You are on page 1of 351

LAM v. KODAK PHILIPPINES, LTD. (Ram) Colorkwik Laboratories, Inc.

with three (3) units Kodak Minilab System


January 11, 2016 | Leonen, J. | Definition of a Contract of Sale 22XL . . . for your proposed outlets in Rizal Avenue (Manila), Tagum (Davao
del Norte), and your existing Multicolor photo counter in Cotabato City
PETITIONER: ALEXANDER and JULIE LAM, doing business under the name under the following terms and conditions:
and style “COLORKWIK LABORATORIES” and “COLORKWIK PHOTO
SUPPLY a) Said Minilab Equipment packages will avail a total of 19% multiple
RESPONDENTS: KODAK PHILIPPINES, LTD. order discount based on prevailing equipment price provided said
equipment packages will be purchased not later than June 30, 1992.
SUMMARY: On January 8, 1992, the Lam Spouses and Kodak Philippines, Ltd. b) 19% Multiple Order Discount shall be applied in the form of
entered into an agreement (Letter Agreement) for the sale of three units of the merchandise and delivered in advance immediately after signing of the
Kodak Minilab System 22XL (Minilab Equipment) in the amount of contract. * Also includes start-up packages worth P61,000.00.
₱1,796,000.00 per unit. Kodak delivered one unit of the Minilab Equipment in c) NO DOWNPAYMENT.
Tagum, Davao Province and the Lam Spouses issued postdated checks amounting d) Minilab Equipment Package shall be payable in 48 monthly
to ₱35,000.00 each for 12 months as payment for the first delivered unit, with the installments at THIRTY FIVE THOUSAND PESOS (P35,000.00)
first check due on March 31, 1992. The depository bank honored the first two inclusive of 24% interest rate for the first 12 months; the balance shall
checks, but the ten other checks were subsequently dishonored after the Lam be re-amortized for the remaining 36 months and the prevailing
spouses ordered the bank to stop payment. Kodak Philippines, Ltd. canceled the interest shall be applied.
sale and demanded that the Lam Spouses return the unit. The Lam Spouses ignored e) Prevailing price of Kodak Minilab System 22XL as of January 8, 1992
the demand but also rescinded the contract because of failure to deliver the two is at ONE MILLION SEVEN HUNDRED NINETY SIX THOUSAND
other Minilab Equipment units. The issues in this case are WoN the contract PESOS.
between the parties is an obligation that is severable, divisible, and susceptible of f) Price is subject to change without prior notice. *Secured with PDCs;
partial performance and WoN rescission of the contract was correctly ordered. The 1st monthly amortization due 45 days after installation.”
SC held that the Letter Agreement contained an indivisible obligation since the 2. Kodak Philippines, Ltd. delivered one (1) unit of the Minilab Equipment in
intention of the parties shows that there be a single transaction covering all three Tagum, Davao Province. The delivered unit was installed by Noritsu
units of the Minilab Equipment. Moreover, since Kodak failed to deliver the representatives. The Lam Spouses issued postdated checks amounting to
remaining Minilab Equipment and Spouses Lam stopped their payments, ₱35,000.00 each for 12 months as payment for the first delivered unit, with
rescission was correctly applied since the obligors did not comply with what was the first check due on March 31, 1992.
incumbent upon them. Rescission under Article 1191 has the effect of mutual 3. The Lam Spouses requested that Kodak Philippines, Ltd. not negotiate the
restitution as if the contract was never entered into. Hence, petitioners must check dated March 31, 1992 allegedly due to insufficiency of funds. The
relinquish possession of the delivered Minilab Equipment unit and accessories, same request was made for the check due on April 30, 1992.
while respondent must return the amount tendered by petitioners as partial payment 4. However, both checks were negotiated by Kodak Philippines, Ltd. and were
for the unit received. honored by the depository bank. The 10 other checks were subsequently
dishonored after the Lam Spouses ordered the depository bank to stop
DOCTRINE: The contract between the parties is one of sale, where one party payment.
obligates himself or herself to transfer the ownership and deliver a determinate 5. Kodak Philippines, Ltd. canceled the sale and demanded that the Lam
thing, while the other pays a certain price in money or its equivalent. A contract of Spouses return the unit. The Lam Spouses ignored the demand but also
sale is perfected upon the meeting of minds as to the object and the price, and the rescinded the contract through the letter dated November 18, 1992 on account
parties may reciprocally demand the performance of their respective obligations of Kodak Philippines, Ltd.’s failure to deliver the two (2) remaining Minilab
from that point on. (In relation to the Sales outline) Equipment units.
6. Kodak Philippines, Ltd. filed a Complaint for replevin and/or recovery of
FACTS: sum of money. The Lam Spouses failed to appear during the pre-trial
1. On January 8, 1992, the Lam Spouses and Kodak Philippines, Ltd. entered conference. Thus, they were declared in default.
into an agreement (Letter Agreement) for the sale of three (3) units of the 7. TRIAL COURT FIRST DECISION:
Kodak Minilab System 22XL (Minilab Equipment) in the amount of a. Kodak Philippines, Ltd. presented evidence ex-parte.
₱1,796,000.00 per unit, with the following terms: b. The trial court issued the Decision in favor of Kodak Philippines,
“This confirms our verbal agreement for Kodak Phils., Ltd. To provide Ltd. ordering the seizure of the Minilab Equipment.
c.
Based on this Decision, Kodak Philippines, Ltd. was able to obtain c. There was only one agreement covering all three (3) units of the
a writ of seizure for the Minilab Equipment installed at the Lam Minilab Equipment and their accessories.
Spouses’ outlet in Tagum, Davao Province. The writ was enforced d. The Letter Agreement specified only one purpose for the buyer,
and Kodak Philippines, Ltd. gained possession of the Minilab which was to obtain these units for three different outlets.
Equipment unit, accessories, and the generator set. e. The 19% multiple order discount as contained in the Letter
8. CA DECISION: Agreement was applied to all three acquired units.
a. The Lam Spouses then filed before the CA a Petition to Set Aside f. The "no downpayment" term contained in the Letter Agreement was
the Orders issued by the trial court. also applicable to all the Minilab Equipment units.
b. These Orders were subsequently set aside by the CA, and the case g. The fourth clause of the Letter Agreement clearly referred to the
was remanded to the trial court for pre-trial. object of the contract as "Minilab Equipment Package."
9. REMANDED BACK TO TRIAL COURT h. This intent must prevail even though the articles involved are
a. In its Decision, the RTC dismissed the case and ordered the Kodak physically separable and capable of being paid for and delivered
to pay Lam Spouses individually
10. CA SECOND DECISION i.
a. Lam Spouses filed their Notice of Partial Appeal. Kodak 2. Rescission was correctly applied since the obligors did not comply with what
Philippines, Ltd. also filed an appeal. was incumbent upon them. Kodak failed to deliver the remaining Minilab
b. The CA dismissed it for Kodak Philippines, Ltd.’s failure to file its Equipment and Spouses Lam stopped their payments.
appellant’s brief, without prejudice to the continuation of the Lam a. (NOTE: Before discussing rescission, the case defined a Contract of
Spouses’ appeal. The Resolution became final and executory. Sale)
c. CA modified the decision of the RTC. i. The contract between the parties is one of sale, where one
party obligates himself or herself to transfer the ownership
ISSUES: and deliver a determinate thing, while the other pays a
1. WoN the contract between the parties is an obligation that is severable, certain price in money or its equivalent.
divisible, and susceptible of partial performance – YES. ii. A contract of sale is perfected upon the meeting of minds
2. WoN rescission of the contract was correctly ordered – YES. as to the object and the price, and the parties may
reciprocally demand the performance of their respective
RULING: WHEREFORE, the Petition is DENIED. The Amended Decision dated obligations from that point on.
September 9, 2005 is AFFIRMED with MODIFICATION. Respondent Kodak b. The power to rescind obligations is implied in reciprocal ones, in case
Philippines, Ltd. is ordered to pay petitioners Alexander and Julie Lam: one of the obligors should not comply with what is incumbent upon
(a) P270,000.00, representing the partial payment made on the Minilab Equipment; him.
(b) P130,000.00, representing the amount of the generator set, plus legal interest at c. Rescission under Article 1191 has the effect of mutual restitution as
12% per annumfrom December 1992 until fully paid; if the contract was never entered into.
(c) P440,000.00 as actual damages; d. Petitioners must relinquish possession of the delivered Minilab
(d) P25,000.00 as moral damages; Equipment unit and accessories, while respondent must return the
(e) P50,000.00 as exemplary damages; and amount tendered by petitioners as partial payment for the unit
(f) P20,000.00 as attorney’s fees. received.
Petitioners are ordered to return the Kodak Minilab System 22XL unit and its standard e. Further, respondent cannot claim that the two (2) monthly
accessories to respondent. SO ORDERED. installments should be offset against the amount awarded by the
Court of Appeals to petitioners because the effect of rescission under
RATIO: Article 1191 is to bring the parties back to their original positions
1. The Letter Agreement contained an indivisible obligation. before the contract was entered into.
a. Intention of the parties is for there to be a single transaction covering f. When rescission is sought under Article 1191 of the Civil Code, it
all three (3) units of the Minilab Equipment. need not be judicially invoked because the power to resolve is implied
b. Respondent’s obligation was to deliver all products purchased under i. Court intervention only becomes necessary when the party
a "package," and, in turn, petitioners’ obligation was to pay for the who allegedly failed to comply with his or her obligation
total purchase price, payable in installments. disputes the resolution of the contract.
ii. Since both parties in this case have exercised their right to
resolve under Article 1191, there is no need for a judicial
decree before the resolution produces effects.
3. The issue of damages is a factual one.
a. A petition for review on certiorari under Rule 45 shall only pertain
to questions of law.
b. The findings of fact of the trial court as affirmed by the Court of
Appeals are conclusive upon this court.
c. The damages awarded by the Court of Appeals were supported by
documentary evidence, especially in light of their failure to produce
receipts or check payments to support their other claim for actual
damages.
d. The award for moral and exemplary damages also appears to be
sufficient.
GAITE v. FONACIER to him, on the same royalty basis. Gaite embarked upon the development
July 31, 1961 | Reyes, J.B.L., J | Contract of Sale and exploitation of the mining claims, opening and paving roads, making
PLAINTIFF-APPELLEE: Fernando Gaite toher improvements and installing facilities used for extracting
DEFENDANTS-APPELLANTS: Isabelo Fonacier, George Krakower Larap approximately 24,000 metric tons of iron ore.
Mines & Smelting Co., Inc, Segundina Vivas, Franscisco Dante, Pacifico Escandor 4. Fonacier decided to revoke the authority granted by him to Gaite to exploit
and Fernando Ty and develop the mining claims. A document entitle “Revocation of Power
of Attorney and Contract” was executed wherein Gaite transferred to
SUMMARY: Fonacier owns 11 iron Iode mineral claims and assigned Gaite as his Fonacier for the consideration of P20,000.00, plus 10% of the royalties that
attorney-in-fact to enter into contracts for the exploration and development of the Fonacier would receive from the mining claims, all his rights and interests
mining claims. Gaite conveyed the mining claims to Larap Iron Mines, which on all the roads, improvements and facilities in or outside said claims as
belong to him and embarked on developing the mins and extracted approximately well as all his rights and interests over the “24,000 tons of iron ore,” in
24,000 metric tons of iron ore. Fonacier revoked Gaite’s authority and executed a consideration of the sum of P75,000, P10,000 of which was paid upon the
document wherein Gaite transferred all roads, improvements, facilities and rights signing of the agreement (P65,000 will be paid from and out of the first
and interests over the 24,000 tons of iron ore to Fonacier in consideration of the letter of credit)
sum of P75,000 (P10,000 of which was paid signing the agreement). A surety bond 5. A surety bond was executed in favor of Gaite to secure the payment of the
was executed in favor of Gaiteto secure the paymentof the balance of P65,000 but said balance of P65,000, with Fonacier as principal and the Larap Mines
Gaite refused to sign and insisted another surety bond is underwritten by a bonding and Smelting Co. and its stockholders George Krakower, Segundina Vivas,
company. The second bond was executed with Far Eastern Surety and Insurance Pacifico Escandor, Francisco Dante, and Fernando Ty as sureties. Gaite,
Co., under the condition that there be an actual sale of the iron ore by Larap Iron however, refused to sign unless another bond under written by a bonding
Mines, but the bond company’s liability automatically expires on December 8, company was put up. Hence, the second bond was executed with the Far
1955. Upon the expiration of the bond, there was neither a sale by Larap Iron Mines Eastern Surety and Insurance Co. as additional surety but it provided that
nor payment to Gaite from Fonacier (P65,000) so Gaiter filed a complaint. SC held the liability of the surety company would attach only when there had been
that the obligation to pay became due and demandable upon the expiration of the an actual sale of iron ore by the Larap Mines & Smelting Co. for not less
Far Eastern Surety bond (failure to put up a good and sufficient security under than P65,000 and that the liability of the surety company would
Article 1198 of the New Civil Code). automatically expire on December 8, 1955.
6. The second bond expired but there was neither sale of the 24,000 tons of
DOCTRINE: A contract of sale is normally commutative and onerous: not only iron ore was made by Larap Mines nor payment of the P65,000 balance of
does each one of the parties assume a correlative obligation (the seller to deliver the price to Gaite by Fonacier. Gaite filed a complaint against Fonacier for
and transfer ownership of the thing sold and the buyer to pay the price),but each their failure to pay the balance.
party anticipates performance by the other from the very start. While in a sale the 7. The uniform defense is that the obligation was subject to a condition that
obligation of one party can be lawfully subordinated to an uncertain event, so that the amount of P65,000 would be payable out of the first letter of credit
the other understands that he assumes the risk of receiving nothing for what he gives covering the first shipment of iron ore and/or the first amount derived from
(as in the case of a sale of hopes or expectations, emptio spei), it is not in the usual the local sale of the iron ore by the Larap Mines. Since no sale of the iron
course of business to do so; hence, the contingent character of the obligation must ore had been made, the obligation was not yet due and demandable.
clearly appear. 8. Fonacier also contended that only 7,573 tons of the estimated 24,000 tons of
iron ore sold to him by Gaite was actually delivered, and counterclaimed for
FACTS: more than P200,000.00 damages.
1. Fonacier is the owner/holder of 11 iron Iode mineral claims, known as the
Dawahan Group in Camarines Norte. ISSUES:
2. By a Deed of Assignment (DOA), Fonacier appointed Fernando Gaite as his 1. Whether or not the lower court erred in holding that the obligation of
true and lawful attorney-in-fact to enter into a contract with any individual Fonacier to pay Gaite (P65,000) is subjected to a period/term or a
or juridical person for the exploration and development of the mining suspensive condition (that the term expired on December 8, 1955) – NO
claims on a royalty basis of not less than P0.50 per ton of ore that might be 2. Whether Fonacier and his sureties still have the right to insist that Gaite
extracted. should wait for the sale or shipment of the ore before receiving
3. Gaite executed a general assignment conveying the development and payment/whether or not they are entitled to take full advantage of the period
exploitation of the mining claims into the Larap Iron Mines, which belongs granted them for making the payment – NO
3. Whether there were really 24,000 tons of iron ore in the stockpiles sold by expiration of the bonding company’s undertaking on December 8
appellee Gaite to appellant Fonacier - YES substantially reduced the security of the vendor’s rights as creditor for the
4. Whether, if there had been a short-delivery as claimed by appellants, they unpaid balance, a security that Gaite considered essential and upon which he
are entitled to the payment of damages – NO had insisted when he executed the deed of sale of the ore to Fonacier. The
case squarely comes under paragraphs 2 and 3 of Article 1198 of the Civil
RULING: Finding no error in the decision appealed from, we hereby AFFIRM the Code of the Philippines: "ART. 1198. The debtor shall lose every right to
same. make use of the period:
(1) . . .
RATIO: (2) When he does not furnish to the creditor the guaranties or
1. We find the court below to be legally correct in holding that the shipment or securities which he has promised.
local sale of the iron ore is not a condition precedent (or suspensive) to the (3) When by his own acts he has impaired said guaranties or
payment of the balance of P65,000.00, but was only a suspensive period or securities after their establishment, and when through fortuitous
term. That the parties to the contract Exhibit "A" did not intend any such state event they disappear, unless he immediately gives new ones equally
of things to prevail is supported by several circumstances: satisfactory.
a. "The balance of Sixty-Five Thousand Pesos (P65,000.00) will be
paid out of the first letter of credit covering the first shipment of iron Appellants' failure to renew or extend the surety company's bond
ores . . ." etc. By the very terms of the contract, the existence of the upon its expiration plainly impaired the securities given to the creditor
obligation to pay is recognized; only (appellee Gaite), unless immediately renewed or replaced.
its maturity or demandability is deferred. 3. There is no charge in this case that Gaite did not deliver to appellants all the
b. A contract of sale is normally commutative and onerous: not only ore found in the stockpiles in the mining claims in questions; Gaite had,
does each one of the parties assume a correlative obligation (the therefore, complied with his promise to deliver, and appellants in turn are
seller to deliver and transfer ownership of the thing sold and the bound to pay the lump price.
buyer to pay the price),but each party anticipates performance by the 4. No short-delivery in this case as would entitle appellants to the payment of
other from the very start. While in a sale the obligation of one party damages, nor could Gaite have been guilty of any fraud in making any
can be lawfully subordinated to an uncertain event, so that the other misrepresentation to appellants as to the total quantity of ore in the stockpiles
understands that he assumes the risk of receiving nothing for what of the mining claims in question, as charged by appellants, since Gaite's
he gives (as in the case of a sale of hopes or expectations, emptio estimate appears to be substantially correct.
spei), it is not in the usual course of business to do so; hence, the
contingent character of the obligation must clearly appear. Nothing
is found in the record to evidence that Gaite desired or assumed to
run the risk of losing his right over the ore without getting paid for
it, or that Fonacier understood that Gaite assumed any such risk.
This is proved by the fact that Gaite insisted on a bond a to guarantee
payment of the P65,000.00, and not only upon a bond by Fonacier,
the Larap Mines & Smelting Co., and the company's stockholders,
but also on one by a surety company; and the fact that appellants did
put up such bonds indicates that they admitted the definite existence
of their obligation to pay the balance of P65,000.00.
c. The rules of interpretation would incline the scales in favor of "the
greater reciprocity of interests", since sale is essentially onerous.
Article 1378, paragraph 1, in fine, provides: If the contract is
onerous, the doubt shall be settled in favor of the greatest reciprocity
of interests.
2. The obligation became due and demandable under Article 1198 of the New
Civil Code when Caltex failed to put up a good and sufficient security in lieu
of the Far Eastern Surety bond which expired on December 8, 1995. The
BUENAVENTURA v. CA
November 20, 2003 | Carpio J. | Validity of a contract of sale; a) Firstly, there was no actual valid consideration for the deeds of sale over
Gross inadequacy of price does not affect a contract of sale the properties in litis;
b) Secondly, assuming that there was consideration in the sums reflected in
the questioned deeds, the properties are more than three-fold times more
PETITIONERS/PLAINTIFFS: Spouses Bernardo Buenaventura and valuable than the measly sums appearing therein;
Consolacion Joaquin, Spouses Juanita Edra and Nora Joaquin c) Thirdly, the deeds of sale do not reflect and express the true intent of the
RESPONDENTS/DEFENDANTS: Court of Appeals, Joaquin parents and parties (vendors and vendees); and
siblings d) Fourthly, the purported sale of the properties in litis was the result of a
deliberate conspiracy designed to unjustly deprive the rest of the
SUMMARY: Petitioners, who are the children and siblings of the respondents, compulsory heirs (plaintiffs herein) of their legitime.
sought to declare as null and void the deeds of sale of real property that was sold by
the respondent parents in favor of their co-respondent children. The petitioners 3. The trial court noted that compulsory heirs have the right to a legitime but
assailed the validity of the sale on the grounds that their legitime was impaired such right is contingent since said right commences only from the moment of
because of the sale, and that there was no actual valid consideration for the deeds death of the decedent pursuant to Article 777 of the Civil Code of the
of sale due to the failure to pay the price. And assuming that there was a valid Philippines. After trial, the trial court ruled in favor of the defendants and
consideration, there was gross inadequacy in the said price. However, before dismissed the complaint.
touching the issue on the validity of the sale, the Court held that the petitioners are
not the real party-in-interest because their right to the property is merely inchoate 4. The Court of Appeals affirmed the decision of the trial court. The appellate
and will vests only upon the death of their parents. On the issue on the validity of court ruled:
the sale, the petitioners failed to show that the contract of sale was simulated due to
lack of consideration. As a matter of fact, the evidence that the petitioners presented Upon this point, there is no question that plaintiffs-appellants, like their
showed that there was a cost for each lots sold, thus showing that there was a defendant brothers and sisters, are compulsory heirs of defendant spouses,
meeting of minds between the respondents as regards the price. And even assuming Leonardo Joaquin and Feliciana Landrito, who are their parents. However,
that there was gross inadequacy on the price, the Court held that the law expressly their right to the properties of their defendant parents, as compulsory heirs,
provides mere gross inadequacy on the price, as a rule, will not invalidate the sale is merely inchoate and vests only upon the latters death. While still alive,
unless it indicates a defect in the consent or that the parties intended some other act defendant parents are provided that such dispositions are not made in fraud
or contract. of creditors.

DOCTRINE: Failure to pay the consideration is different from lack of ISSUES:


consideration. The former results in a right to demand the fulfillment or cancellation
of the obligation under an existing valid contract while the latter prevents the 1. W/N petitioners have a legal interest over the properties subject of the Deeds
existence of a valid contract. Article 1470 of the Civil Code further provides: Gross of Sale – NO
inadequacy of price does not affect a contract of sale, except as may indicate a 2. W/N the Deeds of Sale are void for lack of consideration – NO
defect in the consent, or that the parties really intended a donation or some other act 3. W/N the Deeds of Sale are void for gross inadequacy of price – NO
or contract.
RULING: WHEREFORE, we AFFIRM the decision of the Court of Appeals in toto.

FACTS: RATIO:

1. Sought to be declared null and void ab initio are certain deeds of sale of real 1. It is evident from the records that petitioners are interested in the properties
property executed by defendant parents Leonardo Joaquin and Feliciana subject of the Deeds of Sale, but they have failed to show any legal right to
Landrito in favor of their co-defendant children and the corresponding the properties. The trial and appellate courts should have dismissed the
certificates of title issued in their names. action for this reason alone. An action must be prosecuted in the name of the
real party-in-interest. Petitioners do not have any legal interest over the
2. In seeking the declaration of nullity of the aforesaid deeds of sale and properties subject of the Deeds of Sale. As the appellate court stated,
certificates of title, plaintiffs, in their complaint, aver the ff:
petitioners right to their parents properties is merely inchoate and vests
only upon their parent’s death. While still living, the parents of petitioners
are free to dispose of their properties.

2. A contract of sale is not a real contract, but a consensual contract. As a


consensual contract, a contract of sale becomes a binding and valid
contract upon the meeting of the minds as to price. It is not the act of
payment of price that determines the validity of a contract of sale.
Payment of the price has nothing to do with the perfection of the contract.
Payment of the price goes into the performance of the contract. Failure to
pay the consideration is different from lack of consideration. The former
results in a right to demand the fulfillment or cancellation of the obligation
under an existing valid contract while the latter prevents the existence of a
valid contract.

Petitioners failed to show that the prices in the Deeds of Sale were
absolutely simulated. To prove simulation, petitioners presented Emma
Joaquin Valdozs testimony stating that their father, respondent Leonardo
Joaquin, told her that he would transfer a lot to her through a deed of sale
without need for her payment of the purchase price. The trial court did not
find the allegation of absolute simulation of price credible. Petitioners’ failure
to prove absolute simulation of price is magnified by their lack of knowledge
of their respondent siblings financial capacity to buy the questioned lots. On
the other hand, the Deeds of Sale which petitioners presented as evidence
plainly showed the cost of each lot sold. Not only did respondent’s minds
meet as to the purchase price, but the real price was also stated in the
Deeds of Sale. As of the filing of the complaint, respondent siblings have
also fully paid the price to their respondent father.

3. Articles 1355 of the Civil Code states:


Art. 1355. Except in cases specified by law, lesion or inadequacy of cause
shall not invalidate a contract, unless there has been fraud, mistake or undue
influence.

Article 1470 of the Civil Code further provides:

Art. 1470. Gross inadequacy of price does not affect a contract of sale,
except as may indicate a defect in the consent, or that the parties really
intended a donation or some other act or contract.

Petitioners failed to prove any of the instances mentioned in Articles 1355


and 1470 of the Civil Code, which would invalidate, or even affect, the Deeds
of Sale. Indeed, there is no requirement that the price be equal to the exact
value of the subject matter of sale. All the respondents believed that they
received the commutative value of what they gave.
CELESTINO CO AND CO. v. COLLECTOR OF INTERNAL REVENUE 2. Said article reads as follows: “A contract for the delivery at a certain price
(Mark) of an article which the vendor in the ordinary course of his business
August 31, 1956 |Bengzon J. | sales manufactures or procures for the general market, whether the same is on
hand at the time or not, is a contract of sale, but if the goods are to be
PETITIONER: Celestino Co and Company manufactured specially for the customer and upon his special order, and not
RESPONDENTS: Collector of Internal Revenue for the general market, it is contract for a piece of work.”

SUMMARY: Petitioner’s company, Oriental Sash Factory, wants to pay less tax 3. The company habitually makes Sash, windows, and doors as it has been
by claiming that they are contractors, and not manufacturers. CTA said that they represented to the public. The fact that the windows and doors are made only
are manufacturers because they are a “factory” and they offer ready-mad products. when customers place their orders, does not alter the nature of the
SC held that petitioners do not need special machinery to address special orders, establishment, for it is obvious that they accept special orders other than
thus they are not contractors as provided by Art. 1467. Oriental Sash Factory is a making ready-made products. The factory does nothing more than sell the
manufacturer. goods that it mass produces or habitually makes.
DOCTRINE: The orders herein exhibited were not shown to be special. Nothing
is shown to call them special requiring extraordinary service of the factory, it does 4. When this Factory accepts a job that requires the use of extraordinary or
not fall under Art. 1467 as a contract piece of work. additional equipment, or involves services not generally performed by it-it
thereby contracts for a piece of work — filing special orders within the
FACTS: meaning of Article 1467. The orders herein exhibited were not shown to be
1. Celestino Co doing business under the name of “Oriental Sash Factory”. special. They were merely orders for work — nothing is shown to call them
From 1956-1951 it paid percentage tax of 7% (National Revenue Code sec. special requiring extraordinary service of the factory.
186) on the gross receipts of its sash, door, and window factory. However, on
1952 it began to claim liability only to contractor’s 3% tax (Instead of 7%) 5. As the doors and windows had been admittedly "manufactured" by the
under sec. 191. Oriental Sash Factory, such transactions could be, and should be taxed as
2. Celestino claims that they do not manufacture ready-made doors, sash, and "transfers" thereof under section 186 of the National Revenue Code.
windows for the public and that they are contractors. He claims that they only
do Special Orders for customers, thus, contending they are not manufacturers.
This did not convince the BIR and the Court of Tax Appeals.
3. CTA said that their tradename gives an impression they do engage in
manufacturing and their records suggest that their huge earnings (P188,
754.69) cannot be from special orders from ther few customers, but because
it was from ready made products. They also offered themselves as a “factory”
to the public.
4. Petitioners now assail the decision of the CTA

ISSUES:
1. WoN EO petitioner is a manufacturer – YES

RULING: The appealed decision is consequently affirmed. So ordered.

RATIO:
1. Appellant invokes Article 1467 of the New Civil Code to bolster its
contention that in filing orders for windows and doors according to
specifications, it did not sell, but merely contracted for pieces of work or
"merely sold its services".
Commissioner of Internal Revenue v. Engineering Equipment and Supply 4. It was recommended to CIR that EES be assessed for P480,912.01 as deficiency
Company advance sales tax on the theory that it misdeclared its importation of air
June 30, 1975 Esguerra, J. | Contract for Piece of Work conditioning units and parts and accessories thereof which are subject to tax under
Section 185(m)1 of the Tax Code, instead of Section 186 of the same Code.
PETITIONER: Commissioner of Internal Revenue
RESPONDENTS: Engineering Equipment and Supply Company 5. This assessment was revised on January 23, 1959, in line with the observation of
the Chief, BIR Law Division, and was raised to P916,362.56 representing
SUMMARY: Engineering Equipment & Supply (EES) was engaged in the deficiency advance sales tax and manufacturers sales tax, inclusive of the 25% and
business of designing and installing central air-conditioning systems. Pursuantto 50% surcharges.
Section 185 of the Tax Code, EES was assessed by the CIR for 30% advanced
sales tax plus surcharges (of 25% and 50%) for misdeclaring its importation of 6. EES appealed to Court of Tax Appeals (CTA), arguing that they are contractors
air conditioning units and parts and accessories. EES appealed to Court of Tax and not manufacturers, and thus, should only be liable for the 3% tax on sales of
Appeals (CTA), arguing that they are contractors and not manufacturers, and thus, services or pieces of work.
should only be liable for the 3% tax on sales of services or pieces of work. The
Court of Tax Appeals reversed the order of the CIR, declaring that EES is a 7. The Court of Tax Appeals rendered judgement and said that: the decision of
contractor. The Court ruled that EES is a contractor, subject to tax stated in respondent appealed from is hereby modified, and petitioner, as a contractor, is
Section 191 of the Code. Though EES imported such items, they were NOT for declared exempt from the deficiency manufacturers sales tax covering the period
sale to the general public and were used as mere components for the design of the from June 1, 1948. to September 2, 1956. However, petitioner is ordered to pay
centralized air-conditioning system, wherein its designs and specifications are respondent, or his duly authorized collection agent, the sum of P174,141.62 as
different for every client. Various technical factors must be considered and it can compensating tax and 25% surcharge for the period from 1953 to September 1956.
be argued that no 2 plants are the same; all are engineered separately and With costs against petitioner.
distinctly. Each project requires careful planning and meticulous layout. Such
central air- conditioning systems and their designs would not have existed were 8. Hence, this appeal of the CIR.
it not for the special order of the party desiring to acquire it. This implies that EES
did not intend to sell the said aircon units to the general public. Thus, EES is not ISSUES:
liable for the sales tax of 30%.
1. Whether or not EES is a manufacturer of air conditioning units under Section
DOCTRINE: If the thing is specially done only upon the specific order of 185 of the Code or a contractor (piece of work) under Section 191. -
another, this is a contract for a piece of work; if the thing is manufactured or CONTRACTOR
procured for the general market in the ordinary course of business, it is a contract
of sale. 2. Whether or not EES is guilty of fraud - YES

3. Whether or not the tax assessment has prescribed - NO


FACTS: RULING: WHEREFORE, the decision appealed from is affirmed with the
1. Engineering Equipment and Supply Co. (EES), a domestic corporation, is an modification that Engineering is hereby also made liable to pay the 50% fraud
engineering and machinery firm. It is engaged, among others, in the design and surcharge.
installation of central type air conditioning system, pumping plants and steel
fabrications.
RATIO:
2. On July 27, 1956, certain Juan de la Cruz wrote to the Commissioner of Internal
Revenue denouncing EES for tax evasion. 1. There is a distinction between a contract of sale (manufacturer) and a contract for
furnishing services, labor and materials. Such difference is tested by the inquiry of
3. Pursuant to Section 185 of the Tax Code, EES was assessed by the CIR for 30% whether the thing transferred is one not in existence and which never would have
advanced sales tax plus surcharges (of 25% and 50%) for misdeclaring its existed but for the order of the party desiring to acquire it, or a thing which would have
importation of air conditioning units and parts and accessories. existed and has been the subject of sale to some other persons even if the order had not
been given.
2. Art. 1467 of the Civil Code–a contract for the delivery at a certain price of an article
which the vendor in the ordinary course of his business manufactures or procures for 3. BUT, because the fraud is too glaring, it was held that EES could not be absolved
the general market, whether the same is on hand at the time or not, is a contract of sale, from the 50% fraud surcharge. Otherwise, it would give premium to an intolerable act
but if the goods are to be manufactured specially for the customer and upon his special of tax evasion.
order and not for the general market, it is a contract for a piece of work.
(3) 1. No. EES contends that the prescriptive period is 5 years from importation. But
3. A contractor is a person who,in pursuit of independent business undertakes to do a the SC held that Sec 332 of the code provides for the exceptions as to the period of
specific job or piece of work for other persons, using his own means and methods limitation of assessment and collection of taxes
without submitting himself to control as to the petty details 2. Sec. 332–in case of a false and fraudulent return with intent to evade tax..., the tax
may be assessed, or a proceeding in court for the collection of such tax may be begun
4. Though EES imported such items, they were NOT for sale to the general public and without assessment at any time within 10 years after the discovery of the falsity, fraud
were used as mere components for the design of the centralized air-conditioning or omission.
system, wherein its designs and specifications are different for every client. Various
technical factors must be considered and it can be argued that no 2 plants are the same;
all are engineered separately and distinctly. Each project requires careful planning and
meticulous layout. Such central air- conditioning systems and their designs would not
have existed were it not for the special order of the party desiring to acquire it. This
implies that EES did not intend to sell the said aircon units to the general public. Thus,
EES is not liable for the sales tax of 30%.

5. EES should be held liable to pay the taxes prescribed in Section 190 of the Code.
This compensating tax is not a tax on the importation of goods but a tax on the use of
imported goods not subject to sales tax. Hence, it should be held liable to the payment
of 30% compensating tax in accordance with Sec 190, but without the 50% mark up
provided in Section 183 (b)

6. Also, EES should be subjected to 25% surcharge for delinquency in the payment of
the said tax, as provided in Section 190:

7. Sec 190 –If any article withdrawn from the customhouse or the post office without
payment of the compensating tax is subsequently used by the importer for other
purposes, corresponding entry should be made in the books of accounts if any are kept
or a written notice thereof sent to the Collector and payment of the corresponding
compensating tax made within 30 days from the date of entry or notice, and if tax is
not paid within such period the amount of tax shall be increased by 25%.

MINOR ISSUES:

(2) 1. Yes. As proven by correspondences of EES with foreign companies wherein


EES requested that words of air conditioning equipment should not be mentioned in
shipping documents.

2. The CTA absolved EES from paying the 50% surcharge prescribed in Sec 183 (a)
because the surcharge Section 190 of the tax Code (where EES is subjected to as a
contractor) does not provide it. According to CTA, where a particular provision of the
tax code does not impose a 50% surcharge as fraud penalty, it cannot be enforced.
QUIROGA v. PARSONS 3. The defendant violated the following obligations: not to sell the beds at higher
August 23, 1918 | Avancena, J. | Essential features of a contract of sale prices than those of the invoices; to have an open establishment in Iloilo; itself
to conduct the agency; to keep the beds on public exhibition, and to pay for
PETITIONER: Andres Quiroga the advertisement expenses for the same; and to order the beds by the dozen
RESPONDENTS: Parsons Hardware Co. and in no other manner
4. He alleged that the defendant was his agent for the sale of his beds in Iloilo,
SUMMARY: Quiroga and Parsons Hardware entered into a contract where the and that said obligations are implied in a contract of commercial agency.
former granted the latter the exclusive right to sell Quiroga Beds in the Visayas.
It provided for a discount of 25% as commission for the sales, among other ISSUES:
conditions. Quiroga alleged that Parsons breached its contractual obligations by 1. WoN the contract between Quiroga and Parsons was one of an agency and
selling the beds at a higher price, not having an open establishment in Iloilo, not not sale – NO, it was that of a simple purchase and sale.
maintaining a public exhibition, and for not ordering beds by the dozen. Only the
last imputation was provided for by the contract, the others were NEVER RULING: The judgment appealed from is affirmed, with costs against
stipulated. Quiroga argued that since there was a contract of agency between the appellant. So ordered.
them, such obligations were necessarily implied. The issue in this case is WON
the contract between them was one of agency, and not sale. The Court held that RATIO:
the agreement between Quiroga and Parsons was that of a simple purchase and 1. In order to classify a contract, due regard must be given to its essential
sale - NOT AN AGENCY. Quiroga supplied beds, while Parsons had the clauses. In the contract in question, there was the obligation on the part of the
obligation to pay their purchase price. These are characteristics of a purchase and plaintiff to supply the beds, and on the part of the defendant to pay their price.
sale. In a contract of agency (or order to sell), the agent does not pay its price yet, These are the characteristics of a purchase and sale.
and sells the products, remitting to the principal its proceeds. Contracts are what 2. In a contract of agency (or order to sell), the agent does not pay its price yet,
the law defines it to be, not what the parties call it. and sells the products, remitting to the principal its proceeds. Unsold products
must also be returned to the principal. The provisions on commission and the
DOCTRINE: These are precisely the essential features of a contract of purchase use of the word “agency” in the contract as well as the testimonies in court
and sale. There was the obligation on the part of the plaintiff to supply the beds, do not affect its nature. Contracts are what the law defines it to be, not what
and, on that of the defendant, to pay their price. These features exclude the legal the parties call it.
conception of an agency or order to sell whereby the mandatary or agent receives 3. By virtue of the contract between the plaintiff and the defendant, the latter,
the thing to sell it, and does not pay its price, but delivers to the principal the price on receiving the beds was necessarily obliged to pay their price within the
he obtains from the sale of the thing to a third person, and if he does not succeed term fixed, without any other consideration, and regardless as to whether he
in selling it, he returns it. had or had not sold the beds. With respect to the defendant's obligation to
order by the dozen (the only one expressly imposed by the contract), the effect
of its breach would only entitle the plaintiff to disregard the orders which the
defendant might place under other conditions. But if the plaintiff consents to
FACTS: fill them, he waives his right and cannot complain for having acted thus at his
1. January 24, 1911 - Andres Quiroga and J. Parsons, both merchants, entered own free will.
into a contract for the exclusive sale of Quiroga Beds in the Visayan Islands.
It was agreed, among others, that Andres Quiroga grants the exclusive right
to sell his beds in the Visayan Islands to J. Parsons, subject to some conditions
provided in the contract.
2. It was agreed that in compensation for the expenses of advertisement which,
for the benefit of both contracting parties, Mr. Parsons may find himself
obliged to make Mr. Quiroga assumes the obligation to offer and give the
reference to Mr. Parsons in case anyone should apply for the exclusive agency
for any island not comprised with the Visayan group; and that Mr. Parsons
may sell, or establish branches of his agency for the sale of Quiroga Beds in
all the towns of the Archipelago where there are no exclusive agents, and
shall immediately report such action to Mr. Quiroga for his approval.
07 Gonzalo Puyat & Sons, Inc. v. Arco Amusement Company (Paolo) and sale. The appellate court, however, held that the relation between Arco
June 20, 1941 | Laurel, J. | Not an agent and Gonzalo Puyat was that of and principal and its agent and held Gonzalo
Puyat responsible for $1,335 or 2,671 pesos for the total sum that was
PETITIONER: Gonzalo Puyat & Sons, Inc. overcharged in both transactions.
RESPONDENT: Arco Amusement Company (formerly known as Teatro Arco)
ISSUES: Whether or not the relationship between the Arco and Gonzalo Puyat was
SUMMARY: Arco was in the business of operating cinematographs. Gonzalo Puyat that of a principal and its agent? - NO, it is that of a sale (vendor and vendee).
& Sons was the exclusive agent of Starr Piano, which provided sound reproducing
RULING: Petition granted. Gonzalo Puyat & Sons, Inc. is absolved.
devices for cinematographs. Arco made two agreements with Gonzalo Puyat to order
RATIO:
such equipment from Starr piano and paid two separate transactions. Arco discovered
1. Arco accepted the prices of $1,700 and $1,600, respectively, for the sound
three years later that they paid for the list price (standard) of the equipment, and not
reproducing equipment subject of its contract with the petitioner, are clear in
the net price (discounted). They filed a civil case against Gonzalo Puyat & Sons for
their terms and admit no other interpretation that the respondent in question
the reimbursement of the excess payments for the overcharged equipment. The SC
at the prices indicated which are fixed and determinate.
ruled that this was a sale and not a relationship of agency. The terms of the contract
were clear in the object and price to be paid. There could be no agency just because
2. Gonzalo Puyat received ten per cent (10%) commission, this does not
of a commission on the part of Gonzalo Puyat for the transaction as this is not
necessarily make the petitioner an agent of the respondent, as this provision
incompatible with a contract of purchase and sale. The Court cannot grant relief for
is only an additional price which the respondent bound itself to pay, and
bad judgement in commercial transactions
which stipulation is not incompatible with the contract of purchase and sale.
DOCTRINE: The contract is the law between the parties and should include all the
3. To hold Gonzalo Puyat as an agent of Arco in the purchase of equipment and
things they are supposed to have agreed upon.
machinery from Starr Piano, is incompatible with the admitted fact that the
Gonzalo Puyat is the exclusive agent of the same company in the Philippines.
FACTS:
It is out of the ordinary for one to be the agent of both the vendor and the
1. Arco Amusement Company was engaged in the business of operating
purchaser.
cinematographs (motion picture film camera used as a film projector).
Gonzalo Puyat & Sons was another corporation acting as the exclusive agent 4. It follows that Gonzalo Puyat as vendor is not bound to reimburse Arco as
of Starr Piano Company of Richmond, Indiana, USA. This company dealt vendee for any difference between the cost price and the sales price which
with cinematograph equipment and machinery.
represents the profit realized by the vendor out of the transaction. This is the
2. Arco was interested in equipping its cinematographs with sound reproducing very essence of commerce without which merchants or middleman would not
devices that it could get from Starr Piano. The president and general manager exist.
of Arco approached the acting manager of Gonzalo Puyat.
3. It was agreed that Gonzalo Puyat would order the equipment on behalf of 5. The contract is the law between the parties and should include all the things
Arco who would then pay the price of the equipment plus a 10 percent
they are supposed to have agreed upon. What does not appear on the face of
commission, and all freight, insurance, banking, and cable charges. the contract should be regarded merely as ‘dealer’s’ or ‘trader’s talk,’ which
4. So the equipment arrived and was delivered to Arco. In this initial transaction,
can not bind either party.
$1700 was the cost of the equipment, excluding the commission and the
charges. 6. If the vendee later on discovers itself at the short end of a bad bargain, it alone
5. The following year, they came into a similar agreement for the acquisition of
must bear the blame, and it cannot rescind the contract, much less compel a
sound reproducing devises. This time it costed around $1600 for the cost of
reimbursement of the excess price, on that ground alone. It willingly paid the
the equipment, with the commission and an additional $160 for all expenses price quoted and received the equipment and machinery as represented.
and charges.
6. Three years later, they discovered that they paid for the list price of the sound 7. Not every concealment is fraud; and short of fraud, it were better that, within
devices and not the net price which had an applied discount from Starr Piano certain limits, business acumen permit of the loosening of the sleeves and of
to the benefit of Gonzalo Puyat. Arco filed a civil suit to obtain a reduction the sharpening of the intellect of men and women in the business world.
or reimbursement for overpriced charges.
7. The trial court ruled in favor of Arco, saying this was an outright purchase
LO V. KJS ECO-FORMWORK SYSTEM PHIL., INC. (DOM) • February 22, 1990, Petitioner Ordered scaffolding equipment from
08 October 2003 | Ynares-Santiago. | Assignment of Credit and Dacion En Pago respondent worth P540,425.80.
• Petitioner paid the downpayment in the amount of 150,000. The balance was
PETITIONER: Sonny Lo able made payable in 10 monthly installments
RESPONDENT: KJS Eco-Formwork System Phil Inc. • Respondent was able to deliver the scaffoldings to the petitioner
• Unfortunately, petitioner was only able to pay the first two monthly
SUMMARY: Petitioner Sonny Lo is a building contractor doing business under the installments for his business encountered financial difficulties and he was
name San’s Enterprise while the respondent KJS Eco-Formwork is a corporation unable to settle his obligation to the respondent despite oral and written
engaged in the sale of steel scaffoldings. February 22, 1990, Petitioner Ordered demands made against him
scaffolding equipment from respondent worth P540,425.80. Petitioner paid the • On October 11, 1990, Petitioner and respondent executed a deed of
assignment whereby the petitioner assigned to respondent his receivables in
downpayment in the amount of 150,000. The balance was able made payable in 10
the amount of 335, 462.14 from Jomero Realty Corporation (the owner of the
monthly installments. Respondent was able to deliver the scaffoldings to the residential house to be constructed by the petitioner-assignor)
petitioner. Unfortunately, petitioner was only able to pay the first two monthly • A relevant part of the deed of assignment stipulates that:
installments for his business encountered financial difficulties and he was unable to o the ASSIGNOR further agrees and stipulates as aforesaid that the
settle his obligation to the respondent despite oral and written demands made against said ASSIGNOR, his heirs, executors, administrators, or assigns,
him. Both parties executed a deed of assignment because Sonny Lo claimed that shall and will at times hereafter, at the request of said ASSIGNEE,
Jomero Realty is indebted to him. Sonny Lo assigned his collection rights to KJS. its successors or assigns, at his cost and expense, execute and do all
such further acts and deeds as shall be reasonably necessary to
However, when KJS tried to collect the sum of money from Jomero they refused
effectually enable said ASSIGNEE to recover whatever collectibles
saying Sonny Lo is also indebted to them. said ASSIGNOR has in accordance with the true intent and meaning
of these presents
When KJS requested the payment from Sonny Lo, the petitioner claims that the • However, when respondent tried to collect the said credit from Jomero Realty
assignment of credit already extinguished his obligation. Corporation, the latter refused to honor the deed of assignment because it
claimed that petitioner was also indebted to it
DOCTRINE: An assignment of credit is an agreement by virtue of which the owner • November 26, 1990, respondent sent a letter to petitioner demanding payment
of a credit, known as the assignor, by a legal cause, such as sale, dacion en pago, of his obligation, but petitioner refused to pay claiming his obligation had
exchange or donation, and without the consent of the debtor, transfers his credit and been extinguished when they executed the deed of assignment
accessory rights to another, known as the assignee, who acquires the power to enforce • January 10, 1991, respondent filed am action for recovery of a sum of money
it to the same extent as the assignor could enforce it against the debtor against the petitioner before the RTC
• During the trial petitioner argued that his obligation was extinguished with
In dacion en pago, as a special mode of payment, the debtor offers another thing to the execution of deed of assignment of credit
the creditor who accepts it as equivalent of payment of an outstanding debt.16 In order
• Respondent for its part, presented the testimony of its employee Banaga who
that there be a valid dation in payment, the following are the requisites:
testified that Jomero Realty refused to honor the assignment of credit
o There must be the performance of the prestation in lieu of payment which
• Trial Court rendered a decision dismissing the complaint on the ground the
may consist in the delivery of a corporeal thing or real right or a credit against third
credit extinguished the obligation
person
• CA reversed the RTC decision and ordered Sonny Lo to pay the respondent
o There must be some difference between the prestation due and that which is
given in substitution
o Agreement between the creditor and debtor that the obligation is immediately ISSUE/S:
extinguished by reason of the performance of a prestation from that due • WON the assignment of credit extinguishes the obligation of Sonny
Lo to pay KJS Eco-Framework System
FACTS: RULING:
• Petitioner Sonny Lo is a building contractor doing business under the name • The assignment of credit did not extinguish the obligation
San’s Enterprise while the respondent KJS Eco-Formwork is a corporation because it cannot be considered as a dacion en pago moreover
engaged in the sale of steel scaffoldings the petitioner breached his obligation under the deed of
assignment

RATION:
• An assignment of credit is an agreement by virtue of which the
owner of a credit, known as the assignor, by a legal cause, such as
sale, dacion en pago, exchange or donation, and without the consent
of the debtor, transfers his credit and accessory rights to another,
known as the assignee, who acquires the power to enforce it to the
same extent as the assignor could enforce it against the debtor
• In dacion en pago, as a special mode of payment, the debtor offers
another thing to the creditor who accepts it as equivalent of payment
of an outstanding debt.16 In order that there be a valid dation in
payment, the following are the requisites:
o There must be the performance of the prestation in lieu of
payment which may consist in the delivery of a corporeal
thing or real right or a credit against third person
o There must be some difference between the prestation due
and that which is given in substitution
o Agreement between the creditor and debtor that the
obligation is immediately extinguished by reason of the
performance of a prestation from that due
• In dacion it really partakes in one sense of the nature of sale that is the creditor
is really buying the property of the debtor, payment of which is to be charged
from the debtor’s debt.
• Hence, it may well be that the assignment of credit, which is in the nature of
a sale of personal property produced the effects of a dation in payment which
may extinguish the obligation.20 However, as in any other contract of sale,
the vendor or assignor is bound by certain warranties. More specifically, the
first paragraph of Article 1628 of the Civil Code provides:
The vendor in good faith shall be responsible for the existence and
legality of the credit at the time of the sale, unless it should have been sold as
doubtful; but not for the solvency of the debtor, unless it has been so expressly
stipulated or unless the insolvency was prior to the sale and of common
knowledge.
• From the above provision, the petitioner as vendor or assignor is bound to
warrant the existence and legality of the credit at the time of the sale. When
Jomero claimed it was no longer indebted to petitioner, It is essentially meant
that its obligation to petitioner has been extinguished by compensation. In
other words, it behooved on petitioner to make good its warranty and paid
the obligation
• The petitioner also breached the obligation of the deed of assignment.
Petitioner should have ensured the performance thereof in case the same is
later found to be inexistent. He should be held liable to the payent of his
indebtedness
RATIO:
PARAGAS v. HEIRS OF DOMINADOR BALACANO a. Their grandfather Gregorio could not have appeared before the
Aug. 31, 2005 | Chico-Nazario, J. | Capacity of Parties notary public on July 22, 1996 because he was then confined
b. At the time of the alleged execution of the deed of sale, Gregorio
PETITIONER: Sps. Rudy Paragas and Corazon Paragas was seriously ill, in fact dying at that time, which vitiated his
RESPONDENTS: Hrs. of Dominador Balacano, namely: Dominic, Rodolfo, consent. That at 81 years old he was old, weak, sick and could no
Nanette, and Cyric, all surnamed Balacano, represented by Nanette Balacano and longer talk and whose condition had worsened. Nanette Balacano
Alfredo Balacano claimed that Gregorio could not have signed a deed of sale on July
19, 1996 because she stayed at the hospital the entire day and saw
SUMMARY: Gregorio was an octogenarian who was suffering from liver no visitors.
cirrhosis. On his last week he purportedly sold portions of lot 1175-E and the whole c. Catalino manipulated the execution of the deed and prevailed upon
of 1175-F to Sps. Paragas. The grandchildren of Gregorio filed a complaint alleging the dying Gregorio to sign his name on a paper the contents of which
that their grandfather couldn’t have consented to the alleged sale because he was he never understood because of his serious condition
too weak, old, and sick to do so. The Court affirmed the CA and lower court’s d. And that the other half portion of the lots belonged to Lorenza
findings declaring the deed of sale null and void and denied the petition of Sps. 9. Catalino and Sps. Paragas moved to dismiss the complaint on the ff grounds:
Paragas. a. Plaintiffs have no legal capacity because Domingo was still alive
b. Alfredo, an indispensable party is not impleaded
DOCTRINE: The general rule is that a person is not incompetent to contract c. That there was no alleged mistake, violence, intimidation, undue
merely because of advanced years or by reason of physical infirmities. However, influence or fraud
when such age or infirmities have impaired the mental faculties so as to prevent the 10. Lower court denied the motion to dismiss
person from properly, intelligently, and firmly protecting her property rights then 11. Sps. Paragas filed their Answer with Counter-Claim
she is undeniably incapacitated. a. That the deed of sale was actually executed on July 19, 1996 and
that the notary public went to the hospital in Nueva Vizcaya on July
FACTS: 18, 1996 to notarize the deed of sale.
1. Petition for review seeking to annul the Decision of the CA affirming with b. That Gregorio of sound and disposing mind
modification the Decision of RTC. Petition likewise seeks to annul the c. That the lots were Gregorio’s separate capital
Resolution denying petitioners’ MR. d. That the entire areas of the lots were sold to Sps. Paragas.
2. Gregorio Balacano, married to Lorenza Sumigacay was the registered owner e. Sps. Paragas presented as witnesses Notary Public de Guzman and
of two lots (1175-E and 1175-F) located at Isabela. instrumental witness Antonio to prove Gregorio’s execution of the
3. Gregorio had three children, namely: Domingo, Catalino, and Alfredo sale. That Atty. De Guzman went to the hospital himself and that
4. Lorenza died on December 11, 1991 while Gregorio died on July 28, 1996. Antonio took pictures of Gregorio while signing the deed of sale.
5. Prior to Gregorio’s death he was admitted at the Veterans General Hospital f. Atty. De Guzman claimed that the deed of sale was executed on July
in Nueva Vizcaya and stayed there until July 19, 1996 and was transferred to 18, 1996 but he only entered it in his registry on July 22, 1996.
Veterans Memorial Hospital in QC where he stayed until his death. g. That Gregorio received P50,000 of the P500,000 consideration.
6. On July 22, 1996, barely a week prior to his death, Gregorio purportedly sold 12. After the trial, the lower court declared the deed of sale null and void and that
a portion of of 1175-E and the whole of 1175-F to Sps. Paragas for the total the lots were conjugal properties.
consideration of P500,000. 13. The Court of Appeals affirmed the Decision of the lower court with
7. Sps. Paragas then sold a portion of 1175-E to Catalino. modification that the lots were adjudged as belonging to the estate of
8. Domingo’s children filed a complaint against Catalino and Sps. Paragas Gregorio Balacano. The CA also denied petitioners’ MR. Hence this petition.
alleging:
ISSUE: Guzmans bare assertion that Gregorio asked him to prepare a deed,
1. W/N the CA erred in finding that there was no perfected and partially as Atty. de Guzman was not personally aware of the agreed
executed contract of sale due to absence of meeting of the minds and lack of consideration in the sale of the lots, not being privy to the parties
Gregorio’s consent to the sale. agreement. To us, Rudy could have been a competent witness to
testify on the perfection of this prior contract; unfortunately, the
RULING: WHEREFORE, the present petition is DENIED. Accordingly, the Decision defendants-appellants did not present Rudy as their witness.
and the Resolution of the Court of Appeals are hereby AFFIRMED.
We seriously doubt too the credibility of Atty. de Guzman as a
Ratio: witness. We cannot rely on his testimony because of his tendency
1. SC is not a trier or facts and ruled not to disturb the factual findings of the to commit falsity.”
CA which were in full agreement with that of the trial court b. The lower court also did not consider Antonio Agcaoili, petitioner
2. In affirming the trial court that there was no prior and perfected contract of Rudy Paragas’ driver, a convincing witness, concluding that he was
sale, the CA explained – telling a rehearsed story. The lower court said -
a. “In the absence of any note, memorandum or any other written i. The only portion of his testimony that is true is that he
instrument evidencing the alleged perfected contract of sale, we signed the document. How could the Court believe that he
have to rely on oral testimonies, which in this case is that of Atty. de brought a camera with him just to take pictures of the
Guzman whose testimony on the alleged oral agreement may be signing? If the purpose was to record the proceeding for
summarized as follows: (1) that sometime in the first week of June posterity, why did he not take the picture of Atty. De
1996, Gregorio requested him (Atty. de Guzman) to prepare a deed Guzman when the latter was reading and explaining the
of sale of two lots; (2) Gregorio came to his firms office in the document to Gregorio Balacano? Why did he not take the
morning with a certain Doming Balacano, then returned in the picture of both Gregorio Balacano and Atty. de Guzman
afternoon with Rudy; (3) he (Atty. de Guzman) asked Gregorio while the old man was signing the document instead of
whether he really intends to sell the lots; Gregorio confirmed his taking a picture of Gregorio Balacano alone holding a ball
intention; (4) Gregorio and Rudy left the law office at 5:00 p.m., pen without even showing the document being signed?
leaving the certificates of title; (5) he prepared the deed a day after Verily there is a picture of a document but only a hand with
Rudy (Paragas) and Gregorio came. With regard to the alleged a ball pen is shown with it. Why? Clearly the driver
partial execution of this agreement, Atty. de Guzman said that he Antonio Agcaoili must have only been asked by Rudy
was told by Rudy that there was already a partial payment Paragas to tell a concocted story which he himself would
of P50,000.00. not dare tell in Court under oath
c. Art. 24 also provides that when one of the parties is at a disadvantage
We do not consider Atty. de Guzmans testimony sufficient on account of his moral dependence, ignorance, indigence, mental
evidence to establish the fact that there was a prior agreement weakness, tender age or other handicap, the courts must be vigilant
between Gregorio and the Spouses Paragas on the sale of Lots for his protection”
1175-E and 1175-F. This testimony does not conclusively establish 3. SC affirmed the trial court and the CA’s uniform decision based on the whole
the meeting of the minds between Gregorio and the Spouses Paragas evidence in record holding the Deed of Sale in question to be null and void.
on the price or consideration for the sale of Lots 1175-E and 1175- 4. As in the case of Domingo v. CA, the SC declared the deed of sale null and
F Atty. de Guzman merely declared that he was asked by Gregorio void inasmuch as the seller at the time of the execution of the alleged contract
to prepare a deed; he did not clearly narrate the details of this was already of advanced age and senile.
agreement. We cannot assume that Gregorio and the Spouses a. The general rule is that a person is not incompetent to contract
Paragas agreed to a P500,000.00 consideration based on Atty. de merely because of advanced years or by reason of physical
infirmities. However, when such age or infirmities have impaired
the mental faculties so as to prevent the person from properly,
intelligently, and firmly protecting her property rights then she is
undeniably incapacitated.
5. Gregorio was an octogenarian at the time of the alleged execution of the
contract and suffering from liver cirrhosis at that- circumstances which raise
grave doubts on his physical and mental capacity to freely consent to the
contract.
10 Matabuena v. Cervantes (Paolo) between spouses, should also apply to common-law relationships? - YES
March 31, 1971 | Fernando, J. | Donations between unmarried partners are void
RULING: The lower court decision which dismissed the complaint of Cornelia
Matabuena was reversed. The questioned donation should be void. The plaintiff
PLAINTIFF-APPELLANT: Cornelia Matabuena
and the defendant should be considered pro indiviso heirs to the parcel of land.
DEFENDANT-APPELLEE: Petronila Cervantes
RATIO:
SUMMARY: Felix Matabuena and Petronila Cervantes had a common-law
relationship when Felix decided to donate a parcel of land to Petronila in 1956. They
8. While Article 133 of the Civil Code considers as void a “donation between
got married in 1963, and as the years passed, Felix died. Cornelia Matabuena, the
the spouses during the marriage”, policy considerations of the most exigent
sister of Felix, went after the parcel of land as the nearest collateral relative,
character as well as the dictates of morality require that the same
disputing Petronila's claim of absolute ownership. Cornelia's argument is grounded
prohibition should apply to a common-law relationship.
on Art. 133 of the Civil Code which prohibits spouses from donating property to one
another. Petronila countered this with the fact that she wasn't married to Felix when
9. The Court cited its decision in the case of Buenaventura v. Bautista, where
he donated the parcel of land. The lower court agreed with Petronila. The Supreme
Justice J.B.L. Reyes wrote "to prohibit donations in favor of the other
Court reversed the lower court's decision on the basis that the same prohibition to
consort and his descendants because of fear of undue and improper
married spouses in Art. 133 of the Civil Code should apply to common-law
pressure and influence upon the donor”, which leads to the conclusion that
relationships. The court used principles of statutory construction and prevailing
“there is every reason to apply the same prohibitive policy to persons living
jurisprudence to affirm that the spirit of the Civil Code should trump and literal
together as husband and wife without the benefit of nuptials. Justice Reyes
interpretation of the written provisions that would be repulsive to public policy and
also wrote a line that said: “So long as marriage remains the cornerstone
morality.
of our family law, reason and morality alike demand that the disabilities
attached to marriage should likewise attach to concubinage.”
DOCTRINE: A donation between common-law spouses falls within the prohibition
in Art. 133 of the Civil Code and is null and void for being contrary to public policy.
10. The Court justified this application of the Civil Code provision by using
statutory construction; looking beyond the written text and into the spirit of
FACTS: the law. Whatever omission may be apparent in an interpretation purely
8. Cornelia Matabuena, the sister of the deceased Felix Matabuena, filed a literal of the language used must be remedied by an adherence to its
complaint which claimed that she had absolute ownership of a parcel of land. avowed objective. The frames of the Civil Code would not have allowed
This particular parcel of land was donated by Felix Matabuena to his
such a irregular relationship to have certain advantages over a lawful
common-law partner, Petronila Cervantes, when he was still alive. Felix died relationship
intestate, so the parcel of land's ownership was being questioned by the
parties. 11. The lack of validity of the donation made by the deceased Felix to the
defendant Petronila Cervantes does not necessarily result in plaintiff having
9. Cornelia argues that the deed of donation inter vivos was invalid because of
exclusive right to the disputed property. Prior to the death of Felix
Art. 133 of the Civil Code, which renders donations between spouses during Matabuena, the relationship between him and the defendant was legitimated
the marriage as void. Petronila counters this argument by taking note of the
by their marriage. She is therefore his widow. As provided for in the Civil
year when the donation took place, which was 1956. Petronila and Felix got
Code, Petronila is entitled to one-half of the inheritance as the widow, and
married in 1963, so she argues that the Civil Code provision could not apply the Cornelia, as the surviving sister, to the other half.
to the particular donation at the time it was done.
NOTES:
10. Cornelia says she has a better claim because of being the nearest collateral
relative and that she had declared the land under her name while paying the
• Donation is an act of liberality; Sale is an essentially onerous contract.
estate and inheritance taxes. The lower court ruled in favor of Petronila's
claim because they were not lawfully married when Felix donated the parcel • Donation is a solemn contract; Sale is perfected by mere consent.
of land to her. Cornelia appealed this decision to the Supreme Court. • Distinctions important to know if Law on Sales/Law on Donations appli

ISSUES: Whether Article 133 of the Civil Code, which renders void donations
PHILIPPINE TRUST COMPANY v. SOCORRO ROLDAN claimed that Socorro, in effect, sold to herself the properties of her ward
May 31, 1956 | Bengzon, J. | Purchase of ward’s property by guardian which was in violation of Article 1459 of the Civil Code prohibiting the
guardian from purchasing either in person or through the mediation of another
PETITIONER: Philippine Trust Company, as guardian of the minor Mariano L. the property of her ward. Thus, the sale should be annulled.
Bernardo
RESPONDENTS: Socorro Roldan, Francisco Hermoso, Fidel C. Ramos and 5. The CFI held that there was no proof that Dr. Ramos was a mere intermediary
Emilio Cruz or that the latter had previously agreed with Socorro to buy the
parcels for her benefit. However, taking the former guardian at her word-she
SUMMARY: Mariano Bernardo, a minor, inherited 17 parcels of land from his swore she had repurchased the lands from Dr. Ramos to preserve it and to
deceased father. Soccoro Roldan was appointed as his guardian. Soccoro sought give her protege opportunity to redeem-the court rendered judgment
and was granted authority to sell the lots to her brother-in-law, Dr. Fidel Ramos upholding the contracts but allowing the minor to repurchase all the parcels
for P 14,700. Very shortly after, Dr. Ramos sold back to Soccoro the same by paying P 15,000, within one year. The CA affirmed the judgment.
properties for P 15,000. She then sold 4 parcels to Emilio Cruz. Philippine Trust
Company replaced Soccoro as guardian and sought to annul all the aforesaid sales.
The issue is whether or not the sales are valid. The Court held that the 3 sales ISSUE: Whether or not the sales are valid. -NO
should not be sustained: the first two for violation of Article 1459 of the Civil
Code; and the third because Socorro Roldan could pass no title to Emilio Cruz. RULING: The following is the judgment of the Court:
The Court ruled that Socorro Roldan violated the general doctrine of guardianship a. Annulling the three contracts of sale in question;
when she purchased Mariano’s properties through Dr. Fidel C. Ramos. b. declaring the minor as the owner of the 17 parcels of land, with, the obligation to
Guardianship is a trust of the highest order and the trustee cannot be allowed to return to Socorro Roldan the price of P 14,700 with legal interest from August
have any inducement to neglect his ward's interest. 12, 1947;
c. ordering Socorro Roldan and Emilio Cruz to deliver said parcels of land to the
DOCTRINE: Guardianship is a trust of the highest order and the trustee cannot minor;
be allowed to have any inducement to neglect his ward's interest. d. requiring Socorro Roldan to pay him beginning with 1947 the fruits, which her
attorney admits, amounted to P l,522 a year;
e. authorizing the minor to deliver directly to Emilio Cruz, out of the price of P
FACTS:
14,700 above.
1. Mariano L. Bernardo (Mariano), minor, inherited from his father, Marcelo
Bernardo, 17 parcels located in Guiguinto, Bulacan. In view of his minority,
RATIO:
guardianship proceedings were instituted and Socorro Roldan (Socorro), his
1. The Court ruled that Socorro Roldan violated the general doctrine of
stepmother, was appointed as his guardian.
guardianship when she purchased Mariano’s properties through Dr. Fidel C.
Ramos. Guardianship is a trust of the highest order and the trustee cannot be
2. On July 27, 1947, Socorro filed a motion asking for authority to sell as
allowed to have any inducement to neglect his ward's interest. Therefore,
guardian the 17 parcels for P 14,700 to Dr. Fidel C. Ramos (Dr. Ramos),
Article 1459 of the Civil Code applies.
brother-in-law, the purpose of the sale being allegedly to invest the money in
a residential house, which Mariano desired to have on Tindalo Street, Manila.
2. Although it may be true that she acted without malice and that there may
The motion was granted.
have been no previous agreement between her and Dr. Ramos to the effect
that the latter would buy the lands for her, the stubborn fact remains that she
3. On August 5, 1947 Socorro executed the proper deed of sale in favor of Dr.
acquired her protege's properties, through her brother-in-law. That she
Ramos and on August 12, 1947 she obtained judicial confirmation of the sale.
planned to get them for herself at the time of selling them to Dr. Ramos, may
The next day, Dr. Ramos executed in favor of Socorro a deed of conveyance
be deduced from the very short time between the two sales (one week). The
covering the same 17 parcels for P 15,000. On October 21, 1947 Socorro then
temptation which naturally besets a guardian so circumstanced, necessitates
sold 4 parcels out of the 17 to Emilio Cruz for P 3,000, reserving to herself
the annulment of the transaction, even if no actual collusion is proved (so
the right to repurchase.
hard to prove) between such guardian and the intermediate purchaser. This
would uphold a sound principle of equity and justice.
4. The Philippine Trust Company (PTC) replaced Socorro as guardian, on
August 10, 1948. Thereafter, PTC seeks to undo what Socorro had done. PTC
3. However, the Court held that the doctrine in Rodriguez vs. Mactal is not
applicable since the guardian bought back the property of her ward after two
years. In the case at bar, only one week had elapsed. And if we were technical,
we could say, only one day had elapsed from the judicial approval of the sale
(August 12), to the purchase by the guardian (Aug. 13).

4. Attempting to prove that the transaction was beneficial to the minor,


appellee’s attorney alleges that the money (P 14,700) invested in the house
on Tindalo Street produced for him rentals of P 2,400 yearly; the parcels of
land yielded to his step-mother only an average of P 1,522 per year. The
argument would carry some weight if that house had been built out of the
purchase price of P 14,700 only. One thing is certain: the calculation does not
include the price of the lot on which the house was erected. Estimating such
lot at P 14,700 only, (ordinarily the city lot is more valuable than the building)
the result is that the price paid for the 17 parcels gave the minor an income of
only P 1,200 a year, whereas the harvest from the 17 parcels netted his step-
mother a yearly profit of P 1,522. The minor was thus on the losing end.

5. Hence, from both the legal and equitable standpoints these 3 sales should not
be sustained: the first two for violation of Article 1459 of the Civil Code; and
the third because Socorro Roldan could pass no title to Emilio Cruz. The
annulment carries with it (Article 1303 Civil Code) the obligation of Socorro
Roldan to return the 17 parcels together with their fruits and the duty of the
minor, through his guardian to repay P 14,700 with legal interest.
FABILLO v. INTERMEDIATE APPELLATE COURT a. Filing and sheriffs fee of not less than P86.00
11 March 1991 | Fernan, C.J. | Contracts of Services: Contingent Fees b. 40% of the money value of the house and lot as a contingent fee in
case of success
PETITIONER: Florencio Fabillo & Josefa Tana 4. A contract was then entered into by both parties, stating that:
RESPONDENTS: IAC, Alfred Murillo a. Florencio hired Atty. Murillo as his legal counsel to represent
Florencio and his heirs
SUMMARY: Florencio hired Atty. Murillo as his legal counsel for two cases. b. Atty. Murillo is constituted as attorney-in-fact to sell and convey
Both parties entered into a contract of services which states that as a contingent said huse and lot (he gets 40% of total purchase price if this happens)
fee, Atty. Murillo is entitled to 40% of the properties (please see Fact #4 for c. If said property is mortgaged, Atty. Murillo shall be given 40% the
specificities). sum equivalent of the proceeds
d. 40% of whatever damages Atty. Murillo can collect in the two cases
Florencio, who used to be the owner of the San Salvador property in Leyte filed a he is handling for Florencio; full amount of attorney’s fees if
case for recovery of property against Gregorio. The case was terminated through awarded
a compromise agreement and Florencio was awarded not only said property but e. In case the property is not sold, a 60-40 ownership (same percentage
also the Pugahanay property. for maintenance and tax expenses)
5. Atty. Murillo then filed for Florencio Civil Case No. 3532 against Gregorio
Atty. Murillo proceeded to implement said contract of services. After a while, to recover the San Salvador property. Said case was terminated (through a
Florencio claimed exclusive rights over the properties and refused to give Atty. compromise agreement) and Florencio was awarded both the San Salvador
Murillo what he was entitled to. Florencio’s side claimed that Florencio’s consent and Pugahanay properties)
was vitiated when he entered into said contract (old age). The lower court ruled in 6. Atty. Murillo then proceeded to implement the contract of services by taking
favor of Atty. Murillo stating that there was no proof of said vitiation of consent possession and exercising rights of ownership over 40% of said properties.
(his educated children were even present then). Moreover, it upheld the 40% He also installed a tenant in the Pugahanay property.
contingent fee. 7. In 1966, Florencio claimed exclusive right over both properties and refused
to give Murillo his share of produce.
The SC affirmed the lower court’s decision, stating that said contract did not 8. Murillo then fileda complaint against Florencio, and prayed that he be
violate Section 5 of Article 1491 of the Civil Code as the said prohibition applies declared the lawful owner of 40% of the properties, among others.
only if the sale or assignment of the property takes place during the pendency of 9. Florencio’s side stated that the consent of Florencio to the contract was
the litigation involving the client's property. vitiated by old age, and the contingent fee of 40% was excessive, unfair and
unconscionable.
DOCTRINE: Hence, a contract between a lawyer and his client stipulating a 10. The lower court ruled that there was insufficient evidence to prove that there
contingent fee is not covered by said prohibition under Article 1491 (5) of the was vitiated consent. Moreover, ruling that the contract of services did not
violate Article 1491 of the Civil Code as said contract stipulated a contingent
Civil Code because the payment of said fee is not made during the pendency of
fee, the court upheld Murillo's claim for "contingent attorney's fees of 40%
the litigation but only after judgment has been rendered in the case handled by the of the value of recoverable properties.”
lawyer.
ISSUE/s:
FACTS: 1. W/N the contract of services is in violation of Article 1491 of the Civil Code
1. In her last will and testament dated August 16, 1957, Justina Fabillo – NO.
bequeathed to her brother, Florencio, a house in Leyte, and to her husband,
Gregorio, a piece of land in Leyte as well. (San Salvador property to RULING: WHEREFORE, the decision of the then Intermediate Appellate Court is
Florencio, Pugahanay property to Gregorio). hereby reversed and set aside and a new one entered (a) ordering the petitioners to pay
2. After Justina’s death, Florencio filed a petition for the probate of said will. Atty. Alfredo M. Murillo or his heirs the amount of P3,000.00 as his contingent fee
Probate court approved said petition. with legal interest from October 29, 1964 when Civil Case No. 3532 was terminated
3. Two years later, Florencio sought the assistance of Atty. Murillo in until the amount is fully paid less any and all amounts which Murillo might have
recovering the San Salvador property. Atty. Murillo replied with a received out of the produce or rentals of the Pugahanay and San Salvador properties,
handwritten letter stating: and (b) ordering the receiver of said properties to render a complete report and
accounting of his receivership to the court below within fifteen (15) days from the
finality of this decision. Costs against the private respondent.

RATIO:
1. The contract of services did not violate said provision, because Article 1491,
specifically paragraph 5, prohibits lawyers from acquiring by purchase
even at a public or judicial auction, properties and rights which are the
objects of litigation in which they may take part by virtue of their
profession.
2. The said prohibition, however, applies only if the sale or assignment of the
property takes place during the pendency of the litigation involving the
client's property.
3. Hence, a contract between a lawyer and his client stipulating a contingent fee
is not covered by said prohibition under Article 1491 (5) of the Civil Code
because the payment of said fee is not made during the pendency of the
litigation but only after judgment has been rendered in the case handled by
the lawyer.
4. As long as the lawyer does not exert undue influence on his client, that no
fraud is committed or imposition applied, or that the compensation is clearly
not excessive as to amount to extortion, a contract for contingent fee is valid
and enforceable.
5. Moreover, contingent fees were impliedly sanctioned by No. 13 of the
Canons of Professional Ethics which governed lawyer-client relationships
when the contract of services was entered into between the Fabillo spouses
and Murillo.
6. Although the contract is vague in some aspects, it is against Murillo that it
should be resolved against as it was he himself who drafted the contract.
RUBIAS vs. BATILLER (Elach) updated 7. The same land was claimed by the Batiller (defendant) as his own and was
May 29, 1973 | Teehankee, J. | Prohibited purchase surveyed on June 6 and 7,1956, and a plan approved by Director of Land on
November 15, 1956 was issued, identified as Psu 155241
PETITIONER: Domingo Rubias 8. On April 22, 1960, the plaintiff filed forcible Entry and Detainer case against
RESPONDENTS: Isaias Batiller Isaias Batiller in the Justice of the Peace Court of Barotac Viejo Province of
Iloilo to which the defendant Isaias Batiller riled his answer on August 29,
SUMMARY: Before the war, Francisco Militante filed an application for 1960.
registration of the parcel of land in question. After the war, the petition was heard 9. The Municipal Court of Barotac Viejo after trial, decided the case on May
and denied. Pending appeal, Militante sold the land to petitioner, his son-in-law 10, 1961 in favor of the defendant and against the plaintiff.
and his counsel for the pending application for registration of land, which was 10. The plaintiff appealed from the decision of the Municipal Court which
subsequently denied. Plaintiff filed an action for forcible entry against respondent. decided in favor of the defendant, Isaias Batiller and against the plaintiff
Defendant Batiller claims the complaint of the plaintiff does not state a cause of 11. Rubias tried to prove that the land he purchased from Francisco Militante
action, the truth of the matter being that he and his predecessors-in-interest have under was formerly owned and possessed by Liberato Demontaño but that on
always been in actual, open and continuous possession since time immemorial September 6, 1919 the land was sold at public auction by virtue of a judgment
under claim of ownership of the portions of the lot in question. The issue in this in a Civil Case entitled "Edw J. Pflieder plaintiff vs. Liberato Demontaño
case is whether or not the contract of sale between appellant and his father-in-law Francisco Balladeros and Gregorio Yulo, defendants", of which Yap Pongco
was void because it was made when plaintiff was counsel of his father-in-law in a was the purchaser
land registration case involving the property in dispute. The court rules that it was 12. Batiller on the other hand, proved that lot No. 2 of the Psu-1552 was
indeed void because it fell under the prohibited purchases enumerated in the Civil originally owned and possessed by Felipe Batiller, grandfather of the
Code. defendant Basilio Batiller, on the death of the former in 1920, as his sole heir.
Isaias Batiller succeeded his father , Basilio Batiller, in the ownership and
DOCTRINE: Prohibition against purchase by lawyer of property in litigation possession of the land in the year 1930, and since then up to the present, the
from his client; Prohibited purchases are void and produce no legal effect. land remains in the possession of the defendant, his possession being actual,
open, public, peaceful and continuous in the concept of an owner
FACTS: ISSUE/s
1. Francisco Militante claimed ownership of a parcel of land located in the Whether or not the contract of sale between appellant Rubias and his father-in-law,
province of Iloilo, which he caused to be surveyed on July 18-31, 1934, Francisco Militante, was void because it was made when plaintiff was counsel of his
whereby he was issued a plan Psu-99791 father-in-law in a land registration case involving the property in dispute– YES it is a
2. Before the war with Japan, Francisco Militante filed with the Court of First prohibited sale
Instance of Iloilo an application for the registration of the title of the land
technically described in psu-99791 opposed by the Director of Lands, the RULING: ACCORDINGLY, the order of dismissal appealed from is hereby
Director of Forestry and other oppositors. affirmed, with costs in all instances against plaintiff-appellant. So ordered.
3. However, during the war with Japan, the record of the case was lost before it
was heard, so after the war Francisco Militante petitioned this court to RATIO:
reconstitute the record of the case. The record was reconstituted on the Court 1. The stipulated facts and exhibits of record indisputably established plaintiff's
of the First Instance of Iloilo lack of cause of action and justified the outright dismissal of the complaint.
4. The Court of First Instance heard the land registration case on November 14, Plaintiff's claim of ownership to the land in question was predicated on the
1952, and after the trial this court dismissed the application for registration. sale thereof for P2,000.00 made in 1956 by his father-in- law, Francisco
The appellant, Francisco Militante, appealed from the decision of this Court Militante, in his favor, at a time when Militante's application for registration
to the Court of Appeals thereof had already been dismissed by the Iloilo land registration court and
5. Pending the disposal of the appeal in CA, Francisco Militante sold to the was pending appeal in the Court of Appeals.
plaintiff, Domingo Rubias the land technically described in psu-99791. The 2. With the Court of Appeals' 1958 final judgment affirming the dismissal
sale was duly recorded in the Office of the Register of Deeds for the province of Militante's application for registration, the lack of any rightful claim
of Iloilo or title of Militante to the land was conclusively and decisively judicially
6. The CA promulgated its judgment which dismissed the application for determined. Hence, there was no right or title to the land that could be
Registration filed by Francisco Militante
transferred or sold by Militante's purported sale in 1956 in favor of
plaintiff.
3. No error could be attributed either to the lower court's holding that the
purchase by a lawyer of the property in litigation from his client is
categorically prohibited by Article 1491, paragraph (5) of the Philippine
Civil Code, reproduced and that consequently, plaintiff's purchase of the
property in litigation from his client (assuming that his client could sell the
same since as already shown above, his client's claim to the property was
defeated and rejected) was void and could produce no legal effect, by virtue
of Article 1409, paragraph (7) of our Civil Code which provides that contracts
"expressly prohibited or declared void by law' are "inexistent and that
"(T)hese contracts cannot be ratified. Neither can the right to set up the
defense of illegality be waived."
4. Article 1491 says that “The following persons cannot acquire any purchase,
even at a publicor judicial auction, either in person or through the mediation
of another…. (5) Justices, judges, prosecuting attorneys, clerks of superior
and inferiorcourts, and other officers and employees connected with the
administration of justice, the property andrights in litigation or levied upon
an execution beforethe court within whose jurisdiction or territory
theyexercise their respective functions; this prohibitionincludes the act of
acquiring by assignment and shallapply to lawyesr, with respect to the
property and rights which may be the object of any litigation inwhich they
may take part by virtue of their profession.”
5. The present case clearly falls under this, especially since the case was still
pending appeal when the sale was made
6. Fundamental consideration of public policy render void and inexistent such
expressly prohibited purchase has been adopted in a new article of our Civil
Code, viz, Article 1409 declaring such prohibited contracts as "inexistent and
void from the beginning.”
7. Indeed, the nullity of such prohibited contracts is definite and permanent and
cannot be cured by ratification. The public interest and public policy remain
paramount and do not permit of compromise or ratification.
8. In his aspect, the permanent disqualification of public and judicial officers
and lawyers grounded on public policy differs from the first three cases of
guardians, agents and administrators (Article 1491, Civil Code), as to whose
transactions it had been opined that they may be "ratified" by means of and
in "the form of a new contact, in which cases its validity shall be determined
only by the circumstances at the time the execution of such new contract.
9. The causes of nullity which have ceased to exist cannot impair the validity of
the new contract. Thus, the object which was illegal at the time of the first
contract, may have already become lawful at the time of the ratification or
second contract; or the service which was impossible may have become
possible; or the intention which could not be ascertained may have been
clarified by the parties. The ratification or second contract would then be
valid from its execution; however, it does not retroact to the date of the first
contract."
PUP v. CA (CYRIA) hectare property located along Pureza St., Sta. Mesa, Manila. The estate was
November 14, 2001 | Bellosillo, J. | Price must be in money or its equivalent popularly known as the NDC compound and covered by Transfer Certificates
of Title Nos. 92885, 110301 and 145470.
PETITIONER: Polytechnic University of the Philippines 12. On 24 August 1965 NDC and FIRESTONE entered into a contract of lease
RESPONDENTS: Court of Appeals and Firestone Ceramics denominated as Contract No. C-30-65 covering a portion of the property
measured at 2.90118 hectares for use as a manufacturing plant for a term of
SUMMARY: NDC and Firestone entered into several contracts of lease of a ten (10) years, renewable for another ten (10) years under the same terms and
parcel of land located in Sta. Mesa, Manila (NDC Compound). The first contract conditions. In consequence of the agreement, FIRESTONE constructed on
covered 2.9 hectares of the NDC Compound for use as manufacturing plant for 10 the leased premises several warehouses and other improvements needed for
years, renewable for another 10 years. The second contract involved four NDC’s the fabrication of ceramic products.
steel warehouse in Davao to be shipped in Manila and used in the NDC Compound 13. Three and a half (3-1/2) years later, or on 8 January 1969, FIRESTONE
as well as 2.6 hectare-lot. The 3rd contract involved lease of six steel warehouse. entered into a second contract of lease with NDC over the latter's four (4)-
Prior to the expiration of the 3rd contract, NDC and Firestone entered into a new unit pre-fabricated reparation steel warehouse stored in Daliao,
agreement of lease for 10 years, renewable for another 10 years and expressly Davao. FIRESTONE agreed to ship the warehouse to Manila for eventual
granting Firestone the first option to purchase the leased premises in the event that assembly within the NDC compound. The second contract, denominated as
NDC decides to sell its properties incuding the lot. Before the expiration of the Contract No. C-26-68, was for similar use as a ceramic manufacturing plant
“new agreement,” Firestone notified NDC of its plan of renewing the contract. and was agreed expressly to be "co-extensive with the lease of LESSEE with
Firestone then learned of the rumors of NDC’s plan to dispose of the property in LESSOR on the 2.60 hectare-lot."
favor of PUP. Firestone filed a complaint for specific performance to compel NDC 14. On 31 July 1974 the parties signed a similar contract concerning a six (6)-
to sell the land in its favor since Firestone has the right of refusal. On the other unit pre-fabricated steel warehouse which, as agreed upon by the parties,
hand, PUP referred to Memorandum Order No. 214 issued by President Cory would expire on 2 December 1978. Prior to the expiration of the
Aquino ordering the transfer of the whole NDC compound to the National aforementioned contract, FIRESTONE wrote NDC requesting for an
Government, which in turn would convey the property to PUP. PUP also argues extension of their lease agreement. Consequently on 29 November 1978 the
that the lease contract between Firestone and NDC had expired already and Board of Directors of NDC adopted Resolution No. 11-78-117 extending the
Firestone’s right of first refusal applied only to the six-unit pre-fabricated term of the lease, subject to several conditions among which was that in the
warehouse and not the lot upon which it stood. event NDC "with the approval of higher authorities, decide to dispose and
sell these properties including the lot, priority should be given to the
The SC ruled that there was sale between NDC and PUP as the three requisites LESSEE".
were present – (1) consent – as seen in the Memorandum Order NO. 214, (2) object 15. On 22 December 1978, in pursuance of the resolution, the parties entered into
– NDC Compound, (3) consideration – cancellation of NDC’s liabilities in a new agreement for a ten-year lease of the property, renewable for another
favor of the National Government. Firestone can exercise its right of first ten (10) years, expressly granting FIRESTONE the first option to purchase
refusal. Lastly, the consideration should only be P554.74 but since Firestone did the leased premises in the event that it decided "to dispose and sell these
not appeal the RTC’s ruling re the consideration (P1500), there is no reason to properties including the lot . . . . "
modify it. 16. The contracts of lease conspicuously contain an identically worded provision
requiring FIRESTONE to construct buildings and other improvements within
DOCTRINE: The Civil Code provision is, in effect, a "catch-all" provision which the leased premises worth several hundred thousands of pesos.
effectively brings within its grasp a whole gamut of transfers whereby ownership 17. The parties' lessor-lessee relationship went smoothly until early 1988 when
of a thing is ceded for a consideration. FIRESTONE, cognizant of the impending expiration of their lease agreement
with NDC, informed the latter through several letters and telephone calls that
When a lease contract contains a right of first refusal, the lessor is under a legal it was renewing its lease over the property. While its letter of 17 March 1988
duty to the lessee not to sell to anybody at any price until after he has made an was answered by Antonio A. Henson, General Manager of NDC, who
offer to sell to the latter at a certain price and the lessee has failed to accept it. promised immediate action on the matter, the rest of its communications
remained unacknowledged.
18. FIRESTONE's predicament worsened when rumors of NDC's supposed plans
FACTS: to dispose of the subject property in favor of petitioner Polytechnic
11. In the early sixties, petitioner National Development Corporation (NDC), a University of the Philippines (PUP) came to its knowledge. Forthwith,
government owned and controlled corporation had in its disposal a ten (10)-
FIRESTONE served notice on NDC conveying its desire to purchase the a. PUP was ordered and directed to sell to FIRESTONE the "2.6
property in the exercise of its contractual right of first refusal. hectare leased premises or as may be determined by actual
19. Apprehensive that its interest in the property would be disregarded, verification and survey of the actual size of the leased properties
FIRESTONE instituted an action for specific performance to compel NDC to where plaintiff's fire brick factory is located" at P1,500.00 per
sell the leased property in its favor. FIRESTONE averred that it was pre- square meter considering that, as admitted by FIRESTONE, such
empting the impending sale of the NDC compound to petitioner PUP in was the prevailing market price thereof.
violation of its leasehold rights over the 2.60-hectare property and the b. Contracts of lease executed between FIRESTONE and NDC were
warehouses thereon which would expire in 1999. interrelated and inseparable because "each of them forms part of the
20. FIRESTONE adduced in evidence a letter of Antonio A. Henson dated 15 integral system of plaintiff's brick manufacturing plant x x x if one
July 1988 addressed to Mr. Jake C. Lagonera, Director and Special Assistant of the leased premises will be taken apart or otherwise detached
to Executive Secretary Catalino Macaraeg, reviewing a proposed from the two others, the purpose of the lease as well as plaintiff's
memorandum order submitted to then President Corazon C. Aquino business operations would be rendered useless and inoperative." It
transferring the whole NDC compound, including the leased property, in thus decreed that FIRESTONE could exercise its option to purchase
favor of petitioner PUP. Attached to the letter was a draft of the proposed the property until 2 June 1999 inasmuch as the 22 December 1978
memorandum order as well as a summary of existing leases on the subject contract embodied a covenant to renew the lease for another ten (10)
property. The survey listed FIRESTONE as lessee of a portion of the years at the option of the lessee as well as an agreement giving the
property, whose contract with NDC was set to expire on 31 December 1989 lessee the right of first refusal.
renewable for another ten (10) years at the option of the lessee. The report c. The trial court also sustained the constitutionality of Memorandum
expressly recognized FIRESTONE's right of first refusal to purchase the Order No. 214 which was not per se hostile to FIRESTONE's
leased property "should the lessor decide to sell the same." property rights, but deplored as prejudicial thereto the "very manner
21. Meanwhile, on 21 February 1989 PUP moved to intervene. PUP referred with which defendants NDC and PUP interpreted and applied the
to Memorandum Order No. 214 issued by then President Aquino ordering same, ignoring in the process that plaintiff has existing contracts of
the transfer of the whole NDC compound to the National Government, which lease protectable by express provisions in the Memorandum No. 214
in turn would convey the aforementioned property in favor of PUP at itself." It further explained that the questioned memorandum was
acquisition cost. The issuance was supposedly made in recognition of PUP's issued "subject to such liens/leases existing thereon" and petitioner
status as the "Poor Man's University" as well as its serious need to extend its PUP was under express instructions "to enter, occupy and take
campus in order to accommodate the growing student population. The order possession of the transferred property subject to such leases or liens
of conveyance of the 10.31-hectare property would automatically result in and encumbrances that may be existing thereon"
the cancellation of NDC's total obligation in favor of the National 25. CA:
Government in the amount of P57,193,201.64. a.
FIRESTONE was given a grace period of six (6) months from
22. FIRESTONE sought the annulment of Memorandum Order No. finality of the court's judgment within which to purchase the
214. FIRESTONE alleged that although Memorandum Order No. 214 was property in questioned in the exercise of its right of first refusal.
issued "subject to such liens/leases existing [on the subject property]," PUP b. As there was a sale of the subject property, NDC could not excuse
disregarded and violated its existing lease by increasing the rental rate itself from its obligation TO OFFER THE PROPERTY FOR SALE
at P200,000.00 a month while demanding that it vacated the premises FIRST TO FIRESTONE BEFORE IT COULD TO OTHER
immediately. In the event Memorandum Order No. 214 was not declared PARTIES.
unconstitutional, the property should be sold in its favor at the price for which 26. PUP argues that
it was sold to PUP - P554.74 per square meter or for a total purchase price a. In ordering the sale of the property in favor of FIRESTONE the
of P14,423,240.00. courts a quo unfairly created a contract to sell between the parties.
23. Petitioner PUP, in its answer to the amended complaint, argued in essence b. The "court cannot substitute or decree its mind or consent for that of
that the lease contract covering the property had expired long before the the parties in determining whether or not a contract (has been)
institution of the complaint, and that further, the right of first refusal invoked perfected between PUP and NDC."
by FIRESTONE applied solely to the six-unit pre-fabricated warehouse and c. Since "a real property located in Sta. Mesa can readily command a
not the lot upon which it stood. sum of P10,000.00 per square (meter)," the lower court gravely
24. RTC: erred in ordering the sale of the property at only P1,500.00 per
square meter.
d. The enactment of Memorandum Order No. 214 amounted to a 3. Contrary to what petitioners PUP and NDC propose, there is not just one
withdrawal of the option to purchase the property granted to party involved in the questioned transaction. Petitioners NDC and PUP have
FIRESTONE. NDC, for its part, vigorously contended that the their respective charters and therefore each possesses a separate and distinct
contracts of lease executed between the parties had expired without individual personality. The inherent weakness of NDCs proposition that there
being renewed by FIRESTONE; consequently, FIRESTONE was was no sale as it was only the government which was involved in the
no longer entitled to any preferential right in the sale or disposition transaction thus reveals itself. Beyond cavil, a government owned and
of the leased property. controlled corporation has a personality of its own, distinct and separate from
that of the government.
ISSUE/s: 4. Since the conduct of the parties to a contract may be sufficient to establish
2. WoN there was sale between NDC and PUP – YES (impt) the existence of an agreement and the terms thereof, it becomes necessary for
3. WoN Firestone should be allowed to exercise its right of first refusal – YES the courts to examine the contemporaneous behavior of the parties in
4. WoN the courts a quo were correct in fixing the proper consideration of the establishing the existence of their contract.
sale at P1,500.00 per square meter 5. The preponderance of evidence shows that NDC sold to PUP the whole NDC
compound, including the leased premises, without the knowledge much less
RULING: DENIED. Inasmuch as the first contract of lease fixed the area of the leased consent of private respondent FIRESTONE which had a valid and existing
premises at 2.90118 hectares while the second contract placed it at 2.60 hectares, let a right of first refusal.
ground survey of the leased premises be immediately conducted by a duly licensed, 6. All three (3) essential elements of a valid sale, without which there can be no
registered surveyor at the expense of private respondent FIRESTONE CERAMICS, sale, were attendant in the "disposition" and "transfer" of the property from
INC., within two (2) months from finality of the judgment in this case. Thereafter, NDC to PUP - consent of the parties, determinate subject
private respondent FIRESTONE CERAMICS, INC., shall have six (6) months from matter, and consideration therefor.
receipt of the approved survey within which to exercise its right to purchase the leased a. Consent to the sale is obvious from the prefatory clauses
property at P1,500.00 per square meter, and petitioner Polytechnic University of the of Memorandum Order No. 214 which explicitly states the
Philippines is ordered to reconvey the property to FIRESTONE CERAMICS, INC., in acquiescence of the parties to the sale of the property - WHEREAS,
the exercise of its right of first refusal upon payment of the purchase price thereof. PUP has expressed its willingness to acquire said NDC properties
and NDC has expressed its willingness to sell the properties to
RATIO: PUP.
Sale between NDC and PUP b. The cancellation of NDC's liabilities in favor of the National
1. Petitioner PUP posited that if we were to place our imprimatur on the Government in the amount of P57,193,201.64 constituted the
decisions of the courts a quo, "public welfare or specifically the "consideration" for the sale. For it is axiomatic that every sale
constitutional priority accorded to education" would greatly be prejudiced. imposes upon the vendor the obligation to transfer ownership as an
Paradoxically, our paramount interest in education does not license us, or essential element of the contract. Transfer of title or an agreement
any party for that matter, to destroy the sanctity of binding to transfer title for a price paid, or promised to be paid, is the very
obligations. Education may be prioritized for legislative or budgetary essence of sale.
purposes, but we doubt if such importance can be used to confiscate private c. What is more, the conduct of petitioner PUP immediately after the
property such as FIRESTONE's right of first refusal. transaction is in itself an admission that there was a sale of the NDC
2. A contract of sale, as defined in the Civil Code, is a contract where one of the compound in its favor. Thus, after the issuance of Memorandum
parties obligates himself to transfer the ownership of and to deliver a Order No. 214 petitioner PUP asserted its ownership over the
determinate thing to the other or others who shall pay therefore a sum certain property by posting notices within the compound advising residents
in money or its equivalent. It is therefore a general requisite for the existence and occupants to vacate the premises. In its Motion for
of a valid and enforceable contract of sale that it be mutually obligatory, i.e., Intervention petitioner PUP also admitted that its interest as a
there should be a concurrence of the promise of the vendor to sell a "purchaser pendente lite" would be better protected if it was joined
determinate thing and the promise of the vendee to receive and pay for the as party-defendant in the controversy thereby confessing that it
property so delivered and transferred. The Civil Code provision is, in effect, indeed purchased the property.
a "catch-all" provision which effectively brings within its grasp a whole
gamut of transfers whereby ownership of a thing is ceded for a Firestone’s right of first refusal
consideration.
1. Firestone’s right of first refusal was expressly stated by NDC and the property to a third person, again, under the same terms as offered to the
FIRESTONE in par. XV of their third contract which, as found by the grantee.
courts a quo, was interrelated to and inseparable from their first contract and 2. It appearing that the whole NDC compound was sold to PUP for P554.74
their second contract. per square meter, it would have been more proper for the courts below
a. Should the LESSOR desire to sell the leased premises to have ordered the sale of the property also at the same price. However,
during the term of this Agreement, or any extension since FIRESTONE never raised this as an issue, while on the other hand
it admitted that the value of the property stood at P1,500.00 per square
thereof, the LESSOR shall first give to the LESSEE, meter, then we see no compelling reason to modify the holdings of the
which shall have the right of first option to purchase the courts a quothat the leased premises be sold at that price.
leased premises subject to mutual agreement of both
parties. 3. One final word. Petitioner PUP should be cautioned against bidding for
2. In the instant case, the right of first refusal is an integral and indivisible public sympathy by bewailing the dismissal of its petition before the
part of the contract of lease and is inseparable from the whole press. Such advocacy is not likely to elicit the compassion of this Court or of
contract. The consideration for the right is built into the reciprocal any court for that matter. An entreaty for a favorable disposition of a case not
obligations of the parties. Thus, it is not correct for petitioners to insist that made directly through pleadings and oral arguments before the courts do not
there was no consideration paid by FIRESTONE to entitle it to the exercise persuade us, for as judges, we are ruled only by our forsworn duty to give
of the right, inasmuch as the stipulation is part and parcel of the contract of justice where justice is due.
lease making the consideration for the lease the same as that for the option.
3. It is a settled principle in civil law that when a lease contract contains a
right of first refusal, the lessor is under a legal duty to the lessee not to
sell to anybody at any price until after he has made an offer to sell to the
latter at a certain price and the lessee has failed to accept it. The lessee
has a right that the lessor's first offer shall be in his favor.
4. Consistent with their agreement, it was then implicit for NDC to have first
offered the leased premises of 2.60 hectares to FIRESTONE prior to the sale
in favor of PUP. Only if FIRESTONE failed to exercise its right of first
priority could NDC lawfully sell the property to petitioner PUP.
5. Our attention is invited by petitioners to Ang Yu Asuncion v. CA in
concluding that if our holding in Ang Yu would be applied to the facts of this
case then FIRESTONE's "option, if still subsisting, is not enforceable," the
option being merely a preparatory contract which cannot be enforced.
6. The contention has no merit. At the heels of Ang Yu came Equatorial Realty
Development, Inc., v. Mayfair Theater, Inc., where after much deliberation
we declared, and so we hold, that a right of first refusal is neither
"amorphous nor merely preparatory" and can be enforced and executed
according to its terms. Thus, in Equatorial we ordered the rescission of the
sale which was made in violation of the lessee's right of first refusal and
further ordered the sale of the leased property in favor of Mayfair Theater, as
grantee of the right.

P1,500 per square meter consideration of the sale


1. In contracts of sale, the basis of the right of first refusal must be the current
offer of the seller to sell or the offer to purchase of the prospective
buyer. Only after the lessee-grantee fails to exercise its right under the same
terms and within the period contemplated can the owner validly offer to sell
ATILANO vs. ATILANO (GUECO) RULING: WHEREFORE, the judgment appealed from is reversed. The plaintiffs are
May 21, 1969 | Makalintal, J. | ordered to execute a deed of conveyance of lot No. 535-E in favor of the defendants,
and the latter in turn, are ordered to execute a similar document, covering lot No. 595-
PETITIONER: Asuncion Atilano, Cristina Atilano, Rosario Atilano, Hilario A, in favor of the plaintiffs.
Romano, Feliper Bernardo, Maximo Lacandalo, Isabel Atilano and Gregorio
Atilano RATIO:
RESPONDENTS: Ladislao Atilano and Gregorio M. Atilano
1. What took place was a simple mistake in drafting the instrument evidencing
SUMMARY: Eulogio I purchased Lot 535 and had it subdivided into 5 parts (A the agreement between the brothers. One sells or buys property as he sees it
to E). He occupied Lot A and sold Lot E to his brother, Eulogio II. Both brothers in actual setting and not by the mere lot number in the certificate of title. The
died and their heirs found out after a survey that Eulogio I actually occupied Lot brothers remained in possession of their respective portions throughout their
E and Eulogio II occupied Lot A. Thus, the heirs of Eulogio II offered to exchange lives unaware of the mistake in the designation of the lots. In this case, the
the properties. However, the heirs of Eulogio I refused because Lot E was bigger instrument simply failed to reflect the true intention of the parties; thus, an
than Lot A. The heirs of Eulogio II then filed an action to have Lot E returned. exchange of the properties is unnecessary. All the heirs should do is to
ISSUE: WON an exchange of the properties should be made. The SC ruled in the execute mutual deeds of conveyance.
negative (See doctrine for the ratio).

DOCTRINE:
What took place was a simple mistake in drafting the instrument evidencing the
agreement between the brothers. One sells or buys property as he sees it in actual
setting and not by the mere lot number in the certificate of title. The brothers
remained in possession of their respective portions throughout their lives unaware
of the mistake in the designation of the lots. In this case, the instrument simply
failed to reflect the true intention of the parties; thus, an exchange of the properties
is unnecessary. All the heirs should do is to execute mutual deeds of conveyance.

FACTS:
1. Eulogio Atilano I (“Eulogio I”) purchased Lot 535 from Gerardo Villanueva.
Afterwards, he had the lot subdivided into five parts (A to E).
2. Among the subdivided lots, Eulogio I occupied Lot A and sold lot E to his
brother, Eulogio II. He also sold B, C, and D to other persons.
3. Eulogio II’s wife died and his children obtained title over lot E. They also
had the land resurveyed so that it could properly be subdivided.
4. It was then that they discovered that the land they were actually occupying
was lot A and not lot E. Meanwhile, the land which remained in the
possession of Eulogio I, and which passed to his successors was lot E and not
lot A.
5. The heirs of Eulogio II then offered to exchange the properties. However, the
heirs of Eulogio I refused because Lot E was bigger than Lot A.
6. The heirs of Eulogio II then filed an action to have Lot E returned.
7. The trial court ruled in favor of Eulogio II.

ISSUES:
1. WoN an exchange of the properties should be made—NO.
Yu Tek & Co v Gonzales (Reine) damages.
February 1, 1915 | Trent, J. | Perfected Sale 2. Yu Tek & Co. proved that no sugar had been delivered to it under the
contract nor had it been able to recover the P3000. Yu Tek and Co prayed
PETITIONER: Yu Tek & Co for the judgment for the P3000 and in addition P1200. Judgment was
RESPONDENTS: Basilio Gonzales rendered for P3000 only and from this judgment both, Gonzales & Yu
Tek & Co appealed.
SUMMARY: Basilio Gonzales (Gonzales) received P3000 from Yu Tek & Co 3. First, according to Gonzales, the court erred in refusing to permit parol
and in consideration of said sum, Gonzales obligated himself to deliver to Yu Tek evidence showing that the parties intended that the sugar was to be
& Co 600 piculs of sugar. In the contract, it states that in case Gonzales does not secured from the crop which Gonzales raised on his plantation, and that
deliver said piculs of sugar, the contract will be rescinded and Gonzales will be he was unable to fulfill the contract by reason of the almost total failure
obligated to return to Yu Tek & Co the 3000 received and indemnity for loss and of his crop.
damages in the sum of P1200. 4. However, there is not the slightest intimation in the contract that the
sugar was to be raised by Gonzales. Parties are presumed to have reduced
There was no delivery of sugar by Gonzales hence Yu Tek & Co prayed for the to writing all the essential conditions of their contract. While parol
return of the P3000 plus the P1200. Judgment was rendered for P3000 only and evidence is admissible in a variety of ways to explain the meaning of
due to this, both Gonzales and Yu Tek & Co appealed. written contracts, it cannot serve the purpose of incorporating into the
contract additional contemporaneous conditions which are not
The Court ruled that there was no perfected sale, only a promise of sale since the mentioned at all in the writing, unless there has been fraud or mistake.
thing, which was “sugar” was simply a generic name and there was no 5. Gonzales undertook to deliver a specified quantity of sugar within a
appropriation for a particular lot of sugar. Furthermore, Gonzales must pay the specified time. The contract placed no restriction upon Gonzales in the
P1200 aside from the P3000 by way of indemnity for loss and damages. matter of obtaining the sugar. He was equally at liberty to purchase it on
the market or raise it himself. It may be true that Gonzales owned a
DOCTRINE: Art. 1450: The sale shall be perfected between vendor and plantation and expected to raise the sugar himself, but he did not limit
vendee and shall be binding on both of them, if they have agreed upon the his obligation to his own crop of sugar. The rights of the parties must be
thing which is the object of the contract and upon the price, even when determined by the writing itself.
neither has been delivered. 6. Second, Gonzales assumes that the contract was limited to the sugar he
There was no perfected sale, merely an executory agreement, a promise of sale. might raise upon his own plantation; that the contract represented a
Gonzales merely used the generic name for the thing sold and there was no perfected sale; and that by failure of his crop he was relieved from
appropriation of any particular lot of sugar. complying with his undertaking by loss of the thing due.

ISSUE/s:
FACTS:
5. WoN there was a perfected sale?- NO (Promise of sale only)
1. A written contract (Exhibit A) was the basis of the contract wherein the
6. WoN Yu Tek & Co can recover the additional sum of P1200?- YES
pertinent portion contain:
a.) Mr. Basilio Gonzales (Gonzales) acknowledges the receipt of the
RULING: The judgment appealed from is modified by allowing the recovery of
sum of P3000 from Messrs. Yu Tek and Co and in consideration of
P1,200 as stipulated in the contract.
said sum, Gonzales obligates himself to deliver to Yu Tek & Co 600
piculs of sugar according to the result of polarization within the
RATIO:
period of 3 months beginning on Jan 1, 1912 and ending on March
7. Art. 1450: The sale shall be perfected between vendor and vendee and
31, 1912.
shall be binding on both of them, if they have agreed upon the thing
b.) Gonzales obligates himself to deliver to Yu Tek & Co. the 600 piculs
which is the object of the contract and upon the price, even when neither
of sugar at any place within the municipality of Santa Rosa which
has been delivered.
the Yu Tek & Co or a representative of the same may designate.
8. The contract was merely an executory agreement, a promise of sale and
c.) In case Gonzales does not deliver to Yu Tek & Co the 600 piculs of
not a sale. Therefore, Art. 1452: "The injury to or the profit of the thing
sugar within the period of 3 months, the contract will be rescinded
sold shall, after the contract has been perfected, be governed by the
and Gonzales will be obligated to return to Yu Tek & Co the P3000
provisions of articles 1096 and 1182"is not applicable
received and the sum of P1200 by way of indemnity for loss and
9. Gonzales having defaulted in his engagement, Yu Tek & Co is entitled to
recover the P3000 which Yu Tek & Co advanced to Gonzales.
With regard to thing:
1. There is a perfected sale with regard to the "thing" whenever the article of
sale has been physically segregated from all other articles.
2. It is clear that Gonzales could only say that it was "sugar." He could only use
this generic name for the thing sold. There was no "appropriation" of any
particular lot of sugar.
Recovery of P1200:
1. The contract plainly states that if Gonzales fails to deliver the 600 piculs of
sugar within the time agreed on, the contract will be rescinded and he will be
obliged to return the P3,000 and pay the sum of P1,200 by way of indemnity
for loss and damages. There cannot be the slightest doubt about the meaning
of this language or the intention of the parties.
Melliza v. Iloilo (Steph) to the Government Municipal of Iloilo the lots and portions of the
April 30, 1968 | Bengzon, J.P., J. | Arellano Plan, an object is determinate if there is same that next are specified to know: the lot No. 5 in all their
no need for another contract to determine what the buyer and seller are talking about. extension; a portion of 7669 square meters Lot No. 2, which portion
is designated as sub-batches Nos. 2-B and 2-C piano subdivision of
PETITIONER: Pio Sian Melliza (notary public) the lots prepared by the Certainty Surveying Co., Inc., and a portion
RESPONDENTS: City of Iloilo, University of the Philippines, the Court of of 10,788 square meters lot No. 1214 - which portion is designated
Appeals as sublots 1214-B-2 and B-3 1214-the same plane subdivision..
SUMMARY: Pio Sian Melliza has a TCT over a parcel of land, over which UP Also Nago noted that the assignment and handover ariba mentioned
Iloilo presently stands. He filed an action to recover the lot, but UP claims that is difinitiva sale, and for the better identification of lots and portions
the lot was bought by the City Government of Iloilo, which bought the said lot thereof which are the subject of this, I note that these lots and lots
from Juliana Melliza (see fact 6 for excerpt of instrument) for the construction of are needed by the Municipal Government of Iloilo for the
the municipal hall with parks and avenues, according to the Arellano Plan. Both construction of avenues, parks and City Hall site of the
the CFI and the CA ruled in favor of UP Iloilo, that they are the rightful owners Municipal Government Center of iloilo, according to the
of the land. The issue in this case is whether or not the instrument executed in fact Arellano plan.
6 included the lot occupied by UP Ilolo, and whether or not the instrument covers 7. Later Juliana sold her remaining interest in lot1214 to Remedios Sian
a determinable thing. The SC affirmed the CFI and CA rulings, stating that Villanueva, who registered it through a TCT.
according to the Art. 1273 of the Civil Code an object is determinable if there is 8. Remedios Sian Villanueva transferred her rights in that parcel of land to Pio
no need to execute another agreement/contract to determine which thing the seller Sian Melliza (petitioner) who got TCT over it. Annotated at the back of said
and buyer are talking about. The SC said that because the instrument referred to TCT is the following:
whatever lots were needed to follow the Arellano Plan, which was in existence ... (a) that a portion of 10,788 square meters of Lot 1214 now designated as
even before the questioned instrument, there is no need for another contract to Lots Nos. 1214-B-2 and 1214-B-3 of the subdivision plan belongs to the
determine what lot the seller intended to sell, and therefore the object is Municipality of Iloilo as per instrument dated November 15, 1932 (the
determinate. one referred to in fact 6)
9. The City of Iloilo, which succeeded the Municipality of Iloilo, donated the
DOCTRINE: The object of the sale is a determinate object when there is no need city hall site with the building to UP Iloilo (it consisted of Lot 1214B, 1214C,
for a new agreement between the parties to designate which thing the seller wants 1214D).
to sell and the buyer wants to buy, in accordance with Art. 1273 of the Civil Code. 10. UP then enclosed the site donated with a wire fence.
11. Pio Sian Melliza made representations through his lawyer, with the city
FACTS: authourity for payment of Lot1214B. He was not able to get any money.
1. Juliana Melliza owned three parcels of residential land in Iloilo, registeredin 12. The lands were subsequently registered under UP’s name through TCT.
her name under an OCT. These lots are numbered 2, 5, 1214. Lot 1214 was 13. Pio Sian Melliza filed an action in CFI against UP to recover Lot1214B or of
the biggest (29 073 sq m). its value.
2. She donated a part of it (9 000 sq m) of lot 1214 to serve as the site ofr 14. UP claims that Lot1214B was included in the public instrument executed by
municipal city hall. Juliana Melliza in favor of Iloilo municipality, and the CFI ruled in favor of
3. However, this donation was revoked because the portion donated was found UP, pointing to bold portion of fact 6.
too small to meet the plan (the Arellano Plan). 15. Pio appealed to CA, and CA affirmed CFI ruling, but ordered the remand
4. So, Lot 1214 was divided by Certeza Surveying Co. Inc. into lots 1214 A and of the case to determine which parts of the land Iloilo needed to construct
1214B. avenues and parks for the city hall site.
5. Lot 1214 B was further divided into lots 1214B1, 1214B2, and 1214D. 16. Pio now appeals to SC, maintaining that Lot 1214B is excluded from that
6. After division, Juliana executed an instrument without any caption (original instrument—that to adhere to CFI and CA interpretation would render
in Spanish, that’s google translate hehe. Basically she sold a bigger portion the contract invalid because the law requires that a sale must involve a
to Iloilo government!) “determinate object”.
In consideration of the sum total of SIX HUNDRED TWENTY 17. UP says that the present appeal only concerns questions of fact, and that the
THOUSAND PESOS FOUR (P6,422.00), Philippine currency document in question really intended to include Lot 1214B. To exclude
hereby state that I received to my satisfaction the Municipal Lot1214B would be absurd because they’re all contiguous. They also rebut
Government of Iloilo, I give and transfer real and difinitiva sale Pio’s claims and say that the object of the sale is determinate because it could
be ascertained what lots were needed by Iloilo municipality for avenues,
parks, and the city hall site at the time of the execution for the contract
ISSUES:
1. WoN the conveyance by Juliana Melliza to Iloilo municipality included Lot
1214B – YES.
2. WoN the instrument covers a determinable object and is therefore valid –
YES.

RULING: WHEREFORE, the decision appealed from is affirmed insofar as it


affirms that of the Court of First Instance, and the complaint in this case is dismissed.
No costs. So ordered.

RATIO:

1. Reading the public instrument in toto, with special reference to the


paragraphs describing the lots included in the sale, shows that said instrument
describes four parcels of land by their lot numbers and area; and then it goes
on to further describe, not only those lots already mentioned, but the
lots object of the sale, by stating that said lots are the ones needed for the
construction of the city hall site, avenues and parks according to the Arellano
plan.
2. The object of the sale is a determinate object because there is no need for a
new agreement between the parties to designate which thing the seller wants
to sell and the buyer wants to buy, in accordance with Art. 1273 of the Civil
Code. (in other words, they were on the same page from the start, referring
to the same thing, without the need to execute another agreement/contract).
3. The Arellano plan was already in existence before the public instrument
conveying the questioned lot was executed.
4. The Arellano plan provides enough guidance to ascertain which lots the seller
intended to sell to make sure Iloilo had a municipal hall with avenues and
parks.
5. Furthermore, Pio Sian Melliza, from the stipulation of facts, was the notary
public of the public instrument. As such, he was aware of its terms. Said
instrument was also registered with the Register of Deeds and such
registration was annotated at the back of the corresponding title certificate of
Juliana Melliza. From these stipulated facts, it can be inferred that Pio
Sian Melliza knew of the aforesaid terms of the instrument or is
chargeable with knowledge of them; that knowing so, he should have
examined the Arellano plan in relation to the public instrument.
6. The fact remains that, instead, for twenty long years, Pio Sian Melliza and
his predecessors-in-interest, did not object to said possession, nor exercise
any act of possession over Lot 1214-B. Applying, therefore, principles of
civil law, as well as laches, estoppel, and equity, said lot must necessarily be
deemed included in the conveyance in favor of Iloilo municipality, now Iloilo
City.
NGA v. IAC (Arielle) Certificate.
March 8, 1989 | Medialdea, J. | Contract of Sale 4. Soriano’s certificate was processed and he was given a quota of 2,640 cavans
of palay. The quota represented the maximum number of palay that Soriano
PETITIONER: National Grains Authority can sell to NFA.
RESPONDENTS: Intermediate Appellate Court 5. Soriano then delivered 630 cavans of palay. The palay delivered were not
rebagged, classified, and weighed. When Soriano demanded for payment, he
SUMMARY: Soriano offered to sell palay to the NFA and submitted the required was informed that the payment will be held in abeyance since Mr. Cabal was
documents. Soriano’s certificate was processed and was given a quota of 2,640 investigating on an information he received that Soriano was not a real farmer
cavans of palay. This represented the maximum number of palay that Soriano can and that the palay he delivered was taken from the warehouse of a rice trader,
sell to NFA. Soriano delivered 630 cavans of palay but the same were not Ben de Guzman.
rebagged, classified, or weighed (which was the standard procedure for the NFA). 6. Cabal wrote Soriano advising him to withdraw the 630 cavans of palay from
When Soriano demanded for payment, he was informed that the payment will be the warehouse stating that NFA cannot legally accept the said delivery on the
held in abeyance since the NFA received information that Soriano is not a bona basis that Soriano is not a bona fide farmer.
fide farmer. NFA ordered Soriano to withdraw the 630 cavans of palay from the 7. Instead of withdrawing the cavans of palay, Soriano insisted that the palay
warehouse, because NFA cannot legally accept the said delivery on the ground grains he delivered be paid. He filed a complaint for specific performance
that Soriano is not a bona fide farmer. Soriano filed a complaint for specific with damages against the NFA.
performance with damages against the NFA. The trial court rendred judgment in 8. The trial court rendered judgment ordering NFA to pay Soriano P47,250
favor of Soriano. The CA affirmed the trial court’s decision. representing the unpaid price of the 630 cavans of palay plus legal interest.
9. NFA appealed to the IAC but the latter upheld the findings of the trial court.
The SC held that there was indeed a contract of sale. Art. 1458 defines sale as a Hence this petition.
contract whereby one of the contracting parties obligate himself to transfer the ISSUE/s:
ownership of and to deliver a determinate thing, and the other party to pay a price 1. WoN there was a contract of sale – YES
certain in money or its equivalent. In the case at bar, Soriano initially offered to
sell palay to NFA. When NFA accepted the offer noting the 2,460 cavans of palay, RULING: The instant petition for review is DISMISSED. The assailed decision of
there was already a meeting of minds between the parties. Sale is a consensual the IAC (now the CA) is affirmed.
contract, there is perfection when there is consent upon the subject matter and
price, even if neither is delivered. NFA contends further that one of the essential RATIO:
requisites of contract, which is consent. The acceptance referred to which 1. NFA contends that the 630 cavans of palay was made only for purposes of
determines consent is the acceptance of the offer of one party by the other and not having it offered for sale. Moreover, NFA stated that the procedure then
of the goods delivered as contended by NFA. Hence, there is already a perfected prevailing in matters of palay procurement from qualified farmers were:
a. There is a rebagging wherein palay is transferred from a private sack to the
contract of sale between NFA and Soriano. NFA sack;
b. After rebagging, classification of palay is made to determine variety;
DOCTRINE: c. The palay will be weighed to determine the number of kilos;
Sale is a consensual contract, there is perfection when there is consent upon the d. The palay will be piled inside the warehouse after the preparation of the
subject matter and price, even if neither is delivered. Warehouse Stock Receipt indicating the number of kilos, the variety, and
the number of bags.
FACTS: 2. Under this procedure, rebagging is the initial operative act signifying
1. Petitioner National Grains Authority (now National Food Authority or NFA) acceptance, and acceptance will be considered complete only after the
is a government agency created under PD No. 4. One of its incidental preparation of the Warehouse Stock Receipt.
functions is the buying of palay grains from qualified farmers. 3. When the 630 cavans of palay were brought by Soriano to the warehouse of
2. Private respondent Leon Soriano offered to sell palay grains to NFA through NFA, they were only offered for sale. Since the same were not rebagged,
William Cabal, the Provincial Manager of NFA in Cagayan. classified and weighed, there was no acceptance of the offer which, to NFA’s
3. Soriano submitted documents required by the NFA namely: 1) An mind is a clear case of policitation or an unaccepted offer to sell.
accomplished Farmer’s Information sheet certified by the Bureau of 4. Art. 1458 of the Civil Code defines sale as a contract where by one of the
Extension; 2) Photocopies of four tax declarations of the lease contract contracting parties obligates himself to transfer the ownership of and to
between him and Judge Concepcion Salud; and 3) his Residence Tax deliver a determinate thing, and the other party to pay a price certain in money
or its equivalent.
5. A contract, on the other hand, is a meeting of minds between two or more
persons whereby one binds himself, with respect to the other, to give
something or to render some service.
6. In the case at bar, Soriano initially offered to sell palay grains produced in his
farmland to NFA. When NFA accepted the offer noting in Soriano’s
Information Sheet a quota of 2,640 cavans, there was already a meeting of
minds between the parties.
7. The object of the contract, being the palay grains produced in Soriano’s
farmland and the NFA to pay the same depending on its quality. The fact that
the exact number of cavans of palay to be delivered has not been determined
does not affect the perfection of the contract.
8. Art.1349 states that the fact that the quantity is not determinate shall not be
an obstacle to the existence of the contract, provided it is possible to
determine the same, without the need of a new contract.
9. In this case, there was no need for NFA and Soriano to enter into a new
contract to determine the exact number of cavans of palay to be sold.
10. NFA further contend that there was no contract of sale because of the absence
of an essential requisite in contacts, which is consent. This is not correct. Sale
is a consensual contract, there is perfection when there is consent upon the
subject matter and price, even if neither is delivered.
11. The acceptance referred to which determines consent is the acceptance of the
offer of one party by the other and not of the goods delivered as contended
by NFA.
12. From the moment the contract of sale is perfected, it is incumbent upon the
parties to comply with their mutual obligations, or the parties may
reciprocally demand performance.
13. The reason why NFA initially refused acceptance of the 630 cavans of palay
is that it cannot legally accept the said delivery because Soriano is allegedly
not a bona fide farmer. The trial court and the appellate court found that
Soriano was a bona fide farmer and therefore, he was qualified to sell palay
grains to NFA.
14. NFA’s refusal is without just cause.
SCHUBACK & SONS PHIL. TRADING CORP. v. CA (Jolina) Reichert (general manager of petitioner). Using the PO from earlier, he wrote
Nov. 11, 1993 | Romero | Art. 1349; PO w/o quantity and unopened letter of credit “NOTE: Above P.O. will include a 3% discount. The above will serve as our
PETITIONER: Johannes Schuback & Sons Philippine Trading Corporation initial P.O.”
RESPONDENTS: Court of Appeals, Ramon San Jose, Jr. doing business under 31. Petitioner ordered the items from Schuback Hamburg that in turn ordered
the name and style “Philippine SH Industrial Trading from NDK, a supplier of MAN spare parts from Germany. Schuback
Hamburg sent petitioner a proforma invoice for applying for a letter of credit.
SUMMARY: Roman talked with petitioner to purchase german bus spare parts. The said invoice require the letter of credit to be opened in favor of Schuback
After a few exchanges in offers, respondent sends petitioner a purchase order with Hamburg. Respondent acknowledged receipt of the invoice.
the item number, part number and description (no quantity of the items). He 32. An order confirmation was sent by Schuback Hamburg to plaintiff which was
promises to send this in the future, which he later did to the general manager. A forwarded to and received by defendant. Petioner reminds the respondent
line of credit was given to respondent for the payment of the partial deliveries of several times to open the letter of credit to avoid the delay in shipment and
the parts. Roman contends that he didn’t make any valid PO and that there was no payment of interests. Respondent avers the difficulty he’s experiencing in
valid contract between them. The SC ruled that there is a perfected contract when secuing the required $ allocations and applying for the letter of credit,
the PO was sent. The absence of the quantity of the items is of no effect for it is procuring a loan and looking for a partner-financier, and of finding ways to
not an essential requirement of a contract of sale. proceed with the orders.
33. Meanwhile, Schuback Hamburg paid NDK for the partial deliveries of the
DOCTRINE: A contract of sale is perfected at the moment there is a meeting of items, later on deducted from petitioner. Petitioner reminds respondent once
minds upon the thing which is the object of the contract and upon the price. Art. again of the order. Respondent claims that he didn’t make any valid PO and
1319 states “Consent is manifested by the meeting of the offer and acceptance that there was no valid contract between them. Petitioner contends that there
upon the thing and the cause which are to constitute the contract. The offer must is a valid PO, giving him a choice to proceed with the order to pay for the
be certain and the acceptance absolute. A qualified acceptance constitutes a parts or to pay the cancellation fee of 30% of FOB value, or petitioner will
counter offer.” file a suit.

The opening of a letter of credit in favor of the vendor is only a mode of payment. ISSUE/s:
It it not among the essential requirements of a contract of sale enumerated in Art. 7. WoN a contract of sale has been perfected between the parties – Yes
1305 and 1474 and therefore does not prevent the perfection of the contract 8. WoN the absence of the quantity of items in the PO invalidates the contract?
between the parties. – No

RULING: SC reinstated the trial court’s decision and reversed the decision of the CA.
FACTS:
27. Through the Philippine Consulate General in Hamburg, West Germany, RATIO:
Roman purchased german MAN bus spare parts from petitioner. Petitioner 10. Where the seller quoted items offered for sale, by item number, part number,
communicated with its trading partner Johannes Schuback and Sohne description and unit price, and buyer had sent a reply in a purchase order
Handelsgesellschaft & Co. (herein as Schuback Hamburg) regarding the without indicating the quantity being ordered, there was already a perfected
spare parts Roman ordered. contract of sale, even when the required letter of credit had not been opened
28. Roman gave petitioner a list of parts with specific part numbers and by the buyer.
description. Upon the quotations of the trading partner, petitioner sent the 11. The opening of a letter of credit in favor of the vendor is only a mode of
letter while enclosing its offer on the items listed. Roman replies that he payment. It it not among the essential requirements of a contract of sale
prefers the genuine over the replacement parts and requested for a 15% enumerated in Art. 1305 and 1474 and therefore does not prevent the
discount on all the items. perfection of the contract between the parties.
29. Petitioner submitted its formal offer containing the item number, quantity, 12. A contract of sale is perfected at the moment there is a meeting of minds upon
part number, description, unit price and total to Roman. In turn, Roman the thing which is the object of the contract and upon the price. Art. 1319
informed plaintiff of his desire to avail of the prices of the parts at that time states “Consent is manifested by the meeting of the offer and acceptance upon
and enclosed its Purchase Order. The PO contained the item number, part the thing and the cause which are to constitute the contract. The offer must
number and description (no quantity of the items). Roman promises to be certain and the acceptance absolute. A qualified acceptance constitutes a
submit the quantity he wants in a future date. counter offer.”
30. Roman personally submits the quantities he wants for the order to Dieter
13. Upon the sending of respondent to petitioner the PO, a meeting of the minds
between the vendor and vendee had occurred, the object of the contract being
the spare parts and the consideration, the price stated in petitioner’s offer from
the earlier letter and accepted by respondent.
14. Although the said PO didn’t contain the quantity respondent wanted to order,
he promised to communicate the same, which he later followed through. In
this case, respondent was already in the process of executing the agreement
previously reached between the parties.
MAPALO v. MAPALO (Megan) 45. The Mapalo spouses filed their answer seeking the cancellation of the TCT
May 19, 1966 | Bengzon, J. | Absence of Consideration on the grounds that their signature was procured by fraud and that the
Narcisos were buyers in bad faith. They further asked that the deed of sale be
PETITIONER: Miguel Mapalo, et al. declared null and void as to the western half of said land.
RESPONDENTS: Maximo Mapalo, et al. 46. The CFI ruled in favor of the Mapalo spouses.
47. The CA however reversed the decision of the CFI and declared that since the
SUMMARY: Spouses Mapalo donated the eastern half of their land in favor of deed of sale were obtained by fraud, the same was voidable, and not void ab
Miguel’s (husband) brother, Maximo, since he was about to get married. They initio.
were deceived into signing a deed of absolute sale covering the entire land. The 48. The Mapalo spouses appealed to the SC.
documaent stated a consideration of P500 but they did not receive anything.
Maximo was able to name the entire land as his. Years later he sold the entire land ISSUE/s:
to the Narcisos. The Narcisos at first occupied the eastern part of the land, but 9. WoN cause/consideration is present -NO
subsequently filed a case against the spouses in order to be declared owners of the 10. Whether the deed of sale void ab initio or voidable – void ab initio
entire land. The Mapalo spouses countered by seeking to cancel the TCT and
sought that the deed of sale be declared null and void. The CFI ruled in favor of RULING: Wherefore, the decision of the Court of Appeals is hereby reversed and set
the Mapalo Spouses. The CA however reversed the ruling of the CFI and declared aside
that the deed was merely voidable since consent was present however vitiated.
The SC however reversed the ruling of the CA and declared that although consent RATIO:
and subject matter was present, consideration was absent therefore making the 15. For a contract to exist at all, three essential requisites must concur: (1)
contract null and void. consent, (2) object, and (3) cause or consideration.
16. The Court of Appeals is right in that the element of consent is present as to
DOCTRINE: A contract of purchase and sale is null and void and produces no the deed of sale. For consent was admittedly given, albeit obtained by fraud.
effect whatsoever where the same is without cause or consideration in that the Accordingly, said consent, although defective, did exist. In such case, the
purchase price which appears thereon as paid has in fact never been paid by the defect in the consent would provide a ground for annulment of a voidable
purchaser to the vendor. contract, not a reason for nullity ab initio.
17. The second element of object is likewise present in the deed, namely, the
parcel of land subject matter of the same.
FACTS: 18. However, the third element of cause or consideration is not present.
34. Spouses Miguel Mapalo and Candida Quiba are simple illiterate farmers who 19. The rule under the Civil Code, again be it the old or the new, is that contracts
owned a parcel of land registered under the Torrens title. without a cause or consideration produce no effect whatsoever.
35. Said spouses donated the eastern half of the land to Miguel’s brother, 20. Nonetheless, under the Old Civil Code, the statement of a false consideration
Maximo Mapal, out of love and affection as he was about to get married. renders the contract voidable, unless it is proven that it is supported by
36. However, the spouses were deceived in signing a deed of absolute sale over another real and licit consideration.
the entire land. 21. Since the deed of sale of 1936 is governed by the Old Civil Code, it should
37. Their signatures were procured by fraud since they were made to believe that be asked whether its case is one wherein there is no consideration, or one
they were signing a deed of donation in Maximo’s favor covering the eastern with a statement of a false consideration.
part of their land. 22. If no consideration, it is void and inexistent; if false consideration, only
38. The document of the sale stated a consideration of 500 pesos but they did not voidable.
receive anything of value. 23. What is meant by a contract that states a false consideration is one that has in
39. The spouses built a fence to segregate the eastern and western portion. fact a real consideration but the same is not the one stated in the document.
40. Not known to them, Maximo registered the deed of sale in his favor and 24. As observed earlier, the deed of sale stated that it had for its consideration
obtained in his name the Transfer Certificate of Title over the land. P500.00. In fact, however, said consideration was totally absent.
41. 13 years later, he sold the entire land to the Narcisos. 25. A contract of purchase and sale is null and void and produces no effect
42. The whole land was issued in their names. whatsoever where the same is without cause or consideration in that the
43. The Narcisos took possession only of the eastern portion of the land. purchase price which appears thereon as paid has in fact never been paid
44. After a year, they filed a suit in the CFI to be declared owners of the entire by the purchaser to the vendor.
land.
26. The inexistence of a contract is permanent and incurable and cannot be the
subject of prescription.
27. On the bad faith of The Narcisos: RTC’s finding is conclusive that they had
prior knowledge that the spouses had ownership over the western half when
they recognized such ownership when they occupied only the eastern half.
RONGAVILLA V. CA (MAYUMI) Jimenez (sister of Dolores). The OCT and TCT of this land were kept by
August 17, 1998| Quisumbing, J.| Lack of consent and consideration as void ab initio Juanita.
3. In May 1976, the aunts borrowed P2,000 from Spouses Rongavilla in order
PETITIONER: Spouses Narciso Rongavilla and Dolores Rongavilla to have their dilapidated rooftop repaired.
RESPONDENTS: Court of Appeals and Mercedes Dela Cruz and Florencia Dela 4. A month later, Dolores and Juanita visited their aunts’ home and brough a
Cruz document for their aunts to sign. This document was typewritten in English.
Mercedes asked in Tagalog what the document was about and Dolores told
SUMMARY: Mercedes and Florencia (the aunts) borrowed from Spouses her that it was a document to show that the aunts had a debt of P2,000. On
Rongavilla P2,000 to repair their leaky roof. A month later Dolores Rongavilla account of this representation, the aunts signed the document.
and Juanita (the nieces) visited the aunts and asked them to sign a document that 5. In September 1980 (4 years later), Dolores went to the aunts’ place and asked
was written in English. Because the aunts didn’t know English and only knew them to vacate the land stating that she and her husband were the new owners
how to read and write in Tagalog, they asked Dolores what the document was to of the land.
which Dolores responded that it was a document acknowledging the debt of 6. Surprised, the aunts went to the Office of the Register of Deeds of the
P2,000 of the aunts. The aunts signed the document because of this representation. Province of Rizal to verify the matter. There they discovered that their
4 years later Spouses Rongavilla went to the aunts and told them to vacate their Certificate of Title had been cancelled and a new TCT was issued in favor of
land saying that it now belongs to them because what the aunts signed was a Deed Spouses Rongavilla. Moreover, that the parcel of land had been mortgaged
of Sale. When the aunts went to the Register of Deeds, it was confirmed that a with the Cavite Development Bank by the spouses.
Transfer Certificate of Title was issued in favor of Spouses Rongavilla and that 7. It was only then that the aunts realized that the document they had previously
the said land was mortgaged for P40,000. Thus, the aunts filed with the CFI of been asked by their nieces to sign was a deed of sale.
Rizal a complaint asking that the Deed of Sale be declared void and inexistent. 8. The aunts then filed with the CFI (now RTC) a complaint to have the deed of
According to the Spouses, the aunts voluntarily signed the Deed of Sale, but sale declared void and inexistent for being fictitious and simulated, and
according to the aunts they signed the document thinking it was the secured by means of fraud and misrepresentation.
acknowledgement of debt. They further claim that no consideration was given to 9. According to the aunts:
them for the alleged sale. The lower courts ruled in favor of the aunts. Thus, the a. They did not sell their property (no consent);
Spouses brought the case to the SC. b. They received no consideration for the supposed sale;
c. Spouses Rongavilla mortgaged the said title for P40,000 to the
The Court ruled that the deed of sale was not signed by vitiated consent, but rather damage and prejudice of the aunts; and
there was no consent at all. Moreover, that there was no consideration for the deed d. They claim moral and exemplary damages.
of sale. When there is lack of consent and consideration in a deed of sale, it is 10. Spouses Rongavilla on the other hand allege that:
declared as void ab initio. Because it is a void and inexistent contract, such does a. The aunts sold their parcel of land voluntarily;
not have a prescription period. b. There was consent in the deed of sale;
c. There was consideration for the sale;
DOCTRINE: Lack of consent and consideration made the deeds of sale void d. The document of sale was complete in itself and in due form;
altogether and rendered them subject to attack at any time, conformably to the rule e. That the aunts were fully appraised by the Notary Public, Atty.
in Article 1410 that an action to declare the inexistence of void contracts does not Arcadio G. Espiritu, on what the document was all about, and having
prescribe. understood the explanation made by said Notary Public, they
voluntarily affixed their signatures on said document; and
f. That prescription had already set in
FACTS:
11. The lower court ruled in favor of the aunts which the CA affirmed.
1. Mercedes (60 yrs. old) and Florencia Dela Cruz (71 yrs. old) are the aunts of
12. Spouses Rongavilla claim that "the presumption that contracts are presumed
Dolores Rongavilla. The aunts are both spinsters and earn money from being
to be valid and to be supported by lawful and good consideration of one dollar
embroiderers (“magbuburda”) and dressmakers. The aunts were unschooled
is just as effectual and valuable as a larger sum stipulated or paid''. Moreover
in English, but are able to read and write in Tagalog. Moreover, they are of
that the deed of sale is presumed valid and, being registered, could not be
advanced age, both aunts spend their day confined close to home.
disturbed anymore
2. The aunts own a parcel of land in Las Pias, Rizal (now Metro Manila). They
13. Thus, this review on appeal by certiorari under Rule 45.
own in in proportion of 1/2 pro-indiviso with another niece named Juanita
ISSUES: 7. Lack of consent and consideration made the deeds of sale void altogether and
1. WoN the Deed of Sale is void and inexistent –YES rendered them subject to attack at any time, conformably to the rule in Article
2. WoN the action to annul the Deed of Sale prescribed –NO 1410 that an action to declare the inexistence of void contracts 'does not
prescribe'.
RULING: WHEREFORE, the instant petition is hereby DENIED. The Decision and 8. The defect of inexistence of a contract is permanent and incurable, hence
the Resolution of the Court of Appeals in CA-G.R. CV No. 06543 are hereby it cannot be cured either by ratification or by prescription.
AFFIRMED.

RATIO:
Deed of Sale as Void and Inexistent
1. The aunts allege that the deed of sale had neither their consent nor a
consideration. They also denied going to and appearing before the Notary
Public. Moreover, they allege that the purported deed of sale was obtained by
fraud and misrepresentation because Dolores had misled them to believe the
document was just a paper to evidence a debt of P2,000 they obtained to buy
G.I sheets for the repair of their leaking roof.
2. While it is true that public documents are presumed genuine and regular
under the provisions of the Rules of Court, this presumption is a rebuttable
presumption which may be overcome by clear, strong and convincing
evidence.
3. The Court has already concluded in previous cases that when a contract of
sale lacks cause or consideration, it is therefore null and void and without any
effect whatsoever (Ocejo, Perez & Co. vs. Flores)

Prescription
1. Once the disputed deed is found to be inexistent and void, the statute of
limitations cannot apply. The cause of action for its declaration as such is
imprescriptible.
2. According to the spouses, this ruling is contrary to settled jurisprudence in
Pangadil v. CFI of Cotabato. However, the court mentioned that this case is
different from the case at bar. In Pangadil, there was an inaction for a
considerable period of time. In the case at bar, there is no doubt about the
credibility of the aunts in pursuing their cause promptly and forcefully.
3. They never intended to sell, nor acceded to be bound by the sale of their land.
4. Public policy is also well served in defending the rights of the aged to legal
protection, including their right to property that is their home, as against
fraud, misrepresentation, chicanery and abuse of trust and confidence by
those who owed them candor and respect.
5. While the Civil Code states that an action to annul a contract on the ground
of vitiated consent must be filed within four years from the discovery of the
vice of consent, the case at bar does is not a voidable contract but actually a
contract that is void ab initio.
6. When a deed of sale is signed with the owner not knowing what the deed
is, consent is not merely marred by vices (which would make it voidable),
but actually not given at all.
MATE v CA (Armand) respondent for violation of BP22 on account of the rubber checks, that she,
May 21, 1998 | Martinez, J. | Consideration her mother, sister and brother issued to Mate amounting to P4,435,067.
2. She requested Mate to cede to private respondent his 3 lots in Tacloban City
PETITIONER: Fernando T. Mate in order to calm him down.
RESPONDENTS: Court of Appeals and Inocencio Tan 3. Mate immediately rejected it as he owed private respondent nothing and he
was under no obligation to convey his properties to him. Josie explained to
SUMMARY: On October 6, 1986, Josefina “Josie” Rey and private respondent him that he was in no danger of losing his properties as he will merely execute
Tan went to the residence of petitioner in Tacloban City. Josie solicited his help a simulated document transferring them to private respondent but they will
to stave her off her prosecution by respondent for violation of B. P. 22. Josie be redeemed by her with her own funds. Mate eventually agreed to execute
asked petitioner to cede to respondent his 3 lots. Josie explained to him that he the fictitious deed of sale with right to repurchase covering his 3 lots
was in no danger of losing his property as they will be redeemed by her own mentioned subject to:
funds. After a long discussion, petitioner agreed to execute a fictitious deed of a) amount stated in the document is P1.4M with interest at 5% per month
sale with right to repurchase after 6 months. Josie gave petitioner 2 post-dated b) properties will be repurchased within 6 months or on or before Apr 4,1987
checks worth P1.4M and P420k, to be used in redeeming the property. However, c) Although it would appear that Mate is the vendor, Josie will provide money
the checks were both dishonored. Realizing that he was swindled, he sent a for the redemption of the properties with her own funds.
telegram to Josie, and looked for her in Manila, but she was nowhere to be found. d) Titles to the properties will be delivered to Mate but the sale will not be
Petitioner filed a criminal case against Josie for violation of B. P. 22, but the case registered in the Registry of Deeds and annotated on the titles.
was archived since Josie could not be located. Petitioner filed a civil case for 4. To assure Mate that Josie will redeem the properties, Josie issued to him 2
annulment of contract with damages against Josie and respondent. Josie was BPI checks both postdated Dec 15, 1986. One was P1.4M for the selling price
declared in default and the case proceeded against private respondent. Both the and P420,000 for the interests.
trial court and the Court of Appeals upheld the validity of the sale. The issue is 5. Mate prepared the deed of sale with right to repurchase and after it was signed
WoN the deed of sale is valid or null and void (as there is allegedly no and notarized, it was given to private respondent together with the titles and
consideration) – the sale if valid and with consideration. The court held that it is the latter did not register them in the Register of Deeds as agreed upon.
plain that consideration existed at the time of the execution of the deed of sale 6. Jan 14, 1987 – Mate deposited the check for P1.4M at UCPB and the other
with right of repurchase. It is not only appellant's kindness to Josefina, being his check for P420,000 at Metrobank. Both checks were dishonored by the
cousin, but also his receipt of P420,000.00 that impelled him to agree to the drawee bank for having been drawn against a closed account.
contract. Also, despite him not receiving the P1.4M, he had in his possession 7. He sent Josie a telegram about her checks and when she failed to respond, he
postdated checks in an equivalent amount to repurchase the lots on or before the went to Manila to look for her but she could not be found.
sixth month. There is absolutely no basis for petitioner to file a complaint against 8. He filed criminal cases against Josie for violation of BP 22 but the cases were
private respondent Tan and Josie Rey to annul the pacto de retro sale on the archived as the accused could not be found. He then filed a civil case against
ground of lack of consideration, invoking his failure to encash the two her at the RTC of Leyte for annulment of contract with damages. Josie was
checks. Petitioner's cause of action was to file criminal actions against Josie Rey declared in default and the case proceeded against private respondent. But
under B.P. 22, which he did. The filing of the criminal cases was a tacit admission during the trial the RTC asked private respondent to file an action for
by petitioner that there was a consideration of the pacto de retro sale. consolidation of ownership of the properties subject of the sale.
9. He filed another case that was consolidated with the case he filed earlier
DOCTRINE: The filing of the criminal cases was a tacit admission by petitioner which were decided jointly by the RTC in favor of the private respondent and
that there was a consideration of the pacto de retro sale. Despite not receiving the subsequently appealed to the CA which affirmed it with modification. Mate
money as consideration, the petitioner had postdated checks in an equivalent filed an MR but was denied, hence the instant petition for review.
amount, hence there is consideration. Between two innocent parties, the one who
made it possible for the wrong to be done should be the one to bear the resulting ISSUES:
loss. 1. WoN the deed of sale with right to repurchase is valid or null and void for
lack of consideration (because allegedly, no consideration since no money
FACTS: changed hands when he signed it and the checks that were issued were
1. October 6, 1986 – Josefina Rey (Josie) and private respondent went to the dishonored) – The deed of sale is VALID and with consideration.
residence of Mate at Tacloban City. Josie who is a cousin of Mate’s wife
solicited his help to stave off her and her family’s prosecution by private RULING: WHEREFORE, the Decision of the Court of Appeals dated August 29,
1994 is hereby AFFIRMED. The petition for review is hereby DENIED DUE “where one or two innocent persons must suffer, that person who gave
COURSE for lack of merit. occasion for the damages to be caused must bear consequences."
8. Mate's reliance on this doctrine is misplaced. He is not an innocent person.
RATIO: As a matter of fact, he gave occasion for the damage caused by virtue of the
1. The CA held: “In preparing and executing the deed of sale with right of deed of sale with right to repurchase which he prepared and signed. Thus,
repurchase and in delivering to Tan the land titles, appellant actually there is the equitable maxim that between two innocent parties, the one
accommodated Josefina so she would not be charged criminally by Tan. To who made it possible for the wrong to be done should be the one to bear
ensure that he could repurchase his lots, appellant got a check of the resulting loss.
P1,400,000.00 from her. Also, by allowing his titles to be in possession of 9. Mate even insinuates that private respondent deceived him into signing the
Tan for a period of six months, appellant secured from her another check for deed of sale with right to repurchase. There is no evidence of this. Contrary
P420,000.00. With this arrangement, appellant was convinced he had a good to Mate’s pretension, respondent Tan did not employ any devious scheme to
bargain. Unfortunately his expectation crumbled. For this tragic incident, not make the former sign the deed of sale. It is to be noted that Tan waived his
only Josefina, but also Tan, according to appellant(Mate) must be right to collect from Josie by virtue of the pacto de retro sale.Mate, a lawyer,
answerable.” should have known that the transaction was fraught with risks since Josie
2. Mate did not receive the P1.4 Million purchase prices from respondent Tan, already had a history of issuing worthless checks.
instead, he had a postdated check of Josie in an equivalent amount precisely 10. It was Mate’s own greed for huge profit that impelled him to accede to the
to repurchase the two lots on or before the sixth month. scheme of Josie despite knowing of its risks and dangers. He was in fact
3. As admitted by Mate, Josie thus assumed the responsibility of paying the negligent.
repurchase price on behalf of petitioner to private respondent. Unfortunately, 11. A contract is a contract. Once agreed upon, and provided all the essential
the two checks issued by Josie Rey were worthless and dishonored. However, elements are present, it is valid and binding as between the parties. Mate has
there is absolutely no basis for petitioner to file a complaint against private no one to lame but himself for his misfortune.
respondent Tan and Josie to annul the pacto de retro sale on the ground of
lack of consideration, invoking his failure to encash the two checks.
Petitioner's cause of action was to file criminal actions against Josie Rey
under B.P. 22, which he did. The filing of the criminal cases was a tacit
admission by petitioner that there was a consideration of the pacto de
retro sale.
4. Mate further claims that the pacto de retro sale was subject to the condition
that in the event the checks given by Josie to him for the repurchase of the
property were dishonored, then the document shall be declared null and void
for lack of consideration. However, the Court was not persuaded.
5. Tan was already poised to file criminal cases against Josie and her family. It
would not be logical for respondent Tan to agree to the conditions allegedly
imposed by petitioner. Mate knew that he was bound by the deed of sale with
right to repurchase, as evidenced by his filing criminal cases against Josie
Rey when the two checks bounced.
6. The SC agreed with the CA in ruling that: “If there is anybody to blame for
his predicament, it is appellant himself. He is a lawyer. He was the one who
prepared the contract. He knew what he was entering into. Surely, he must
have been aware of the risk involved. When Josefina's checks bounced, he
should have repurchased his lots with his own money. Instead, he sued not
only Josefina but also Tan for annulment of contract on the ground of lack of
consideration and false pretenses on their part.”
7. Mate then claims that "it is not only illegal but immoral to require him to
repurchase his own properties with his own money when he did not derive
any benefit from the transaction." even citing a case, when the SC held:
GUAN v. ONG (IYA) simulated.
October 18, 2001| Panganiban, J. | Price must be real: When simulated 6. Ong then executed a Deed of Sale with the consideration of P200,000 (which
was not payed). A new title was issued under the name of Guan.
7. Guan made insistent demands (using physical violence and threats) for the
PETITIONER: Yu Bun Guan
delivery of the owner’s copy of the title. Guan also refused to perform his
RESPONDENTS: Elvira Ong
promise. This led to Ong executing an Affidavit of Adverse Claim.
8. Guan filed before the RTC a Petition for Replacement of an owner’s duplicate
SUMMARY: Guan and Ong are husband and wife. During their marriage they
title. Attached to the petition is an Affidavit of Loss, where he made it appear
bought a house and lot using their conjugal funds. Ong also bought the Rizal
that the owner’s copy of the title was lost or misplaced.
property using her person funds, therefore making it paraphernal property. In the
9. The RTC granted such and issued a new owner’s copy of title to Guan.
year 1992, they separated due to Guan’s promiscuity, temper, and other vices.
10. Ong, upon discovery of the fraudulent acts of Guan, filed a case before the
courts to declare the sale of the Rizal property as null and void.
Guan insisted on Ong to transfer the Rizal property to him through a fictitious
11. As defense Guan posits that before he became a Filipino citizen, the Rizal
Deed of Sale wherein he (Guan) did not pay any consideration. To this Ong
property was already beings offered to him, but as he is an alien he cannot
agreed on the condition that he build a commercial building for the benefit of their
acquire for himself.
children.
a. He therefore used Ong as his ‘dummy’ and claims he used his
personal funds to buy said property.
However, Guan did not comply with his promises and performed fraudulent acts
b. He also claims that Ong is in pari delicate and cannot claim relief
to acquire the owner’s title of the Rizal property for himself. Because of this Ong
from courts.
filed a case for the cancellation of the fictitious sale. The issue before the courts
12. The RTC found that the Rizal property was the paraphernalia property of
is whether or not the Deed of Sale is void.
Ong.
a. That the in pari delicate rule does not apply because it would only
The RTC and CA ruled in Ong’s favor declaring the sale void. Upon reaching the
apply to existing contracts with illegal cause or object, not to
Supreme Court, the lower court decisions were affirmed. It was held that contracts
simulated or fictitious contracts.
of sale where the consideration is absolutely simulated is null and void.
b. The Deed of Sale is likewise voided for having been simulated and
executing during the marriage of the parties.
DOCTRINE: A contract of purchase and sale is null and and void and
13. On appeal to the CA, the court upheld the RTC ruling:
produces no effect whatsoever where the same is without cause or
14. Hence, this petition.
consideration in that the purchase price which appears thereon as paid has in fact
never been paid by the purchaser to vendor.
ISSUE/s:
1. WON the Deed of Sale is void? - YES, consideration is absolutely simulated

FACTS: RULING: WHEREFORE, the Petition is hereby DENIED and the assailed. Decision
1. Elvira Ong and Yu Bun Guan are husband and wife, married under Chinese AFFIRMED.
rites on April 1961. They lived together and had 3 children until Guan
abandoned them due to his promiscuity, volcanic temper, and other vices. RATIO:
2. Out of Ong’s personal funds, she bought a parcel of land referred to as the 1. In Roganvilla v. Court of appeals the Court declared that a deed of sale, in
Rizal Property from a certain Aurora Seneris. This was registered under her which the stated consideration had not in fact been paid, is null and void:
name. a. “..a contract of purchase and sale is null and and void and
3. During their marriage, they purchased, out of their conjugal funds, a house produces no effect whatsoever where the same is without cause
and lot registered under their names. or consideration in that the purchase price which appears thereon
4. During their separation in 1992, Ong agreed to Guan’s insistence that she as paid has in fact never been paid by the purchaser to vendor.”
execute a Deed of Sale of the Rizal property in his favor. This is on the 2. In the present case, it is clear from the factual findings of both lower courts
promise that he would construct a commercial building for the benefit of their that the Deed of Sale was completely simulated and, hence, void and
children. without effect. No portion of the P200,000 consideration stated in the Deed
5. Guan suggested that the property be named to him alone so that she would was ever paid.
not incur any obligation. The consideration or price for the sale was
3. The principle of in pari delicto provides that when two parties are equally
at fault, the law leaves them as they are and denies recovery by either
one of them. However, this principle does not apply with respect to inexistent
and void contracts.
a. The principle of in pari delicto non oritur actio denies all recovery
to the guilty parties. It applies to cases where the nullity arises from
the illegality of the consideration or the purpose of the contract.
When two persons are equally at fault, the law does not relieve them.
The exception to this general rule is when the principle is
invoked with respect to inexistent contracts.
Ong v Ong (Lij) no moment since it is usual practice to place a nominal amount although there
October 8, 1985 | Relova, J. | Consideration is valuable consideration given.
54. Not satisfied, petitioners came to the court to question the interpretation of
PETITIONER: Imelda Ong, et al the said document.
RESPONDENTS: Alfredo Ong, et al 55. April 15, 1985, Sandra replaced her guardian ad litem since she reached age
of majority.
SUMMARY: Imelda Ong executed a Quitclaim deed in favor of Sandra Maruzzo ISSUE/s:
(minor). The quitclaim conveyed the ½ undivided portion of a piece of land in 11. W/n there was consideration to constitute the deed as a sale? Yes
Makati. The deed itself stated that the conveyance was for 1 peso and other
valuable considerations. 4 years later, Imelda revoked the Deed of Quitclaim and RULING: WHEREFORE, the appealed decision of the Intermediate Apellate Court
donated her entire property to her son Rex. should be, as it is hereby AFFIRMED, with costs against herein petitioners.
Sandra, through her guardian filed a case to recover ownership. The trial court
ruled in favor of Sandra and held that it was a deed of sale. The IAC affirmed RATIO:
stating that it was a deed of sale with a valid cause or consideration. The SC ruled 28. A careful perusal of the subject deed reveals that the conveyance of the ½
that the deed itself states “for other valuable consideration”. The consideration undivided portion was in consideration of 1 peso and other valuable
was not only 1 peso. The document in itself is prima facie evidence of sufficient considerations paid by respondent Sandra through her representative. Alfred
cause or consideration andit is the burden of the person alleging the lack thereof Ong to Imelda Ong. The consideration was not only 1 peso.
to the court. Even if a consideration of 1 peso is suspicious, that circumstance 29. The execution of a deed purporting to convey ownership of a realty is in itself
alone will not justify the inference that the parties were not purchasers in good prima facie evidence of the existence of a valuable consideration. The party
faith and for value. alleging lack of consideration has the burden of proving such allegations.
Also if it was a donation, acceptance would not be needed since it was a pure There is a legal presumption of sufficient cause or consideration supporting
donation with no condition imposed hence the donor would have no right to a contract even if such cause is not stated therein(art.1345).
protect his interest nor would the donee have any obligation to the donor. 30. Even if the quitclaim was a deed, the acceptance of the donation in favor of
a minor by parents or guardian applies to onerous and conditional donations
DOCTRINE: Execution of a deed purpoting to convey ownership of realty is in where there are certain burdens. Acceptance by a guardian of a simple or pure
itself prima facie evidence of the existence of valuable consideration. donation does not seem to be necessary. Kapunan v Casilan: the donation to
an incapacitated donnee does not need acceptance when it does notcontain a
condition.
FACTS: 31. The pronouncement of the IAC finds support in Court of Morales
49. February 25, 1976, Imelda Ong, executed in favor of private respondent Development Co., Inc. vs. CA which states the the 1 peso consideration is not
Sandra Maruzzo (minor) a Quitclaim deed for the consideration of P1 and unusual. Deeds of conveyance adheringto the Anglo-Saxon practice of stating
other valuable considerations. The quitclaim transferred, realesed, assigned the consideration is 1 peso when in fact the consideration is much more. Even
and forever quit-claimed to Sandra Maruzzo all right to one half undivided if the 1 peso consideration is suspicious, it does not necessarily justify the
portion of the parcel of land. The property was in Makati and had an are of inference that Reyes and the Abellas were not purchasers in good faith and
125 square meters. for value nor does it render it void ab initio. Bad faith and inadequacy of
50. November 19, 1980, Imelda revoked the Deed of Quitclaim and on Januart monetary consideration do not render a conveyance inexistent.
20, 1982 dontaed the entire property to her son Rex Ong-Jimenez.
51. 1983, Sandra, through her guardian (ad litem) Alfredo Ong, filed for recovery
of ownership and null of deed of donation over the portion belonging to her.
52. The petitioners plead that the Quitclaim deed is null and void inasmuch as it
is equivalent to a Deed of Donation, acceptance of which by the done is
necessary to give it validity. They also argued the minority of Sandra.
53. The trial court rendered judgement in favor of Sandra and held that the
Quitclaim is a deed of Sale, hence was a valid conveyance. The IAC affirmed
the appealed judgement and the held that the Quitclaim Deed is a conveyance
of property with a valid cause or consideration. That the consideration of One
peso which is clearly stated in the deed itself, the apparent inadequacy is of
BAGNAS v. COURT OF APPEALS (PERRAL) FACTS:
Aug. 10, 1989| NARVASA J.:| Price must be in “Money or its Equivalent” 1. Hilario Mateum of Kawit, Cavite, died on March 11,1964, SINGLE L,
without ascendants or descendants, and survived only by collateral relatives,
PETTIONER: ISAAC, ENCARNATION, SILVESTRE, MAXIMINA, SIXTO, all of whom petitioners herein, his first cousins, were the nearest.
surnamed BAGNAS, and AGATONA ENCARNACION 2. Matueum left no will, no debt, and an estate consisting of 29 parcels of land
RESPONDENT: COURT OF APPEAL, ROSA RETONI, TEOFILO in Kawit and Imus cavite, 10 of which are involved in this case.
ENCARNACION, JOSE NAMBAYAN 3. April 1964: Mateum’s distant or remote collateral relatives, herein private
respondents, registered 2 deeds of sale purportedly executed by Mateum in
SUMMARRY: Hilario Mateum, the owner of the disputed parcels of land in this case, their favor covering the disputed 10 parcels of land.
died leaving no ascendant and descendants, and was survived only by collateral 4. The 2 deeds covered 5 parcels of land each and both antedating Mateum’s
relatives. Mateum left no will and an estate consisting of 29 parcels of land wherein 10 death by more than a year. (Feb. 6, 1963; March 4,1963)
of which are the subject of this proceeding. The conflict in this case started when the 5. Both deeds recited the consideration of the sale to be: “the Sum of ONE
distant and remote relatives of Mateum, private respondents, registered 2 deeds of sale, PESO, Phil. Currency, and Services rendered, being rendered and to be
each covering 5 parcels of land, purportedly executed by Mateum himself in favor of rendered for my benefit.” (originally stated in Tagalog)
private respondents, wherein private respondents were able to secure a title over 3 out 6. What initially prompted the filing of this case, is that, on the strength of the
of 10 parcels of land. Also, written on the face of the deeds is a stipulation of the price aforementioned deeds of sale, the private respondents were able to secure title
of said properties amounting only to P1.00 and services rendered or being rendered in in their favor over 3 out of the 10 parcels of land.
his favor as to constitute as consideration of the sale. Upon the knowledge of the nearest 7. May 1964: The petitioner filed a suit against private respondents in the CFI
collateral relatives of Mateum, his first cousins, about the said registration, they (RTC), questioning the validity of said deeds of sale, advancing the following
immediately file an action against private respondents seeking to recover the ownership arguments:
and possession of the said lands; to Annul the deeds of sale, and declare such sale as a. PETIONERS’ ARGUMENT:
VOID for lack of Consideration thus, fictitious or simulated. Private respondents file a i. The deeds of sale should be annulled for being fictitious,
motion to dismiss on the ground that petitioners have no capacity to impugn the fraudulent or falsified for lack of good and valuable
disposition of deceased with his property since they are mere collateral relative who consideration;
are not bound subsidiarily or principally and for lack of evidence which would ii. Assuming that both sales, in reality, were donations, the
substantiate the claim of fraud. Trial Court dismissed the case agreeing with private same are void for want of acceptance embodied in a public
respondents that petitioners were not able to present proof of Fraud and that as instrument;
collateral rel. they do not have the right to question deceased disposition of his land. iii. Claiming ownership pro indiviso of the lands subject of the
Petitioner appealed with the CA but unfortunately the latter affirmed the ruling of trial deeds by virtue of being intestate heirs of Mateum.,
court that they have no right to impugn, no sufficient evidence of fraud and that there petitioners prayed for recovery of ownership and
is a presumption of the existence of consideration in duly registered deeds. Hence, possession of said lands, accounting of the fruits thereof
petitioner appealed to the SC. SC reversed the ruling of both trial court and CA and and damages.
declared that there was no valid contract of sale at all since the element of 8. Originally, the complaint seeks the recovery of all 29 parcels of land,
consideration, price certain in money or its equivalent, is absent. Court also Annulled however at the pre- trial the parties agreed that it be limited to the 10 parcels
the deeds of sale covering the 10 parcels of land, and awarded all the properties to subject of the questioned sales. Of the 10 parcels under litigation, 9 were
Petitioners. assessed having an aggregate value of P10,500.
9. The respondent filed their answer to the complaint positing the following
DOCTRINE: At the onset, if a contract has no consideration, it is not merely contentions:
voidable, but VOID—and even collateral heirs may assail the contract. In this a. PRIVATE RESPONDENTS’ POSITION:
case, there was no consideration. Price must be in money or its equivalent; services i. They denied the alleged fictitious or fraudulent character
are not the equivalent of money insofar as the requirement of price is concerned. of the sales in their favor;
A contract is not one for sale if the consideration consists of services. Not only are ii. Asserting that said sales were made for good and valuable
they vague, they are unknown and not susceptible of determination without a new consideration;
agreement between the parties. iii. While they may have the effect of a donations, yet the
formalities and solemnities of donation are not required for
their validity and effectivity
iv. That they are collateral relatives of Mateum and had done
many good things for him, nursing him in his last illness, RULING: Appealed decision of the CA is reversed. The Questioned transfers are
which services constituted the bulk of the consideration of declared VOID and of no force or effect. Certificates of title as the private respondents
the sales; may have obtained over the properties subject of said transfers are hereby
v. (by way of affirmative defense) that the plaintiffs could not ANNULLED. Respondent is ordered to return all the properties and to account for
question or seek annulment of the sales because they are their fruits during the time of their possession.
mere collateral relatives of the deceased vendor and were
not bound, principally or subsidiarity, thereby. RATIO:
10. After petitioners presented their evidence, private respondents filed a motion 1. FIRST ISSUE
to dismiss reasserting their affirmative defense and that no evidence of fraud a. CITING AUTHORITIES:
tainting said transfers had been presented. i. Citing Montinola: a price of P1 for the sale of things worth
11. The Trial Court granted the motion to dismiss, holding: 20K is so insignificant as to amount to no price at all,
a. On the Authority laid down in the case of Armentia, that the and does not satisfy the law which, while not requiring for
petitioners, as mere collateral relatives, not forced heirs, of Mateum, the validity of a sale that the price be adequate, prescribes
could not legally question the disposition made by said deceased that it must be real and not fictitious.
during his lifetime, regardless of whether said dispositions were ii. Citing Manresa: that true price, while is essential to the
valid or not. validity of a sale, means existent, real and effective price;
b. That plaintiffs’ evidence of alleged fraud was insufficient, the fact that is not the same as the concept of just price which
that the deeds of sale each stated a consideration of P1 not being in entails weighing and measuring, for economic
itself evidence of fraud or simulation. equivalence; but there is no need of such a close
12. From this ruling, the Petitioners appealed to the CA: examination when the immense disproportion between
a. CA affirmed, adverting with approval the Trial Court’s reliance of such economic values is patent (a case of insignificant or
the Armentia ruling which, both courts saw as denying, without ridiculous price), the unbelievable amount of which at
exception, to collaterals, of a decedent, not forced heirs, the right to once points out its inexistence.
impugn the latter’s disposition inter vivos of his property. iii. Art 1458 of CC, prescribing that a sale be for a “price
b. Declared that the testimonies of petitioners’ witnesses failed to certain in money or its equivalent”, requires that the
establish fraud of any kind or that Mateum had continued paying equivalent is something representative of money. To the
taxes on the lands in question even after executing the deeds. effect that SERVICES ARE NOT EQUIVALENT OF
c. Ruled that since in duly notarized and registered deeds of sale MONEY insofar as said requirement is concerned and a
consideration is presumed, CA did not find any alternative contract is NOT A TRUE SALE where the PRICE
allegations of the appellants that the said deed of sail were in reality CONSISTS OF SERVICES or PRESTATION.
donations. b. COURT’S RULING:
13. Hence, this appeal to the SC. i. Without necessarily according all these assertions its full
concurrence, but upon the consideration alone that the
ISSUE: apparent gross, not to say enormous, disproportion
1. W/N the immense disproportion between the assessed value of the between the stipulated price (in each deed) of P l.00 plus
parcels of land (10K+) and the actual price (1 peso) given under the unspecified and unquantified services and the indisputably
said sales plus the services rendered by Private respondents to valuable real estate allegedly sold worth at least
Mateum prior to his death constitutes a consideration as P10,500.00 going only by assessments for tax purposes
contemplated in the law.—NO which, it is well-known, are notoriously low indicators of
actual value plainly and unquestionably demonstrates
a. W/N there was a valid contract of sale between Mateum and that they state a false and fictitious consideration, and
herein private respondents? – NONE no other true and lawful cause having been shown, the
2. W/N Bagnas et. al (petitioners) have the right to impugned or seek Court finds both said deeds, insofar as they purport to be
annulment of the sales even if they were mere collateral relatives of sales, not merely voidable, but void ab initio.
Mateum. – YES. Because deeds of sale is VOID.
ii. Neither can the validity of said conveyances be defended
on the theory that their true causa is the liberality of the
transferor and they may be considered in reality
donations because the law also prescribes that donations of
immovable property, to be valid, must be made and
accepted in a public instrument, and it is not denied by the
respondents that there has been no such acceptance which
they claim is not required.
iii. THEREFOR, there was no Valid sale between deceased
Mateum and herein private respondents for want of
consideration that is a PRICE in MONEY or Its
EQUIVALENT.
2. SECOND ISSUE:
a. Citing Justice JBL Reyes in his opinion in Armentia:
I cannot bring myself to agree to the proposition that the heirs’ intestate
would have no legal standing to contest the conveyance made by the deceased
if the same were made without any consideration, or for a false and fictitious
consideration. For under the Civil Code, Art. 1409, par. 3, contracts with a
cause that did not exist at the time of the transaction are inexistent and void
from the beginning. The same is true of contracts stating a false cause
(consideration) unless the persons interested in upholding the contract
should prove that there is another true and lawful consideration therefor.

If therefore the contract has no causa or consideration, or the causa is false


and fictitious (and no true hidden causa is proved) the property allegedly
conveyed never really leaves the patrimony of the transferor, and upon the
latter's death without a testament, such property would pass to the
transferor's heirs intestate and be recoverable by them or by the
Administrator of the transferor's estate.

b. It therefore seems clear that insofar as it may be considered as


setting or reaffirming precedent, Armentia only ruled that transfers
made by a decedent in his lifetime, which are voidable for having
been fraudulently made or obtained, cannot be posthumously
impugned by collateral relatives succeeding to his estate who are not
principally or subsidiarily bound by such transfers. For the reasons
already stated, that ruling is not extendible to transfers which,
though made under closely similar circumstances, are void ab
initio for lack or falsity of consideration.
REPUBLIC v. PHIL. RESOURCES DEV. CORP. (PELINO) belonged to the company, without the president and stockholders’ consents,
January 31, 1958 | Padilla, J. | Payment & Price to the BOP in an attempt to settle his personal debts.
5. PRDC took steps to recover the said goods from the BOP but BOP refused,
Petitioner: Republic of the Philippines which is why they wanted to intervene.
Respondents: Philippine Resources Development Corporation and the Court of 6. CFI denied the motion to intervene, but the CA granted.
Appeals 7. CA found no merit in the contention of the respondents. Although the subject
of the original case is a sum of money, the materials belonging to the
Summary: BOP filed a case against Apostol for the remaining amount unpaid for corporation have been assigned by Apostol as tokens of payment to his
Apostol’s purchase of logs and other goods. PRDC filed a motion to intervene on the private debts with the BOP.
ground that Apostol, as president of PRDC but without the consent of the stockholders, 8. In this case, RP contends that:
disposed of goods that the company owns to BOP in order to pay his private debt; they a. PRDC has no legal interest in the matter in litigation because the suit is
were not able to recover from BOP because BOP refused, which is why they wanted to
only for a collection of a sum of money and that what PRDC seeks to
intervene. CFI denied the motion but the CA granted it since the materials paid off by
recover is ownership and possession of the G.I. sheets, black sheets, M.S.
Apostol to the BOP belonged to PRDC. Hence, this petition. RP contends that PRDC
plates, round bars and G.I. pipes that it claims to own.
has no legal interest in the case and that price is always paid in terms of money, and
since what was paid was “in kind”, such is no payment at all. The issue in this case is b. Allowing the intervention would change the personal action into one ad
whether or not payment is limited to money. The SC held that payment is not limited rem and would delay the disposition of the case.
to money. RP based its argument on Art. 1458 of the CC in that since price is always c. Price is always paid in terms of money, and since what was paid was
paid in money, if the payment is in kind, it is not payment at all, but the SC said that “in kind”, such is no payment at all.
the same article provides that payment may be “a price certain in money or its
equivalent”, which means that it need not be necessarily money. Issue: WON payment is limited to money. – NO.

Doctrine: Although Article 1458 of the Civil Code provides that “price is always paid Held: The judgment under review is AFFRIMED, without pronouncement as to costs.
in terms of money and the supposed payment being in kind is no payment at all”, it also
provides that the purchaser can pay “a price certain in money or its equivalent” which Ratio:
means that the payment of the price need not be money. On whether payment is limited to money
• RP: Based on Art. 1458 of the CC, price is always paid in terms of money
Facts: and since the payment is in kind, it is no payment at all.
1. Republic of the Philippines (RP), in representation of the Bureau of Prisons • SC: The same article provides that the purchaser may pay “a price certain in
(BOP) filed a case against Macario Apostol (Apostol) and Empire Insurance money or its equivalent”, which means that payment need not be in money.
Co. (EIC). • Whether the aforementioned sheets, bars, plates, or pipes claimed by PRDC
2. The complaint against Apostol avers the following: belong to it and delivered to BOP by Apostol in payment of his account is
a. Apostol submitted the highest bid (Php 450.00 per ton) for 100 tons of sufficient payment. Should the CFI hold that it is as to credit Apostol with
Palawan Almaciga from the BOP; a contract was drawn and Apostol the value or price of the materials, PRDC would be affected if indeed their
obtained goods from the BOP valued at Php 15,878.59, but he only paid claim is true.
Php 691.10, leaving a balance. •
b. Apostol also submitted the best bid with the BOP for 3 million board feet On whether counsel for PRDC has no authority to represent/sue in its behalf
of logs at Php 88.00 per 1k board feet and a contract was drawn, to which • RP: Counsel of PRDC has no authority to sue in its behalf.
Apostol received logs valued at Php 65,830 and he failed to pau a balance • SC: Power of a corporation to sue and be sued in any court is lodged in the
of Php 18,827.57. board of directors which exercises its corporate powers, and not in the
c. EIC was also included in the complaint since they executed a president, as contended by RP.
performance bond of Php 10k in favor of Apostol. o CA was satisfied that counsel was duly authorized by his client to file
3. Apostol interposed payment as a defense and sought the dismissal of the the complaint-in-intervention and to appear on its behalf; findings of the
complaint. CA should not be disturbed.
4. Phil. Resources Dev. Corp. (PRDC) moved to intervene since when Apostol o Granting that he is not authorized, as stockholder and director of PRDC,
was the president of the corporation, he disposed of certain goods that he may sue on its behalf and file the complain-in-intervention with the
court.
HYATT ELEVATORS AND ESCALATORS CORP. v. CATHEDRAL adjustment and lubrication of machinery, motors, control parts and accessory
HEIGHTS BLDG. COMPLEX ASSOCIATION (MICA) equipments, including switches and electoral wirings (BASICALLY
December 1, 2010 | Peralta, J. | Unjust Enrichment MAINTAIN NGA NAMAN)
4. Hyatt claims that during the period of April 1997 to July 1998, the expenses
amounted to P1,161,933.47. After a series of demand letters, Cathedral still
PETITIONER: Hyatt Elevators and Escalators Corporation
refused to pay.
RESPONDENTS: Cathedral Heights Building Complex Association, Inc.
5. Hyatt filed a complaint for sum of money against the respondent before the
RTC.
SUMMARY: Hyatt and Cathedral entered into a service agreement wherein,
6. RTC: ruled in favor of the petitioner ordering the latter to pay:
Hyatt would maintain 4 of the elevators of Cathedral. Hyatt sent the respondent
a. P1,161,933.27 – costs of the elevator parts used, and for services
demand letters for the payment of the maintenance expenses but Cathedral refused
and maintenance, with legal rate of interest from filing
to pay. RTC said that there was a contract of sale. CA reversed this. It ruled that
b. 50,000 – attorney’s fees
Cathedral did not give its consent to the purchase of the spare parts installed. SC
c. cost of suit
held that there is indeed no contract of sale because The fixing of the price can
7. RTC also held that based on the sales invoices presented by petitioner, a
never be left to the decision of one of the contracting parties. But a price fixed
contract of sale of goods was entered into between the parties. Since
by one of the contracting parties, if accepted by the other, gives rise to a
petitioner was able to fulfill its obligation, it was incumbent on respondent to
perfected sale. In this case, Hyatt was the only one who determined the price.
pay for the services rendered.
However, respondent is partly to blame. They received the parts but failed to
8. RTC also said that the claim of the respondent that the parts were never
remind the petitioners to follow the SOP. It is not an uncommon practice for
delivered and that the repairs were questionable was a mere afterthought. MR
contractors to deliver materials and to bill the client at a later date, specially
was denied.
since the parties in the present action have an existing Service Agreement.
9. CA reversed the decision. It said that Cathedral did not give its consent to the
Withal, it is indisputable that the repairs made on the elevators ultimately
purchase of the spare parts allegedly installed in the defective elevators. Also,
redounded to the benefit of respondent for without said repairs, the elevators
there was no perfected contract of sale because there was no meeting of minds
would not be operational. Under Article 2142 of the Civil Code, such acts
upon the price.
“give rise to the juridical relation of quasi-contract to the end that no one
10. CA ruled that the Service Agreement did not give petitioner the unbridled
shall be unjustly enriched or benefited at the expense of another.” It would
license to purchase and install any spare parts and demand payment accdg to
certainly be unjust for respondent to benefit from the repairs done by
its own dictated price. MR was denied
petitioner only to refuse payment because the papers submitted were not in
order.
ISSUE/s:
12. WoN there is a perfected contract of sale between petitioner and respondent
with regard to the spare parts delivered and installed by petitioner on the four
DOCTRINE: By the contract of sales, one of the contracting parties obligates elevators of respondent at its hospital under the agreement to service
himself to transfer the ownership of and deliver a determinate thing, and the elevators as to render respondent liable for their prices – No but, since there
other to pay therefor a price certain in money or its equivalent. The absence of was benefit to the respondent, they should still pay.
any of the essential elements will negate the existence of a perfected contract
of sale. The sale is inefficacious when the price is neither certain or RULING: Wherefore, premises considered, the petition is granted.
ascertainable. But if the buyer appropriates the object, he must pay a
reasonable price. (Art. 1474) RATIO:
Procedural issue:
FACTS: 1. Respondent claimed that the petition should be denied due course for raising
1. This case is a petition for review on certiorari under Rule 45 of the Rules of questions of fact.
Court. 2. The determination of whether there exists a perfected contral of sale is
2. Oct. 1, 1994—Hyatt Elevators and Escalators Corp (Hyatt) entered into an essentially a question of fact
“Agreement to Service Elevators” (Service Agreement) with Cathedral 3. It is a well-settled rule that the jurisdiction of the SC is limited to reviewing
Heights Building Complex Association, Inc. (Cathedral). errors of law and findings of fact of the CA are conclusive. However, there
3. Under the Service Agreement: Hyatt would maintain four passenger elevators are exceptions: (listed sa case but yung nag-aapply isJ
installed in Cathedral’s building. This included monthly inspection,
a. When the findings are contrary to those of the trial court the two individuals who were privy to the transactions. Petitioner also did not
4. RTC and the CA arrived at conflicting findings of fact offer any evidence that the engineer accepted the parts.
13. By the contract of sale, one of the contracting parties obligates himself to
Substantive issue: transfer the ownership of and deliver a determinate thing, and the other to pay
1. Petitioner contends that the CA erred when it ruled that there was no perfected therefore a price certain in money or its equivalent. Absence of any of the
contract of sale between petitioner and respondent with regard to the spare essential requirements negates the existence of a perfected contract of sale
parts delivered and installed 14. Based on the record, it was only Hyatt who determined the price, wihout the
2. According to the Service Agreement: respondent “shall pay whatever acceptance or conformity of CHBCAI. Petitioner failed to secure the
additional charges in connection with the repair, supply of parts…” necessary purchase orders form Cathedral’s Board of Directors or Finance
3. Respondnet claimed that petitioner had not complied with the Standard Manager
Operating Procedure (SOP) following a breakdown of an elevator 15. From the moment the determination of the pirce is left to the judgment of one
4. According to Celestino Aguilar, respondent’s witness, the SOP following an of the contracting parties, it cannot be said that there has been an arrangement
elevator breakdown is as follows: (a) Hyatt’s technician will be notified, (b) on the price since it is not possible for the other contracting party to agree on
the technician will evaluate the problem and if it is possible, to repair it right something of which he does not know beforehand
there and then, (c) present the defective parts to the building administrator 16. The fixing of the price can never be left to the decision of one of the
and a quotation is made, (d) this is indorsed to Cathedral’s Finance Dept, (e) contracting parties. But a price fixed by one of the contracting parties, if
a purchase order is prepared and submitted for the Board of Directors for accepted by the other, gives rise to a perfected sale.
approval 17. Respondent is partly to blame because respondent and its employees never
5. Respondent contends that petitioner had failed to follow the SOP since no once questioned the authority of petitioner to install replacement parts during
purchase orders form the Finance Manager or Board of Directors relating to the repairs. They should have reminded them to follow the SOP
the supposed parts used were secured prior to the repairs 18. Also, based on the testimony of Mr. Perfecto Cruz, respondent’s witness: The
6. The SOP was not embodied in the agreement. However, the SOP was an security guards were aware of the installation of parts done; that their
industry practice. employees accepted the defective parts replaced; and that the information
7. Petitioner’s excuse was that it was the respondent who requested that the SOP clerk was authorized to accept deliveries and that the parts received were used
not be followed to repair their elevators.
8. Petitioner presented Wilson Sua, its finance manager: (testimony in Court): 19. The trouble call reports, sales invoices and delivery receipts were also signed
a. Process: call technician, dispatch technicians, request the parts to the by respondent’s employees
office, install parts to be checked for a couple of weeks, if it is 20. On a final note, the Court disagreed with the findings of the CA that the
correct then they will secure the approval. Afterwards, issue claims of petitioner are questionable, because the date of the sales invoice
invoices and delivery receipts and the date stated in the corresponding delivery receipt are too far
b. All are in writing and written within the day of the torouble apart.
c. Install before rdelivery receipt because there was an agreement 21. It is not an uncommon practice for contractors to deliver materials and
between the building engineer and Hyatt’s service manager that the to bill the client at a later date, specially since the parties in the present
elevator should be running in good condition at all times, breakdown action have an existing Service Agreement.
should be at least one day only. If there’s a complaint that the parts 22. Withal, it is indisputable that the repairs made on the elevators
were not actually installed or delivered or did not agree with the ultimately redounded to the benefit of respondent for without said
price indicated, then it should bring its complaint or disagreement to repairs, the elevators would not be operational.
the attention of petitioner 23. Under Article 2142 of the Civil Code, such acts “give rise to the juridical
d. This was a verbal agreement relation of quasi-contract to the end that no one shall be unjustly
9. Petitioner claimed that this was because of the special circumstances of the enriched or benefited at the expense of another.”
building being a hospital (ST. LUKES, IN CASE TANUNGIN) 24. It would certainly be unjust for respondent to benefit from the repairs
10. Following the procedure testified by Sua, there was no complaint or done by petitioner only to refuse payment because the papers submitted
disagreement as to the prices of the spare parts. were not in order.
11. Burden of proof is with the plaintiff. Court found Sua’s testimony insufficient
because a mere allegation is not evidence.
12. Petitioner did not present the service manager and Engr. Tisor who were to
Navarra v. Planters Development bank (Cristelle) FACTS:
July 12, 2007| GARCIA, J.| Manner of Payment” of Price Is Essential (Art. 1179) 1. Spouses Jorge and Carmelita Navarra obtained loan of 1.2 M from Planters
PETITIONER: SPS. JORGE NAVARRA and CARMELITA BERNARDO Bank.
NAVARRA and RRRC DEVELOPMENT CORPORATION, 2. They mortgaged 5 LOTS for security. Couple failed to pay, so the bank
RESPONDENTS: PLANTERS DEVELOPMENT BANK and ROBERTO foreclosed on the mortgage and sold it for more than 1.3 M. Bank was highest
GATCHALIAN REALTY, INC. bidder. One year redemption expired w/o it having been redeemed by couple.
SUMMARY: Navarra spouses are the owners of 5 parcels of land in BF Homes, 3. RRRC Development Corporation on the other hand, a real estate company
Paranaque. In 1982, they obtained a loan of P1.2M from Planters Bank, secured by a owned by the parents of Carmelita, obtained a loan with the same bank.
mortgage over these parcels of land. Unfortunately, they defaulted to pay their 4. They also mortgaged a certain property as security. They also failed to pay
obligation and thus, Planters Bank foreclosed the property. They were not able to and the mortgaged assets was foreclosed. BUT they were able to negotiate
redeem the property as well. On the other hand, RRRC Dev. Corp. is a real estate with the bank by way of concession.
company owned by the parents of Carmelita Navarra. It obtained a loan from Planters 5. Eventually, the foreclosed properties of RRRC were sold to third persons
Bank secured by a mortgage over another set of properties of RRRC. Likewise, it whose payments were directly made to the Bank, were in excess by
defaulted and the properties were foreclosed. However, RRRC was able to negotiate P300,000.00 for the redemption price.
with the Bank for the redemption of the properties by was of a concession whereby 6. In July 1985 - Back to the spouses, Jorge sent a letter to the bank proposing
the Bank allowed RRRC to refer to it would-be buyers of the properties who would to repurchase the said 5 LOTS previously foreclosed.
remit their payments directly to the Bank, which would then be considered as 7. In response, Planters Bank, thru its Vice-President wrote back Navarra via a
redemption price for RRRC. Eventually, these were sold and payments made directly letter agreeing to the request and telling him to see the Head of the bank’s
to the Bank were in excess by P300K for the redemption price. In the meantime, Jorge Acquired Assets Unit for the details of the transaction so that they may work
Navarra requested that they repurchase their house and lot for P300K, which the Bank on the necessary documentation.
agreed. Accordingly, Jorge Navarra requested further that the excess payment of 8. In August 1985 - Jorge went to see the Head with a letter requesting that the
RRRC be applied as down payment for their repurchase. For his failure to submit a excess payment ofP300,000.00 in connection with the redemption made by
board resolution from RRRC authorizing such, the Bank refused to apply the excess the RRRC be applied as down payment for the Navarras’ repurchase of their
to his repurchase. In 1988, a portion of the lots was sold to Gatchalian Realty. Navarra foreclosed properties but because the amount of P300,000.00 was sourced
spouses filed for specific performance against Planters Bank, alleging that there was from a different transaction between RRRC and Planters Bank and involved
a perfected contract of sale (P1.8M, with P300K downpayment). RTC ruled in favor different debtors, the Bank required Navarra to submit a board resolution
of Navarra spouses. CA reversed. from RRRC authorizing him to negotiate for and its behalf and empowering
him to use the amount
ISSUE: W/N there was a valid contract of sale? NO. While the letters indicate the 9. In Jan 1987 - Planters Bank sent a letter to Jorge Navarra informing him
amount of P300K as downpayment, they are completely silent as to how the that it could not proceed with the documentation of the proposed
succeeding installment payment shall be made. At most, the letters merely repurchase of the foreclosed properties on account of his non-
acknowledge that the down payment was agreed upon by the parties. However, this compliance with the Bank’s request for the submission of the needed
fact cannot lead to the conclusion that a contract of sale had been perfected. Before a board resolution of RRRC.
valid and binding contract of sale can exist, the manner of payment of the purchase 10. Navarra claimed having already delivered copies of the required board
price must first be established since the agreement on the manner of the payment goes resolution to the Bank. The Bank, however, did not receive said copies.
into the price such that a disagreement on the manner of payment is tantamount to a 11. In June 1987 - Navarras filed their complaint for Specific Performance
failure to agree on the price. Moreover, the letter/offer failed to specify a definite against bank. Planters Bank asserted however that there was no perfected
amount of the purchase price for the sale/repurchase of the properties. It merely stated contract of sale because the terms and conditions for the repurchase have not
that it will be based on the redemption value plus accrued interest at the prevailing yet been agreed upon
rate up to the date of the sales contract. Clearly, the lack of a definite offer on the part 12. Sep 1988 – Planters bank sold the properties to Gatchalian Realty
of the Navarra spouses could not possibly serve as the basis of their claim that the 13. RTC ruled for the Navarra spouses and said there was perfected Contract of
sale was perfected. Sal.
14. The CA reversed the trial court ruling.
DOCTRINE: A definite agreement on the manner of payment of price is an essential
element in the formation of a binding and enforceable contract sale; without which ISSUES: Whether or not there was a perfected Contract of sale? NO. SC upheld the
the sale is void and an action for specific performance must fail. CA decision.
RULING: WHEREFORE, the petition is DENIED and the assailed decision and Here, what is dramatically clear is that there was no meeting of minds vis-
resolution of the Court of Appeals are AFFIRMED. No pronouncement as to costs. a-vis the price, expressly or impliedly, directly or indirectly.
SO, ORDERED.
Stages of contracts: Negotiation begins from the time the prospective contracting
RATIO: parties manifest their interest in the contract and ends at the moment of their
1. Navarras assert that the following exchange of correspondence between them agreement. Perfection or birth of the contract takes place when the parties agree upon
and Planters Bank constitutes the offer and acceptance. The July 1985 letter the essential elements of the contract, i.e., consent, object and price. Consummation
being the offer from Navarra and the Aug 1985 letter-reply from the Bank occurs when the parties fulfill or perform the terms agreed upon in the contract,
the acceptance. BUT SUCH WERE NOT “CERTAIN OFFER” and culminating in the extinguishment thereof.
“ABSOLUTE ACCEPTANCE”. Negotiation: formally initiated by an offer which should be certain with respect to
2. While the foregoing letters indicate the amount of P300,000.00 as down both the object and the cause or consideration of the envisioned contract. In order to
payment, they are, however, completely silent as to how the succeeding produce a contract, there must be acceptance, which may be express or implied, but it
installment payments shall be made. At most, the letters merely acknowledge must not qualify the terms of the offer. The acceptance of an offer must be
that the down payment of P300,000.00 was agreed upon by the parties. unqualified and absolute to perfect the contract. In other words, it must be identical
3. However, this fact cannot lead to the conclusion that a contract of sale had in all respects with that of the offer so as to produce consent or meeting of the minds.
been perfected. Quite recently, this Court held that before a valid and binding
contract of sale can exist, the manner of payment of the purchase price
must first be established since the agreement on the manner of payment
goes into the price such that a disagreement on the manner of payment
is tantamount to a failure to agree on the price.
4. Navarras’ letter/offer failed to specify a definite amount of the purchase price
for the sale/repurchase of the subject properties. It merely stated that the
"purchase price will be based on the redemption value plus accrued interest
at the prevailing rate up to the date of the sales contract."
5. The ambiguity of this statement only bolsters the uncertainty of the Navarras’
so-called "offer" for it leaves much rooms for such questions.
6. Also, not clear insofar as concerned the exact number of years that will
comprise the long-term payment scheme. As we see it, the absence of a
stipulated period within which the repurchase price shall be paid all the more
adds to the indefiniteness of the Navarras’ offer.
7. Further, the tenor of Planters Bank’s letter-reply negates the contention
of the Navarras that the Bank fully accepted their offer. The letter
specifically stated that there is a need to negotiate on the other details of the
transaction before the sale may be formalized.
8. Such statement in the Bank’s letter clearly manifests lack of agreement
between the parties as to the terms of the purported contract of
sale/repurchase, particularly the mode of payment of the purchase price and
the period for its payment. The law requires acceptance to be absolute and
unqualified.
9. In this case one essential element of a contract of sale is wanting: the price
certain. There can be no contract of sale unless the following elements
concur: (a) consent or meeting of the minds; (b) determinate subject
matter; and (c) price certain in money or its equivalent.
10. Such contract is born or perfected from the moment there is a meeting of
minds upon the thing which is the object of the contract and upon the price.
MANILA METAL CONTAINER CORPORATION v PNB (Clark) FACTS:
December 20, 2006 | Callejo, Sr. J. | Negotiation 1. Manila Metal Container Corporation is an owner of an 8,015 sqm land
located in Mandaluyong, Metro Manila. The property was covered by TCT
PETITIONER: Manila Metal Container Corporation, Reynaldo C. Tolentino in the Registry of Deeds of Rizal. To secure a P 900,000 loan it had obtained
RESPONDENTS: Philippine National Bank, DMCI-Project Developers, Inc from PNB, Manila Metal executed a real estate mortgage over the lot. PNB
later granted Manila Metal a new credit accommodation of P1,000,000. On
November 16, 1973, Manila Metal executed an Amendment of Real Estate
SUMMARY: Manila Metal was the owner of 8,015 square meters of parcel of
Mortgage over its property.
land located in Mandaluyong City, Metro Manila. To secure a P900,000 loan it
2. Manila Metal secured another loan of P653,000 from PNB, payable in
had obtained from respondent Philippine National Bank, Manila Metal executed
quarterly installments of P32,650, plus interests and other charges.
a real estate mortgage over the lot. PNB later granted petitioner a new credit
3. On August 5, 1982, PNB filed a petition for extrajudicial foreclosure of the
accommodation. On August 5, 1982, PNB filed a petition for extrajudicial
real estate mortgage and sought to have the property sold at public auction
foreclosure of the real estate mortgage and sought to have the property sold at
for P911,532.21, Manila Metal's outstanding obligation to PNB as of June
public auction. After due notice and publication, the property was sold at public
30, 1982, plus interests and attorney's fees.
action where PNB was declared the winning bidder. Manila Metal sent a letter to
4. After due notice and publication, the property was sold at public auction on
PNB, requesting it to be granted an extension of time to redeem/repurchase the
September 28, 1982 where PNB was declared the winning bidder
property. Some PNB personnel informed that as a matter of policy, the bank does
for P1,000,000. The Certificate of Sale issued in its favor was registered with
not accept “partial redemption”. Since Manila Metal failed to redeem the property,
the Office of the Register of Deeds of Rizal, and was annotated at the dorsal
the Register of Deeds cancelled TCT No. 32098 and issued a new title in favor of
portion of the title on February 17, 1983. Thus, the period to redeem the
PNB. Meanwhile, the Special Asset Management Department (SAMD) had
property was to expire on February 17, 1984.
prepared a statement of account of Manila Metal’s obligation. It also
5. Manila Metal sent a letter dated August 25, 1983 to PNB, requesting that it
recommended the management of PNB to allow Manila Metal to repurchase the
be granted for a one year extension from February 17, 1984 to
property for P1,574,560. PNB rejected the offer and recommendation of SAMD.
redeem/repurchase the property. PNB informed petitioner that the request had
It instead suggested to Manila Metal to purchase the property for P2,660,000, in
been referred to its Pasay City Branch for appropriate action and
its minimum market value. Manila Metal declared that it had already agreed to
recommendation.
SAMD’s offer to purchase for P1,574,560.47 and deposited a P725,000. Manila
6. Some PNB Pasay City Branch personnel informed Manila Metal that as a
Metal filed a complaint against PNB for specific performance. RTC ruled in favor
matter of policy, the bank does not accept “partial redemption.” Since Manila
PNB. CA affirmed. The issue is WoN Manila Metal and PNB had entered into a
Metal failed to redeem the property, the Register of Deeds cancelled TCT
perfected contract for Manila Metal to repurchase the property from PNB. The SC
No. 32098 on June 1, 1984, and issued a new title in favor of PNB. Manila
held that there is no perfected contract of sale. The statement of account prepared
Metal’s offers had not yet been acted upon by PNB.
by the SAMD stating that the net claim of respondent as of June 25, 1984
7. Meanwhile, the Special Assets Management Department (SAMD) had
was P1,574,560.47 cannot be considered an unqualified acceptance to petitioner's
prepared a statement of account, and as of June 25, 1984 Manila Metal’s
offer to purchase the property. The statement is but a computation of the amount
obligation amounted to P1,574,560.47. This included the bid price
which petitioner was obliged to pay in case respondent would later agree to sell
of P1,056,924.50,.
the property. What the parties had at best was a negotiation wherein there is still
8. When apprised of the statement of account, Manila Metal remitted P725,000
no acceptance of the offer.
to PNB as “deposit to repurchase,” and Official Receipt No. 978191 was
issued to it. In the meantime, the SAMD recommended to the management
DOCTRINE: A negotiation is formally initiated by an offer, which, however, of PNB that Manila Metal be allowed to repurchase the property
must be certain. At any time prior to the perfection of the contract, either for P1,574,560.
negotiating party may stop the negotiation. At this stage, the offer may be 9. On November 14, 1984, the PNB management informed Manila Metal that it
withdrawn; the withdrawal is effective immediately after its manifestation. To was rejecting the offer and the recommendation of the SAMD. It was
convert the offer into a contract, the acceptance must be absolute and must not suggested that Manila Metal purchase the property for P2,660,000, its
qualify the terms of the offer; it must be plain, unequivocal, unconditional and minimum market value.
without variance of any sort from the proposal. a. PNB gave Manila Metal until December 15, 1984 to act on the
proposal; otherwise, its P725,000.00 deposit would be returned and
the property would be sold to other interested buyers.
b. Manila Metal declared that it had already agreed to the SAMD's should be lowered to P1,574,560.47. Clearly therefore, there was no meeting
offer to purchase the property for P1,574,560.47, and that was why of the minds between the parties as to the price or consideration of the sale.
it had paid P725,000.00. Manila Metal warned PNB that it would a. Petitioner's original offer to purchase the subject property had not
seek judicial recourse should PNB insist on the position. been accepted by respondent PNB. In fact, it made a counter-offer
10. On June 4, 1985, PNB informed Manila Metal that the PNB Board of through its June 4, 1985 letter specifically on the selling price;
Directors had accepted Manila Metal's offer to purchase the property, but petitioner did not agree to the counter-offer; and the negotiations did
for P1,931,389.53 in cash less the P725,000.00 already deposited with it. not prosper.
Manila Metal did not respond, so PNB requested Manila Metal to submit an b. Moreover, petitioner did not pay the balance of the purchase price
amended offer to repurchase. Manila Metal rejected PNB's proposal. within the sixty-day period set in the June 4, 1985 letter of
11. Manila Metal filed a complaint against PNB for “Annulment of Mortgage respondent PNB. Consequently, there was no perfected contract of
and Mortgage Foreclosure, Delivery of Title, or Specific Performance with sale, and as such, there was no contract to rescind.
Damages.”
a. PNB averred, as a special and affirmative defense, that it had ISSUE:
acquired ownership over the property after the period to redeem had 1. WoN Manila Metal and PNB had entered into a perfected contract for Manila
elapsed. It claimed that no contract of sale was perfected between it Metal to repurchase the property from PNB – NO
and Manila Metal after the period to redeem the property had
expired. RULING: In sum, then, there was no perfected contract of sale between petitioner
12. On March 18, 1993, Manila Metal offered to repurchase the property and respondent over the subject property. IN LIGHT OF ALL THE
for P3,500,000. The offer was however rejected by PNB. According to it, the FOREGOING, the petition is DENIED. The assailed decision is AFFIRMED.
prevailing market value of the property was approximately P30,000,000, and Costs against petitioner Manila Metal Container Corporation.
as a matter of policy, it could not sell the property for less than its market
value On June 21, 1993, Manila Metal offered to purchase the property RATIO:
for P4,250,000.00 in cash. The offer was again rejected by PNB on 1. The ruling of the appellate court that there was no perfected contract of sale
September 13, 1993. between the parties on June 4, 1985 is correct. The absence of any of the
13. RTC ruled against Manila Metal. It ordered PNB to refund the P725,000 essential elements will negate the existence of a perfected contract of sale.
deposit petitioner had made. The trial court ruled that there was no perfected 2. A definite agreement as to the price is an essential element of a binding
contract of sale between the parties; hence, petitioner had no cause of action agreement to sell personal or real property because it seriously affects the
for specific performance against respondent. rights and obligations of the parties. Price is an essential element in the
a. Respondent had rejected petitioner's offer to repurchase the formation of a binding and enforceable contract of sale. The fixing of the
property. Petitioner, in turn, rejected the terms and conditions price can never be left to the decision of one of the contracting parties. But a
contained in the June 4, 1985 letter of the SAMD. price fixed by one of the contracting parties, if accepted by the other, gives
b. While petitioner had offered to repurchase the property per its letter rise to a perfected sale.
of July 14, 1988, the amount of P643,422.34 was way below 3. A contract of sale is consensual in nature and is perfected upon mere meeting
the P1,206,389.53 which respondent PNB had demanded. of the minds. When there is merely an offer by one party without acceptance
c. The P725,000.00 remitted by petitioner to respondent PNB on June of the other, there is no contract. When the contract of sale is not perfected,
4, 1985 was a “deposit,” and not a downpayment or earnest money. it cannot, as an independent source of obligation, serve as a binding juridical
d. Meanwhile, Manila Metal’s Board of Directors approved Resolution relation between the parties.
No. 3-004, where it waived, assigned and transferred its rights over 4. The stages of a contract of sale are as follows:
the property covered by TCT No. 33099 and TCT No. 37025 in a. Negotiation, covering the period from the time the prospective
favor of Bayani Gabriel, one of its Directors. Thereafter, Bayani contracting parties indicate interest in the contract to the time the
Gabriel executed a Deed of Assignment over 51% of the ownership contract is perfected;
and management of the property in favor of Reynaldo Tolentino, b. Perfection, which takes place upon the concurrence of the essential
who later moved for leave to intervene. elements of the sale which are the meeting of the minds of the parties
14. CA affirmed the decision of the RTC. It declared that petitioner obviously as to the object of the contract and upon the price; and
never agreed to the selling price proposed by respondent PNB
(P1,931,389.53) since petitioner had kept on insisting that the selling price
c. Consummation, which begins when the parties perform their 12. We do not agree with petitioner's contention that the P725,000.00 it had
respective undertakings under the contract of sale, culminating in remitted to respondent was “earnest money” which could be considered as
the extinguishment thereof. proof of the perfection of a contract of sale under Article 1482 of the New
5. A negotiation is formally initiated by an offer, which, however, must be Civil Code: ART. 1482. Whenever earnest money is given in a contract of
certain. At any time prior to the perfection of the contract, either sale, it shall be considered as part of the price and as proof of the perfection
negotiating party may stop the negotiation. At this stage, the offer may of the contract.
be withdrawn; the withdrawal is effective immediately after its 13. This contention is likewise negated by the stipulation of facts which the
manifestation. To convert the offer into a contract, the acceptance must parties entered into in the trial court: The deposit of P725,000 was accepted
be absolute and must not qualify the terms of the offer; it must be plain, by PNB on the condition that the purchase price is still subject to the approval
unequivocal, unconditional and without variance of any sort from the of the PNB Board. Thus, the P725,000 was merely a deposit to be applied as
proposal. *highlight of the syllabus part of the purchase price of the property, in the event that respondent would
6. The rule is that except where a formal acceptance is so required, although the approve the recommendation of SAMD for respondent to accept petitioner's
acceptance must be affirmatively and clearly made and must be evidenced by offer to purchase the property for P1,574,560.47. Unless and until the
some acts or conduct communicated to the offeror, it may be shown by acts, respondent accepted the offer on these terms, no perfected contract of sale
conduct, or words of the accepting party that clearly manifest a present would arise. Absent proof of the concurrence of all the essential elements of
intention or determination to accept the offer to buy or sell. Thus, acceptance a contract of sale, the giving of earnest money cannot establish the existence
may be shown by the acts, conduct, or words of a party recognizing the of a perfected contract of sale.
existence of the contract of sale. 14. Per its letter to petitioner dated June 4, 1985, the respondent had decided to
7. A qualified acceptance or one that involves a new proposal constitutes a accept the offer to purchase the property for P1,931,389.53. However, this
counter-offer and a rejection of the original offer. A counter-offer is amounted to an amendment of respondent's qualified acceptance, or an
considered in law, a rejection of the original offer and an attempt to end the amended counter-offer, because while the respondent lowered the purchase
negotiation between the parties on a different basis. Consequently, when price, it still declared that its acceptance was subject to the following terms
something is desired which is not exactly what is proposed in the offer, such and conditions.
acceptance is not sufficient to guarantee consent because any modification or 15. It appears that although respondent requested petitioner to conform to its
variation from the terms of the offer annuls the offer. The acceptance must amended counter-offer, petitioner refused and instead requested respondent
be identical in all respects with that of the offer so as to produce consent or to reconsider its amended counter-offer. Petitioner's request was ultimately
meeting of the minds. rejected and respondent offered to refund its P725,000 deposit.
8. In this case, petitioner had until February 17, 1984 within which to redeem
the property. However, since it lacked the resources, it requested for more
time to redeem/repurchase the property under such terms and conditions
agreed upon by the parties. The request, which was made through a letter
dated August 25, 1983, was referred to the respondent's main branch for
appropriate action.
9. When the petitioner was told that PNB did not allow “partial redemption,” it
sent a letter to PNB’s President reiterating its offer to purchase the
property. There was no response to petitioner's letters dated February 10 and
15, 1984.
10. The statement of account prepared by the SAMD stating that the net claim of
respondent as of June 25, 1984 was P1,574,560.47 cannot be considered an
unqualified acceptance to petitioner's offer to purchase the property. The
statement is but a computation of the amount which petitioner was obliged to
pay in case respondent would later agree to sell the property.
11. There is no evidence that the SAMD was authorized by respondent's Board
of Directors to accept petitioner's offer and sell the property
for P1,574,560.47. Any acceptance by the SAMD of petitioner's offer would
not bind respondent.
TAYAG v. LACSON (Ella) obligation on the person holding the option aside from the consideration for the
March 25, 2004 | Callejo Sr., J. | Option Contract offer. Until accepted (exercised), it is not treated as a sale.

PETITIONER: Herminio Tayag


RESPONDENTS: Angelica Tiotuyco Vda. de Lacson, and her children Felicidad FACTS:
Mardo 1. Respondents Angelica Tiotuyco Vda. de Lacson,[3] and her children
Amancia, Antonio, Juan, and Teodosia, all surnamed Lacson, were the
SUMMARY: The Lacsons are registerd owners of 3 parcels of land in Pampanga. registered owners of three parcels of land located in Mabalacat, Pampanga.
The properties are tenanted agricultural lands administered by Renato Espinosa
2. The properties, which were tenanted agricultural lands,[4] were
for the owner. On Mar. 17, 1996, 2 group of original farmers/tillers (35 farmers in
administered by Renato Espinosa for the owner.
total) individually executed in favor of Tayag separate Deeds of Assignment in
which the assignees assigned to Tayag their respective rights as tenants/tillers of 3. On March 17, 1996, a group of original farmers/tillers and another group
the landholdings possessed and tilled by them for and in consideration of P50.00 individually executed in favor of the petitioner separate Deeds of Assignment
per sqm. Said amount is payable when the legal impediemnts to the sale of the in which the assignees assigned to the petitioner their respective rights as
property to Tayag no longer existed. Tayag was also granted the exclusive right to tenants/tillers of the landholdings possessed and tilled by them for and in
buy if and when the Lacsons, with the concurrence of the tenants, agreed to sell consideration of P50.00 per square meter.
the property. Meanwhile, Tayag gave different sums of money as partial payments 4. The said amount was made payable when the legal impediments to the sale
and the tenants issued receipts. On July 24, 1996, Tayag called a meeting with the of the property to the petitioner no longer existed. The petitioner was also
tenants to discuss the implementation of the terms of their separate Deeds of granted the exclusive right to buy the property if and when the respondents,
Assignments. But the farmer-tenants ultimately decided to sell their rights & with the concurrence of the defendants-tenants, agreed to sell the property.
interests to Lacsons, the owners. So Tayag filed a complaint against the tenants
and the Lacsons, for the court to fix a period within which to pay the agreed 5. In the interim, the petitioner gave varied sums of money to the
purchase price as provided for in the Deeds of Assignment. The SC decided tenants as partial payments, and the latter issued receipts for the
against Tayag. First, the contract between Tayag and the farmer-tenants is an said amounts.
Option Contract. The obligation to pay to each of the farmers the balane of the 6. However, on August 8, 1996, the defendants-tenants, through Joven Mariano,
purchase price was conditioned on the occurrence of 1) the Lacsons agreeing to wrote the petitioner stating that they were not attending the meeting and
sell the property to Tayag, 2) legal impediments on the land no longer exist, and instead gave notice of their collective decision to sell all their rights and
3) Tayag decided to buy the property. However, the first 2 conditions are absent interests, as tenants/lessees, over the landholding to the respondents.
in this case. Second, there is actually no perfected option contract. An Option 7. On August 19, 1996, the petitioner filed a complaint with the Regional Trial
Contract is a separate & distinct contract from which the parties may enter upon Court of San Fernando, Pampanga, Branch 44, against the defendants-
the conjunction of the option. An option imposes no binding obligation on the tenants, as well as the respondents, for the court to fix a period within which
person holding the option aside from the consideration for the offer. Until accepted to pay the agreed purchase price of P50.00 per square meter to the defendants,
(exercised), it is not treated as a sale. In this case, the tenant-farmers are not the as provided for in the Deeds of Assignment.
registered owners of the land so they could not legally grant to Tayag the option
and the exclusive right to buy the property. Furthermore, it is against public policy ISSUE/s:
because the Land Reform Code and the CARP law specifically provides that if the 13. WON the contracts between the farmer-tenants and Tayag are perfected
Lacsons agreed to sell the property, the tenant-farmers shall have preferential right Option Contract = YES
to buy. Tayag wanted to get the rights over the lands even before the tenants could 14. WON the Option Contract obligated Tayag to pay the farmer-tenants the
actually get the land. balance of the purchase price = NO

DOCTRINE: An Option Contract is a condition offered by which the owner RULING: IN LIGHT OF ALL THE FOREGOING, the petition is PARTIALLY
stipulates with another that the latter shall have the right to by the property at a GRANTED. The Decision of the Court of Appeals nullifying the February 13, 1996
fixed certain time, or under, or in compliance with certain terms and conditions. and April 16, 1997 Orders of the RTC is AFFIRMED.
An option contract is a separate and distinct contract from which the parties may
enter into upon the conjunction of the option. An option imposes no binding
RATIO:
1. Under the Deeds of Assignment, the obligation of the petitioner to pay to each
of the defendants-tenants the balance of the purchase price was conditioned
on the occurrence of the following events: (a) the respondents agree to sell
their property to the petitioner; (b) the legal impediments to the sale of the
landholding to the petitioner no longer exist; and, (c) the petitioner decides
to buy the property. It is only upon the occurrence of the foregoing conditions
that the petitioner would be obliged to pay to the defendants-tenants the
balance of the P50.00 per square meter under the deeds of assignment.
2. When Tayag testified, the petitioner admitted that the legal impediments
referred to in the deeds were (a) the respondents refusal to sell their property;
and, (b) the lack of approval of the Department of Agrarian Reform.
3. There is no showing in the petitioners complaint that the respondents had
agreed to sell their property, and that the legal impediments to the agreement
no longer existed. The petitioner and the defendants-tenants had yet to submit
the Deeds of Assignment to the Department of Agrarian Reform which, in
turn, had to act on and approve or disapprove the same. In fact, Tayag was
yet to meet with the defendants-tenants to discuss the implementation of the
deeds of assignment.
4. Unless and until the Department of Agrarian Reform approved the said
deeds, if at all, the petitioner had no right to enforce the same in a court
of law by asking the trial court to fix a period within which to pay the
balance of the purchase price and praying for injunctive relief.
5. No perfect option contract. An option is a contract by which the owner of
the property agrees with another person that he shall have the right to buy his
property at a fixed price within a certain time. It imposes no binding
obligation on the person holding the option, aside from the consideration for
the offer. Until accepted, it is not, properly speaking, treated as a contract. An
option contract is a separate and distinct contract from which the parties may
enter into upon the conjunction of the option
6. In this case, Tayag receives not lands, not an agreement that he shall have
the lands, but the right to call for and receive lands if he elects.
7. In this case, the tenant-farmers, under the deeds of assignment, granted to the
petitioner not only an option but the exclusive right to buy the
landholding. But the grantors were merely the defendants-tenants, and not the
Lacsons, the registered owners of the property.
8. Not being the registered owners of the property, the defendants-tenants could
not legally grant to the petitioner the option, much less the exclusive right to
buy the property.
9. Furthermore, it is against public policy because the Land Reform Code and
the CARP law specifically provides that if the Lacsons agreed to sell the
property, the tenant-farmers shall have preferential right to buy. Tayag
wanted to get the rights over the lands even before the tenants could actually
get the land.
VILLAMOR v. RESPONDENT (EMAR) Reyeses have agreed to sell and convey to them the remaining one-
October 10, 1991| Medialdea, J. | Consideration in Option: No need to be in cash half portion whenever the need of such sale arises, either on our
part or on the part of the Villamors, at the same price of P70/sqm,
PETITIONER: SPOUSES JULIO D. VILLAMOR AND MARINA VILLAMOR excluding whatever improvement may be found (see next and last
(M. Villamor) (Villamors) page for the full Deed of Option)
4. According to M. Reyes, when her husband retired, they offered to
RESPONDENTS: THE HON. COURT OF APPEALS AND SPOUSES repurchase the lot sold by them to the spouses but M. Villamor refused and
MACARIA LABINGISA REYES (M. Reyes) AND ROBERTO REYES (Reyeses) reminded them instead that the Deed of Option in fact gave them the option
to purchase the remaining portion of the lot.
SUMMARY: M. Reyes sold ½ (300sqm) of her land (600sqm) to the Villamors for 5. The Villamors claimed that they had expressed their desire to purchase the
P70/sqm. It is provided in their Deed of Option that the Villamors paid an amount remaining portion of the lot but the Reyeses had been ignoring them.
significantly larger than the prevailing price of the land (P25/sqm) because the 6. July 1987: after conciliation proceedings in the barangay level failed, the
Reyeses agreed to sell the other half of the land for the same amount whenever either Villamors filed a complaint for specific performance against the Reyeses.
party finds the need to do so. The Villamors claimed that they had expressed their 7. The RTC ruled in favor of the Villamor spouses, ordering the Reyeses to sell
desire to purchase the remaining portion of the lot but the Reyeses had been ignoring to the Villamors the remaining land owned by them.
them and so they filed a complaint before the barangay which failed to fix their 8. The Reyeses appealed to the CA which didn’t believe that the Villamors
problem. The Villamors then, filed for specific performance before the lower court bought the land for P70/sqm instead of its prevailing price of P25/sqm for the
which ruled in favor of them, ordering the Reyeses to sell to the Villamors the consideration of the option to buy the remainder of the land because
remaining land. The Reyeses appealed in the CA which reversed the ruling of the a. A deed of sale was never produced by the Villamors to prove their
lower court, finding the deed of option void for lack of consideration. The Villamors claim and
petitioned before the SC. The issue is WON the deed of option whereby the Reyeses b. They see no reason why it should be reiterated in the Deed of
agreed to sell their lot to Villamors "whenever the need of such sale arises, either on Option.
our (Reyeses) part or on the part of Julio Villamor and Marina Villamor (Villamors)” 9. The CA reversed the decision and found that Deed of Option is void for lack
is valid and the court held that yes, it is valid (and not void for lack of consideration) of consideration; hence this petition by the Villamors.
because there is a consideration on the part of Reyeses executing the deed of option
as appearing in the deed itself which is the Villamors' having agreed to buy the ISSUE: WON the Deed of Option whereby the Reyeses agreed to sell their lot to
300sqm portion of Reyeses’ land at P70/sqm "which was greatly higher than the Villamors "whenever the need of such sale arises, either on our (Reyeses) part or on
actual reasonable prevailing price.” However, Villamors’ right of action has already the part of Julio Villamor and Marina Villamor (Villamors)” is valid – YES, but right
prescribed because they only filed for specific performance in 1987, 17y from the of action already prescribed.
time of the execution of the contract which is beyond the prescription period of 10
years within actions upon written contracts must be brought. RULING: ACCORDINGLY, the petition is DENIED. The decision of the CA is
AFFIRMED for reasons cited in this decision. Judgement is rendered dismissing the
DOCTRINE: “Separate consideration” in an option may be anything of value, complaint in Civil Case on the ground of prescription and laches.
unlike in sale where it must be the price certain in money or its equivalent.
RATIO:
1. When the terms of an agreement have been reduced to writing it is to be
FACTS: considered as containing all such terms, and therefore, there can be,
1. M. Reyes was the owner of a 600-sq m lot located in Caloocan City. between the parties and their successors in interest no evidence of their
2. July 1971: M. Reyes sold ½ of her land (300 sqm) to the Villamors for 21m. terms of the agreement, other than the contents of the writing.
2m was deducted from the purchase price as a previous debt of M. Reyes 2. It is a general and most inflexible rule that wherever written instruments
from the Villamors. The lots of M. Reyes and the Villamors became covered are appointed either by the requirements of law, or by the contract of
by different TCTs. the parties, to be the repositories and memorials of truth, any other
3. Nov 1971: Macaria executed a "Deed of Option" in favor of the spouses evidence is excluded from being used, either as a substitute for such
stating that instruments, or to contradict or alter them.
a. They sold the first half of the lot for P70/sqm which is greatly higher 3. Gonzales v. Trinidad defined “consideration” as "the why of the contracts,
compared to the value (P25/sqm) of the lots that time; the essential reason which moves the contracting parties to enter into the
b. The spouses bought their land for that price because the vendor
contract." The cause/ impelling reason on the part of private respondent for the property or the petitioners, who may compel the private
executing the deed of option as appearing in the deed itself is the petitioner's respondents to deliver the property.
having agreed to buy the 300sqm portion of private respondents' land at 9. Deed of Option did not provide for the period within which the parties may
P70/sqm "which was greatly higher than the actual reasonable demand the performance of their respective undertakings in the instrument.
prevailing price." The parties could not have contemplated that the delivery of the property and
4. In 1969, Villamors bought an adjacent lot from the brother of M. Reyes for the payment thereof could be made indefinitely and render uncertain the
only P18/sqm which the private respondents did not rebut. Thus, expressed status of the land. The failure of either parties to demand performance of
in terms of money, the consideration for the deed of option is the the obligation of the other for an unreasonable length of time renders the
difference between the purchase price of the 300 sqm portion of the lot contract ineffective.
in 1971 P70/sqm and the prevailing reasonable price of the same lot in 10. Actions upon written contract must be brought within 10y. The Deed of
1971. Whatever it is, (P25 or P18) though not specifically stated in the deed Option was executed on 1971. The acceptance was already in the same
of option, was ascertainable. Petitioner's allegedly paying P52/sqm for instrument. The complaint in this case was filed by the petitioners on 1987,
the option may, as opined by the appellate court, be improbable but 17y from the time of the execution of the contract. Hence, the right of action
improbabilities does not invalidate a contract freely entered into by the had prescribed. The Villamors allege that they demanded from the private
parties. respondents as early as 1984 the enforcement of their rights under the contract
5. The “deed of option" entered into by the parties in this case had unique but it’s still beyond the 10y prescription period.
features. Ordinarily, an optional contract is a privilege existing in one person,
for which he had paid a consideration and which gives him the right to buy,
for example, certain merchandise or certain specified property, from another DEED OF OPTION
person, if he chooses, at any time within the agreed period at a fixed price.
6. In this Deed of option, the first part covered the statement on the sale of the This Deed of Option, entered into in the City of Manila, Philippines, this 11th day of
300 sqm portion to theVillamors at the price of P70/sqm "which was higher November, 1971, by and between Macaria Labing-isa, of age, married to Roberto
than the actual reasonable prevailing value of the lands in that place at Reyes, likewise of age, and both resideing on Reparo St., Baesa, Caloocan City, on
that time (of sale)." The second part stated that the only reason why the the one hand, and on the other hand the spouses Julio Villamor and Marina V.
Villamor spouses agreed to buy the said lot at a much higher price is Villamor, also of age and residing at No. 552 Reparo St., corner Baesa Road, Baesa,
because the Reyeses also agreed to sell to the Villamors the other half- Caloocan City.
portion of the land. But the deed went on and stated that the sale of the
other half would be made "whenever the need of such sale arises, either WITNESSETH
on our (Reyeses) part or on the part of the Spouses Julio Villamor and
Marina V. Villamor.” While the option to buy was granted to the Villamors, That, I Macaria Labingisa, am the owner in fee simple of a parcel of land with an
the Reyeses were likewise granted an option to sell. In other words, it was area of 600 square meters, more or less, more particularly described in TCT No.
not only the Villamors who were granted an option to buy for which they (18431) 18938 of the Office of the Register of Deeds for the province of Rizal,
paid a consideration. The Reyeses as well were granted an option to sell issued in may name, I having inherited the same from my deceased parents, for
should the need for such sale on their part arise. which reason it is my paraphernal property;
7. The option offered by the Reyeses had been accepted by the petitioner, the
promise, in the same document. The acceptance of an offer to sell for a That I, with the conformity of my husband, Roberto Reyes, have sold one-half
price certain created a bilateral contract to sell and buy and upon thereof to the aforesaid spouses Julio Villamor and Marina V. Villamor at the price
acceptance, the offer, ipso facto assumes obligations of a vendee. of P70.00 per sq. meter, which was greatly higher than the actual reasonable
8. A contract of sale is "perfected at the moment there is a meeting of minds prevailing value of lands in that place at the time, which portion, after segregation, is
upon the thing which is the object of the contract and upon the price. From now covered by TCT No. 39935 of the Register of Deeds for the City of Caloocan,
that moment, the parties may reciprocally demand perform of contracts." (Art issued on August 17, 1971 in the name of the aforementioned spouses vendees;
1475) Since there was, between the parties, a meeting of minds upon the
object and the price, there was already a perfected contract of sale. What That the only reason why the Spouses-vendees Julio Villamor and Marina V.
wa left to be done was for either party to demand from the other their Villamor, agreed to buy the said one-half portion at the above-stated price of about
respective undertakings under the contract. It may be demanded at any time P70.00 per square meter, is because I, and my husband Roberto Reyes, have agreed
either by the private respondents, who may compel the petitioners to pay to sell and convey to them the remaining one-half portion still owned by me and now
covered by TCT No. 39935 of the Register of Deeds for the City of Caloocan,
whenever the need of such sale arises, either on our part or on the part of the spouses Notary Public
(Julio) Villamor and Marina V. Villamor, at the same price of P70.00 per square Until December 31, 1972
meter, excluding whatever improvement may be found the thereon; PTR No. 338203, Manila
January 15, 1971
That I am willing to have this contract to sell inscribed on my aforesaid title as an
encumbrance upon the property covered thereby, upon payment of the corresponding Doc. No. 1526;
fees; and Page No. 24;
Book No. 38;
That we, Julio Villamor and Marina V. Villamor, hereby agree to, and accept, the Series of 1971. (pp. 25-29, Rollo)
above provisions of this Deed of Option.

IN WITNESS WHEREOF, this Deed of Option is signed in the City of Manila,


Philippines, by all the persons concerned, this 11th day of November, 1971.

JULIO VILLAMOR MACARIA LABINGISA

With My Conformity:

MARINA VILLAMOR ROBERTO REYES

Signed in the Presence Of:

MARIANO Z. SUNIGA
ROSALINDA S. EUGENIO

ACKNOWLEDGMENT

REPUBLIC OF THE PHILIPPINES)


CITY OF MANILA ) S.S.

At the City of Manila, on the 11th day of November, 1971, personally appeared
before me Roberto Reyes, Macaria Labingisa, Julio Villamor and Marina Ventura-
Villamor, known to me as the same persons who executed the foregoing Deed of
Option, which consists of two (2) pages including the page whereon this
acknowledgement is written, and signed at the left margin of the first page and at the
bottom of the instrument by the parties and their witnesses, and sealed with my
notarial seal, and said parties acknowledged to me that the same is their free act and
deed. The Residence Certificates of the parties were exhibited to me as follows:
Roberto Reyes, A-22494, issued at Manila on Jan. 27, 1971, and B-502025, issued at
Makati, Rizal on Feb. 18, 1971; Macaria Labingisa, A-3339130 and B-1266104,
both issued at Caloocan City on April 15, 1971, their joint Tax Acct. Number being
3028-767-6; Julio Villamor, A-804, issued at Manila on Jan. 14, 1971, and B-138,
issued at Manila on March 1, 1971; and Marina Ventura-Villamor, A-803, issued at
Manila on Jan. 14, 1971, their joint Tax Acct. Number being 608-202-6.

ARTEMIO M. MALUBAY
NICOLAS SANCHEZ v. SEVERINA RIGOR (SARMIENTO) ISSUE/s:
JUNE 14, 1972| C.J. CONCEPCION | OPTION TO PURCHASE 15. WoN the seller is an Option to Purchase without consideration, automatically
sells the property to the buyer at the moment the buyer exercises such option.
PETITIONER: NICOLAS SANCHEZ
RESPONDENTS: SEVERINA RIGOR RULING: YES

SUMMARY: Nicolas Sanchez and Severina Rigor entered into an Option to WHEREFORE, the decision appealed from is hereby affirmed, with costs
Purchase, without consideration, a parcel of land owned by the latter. Sanchez against defendant-appellant Severina Rigos.
tendered payment to Rigor but the latter refused to accept the payment. This
RATIO:
prompted Sanchez to consign the payment to the court and instiuted an action of
1. This case admittedly hinges on the proper application of Article 1479 of our
specific performance to compel Rigor to convey the property. Lower court ruled
Civil Code, which provides:
in favor of Sanchez. Rigor filed an appeal. Supreme court said that the Rigor as
"ART. 1479. A promise to buy and sell a determinate thing for a price certain
seller is bound by the option to purchase even though without consideration.
is reciprocally demandable.
Nonetheless, seller can revoke the option anytime before the buyer exercises the
"An accepted unilateral promise to buy or to sell a determinate thing for a
option. In this case, Rigor did not withdraw the option before Sanchez exercised
price certain is binding upon the promissor if the promise is supported by a
the option. Thus, there is an ipso facto sale of the parcel of land in favor of the
consideration distinct from the price."
buyer.
2. Plaintiff alleges that, by virtue of the option under consideration, "defendant
agreed and committed to sell" and "the plaintiff agreed and committed to buy"
DOCTRINE: Even supposing that the Option to purchase granted an option
the land described in the option,
which is not binding for lack of consideration, the authorities hold that If the
• Hence, plaintiff maintains that the promise contained in the contract
option is given without a consideration, it is a mere offer of a contract of sale,
is "reciprocally demandable," pursuant to the first paragraph of said
which is not binding until accepted. If, however, acceptance is made before a
Article 1479.
withdrawal, it constitutes a binding contract of sale, even though the option was
3. Although defendant had really "agreed, promised and committed" herself to
not supported by a sufficient consideration.
sell the land to the plaintiff, it is not true that the latter had, in turn, "agreed
and committed himself" to buy said property.
4. The option did not impose upon plaintiff the obligation to purchase
FACTS: defendant's property. It is not a "contract to buy and sell." It merely granted
1. Plaintiff Nicolas Sanchez and defendant Severina Rigos executed an plaintiff an "option" to buy.
instrument, entitled "Option to Purchase," whereby Mrs. Rigos "agreed, 5. Under the provisions thereof, the defendant "agreed, promised and
promised and committed * * * to sell" to Sanchez, for the sum of P1,510.00, committed" herself to sell the land therein described to the plaintiff for
a parcel of land within two (2) years from said date, with the understanding P1,510.00, but there is nothing in the contract to indicate that her
that said option shall be deemed "terminated and elapsed," if "Sanchez shall aforementioned agreement, promise and undertaking is supported by a
fail to exercise his right to buy the property" within the stipulated period. consideration "distinct from the price" stipulated for the sale of the
2. Inasmuch as several tenders of payment of the sum of P1,510.00, made by land.
Sanchez within said period, were rejected by Mrs. Rigos, the former 6. It should be noted that:
deposited said amount with the Court of First Instance of Nueva Ecija and (1) In order that said unilateral promise may be "binding" upon the
commenced against the latter the present action, for specific performance and promisor. Article 1479 requires the concurrence of a condition, namely, that
damages. the promise be "supported by a consideration distinct from the price."
3. Defendant alleging, as special defense that the contract between the parties • Accordingly, the promisee cannot compel the promisor to comply
"is a unilateral promise to sell, and the same being unsupported by any with the promise, unless the former establishes the existence of said
valuable consideration, by force of the New Civil Code, is null and void" distinct consideration. Plaintiff herein has not even alleged the
4. Lower court rendered judgment for Sanchez, ordering Mrs. Rigos to accept existence thereof in his complaint.
the sum judicially consigned by him and to execute, in his favor, the requisite (2) Upon the other hand, defendant explicitly averred in her answer, and
deed of conveyance. pleaded as a special defense, the absence of said consideration for her promise
5. Hence, this appeal by Mrs. Rigos. to sell.
7. Southwestern Sugar & Molasses Co. vs. Atlantic Gulf & Pacific Co., from
which We quote: 8. However, this Court itself, in the cases decided later than Southwestern Sugar
• "The main contention of appellant is that the option granted to & Molasses Co. vs. Atlantic Gulf & Pacif ic Co., where a unilateral promise
appellee to sell to it barge No. 10 for the sum of P30,000 under the to sell similar to the one sued upon here was involved, treating such promise
terms stated above has no legal effect because it is not supported by as an option which, although not binding as a contract in itself for lack of a
any consideration and in support thereof it invokes article 1479 of separate consideration, nevertheless generated a bilateral contract of purchase
the new Civil Code. and sale upon acceptance.
• "On the other hand, appellee contends that, even granting that the 9. Speaking through Associate Justice, later Chief Justice, Cesar Bengzon, this
'offer of option' is not supported by any consideration, that option Court said:
became binding on appellant when the appellee gave notice to it of • "Furthermore, an option is unilateral: a promise to sell at the price
its acceptance, and that having accepted it within the period of fixed whenever the offeree should decide to exercise his option
option, the offer can no longer be withdrawn and in any event such within the specified time.
withdrawal is ineffective. • After accepting the promise and before he exercises his option, the
o In support of this contention, appellee invokes article 1324 holder of the option is not bound to buy. He is free either to buy or
of the Civil Code which provides: not to buy later.
'Art. 1324. When the offerer has allowed the offeree a 10. In this case, upon accepting herein petitioner's offer a bilateral promise
certain period to accept, the offer may be withdrawn at any time to sell and to buy ensued, and the respondent ipso facto assumed the
before acceptance by communicating such withdrawal, except when obligations of a purchaser.
the option is founded upon consideration, as something paid or 11. Even supposing that the Option to purchase granted an option which is not
promised.' binding for lack of consideration, the authorities hold that
• There is no question that under article 1479 of the new Civil Code • 'If the option is given without a consideration, it is a mere offer of a
'an option to sell,' or 'a promise to buy or to sell,' as used in said contract of sale, which is not binding until accepted.
article, to be valid must be 'supported by a consideration distinct • If, however, acceptance is made before a withdrawal, it constitutes
from the price.' a binding contract of sale, even though the option was not supported
o Here it is not disputed that the option is without by a sufficient consideration.
consideration. It can therefore be withdrawn 12. Since there may be no valid contract without a cause or consideration, the
notwithstandingthe acceptance made of it by appellee. promisor is not bound by his promise and may, accordingly, withdraw it.
• "It is true that under article 1324 of the new Civil Code, the general • Pending notice of its withdrawal, his accepted promise partakes,
rule regarding offer and acceptance is that, when the offerer gives to however, of the nature of an offer to sell which, if accepted, results
the offeree a certain period to accept, 'the offer may be withdrawn at in a perfected contract of sale.
any time before acceptance' except when the option is founded upon 13. What is more, the reference, in both the second paragraph of Art. 1479 and
consideration. Art. 1324, to an option or promise supported by or founded upon a
o But this general rule must be interpreted as modified by the consideration, strongly suggests that the two (2) provisions intended to
provision of article 1479 above referred to, which applies enforce or implement the same principle.
to 'a promise to buy and sell' specifically. 14. The view adhered to in the Southwestern Sugar & Molasses Co. case should
• "We are not oblivious of the existence of American authorities be deemed abandoned or modified.
which hold that an offer, once accepted, cannot be withdrawn,
regardless of whether it is supported or not by a consideration.
o But we are prevented from applying them in view of the
specific provision embodied in article 1479.
o While under the 'offer of option' in question appellant has
assumed a clear obligation to sell to appellee and the option
has been exercised in accordance with its terms and there
appears to be no valid or justifiable reason for appellant to
withdraw its offer, this Court cannot adopt a different
attitude because the law on the matter is clear. Our
imperative duty is to apply it unless modified by Congress."
TUAZON v. DEL ROSARIO-SUAREZ (Jenn) Miguel and Rommel (De Leon) for only P2,750,000. The TCT was then
December 13, 2010 | Del Castillo, J. | Option to Purchase without any consideration reissued in the TCT in the De Leons names.
4. The new owners, through their attorney-in-fact, tolf Roberto to vacate the
PETITIONER: Roberto D. Tuazon land but Roberto refused so the De Leons filed a complaint for Unlawful
RESPONDENTS: Lourdes Q. Del-Rosario-Suarez, Catalina R. Suarez-De Leon, Detainer. The MTC rendered a decision ordering Roberto to vacate.
Wilfredo De Leon, Miguel Luis S. De Leon, Rommel Lee S. De Leon, and 5. While the ejectment case was on appeal, Roberto filed with the RTC a
Guillerma L. Sandico-Silva, as attorney-in-fact of the defendants, except Lourdes complaint for annulment of the deed of absolute sale and reconveyance,
Q. Del Rosario damages and application for preliminary injuction against Lourdes and the
De Leons. He also filed a notice of Lis Pendens with the Registry of Deeds.
SUMMARY: Roberto leased a property owned by Lourdes for 3 years. After the 6. Lourdes and De Leons filed an answer and a counter claim praying that the
contract of lease was perfected, Lourdes sent a letter to Roberto offering to sell complaint be dismissed. Lourdes and De Leons were later declared in default
the land to him for P37,541,000 said offer to be accepted by Roberto within 2 for failing to appear before the court. The RTC then rendered a decision in
years if he wants to buy the land. After the expiration of the lease, Lourdes sold favor of Lourdes and the De Leons. It declared the deed of absolue sale
the land to the De Leons (family of her child). Roberto now claims that Lourdes between the 2 valid and that the offer made by Lourder to Roberto did not
violated his right to buy subject property under the principle of "right of first ripen into a contract to sell because the price offered by Lourdes was not
refusal" by not giving him "notice" and the opportunity to buy the property under accepted by Roberto and that the offer was no longer binding on Lourdes
the same terms and conditions or specifically based on the much lower price paid when she sold the land to the De Leons.
by the De Leons. The RTC and CA ruled in favor of the De Leons. The issue in 7. The CA affirmed the RTC decision. Hence this petition.
this case is whether or not the letter sent by Lourdes to Roberto involved an option 8. Roberto claims the following:
contract or a contract of a right of first refusal. The SC held that it was an option a. that Lourdes violated his right to buy subject property under the
contract since it grants Roberto a fixed period of only 2 years to buy the property principle of "right of first refusal" by not giving him "notice" and
at a fixed price of P37,541,000. And If there is no consideration, the offeror may the opportunity to buy the property under the same terms and
withdraw his offer by communicating such withdrawal to the offeree at anytime conditions or specifically based on the much lower price paid by the
before acceptance. If there is consideration, the offeror cannot withdraw his offer De Leons
before the lapse of the period agreed upon. Since there was no consideration, b. that he is enforcing his "right of firrst refusal" based on Equatorial
Lourdes was not bound to sell the property to Roberto and also, the sale happened Realty Development, Inc. v. Mayfair Theater, Inc. which is the
long after the 2 years expired so Roberto can’t compel Lourdes to sell the land to leading case on the "right of first refusal."
him. 9. Lourdes and the De Leons claim the following:
a. this case is not covered by the principle of "right of first refusal" but
DOCTRINE: If the option is without any consideration, the offeror may withdraw an unaccepted unilateral promise to sell or, at best, a contract of
his offer by communicating such withdrawal to the offeree at any time before option which was not perfected. The letter of Lourdes to Roberto
acceptance. If it is founded upon a consideration, the offeror cannot withdraw his clearly embodies an option contract as it grants the latter only two
offer before the lapse of the period agreed upon. years to exercise the option to buy the subject property at a price
certain of P37,541,000.00. As an option contract, the said letter
would have been binding upon Lourdes without need of any
FACTS: consideration, had Roberto accepted the offer. But in this case there
1. Respondent Lourdes was the owner of a 1,211sqm land in Tandang Sora St., was no acceptance made neither was there a distinct consideration
Quezon City which was covered by a TCT. for the option contract.
2. On June 24, 1994, Lourdes and Roberto, herein petitioner, executed a
Contract of Lease over the land for a period of 3 years. The lease started on ISSUE/s:
March 1994 and ended on February 1997. During the effectivity of the lease, 16. WoN the letter of Lourdes to Roberto involved an option contract or a
Lourdes sent a letter dated January 2, 1995 to Roberto wherein she offered to contract of a right of first refusal? –Option Contract
sell the land to the latter for P37,541,000 which Roberto had to buy within 2
years from January 2, 1995 if he wanted to. So Roberto had until January 1, RULING: WHEREFORE, the instant petition for review on certiorari is D E N I E D. The
1997 to decide. assailed Decision of the Court of Appeals in CA-G.R. CV No. 78870, which affirmed the
3. On June 19, 1997, 4 months after the expiration of the lease, Lourdes sold the Decision dated November 18, 2002 of the Regional Trial Court, Branch 101, Quezon City in
land to her only child Catalina, her son-in-law, Wilfredo, and her 2 grandkids, Civil Case No. Q-00-42338 is AFFIRMED. SO ORDERED.
RATIO: Thank you,
This case involves an option contract and not a contract of a right of first refusal. Lourdes Q. del Rosario vda. de Suarez
1. An option contract is 'A contract by virtue of which A, in consideration of the 6. It is clear that the letter is an option contract since it grants Roberto a fixed
payment of a certain sum to B, acquires the privilege of buying from, or selling to, B period of only 2 years to buy the property at a fixed price of P37,541,000.
certain securities or properties within a limited time at a specifed price. (Story vs. 7. Since it is an option contract, Art. 13241 and Art. 14792 of the Civil Code
Salamon, 71 N.Y., 420.)' applies.
2. It is simply a contract by which the owner of property agrees with another 8. Based on Art. 1324, there is a great difference if the option was without a
person that he shall have the right to buy his property at a fixed price within consideration or with a consideration. If there is no consideration, the
a certain time. He does not sell his land; he does not then agree to sell it; but offeror may withdraw his offer by communicating such withdrawal to
he does sell something; that is, the right or privilege to buy at the election or the offeree at anytime before acceptance. If there is consideration, the
option of the other party. offeror cannot withdraw his offer before the lapse of the period agreed
3. On the other hand, a right of first refusal is not a perfected contract of sale under upon.
Article 1458 of the Civil Code. In a right of first refusal, while the object might be 9. Under Art. 1479, “an accepted unilateral promise to buy or to sell a
made determinate, the exercise of the right, however, would be dependent not only on determinate thing for a price certain is binding upon the promisor if the
the grantor's eventual intention to enter into a binding juridical relation with another
but also on terms, including the price, that obviously are yet to be later rmed up.
promise is supported by a consideration distinct from the price.”
Prior thereto, it can at best be so described as merely belonging to a class of 10. In the case at bar, Roberto did not accept the terms stated in the letter of
preparatory juridical relations governed not by contracts (since the essential elements Lourdes since he negotiated for a much lower price. Roberto’s act of
to establish the vinculum juris would still be indefinite and inconclusive) but by, negotiating for a much lower price was a counter-offer and is therefore not
among other laws of general application, the pertinent scattered provisions of the Civil an acceptance of the offer. And since the counter-offer for a much lower price
Code on human conduct. was not accepted by Lourdes, there is no contract between the, with regard to
4. An option contract is entirely different and distinct from a right of first the sale of the property. So Roberto does not have the right to demand that
refusal. In an option contract, the option granted to the offeree is for a fixed the property be sold to him.
period and at a determined price. Absent these 2, it is only a right of first
refusal. Equatorial Realty Dev’t v. Mayfair Theatre is not applicable.
5. The letter is as follows: 1. The facts between the 2 cases are almost the same except that in Equatorial,
Dear Mr. Tuazon, the Contract of Lease had the following provision:
a. (i)f the LESSOR should desire to sell the leased properties, the
I received with great joy and happiness the big box of sweet grapes and ham, fit for a king's LESSEE shall be given 30-days exclusive option to purchase the
party. Thanks very much.
same.
I am getting very old (79 going 80 yrs. old) and wish to live in the U.S.A. with my only 2. There is no such provision in the case at bar. What is involved here is a
family. I need money to buy a house and lot and a farm with a little cash to start. separate and distinct offer made by Lourdes to sell the land for a definite price
and within a definite period of time. Roberto was not given a right of first
I am offering you to buy my 1211 square meter at P37,541,000.00 you can pay me in refusal. The letter of Lourdes was not part of the contract since it was made
dollars in the name of my daughter. I never offered it to anyone. Please shoulder the 6 months after the perfection of the lease contract.
expenses for the transfer. I wish the Lord God will help you buy my lot easily and you will 3. Also, in Equatorial, the property was sold within the lease period while in this
be very lucky forever in this place. You have all the time to decide when you can, but not case, the property was sold after the lease.
for 2 years or more. HCEaDI 4. Moreover, even if the offer of Lourdes was accepted by Roberto, still the former
is not bound thereby because of the absence of a consideration distinct and
I wish you long life, happiness, health, wealth and great fortune always! separate from the price. The argument of Roberto that the separate
consideration was the liberality on the part of Lourdes cannot stand. A perusal
I hope the Lord God will help you be the recipient of multi-billion projects aid from other of the letter-offer of Lourdes would show that what drove her to offer the
countries. property to Roberto was her immediate need for funds as she was already very

1
Art. 1324. When the offerer has allowed the offeree a certain period to accept, the offer may be withdrawn An accepted unilateral promise to buy or to sell a determinate thing for a price certain is binding upon the
at any time before acceptance by communicating such withdrawal, except when the option is founded upon promissor if the promise is supported by a consideration distinct from the price.
a consideration, as something paid or promised.
2
Art. 1479. A promise to buy and sell a determinate thing for a price certain is reciprocally demandable.
old. Offering the property to Roberto was not an act of liberality on the part of
Lourdes but was a simple matter of convenience and practicality as he was the
one most likely to buy the property at that time as he was then leasing the same.
VASQUEZ v. CA (Siapno) option to buy since it is not embodied in the same document of sale but in a separate
July 12, 1991 | Gutierrez Jr., J. | Right to Repurchase document, and since such option is not supported by a consideration distinct from
the price, said deed for right to repurchase is not binding upon them pursuant to
PETITIONERS: Spouses Cipriano Vasquez and Valeriana Gayanelo Article 1479 of the Civil Code.
RESPONDENTS: Spouses Martin Vallejera and Apolonia Olea 7. After trial, the court below rendered judgment against Vasquez, ordering the
spouses to resell lot No. 1860 of the Himamaylan Cadastre to the respondents for
SUMMARY: Petitioner Spouses Martin Vallejera and Apolonia Olea sold their the repurchase price of P24,000.00, which amount combines the price paid for the
land in Negros Occidental to Respondents Vasquez and Gayanelo. A Deed of Sale first sale and the price paid by Vasquez to Benito Derrama, Jr.
was executed and on the same day, in a separate instrument, respondents were 8. CA applied the principles laid down in the case of Sanchez v Rigos:
granted a Right to Repurchase. Respondents availed of the right to repurchase but a. SC arrived at the conclusion that Article 14793 and Article 13244 of the Civil
Vasquez resisted because of lack of consideration separate from the purchase price Code should be reconciled and harmonized to avoid a conflict between the two
and on the ground that the right to repurchase was made in a separate document. provisions.
Issue is WON the respondents have the right to repurchase under the contract. SC b. Sanchez and Rigos executed a document entitled "Option to Purchase,"
held that respondents cannot avail of the right to repurchase because it was not whereby Mrs. Rigos agreed, promised and committed to sell to Sanchez for a
accepted by the respondents and no consideration was given. The document certain sum of money, a registered parcel of land within 2 years from execution
of the right to repurchase was not signed by the respondents and as such, shows of the document with the condition that said option shall be deemed terminated
no acceptance from the promisee. and lapsed, if Sanchez shall fail to exercise his right to buy the property within
the stipulated period. In the same document, Sanchez agreed and conformed
DOCTRINE: For there to be a valid reservation of the right to repurchase, the with all the conditions set forth in the option to purchase executed in my
right must bereserved by the vendor in the same instrument of sale as one of the favor, that I bind myself with all the terms and conditions.
stipulations of the contract and not in a subsequent document. Otherwise, it is c. The Court said that although promise to sell was not supported by a
just an option to to buy or a mere promise to resell. consideration distinct from the price, it was obvious that Sanchez, the promisee,
accepted the option to buy before Rigos, the promisor, withdrew the same.
d. Under such circumstances, the option to purchase was converted into a bilateral
FACTS:
contract of sale which bound both parties.
1. Spouses Martin Vallejera and Apolonia Olea (respondents) filed an action against
Spouses Cipriano Vasquez and Valeriana Gayanelo (Vasquez) seeking to redeem
ISSUE: WON respondents have the right to repurchase? NO
Lot No. 1860 of the Himalayan Cadastre which was previously sold by the
respondents to Vasquez.
HELD: WHEREFORE, the petition is GRANTED. The questioned decision and
2. Said lot was registered in the name of respondents. It was was leased by
resolution of the Court of Appeals are hereby REVERSED and SET ASIDE. The
respondents to Vasquez and after the execution of the lease, Vasquez took
complaint in Civil Case No. 839 of the then Court of First Instance of Negros
possession of the lot and devoted the same to the cultivation of sugar.
Occidental 12th Judicial District Branch 6 is DISMISSED. No costs.
3. The plaintiffs sold the lot to the defendants under a Deed of Sale for the amount of
P9,000.00. On the same day and along with the execution of the Deed of Sale,
RATIO:
a separate instrument, denominated as Right to Repurchase (Exh. E: refer to
1. SC held that the Sanchez v Rigos doctrine cannot be applied. In the instant case
end of digest for exact wording), was executed by the parties granting respondents
and contrary to the appellate court's finding, it is clear that the right to repurchase
the right to repurchase the lot for P12,000.00.
was not supported by a consideration distinct from the price. The rule is that the
4. By virtue of the sale, Vasquez secured TCT No. T-58898 in their name. 5 years
promisee has the burden of proving such consideration. Unfortunately, the
later, respondents sold the same lot to Benito Derrama, Jr., after securing Vasquez’
private respondents, promisees in the right to repurchase, failed to prove such
title, for the sum of P12,000.00.
consideration. They did not even allege the existence thereof in their complaint.
5. Upon the protestations of Vasquez, assisted by counsel, the said second sale was
Therefore, in order that the Sanchez case can be applied, the evidence must show
cancelled after the payment of P12,000.00 by Vasquez to Derrama.
that the private respondents accepted the right to repurchase.
6. Vasquez resisted this action for redemption on the premise that Exh. E is just an
2. The record, however, does not show that the private respondents accepted the

3 4
ART. 1479. A promise to buy and sell a determinate thing for a price certain is reciprocally demandable. ART. 1324. When the offeror has allowed the offeree a certain period to accept, the offer may be
An accepted unilateral promise to buy or to sell a determinate thing for a price certain is binding upon the withdrawn at any time before acceptance by communicating such withdrawal, except when the
promisor if the promise is supported by a consideration distinct from the price. option is founded upon a consideration, as something paid or promised.
"Right to Repurchase" the land in question. We disagree with the appellate court's
finding that the private respondents accepted the "right to repurchase" under the Note:
following circumstances: as evidenced by the annotation and registration of the "RIGHT TO REPURCHASE" (Exhibit E) provides:
same on the back of the transfer of certificate of title in the name of appellants.
This shows that it was signed by appellant himself and witnessed by his wife so "RIGHT TO REPURCHASE
that for all intents and purposes the Vasquez spouses are estopped from KNOW ALL MEN BY THESE PRESENTS:
disregarding its obvious purpose and intention. I, CIPRIANO VASQUEZ, . . ., do hereby grant the spouses Martin Vallejera and
3. The annotation and registration of the right to repurchase at the back of the Apolonia Olea, their heirs and assigns, the right to repurchase said Lot No. 1860 for
certificate of title of the petitioners can not be considered as acceptance of the the sum of TWELVE THOUSAND PESOS (P12,000.00), Philippine Currency, within
right to repurchase. Annotation at the back of the certificate of title of registered the period TEN (10) YEARS from the agricultural year 1969-1970 when my contract
land is for the purpose of binding purchasers of such registered land. of lease over the property shall expire and until the agricultural year 1979-1980.
4. In effect, the annotation of the right to repurchase found at the back of the IN WITNESS WHEREOF, I have hereunto signed my name at Binalbagan, Negros
certificate of title over the subject parcel of land of the private respondents only Occidental, this 21st day of September, 1964.
served as notice of the existence of such unilateral promise of the petitioners to SGD. CIPRIANO VASQUEZ
SGD. VALERIANA G. VASQUEZ SGD.
resell the same to the private respondents. This, however, can not be equated with FRANCISCO SANICAS" (Rollo, p. 47).
acceptance of such right to repurchase by the private respondent.
5. Neither can the signature of the petitioners in the document called "right to
repurchase" signify acceptance of the right to repurchase. The respondents did
not sign the offer. Acceptance should be made by the promisee, in this case,
the private respondents and not the promisors, the petitioners herein. It
would be absurd to require the promisor of an option to buy to accept his own
offer instead of the promisee to whom the option to buy is given.
6. Furthermore, the actions of the private respondents — (a) filing a complaint to
compel resale and their demands for resale prior to filing of the complaint cannot
be considered acceptance.
7. Zulueta case (same situation as this case, right to repurchase executed separately
from the Deed of Sale on the same day):
a. The right of repurchase is not a right granted to the vendor by the vendee in
a subsequent instrument, but is a right reserved by the vendor in the same
instrument of sale as one of the stipulations of the contract.
b. Once the instrument of absolute sale is executed, the vendor can no longer
reserve the right to repurchase, and any right thereafter granted the vendor by
the vendee in a separate instrument cannot be a right of repurchase but some
other right like option to buy.
c. 'Conventional redemption shall take place when the vendor reserves the right
to repurchase the thing sold, with the obligation to comply with the provision
of Article 1616 and other stipulations which may have been agreed upon.'
d. It is clear from the decision that the ruling in the Zulueta case was based
mainly on the finding that the transaction between the parties was not a sale
with right to repurchase and that the option to repurchase was but an option
to buy or a mere promise on the part of Octaviano to resell the property to
Zulueta.
8. In the instant case, since the transaction between the petitioners and private
respondents was not a sale with right to repurchase, the private respondents
cannot avail of Article 1601 of the Civil Code which provides for conventional
redemption.
Nietes vs CA (Mel) 1. That the term will be for a period of 5 years
2. That the price of the rent is 5,000 per year payable in the following manners:
August 18, 1972| Concepcion, C. J. | Exercise of Option Contract
a. That the amount of P5,500 will be paid upon the execution of this Contract of Lease;
b. That the amount of P4,500 is payable on or before the 30th day of October, 1959;
PETITIONER: Aquilino Nietes c. That the remaining balance of P15,000 will be paid on or before March 30, 1960;
RESPONDENTS: Hon. Court of Appeals & Dr. Pablo C. Garcia 3. That all improvements made during the lease by the LESSEE will be owned by the LESSOR after the
expiration of the term of this Contract of Lease;
SUMMARY: Nietes lease the Angeles Educational Institute from Dr. Garcia. Their 4. That the LESSOR agrees to give the LESSEE an option to buy the land and the school building,
for a price of P100,000 within the period of the Contract of Lease;
contract was a “Contract of Lease With Option to Buy”. It stipulated that the unused
5. That should the LESSEE buy the lot, land and the school building within the stipulated period, the
payment will be applied to the purchase price of the school. Nietes paid Garcia certain unused payment for the Contract of Lease will be considered as part payment for the sale of the land and
sums in excess of the rent, which Garcia acknowledged. Later on, Garcia, through counsel, school;
wrote Nietes informing him of his decision to rescind the contract due to certain violations 6. That an inventory of all properties in the school will be made on March 31, 1960;

of the contract—such as poor maintenance, lack of inventory of school equipment, and the 6A. That the term of this Contract will commence in June 1960 and will terminate in June 1965;
use of another name for the said school. Nietes replied by informing Garcia that he decided 7. That the LESSEE will be given full control and responsibilities over all the properties of the school and
to exercise his Option to Buy. Nietes deposited the balance of the price to Agro-Industrial over all the supervisions and administrations of the school;
Bank, but he later withdrew the said amounts. Garcia refused to sell so Nietes commenced 8. That the LESSEE agrees to help the LESSOR to collect the back accounts of students incurred before
the present action for specific performance of Dr. Garcia's alleged obligation. The trial the execution of this contract.
court ruled in favor of Nietes which the CA affirmed. On motion for reconsideration by
Dr. Garcia, the special division of CA ruled in favor of Garcia stating that the full purchase 2. Instead of paying Dr. Garcia in the manner set forth in the contract,
price must be paid before the Option to Buy may be exercised. The issue is WON actual Nietes had made payments as follows:
payment is needed before one may exercise the option to buy. The SC ruled that this was October 6,1960 ....................................... P18,957.00
not needed. There is nothing in the contract that required Nietes to pay the full price before November 23, 1960 ................................. 300.00
December 21, 1960 ................................. 200.00
he could exercise the option. It was sufficient that he informed Garcia of his choice and
January 14, 1961 ..................................... 500.00
that he was at that time ready to pay. The exercise of the option need not be coupled with February 16, 1961 ................................... 3,000.00
actual payment so long as such payment is made upon the fulfillment of the owner’s March 12, 1961 ....................................... 1,000.00
undertaking to deliver the property. This is based on the principle that such option contracts March 13, 1961 ....................................... 700.00
involve reciprocal obligations—and one does not incur delay if the other party fails or August 4, 1961 ........................................ 100.00
refuses to comply with his respective obligation. That being the case, there was no need TOTAL ..................................... P24,757.00
for Nietes to deposit the said amounts—and his withdrawal thereof does not affect his 3. Nietes maintains that, on September 4, 1961, and December 13, 1962, he paid
right. Garcia the additional sums of P3,000 and P2,200, respectively, for which
Garcia issued receipts
DOCTRINE: In the case of an option to buy, the creditor may validly and effectively 4. Dr. Garcia's counsel wrote to Nietes a letter informing Nietes that Dr. Garcia
exercise his right by merely advising the debtor of the former's decision to buy and
will rescind the contract:
expressing his readiness to pay the stipulated price, provided that the same is available and
actually delivered to the debtor upon execution and delivery by him of the corresponding
deed of sale. Unless and until the debtor shall have done this the creditor is not and cannot Sir: I regret to inform you that our client, Dr. Pablo Garcia, desires to rescind your contract,
be in default in the discharge of his obligation to pay. In other words, notice of the 1959 because of the following:
creditor's decision to exercise his option to buy need not be coupled with actual 1. That you had not maintained the building, subject of the lease contract in good condition.
payment of the price, so long as this is delivered to the owner of the property upon 2. That you had not been using the original name of the school — Angeles Institute, thereby
performance of his part of the agreement. extinguishing its existence in the eyes of the public and injuring its prestige.
3. That through your fault, no inventory has been made of all properties of the school.
4. That up to this time, you had not collected or much less helped in the collection of back
accounts of former students.
FACTS:
1. Petitioner Aquilino Nietes and Respondent Dr. Pablo C. Garcia entered into This is to remind you that the foregoing obligations had been one, if not, the principal moving
a “Contract of Lease with Option to Buy” pursuant to the terms and factors which had induced the lessor in agreeing with the terms embodied in your contract of
conditions set forth in the deed namely: lease, without which fulfillment, said contract could not have come into existence. It is not
simply one of those reminders that we make mention, that our client under the circumstances,
That the LESSOR (Dr. Garcia) is an owner of the ANGELES EDUCATIONAL INSTITUTE is not only entitled to a rescission of the contract. He is likewise entitled to damages — actual,
situated at Angeles, Pampanga, a school which is duly recognized by the Government; compensatory and exemplary…”
That the lessor agrees to lease the above stated school to the LESSEE (Nietes) under the
following terms and conditions: 5. Counsel for Nietes replied that Nietes did not violate any provision of the
Contract of Lease with Option to Buy and said that for this reason, there was did "not know" whether the signatures on the receipt for P3,000 and P2,200
no basis for the rescission of the contract nor of the demands contained in the were his, and even said that he was "doubtful" about it.
letter. The letter also stated that “Mr. AQUILINO T. NIETES will exercise
his OPTION to buy the land and building subject matter of the lease and à SC: This testimony is manifestly incredible, for a man of his intelligence — a
that my said client is ready to pay the balance of the purchase price in Doctor of Medicine and the owner of an educational institution — could not possibly
accordance with the contract.” "not know" or entertain doubts as to whether or not the aforementioned signatures are
6. Nietes deposited with the branch office of the Agro-Industrial Bank in his and the payments therein acknowledged had been received by him. Also, the
Angeles City checks amounting to P84,860.50, as balance of the purchase payment of said sums of P3,000 and P2,200 was admitted by Dr. Garcia in his answer.
price of the property, but he withdrew said sum, after the checks had been
cleared. 2. The SC does not agree with the view of the CA that full purchase price
7. Nietes commenced the present action for specific performance of Dr. Garcia's must be paid before the option to buy be exercised. Neither the tenor of
alleged obligation. the contract nor the behaviour of Dr. Garcia justifies such view. The contract
8. Trial Court ruled in favor of Nietes and ordered Dr. Garcia to execute the does not say that Nietes had to pay the stipulated price of P100,000 before
Deed of Absolute Sale of property. exercising his option to buy the property in question. Accordingly, said
9. Both parties appealed. Dr. Garcia insofar as the trial court had neither option is governed by the general principles on obligations, pursuant to
dismissed the complaint nor upheld his counterclaim and failed to order which: In reciprocal obligations, neither party incurs in delay if the other
Nietes to vacate the property in question, and Nietes insofar as the trial court does not comply or is not ready to comply in a proper manner with what is
had granted him no more than nominal damages in the sum of P1,000, as incumbent upon him. From the moment one of the parties fulfills his
attorney's fees. obligation, delay by the other begins.
10. CA affirmed the trial court’s ruling except as regards the attorney’s fees 3. In the case of an option to buy, the creditor may validly and effectively
which were eliminated. exercise his right by merely advising the debtor of the former's decision
11. On motion for reconsideration of Dr. Garcia, the special division of the to buy and expressing his readiness to pay the stipulated price, provided
CA reversed the decision of the trial court. The said decision was mainly that the same is available and actually delivered to the debtor upon execution
predicated upon the theory that, under the contract between the parties, "the and delivery by him of the corresponding deed of sale. Unless and until the
full purchase price must be paid before the option counsel be exercised,"; debtor shall have done this the creditor is not and cannot be in default in the
that the checks for P84,860.50 deposited by Nietes with the Agro-Industrial discharge of his obligation to pay.
Development Bank, did not constitute a proper tender of payment, which was 4. Notice of the creditor's decision to exercise his option to buy need not be
"made beyond the stipulated 5-year period"; that such deposit "was not coupled with actual payment of the price, so long as this is delivered to
seriously made, because the same was withdrawn from the Bank and the owner of the property upon performance of his part of the
ostensibly remains in the lessee's hand"; and that "the fact that such agreement.
deposit was made by the lessee shows that he himself believed that he 5. Nietes need not have deposited with the Agro-Industrial Bank checks
should have paid the entire amount of the purchase price before he could amounting altogether to P84,860.50 and the withdrawal thereof soon
avail of the option to buy, otherwise, the deposit was a senseless gesture after does not and cannot affect his cause of action in the present case. In
... ." making such deposit, he may have had the intent to show his ability to pay
the balance of the sum due to Dr. Garcia as the sale price of his property. In
ISSUE: short, said deposit and its subsequent withdrawal cannot affect the result
WON actual payment is needed before one may exercise the option to buy? – NO of the present case.
6. Nietes was entitled to exercise his option to buy "within the period of the
RULING: Thus modified, the decision of the Court of First Instance of Pampanga is Contract of Lease," which commenced "in June 1960" and was to "terminate
hereby affirmed in all other respects, and that of the Court of Appeals reversed, with in June 1965." As early as September 4, 1961, or well "within the period
costs against respondent herein, Dr. Pablo C. Garcia. It is so ordered. of the Contract of Lease," Nietes had paid Dr. Garcia.
7. Though it is true that Nietes was bound, under the contract to pay certain
RATIO: amounts on specified dates and whereas he did not follow such dates, the
1. Dr. Garcia maintained "that the sums paid" to him "were part of the price of amounts received however was only P243.00 less than the P25,000 due as of
the contract of lease between the parties which were paid late and not within March 30, 1960, so that Nietes may be considered as having complied
the periods and/or schedules fixed by the contract. Dr. Garcia claimed that he substantially with the terms agreed upon. Indeed, Dr. Garcia seems to
have either agreed thereto or not considered that Nietes had thereby
violated the contract, because the letter of Dr. Garcia demanding
rescission of the contract, did not mention said acts or omissions of Nietes
among his alleged violations thereof enumerated in said communication.
In fact, Dr. Garcia issued the receipt stating that said payment had been made
"as per advance pay for the school, the Contract of Lease being paid" — in
other words, in accordance or conformity with said contract.
8. Besides, when Mrs. Nietes delivered the additional sum of P2,200, Dr. Garcia
issued a receipt accepting said amount "as the partial payment on the purchase
price of the property as specified on the original contract," thus further
indicating that the payment, in his opinion, conformed with said contract,
and that, accordingly, the same was in full force and effect.
9. In any event, it is undisputed that, as of September 4, 1961, Dr. Garcia had
received the total sum of P27,757, or P2,757 in excess of the P25,000
representing the rentals for the entire period of the lease, and over P21,200 in
excess of the rentals for the unexpired portion of the lease, from September
4, 1961 to June 1965. This circumstance indicates clearly that Nietes had,
on September 4, 1961, chosen to exercise and did exercise then his option
to buy. What is more, this is borne out by the receipt issued by Dr. Garcia
wherein the payment was referred as a "partial payment on the purchase of
the property as specified on the original contract of 'Contract of Lease with
the First Option to Buy'”
10. Further, confirmation is furnished by the letter of Nietes, also, within the
period of the lease — stating that he "will exercise his OPTION to buy the
land and building subject matter of the lease."
11. It is not correct to construe this expression as implying that the option had
not been or was not yet being exercised, or as a mere announcement of the
intent to avail of it at some future time. This interpretation takes said
expression out of the context of Nietes’ letter, which positively states, also,
that Nietes "is ready to pay the balance of the purchase price in accordance
with the contract," and requests counsel for Dr. Garcia to inform or advise
him "to make available the land title and execute the corresponding Deed of
Sale pursuant to this notice, and that if he fails to do so within fifteen (15)
days ... we shall take the corresponding action to enforce the agreement."
12. The provision in paragraph 5 of the Contract, to the effect that "should the
LESSEE" choose to make use of his option to buy "the unused payment for
the Contract of Lease will be considered as payment for the sale of the land
and school, " simply means that the rental paid for the unused portion of
the lease shall be applied to and deducted from the sale price of P100,000
to be paid by Nietes at the proper time — in other words, simultaneously
with the delivery to him of the corresponding deed of sale, duly executed
by Dr. Garcia.
CARCELLER v. CA (Paul) accelerate payments if the interest for the remaining installments
Feb. 10, ’99 | Quisumbing, J. | Contract of Lease with Option to Purchase will no longer be imposed
d. The option shall be exercised by a written notice to the lessor, at
anytime within the option period and the document of sale has to
PETITIONER: Jose Ramon Carceller be consummated within the month when the option is exercised.
RESPONDENTS: Courts of Appeals and State Investment Houses, Inc. 4. On January 7, 1986, 3 weeks before the expiration of the lease contract,
SIHI notified Carceller of the impending expiration of the lease contract aka
SUMMARY: Carceller and SIHI entered into a contract of lease with option to the period for exercising the option to purchase.
purchase. 3 weeks before the period of redemption expired, SIHI asked Carcerller 5. In a letter dated January 15, 1986, Carceller sent a letter requesting for a 6-
if he was going to exercised the option or not. A few days after, Carceller notified month extension of the lease contract, saying that he needs ample time to
SIHI that he wanted to ask for a 6-month extension so that he can raise funds to raise funds to exercise the option. Carceller also said that he has made a
buy the property. 18 days after, BUT BEYOND THE PERIOD FOR substantial investment on the property, and had been paying the monthly
REDEMPTION, Carceller told SIHI that he was going to exercise the option, but rentals punctually.
SIHI already said that the period had elapsed and therefore, the option can no 6. On February 14, 1986, SIHI disapproved the request. Though, SIHI still
longer be availed of. Because of this, petitioner filed for specific specific offered that the lease be extended for Php 30,000 a month for 1 year. SIHI
performance against SIHI. also told petitioner about its intention to lease property to the public.
7. On February 18, 1986, Carceller notified SIHI with his decision to exercise
The SC holds that Carceller’s notification to SIHI (the one on Feb 18, 1986), was
the option to purchase the property and made the downpayment of Php
fair notice of petitioner’s intent to exercise the option, despite the request for the
360,000 as stipulated in the contract.
extension of the lease contract. As Carcerller said, he only asked the extension so 8. SIHI told petitioner that the period to exercise the option has lapsed so
he can raised the funds needed to purchase the property. These are acts consistent
respondent ordered petitioner to vacate the property and to pay the remaining
with the parties’ primary intent - Carceller wanted to buy, and SIHI wanted
rentals and penalty.
to sell from the very start.
9. Because of this, Carcerller filed a complaint for specific performance and
damages to compel SIHI to execute the deed of sale.
DOCTRINE: An option is a preparatory contract wherein one party grants to 10. RTC: ruled in favor of Carceller, making the consummation of the contract
another, for a fixed period, and under specified conditions, the power to decide, Jan. 1985, the basis of the purchase price - Php 1,800,000.
whether or not to enter into a principal contract. It binds the party who has given 11. CA: Affirmed the RTC but modified the basis of the purchase price - the
the option (SIHI, in this situation), not to enter into the principal contract with any market price at 1995, the date when the decision was rendered
other person during the period, and to enter into such contract with the one whom 12. Both parties raised it to the SC.
the option was granted (Carceller, here). It is a separate, distinct agreement from
ISSUES:
the contract which the parties may enter into option the consummation of the
1. W/N Carceller should still be allowed to exercise the option to purchase
option.
the property, even if there was delay on his part in giving notice to SIHI?
FACTS: YES
1. Respondent State Investment Houses, Inc. (SIHI), is the owner of 2 lands in 2. W/N the market price that should be used is the prevailing market price aka.
Bulacao, Cebu City, covered by TCT’s by the Registry of Deeds of Cebu. at 1995 instead of 1985? NO, market price during the consummation of the
2. On January 10, 1985, petitioner Carceller entered into a LEASE contract should be used.
CONTRACT WITH OPTION TO PURCHASE over the parcels, with a
rental fee of Php 10,000 for 18 months. RULING: The appealed decision of respondent court, insofar as it affirms the
3. Some of the pertinent portions of the contract are as follows: judgment of the trial court in granting petitioner the opportunity to exercise the option
a. Exclusive right, and option to purchase, within the lease period, is to purchase the subject property, is hereby AFFIRMED. However the purchase price
given to the LESSEE (Carceller) for Php 1,800,000. should be based on the fair market value of real property in Bulacao, Cebu City, as of
b. Upon signing the Deed of Sale, lessee shall immediately pay Php February 1986, when the contract would have been consummated.
360,000. RATIO:
c. The balance of Php 1,440,000 shall be paid over 60 consecutive
months with 24% per annum on the balance; and the lessee shall Carceller should still be allowed to exercise the option to purchase despite him
complying after the said lease period 1986 should be used.
1. An option is a preparatory contract wherein one party grants to another, for
a fixed period, and under specified conditions, the power to decide, whether or not to
enter into a principal contract. It binds the party who has given the option (SIHI, in
this situation), not to enter into the principal contract with any other person during the
period, and to enter into such contract with the one whom the option was granted
(Carceller, here). It is a separate, distinct agreement from the contract which the
parties may enter into option the consummation of the option.
2. The SC holds that Carceller’s notification to SIHI (the one on Feb 18, 1986),
was fair notice of petitioner’s intent to exercise the option, despite the request for the
extension of the lease contract.
3. As Carcerller said, he only asked the extension so he can raised the funds
needed to purchase the property. These are acts consistent with the parties’ primary
intent, which will be discussed later on.
4. Carcerller acted in good faith, introduced substantial improvements on the
property, and therefore, he should be allowed to exercise the option to purchase the
property. SIHI will not even be prejudiced with the decision.

Primary Intent
1. Here, the primary intent, was in accordance with CA’s ruling.
2. Oblicon throwback: Construction should not be limited to the words in the contract,
but the paramount consideration is the parties’ intent. Their agreement has the force
of law between them, and therefore, the intent should always prevail. The provisions
should also be construed together, along with the contemporaneous, and subsequent
acts with regard to the execution of the contract.
3. Context: SIHI was already taken over by the Central Bank because it had financial
problems. The company was desperate to dispose its assets to improve its situation.
Among these assets, is the land in dispute here.
4. According to the stipulations, the lease period should have been exercised by
January 30, 1986. What happened instead, was that Carcerller asked for a 6-month
extension for him to be able to raise funds. He then exercised this option, on
February 14, 1986.
5. There was a delay of 18 days but this was not a substantial breach that would
defeat the intention of the parties. As seen, with the circumstances, petitioner and
defendant clearly had the intention to buy and sell the property, respectively.
6. Actions showing intent:
a. SIHI: Wanted to improve its financial standing even if it will only
materialize within years. Kept on reminding Carceller to send notice if he
will exercise the option.
b. Carceller: Went to a company named TRC to loan Php 8,000,000 to pay
the purchase price in one single payment, instead of paying in installments,
because he wanted to avoid paying the 24% interest.

Market price during consummation of the contract


1. It would be a great injustice to allow SIHI to unjustly enrich itself by earning 90%
more than the original price. For equity considerations, the market price back in
ANG YU ASUNCION v. CA (Eliel) Unjieng to sell the property to them.
December 2, 1994 | Vitug, J. | Right of First Refusal 6. The RTC found that Cu Unjieng’s offer to sell was never accepted by
Asuncion for the reason that the parties did not agree upon the terms and
PETITIONER: Ang Yu Asuncion, Arthur Go and Keh Tiong conditions of the proposed sale, hence, there was no contract of sale at all.
RESPONDENTS: CA and Buen Realty Development Corporation 7. Nonetheless, the lower court ruled that should Cu Unjieng subsequently offer
their property for sale at a price of P11M or below, Asuncion will have the
SUMMARY: Asuncion filed a complaint for specific performance against Cu right of first refusal.
Unjieng for the latter to sell the property that Asuncion were leasing. The lower 8. During the appeal of the decision in the CA, Cu Unjieng executed a Deed of
court ruled, however, that there was no meeting of the minds as to the sale Sale transferring the property in question to Buen Realty and Development
between Cu Unjieng and Asuncion. Nonetheless, the court granted to Asuncion Corporation for the consideration of P15M.
the right of first refusal. During the appeal of the lower courts decision by 9. Buen Realty, as the new owner of the subject property, demanded Asuncion
Asuncion, Cu Unjieng sold and transferred to Buen Realty the said property. This to vacate the premises. Asuncion then filed a writ of execution, which was
prompted Asuncion to file a writ of execution to issue a deed of sale to Asuncion granted by the court.
for the same consideration of P15M. Buen Realty appelead the decision, and the 10. On appeal of the decision by Buen Realty, the court set aside ande decalred
CA set aside such writ. Hence this petition. The issue before the SC was whether without force and effect the writ of execution. Hence this petition.
or not the writ of execution was a valid remedy, since Asuncion had the right of
first refusal. The SC ruled that the writ of execution will fail since there is nothing ISSUES:
to execute. The right of first refusal, is independent from the contract of sale. At 1. WoN the writ of execution is valid – NO
best, it is merely a preparatory juridical relation between the parties. It is
dependent on the eventual agreement of the terms of the sale, in consistency with RULING: WHEREFORE, we UPHOLD the Court of Appeals in ultimately setting
the element of consensuality of contracts. Asuncion’s remedy, therefore, is not aside the questioned Orders, dated 30 August 1991 and 27 September 1991, of the
specific performance for the writ of execution, rather an action for recovery of court a quo. Costs against petitioners.
damages under Art 19 fo the Civil Code.
RATIO:
DOCTRINE: In the law on sales, the so-called “right of first refusal” is an 1. A contract undergoes various stages that include its negotiation or
innovative juridical relation. Needless to point out, it cannot be deemed a preparation, its perfection and, finally, its consummation.
perfected contract of sale under Art 1458 of the Civil Code. 2. Negotiation covers the period from the time the prospective contracting
parties indicate interest in the contract to the time the contract is concluded.
FACTS: The perfection of the contract takes place upon the concurrence of the
1. A Second Amended Complaint for Specific Performance was filed by Ang essential elements thereof. The stage of consummation begins when the
Yu Asuncion and Keh Tiong, et. al (Asuncion) against Bobby Cu Unjieng, parties perform their respective undertakings under the contract culminating
Rose Cu Unjieng and Jose Tan (Cu Unjieng) before the RTC. in the extinguishment thereof.
2. Asuncion are tenants or lessees of residential and commercial spaces owned 3. Until the contract is perfected, it cannot, as an independent source of
by Cu Unjieng in Ongpin Street, Binondo, Manila. That they have occupied obligation, serve as a binding juridical relation. In sales, particularly, to
said spaces since 1935 and have been religiously paying the rental and which the topic for discussion about the case at bench belongs, the
complying with all the conditions of the lease contract contract is perfected when a person, called the seller, obligates himself,
3. Cu Unjieng informed Asuncion that they are offering to sell the premises and for a price certain, to deliver and to transfer ownership of a thing or right
are giving them priority to acquire the same; that during the negotations, Cu to another, called the buyer, over which the latter agrees.
Unjieng offered a price of P6M while Asuncion made a counter offer of P5M 4. Art. 1458. By the contract of sale, one of the contracting parties obligates
4. Asuncion thereafter asked Cu Unieng to put their offer in writing to which himself to transfer the ownership of and to deliver a determinate thing,
request Cu Unjieng acceded. Asuncion asked that they specify the terms and and the other to pay therefor a price certain in money or its equivalent.
ocnditions o the offer to sell; that when Asuncion did not receive any reply, A contract of sale may be absolute or conditional.
they sent another letter with the same request 5. When the sale is not absolute but conditions, such as in a “Contract to Sell”
5. Since Cu Unjieng failed to specify the terms and conditions of the offer to where invariably the ownership of the thing sold is retained until the
sell and because of information received that Cu Unjieng were about to sell fulfillment of a positive suspensive condition, the breach of the condition will
the property, Asuncion were compelled to file the complaint to compel Cu prevent the obligation to convey title from acquiring an obligatory force.
6. An unconditional mutual promise to buy and sell, as long as the object is the consideration for the main contract with a right of withdrawal on
made determinate and the price is fixed, can be obligatory on the parties, and the part of the optionee, the main contract could be deemed
compliance therewith may accordingly be exacted. perfected; a similar instance would be an “earnest money” in a
7. An accepted unilateral promise which specifies the thing to be sold and the contract of sale that can evidence is perfection.
price to be paid, when coupled with a valuable consideration distinct and 12. In the law on sales, the so-called “right of first refusal” is an innovative
separate from the price, is what may properly be termed a perfected contract juridical relation. Needless to point out, it cannot be deemed a perfected
of option. contract of sale under Art 1458 of the Civil Code.
8. Observe, however, that the option is not the contract of sale itself. The 13. An option or an offer would require, among other things, a clear certainty on
optionee has the right, but not the obligation to buy. Once the option is both the object and the cause or consideration of the envisioned contract. In
exercised timely, a bilateral promise to sell and to buy ensues and both parties a right of first refusal, while the object might be made determinate, the
are then reciprocally bound to comply with their respective undertakings. exercise of the right, however, would be dependent not only on the
9. A negotiation is formally initiated by an offer. An imperfect promise is grantor’s eventual intention to enter into a binding juridical relation
merely an offer. Public advertisements or solicitations and the like are with another but also on terms, including the price, that obviously are
ordinarily construed as mere invitations to make offers or only as proposals. yet to be later firmed up.
10. These relations, until a contract is perfected, are not considered binding 14. Prior thereto, it can at best be so described as merely belonging to a class
commitments. Thus, at any time prior to the perfection of the contract, either of preparatory juridical relations governed not by contracts but by,
negotiating party may stop the negotiation. The offer, at this stage, may be among other laws of general application, the pertinent scattered
withdrawn; the withdrawal is effective immediately after its manifestation, provision of the Civil Code on human conduct.
such as by its mailing and not necessarily when the offeree learns of the 15. Even on the premise that such right of first refusal has been decreed
withdrawal. under a final judgment, like here, its breach cannot justify
11. Where a period is given to the offeree within which to accept the offer, the correspondingly an issuance of a writ of execution under a judgment that
following rules generally govern: merely recognizes its existence, nor would it sanction an action for
a. If the period is not itself founded upon or supported by a specific performance without thereby negating the indispensible element
consideration, the offeror is still free and has the right to withdraw of consensuality in the perfection of contracts.
the offer before its acceptance, or, if an acceptance has been made, 16. It is not to say, however, that the right of first refusal would be
before the offeror’s coming to know of such fact, by communicating inconsequential for, such as already intimate above, an unjustified disregard
that withdrawal to the offeree. thereof, given, for instance, the circumstances expressed in Article 19 of the
b. The right to withdraw, however, must not be exercised whimsically Civil Code, can warrant a recovery for damages.
or arbitrarily; otherwise, it could give rise to a damage claim under 17. The final judgment of the lower court has merely accorded a “right of
Art. 19 of the Civil Code which ordains that “every person must, in first refusal” in favor of Asuncion. The consequence of such a declaration
the exercise of his rights and in the performance of his duties, act entails no more than what has heretofore been said. In fine, if, as it is
with justice, give everyone his due, and observe honesty and good here so conveyed to us, Asuncion are aggrieved by the failure of Cu
faith.” Unjieng to honor the right of first refusal, the remedy is not a writ of
c. If the period has a separate consideration, a contract of “option” execution on the judgment, since there is none to execute, but an action
is deemed perfected, and it would be a breach of that contract for damages in a proper forum for the purpose.
to withdraw the offer during the agreed period. The option,
however, is an independent contract by itself, and it is to be
distinguished from the projected main agreement which is
obviously yet ot be concluded.
d. If, in fact, the optioner-offeror withdraws the offer before its
acceptance by the optionee-offeree, the latter may not sue for
specific performance on the proposed contract, since it has failed to
reach its own stage of perfection. The optioner-offeror, however,
renders himself liable for damages for breach of the option.
e. In these cases, care should be taken of the real nature of the
consideration given, for if, in fact, it has been intended to be part of
10 EQUATORIAL REALTY DEVT, INC. v MAYFAIR THEATER, INC. other than the LESSEE, the lessor is bound and obligated, as it
(CHIQUI) hereby binds and obligates itself, to stipulate in the Deed of Sale
November 21, 1996 | Hermosisima, J. | Right of First Refusal thereof that the purchaser shall recognize this lease and be
bound by all the terms and conditions thereof.’
PETITIONER: Equatorial Realty Development, Inc. & Carmelo & Bauermann, 5. In August 1974, Mr. Pascal of Carmelo informed Mr. Yang, president of
Inc. Mayfair, that Carmelo wanted to sell the whole property. There was a certain
RESPONDENTS: Mayfair Theater, Inc. Jose Araneta who wanted to buy for $1.2 M so Mr. Pascal asked Mr. Yang if
the latter wanted to buy the property for 6-7 M pesos.
SUMMARY: Mayfair leased a portion of the second floor of Carmelo’s building 6. Mr. Yang reminded Mr. Pascal of their contract and Carmelo did not reply
for a term of 20 years. Mayfair would use the leased properties as Miramar and any further. In September, Mayfair expressed its intent to buy only the leased
Maxim Theatre. (Please check the wording of the contracts in Facts 4a and 4b). premises for a reasonable price.
Carmelo informed Mayfair that they would dispose of the whole property but 7. In July 1978, Carmelo sold the entire property to Equatorial which included
Mayfair only wanted to buy the leased portion. Carmelo was never heard from the leased premises. Thus, Mayfair instituted the action a quo for specific
again. Carmelo then sold the entire property to Equatorial which led to the performance and annulment of the sale of the leased premises.
institution of specific performance and annulment of sale by Mayfair. RTC ruled 8. Carmelo and Equatorial’s side:
in favor of Carmelo and Equatorial because the contract was stated to be an option a. That it had informed Mayfair of its desire to sell the land and offered
contract since it lacks consideration. Meanwhile, the CA decided in favor of the same to Mayfair, and that the latter only expressed their intent
Mayfair asserting that the contract was a right of first refusal. W/N the contract on the leased premises which was impossible due to its structure
provides for a right of refusal? YES and it is also RESCISSIBLE. According to b. That the option to purchase was null and void for lack of
the SC, an option contract must be supported by consideration. Par. 8 was consideration.
incorporated for the benefit of Mayfair and that the right of first refusal is an 9. RTC decision: In favor of Carmelo and Equatorial
integral part of the contracts of lease. The consideration is built into the reciprocal a. Par. 8 is an option clause which is not binding because of the lack
obligations of the parties. The contract is also rescissible because of the bad faith of distinct consideration
shown by Carmelo and Equatorial. 10. CA decision: In favor of Mayfair
a. Since par. 8 does not state a fixed price for the purchase of the leased
DOCTRINE: Right of first refusal contained in a Contract of Lease, when premises, which is an essential element for a contract of sale to be
breached by a promissor allows enforcement by the promissee by way of perfected, then par. 8 must be a right of first refusal and not an option
rescission of the sale entered into with the third party. contract.
b. The 30-day option to purchase cannot be legally categorized as an
option. It is valid and binding.
FACTS:
1. Carmelo owned a parcel of land, together with two 2-storey buildings
ISSUE/s:
constructed along C.M. Recto Avenue and these are issued in its name by the
17. W/N the contractual stipulations provides for a right of first refusal? YES
Register of Deeds of Manila
18. W/N the sale to Equatorial is valid - NO
2. On June 1, 1967, Carmelo entered into a contract with Mayfair for the latter’s
lease (a portion of the second floor of the two-storey building) of Carmelo’s
RULING: WHEREFORE, the petition for review of the decision of the Court of
property. Mayfair would use it as a motion picture theater for a term of 20
Appeals, dated June 23, 1992, in CA-G.R. CV No. 32918, is HEREBY DENIED.
years. Mayfair then constructed on the leased property a movie house known
The Deed of Absolute Sale between petitioners Equatorial Realty Development, Inc.
as Maxim Theatre.
and Carmelo & Bauermann, Inc. is hereby deemed rescinded; petitioner Carmelo &
3. On March 31, 1969, Mayfair entered into a second contract of lease of another
Bauermann is ordered to return to petitioner Equatorial Realty Development the
portion of the second floor for a similar use and for a similar term of 20 years.
purchase price. The latter is directed to execute the deeds and documents necessary
Mayfair put up another movie house called Miramar Theatre.
to return ownership to Carmelo & Bauermann of the disputed lots. Carmelo &
4. Both contracts are worded in Par. 8 as:
Bauermann is ordered to allow Mayfair Theater, Inc. to buy the aforesaid lots for
a. 'That if the LESSOR should desire to sell the leased premises, the
P11,300,000.00.
LESSEE shall be given 30-days exclusive option to purchase the
same. RATIO:
b. In the event, however, that the leased premises is sold to someone 1. The rule so early established in this jurisdiction is that the deed of option or
option clause in a contract, in order to be valid and enforceable, must, not applicable to third persons who acquired possession lawfully
among other things, indicate the definite price at which the person and in good faith.
granting the option, is willing to sell. 3. On the impossibility of the performance because the entire property is
a. In all the cases which discussed option contracts, the selling price of indivisible, common sense and fairness dictate that instead of nullifying the
the object is always predetermined and specified in the option clause agreement on that basis, the stipulation should be given effect by including
in the contract or in the separate deed of option. the indivisible appurtenance in the sale of the dominant portion under the
b. CA correctly ruled that par. 8 grants the right of first refusal to right of first refusal.
Mayfair and is not an option contract. An option is a contract a. The right of first refusal should include not only the property
granting a privilege to buy or sell within an agreed time and at specified in the contracts but also the appurtenant portions sold to
a determined price. It is a separate and distinct contract from Equatorial which are claimed by Carmelo to be indivisible.
that which the parties may enter into upon the consummation b. Even as it recognizes the right of first refusal, this Court should
of the option. It must be supported by consideration. In the also order that Mayfair be authorized to exercise its right of first
instant case, the right of first refusal is an integral part of the refusal under the contract to include the entirety of the
contracts of lease. The consideration is built into the reciprocal indivisible property. The boundaries of the property sold should
obligations of the parties. be the boundaries of the offer under the right of first refusal.
c. CA is correct in stating that par. 8 was incorporated into the c. Since Mayfair has a right of first refusal, it can exercise the right
contracts of lease for the benefit of Mayfair which wanted to be only if the fraudulent sale is first set aside or rescinded. All of these
assured that it shall be given the first crack or the first option to matters are now before us and so there should be no piecemeal
buy the property at the price which Carmelo is willing to accept. determination of this case and leave festering sores to deteriorate
d. It is also wrong to say that there is no consideration in an agreement into endless litigation. The facts of the case and considerations of
of right of first refusal. The stipulation is part and parcel of the justice and equity require that we order rescission here and now.
entire contract of lease. The consideration for the lease includes Rescission is a relief allowed for the protection of one of the
the consideration for the right of first refusal. Thus, Mayfair is in contracting parties and even third persons from all injury and
effect stating that it consents to lease the premises and to pay the damage the contract may cause or to protect some incompatible and
price agreed upon provided the lessor also consents that, should it preferred right by the contract. The sale of the subject real property
sell the leased property, then, Mayfair shall be given the right to by Carmelo to Equatorial should now be rescinded considering that
match the offered purchase price and to buy the property at that Mayfair, which had substantial interest over the subject property,
price. was prejudiced by the sale of the subject property to Equatorial
2. For the consequential rights, obligations and liabilities of Carmelo, Mayfair, without Carmelo conferring to Mayfair every opportunity to
and Equatorial negotiate within the 30-day stipulated period.
a. Carmelo and Equatorial acted in bad faith when they rendered par. 4. Under the Ang Yu Asuncion vs. Court of Appeals decision, the Court stated
8 as “inutile”. It is undisputed that Carmelo recognized the right of that there was nothing to execute because a contract over the right of first
Mayfair when it informed the latter of its intention to sell the said refusal belongs to a class of preparatory juridical relations governed not by
property. However, the former violated such right when it the law on contracts but by the codal provisions on human relations. This may
abandoned negotiations, kept a low profile for some time, then sold, apply here if the contract is limited to the buying and selling of the real
without prior notice to Mayfair, the subject property. property. However, the obligation of Carmelo to first offer the property to
b. Equatorial is buyer in bad faith. It was aware of the lease contracts Mayfair is embodied in a contract. It is Paragraph 8 on the right of first refusal
between Mayfair and Carmelo when it had its lawyers study the which created the obligation. It should be enforced according to the law on
lease contracts. Hence, the sale to it of the property in question is contracts instead of the panoramic and indefinite rule on human relations.
rescissible. The latter remedy encourages multiplicity of suits.
c. Under Art. 1380 to 1381(3) of the Civil Code, a contract otherwise a. There is something to execute and that is for Carmelo to comply
valid may nevertheless be subsequently rescinded by reason of with its obligation to the property under the right of the first refusal
injury to third person, like creditors. It is a remedy by according to the terms at which they should have been offered then
contracting parties, even to third persons, to secure reparation to Mayfair, at the price when that offer should have been made.
for damages caused to them by a contract. But this remedy is Also, Mayfair has to accept the offer. This juridical relation is not
amorphous nor is it merely preparatory. Paragraph 8 of the two
leases can be executed according to their terms.
5. Panganiban, separate concurring:
a. With respect to the sale, Mayfair is not a party so it had no
personality to sue for its annulment. However, it falls under a case
of rescission in Art. 1177.
b. Specific performance or rescission with damages is a proper remedy
to enforce a right of first refusal because consent was established
due to a perfected contract which discussed such right.
6. Romero, concurring and dissenting:
a. The right of first refusal is unlike an option which requires a
certainty as to the object and consideration of the anticipated
contract. When the right of the first refusal is exercised, there is no
perfected contract of sale because the other terms of the sale have
yet to be determined.
b. While Article 1381 (5) itself says that rescission may be granted to
all other contracts specially declared by law to be subject to
rescission, there is nothing in the law that states that an offeree who
failed to exercise his right of refusal because of bad faith on the part
of the offeror may rescind the subsequent contract entered into by
the offeror and a third person. Hence, there is no legal justification
to rescind the contract between Carmelo and Bauermann, Inc. and
Equatorial Realty. (His opinion is that third persons should be
construed as only including wards, creditors, absentees, heirs and
others enumerated under the law who are prejudiced by the contract
sought to be rescinded.)
7. Vitug, dissenting:
a. A "breach" of the right of first refusal can only give rise to an action
for damages primarily under Article 19 of the Civil Code, as well as
its related provisions, but not to an action for specific performance
set out under Book IV of the Code on "Obligations and Contracts.
(Breach of right of first refusal can only give rise to damages, not to
an action for specific performance.)
b. The consent contemplated by the law is that which is manifested by
the meeting of the offer and of the acceptance upon the object and
the cause of the obligation. The offer must be certain and the
acceptance absolute. Thus, a right of first refusal cannot have the
effect of a contract because, by its very essence, certain basic terms
would have yet to be determined and fixed.
Parañaque Kings Enterprises Inc v. CA (Rica) informing Paranaque of the sale. Upon learning this, Paranaque’s
26 February1997| Panganiban, J| Price in Option to buy representative wrote a letter back to Santos, requesting her to rectify the error
and consequently realizing the error, Santos had the properties reconveyed to
PETITIONER: Parañaque Kings Enterprises Inc her for the same consideration. (Basically, Santos bought back the property
RESPONDENT: Court of Appeals, Catalina Santos from Raymundo).
SUMMARY: Santos owned eight parcels of land which was being leased to 7. Santos offered the property to Paranaque for Php15M but Paranaque refused.
Paranaque Kings. The deed of assignment included Paranaque’s right to first option to Instead, Paranaque offered to buy the property for Php5M.
purchase the property should Santos decide to sell it. This, however was violated by 8. A deed of sale was executed in favor of Raymundo for a consideration of
Santos. Santos sold the property to Raymundo for Php5M. After learning of her error, Php9M.
Santos repurchased the property and offered to sell it to Paranaque with a whopping 9. Paranaque demanded from Santos to rectify their unlawful acts that they
Php15M price. Paranaque refused to accept the offer because of the price. Santos then committed but Santos refused and failed to comply with Paranaque’s just and
violated the right again when she sold it to Raymundo for Php9M. Paranaque prays valid demands.
for the annulment of the deed of sale between Raymundo and Santos. The TC and the 10. Paranaque prays that the Deed of Sale between Santos and Raymundo be
CA ruled in favor of Santos. The SC reversed the decision saying that Paranaque’s annulled and the leased properties be sold to Paranaque in the amount of
right was violated because the price that was offered to Paranaque and the price offered Php5M, and damages.
to Raymundo were not the same. For Paranaque to have exercised its right, the 11. Instead of filing their respective answers, Santos filed a motion to dismiss
properties should have been sold in the same terms and conditions. In this case, it was anchored on the grounds of lack of cause of action, estoppel and laches.
not sold in the same terms and conditions. Therefore, violating Paranaque’s right.
DOCTRINE: The basis of the right of first refusal must be the current offer to sell ISSUE: W/N there is a lack of cause of action. – NO, may action.
of the seller or offer to purchase of any prospective buyer. Only after the optionee fails Related to topic: W/N the first option to buy was violated – Yes it was because the
to exercise its right of first priority under the same terms and within the period price offered to Paranaque was not proper.
contemplated, could the owner validly offer to sell the property to a third person under
the same terms as offered to the optionee. RULING: WHEREFORE, the petition is GRANTED. The assailed decisions of the
FACTS: trial court and the Court of Appeals are hereby REVERSED and SET ASIDE. The
1. Paranaque is a private corporation organized and existing under and by virtue case is remanded to the RTC of Makati for further proceedings.
of the laws of the Philippines. While Catalina Santos is of legal age and a
resident at Stockton, California, USA represented by her attorney-in-fact, RATIO:
Protacio. Santos is the one who owns the eight parcels of land in question. 1. Preliminary Issue: Failture to File Sufficient Copies of Brief – compliance
2. A certain Frederick Chua leased the land of Santos and the said lease was with the Rules was not intended to dely and did not result to prejudice Santos.
registered in the Register of Deeds. Because of the court’s equity jurisdiction, the appeal may be stayed.
3. Chua then assigned all his rights and interests and participation in the leased 2. A cause of action exists when:
property to Lee Ching Bing with the conformity of Santos. The assignment a. A right in favor of plaintiff by whatever means and under whatever
was registered. law it arises or is created (the right to first option to buy)
4. Bing then assigned all his rights and interests in the leased property to b. An obligation on the part of the named defendant to respect or not
Paranaque Kungs Enterprises by virtue of a deed of assignment with the to violate such right (Santos’ failure to sell the properties first to
conformity of Santos. The assignment was registered. Paranaque)
5. The deed of assignment included: c. An act or omission on the part of such defendant violative of the
a. That in case the properties subject of the lease agreement are sold or right of plaintiff or constituting a breach of the obligation
encumbered, Lessors shall impose as a condition that the buyer or 3. In this case, the first option or priority to purchase the leased properties was
mortgagee thereof shall recognize and be bound by all the terms and evident when the different past lessees of the property assigned their rights
conditions of this lease agreement and shall respect this Contract of and eventually ended with Paranaque. The assigned rights included the right
Lease as if they are the Lessors thereof and in case of sale, Lessee to first option or priority.
shall have the first option or priority to buy the properties subject 4. Santos first sold the properties to Raymundo and after realizing the her error,
of the lease she repurchased the properties and offered them to Paranaque. The price of
6. On September 21, 1988 Santos sold the eight parcels of land subject of the the property was, however, higher than that offered to Raymundo. Paranaque
lease to David Raymundo for Php 5M. Santos wrote a letter to Paranaque refused to buy the property because of the ridiculous price. Santos then again
violated the the right to first purchase when it sold the properties to
Raymundo without first offering them to Paranaque at the same price.
5. To have full compliance with the contractual right granting Paranaque the
first option to purchase, the sale of the properties for the amount of Php9M,
the price for which they were finally sold to Raymundo, should have likewise
been first offered to petitioner.
6. Guzman v Bonnevie was used by the court which is essentially the same. The
court in this case said that only if the Bonnevies failed to exercise their right
of first priority could Reynoso thereafter lawfully sell the subject property to
others, and only under the same terms and conditions previous offered to
the Bonnevies.
7. The basis of the right of first refusal must be the current offer to sell of the
seller or offer to purchase of any prospective buyer. Only after the optionee
fails to exercise its right of first priority under the same terms and within the
period contemplated, could the owner validly offer to sell the property to a
third person under the same terms as offered to the optionee.
ROSENCOR VS. INQUING (Nikki V) from Eufrocina de Leon offering to sell to them the property they were leasing for
J. Gonzaga-Reyes | G.R. No. 140479 | March 8, 2001 | Buyer in Good Faith P2,000,000.00.
5. The lessees offered to buy the property from de Leon for the amount of
Petitioners: Rosencor Development Corporation and Rene Joaquin P1,000,000.00. De Leon told them that she will be submitting the offer to the other
Respondents: Paterno Inquing, Irene Guillermo, Federico Bantugan, Fernando heirs. Since then, no answer was given by de Leon as to their offer to buy the property.
Magbanua, Lizza Tiangco. However, in November 1990, Rene Joaquin came to the leased premises introducing
himself as its new owner.
Summary: The lessees in this case were granted a right of first refusal by Sps. 6. In January 1991, the lessees again received another letter from Atty. Aguila
Tiangco. Upon the death of Sps. Tiangco, the management of the property was demanding that they vacate the premises. A month thereafter, the lessees received a
adjudicated to their heirs who were represented by De Leon. The heirs allegedly the letter from de Leon advising them that the heirs of the late spouses Tiangcos have
same right to the lessees as the former knew the such right was extended by Sps. already sold the property to Rosencor. The following month Atty. Aguila wrote them
Tiangco. However, the lessees received a letter from a certain Atty. Aguila another letter demanding the rental payment and introducing herself as counsel for
demanding them to leave the premises. The lessees refused to leave. Thereafter, Rosencor/Rene Joaquin, the new owners of the premises.
they received a letter from De Leon offering to sell to them the property for 2M. 7. The lessees requested from de Leon why she had disregarded the pre-emptive right
The lessees offered to buy the property for 1M but no answer was given by De Leon she and the late Tiangcos have promised them. The lessees offered to tender their rental
since then. Moreover, a certain Rene Joaquin came to the premises introducing payment to de Leon but she refused to accept the same.
himself as the new owner. It turns out that a sale between Rene Joaquin (VP of 8. In April 1992 before the demolition can be undertaken by the Building Official,
ROSENCOR) and De Leon took place in Sept. 1990 (a month before De Leon made Rosencor raised the issue as to the rental payment of the premises. It was also at this
the offer to the lessees). The issue in this case is WON a Contract of Sale entered instance that the lessees were furnished with a copy of the Deed of Sale and discovered
into in violation of a third party's right of first refusal be rescinded in order that such that they were deceived by de Leon since the sale between her and Rene
third party can exercise said right. The prevailing doctrine is that a contract of sale Joaquin/Rosencor took place in September 4, 1990 while de Leon made the offer to
entered into in violation of a right of first refusal of another person, while valid, is them only in October 1990 or after the sale with Rosencor had been consummated.
rescissible. The lessees also noted that the property was sold only for P726,000.00.
9. The lessees offered to reimburse de Leon the selling price of P726,000.00 plus an
DOCTRINE: In this case, the contract of sale cannot be rescinded because under additional P274,000.00 to complete their P1,000,000.00 earlier offer. When their offer
Article 1385, rescission shall not take place "when the things which are the object was refused, they filed an action praying for the following: a) rescission of the Deed
of the contract are legally in the possession of third persons who did not act in bad of Absolute Sale between de Leon and Rosencor dated September 4, 1990; b) the
faith." In order to hold that petitioners were in bad faith, there must be clear and defendants Rosencor/Rene Joaquin be ordered to reconvey the property to de Leon;
convincing proof that petitioners were made aware of the said right of first refusal and c) de Leon be ordered to reimburse the plaintiffs for the repairs of the property, or
either by the respondents or by the heirs of the spouses Tiangco. apply the said amount as part of the price for the purchase of the property in the sum
FACTS: of P100,000.00.
1. Plaintiffs and plaintiffs-intervenors averred that they are the lessees since 1971 of a 10. The trial court held that the right of redemption on which the complaint was based
two-story residential apartment owned by spouses Faustino and Cresencia Tiangco. was merely an oral one and as such, is unenforceable under the law.
The lease was not covered by any contract. The lessees were renting the premises and 11. Not satisfied with the decision of the trial court, repondents went to the Court of
were allegedly verbally granted by the lessors the pre-emptive right to purchase the Appeals which rendered its decision reversing the decision of the trial court. Hence,
property if ever they decide to sell the same. this petition for review on certiorari filed by petitioners Rosencor Development
2. Upon the death of the spouses Tiangcos in 1975, the management of the property Corporation and Rene Joaquin.
was adjudicated to their heirs who were represented by Eufrocina de Leon. The lessees ISSUES:
were allegedly promised the same pre-emptive right by the heirs of Tiangcos since the WON a right of first refusal is covered by the Statute of Frauds - NO
latter had knowledge that this right was extended to the former by the late spouses WON respondents have satisfactorily proven their right of first refusal over the subject
Tiangcos. property of the Deed of Sale between ROSENCOR and De Leon - YES
4. In June 1990, the lessees received a letter from Atty. Erlinda Aguila demanding that WON a Contract of Sale entered into in violation of a third party's right of first
they vacate the premises so that the demolition of the building be undertaken. They refusal be rescinded in order that such third party can exercise said right - YES,
refused to leave the premises. In that same month, de Leon refused to accept the provided that the buyer is in bad faith. OTHERWISE, the sale must be valid as regards
lessees' rental payment claiming that they have run out of receipts and that a new the buyer and seller without prejudice to the right of the third party (with a right of
collector has been assigned to receive the payments. Thereafter, they received a letter first refusal) to claim damages against the seller who disregarded such right.
2. It is true that the acquisition by a third person of the property subject of the contract
RULING: is an obstacle to the action for its rescission where it is shown that such third person is
WHEREFORE, premises considered, the decision of the Court of Appeals dated June in lawful possession of the subject of the contract and that he did not act in bad faith.
25, 1999 is REVERSED and SET ASIDE. The Decision dated May 13, 1996 of the Clearly, this rule is not applicable when the third person acted in bad faith.
Quezon City Regional Trial Court, Branch 217 is hereby REINSTATED insofar as it 3. A purchaser in good faith and for value is one who buys the property of another
dismisses the action for rescission of the Deed of Absolute Sale dated September 4, without knowledge that some other person has a right to or interest in such property
1990 and orders the payment of monthly rentals of P1,000.00 per month reckoned and pays a full and fair price for the same at the time of such purchase or before he has
from May 1990 up to the time respondents leave the premises. notice of the claim or interest of some other person in the property. Good faith
connotes an honest intention to abstain from taking unconscientious advantage of
RATIO: another.
RIGHT OF FIRST REFUSAL NOT COVERED BY THE STATUTE OF FRAUDS 4. In Equatorial Realty and Development, Inc. vs. Mayfair Theater, Inc., the Court
A right of first refusal is not among those listed as unenforceable under the statute of ordered the rescission of a contract entered into in violation of a right of first refusal
frauds. Furthermore, the application of Article 1403, par. 2(e) of the New Civil Code saying that since respondent therein had a right of first refusal over the said property,
presupposes the existence of a perfected, albeit unwritten, contract of sale. A right of it could only exercise the said right if the fraudulent sale is first set aside or rescinded.
first refusal, such as the one involved in the instant case, is not by any means a 5. This Court upheld the right of first refusal of the lessee Mayfair, and rescinded the
perfected contract of sale of real property. At best, it is a contractual grant, not of the sale of the property by the lessor Carmelo to Equatorial Realty "considering that
sale of the real property involved, but of the right of first refusal over the property Mayfair, which had substantial interest over the subject property, was prejudiced by
sought to be sold. It is thus evident that the statute of frauds does not contemplate cases its sale to Equatorial without Carmelo conferring to Mayfair every opportunity to
involving a right of first refusal. As such, a right of first refusal need not be written to negotiate within the 30-day stipulated period." The prevailing doctrine, as enunciated
be enforceable and may be proven by oral evidence. in the Equatorial, is that a contract of sale entered into in violation of a right of first
RESPONDENTS HAVE THE RIGHT OF FIRST REFUSAL refusal of another person, while valid, is rescissible.
1. Respondents have adequately proven the existence of their right of first refusal. 6. There is, however, a circumstance which prevents the application of this doctrine in
Federico Bantugan, Irene Guillermo, and Paterno Inquing uniformly testified that they the case at bench. In the case cited, the Court ordered the rescission of sale made in
were promised by the late spouses Faustino and Crescencia Tiangco and, later on, by violation of a right of first refusal precisely because the vendees therein could not have
their heirs a right of first refusal over the property they were currently leasing should acted in good faith as they were aware or should have been aware of the right of first
they decide to sell the same. Moreover, respondents presented a letter where Eufrocina refusal granted to another person by the vendors therein. The rationale for this is found
de Leon, the representative of the heirs of the spouses Tiangco, offering the property in the provisions of the New Civil Code on rescissible contracts. Under Article 1381
for sale. Verily, if Eufrocina de Leon did not recognize respondents' right of first of the New Civil Code, paragraph 3, a contract validly agreed upon may be rescinded
refusal over the property they were leasing, then she would not have bothered to offer if it is "undertaken in fraud of creditors when the latter cannot in any manner collect
the property for sale to the respondents. the claim due them." Moreover, under Article 1385, rescission shall not take place
2. It must be noted that petitioners did not present evidence before the trial court "when the things which are the object of the contract are legally in the possession
contradicting the existence of the right of first refusal of respondents over the disputed of third persons who did not act in bad faith."
property. They only presented petitioner Rene Joaquin, the vice-president of petitioner 6. The right of first refusal involved in the instant case was an oral one given to
Rosencor, who admitted having no personal knowledge of the details of the sales respondents by the deceased spouses Tiangco and subsequently recognized by their
transaction between Rosencor and the heirs of the spouses Tiangco. They also heirs. As such, in order to hold that petitioners were in bad faith, there must be clear
dispensed with the testimony of Eufrocina de Leon who could have denied the and convincing proof that petitioners were made aware of the said right of first refusal
existence or knowledge of the right of first refusal. As such, there being no evidence either by the respondents or by the heirs of the spouses Tiangco.
to the contrary, the right of first refusal claimed by respondents was substantially 7. It is axiomatic that good faith is always presumed unless contrary evidence is
proven by respondents before the lower court. adduced. In this regard, the rule on constructive notice would be inapplicable as it is
undisputed that the right of first refusal was an oral one and that the same was never
CONTRACT OF SALE CANNOT BE RESCINDED BECAUSE ROSENCOR IS reduced to writing, much less registered with the Registry of Deeds. In fact, even the
A BUYER IN GOOD FAITH lease contract by which respondents derive their right to possess the property involved
1. Under Article 1380 to 1381 (3) of the Civil Code, a contract otherwise valid may was an oral one.
nonetheless be subsequently rescinded by reason of injury to third persons, like 8. On this point, we hold that the evidence on record fails to show that petitioners acted
creditors. Rescission implies a contract which, even if initially valid, produces a lesion in bad faith in entering into the deed of sale over the disputed property with the heirs
or pecuniary damage to someone that justifies its invalidation for reasons of equity. of the spouses Tiangco. Respondents failed to present any evidence that prior to the
sale of the property on September 4, 1990, petitioners were aware or had notice of the unjustified violation of their right of first refusal. Their remedy however is not an
oral right of first refusal. action for the rescission of the Deed of Absolute Sale but an action for damages against
9. Respondents point to the letter dated June 1, 1990 indicative of petitioners' the heirs of the spouses Tiangco for the unjustified disregard of their right of first
knowledge of the said right where certain Atty. Erlinda Aguila demanded that refusal.
respondent Irene Guillermo vacate the structure they were occupying to make way for
its demolition. We fail to see how the letter could give rise to bad faith on the part of
the petitioner. No mention is made of the right of first refusal granted to respondents.
The name of petitioner Rosencor or any of its officers did not appear on the letter and
the letter did not state that Atty. Aguila was writing in behalf of petitioner. In fact,
Atty. Aguila stated during trial that she wrote the letter in behalf of the heirs of the
spouses Tiangco. Moreover, even assuming that Atty. Aguila was indeed writing in
behalf of petitioner Rosencor, there is no showing that Rosencor was aware at that
time that such a right of first refusal existed.
10. Neither was there any showing that after receipt of this June 1, 1990 letter,
respondents notified Rosencor or Atty. Aguila of their right of first refusal over the
property. Respondents did not try to communicate with Atty. Aguila and inform her
about their preferential right over the disputed property. There is even no showing that
they contacted the heirs of the spouses Tiangco after they received this letter to remind
them of their right over the property.
11. Respondents likewise point to the letter dated October 9, 1990 of Eufrocina de
Leon, where she recognized the right of first refusal of respondents, as indicative of
the bad faith of petitioners. We do not agree. Eufrocina de Leon wrote the letter on her
own behalf and not on behalf of petitioners and, as such, it only shows that Eufrocina
de Leon was aware of the existence of the oral right of first refusal. It does not show
that petitioners were likewise aware of the existence of the said right. Moreover, the
letter was made a month after the execution of the Deed of Absolute Sale on September
4, 1990 between petitioner Rosencor and the heirs of the spouses Tiangco. There is no
showing that prior to the date of the execution of the said Deed, petitioners were put
on notice of the existence of the right of first refusal.
12. Clearly, if there was any indication of bad faith based on respondents' evidence, it
would only be on the part of Eufrocina de Leon as she was aware of the right of first
refusal of respondents yet she still sold the disputed property to Rosencor. However,
bad faith on the part of Eufrocina de Leon does not mean that petitioner Rosencor
likewise acted in bad faith. There is no showing that prior to the execution of the Deed
of Absolute Sale, petitioners were made aware or put on notice of the existence of the
oral right of first refusal. Thus, absent clear and convincing evidence to the contrary,
petitioner Rosencor will be presumed to have acted in good faith in entering into the
Deed of Absolute Sale over the disputed property.
13. Considering that there is no showing of bad faith on the part of the petitioners, the
Court of Appeals thus erred in ordering the rescission of the Deed of Absolute Sale
dated September 4, 1990 between petitioner Rosencor and the heirs of the spouses
Tiangco. The acquisition by Rosencor of the property subject of the right of first
refusal is an obstacle to the action for its rescission where, as in this case, it was shown
that Rosencor is in lawful possession of the subject of the contract and that it did not
act in bad faith.
14. This does not mean however that respondents are left without any remedy for the
VAZQUEZ v. AYALA (JP) 5.7 The Buyer (Ayala) hereby commits that it will develop the
November 19, 2004 | Tinga, J. | Right of First Refusal 'Remaining Property' into a first class residential subdivision of the same
class as its New Alabang Subdivision, and that it intends to complete the
PETITIONER: Dr. Daniel Vazquez and Ma. Luisa M. Vazquez first phase under its amended development plan within three (3) years
RESPONDENTS: Ayala Corporation from the date of this Agreement. . . ."
5.15 The Buyer (Ayala) agrees to give the sellers (Vazquez
SUMMARY: The Vazquez Spouses entered into a MOA with Ayala where they Spouses) a first option to purchase four developed lots next to the
sold to Ayala their stocks in Conduit Dev’t Inc. Conduit’s main asset then was the “Retained Area” at the prevailing market price at the time of purchase.
Ayala Alabang Village. As part of the MOA, Ayala was to reserve an area for the 64. The parties are agreed that the development plan referred to in paragraph 5.7
Vazquez Spouses called the Retained Area. In addition to the Retained Area, par. is not Conduit's development plan, but Ayala's amended development plan
5.15 of the MOA also shows that Ayala agrees to give the Vazquez Spouses a first which was still to be formulated as of the time of the MOA.
option of repurchase of the four developed lots next to the Retained Area. These 65. While in the Conduit plan, the 4 lots to be offered for sale to the Vasquez
4 lots were then offered to be sold to the Vazquez Spouses in 1990 after its Spouses were in the first phase thereof or Village 1, in the Ayala plan which
development. This was however rejected by the latter because they prefer to pay was formulated a year later, it was in the third phase, or Phase II-c.
with the 1984 prices where they allege they sent demand letters. RTC ruled in 66. Taking the position that Ayala was obligated to sell the 4 lots adjacent to the
favor of Vazquez Spouses, CA reversed. "Retained Area" within 3 years from the date of the MOA, the Vasquez
spouses sent several "reminder" letters of the approaching so-called deadline.
The SC held that 5.15 is unmistakably not an option contract but a mere right of However, no demand after April 23, 1984, was ever made by the Vasquez
refusal. While the subject is determinate, the 4 lots, the period for within they will spouses for Ayala to sell the 4 lots.
offered for sale and the price is not specified. According then to the Civil Code, 67. In the contrary, one of the letters signed by their authorized agent, Engr.
this offer by Ayala can be withdrawn. Ayala offered the lots for sale with the 1990 Eduardo Turla, categorically stated that they expected "development of Phase
price but the Vazquez Spouses rejected this and gave their counteroffer. Ayala 1 to be completed by February 19, 1990, three years from the settlement of
rightfully then declined this counteroffer. It cannot be then said that Ayala is the legal problems with the previous contractor."
obliged to sell the 4 lots. 68. By early 1990 Ayala finished the development of the vicinity of the 4 lots to
be offered for sale. The four lots were then offered to be sold to the
DOCTRINE: In a right of first refusal while the object might be made Vasquez spouses at the prevailing price in 1990. This was rejected by the
determinate, the exercise of the right would be dependent not only on the grantor's Vasquez spouses who wanted to pay at 1984 prices, thereby leading to
eventual intention to enter into a binding juridical relation with another but also the suit.
on terms, including the price, that are yet to be firmed up. 69. RTC: In favor of Vazquez Spouses, ordering Ayala to sell the relevant lots in
the Ayala Alabang Village at the price of P460/square meter, amounting to
P1,349,540 (the 1984 prices).
FACTS: 70. The RTC sided with the Vazquez Spouses that the option to purchase, as can
56. Spouses Daniel and Ma. Luisa Vazquez (Vazquez spouses) entered into a be gleaned from paragraph 5.15 was valid because it was incorporated in the
Memorandum of Agreement (MOA) with Ayala Corporation (Ayala). MOA and the consideration therefor was the commitment by Ayala
57. Vazquez spouses were to sell their shares of stock in Conduit Development, Corporation to the Vazquez Spouses embodied in the MOA.
Inc. (Conduit) to Ayala. 71. Upon appeal, the CA reversed. According to it, par. 5.15 is not an option
58. The main asset of Conduit was a 49.9 hectare property in Ayala Alabang, contract but a right of first refusal there being no separate consideration
Muntinlupa, which was then being developed by Conduit under a therefor. Hence this appeal by the Vazques Spouses.
development plan where than land was divided into Villages 1, 2 and 3.
59. Under the MOA, Ayala was to develop the entire property, less what was ISSUE/s:
defined as the "Retained Area" consisting of 18,736 square meters. 19. WoN Ayala was in default or in delay in the fulfillment of the obligation.–
60. This "Retained Area" was to be retained by the Vazquez spouses. NO
61. The area to be developed by Ayala was called the "Remaining Area". 20. WoN the contract at bar is an option contract (not a contract with a right
62. In this "Remaining Area" were 4 lots adjacent to the "Retained Area" and of first refusal) - NO
Ayala agreed to offer these lots for sale to the Vazquez spouses at the
prevailing price at the time of purchase. RULING: Judgment in question is affirmed. Instant petition is denied.
63. Relevant provisions of the MOA:
they have not been provided with the specifications of these lots.
RATIO: 43. On the issue of whether or not paragraph 5.15 can properly be construed
32. On the issue on delay: According to the Civil Code, in order that the debtor as an option contract or a right of first refusal: The Court has clearly
may be in default it is necessary, unless he qualifies under the exceptions, that distinguished between an option contract and a right of first refusal.
the following requisites be present: (1) that the obligation be demandable and 44. An option is a preparatory contract in which one party grants to another, for
already liquidated; (2) that the debtor delays performance; and (3) that the a fixed period and at a determined price, the privilege to buy or sell, or to
creditor requires the performance judicially or extrajudicially. decide whether or not to enter into a principal contract. It binds the party who
33. Under Article 1193 of the Civil Code, obligations for whose fulfillment a day has given the option not to enter into the principal contract with any other
certain has been fixed shall be demandable only when that day comes. person during the period designated, and within that period, to enter into such
However, no such day certain was fixed in the MOA. contract with the one to whom the option was granted, if the latter should
34. Petitioners Vazquez Spouses, therefore, cannot demand performance after the decide to use the option. It must be supported by consideration.
three (3) year period fixed by the MOA for the development of the first phase 45. In a right of first refusal, on the other hand, while the object might be made
of the property since this is not the same period contemplated for the determinate, the exercise of the right would be dependent not only on the
development of the subject lots. grantor's eventual intention to enter into a binding juridical relation with
35. Since the MOA does not specify a period for the development of the subject another but also on terms, including the price, that are yet to be firmed
lots, petitioner Vazquez Spouses should have petitioned the court to fix the up.
period in accordance with Article 1197 of the Civil Code. 46. Applied to the instant case, paragraph 5.15 is obviously a mere right of
36. As no such action was filed by petitioner Vazquez Spouses, their complaint first refusal and not an option contract.
for specific performance was premature, the obligation not being demandable 47. Although the paragraph has a definite object, i.e., the sale of subject lots,
at that point. Accordingly, Ayala Corporation cannot likewise be said to have the period within which they will be offered for sale to petitioner
delayed performance of the obligation. Vazquez Spouses and, necessarily, the price for which the subject lots
37. Even assuming that the MOA imposes an obligation on Ayala Corporation to will be sold are not specified.
develop the subject lots within three (3) years from date thereof, Ayala 48. The phrase "at the prevailing market price at the time of the purchase"
Corporation could still not be held to have been in delay since no demand connotes that there is no definite period within which Ayala Corporation
was made by petitioner Vazquez Spouses for the performance of its is bound to reserve the subject lots for petitioner Vazquez Spouses to
obligation. exercise their privilege to purchase.
38. As found by the appellate court, petitioners (Vazquez Spouses)' letters which 49. Neither is there a fixed or determinable price at which the subject lots
dealt with the three (3)-year timetable were all dated prior to April 23, 1984, will be offered for sale. The price is considered certain if it may be
the date when the period was supposed to expire. determined with reference to another thing certain or if the
39. In other words, the letters were sent before the obligation could become determination thereof is left to the judgment of a specified person or
legally demandable. Moreover, the letters were mere reminders and not persons.
categorical demands to perform. 50. Further, paragraph 5.15 was inserted into the MOA to give petitioners the
40. More importantly, petitioner Vazquez Spouses waived the three (3)-year first crack to buy the subject lots at the price which Ayala Corporation would
period as evidenced by their agent, Engr. Eduardo Turla's letter to the effect be willing to accept when it offers the subject lots for sale. It is not supported
that petitioners agreed that the three (3)-year period should be counted from by an independent consideration. As such it is not governed by Articles 1324
the termination of the case filed by Lancer (Context: Ayala had a case with and 1479 of the Civil Code.
Lancer, subcontractor of GP Construction who worked the lots, wherein 51. Art. 1324: When the offeror has allowed the offeree a certain period to accept,
Ayala failed to pay). the offer may be withdrawn at any time before acceptance by communicating
41. Manifestly, the letter expresses not only petitioners' acknowledgement that such withdrawal, except when the option is founded upon a consideration, as
the delay in the development of Phase I was due to the legal problems with something paid or promised.
GP Construction, but also their acquiescence to the completion of the 52. Consequently, the "offer" may be withdrawn anytime by communicating the
development of Phase I at the much later date of February 19, 1990. withdrawal to the other party.
42. More importantly, by no stretch of semantic interpretation can it be construed 53. In this case, Ayala Corporation offered the subject lots for sale to petitioners
as a categorical demand on Ayala Corporation to offer the subject lots for sale at the price of P6,500.00/square meter, the prevailing market price for the
to petitioners as the letter merely articulates petitioners' desire to exercise property when the offer was made on June 18, 1990.
their option to purchase the subject lots and concern over the fact that 54. Insisting on paying for the lots at the prevailing market price in 1984 of
P460.00/square meter, petitioners rejected the offer. Ayala Corporation
reduced the price to P5,000.00/square meter but again, petitioners
rejected the offer and instead made a counter-offer in the amount of
P2,000.00/square meter.
55. Ayala Corporation rejected petitioners' counter-offer. With this
rejection, petitioner Vazquez Spouses lost their right to purchase the
subject lots.
56. It cannot, therefore, be said that Ayala Corporation breached petitioners' right
of first refusal and should be compelled by an action for specific performance
to sell the subject lots to petitioners at the prevailing market price in 1984.
RIVIERA FILIPINA v. CA (HENRY) property as security. With the loan unpaid, Prudencial extrajudicially
April 5, 2002 | De Leon, Jr., J. | Right of First Refusal foreclosed the mortgage.
PETITIONER: Riviera Filipina, Inc. 3. At a public auction with the bank as the highest bidder, Reyes was left with
RESPONDENTS: CA, Juan Reyes (deceased), Estefania Reyes, et al. no choice but to sell the property due to his inability to raise enough money
to redeem it. (but note: redemption period was set to expire: March 7, 1989)
SUMMARY: This is a case that basically revolves around the right of first refusal 4. In his pursuit of selling the property, it was noted that Par 11 of his lease
of Riviera as far as the sale of the property leased to it by Reyes is concerned. contract with petitioner Riviera indicated that the latter had the right of first
refusal, and hence, he offered to sell the property for 5k/sqm through its
The subject property is actually owned by Ryees, who mortaged the same to President, Angeles.
Prudencial Bank, who eventually was unable to pay its loan and so the property 5. Riviera bargained for 3.5k/sqm, but Reyes pushed for his initial offer, to
was foreclosed and sold to the bank as the highest bidder, with March 1989 as the which Angeles said he’ll consult with the Board of Directors of Riviera.
expiry of the redemption date. With not enough funds left, he finally decided to 6. Sometime October 1988, Angeles informed Reyes of the new offer: 4k/sqm,
sell the property so he can redeem it from the bank. Reyes did not only decline this offer, but also told them that the asking price
is 6k/sqm.
With Rivieara as lessee having the right to first refusal, the property was offered 7. In a letter dated Nov 2, 1988, Atty Juan, counsel of Reyes, informed Riviera
at 5k/sqm, to which the former made a counter offer of 3.5k/sqm (which was of the former’s asking price (still 6k/sqm), and further, that this is served to
obviously not accepted as it was too low. Eventually though, petitioner Riviera them for their exercise of their right to first refusal, and is given 10 days to
offered to buy at 4k/sqm, but Reyes was firm with not going lower, and even went decide—to which upon the period’s expiry, Reyes will be allowed to sell it
up to 6k/sqm. to other interested buyers.
8. Nov 22, 1988 (10 days after), Riviera through a letter informed Reyes of their
Riviera was informed of its right of first refusal, with the offer standing at 6k/sqm, intent to purchase, with its offer at 5k/sqm
to which it did not accept, but offered 5k/sqm which was not accepted. This 9. Dec 2, 1988, Angeles wrote to Reyes confirming their intent to purchase at
entitled Reyes to look for other potential buyers, and successfully, found one: the final price of 5k/sqm, to which they even indicated that such offer is what
Cypress and Cornhill will buy it. After negotiations, the offer was settled at they feel should be the market price of the said property. That, Reyes should
5.3k/sqm. The buyer then gave Reyes payment, which enabled Reyes to redeem make up his mind in 15 days as they are being offered other properties as
the property from the bank. well.
10. Reyes, via Atty. Juan, in a letter, informed Riviera that their offer is not
Now, Riviera is claiming that its right of first refusal was violated. RTC and CA accepted, and further, that they have failed to take advantage of their right of
both ruled against Riviera, and held that no right of first refusal was violated as it first refusal which now is lost.
specifically indicated in its letter to Reyes that it wouldn’t make an offer higher 11. December 1988, Reyes confided to Traballo (close family friend and
than 5k/sqm. President of Cypress) about the nearing of the expiry of the redemption period
for the foreclosed mortgaged property.
Upon petitioner’s appeal to the SC, the ruling of the appellate court was upheld, 12. Traballo expressed interest in buying the said property, and gave an offer for
with the SC recognizing the acts of Riviera as far as the contract is concerned, and 5.3k/sqm, Reyes accepted.
that it did not express dissent when it was informed that such right of first refusal 13. However, since Traballo did not have the amount on hand, he informed Reyes
was lost after it declined the last offer of 6k/sqm. that he’ll look first for a partner.
14. Sometime January 1989, with the expiration of the redemption period fast
DOCTRINE: A right of first refusal means identity of terms and conditions to be approaching, Reyes once again went to Riviera, who after further
offered to the lessee and all other prospective buyers and a contract of sale entered negotiations, gave his final offer at 5.3k/sqm.
into in violation of a right of first refusal of another person, while valid, is 15. February 1989 (1 month before expiration of redemption period!!!), Cypress
rescissible. finally found a partner, Cornhill Trading Corp, and both finally came up with
the amount sufficient to cover the redemption money, to which Reyes paid to
FACTS: Prudencial, to redeem the property.
1. November 1982, respondent Reyes executed a Contract of Lease (for 10 16. May 1 1989, Reyes executed a Deed of Absolute Sale covering the property
years, renewable) with petitioner Riviera for a 1,018-sqm parcel of land along in favor of Cypress and Cornhill, for Php 5,395,400.00.
EDSA. 17. On the same date, Cypris and Cornhill mortgaged the property to Urban
2. Reyes loaned from Prudencial Bank, to which he mortgaged the leased
Development Bank for Php 3M. the seller or offer to purchase of any prospective buyer.
18. Later on, Riviera sought from Reyes, Cypress, and Cornhill, a resale of the 7. Consequently, the court mentioned that the prevailing doctrine is that, a right
property and claims its right of first refusal was violated. A suit was of first refusal means identity of terms and conditions to be offered to the
eventually filed by petitioner to compel the respondents to transfer the lessee and all other prospective buyers and a contract of sale entered into
disputed lands in their favor. in violation of a right of first refusal of another person, while valid, is
19. RTC dismissed the case, and ruled that the right of first refusal of the rescissible.
petitioners was not at all violated, when it informed Reyes that it wasn’t 8. However, the SC noted that general propositions do not decide specific cases.
accepting any offer higher than 5k/sqm (with Reyes’ offer at 6k/sqm). Rather, laws are interpreted in the context of the peculiar situation of
20. An appeal was filed before the CA who affirmed the RTC decision in its each proceeding. Each case has its own flesh and blood and cannot be rueld
entirety. It further mentioned the earnest efforts of Reyes to respect Riviera’s uponthe basis of isolated clinical classroom principles. Analysis and
contractual right to first refusal, for multiple times, when it offered and construction should NOT be limited to the words used in the contract, as
offered but the latter did not accept. Hence, this petition. they may not reflect the true intent of the parties. The court MUST read
ISSUE/s: a contracat as the average person would readi t and should not give it a
1. WoN Riviera’s right to first refusal was violated – NO strained or forced construction.
9. In relation to the present case, the SC finds this doctrine relevant and pressing.
RULING: SC denied the petition and affirmed the CA ruling. Reyes and Riviera shaped their understanding and interpretation of the lease
provision on the right of first refusal to mean simply that should Reyes decide
RATIO: to sell the subject property during the term of the lease, Riviera will be the
1. Riviera claims that its right of first refusal was totally violated by Reyes’ sale first to whom it will be offered. Which indeed was evidenced by the
to Cypress and Cornhill. That, the right of first refusal principally amounts to negotiations that took place after Reyes’s decision to sell the property.
a right to match in the sense that it needs another offer for the right to be 10. It is evident in the letters dated December 1988 and Feb 1989 that Riviera
exercised. was so sure of its position and took obvious advantage of the knowledge of
2. The concept of such right began in the case of Guzman, Bocaling & Co v. the time element in its negotiations with Reyes as the redemption period of
Bonnevie, where the Court held that a lease with a proviso granting the lessee the subject foreclosed property drew near.
the right of first priority all things and conditions being equal meant that 11. Riviera basically exhibited the “take-it-or-leave-it” vibe in its negotiations. It
there should be identity of the terms and conditions to be offered to the lessee was firm in its 5k/sqm offer, no more no less. It voiced that it had other
and all other prospective buyers, with lessee to enjoy the right of first priority. properties to consider, and even pressured Reyes to decide in 15 days.
A deed of sale executed in favor of a third party who cannot be deemed a Naturally, given such circumstance, Reyes would disagree as it is a highly
purchaser in good faith, and which is in violation of the right of first refusal disadvantageous offer.
granted to the lessee is not voidable but rescissible under the Statute of 12. It was evident that there was no violent reaction when Reyes’ counsel
Frauds. informed Riviera that it had lost its right of first refusal. Now, petitioner
3. Subsequently in the case of Ang Yu Asuncion v. CA the SC departed from the cannot be heard that had it been informed of the offer of Cypress (5.3k/sqm),
doctrine of Guzman and refused to rescind a contract of sale which violated it would’ve matched the said price.
the right of first refusal. That, such right of first refusal cannot be deemed a 13. In sum, the SC finds that in the interpretation of the right of first refusal as
perfected contract of sale, hence a breach thereof does not entitle the understood by the parties of this case, the question as to what is to be included
aggrieved party to a writ of execution of judgment but to an action for therein or what is meant by the same, as in all other provisions of the contract,
damages in a proper forum for the purpose. is for the parties and not for the court to determine, and is a question that
4. In the case of Equitorial Realty Devt v. Mayfair, the Court reverted to the cannot be resolved by what the parties might have provided had they thought
Guzman doctrine and said that rescission is the appropriated relief for about it, which is evident from Riviera claims, or by what the court might
parties whom their right of first refusal is violated. include regarding abstract fairness.
5. Accordingly, in Paranaque Kings Enterprises, Inc v. CA the SC affirmed and 14. The SC held that it would be rewriting the contract between Reyes and
summarized the doctrines of the above cases and held that, in order to have Riviera under the guise of construction were they to interpret the right
full compliance with the contractual right of first refusal, the sale of the of first refusal as Riviera propounds it, despite a contrary construction
properties for the price for which they were finally sold to a third person as exhibited by its actions.
should have likewise been first offered to the former. 15. SC has no right to make new contracts for parties or ignore what they have
6. The basis of the right of first refusal must be the current offer to sell of already made.
15 MACION v. GUIANI (Salve) 2. Macion and De la Vida entered into a contract to sell where De la Vida
August 4, 1993 | Romero, J. | Contract to sell assured Macion that they would buy the lots on or before July 31, 1993 for
PETITIONER: Henry Macion and Angeles Macion 1,750,000. In the meantime, Macion surrendered the physical possession of
RESPONDENTS: Hon. Japal M. Guiani, RTC and De la Vida Institute the lots to them. Because of this, de la vida built a building worth 800,000.
3. On July 31, 1993, the sale didn’t materialize. Thus, Macion filed a complaint
SUMMARY: Macion owned 2 parcels of lot. These were proposed to be the for unlawful detainer against De la Vida. De la vida filed a complaint for
extension sites of the school, De la Vida. Macion and De la Vida entered into a reformation of the contract to sell.
contract to sell. On the agreed date, the sale didn’t materialize. Macion filed for 4. On February 6, 1992, the parties entered into a compromise agreement.
unlawful detainer against De la Vida while De la Vida filed for reformation of the Maciano would give De la Vida 5 months to raise 2,060,000. Failure to do
contract to sell. Subsequently, Macion and De la Vida entered into a compromise so, De la Vida should vacate the premises.
agreement where Maciano would give De la Vida 5 months to raise 2,060,000 (or 5. Compromise Agreement:
to secure a loan) and failure to do so, De la Vida should vacate the premises. De 6. that upon the execution of this agreement, the defendant will furnish the
la Vida now requests Maciano to execute the contract to sell because they have plaintiff with xerox copy of the land title for each lot which the latter may use
allegedly negotiated a loan from BPI. Maciano filed for a motion for execution for the purpose of providing information in securing a loan from any
of judgment since De la Vida failed to settle their obligations. Judge: denied financing or banking institution of their choice.
motion for execution. Ordered Maciano to execute a contract to sell because the 7. that if within the period of five (5) months from and after February 6, 1992,
failure of De la Vida was due to the refusal of Maciano to execute the contract to the plaintiff succeeds in obtaining funds for the purpose of settling their
sell. obligations with defendants in the total sum of P2,060,000.00 the latter shall
Issue: WoN respondent Judge committed grave abuse of discretion in ordering oblige themselves to execute, sign and deliver to the former the
Maciano to execute the contract to sell? NO corresponding Deed of Sale for the two (2) lots which is the subject of this
Despite par 7 of the compromise agreement obligates the execution of a deed of case and turn-over to said plaintiff the owner's duplicate copy of TCT Nos.
sale and not a contract to sell, the contemporaneous interpretation of the T-22004 and T-22005 of the Registry of Deeds for the City of Cotabato.
agreement showed that prior to the compromise agreement, the parties already 8. De la Vida wrote to Maciano that they received a different compromise
entered into a contract to sell. Where the seller promised to execute a deed of agreement as to what was agreed upon. It was supposed to be 2,000,000 and
absolute sale upon completing payment of the price, it is a contract to sell. Here, not 2,060,000
the sale is still in the executory stage since the passing of title is subject to a 9. RTC: approved the compromise agreement dated February 6, 1992.
suspensive condition, namely, that if private respondent is able to secure the 10. De la Vida now requests Maciano to execute the contract to sell because they
needed funds to be used in the purchased of the 2 lots owned by Maciano. Hence, have allegedly negotiated a loan from BPI.
no transfer of ownership yet. However, since the period in the compromise 11. Maciano filed for a motion for execution of judgment since De la Vida failed
agreement already lapsed, SC ordered RTC to fix a new period. to settle their obligations.
12. Judge: denied motion for execution. Ordered Maciano to execute a contract
DOCTRINE: to sell because the failure of De la Vida was due to the refusal of Maciano to
The compromise agreement, having been signed by both parties, is tantamount to execute the contract to sell.
a bilateral promise to buy and sell a certain thing for a price certain. Hence, this
gives the contracting parties rights in personam, such that each has the right to ISSUES:
demand from the other the fulfillment of their respective undertakings. 1. WoN respondent Judge committed grave abuse of discretion in ordering
Demandability may be exercised at any time after the execution of the Deed. Maciano to execute the contract to sell? NO

In a contract to sell, there is no immediate transfer of ownership. In contracts to RULING: WHEREFORE, the instant petition is DISMISSED. Petitioners are hereby
sell, payment is a positive suspensive condition, failure of which does not ordered to EXECUTE a contract to sell in favor of private respondents. On the other
constitute a breach but an event that hand, private respondent is ordered to DEPOSIT with the trial court current rentals
pending consummation of the transaction between the parties. The trial court is
FACTS: ordered to FIX anew the period within which private respondents may be given the
1. The subject of this litigation revolves around 2 parcels of lot owned by opportunity to raise funds for the purchase of the two (2) adjoining lots owned by
Macion. These lots are proposed to be extension sites of De la Vida Institute, petitioners.
a school in Cotabato City. RATIO:
1. Par. 7 of the compromise agreement: thereafter with the RTC.
2. that if within the period of five (5) months from and after February 6, 1992,
the plaintiff succeeds in obtaining funds for the purpose of settling their
obligations with defendants in the total sum of P2,060,000.00 the latter shall
oblige themselves to execute, sign and deliver to the former the
corresponding Deed of Sale for the two (2) lots which is the subject of this
case and turn-over to said plaintiff the owner's duplicate copy of TCT Nos.
T-22004 and T-22005 of the Registry of Deeds for the City of Cotabato.
3. From the aforecited paragraph, it is clear that the seller is obliged to execute
a Deed of Sale and not a Contract to Sell upon payment of the full price of
P2.06 million. Thereafter, the sellers would turn over to the buyers,
respondents herein, the owner's duplicate copy of Transfer Certificate of Title
Nos. T-22004 and T-22005.
4. Interpretation of the compromise agreement with the use of the
contemporaneous and subsequent acts of the parties:
5. Prior to the signing of the compromise agreement, the parties already entered
into a contract to sell.
6. Within the time frame agreed upon by the parties, De La Vida wrote three (3)
letters dated may 19, 20 and 26 requesting Maciano to execute a contract to
sell in its favor.
7. Where the seller promised to execute a deed of absolute sale upon completing
payment of the price, it is a contract to sell. In the case at bar, the sale is still
in the executory stage since the passing of title is subject to a suspensive
condition, namely, that if private respondent is able to secure the needed
funds to be used in the purchased of the two (2) lots owned by petitioners. A
mere executory sale, one where the sellers merely promise to transfer the
property at some future date, or where some conditions have to be fulfilled
before the contract is converted from an executory to an executed one, does
not pass ownership over the real estate being sold.
8. The compromise agreement, having been signed by both parties, is
tantamount to a bilateral promise to buy and sell a certain thing for a price
certain. Hence, this gives the contracting parties rights in personam, such that
each has the right to demand from the other the fulfillment of their respective
undertakings. Demandability may be exercised at any time after the execution
of the Deed.
9. In a contract to sell, there is no immediate transfer of ownership. In contracts
to sell, payment is a positive suspensive condition, failure of which does not
constitute a breach but an event that prevents the obligation of the vendor to
convey title from materializing.
10. Petitioners as promisors were never obliged to convey title before the
happening of the suspensive condition.
11. Since the period given by the petitioners under the compromise agreement
has already lapsed, the Court orders the trial court to fix anew a period within
which private respondents could secure the needed funds for the purchase.
12. (not part of the topic) De la Vida consigned their rent payments. The SC said
that it is proper that they be ordered to deposit monthly rentals collected
MANILA METAL CONTAINER CORP v. PNB (LORENA) 2. PNB later granted Manila Corp a new credit accommodation of P1,000,000.
December 20, 2006 | Collejo, Sr., J. | Essential Elements of a contract of Sale On November 16, 1973, Manila Corp executed an Amendment of Real Estate
PETITIONER: Manila Metal Container Corporation, Reynaldo C. Tolentino Mortgage over its property
[intervenor] 3. On May 31, 1981, Manila Corp secured another loan of P653,000 from PNB,
RESPONDENTS: Philippine National Bank, DMCI-Project Developers, payable in quarterly installments of P32,650 plus interests and other charges.
inc.[intervenor] 4. On Augyst 5, 1982, PNB filed a petition for extrajudicial foreclosure of the
real estate mortgage and sought to have the property sold at public auction
SUMMARY: Manila Corp executed a real estate mortgage over a parcel of land in for P911,532.21, Manila Corp’s outstanding obligation to PNB as of June 30,
favor of PNB to secure a P900,000 loan, a credit accommodation of P1000,000 and 1982, plus interests and attorney's fees.
another loan of P653,000 it obtained from the said bank. On August 5, 1982, PNB 5. After due notice and publication, the property was sold at public auction on
filed a petition for extrajudicial foreclosure of the real estate mortgage and sought September 28, 1982 where PNB was declared the winning bidder
to have the property sold at public auction for P911,532.21. After due notice and for P1,000,000. The period to redeem was to expire on February 17, 1984
publication, the property was sold at public auction on September 28, 1982 where because the contract of sale was registered on Feb 17, 1983.
PNB was declared the winning bidder for P1,000,000. Manila Corp sent a letter to 6. Manila Corp sent a letter dated August 25, 1983 to PNB, requesting that it be
PNB, requesting that it be granted an extension of time to redeem/repurchase the granted an extension of time to redeem/repurchase the property. PNB replied
property. some PNB Pasay City Branch personnel informed Manila Corp that as a on August 30, 1983, informing Manila Corp hat the request had been referred
matter of policy, the bank does not accept "partial redemption. Since Manila Corp to its Pasay City Branch for appropriate action and recommendation.
failed to redeem the property, the Register of Deeds issued a new title in favor of 7. In a letter dated February 10, 1984, Manila Corp reiterated its request for a
PNB. one year extension from February 17, 1984 within which to
Meanwhile, SAMD had prepared a statement of account in which Manila Corp's redeem/repurchase the property on installment basis. Meanwhile, some PNB
obligation amounted to P1,574,560.47 and SAMD recommended to the Pasay City Branch personnel informed Manila Corp that as a matter of policy,
management of PNB that Manila Corp be allowed to repurchase the property the bank does not accept "partial redemption.
for P1,574,560.00. The PNB management informed Manila Corp that it was 8. Since Manila Corp failed to redeem the property, the Register of Deeds issued
rejecting the offer and the recommendation of the SAMD. Instead, PNB suggested a new title in favor of PNB. Manila Corp’s offers had not yet been acted upon
that Manila Corp purchase the property for P2,660,000, its minimum market value. by PNB.
Manila Corp declared that it had already agreed to the SAMD's offer to purchase 9. Meanwhile, the Special Assets Management Department (SAMD) had
the property for P1,574,560.47, and deposited P725,000. The issue in this case is prepared a statement of account, and as of June 25, 1984 Manila Corp's
WoN Manila Corp and PNB had entered into a perfected contract for Manila Corp obligation amounted to P1,574,560.47. This included the bid price
to repurchase the property from PNB. The SC ruled in the negative. The P725,000 of P1,056,924.50, interest, advances of insurance premiums, advances on
Manila Corp had remitted to PNB was not "earnest money" which could be realty taxes, registration expenses, miscellaneous expenses and publication
considered as proof of the perfection of a contract of sale under Article 1482 of the cost.
New Civil Code. It was merely a deposit to be applied as part of the purchase price 10. When apprised of the statement of account, Manila Corp remitted P725,000
of the property, in the event that PNB would approve the recommendation of SAMD to PNB as "deposit to repurchase," and Official Receipt No. 978191 was
for PNB to accept Manila Corp's offer to purchase the property for P1,574,560.47. issued to it.
Unless and until the PNB accepted the offer on these terms, no perfected contract of 11. SAMD recommended to the management of PNB that Manila Corp be
sale would arise. allowed to repurchase the property for P1,574,560. In a letter dated
November 14, 1984, the PNB management informed Manila Corp that it was
DOCTRINE: Absent proof of the concurrence of all the essential elements rejecting the offer and the recommendation of the SAMD. It was suggested
(Consent, Subject Matter, and Consideration) of a contract of sale, the giving that Manila Corp purchase the property for P2,660,000, its minimum market
of earnest money cannot establish the existence of a perfected contract of value. PNB gave Manila Corp until December 15, 1984 to act on the proposal;
sale otherwise, its P725,000 deposit would be returned and the property would be
sold to other interested buyers.
FACTS:
12. Manila Corp, however, did not agree to PNB's proposal. Instead, it wrote
1. Manila Metal Container Corporation (Manila Corp) was the owner of a 8,015
another letter dated December 12, 1984 requesting for a reconsideration.
square meter parcel of land located in Mandaluyong. To secure a P900,000
13. PNB replied in a letter dated December 28, 1984, wherein it reiterated its
loan it had obtained from Philippine National Bank (PNB), Manila Corp
proposal that Manila Corp purchase the property for P2,660,000 PNB again
executed a real estate mortgage over the lot.
informed Manila Corp that it would return the deposit should Manila Corp performance against PNB. The P725,000 remitted by Manila Corp to PNB
desire to withdraw its offer to purchase the property. on June 4, 1985 was a "deposit," and not a downpayment or earnest money.
14. On February 25, 1985, Manila Corp, through counsel, requested that PNB 25. The CA rendered judgment on May 11, 2000 affirming the decision of the
reconsider its letter dated December 28, 1984. Manila Corp declared that it RTC.
had already agreed to the SAMD's offer to purchase the property 26. Manila Corp filed a motion for reconsideration, which the CA denied.
for P1,574,560.47, and that was why it had paid P725,000.00. Manila Corp 27. Thus, Manila Corp filed the instant petition for review on certiorari
warned PNB that it would seek judicial recourse should PNB insist on the
position. ISSUE/s:
15. PNB informed Manila Corp that the PNB Board of Directors had accepted its 1. WoN Manila Corp and PNB had entered into a perfected contract for Manila
offer to purchase the property, but for P1,931,389.53 in cash less Corp to repurchase the property from PNB – NO
the P725,000 already deposited with it. On page two of the letter was a space
above the typewritten name of Manila Corp's President, Pablo Gabriel, where RULING: IN LIGHT OF ALL THE FOREGOING, the petition is DENIED.
he was to affix his signature. However, Pablo Gabriel did not conform to the The assailed decision is AFFIRMED. Costs against petitioner Manila Metal
letter but merely indicated therein that he had received it. Manila Corp did
Container Corporation.
not respond, so PNB requested Manila Corp in a letter dated June 30, 1988
to submit an amended offer to repurchase.
16. Manila Corp rejected PNB's proposal in a letter dated July 14, 1988. It RATIO:
maintained that PNB had agreed to sell the property for P1,574,560.47, and 1. A contract is a meeting of minds between two persons whereby one binds
that since its P725,000.00 downpayment had been accepted, PNB was himself, with respect to the other, to give something or to render some
proscribed from increasing the purchase price of the property. Manila Corp service. Under Article 1318 of the New Civil Code, there is no contract unless
averred that it had a net balance payable in the amount of P643,452.34. the following requisites concur:
17. PNB, however, rejected Manila Corp's offer to pay the balance a. Consent of the contracting parties;
of P643,452.34 in a letter dated August 1, 1989. b. Object certain which is the subject matter of the contract;
18. On August 28, 1989, Manila Corp filed a complaint against PNB for c. Cause of the obligation which is established
"Annulment of Mortgage and Mortgage Foreclosure, Delivery of Title, or 2. By the contract of sale, one of the contracting parties obligates himself to
Specific Performance with Damages." transfer the ownership of and deliver a determinate thing, and the other to pay
19. Manila Corp later filed an amended complaint and supported its claim for therefor a price certain in money or its equivalent. The absence of any of the
damages essential elements will negate the existence of a perfected contract of sale.
20. In its Answer to the complaint, PNB averred, as a special and affirmative 3. When the contract of sale is not perfected, it cannot, as an independent source
defense, that it had acquired ownership over the property after the period to of obligation, serve as a binding juridical relation between the parties.
redeem had elapsed. It claimed that no contract of sale was perfected between 4. the stages of a contract of sale are as follows:
it and Manila Corp after the period to redeem the property had expired. a. negotiation
21. While the case was pending, PNB demanded, on September 20, 1989, that i. covering the period from the time the prospective
Manila Corp vacate the property within 15 days from notice, but Manila Corp contracting parties indicate interest in the contract to the
refused to do so. time the contract is perfected;
22. On March 18, 1993, Manila Corp offered to repurchase the property ii. formally initiated by an offer, which, however, must be
for P3,500,000. The offer was however rejected by PNB, in a letter dated certain
April 13, 1993. According to it, the prevailing market value of the property iii. At any time prior to the perfection of the contract, either
was approximately P30,000,000, and as a matter of policy, it could not sell negotiating party may stop the negotiation.
the property for less than its market value b. perfection
23. On June 21, 1993, Manila Corp offered to purchase the property i. takes place upon the concurrence of the essential elements
for P4,250,000 in cash. The offer was again rejected by PNB on September of the sale which are the meeting of the minds of the parties
13, 1993. as to the object of the contract and upon the price;
24. On May 31, 1994, the trial court ordered PNB to refund the P725,000 deposit c. consummation
petitioner had made. It ruled that there was no perfected contract of sale i. begins when the parties perform their respective
between the parties; hence, Manila Corp had no cause of action for specific undertakings under the contract of sale, culminating in the
extinguishment thereof.
5. To convert the offer into a contract, the acceptance must be absolute and must previously conformed, PNB set the purchase price at P2,660,000.00.
not qualify the terms of the offer; it must be plain, unequivocal, unconditional f. PNB's acceptance of Manila Corp's offer was qualified, hence can
and without variance of any sort from the proposal. be at most considered as a counter-offer. If Manila Corp had
6. Consequently, when something is desired which is not exactly what is accepted this counter-offer, a perfected contract of sale would have
proposed in the offer, such acceptance is not sufficient to guarantee consent arisen; as it turns out, however, Manila Corp merely sought to have
because any modification or variation from the terms of the offer annuls the the counter-offer reconsidered. This request for reconsideration
offer. The acceptance must be identical in all respects with that of the offer would later be rejected by PNB.
so as to produce consent or meeting of the minds. g. The P725,000.00 Manila Corp had remitted to PNB was not "earnest
7. There was no perfected contract of sale between Metro Corp and PNB money" which could be considered as proof of the perfection of a
over the subject property contract of sale under Article 14825 of the New Civil Code.
a. Manila Corp had until February 17, 1984 within which to redeem i. This was affirmed by the stipulation of facts which the
the property. However, since it lacked the resources, it requested for parties entered into in the trial court
more time to redeem/repurchase the property under such terms and 1. On June 25, 1984, MMCC paid P725,000.00 to
conditions agreed upon by the parties. The request, which was made PNB as deposit to repurchase the property. The
through a letter dated August 25, 1983, was referred to the PNB's deposit of P725,000 was accepted by PNB on the
main branch for appropriate action. Before PNB could act on the condition that the purchase price is still subject to
request, Manila Corp again wrote respondent. the approval of the PNB Board.
b. When Manila Corp was told that PNB did not allow "partial ii. The P725,000 was merely a deposit to be applied as part of
redemption," it sent a letter to PNB's President reiterating its offer the purchase price of the property, in the event that PNB
to purchase the property. There was no response to Manila Corp 's would approve the recommendation of SAMD for PNB to
letters dated February 10 and 15, 1984. accept Manila Corp's offer to purchase the property
c. The statement of account prepared by the SAMD stating that the net for P1,574,560.47. Unless and until the PNB accepted the
claim of PNB as of June 25, 1984 was P1,574,560.47 cannot be offer on these terms, no perfected contract of sale would
considered an unqualified acceptance to Manila Corp 's offer to arise. Absent proof of the concurrence of all the
purchase the property. The statement is but a computation of the essential elements of a contract of sale, the giving of
amount which Manila Corp was obliged to pay in case PNB would earnest money cannot establish the existence of a
later agree to sell the property, including interests, advances on perfected contract of sale.
insurance premium, advances on realty taxes, publication cost, h. It appears that, per its letter to Manila Corp dated June 4, 1985, the
registration expenses and miscellaneous expenses. PNB had decided to accept the offer to purchase the property
d. There is no evidence that the SAMD was authorized by PNB's Board for P1,931,389.53. However, this amounted to an amendment of
of Directors to accept Manila Corp's offer and sell the property PNB's qualified acceptance, or an amended counter-offer, because
for P1,574,560.47. Any acceptance by the SAMD of Manila Corp's while the PNB lowered the purchase price, it still declared that its
offer would not bind PNB. A corporation can only execute its acceptance was subject to the some terms and condition
powers and transact its business through its Board of Directors and i. It appears that although PNB requested Manila Corp to conform to
through its officers and agents when authorized by a board its amended counter-offer, Manila Corp refused and instead
resolution or its by-laws. requested PNB to reconsider its amended counter-offer. Manila
e. It appears that the SAMD had prepared a recommendation for PNB Corp's request was ultimately rejected and PNB offered to refund
to accept Manila Corp's offer to repurchase the property even its P725,000 deposit.
beyond the one-year period; it recommended that Manila Corp be
allowed to redeem the property and pay P1,574,560 as the purchase
price. PNB later approved the recommendation that the property be
sold to Manila Corp. But instead of the P1,574,560.47
recommended by the SAMD and to which Manila Corp had

5
ART. 1482. Whenever earnest money is given in a contract of sale, it shall be
considered as part of the price and as proof of the perfection of the contract.
Bank of the Phil (DBP) on April 21, 1959 as security for a loan of P441,000.
Villonco v. Bormaheco (Celaje) The mortgage debt was fully paid on July 10, 1969.
G.R. No. L-26872 | July 25, 1975 | Aquino, J. 2. Cervantes is the president of Bormaheco, Inc., a dealer and importer of
PETITIONER: VILLONCO REALTY COMPANY and EDITH PEREZ DE industrial and agricultural machinery. The entire lots are occupied by the
TAGLE. building, machinery and equipment of Bormaheco, Inc. and are adjacent to
the property of Villonco Realty Company situated at 219 Buendia Avenue.
RESPONDENTS: BORMAHECO, INC., FRANCISCO N. CERVANTES and
ROSARIO N. CERVANTES, 3. In the early part of February, 1964 there were negotiations for the sale of the
said lots and the improvements thereon between Romeo Villonco of Villonco
SUMMARY: In the early part of February, 1964 there were negotiations for the Realty Company "and Bormaheco, Inc., represented by its president,
sale of the 3 lots located at 245 Buendia Avenue, Makati, Rizal with a total area Francisco N. Cervantes, through the intervention of Edith Perez de Tagle, a
of 3,500 square meters (actually owned by Cervantes spouses, but represented by real estate broker".
the present of Bormaheco, Inc. as being owned by Bormaheco), between Romeo
Villonco of Villonco Realty Company and Bormaheco, Inc. represented by its 4. During the negotiations, Villonco Realty Company assumed that the lots
president, Francisco N. Cervantes. A contract of sale was eventually signed. But, belonged to Bormaheco, Inc. and that Cervantes was duly authorized to sell
twenty-six days after the signing of the contract of sale, Cervantes rescinded the same. Cervantes did not disclose to the broker and to Villonco Realty
the contract, for failure of the condition that another property was to be Company that the lots were conjugal properties of himself and his wife and
acquired to materialize. Villonco eventually filed a complaint for specific that they were mortgaged to the DBP.
performance against Bormaheco, Inc. 5. Bormaheco, Inc., through Cervantes, made a written offer dated February 12,
Supreme Court ruled in favor of Villonco Realty Company. Bormaheco's 1964, to Romeo Villonco for the sale of the property. (3,500 sq. meters)
acceptance of Villonco Realty Company's offer to purchase the Buendia Avenue Pertinent parts of the offer are: (1) Offering to sell the Buendia property for
property, indubitably proves that there was a meeting of minds upon the subject P400.00 per square meter. (2) The sale is to be consummated only after
matter and consideration of the sale. Therefore, on that date the sale was Bormaheco has consummated purchase of another property located at Punta,
perfected. Cervantes also claims that it made a counter off that Villonco should Sta. Ana, Manila. The property mentioned in Bormaheco's letter was the land
have re-accepted, but the alleged changes or qualifications in the revised counter- of the National Shipyards & Steel Corporation (Nassco). At the bidding held
offer are not material or are mere clarifications of what the parties had previously on January 17, 1964 that land was awarded to Bormaheco, Inc., the highest
agreed upon, thus nullifying the need for another acceptance. Examples of the bidder, for the price of P552,000.
changes are, crossing out the word “Nassco” and putting “another property” 6. In the meanwhile, Bormaheco, Inc. and Villonco Realty Company continued
instead. Or putting the word “per annum” after the word interest did not make their negotiations for the sale of the Buendia Avenue property. As a result of
Cervantes letter a counter offer, insofar as the parties must have intended the a final conference Villonco Realty Company, through Teofilo Villonco, in its
interest to be per annum, or else it would have been an usurious interest. Further, letter of March 4, 1964 made a revised counter- offer for the purchase of the
Cervantes mislead Villonco that Bormahecho had the authority to sell the 3 lots, property. The counter-offer was accepted by Cervantes. Pertinent portions
when it fact it didn’t. Justice demands that Cervantes should be estopped from are: (1) Accepted P400.00 per sq.m, including improvements. (2) Deal
raising the defense of lack of authority to sell. cancelled if Nassco’s deal is not consummated.
DOCTRINE: It is true that an acceptance may contain a request for certain 7. Then, unexpectedly, in a letter dated March 30, 1964, or twenty-six days
changes in the terms of the offer and yet be a binding acceptance. 'So long as it is after the signing of the contract of sale, Cervantes returned the earnest
clear that the meaning of the acceptance is positively and unequivocally to accept money, with interest amounting to P694.24 (at ten percent per annum).
the offer, whether such request is granted or not, a contract is formed. Thus, it Cervantes cited as an excuse the circumstance that "despite the lapse of 45
was held that the vendor's change in a phrase of the offer to purchase, which days from February 12, 1964 there is no certainty yet" for the acquisition of
change does not essentially change the terms of the offer, does not amount to a the Punta property. Villonco Realty Company refused to accept the letter
rejection of the offer and the tender of a counter-offer and the checks of Bormaheco, Inc. Cervantes sent them by registered
FACTS: mail. When he rescinded the contract, he was already aware that the
Punta lot had been awarded to Bormaheco, Inc.
1. Francisco N. Cervantes and his wife, Rosario P. Navarra-Cervantes, are the
owners of 3 lots located at 245 Buendia Avenue, Makati, Rizal with a total 8. Villonco eventually filed a complaint for specific performance against
area of 3,500 square meters. The lots were mortgaged to the Development Bormaheco, Inc.
9. After trial, the lower court rendered a decision ordering the Cervantes spouses 5. It is safe to assume that the alleged changes or qualifications made by
to execute in favor of Bormaheco, Inc. a deed of conveyance for the three lots Cervantes were approved by Villonco Realty Company and that such
in question and directing Bormaheco, Inc. (a) to convey the same lots to approval was duly communicated to Cervantes or Bormaheco, Inc. by the
Villonco Realty Company, (b) to pay the latter, as consequential damages, broker as shown by the fact that Villonco Realty Company paid, and
the sum of P10,000 monthly from March 24, 1964 up to the consummation Bormaheco, Inc. accepted, the sum of P100,000 as earnest money or down
of the sale, (c) to pay Edith Perez de Tagle the sum of P42,000 as broker's payment. That crucial fact implies that Cervantes was aware that Villonco
commission and (d) pay P20,000 as to attorney's fees (Civil Case No. 8109). Realty Company had accepted the modifications which he had made in
Villonco's counter-offer. The fact that Villonco Realty Company allowed its
10. Hence this petition.
check to be cashed by Bormaheco, Inc. signifies that the company was in
conformity with the changes made by Cervantes and that Bormaheco, Inc.
was aware of that conformity.
ISSUES:
6. The truth is that the alleged changes or qualifications in the revised counter
1. (1) W/N that no contract of sale was perfected because Cervantes made a
— offer are not material or are mere clarifications of what the parties had
supposedly qualified acceptance of the revised offer contained in Exhibit D, previously agreed upon.
which acceptance amounted to a counter-offer, and because the condition that
Bormaheco, inc. would acquire the Punta land within the forty-five-day 7. Thus, Cervantes' alleged insertion in his handwriting of the figure and the
period was not fulfilled. No and No. words "12th and" in Villonco's counter-offer is the same as the statement
found in the voucher-receipt for the earnest money.
2. (2) W/N Francisco N. Cervantes, as president of Bormaheco, Inc., can sell
the 3 lots which are owned by the conjugal partnership. Yes. 8. Cervantes allegedly crossed out the word "Nassco" in paragraph 3 of
Villonco's revised counter-offer and substituted for it the word "another" so
RULING: The Court ordered Spouses Cervantes to convey the 3 lots to Bormaheco, that the original phrase, "Nassco's property in Sta. Ana", was made to read as
Inc. Then the Court ordered Bormaheco Inc. to sell the property to Villonco Realty "another property in Sta. Ana". That change is trivial. What Cervantes did
Company for P1,300,000.00, at the price of P400.00 square meter.
was merely to adhere to the wording of paragraph 3 of Bormaheco's original
offer (Exh. B) which mentions "another property located at Sta. Ana." His
obvious purpose was to avoid jeopardizing his negotiation with the Nassco
RATIO: for the purchase of its Sta. Ana property by unduly publicizing it.
1. We hold that the appeal, is devoid of merit. 9. Similarly, Cervantes' alleged insertion of the letters "PA" ( per annum) after
2. Bormaheco's acceptance of Villonco Realty Company's offer to purchase the word "interest" in that same paragraph 3 of the revised counter-offer (Exh.
the Buendia Avenue property, as shown in Teofilo Villonco's letter dated D) could not be categorized as a major alteration of that counter-offer that
March 4, 1964, indubitably proves that there was a meeting of minds prevented a meeting of the minds of the parties. It was understood that the
upon the subject matter and consideration of the sale. Therefore, on that parties had contemplated a rate of ten percent per annum since ten percent a
date the sale was perfected. Acceptance by Bormaheco of P100,000 shows month or semi-annually would be usurious.
sale was partially consummated subject to acquisition of Punta property. 10. Appellants Bormaheco, Inc. and Cervantes further contend that Cervantes, in
3. On February 18, 1964 Bormaheco's bid for the Punta property was already clarifying in the voucher for the earnest money of P100,000 that Bormaheco's
accepted by the Nassco which had authorized its General Manager to sign the acceptance thereof was subject to the terms and conditions embodied in
corresponding deed of sale. Bormaheco's letter of February 12, 1964 and your (Villonco's) letter of March
4, 1964" made Bormaheco's acceptance "qualified and conditional".
4. Bormaheco, Inc. and the Cervantes spouses contend that the sale was not
perfected because Cervantes allegedly qualified his acceptance of Villonco's 11. That contention is not correct. There is no incompatibility between
revised offer and, therefore, his acceptance amounted to a counter-offer Bormaheco's offer of February 12, 1964 and Villonco's counter-offer of
which Villonco Realty Company should accept but no such acceptance was March 4, 1964. The revised counter-offer merely amplified Bormaheco's
ever transmitted to Bormaheco, Inc. which, therefore, could withdraw its original offer. It is true that an acceptance may contain a request for certain
offer. That contention is not well-taken. No evidence as to what changes changes in the terms of the offer and yet be a binding acceptance.
were made by Cervantes. Also no evidence that Villonco did not assent to the 12. Thus, it was held that the vendor's change in a phrase of the offer to purchase,
changes. which change does not essentially change the terms of the offer, does not
amount to a rejection of the offer and the tender of a counter-offer (Stuart vs. properties of the Cervantes spouses. They aver that Cervantes in dealing with
Franklin Life Ins. Co., supra). the Villonco brothers acted as president of Bormaheco, Inc. and not in his
individual capacity and, therefore, he did not bind the conjugal partnership
13. Appellants' next contention is that the contract was not perfected because the
nor Mrs. Cervantes who was allegedly opposed to the sale.
condition that Bormaheco, Inc. would acquire the Nassco land within forty-
five days from February 12, 1964 or on or before March 28, 1964 was not 20. In truth, he concealed the fact that the three lots were registered in the name
fulfilled. That contention is predicated on the erroneous assumption that of the conjugal partnership. He certainly led the Villonco brothers to believe
Bormaheco, Inc. was to acquire the Nassco land within forty-five days or on that as president of Bormaheco, Inc. he could dispose of the said lots. The
or before March 28, 1964. pleadings disclose that Bormaheco, Inc. and Cervantes deliberately and
studiously avoided making the allegation that Cervantes was not authorized
14. The record does not support the theory of Bormaheco, Inc. and the Cervantes
by his wife to sell the three lots or that he acted merely as president of
spouses that the forty-five-day period was the time within which (a) the
Bormaheco, Inc.
Nassco property and two Pasong Tamo lots should be acquired, (b) when
Cervantes would secure his wife's consent to the sale of the three lots and (c) 21. It its first formal offer, Bormaheco, Inc. said it was the owner of the 3 lots in
when Bormaheco, Inc. had to decide what to do with the DBP encumbrance. question. However, it filed an amended answer dated May 25, 1964 wherein
it denied that it was the owner of the three lots. It revealed that the three lots
15. Cervantes in paragraph 3 of his offer of February 12, 1964 stated that the sale
"belong and are registered in the names of the spouses Francisco N. Cervantes
of the Buendia lots would be consummated after he had consummated the
and Rosario N. Cervantes."
purchase of the Nassco property. Then, in paragraph 5 of the same offer he
stated "that final negotiations on both properties can be definitely 22. In its affirmative defense, Bormaheco, Inc. pretended that it needed forty-
known after forty-five days.” five days within which to acquire the Nassco property and "to negotiate" with
the registered owner of the three lots. The absurdity of that pretension stands
16. It is deducible from the tenor of those statements that the consummation of
out in bold relief when it is borne in mind that the answers of Bormaheco,
the sale of the Buendia lots to Villonco Realty Company was conditioned on
Inc. were verified by Cervantes and that the registered owner of the three lots
Bormaheco's acquisition of the Nassco land. But it was not spelled out that
is Cervantes himself.
such acquisition should be effected within forty-five days from February 12,
1964. Had it been Cervantes' intention that the forty-five days would be the 23. Inasmuch as the sale was perfected and even partly executed, Bormaheco,
period within which the Nassco land should be acquired by Bormaheco, then Inc., and the Cervantes spouses, as a matter of justice and good faith, are
he would have specified that period in paragraph 3 of his offer. He could bound to comply with their contractual commitments.
have also specified that period in his "conforme" to Villonco's counter-offer
of March 4, 1964.
17. No such specification was made. The term of forty-five days was not a part
of the condition that the Nassco property should be acquired. It is clear that
the statement "that final negotiations on both property can be definitely
known after 45 days" simply is a surmise that after forty-five days it would
be known whether Bormaheco, Inc. would be able to acquire the Nassco
property and whether it would be able to sell the Buendia property.
18. It should be underscored that the condition that Bormaheco, Inc. should
acquire the Nassco property was fulfilled. As admitted by the appellants, the
Nassco property was conveyed to Bormaheco, Inc. on June 26, 1964. As early
as January 17, 1964 the property was awarded to Bormaheco, Inc. as the
highest bidder. It is reasonable to assume that had Cervantes been more
assiduous in following up the transaction, the Nassco property could have
been transferred to Bormaheco, Inc. on or before March 28, 1964, the
supposed last day of the forty-five-day period.
19. The appellants, in their fifth assignment of error, argue that Bormaheco, Inc.
cannot be required to sell the three lots in question because they are conjugal
FIRST OPTIMA REALTY CORP vs SECURITRON SECURITY despite the fact that they were still undecided whether to sell the property,
SERVICES, INC. (Abs) that no Board Resolution was issued that will authorize the sale, and that they
January 28, 2015 | Del Castillo, J. | Payment of the earnest money will not result in a have no Contract for the earnest money nor Contract to Sell the said property
perfected contract of sale when the owner did not agree to sell his property with Securitron.
6. Securitron, then, filed a case against First Optima for specific performance.
PETITIONER: First Optima Realty Corporation The RTC ruled in favor of Securitron on the ground that the acceptance of
RESPONDENTS: Securitron Security Services, Inc. the earnest money by First Optima indicated the existence of a perfected
contract of sale, that they were not forced to accept the earnest money, and
SUMMARY: Securitron offered to buy the subject property from First Optima, that Young was free to represent the corporation in negotiating with the
which the latter declined. Thereafter, Securitron sent a letter and a check to First respondent for the sale thereof, thus, not needing a Board Resolution for such
Optima saying that the check is an earnest money for the property. Said check transaction.
was eventually deposited to First Optima’s bank account. After some time, 7. First Optima then appealed to CA but it merely affirmed the decision of the
Securitron, through a demand letter, asked First Optima to proceed with the sale RTC.
of the property considering they accepted the earnest money. First Optima refused
saying that they did not agree yet to sell the property. The Court held that *see ISSUES:
Doctrine* 1. WoN the acceptance of the earnest money resulted in the perfection of a
contract of sale between the parties. –NO
DOCTRINE: In a potential sale transaction, prior payment of earnest money
even before the owner can agree to sell his property is irregular, and cannot be RULING: Wherefore, the Petition is GRANTED.
used to bind the owner to the obligations of a seller under an otherwise perfected
contract of sale. Property owner/prospective seller may not be legally obliged to RATIO:
enter into a sale with a prospective buyer through the latter's employment of The trial and appellate courts failed to appreciate that respondent’s offer to
questionable practices which prevent the owner from freely giving his consent to purchase the subject property was never accepted by the petitioner at any instance,
the transaction. even after negotiations were held between them. Thus, as between them, there is no
sale to speak of. "When there is merely an offer by one party without acceptance of
FACTS: the other, there is no contract."
1. Looking to expand its business, respondent Securitron, through its General Respondent’s subsequent sending of the February 4, 2005 letter and check to
Manager Eleazar, offered to purchase the subject property from petitioner petitioner – without awaiting the approval of petitioner’s board of directors and
First Optima. Young’s decision, or without making a new offer – constitutes a mere reiteration of its
2. Eleazar went to the office of the Executive VP of First Optima, Ms. Young, original offer which was already rejected previously; thus, petitioner was under no
with cash at hand, offering to buy the subject property. However, Young obligation to reply to the February 4, 2005 letter.
declined to accept the payment and told Eleazar that prior approval of the Since there is no perfected sale between the parties, respondent had no
petitioner’s Board of Directors was required for the transaction, to which obligation to make payment through the check; nor did it possess the right to deliver
Eleazar remark that they will instead await such approval. earnest money to petitioner in order to bind the latter to a sale. As contemplated under
3. On February 4, 2005, Securitron sent a Letter to the petitioner. This was Art. 1482 of the Civil Code, "there must first be a perfected contract of sale before we
accompanied by a check issued for 100,000php payable to the petitioner. The can speak of earnest money." "Where the parties merely exchanged offers and counter-
Letter states that the check was the earnest money for the subject property offers, no contract is perfected since they did not yet give their consent to such offers.
and that full payment will be made upon clearing of the tenants of the Earnest money applies to a perfected sale."
property and signing of the Deed of Sale. This Court is inclined to accept petitioner’s explanation that since the check
4. However, said Letter and check was not delivered directly to Young but was was mixed up with all other checks and correspondence sent to and received by the
instead coursed through a receptionist who then issued a Provisional Receipt. corporation during the course of its daily operations, Young could not have timely
The check was eventually deposited and credited to petitioner’s bank discovered respondent’s check payment; petitioner’s failure to return the purported
account. earnest money cannot mean that it agreed to respondent’s offer.
5. Thereafter, Securitron, through a demand letter, asked First Optima to In a potential sale transaction, the prior payment of earnest money even
proceed with the sale of the property. The petitioner replied stating that it was before the property owner can agree to sell his property is irregular, and cannot be
Securitron who offered to buy the property, that it tendered an earnest money used to bind the owner to the obligations of a seller under an otherwise perfected
contract of sale; to cite a well-worn cliché, the carriage cannot be placed before the
horse. The property owner-prospective seller may not be legally obliged to enter into
a sale with a prospective buyer through the latter’s employment of questionable
practices which prevent the owner from freely giving his consent to the transaction;
this constitutes a palpable transgression of the prospective seller’s rights of ownership
over his property, an anomaly which the Court will certainly not condone. An
agreement where the prior free consent of one party thereto is withheld or suppressed
will be struck down, and the Court shall always endeavor to protect a property owner’s
rights against devious practices that put his property in danger of being lost or unduly
disposed without his prior knowledge or consent. As this ponente has held before,
"[t]his Court cannot presume the existence of a sale of land, absent any direct proof of
it."
OESMER v. PARAISO DEVELOPMENT CORPORATION (Ram) FACTS:
February 5, 2007 | Chico-Nazario, J. | Earnest Money v. Option Money 1. Oesmers are co-owners of undivided shares of 2 parcels of agricultural and
tenanted land in Carmona Cavite, which are unregistered and originally
PETITIONER: Rizalino, substistuted by his heirs, Josefina Rolando and owned by their parents. When their parents died, they acquired the lots as
Fernando, Ernesto, Leonora, Bibiano, Jr., Librado and Enriqueta, all surnamed heirs by right of succession.
Oesmer 2. In 1989, Paular, a resident and former Mun. Sec. of Carmona Cavite, brought
RESPONDENTS: Paraiso Development Corporation Ernesto Oesmer (one of the heirs) to meet with Lee, President of Paraiso
Development Corp, in Manila for the purpose of brokering the sale of
SUMMARY: Oesmers are co-owners of undivided shares of 2 parcels of Ernesto's properties to Paraiso Dev. Corp.
agricultural and tenanted land in Carmona Cavite by virtue of right of succession. 3. A contract to sell was entered into between Paraiso Dev. Corp and Ernesto as
Ernesto Oesmer (one of the heirs) met with Lee, President of Paraiso well as Enriqueta. A check in the amount of P100,000 payable to Ernesto was
Development Corp, in Manila for brokering the sale of the properties. A contract given as “option money”. Eventually, Rizalino, Leonora, Bibiano Jr, and
to sell was entered into between Paraiso Dev. Corp and Ernesto as well as Librado also signed the Contract to Sell.
Enriqueta. A check in the amount of P100,000 payable to Ernesto was given as 4. However, 2 of their brothers, Adolfo and Jesus, refused to sign the document.
“option money”. Eventually, Rizalino, Leonora, Bibiano Jr, and Librado also 5. A couple of months after, the Oesmers informed Paraiso (through a letter)
signed the Contract to Sell. However, 2 of their brothers, Adolfo and Jesus, that it is rescinding the Contract to Sell and returning the option money.
refused to sign the document. A couple of months after, the Oesmers informed 6. However, Paraiso did not respond.
Paraiso (through a letter) that it is rescinding the Contract to Sell and returning 7. Oesmers filed a complaint for declaration of nullity of the Contract to Sell
the option money. Petitioners who signed the document are contending that with damages with the RTC, which ruled in favor of Paraiso Dev. Corp.
Ernesto had no written authority to sell the property, that their signatures merely a. Contract to Sell is valid and binding only to the undivided
conferred conditional consent subject to a suspensive condition (approval of the proportionate share of Ernesto who signed the document and
sale by all the co-owners) and that the document did not bear the signature of the received the check.
respondent corporation. Hence, the Contract to Sell was not valid. RTC ruled in b. Ernesto was ordered to execute the Contract of Absolute Sale as to
favor of Paraiso, but only as to Ernesto’s 1/8 share. CA modified and declared the his 1/8 share over the subject two parcels of land.
Contract to Sell as valid and binding to all the signatories of the document. The 8. On appeal, CA modified by declaring that the Contract to Sell is valid and
issue in this case is WoN the Contract to Sell is valid as to all co-owners and WoN binding as to the undivided shares of the six signatories of the document.
the P100,000 is considered as option money. The SC held that the Contract to Sell 9. Petitioners who signed the document are contending that:
was valid to all signatories, but not valid to those who did not. The co-owners a. Ernesto had no written authority to sell the property
who affixed their signatures signify their act of directly selling their personal b. Their signatures merely conferred conditional consent subject to a
shares to Paraiso Dev. Corp. and acceptance of what has been stipulated in the suspensive condition (approval of the sale by all the co-owners) .
contract. Moreover, a careful examination of the words used in the contract c. Document did not bear the signature of the respondent corporation.
indicates that the money is not option money but earnest money. It is shown by d. Hence, the Contract to Sell was not valid.
its partial performance of tendering the P100,000 as part of the purchase price.
ISSUES:
DOCTRINE: 1. WON the Contract to Sell is valid as to all co-owners? – NO. Valid to all
OPTION MONEY EARNEST MONEY those who signed it, but not valid to those who did not.
Part of the purchase price The money given as a distinct 2. ISSUE related to the syllabus: WoN the P100,000 is considered as option
consideration for an option contrac money? NO. It is not option money, but earnest money.
Given only where there is already a Applies to a sale not yet perfected
sale RULING: Wherefore, petition is DENIED. CA decision is affirmed.
When earnest money is given, the When the would-be buyer gives
buyer is bound to pay the balance option money, he is not required to RATIO:
buy, but may even forfeit it depending Affixing their Signatures
on the terms of the option. 1. It is true that the signatures of the 5 siblings did not confer authority on
Ernesto as agent to sell their respective shares in the properties, because such
authority to sell an immovable is required to be in writing.
2. However, those signatures signify their act of directly (not through an agent) referred to as “option money.” However, a careful examination of the words
selling their personal shares to Paraiso Dev. Corp. used in the contract indicates that the money is not option money but earnest
3. The Contract to Sell was perfected when the petitioners consented to the sale money.
to the respondent of their shares in the subject parcels of land by affixing 3. Settled is the rule that in the interpretation of contracts, the ascertainment of
their signatures on the said contract. the intention of the contracting parties is to be discharged by looking to the
4. Such signatures show their acceptance of what has been stipulated in the words they used to project that intention in their contract, all the words, not
Contract to Sell and such acceptance was made known to Paraiso just a particular word or two, and words in context, not words standing alone.
Development Corporation when the duplicate copy of the Contract to Sell 

was returned to the it bearing petitioners’ signatures.
5. It cannot also have been subject to a suspensive condition because the terms
of the Contract to Sell did not mention of any condition. It is a cardinal rule
in the interpretation of contracts that if the terms of a contract are clear and
leave no doubt upon the intention of the contracting parties, the literal
meaning of its stipulation shall control.
6. As to petitioner Enriqueta’s claim that she merely signed as a witness to the
said contract, the contract itself does not say so.
7. There was no single indication in the said contract that she signed the same
merely as a witness.
8. The fact that her signature appears on the right-hand margin of the Contract
to Sell is insignificant.
9. The contract indisputably referred to the “Heirs of Bibiano and Encarnacion
Oesmer,” and since there is no showing that Enriqueta signed the document
in some other capacity, it can be safely assumed that she did so as one of the
parties to the sale.

Co-ownership
1. The signatories being owners of their respective undivided shares in the
subject properties, can dispose of their shares even without the consent of all
the co-heirs.
2. Article 493. Each co-owner shall have the full ownership of his part and of
the fruits and benefits pertaining thereto, and he may therefore alienate,
assign or mortgage it, and even substitute another person in its enjoyment,
except when personal rights are involved. But the effect of the alienation or
the mortgage, with respect to the co-owners, shall be limited to the portion
which may be allotted to him in the division upon the termination of the
coownership.
3. Even without the consent of the two co-heirs, Adolfo and Jesus, the Contract
to Sell is still valid and binding with respect to the 6/8 proportionate shares
of the petitioners.

Option Money vs. Earnest Money


1. The Contract to Sell is not void merely because it does not bear the signature
of the respondent corporation. The corporation’s consent to be bound by the
terms of the contract is shown by its partial performance of tendering the
P100,000 as part of the purchase price.
2. The consideration of P100,000.00 paid by respondent to petitioners was
SPOUSES DALION V. CA (Nicolin) d. Dalion alleges that the suit was merely intended to harass, preempt
February 28, 1990 | Medialdea, J. | How Form is Important in Contracts of Sale and forestall Dalion’s threat to sue for the unpaid commissions.
2. Lower court ruled in favor of Sabesaje and ordered Spouses Dalion to deliver
PETITIONERS: SPS. SEGUNDO DALION AND EPIFANIA SABESAJE- the parcel of land and execute the corresponding formal deed of conveyance
DALION in a public document in favor of Sabesaje.
RESPONDENTS: THE HONORABLE COURT OF APPEALS AND RUPERTO 3. Dalion appealed the decision, but the CA upheld the validity of the sale of
SABESAJE, JR. the parcel of land between Dalion and Sabesaje. Hence, this petition.

SUMMARY: Ruperto Sabesaje sued to recover ownership of a parcel of land from ISSUES:
Segundo Dalion based on a private document of absolute sale executed between 1. W/N the contract of sale of a parcel of land is valid – YES. Authenticity of
them. Dalion denies the sale contending that the document is fictitious, her wife’s Dalion’s signature and genuineness of the document was proven.
signature was forged and that the land is conjugal property sold to them by 2. W/N a public document is necessary to transfer ownership – NO. Public
Saturnina Sabesaje. Spouses Dalion deny the allegation that they pleaded with instrument is only for convenience and not a requirement for the validity of
Sabesaje to continue administering the land after the execution of the sale They a contract of sale.
admitted, however, administrating 5 parcels of land of Sabesaje’s grandfather to
which they were entitled to 5% and 10% commission (which they never got). Trial RULING: ACCORDINGLY, the petition is DENIED and the decision of the Court
court ruled in favor of Sabesaje and ordered Dalion to deliver the land. On appeal, of Appeals upholding the ruling of the trial court is hereby AFFIRMED. No costs.
CA affirmed. Dalion now contests the validity of the sale and the necessity of a
public document to effect ownership. SC held that the sale was valid as several RATIO:
witness attest to the authenticity of the document. Furthermore, a public document 1. SC upheld the findings and ruling
was not necessary for validity or enforceability of a sale. of CA:
a. Exhibit A and B have the
DOCTRINE: The provisions of Art. 1358 on the necessity of a public document same description of the
is only for convenience, not for validity or enforceability. It is not a requirement subject land (boundaries
for the validity of a contract of sale of a parcel of land that this be embodied in a delineating the property
public instrument. from adjacent lots are
identical)
b. Witnesses positively testified to the authenticity of the execution of
the subject deed.
FACTS: c. Dalion never presented any witness or evidence to prove his claim
1. May 28, 1973, Ruperto Sabesaje (Sabesaje) sued to recover ownership of a of forgery.
parcel of land based on a private document of absolute sale aka Exhibit A d. The questioned signatures and the specimens are very similar to
(date July 1, 1965), allegedly executed by Dalion each other and appear to be written by one person. It may be noted
a. Dalion denied the sale contending that the document is 1) fictitious that two signatures of Segundo D. Dalion appear on the face of the
and that 2) his wife’s signature was forged, 3) that the land is questioned document (Exh. A), one at the right corner bottom of the
conjugal property, which he and his wife acquired in 1960 from document (Exh. A-2) and the other at the left hand margin thereof
Saturnina Sabesaje (evidenced by the Escritura de Venta (Exh. A-3). The second signature is already a surplusage. A forger
Absoluta aka Exhibit B) would not attempt to forge another signature, an unnecessary one,
b. Dalion also denied Sabesaje’s claims that the Spouses Dalion for fear he may commit a revealing error or an erroneous stroke
pleaded with them to still administer the land (even after the sale) 2. Dalion’s ground that a private document does not convey title or right to the
because they did not have any means of livelihood. lot in question is misplaced. The provisions of Art. 1358 on the necessity of
c. Dalion, however, admitted that they have been administering 5 a public document is only for convenience, not for validity or enforceability.
parcels of land owned by Sabesaje’s grandfather, Leonardo. (but It is not a requirement for the validity of a contract of sale of a parcel of land
they didn’t receive the agreed commission of 10% and 15% of the that this be embodied in a public instrument.
sales of the products) a. A contract of sale is a consensual contract, which means that the sale
is perfected by mere consent. No particular form is required for its
validity. Upon perfection of the contract, the parties may
reciprocally demand performance (Art. 1475, NCC), i.e., the vendee
may compel transfer of ownership of the object of the sale, and the
vendor may require the vendee to pay the thing sold (Art. 1458,
NCC).
b. Under Art. 1498, NCC, when the sale is made through a public
instrument, the execution thereof is equivalent to the delivery of the
thing. Delivery may either be actual (real) or constructive. Thus
delivery of a parcel of land may be done by placing the vendee in
control and possession of the land (real) or by embodying the sale
in a public instrument (constructive).
c. Proper action is recovery of ownership. Sabesaje’s complaint
sufficiently alleged a cause of action to compel Dalion to execute a
formal deed of sale, and the suit for recovery of ownership merely
seeks consummation of said contract.
SECUYA V. SELMA (Josef) Sabellona, whereby the former bound herself and parted with one-third (1/3)
February 22, 2002 | Panganiban J. | Importance of Form in Contracts of Sale in portion of Lot 5679 in favor of the latter.
order to bind third parties 4. Paciencia Sabellona took possession and occupation of that one-third portion
PETITIONER: BENIGNA SECUYA, MIGUEL SECUYA, MARCELINO of Lot 5679 adjudicated to her (however, the latter and her successors-in-
SECUYA, CORAZON SECUYA, RUFINA SECUYA, BERNARDINO SECUYA, interest did nothing to assert their right like registering the said
NATIVIDAD SECUYA, GLICERIA SECUYA and PURITA SECUYA Agreement of Partition, and thus it was sold to a certain Silvestre Aro by
RESPONDENT: GERARDA M. VDA. DE SELMA the heirs of Maxima – relevant to fact no. 8). Later, she sold the three
thousand square meter portion thereof to Dalmacio Secuya, the precedessor-
SUMMARY: Maxima Caballero executed a document entitled "Agreement of in-interest of petitioners, on October 20, 1953, for a consideration of ONE
Partition," wherein she stipulated to transfer one-third (1/3) of the lot to and accepted by THOUSAND EIGHT HUNDRED FIFTY PESOS (P1,850.00), by means of
Paciencia Sabellano (aunt). Paciencia thereafter sold the same to Dalmacio Secuya, a private document which was lost. Such sale was admitted and confirmed
which was embodied in a private instrument. The document was, however, lost. by Ramon Sabellona, only heir of Paciencia Sabellona per that instrument
After the purchase of Dalmacio Secuya, the latter, together with his relatives took denominated CONFIRMATION OF SALE OF UNDIVIDED SHARES.
physical possession of the land. Thereafter, the defendant-respondent Gerarda Selma 5. After the purchase by Dalmacio Secuya of the property in litigation on
alleged that she bought a lot, in which the land of the petitioners was included. October 20, 1953, Dalmacio, together with his brothers and sisters, took
Petitioners, heirs of Dalmacio Secuya, filed an action for quieting of title on the ground physical possession of the land and cultivated the same. Dalmacio Secuya
that respondent's title is a cloud on their title as owners and possessors of the property died on November 20, 1961. Thus his heirs — brothers, sisters, nephews and
subject of litigation. Defendant-respondent alleged that she bought the property from nieces — are the plaintiffs in Civil Case No. CEB-4247 and now the
Cesaria Caballero, who was the widow of the Silvestre Aro, the said registered owner of petitioners;
the lot. The trial court rendered judgment against the petitioners. CA affirmed. 6. In 1972, defendant-respondent Gerarda Selma bought a 1,000 square-meter
portion of Lot 5679. Then on February 19, 1975, she bought the bigger bulk
The main issue is whether or not the petitioners have the requisite title that would enable of Lot 5679, consisting of 9,302 square meters. The land in question, a
them to avail of the remedy of Quieting of Title. The Court held no. Petitioners insist 3,000-square meter portion of Lot 5679, is embraced and included within
that Paciencia sold the disputed property to Dalmacio Secuya (predecessor-in-interest of the boundary of the later acquisition by respondent Selma.
petitioners), and that the sale was embodied in a private document. However, such 7. Defendant-respondent Gerarda Selma lodged a complaint, and had the
document, which would have been the best evidence of the transaction, was never plaintiffs-petitioners summoned, before the Barangay Captain of the place,
presented in court, allegedly because it had been lost. While a sale of a piece of land and in the confrontation and conciliation proceedings at the Lupong
appearing in a private deed is binding between the parties, it cannot be considered Tagapayapa, defendant-respondent Selma was asserting ownership over the
binding on third persons, if it is not embodied in a public instrument and recorded in the land inherited by plaintiffs-petitioners from Dalmacio Secuya of which they
Registry of Property. (See Issues and Ratio part below to also understand how the SC had long been in possession . . . in concept of owner.
resolved the other issues to solve the main issue) 8. Respondent Selma's version of the facts, on the other hand, was summarized
by the appellate court as follows:
DOCTRINE: • She is the registered owner of Lot 5679-C-120 consisting of 9,302
While sale of land appearing in a private deed is binding between the parties, it cannot square meters, having bought the same sometime in February 1975
be considered binding on third persons, if it is not embodied in a public instrument and from Cesaria Caballero and have been in possession of the same
recorded in the Registry of Deeds. since then. Cesaria Caballero was the widow of Silvestre Aro,
registered owner of the mother lot, Lot. No. 5679 with an area of
Facts: 12,750 square meters of the Talisay-Minglanilla Friar Lands Estate,
1. The present Petition is rooted in an action for quieting of title filed before the as shown by Transfer Certificate of Title No. 4752. Upon Silvestre
RTC by the Secuyas against Gerarda M. vda. de Selma. Aro's demise, his heirs executed an "Extrajudicial Partition and
2. The parcel of land subject of this case is a PORTION (3000 sqm.) of Lot 5679 Deed of Absolute Sale" wherein one-half plus one-fifth of Lot No.
of the Talisay-Minglanilla Friar Lands Estate. The property was originally 5679 was adjudicated to the widow, Cesaria Caballero, from whom
sold to Maxima Caballero Vda. de Cariño. Lot 5679 has an area of 12,750 defendant-appellee derives her title.
square meters, more or less. 9. RTC: Dismissed petitioner’s claim of ownership
3. During the lifetime of Maxima Caballero, she entered into that CA: Affirmed RTC
AGREEMENT OF PARTITION dated January 5, 1938 with Paciencia
• It held that respondent's title can be traced to a valid TCT. On the other parties were not co-owners. Rather, it is in the nature of a trust
hand, it ruled that petitioners anchor their claim on an "Agreement of agreement.
Partition" which is void for being violative of the Public Land Act. • As a result of the Agreement, Maxima Caballero held the portion
The CA noted that the said law prohibited the alienation or specified therein as belonging to Paciencia Sabellona when the
encumbrance of land acquired under a free patent or homestead application was eventually approved and a sale certificate was issued in
patent, for a period of five years from the issuance of the said patent. her name. Thus, she (Maxima Caballero) should have transferred the
same to the latter, but she never did so during her lifetime. Instead,
Ruling: WHEREFORE, the Petition is hereby DENIED and the assailed Decision her heirs sold the entire Lot No. 5679 to Silvestre Aro in 1955.
AFFIRMED. • From 1954 when the sale certificate was issued until 1985 when
petitioners filed their Complaint, Paciencia and her successors-in-
Issues: interest did not do anything to enforce their proprietary rights over
Main issue: W/N the petitioners have the requisite title that would enable them the disputed property or to consolidate their ownership over the
to avail of the remedy of Quieting of Title – NO same. In fact, they did not even register the said Agreement with the
In determining w/n the petitioners have the requisite title – a legal or an equitable Registry of Property or pay the requisite land taxes.
title to, or an interest in, the subject real property – that would enable them to avail
themselves of the remedy of Quieting of Title, the Court finds it relevant to resolve the b) The Repudiation of the Express Trust
following issues below: • While no time limit is imposed for the enforcement of rights under
1. What is the real nature of the “Agreement to Partition?” – Express Trust express trusts, prescription may, however, bar a beneficiary's action
2. W/N the deed of sale to Dalmacio Secuya is valid – YES, but not to third for recovery, if a repudiation of the trust is proven by clear and
persons because it was in a private document. (Relevant topic) convincing evidence and made known to the beneficiary.
3. W/N the private respondent has valid title – YES • There was a repudiation of the express trust when the heirs of
Maxima Caballero failed to deliver or transfer the property to
Ratio: Paciencia Sabellona, and instead sold the same to a third person not
First Issue: 
The Real Nature of the "Agreement of Partition" privy to the Agreement. In the memorandum of incumbrances of TCT
No. 308719 issued in the name of Maxima, there was no notation of the
AGREEMENT OF PARTITION Agreement between her and Paciencia. Equally important, the
Agreement was not registered; thus, it could not bind third persons.
I, MAXIMA CABALLERO, Filipina, of legal age, married to Rafael Cariño, now Neither was there any allegation that Silvestre Aro, who purchased
residing and with postal address in the Municipality of Dumaguete, Oriental Negros, the property from Maxima's heirs, knew of it. Consequently, the
depose the following and say: subsequent sales transactions involving the land in dispute and the titles
(Relevant provisions) covering it must be upheld, in the absence of proof that the said
1. That I am the applicant of vacant lot No. 5679 of the Talisay-Minglanilla Estate transactions were fraudulent and irregular.
and the said application has already been indorsed by the District Land Officer,
Talisay, Cebu, for private sale in my favor; Second Issue: The Purported Sale to Dalmacio Secuya
4. That as soon as the application is approved by the Director of Lands, Manila,
in my favor, I hereby bind myself to transfer the one-third (l/3) portion of the a) The Absence of the Purported Deed of Sale (MOST relevant)
above mentioned lot in favor of my aunt, Paciencia Sabellana y Caballero, of legal • Petitioners insist that Paciencia sold the disputed property to Dalmacio
age, single, residing and with postal address in Tungkop, Minglanilla, Cebu. Secuya on October 20, 1953, and that the sale was embodied in a
6. I, Paciencia Sabellana y Caballero, hereby accept and take the portion herein private document. However, such document, which would have been
adjudicated to me by Mrs. Maxima Caballero of Lot No. 5679 Talisay-Minglanilla the best evidence of the transaction, was never presented in court,
Estate and will pay the corresponding portion to the government after the subdivision allegedly because it had been lost. While a sale of a piece of land
of the same; appearing in a private deed is binding between the parties, it cannot
be considered binding on third persons, if it is not embodied in a
a) The Agreement: An Express Trust, Not a Partition public instrument and recorded in the Registry of Property.
• Notwithstanding its purported nomenclature, this Agreement is not one
of partition, because there was no property to partition and the b) The Questionable Value of the Deed Executed by Ramon Sabellona
• To prove the alleged sale of the disputed property to Dalmacio,
petitioners instead presented the testimony of Miguel Secuya, one of the
petitioners; and a Deed confirming the sale executed by Ramon
Sabellona, Paciencia's alleged heir. The testimony of Miguel was a bare
assertion that the sale had indeed taken place and that the document
evidencing it had been destroyed. While the Deed executed by Ramon
ratified the transaction, its probative value is doubtful. His status as
heir of Paciencia was not affirmatively established. Moreover, he was
not presented in court and was thus not quizzed on his knowledge — or
lack thereof — of the 1953 transaction.

c) Petitioners' Failure to Exercise Owners' Rights to the Property


• Petitioners insist that they had been occupying the disputed
property for forty-seven years before they filed their Complaint for
quieting of title. However, there is no proof that they had exercised
their rights and duties as owners of the same. They argue that they
had been gathering the fruits of such property; yet, it would seem that
they had been remiss in their duty to pay the land taxes. If petitioners
really believed that they owned the property, they have should have been
more vigilant in protecting their rights thereto. As noted earlier, they
did nothing to enforce whatever proprietary rights they had over
the disputed parcel of land.

Third Issue: 
The Validity of Private Respondent's Title

• Petitioners contention: private respondent is not a purchaser in good faith


alleging that the latter was aware of the petitioner’s possession of the disputed
land. Thus, they cannot be entitled to the protection under the Torrens title.
• SC: Granting arguendo that private respondent knew that petitioners, through
Superales and his family, were actually occupying the disputed lot, we must
stress that the vendor, Cesaria Caballero, assured her that petitioners
were just tenants on the said lot. Private respondent cannot be faulted for
believing this representation, considering that petitioners' claim was not
noted in the certificate of the title covering Lot No. 5679.
• Moreover, the lot, including the disputed portion, had been the subject of
several sales transactions. The title thereto had been transferred several
times, without any protestation or complaint from the petitioners. In any
case, private respondent's title is amply supported by clear evidence, while
petitioners' claim is barren of proof.
YUVIENCO v. DACUYCUY (Mark) complaint states no cause of action and/or that the claim alleged therein is
May 27, 1981| Barredo J. | Perfection of sale unenforceable under the Statute of Frauds.
7. Petitioners argue that although they did express willingness to sell to private
PETITIONER: Suga Sotto Yuvienco, Britania Sotto, and Marcelino Sotto respondents the subject property, the respondents' reply that they were
RESPONDENTS: Hon. Auxencio C. Dacuycuy, (judge of the CFI of Leyte) Dely agreeable was not absolute, so much so that when Atty. Gamboa went to
Rodriquez et al. Cebu City with a prepared and duly signed contract for the purpose of
SUMMARY: Petitioners own a property in Tacloban City which they intend to sell perfecting and consummating the transaction, respondents and said
for 6.5M. They gave the respondents the right to purchase the property nut only representative found variance between the terms of payment stipulated in the
until July 31, 1978. Respondents replied that they agree to buy the property and prepared document returned and the document remained unsigned by
they will negotiate for details. Petitioner sent another telegram informing respondents.
respondents that their proposal is accepted and a contract will be prepared. Lawyer
of petitioners, Atty. Gamboa, arrived bringing a contact with an altered mode of ISSUES:
payment which says that the balance payment should be paid withing 30 days 1. WoN the complaint in controversy states sufficiently a cause of action. -NO
instead of the former 90 days. (Alleged original terms: 2M payment upon execution. 2. WoN there is perfected contract of sale between petitioners and Yao King
4.5M after 90 days). The issue is WON there was already a perfected contract of Ong. - NO
sale between the parties.
SC held that there was no perfected contract of sale yet because both parties are RULING: ACCORDINGLY, the impugned orders of respondent judge of November
still under negotiation and hence, no meeting of the minds. Atty. Gamboa even 2, 1978 and August 29, 1980 are hereby set aside and private respondents' amended
went to the respondents to negotiate for the sale. Even though there was an complaint, Annex A of the petition, is hereby ordered dismissed and the restraining
agreement on the terms of payment, there was no absolute acceptance because order heretofore issued by this Court on October 7, 1980 is declared permanent. Costs
respondents still insisted on further details. against respondents.

DOCTRINE: We hold that in any sale of real property on installments, the idea of RATIO:
payment on installments must be in the requisite of a note or memorandum.
1. In this respect, the governing legal provision is, of course, Article 1319 of
FACTS: the Civil Code which provides:
1. Petitioners own a property in Tacloban that they offered to sell for P6,500,000
to private respondents through a letter. ART. 1319. Consent is manifested by the meeting of the offer and the
2. They gave the respondents Yao King Ong the right to purchase the property acceptance upon the thing and the cause which are constitute the contract.
only until July 31, 1978. The offer must be certain the acceptance absolute. A qualified acceptance
3. Respondents replied that they agree to buy the property and they will constitute a counter-offer.
negotiate for details.
4. Petitioner sent another telegram informing respondents that their proposal is Acceptance made by letter or telegram does not bind offerer except from the
accepted and a contract will be prepared. time it came to his knowledge. The contract, in a case, is presumed to have
5. Respondent allege that Atty of the petitioners, Pedro Gamboa arrived been entered into in the place where the offer was made.
Tacloban City bringing with him the prepared contract to purchase and to sell
however, to the complete surprise of respondents, the petitioner without
2. In the instant case, We can lay aside, for the moment, petitioners' contention
giving notice to plaintiffs, changed the mode of payment with respect to the
that the letter of July 12, 1978 of Atty. Pedro C. Gamboa to respondents Yao
balance of P4,500,000.00 by imposing upon plaintiffs to pay same amount
King Ong and his companions constitute an offer that is "certain", although
within thirty (30) days from execution of the contract instead of the former
the petitioners claim that it was a mere expression of willingness to sell the
term of ninety (90) days
subject property and not a direct offer of sale to said respondents. What We
6. Petitioners applied for Petition for certiorari and prohibition to declare void
consider as more important and truly decisive is what is the correct juridical
for being in grave abuse of discretion the orders of respondent judge which
significance of the telegram of respondents instructing Atty. Gamboa to
denied the motion filed by petitioners to dismiss the complaint of private
"proceed to Tacloban to negotiate details." We underline the word
respondents for specific performance of an alleged agreement of sale of real
"negotiate" advisedly because to Our mind it is the key word that
property, the said motion being based on the grounds that the respondents'
negates and makes it legally impossible for Us to hold that respondents'
acceptance of petitioners' offer, assuming that it was a "certain" offer Separate Opinions
indeed, was the "absolute" one that Article 1319 above-quoted requires.
3. Importantly, it must be borne in mind that Yao King Ong's telegram simply AQUINO, J., concurring:
says "we agree to buy property". It does not necessarily connote acceptance
of the price but instead suggests that the details were to be subject of I concur in the result. Private respondents cannot prove any perfected sale which they
negotiation. can enforce.
4. Respondents now maintain that what the telegram refers to as "details" to be
"negotiated" are mere "accidental elements", not the essential elements of the (ITO NA YUNG BUONG CONCURRING NIYA HAHAHAHA)
contract. But to Our mind such alleged facts precisely indicate the failure of
any meeting of the minds of the parties, and it is only from the letter and
telegrams above-quoted that one can determine whether or not such meeting
of the minds did materialize.
5. The 90-day term for the balance of P4.5 M insisted upon by respondents
choices not appear in any note, writing or memorandum signed by either the
petitioners or any of them, not even by Atty. Gamboa.
6. Hence, looking at the argument of respondents that there was a perfected
agreement of purchase and sale between them and petitioners under which
they would pay in installments of P2 M down and P4.5 M within ninety 90)
days afterwards it is evident that such oral contract involving the "sale of real
property" comes squarely under the Statute of Frauds
7. We hold that in any sale of real property on installments, the Statute of Frauds
read together with the perfection requirements of Article 1475 of the Civil
Code must be understood and applied in the sense that the idea of payment
on installments must be in the requisite of a note or memorandum therein
contemplated.
8. Secondly, We are of the considered opinion that under the rules on proper
pleading, the ruling of the trial court that, even if the allegation of the
existence of a sale of real property in a complaint is challenged as barred from
enforceability by the Statute of Frauds, the plaintiff may simply say there are
documents, notes or memoranda without either quoting them in or annexing
them to the complaint, as if holding an ace in the sleeves is not correct.
9. Such a procedure is to tolerate and even encourage undue delay in litigation,
for the simple reason that to await the stage of trial for the showing or
presentation of the requisite documentary proof when it already exists and is
asked to be produced by the adverse party would amount to unnecessarily
postponing.
10. It would be inimical to the public interests in speedy justice for plaintiff to
play hide and seek at his own convenience, particularly, when, as is quite
apparent as in the instant case that chances are that there are no more writings,
notes or memoranda of the installment agreement alleged by respondents.
11. We consider it as sufficiently a grave abuse of discretion warranting the
special civil actions herein the failure of respondent judge to properly apply
the laws on perfection of contracts in relation to the Statute of Frauds and the
pertinent rules of pleading and practice
ORTEGA V. LEONARDO (Regine) 4. Leonardo also asserted a similar right, alleging occupancy of a portion of the
May 28, 2956 | Bengzon, J. | Partial Performance land subsequent to Ortega’s.
5. During the investigation of such conflicting interests, Leonardo asked Ortega
to desist from pressing her claim and definitely promised that if and when he
PETITIONER: MARTA C. ORTEGA succeeded in getting title to Lot I , he would sell to her a portion thereof with
RESPONDENTS: DANIEL LEONARDO an area of 55.60 square meters (particularly described) at the rate of P25.00
per square meter, provided she paid for the surveying and subdivision of the
SUMMARY: Long before and until her house had been completely destroyed Lot and provided further that after he acquired title, she could continue
during the liberation of the City of Manila, Ortega occupied a parcel of land, holding the lot as tenant by paying a monthly rental of P10.00 until said
designated as Lot I, Block 3 located at San Andres Street, Malate, Manila. After portion shall have been segregated and the purchase price fully paid.
liberation she re-occupied it. When the administration and disposition of the said 6. Ortega accepted Leonardo’s offer, and desisted from further claiming Lot I.
lot was assigned by the government to Rural Progress Administration, Leonardo 7. Leonardo finally acquired title thereto.
asserted the same right. Leonardo asked Ortega to desist from pressing her claim 8. Relying upon their agreement, Ortega caused the survey and segregation of
and promised that if he would succeed in getting the title to Lot I, he would sell the portion which Leonardo had promised to sell incurring expenses therefor,
to her a portion (55.60 sqm at P25 per sqm). His other conditions included that said portion being now designated as Lot I-B in a duly prepared and approved
she paid for the surveying and subdivision and that she continue as tenant with a subdivision plan; that in remodelling her son's house constructed on a lot
monthly rental of P10 until said portion shall have been segregated and the adjoining Lot I she extended it over said Lot I-B.
purchase price fully paid. Ortega accepted the offer and desisted from the claim. 9. After Leonardo had acquired Lot I, Ortega regularly paid him the monthly
After the plans of subdivision and segregation had been approved by the Bureau rental of P10.00.
of Lands, Ortega tendered to Leonardo the purchase price which the other refused 10. After the plans of subdivision and segregation of the lot had been approved
without reason. Ortega had already caused a survey and segregation of the portion by the Bureau of Lands, Ortega tendered to Leonardo the purchase price
of the land they agreed upon, and in fact extended a portion of her son’s house which the latter refused to accept, without cause or reason.
into the segregated portion. The CFI explained that an oral agreement to sell a 11. Ortega sought to compel Leonardo to comply with their oral contract of sale
piece of land is not enforceable. However, the SC said that there was an of a parcel of land.
enforceable contract because there was partial performance. Ortega made 12. CFI of Manila order the dismissal of the complaint and ruled that oral
substantial improvements on the lot, desisted from the claim, continued agreement to sell a piece of land is not enforceable.
possession, paid for surveying and paid for the rentals. It is enough to hold that 13. Ortega, however, argues that the contract in question, although verbal, was
the combination of all of these acts amounted to partial performance. It would be partially performed because she desisted from claiming the portion of lot I in
a fraud upon the plaintiff if the defendant were permitted to oppose performance question due to the promise of Leonard to transfer said portion to her after
of his part after he has allowed or induced the former to perform in reliance upon the issuance of title to defendant.
the agreement. 14. Hence, this appeal.

DOCTRINE: It is enough to hold that relinquishment of rights, continued ISSUE/s:


possession, building of improvements and tender of payment amounted to partial 1. Whether or not there was an enforceable contract. YES.
performance. Thus, making a contract enforceable.
FACTS: RULING: The judgment will accordingly be reversed and the record remanded for
1. Long before and until her house had been completely destroyed during the further proceedings. With costs against appellee.
liberation of the City of Manila, Ortega occupied a parcel of land, designated
as Lot 1, Block 3 etc. (hereinafter called Lot I) located at San Andres Street, RATIO:
Malate, Manila. 1. If the above means that partial performance of a sale contract
2. After liberation she re-occupied it. occurs only when part of the purchase price is paid, it surely constitutes a
3. When the administration and disposition of the said Lot I (together with other defective statement of the law.
lots in the Ana Sarmiento Estate) were assigned by the Government to the 2. American Jurisprudence in its title "Statute of Frauds" lists other acts of
Rural Progress Administration, Ortega asserted her right thereto (as occupant) partial performance, such as possession, the making of improvements,
for purposes of purchase. rendition of services, payment of taxes, relinquishment of rights, etc.
3. Thus, it is stated that "The continuance in possession may, in a proper case,
be sufficiently referable to the parol contract of sale to constitute a part
performance thereof.
4. Continued possession under an oral contract of sale, by one already in
possession as a tenant, has been held a sufficient part performance, where
accompanied by other acts which characterize the continued possession and
refer it to the contract of purchase.
5. It is also stated that "The making of valuable permanent improvements on
the land by the purchaser, in pursuance of the agreement and with the
knowledge of the vendor, has been said to be the strongest and the most
unequivocal act of part performance by which a verbal contract to sell land is
taken out of the statute of frauds, and is ordinarily an important element in
such part performance.
6. Again, it is stated that "A tender or offer of payment, declined by the
vendor, has been said to be equivalent to actual payment, for the purposes of
determining whether or not there has been a part performance of the contract.
7. And the relinquishment of rights or the compromise thereof has likewise
been held to constitute part performance.
8. It would appear that the complaint in this case described several circumstance
indicating partial performance: relinquishment of rights, continued
possession, building of improvements, tender of payment plus the surveying
of the lot at Ortega's expense and the payment of rentals.
9. It is enough to hold that the combination of all of them amounted to
partial performance; and we do so line with the accepted basis of the
doctrine, that it would be a fraud upon Ortega if the Leonardo were permitted
to oppose performance of his part after he has allowed or induced the former
to perform in reliance upon the agreement.
10. Hence, as there was partial performance, the principle excluding parol
contracts for the sale of realty, does not apply.
Claudel v. CA (Izzy) obtained the corresponding TCTs on their shares.
July 12, 1991 | Sarmiento, J. | Partial execution - Statute of Frauds devised to protect 6. Four years later, on December 7, 1976, private respondents SIBLINGS OF
parties in contracts of sale of real property CECILIO, filed Civil Case No. 5276-P as already adverted to at the outset,
with the then Court of First Instance of Rizal, a “Complaint for Cancellation
of Titles and Reconveyance with Damages,” alleging that 46 years earlier, or
PETITIONER: Heirs of Cecilio (also known as Basilio) Claudel
sometime in 1930, their parents had purchased from the late Cecilio Claudel
RESPONDENTS: Court of Appeals, Heirs of Macario, Esperidiona, Raymond
several portions of Lot No. 1230 for the sum of P30.00.
and Celestina Claudel
7. They admitted that the transaction was verbal. However, as proof of the sale,
the SIBLINGS OF CECILIO presented a subdivision plan of the said land,
SUMMARY: Cecilio Claudel acquired a lot from the Bureau of Lands. He
dated March 25, 1930, indicating the portions allegedly sold to the
occupied the same, declared it in his name and dutifully paid his taxes. After his
SIBLINGS OF CECILIO.
death, his heirs and siblings contested each other claiming ownership thereof. It
8. CFI of Rizal dismissed the complaint, disregarding the subdivision plan
was his heirs who were in possession of the property. They partitioned it amongst
presented by SIBLINGS OF CECILIO
themselves, registered each portion under the Torrens System, and each paid their
a. Complaint was filed in the names of the respondents (as in the
respective taxes. However, SIBLINGS OF CECILIO filed a case for cancellation
summary box), without naming the different heirs involved who
of titles and reconveyance arguing that there was a verbal sale between Cecilio
wish to recover the lots
and their parents over the lot 46 years ago. As evidence, they presented a
b. Court would not be able to apportion the property to the real party
subdivision plan. CA ordered the cancellation of the TCTs in favor of the heirs.
in interest if ever they are entitled to it as the persons indicated
The issue in the case is WoN the sale between Cecilio and his siblings can be
therein is in generic term
proven orally. The Court ruled in the negative stating that even though no form is
c. But most important of all the plaintiffs failed to present any
required in contracts of sale, the heirs had a right to rely upon their Torrens titles
document evidencing the alleged sale of the property to their
which are more credible than the subdivision plans, as protection provided by the
predecessors in interest by the father of the defendants.
Statute of Frauds.
d. Considering that the subject mater of the supposed sale is a real
property the absence of any document evidencing the sale would
DOCTRINE: The purpose of the Statute of Frauds is to prevent fraud and perjury
preclude the admission of oral testimony (Statute of Frauds).
in the enforcement of obligations depending for their evidence upon the
e. Moreover, considering also that the alleged sale took place in 1930,
unassisted memory of witnesses by requiring certain enumerated contracts and
the action filed by the plaintiffs herein for the recovery of the same
transactions to be evidenced in writing.
more than thirty years after the cause of action has accrued has
already prescribed.
9. SIBLINGS OF CECILIO appealed to the CA, and the CA reversed the
FACTS: decision of the trial court
1. December 28, 1922 - Basilio acquired from the Bureau of Lands, Lot No. a. The failure to bring and prosecute the action in the name of the real
1230 of the Muntinlupa Estate Subdivision, located in Muntinlupa, Rizal party in interest, namely the parties themselves, was not a fatal
2. He secured a Transfer Certificate of Title (TCT) No. 7471 issued by the omission since the court a quo could have adjudicated the lots to the
Registry of Deeds for the Province of Rizal in 1923; he also declared the lot SIBLINGS OF CECILIO, the parents of the herein respondents,
in his name, the latest Tax Declaration being No. 5795. leaving it to them to adjudicate the property among themselves.
3. He dutifully paid the real estate taxes until his death in 1937. Thereafter, his b. The fact of residence in the disputed properties by the herein
widow Basilia and later, her son Jose (one of the petitioners) paid the taxes. respondents had been made possible by the toleration of the
4. Two branches of Cecilio’s family contested the ownership over the land—on deceased Cecilio.
one hand the children of Cecilio, namely, Modesta, Loreta, Jose, Benjamin, c. The Statute of Frauds applies only to executory contracts and not to
Pacita, Carmelita, Roberto, Mario, Leonardo, Nenita, Arsenia Villalon, and consummated sales as in the case at bar where oral evidence may be
Felisa Claudel, and their children and descendants, now the herein petitioners admitted as cited in Iñigo v. Estate of Magtoto7 and Diana, et al. v.
(hereinafter referred to as HEIRS OF CECILIO), and on the other, the brother Macalibo.
and sisters of Cecilio, namely, Macario, Esperidiona, Raymunda, and d. The defense of prescription cannot be set up against the herein
Celestina and their children and descendants, now the herein private petitioners despite the lapse of over forty years from the time of the
respondents (hereinafter referred to as SIBLINGS OF CECILIO). alleged sale in 1930 up to the filing of the “Complaint for
5. In 1972, HEIRS OF CECILIO partitioned this lot among themselves and
Cancellation of Titles and Reconveyance x x x” in 1976. provisions of the Statute of Frauds pertinent to the present controversy, state:
10. According to the CA, the action was not for the recovery of possession of real a. Art. 1403 (Civil Code). The following contracts are unenforceable,
property but for the cancellation of titles issued to the HEIRS OF CECILIO unless they are ratified: xxx xxx xxx 2) Those that do not comply
in 1973. SIBLINGS OF CECILIO commenced their complaint for with the Statute of Frauds as set forth in this number. In the
cancellation of titles and reconveyance with damages on December 7, 1976, following cases, an agreement hereafter made shall be
only four years after the HEIRS OF CECILIO partitioned this lot among unenforceable by action unless the same, or some note or
themselves and obtained the corresponding Transfer Certificates of Titles, memorandum thereof, be in writing, and subscribed by the party
then there is no prescription of action yet. charged, or by his agent; evidence, therefore, of the agreement
11. The respondent court ordered the cancellation of the TCTs in the names of cannot be received without the writing, or a secondary evidence of
the HEIRS OF CECILIO and corollarily ordered the execution of the deeds its contents: xxx xxx xxx
of reconveyance to the SIBLINGS: b. An agreement for the leasing for a longer period than one year, or
a. To Celestina Claudel, Lot 1230-A with an area of 705 sq. m. for the sale of real property or of an interest therein;
b. To Raymunda Claudel, Lot 1230-B with an area of 599 sq. m. 5. The purpose of the Statute of Frauds is to prevent fraud and perjury in the
c. To Esperidiona Claudel, Lot 1230-C with an area of 597 sq. m. enforcement of obligations depending for their evidence upon the unassisted
d. To Macario Claudel, Lot 1230-D, with an area of 596 sq. m memory of witnesses by requiring certain enumerated contracts and
12. The rest of the land, lots 1230-E and 1230-F, with an area of 598 and 6,927 transactions to be evidenced in writing.
square meters, respectively would go to Cecilio or his heirs, the herein 6. Therefore, except under the conditions provided by the Statute of Frauds, the
petitioners. Beyond these apportionments, the HEIRS OF CECILIO would existence of the contract of sale made by Cecilio with his siblings can not be
not receive anything else. proved.
7. Above all, the torrens title in the possession of the HEIRS OF CECILIO
ISSUES: carries more weight as proof of ownership than the survey or subdivision
1. WoN there was a valid contract of sale between Cecilio and the SIBLINGS plan of a parcel of land in the name of SIBLINGS OF CECILIO.
OF CECILIO that can be proven orally - NO, there was no valid contract of 8. The Court has invariably upheld the indefeasibility of the torrens title. No
sale possession by any person of any portion of the land could defeat the title of
the registered owners thereof.
RULING: WHEREFORE, the petition is GRANTED. We REVERSE and SET
ASIDE the decision rendered in CAG. R. CV No. 04429, and we hereby REINSTATE Issue of prescription (not for class but just in case)
the decision of the then Court of First Instance of Rizal (Branch 28, Pasay City) in 9. On the second issue, the belated claim of the SIBLINGS OF CECILIO who
Civil Case No. M-5276-P which ruled for the dismissal of the Complaint for filed a complaint in court only in 1976 to enforce a right acquired allegedly
Cancellation of Titles and Reconveyance with Damages filed by the Heirs of Macario, as early as 1930, is difficult to comprehend.
Esperidiona, Raymunda, and Celestina, all surnamed CLAUDEL. Costs against the 10. Art. 1145. The following actions must be commenced within six years: (1)
private respondents. Upon an oral contract
11. If the parties SIBLINGS OF CECILIO had allegedly derived their right of
RATIO: action from the oral purchase made by their parents in 1930, then the action
1. The rule of thumb is that a sale of land, once consummated, is valid filed in 1976 would have clearly prescribed. More than six years had lapsed.
regardless of the form it may have been entered into. 12. We do not agree with the parties SIBLINGS OF CECILIO when they reason
2. For nowhere does law or jurisprudence prescribe that the contract of sale be that an implied trust in favor of the SIBLINGS OF CECILIO was established
put in writing before such contract can validly cede or transmit rights over a in 1972, when the HEIRS OF CECILIO executed a contract of partition over
certain real property between the parties themselves. the said properties.
3. However, in the event that a third party, as in this case, disputes the ownership 13. Bornales v. IAC, the defense of indefeasibility of a certificate of title was
of the property, the person against whom that claim is brought can not disregarded when the transferee who took it had notice of the flaws in the
present any proof of such sale and hence has no means to enforce the transferor’s title. No right passed to a transferee from a vendor who did not
contract. have any in the first place.
4. Thus, the Statute of Frauds was precisely devised to protect the parties
in a contract of sale of real property so that no such contract is
enforceable unless certain requisites, for purposes of proof, are met. The
ALFREDO V. BORRAS (DOM) Adelia. Godofredo and Carmen also accepted payment of the balance of the
17 June 2003 | Carpio, J. | Statute of Frauds and Contract of Sale purchase price.
PETITIONER: Spouses Godofredo and Carmen Alfredo, Spouses Arnulfo and
Editha Savellano, Danton Matawaran, Spouses Delfin and Estela Espiritu and
Elizabeth Tuazon
RESPONDENTS: Spouses Armando and Adelia Borras FACTS:
1. A parcel of land measuring 81, 524 square meters located in Barrio Culis,
SUMMARY: The subject of the controversy is the parcel of land located in Mabiga Hermosa is the subject of the controversy in this case
2. The registered owners were the Spouses Godofredo and Carmen Alfredo.
Hermosa, Bataan owned by petitioners Sps. Alfredo which they mortgaged to the
3. The subject land is covered by an original certificate of title and it was issued
Development Bank of the Philippines. Later on, Sps. Alfredo entered into an oral under a Homestead patent
contract of sale with Sps. Borras. The object of the contract of sale is the land 4. On March 7 1994, the spouses Borras filed a complaint for specific
mortgaged to the DBP. Both parties agreed on the object and with the performance against Godofredo and Carmen before the RTC branch of
consideration of 15,000php plus payment of the mortgage fee in DBP worth Bataan
7,000php. Sps. Alfredo issued a receipt and transferred the owner’s duplicate of 5. The Sps. Borras alleged in their complaint that Sps. Alfredo mortgaged the
the oct. subject land for 7,000php with Development Bank of the Philippines (DBP).
6. To pay the debt, Sps. Alfredo sod the subject land to Sps. Borras for
Few years later, Sps. Borras discovered that there were other people (subsequent 15,000php, the buyers to pay the DBP loan and its accumulated interest and
the balance to be paid in cash to sellers
buyers-herein petitioners) who were claiming the same subject property saying
7. Sps. Borras gave the money to the Sps. Alfredo to pay the loan to DBP which
that Sps. Alfredo sold to them the same parcels of land and they claim to be buyers signed the release of mortgage and returned the owner’s duplicate copy to
in good faith. Sps. Alfredo.
8. Sps. Borras then paid the subsequent balance of the purchase price of the
Sps. Borras filed for an action for specific performance from Sps. Alfredo then subject land for which Carmen Alfredo issued a receipt dated 11 March 1970.
amended the complaint to a reconveyance including now the subsequent buyers 9. Sps. Alfredo then delivered to Adelia Borras the owner’s duplicate copy with
in the complaint. RTC and CA ruled in favor of Sps. Borras saying that there was the document of cancellation of mortgage, official receipts of realty tax
a perfected contract of sale between Sps. Alfredo and Sps. Borras and that there payments, and tax declaration in the name of Godofredo Alfredo.
10. Later on, Sps. Alfredo introduced Sps. Borras, as the new owners of the
was a constructive notice when the Sps. Borras registered the property. It is
subject land, to the Natanawans, who are the old tenants of the subject land.
therefore rightful also to conclude that the subsequent buyers are innocent 11. Sps. Borras then on took possession of the land
purchasers. SC also affirmed the challenged decision and ruled in favor of the Sps. 12. In January 1994, Sps. Borras discovered that hired persons had entered the
Borras. SC emphasized that the case at bar is beyond the ambit of Statutes of fraud subject land and were cutting trees under the instructions of allegedly other
because the contract has been fully executed and evidenced by the receipt issued owners of the subject land.
by Sps. Alfredo 13. Subsequently the Sps. Borras found out that the Sps. Alfredo had re-sold
portions of the subject lands to several persons.
DOCTRINE: The Statute of Frauds provides that a contract for the sale of real 14. 8 February 19994, Sps. Borras filed an adverse claim with the Register of
property shall be unenforceable unless the contract or some note or memorandum deeds of Bataan. Sps. Borras discovered that the Sps. Alfredo had secured an
of the sale is in writing and subscribed by the party charged or his agent. The owner’s duplicate copy after filing a petition in court for the issuance of a
existence of the receipt dated 11 March 1970, which is a memorandum of the sale, new copy. Sps. Alfredo claimed that they lost their duplicate copy and they
removes the transaction from the provisions of the Statute of Frauds. need a new one.
15. Sps. Borras wrote a letter to the Sps. Alfredo complaining about their acts,
The Statute of Frauds applies only to executory contracts and not to contracts but the latter did not reply. Thus Sps. Borras filed a complaint for specific
either partially or totally performed. In addition, a contract that violates the Statute performance
of Frauds is ratified by the acceptance of benefits under the contract. Godofredo 16. 28 March 1994 Sps. Borras amended their complaint to include various
and Carmen benefited from the contract because they paid their DBP loan and people as additional defendant. These people are those considered as the
secured the cancellation of their mortgage using the money given by Armando and “subsequent buyers”.
17. The subsequent buyers who are also petitioners in this case, purchased from c. Sps. Alfredo turned over the documents such as the duplicate copy
Sps. Alfredo the subdivided portions of the subject land. of the subject land, tax declaration, and receipts to Sps. Borras
18. The Register of deeds of bataan issued subsequent buers transfer certificates d. DBP cancelled the mortgage on the subject property upon payment
of titles to the lots they purchased of the loan
19. Sps. Alfredo and the subsequent buyers (collectively the petitioners) argued e. Receipt issued by Carmen Alfredo which serves as an
that the action is unenforceable under the statute of frauds. acknowledgement if not a ratification, of the verbal sale between the
20. Petitioners pointed out that there is no written instrument evidencing alleged sellers
contract of sale over the subject land in favor of Sps. Borras. 28. The trial court also concluded that the subsequent buyers were not innocent
21. Petitioners objected to whatever parole evidence the Sps. Borras introduced purchasers
or offered on the alleged sale unless the same was in writing and subscribed 29. Not one of the bueyers testified in court how they purchased their respective
by Sps. Alfredo lots. They totally depended their testimony to a certain Constancia Calonso,
22. Petitioners also asserted that the subsequent buyers were buyers in good faith a broker who negotiated with Sps. Alfredo
and for value. 30. Calonso, the broker admitted that the subject land was adjacent to her own
23. Trial then ensued both parties presented their own set of witnesses land and she said that she did not inquire on the nature of tenancy of the
24. The Trial court rendered its decision in favor of Sps. Borras and the Natanawans and on who owned the subject land. Instead, she bought out the
dispositive portion of the decision reads: tenants for 150,000php. The buy out was embodied in a Kasnduan
a. Declaring the deeds of absolute sale over the disputed parcel of land 31. Apolinario Natanawan testified that he and his wife accepted the money and
executed by the Sps. Alfredo to the Sps. Sabellano and Espiritu, and signed the Kasunduan because Calonso and the subsequent buyers threatened
Tuazon as null and void them with forcible ejectment.
b. Declaring the TCTs in the name of Sps. Sabellano, Espiritu, and 32. Calonso brought Apolinario to the Agrarian reform office where he was
Matawaran null and void and ordered to cancel said titles asked to produce documents showing that Adelia Borras is the owner of the
c. Ordering the Sps. Alfredo to execute and deliver good and valid subject land and because Apolinario cannot produce the documents the
Deed of Absolute Sale in favor of Sps. Borras within 10 days upon officer told him that he would lose the case and thus Apolinario was
finality of decision constrained to sign the Kasunduan.
d. Ordering the Sps. Alfredo to surrender their owner’s duplicate copy 33. CA affirmed the decision of RTC by saying indeed that there was a perfected
of OCT issued to them by the Registry of Deeds in Bataan. The contract of sale and that the subsequent buyers were not innocent purchasers
registry of deeds is ordered ott cancel the same as there exists in the 34. The CA ruled that the handwritten receipt dated 11 March 1970 is a sufficient
possession of Sps. Alfredo proof that the land was sold to Sps. Borras upon payment of the balance of
e. Sps. Alfredo are ordered to return the amount of their respective the purchase price.
purchase prices of sale to the disputed parcels of land 35. CA found the recitals in the receipt as sufficient to serve as the memorandum
25. Petitioners appealed to the court of appeals and on 26TH of November 1999, or note as a writing under the statute of Frauds
the CA issued its decision affirming the trial court 36. CA grave credence to the testimony of Sps. Borras and their witnesses who
26. RTC ruling: It emphasizes that there was a perfected contract of sale between explained why the title of the subject land was not in the name of the said
the Sps. Alfredo and Borras. The trial court found that all the elements of a spouses
contract of sale were present 37. The witnesses explained that Sps. Alfredo was then busy preparing to leave
a. Object: specifically identified as the 81.524 square meter of lot in for Davao. Godofredo Alfredo proised that he would sign all the papers once
Hermosa Bataan they are ready and since the two families were close, they trusted Godfredo
b. Price: the purchase price was fixed at P15,000, with the buyers and honored Carmen’s commitment
assuming to pay the sellers’ P7,000 DBP mortgage loan including 38. Sps. Borras had no reason to believe that the contract was not perfected
its accumulated interest. considering they had received the duplicate copy of OCT
c. Consent: meeting of the minds as to the two aforementioned 39. CA held also that the contract is not void ieven if only Carmen signed the
elements receipt citing Felipo v. Heirs of Maximo by saying that the sale made by the
27. The trial court found the following facts as proof of a perfected contract of wife without husband’s consent is not void but merely voidable
sale 40. In this case it binds the conjugal partnership because the proceeds were used
a. Sps. Alfredo delivered the subject land to Sps. Borras for the benefit of the conjugal partnership
b. Sps. Borras treated as their own tenants the tenants of Sps. Alfredo 41. Subsequent buyers of the conjugal partnership are not innocent urchasers for
they had a constructive notice of the claims of Sps. Borras Land. This physical delivery of the Subject Land also constituted a
transfer of ownership of the Subject Land to Armando and Adelia.
ISSUES: Whether the alleged sale of the subject land in favor of Armando and Adelia Ownership of the thing sold is transferred to the vendee upon its actual or
is valid and enforceable where: constructive delivery. Godofredo and Carmen also turned over to Armando
• It was orally entered into and not in writing (IMPT. IN THIS CASE: YES- and Adelia the documents of ownership to the Subject Land, namely the
it is still valid, a receipt was issued hence, Statute of frauds is not applicable) owner’s duplicate copy of OCT No. 284, the tax declaration and the receipts
• Carmen Alfredo did not obtain the consent of her husband of realty tax payments.
• It was entered during the 25-year prohibitive period for alienating lands 3. The trial and appellate courts correctly refused to apply the Statute of Frauds
without approval of secretary of agriculture to this case. The Statute of Frauds provides that a contract for the
sale of real property shall be unenforceable unless the contract or
some note or memorandum of the sale is in writing and subscribed
OTHER ISSUES: by the party charged or his agent. The existence of the receipt dated
11 March 1970, which is a memorandum of the sale, removes the
• W/N the action to enforce the alleged oral contract of sale brought after 24 transaction from the provisions of the Statute of Frauds. (for
years from its alleged perfection had been barred by prescription and by emphasis)
laches - NO 4. The Statute of Frauds applies only to executory contracts and not to
contracts either partially or totally performed. Thus, where one party has
• W/N the subsequent buyers are innocent purchasers - NO performed one’s obligation, oral evidence will be admitted to prove the
agreement. In the instant case, the parties have consummated the sale of the
RULING: WHEREFORE, the petition is DENIED and the appealed decision is Subject Land, with both sellers and buyers performing their respective
AFFIRMED. Treble costs against petitioners. obligations under the contract of sale. In addition, a contract that violates
the Statute of Frauds is ratified by the acceptance of benefits under the
SO ORDERED. contract. Godofredo and Carmen benefited from the contract because
they paid their DBP loan and secured the cancellation of their
RATIO: mortgage using the money given by Armando and Adelia. Godofredo
and Carmen also accepted payment of the balance of the purchase
VALIDITY OF THE CONTRACT OF SALE: price. (for emphasis)
a.) enforceability of the oral contract of sale
b.) sale without the consent of the husband
1. The contract of sale between the spouses Godofredo and Carmen and the
spouses Armando and Adelia was a perfected contract. A contract is perfected 1. Godofredo can no longer question the sale. Voidable contracts are susceptible
once there is consent of the contracting parties on the object certain and on of ratification. Godofredo ratified the sale when he introduced Armando and
the cause of the obligation. In the instant case, the object of the sale is the Adelia to his tenants as the new owners of the Subject Land. The trial court
Subject Land, and the price certain is ₱15,000.00. The trial and appellate noted that Godofredo failed to deny categorically on the witness stand the
courts found that there was a meeting of the minds on the sale of the Subject claim of the complainants’ witnesses that Godofredo introduced Armando
Land and on the purchase price of ₱15,000.00. This is a finding of fact that and Adelia as the new landlords of the tenants. That Godofredo and Carmen
is binding on this Court. We find no reason to disturb this finding since it is allowed Armando and Adelia to enjoy possession of the Subject Land for 24
supported by substantial evidence. years is formidable proof of Godofredo’s acquiescence to the sale.
2. Moreover, Godofredo and Carmen used most of the proceeds of the sale to
2. The contract of sale of the Subject Land has also been consummated because pay their debt with the DBP. We agree with the Court of Appeals that the sale
the sellers and buyers have performed their respective obligations under the redounded to the benefit of the conjugal partnership.
contract. In a contract of sale, the seller obligates himself to transfer the 3. As to the approval of the Agriculture Secretary, The failure to secure the
ownership of the determinate thing sold, and to deliver the same, to the buyer approval of the Secretary does not ipso facto make a sale void. The absence
who obligates himself to pay a price certain to the seller. In the instant case, of approval by the Secretary does not nullify a sale made after the expiration
Godofredo and Carmen delivered the Subject Land to Armando and of the 5-year period, for in such event the requirement of Section 118 of the
Adelia, placing the latter in actual physical possession of the Subject
Public Land Act becomes merely directory or a formality. The approval may sale.
be secured later, producing the effect of ratifying and adopting the transaction 5. Thus, to merit protection under the second paragraph of Article 154458 of
as if the sale had been previously authorized. the Civil Code, the second buyer must act in good faith in registering the
4. Section 118 of Commonwealth Act No. 141, as amended, specifically enjoins deed. In this case, the Subsequent Buyers’ good faith hinges on whether they
that the approval by the Department Secretary "shall not be denied except on had knowledge of the previous sale. Petitioners do not dispute that Armando
constitutional and legal grounds." There being no allegation that there were and Adelia registered their adverse claim with the Registry of Deeds of
constitutional or legal impediments to the sales, and no pretense that if the Bataan on 8 February 1994. The Subsequent Buyers purchased their
sales had been submitted to the Secretary concerned they would have been respective lots only on 22 February 1994 as shown by the date of their deeds
disapproved, approval was a ministerial duty, to be had as a matter of course of sale. Consequently, the adverse claim registered prior to the second sale
and demandable if refused. For this reason, and if necessary, approval may charged the Subsequent Buyers with constructive notice of the defect in the
now be applied for and its effect will be to ratify and adopt the transactions title of the sellers, Godofredo and Carmen
as if they had been previously authorized.

OTHER ISSUES:
The action is not barred by prescription and laches

1. Article 1456 of the Civil Code provides that a person acquiring property
through fraud becomes by operation of law a trustee of an implied trust for
the benefit of the real owner of the property. The presence of fraud in this
case created an implied trust in favor of Armando and Adelia. This gives
Armando and Adelia the right to seek reconveyance of the property from the
Subsequent Buyers. The body of the pleading or complaint determines the
nature of an action, not its title or heading. Thus, the present action should be
treated as one for reconveyance.
2. To determine when the prescriptive period commenced in an action for
reconveyance, plaintiff’s possession of the disputed property is material. An
action for reconveyance based on an implied trust prescribes in ten years. The
ten-year prescriptive period applies only if there is an actual need to reconvey
the property as when the plaintiff is not in possession of the property.
However, if the plaintiff, as the real owner of the property also remains in
possession of the property, the prescriptive period to recover title and
possession of the property does not run against him. In such a case, an action
for reconveyance, if nonetheless filed, would be in the nature of a suit for
quieting of title, an action that is imprescriptible.
3. SC disagree with CA that it should be 4 years prescriptive period based on
fraud that should be discussed instead it shoud be ten years for this is an
obligation created by law (particularly an action for reconveyance based on
an implied or constructive trust)
Subsequent sale of portions of lands invalid
4. Godofredo and Carmen had already sold the Subject Land to Armando and
Adelia. The settled rule is when ownership or title passes to the buyer, the
seller ceases to have any title to transfer to any third person. If the seller sells
the same land to another, the second buyer who has actual or constructive
knowledge of the prior sale cannot be a registrant in good faith. Such second
buyer cannot defeat the first buyer’s title. In case a title is issued to the second
buyer, the first buyer may seek reconveyance of the property subject of the
TOYOTA SHAW v. CA (KC) home address, the model series of the vehicle, and that the payment is by
May 23, 1995| Davide, Jr., J. | Receipt proves payment which takes the sale out of installment to be financed by B.A.
the Statute of Frauds 78. The Sales Supervisor of Bernardo even checked and approved the VSP.
79. On June 17, 1989 Bernardo called Gilberto and told him that the vehicle
PETITIONER: Toyota Shaw, Inc. would not be ready for pickup at 10:00 AM but on 2:00 PM that same day.
RESPONDENTS: Court of Appeals and Luna L. Sosa 80. However, respondent Sosa and his son never got the car. Bernardo allegedly
told them “nasulot ang unit ng ibang malakas”
SUMMARY: Respondent Sosa wanted to buy a car. Exhibit A was signed by 81. Petitioner Toyota Shaw on the other hand said that the vehicle was not
Bernardo wherein it only stated that a downpayment of P100,000 was to be paid. delivered because B.A. Finance disapproved the credit financing application
A subsequent document called VSP was then printed which stated the name of the of Sosa. Petitioner Toyota gave respondent Sosa the option to purchase the
customer, the home address, the model series of the vehicle, and that the payment unit by paying the full purchase price in cash but Sosa refused.
is by installment to be financed by B.A. The vehicle was not released because 82. Respondent Sosa asked for the downpayment which Toyota geve by issuing
respondent Sosa’s credit financing application was denied by B.A. Respondent a Far East Bank check.
Sosa asked the downpayment back from petitioner Toyota and filed a case for 83. Respondent Sosa then sent two demand letters to Petitioner Toyota asking
damages. The trial court and CA ruled in favor of respondent Sosa but SC said them to pay the P100,00 with interest and damages. And subsequently
that the two documents did were not contracts of sale. Exhibit A did not make any demanded P1M representing said interest and damages.
reference to the determinate thing nor the purchase price The agreement as to the 84. Petitioner Toyoto refused to accede.
manner of payment goes into the price such that a disagreement on the manner of 85. Respondent Sosa filed a complaint against Petitioner Toyota under Art. 19 &
payment is tantamount to a failure to agree on the price. On the otherhand, the 21 of the Civil Code.
VSP created no obligation whatsoever because the credit financing application 86. Petitioner Toyoto alleged that no sale was entered into between it and
was denied. At most both documents were part of the negotiation phase. respondent Sosa, and that Bernardo had no authority to sign Exhibi A for and
Therefore, there was no meeting of the minds. in its behalf. And that the VSP did not state a date of delivery nor did
respondent Sosa completed the documents required by the financing
DOCTRINE: The memorandum to take the sale transaction out of the coverage company (B.A.) and as a matter of policy the vehicle could not be released.
of the Statute of Frauds, must contain within its four corners “all the essential 87. The trial court and CA ruled in favor of respondent Sosa, saying that Exhibit
terms of the contract” of sale. A was a valid perfect contract of sale and bound Petitioner Toyota to deliver
the vehicle.
88. Hence this appeal
FACTS:
72. Sometime in June 1989, respondent Sosa wanted to purchase a Toyota Lite ISSUE/s:
Ace. It was a seller’s market (I think this means maraming bumibili?) and 21. WoN there was a perfected contract of sale – NO
Sosa was having a hard time finding a dealer with an available unit for sale.
73. Upon contacting Petitioner Toyota Shaw, Inc., he was told that there was an RULING: WHEREFORE, the instant petition is GRANTED. The challenged
available unit.
decision of the Court of Appeals in CA-G.R. CV NO. 40043 as well as that of Branch
74. Respondent Sosa and his son Gilbert went to Shaw and met Popong Bernardo
who was a sales representative of Petitioner Toyota. 38 of the Regional Trial Court of Marinduque in Civil Case No. 89-14 are REVERSED
75. Respondent Sosa needed the car not later than June 17, 1989 because he and and SET ASIDE and the complaint in Civil Case No. 89-14 is DISMISSED. The
his family with a balikbayan friend palnned to go home to their province counterclaim therein is likewise DISMISSED. No pronouncement as to costs.
using the car.
76. Exhibit A titled “Agreements Between Mr. Sosa & Popong Bernardo of RATIO:
Toyota Shaw, Inc.” was then signed by Bernardo. It said that the 57. Art. 1458 of the Civil Code defines a contract of sale as:
downpayment of P100,000 will be paid by respondent Sosa on June 15, 1989 a. By the contract of sale one of the contracting parties obligates
and that the remaining balance of the purchase price would be paid by credit himself to transfer the ownership of and to deliver a determinate
financing through B.A. Finance thing, and the other to pay therefor a price certain in money or its
77. Respondent Sosa paid the downpayment the following day and accomplished equivalent. A contract of sale may be absolute or conditional.

a printed Vehicle Sales Proposal (VSP) on which Gilbert signed under the 58. Art. 1457 specifically provides that:
subheading CONFORME. The VSP stated the name of the customer, the a. The contract of sale is perfected at the moment there is a meeting of
minds upon the thing which is the object of the contract and generation/negotiation stage of a contract of sale. Meanwhile, the VSP was
upon the prince. the second pahase of the generation/negotiation stage.
67. The VSP was a mere proposal which was aborted in lieu of the disapproval
From that moment, the parties may reciprocally demand of respondent Sosa’s credit financing application. In a sale on installment
performance, subject to the provisions of the law governing the form basis there are three parties involved: the buyer who executes a note/notes for
of contracts the unpaid balance of the price, the seller who assigns the notes/discounts
them with a financing company and the financing company which is
59. Exhibit A is not a contract of sale. No obligation on the part of petitioner subrogated in the place of the seller
Toyota to transfer ownership of a determinate thing to respondent Sosa and 68. Since B.A. did not approve respondent Sosa’s application there was then no
no correlative obligation on the part of the latter to pay therefor a price certain meeting of the minds on the sale on installment basis.
appears therein. 69. It follows therefore that the VSP created no demandable right in favor of
Exhibit A: respondent Sosa for the delivery of the vehicle to him, and its non-delivery
did not cause any legally indemnifiable injury. And at the bottom of the claim
for damages it nothing but misplaced pride and ego. (Napahiya daw kasi siya
4 June 1989 nung umuwi siya sa province without the car)

AGREEMENTS BETWEEN MR. SOSA & POPONG BERNARDO OF


TOYOTA SHAW, INC.

1. all necessary documents will be submitted to TOYOTA SHAW, INC.


(POPONG BERNARDO) a week after, upon arrival of Mr. Sosa from the
Province (Marinduque) where the unit will be used on the 19th of June.

2. the downpayment of P100,000.00 will be paid by Mr. Sosa on June 15,


1989.

3. the TOYOTA SHAW, INC. LITE ACE yellow, will be pick-up [sic] and
released by TOYOTA SHAW, INC. on the 17th of June at 10 a.m.

Very truly yours,


(Sgd.) POPONG BERNARDO.
60. The provision on the downpayment made no mention of the specific
reference to a sale of a vehicle.
61. If it was intended for a contract of sale, it could only refer to a sale on
installment basis, as the VSP executed the following day confirmed. But
nothing was mentioned about the full purchase price and the manner the
installments were to be paid.
62. A definite agreement on the manner of payment of the price is an essential
element in the formation of a biding and enforceable contract of sale.
63. The agreement as to the manner of payment goes into the price such that a
disagreement on the manner of payment is tantamount to a failure to agree on
the price.
64. Definiteness as to the price is an essential element of a binding agreement to
sell a personal property
65. There was no meeting of the minds. Respondent Sosa did not even sign it.
66. At most, Exhibit A can only be considered as a part of the initial phase of the
01 SANTOS vs. SANTOS (Elach) FACTS:
October 2, 2001 | Quisumbing, J. | Transferring Ownership; Constructive delivery 1. Petitioner Zenaida M. Santos is the widow of Salvador Santos, a brother of
private respondents Calixto, Alberto, Antonio, all surnamed Santos and Rosa
PETITIONER: Zenaida M. Santos Santos-Carreon.
RESPONDENTS: Calixto Santos, Alberto Santos, Rosa Santos-Carreon, and 2. The spouses Jesus and Rosalia Santos (Parents of respondents and parents-
Antonio Santos in-law of petitioner) owned a parcel of land located at Sta. Cruz Manila. On
it was a four-door apartment administered by Rosalia who rented them out.
SUMMARY: The spouses Jesus and Rosalia were the parents of Calixto, The spouses had five children, Salvador, Calixto, Alberto, Antonio and Rosa.
Alberto, and Rosa, the respondents and Salvador Santos, the husband of the 3. On January 19, 1959, Jesus and Rosalia executed a deed of sale of the
petitioner, Zenaida Santos (So parents-in-law ni Zenaida sila Jesus and Rosalia). properties in favor of their children Salvador and Rosa. Rosa in turn sold her
The spouses owned a parcel of registered land with a four-door apartment share to Salvador on November 20.
administered by Rosalia who rented them out. On January 19, 1959, the spouses 4. Despite the transfer of the property to Salvador, Rosalia continued to lease
executed a deed of sale of the properties in favor of their children Salvador and receive rentals form the apartment units.
Rosa. Rosa in turn sold her share to Salvador on November 20, 1973, which 5. On November 1, 1979, Jesus died. Six years after or on January 9, 1985,
resulted in the issuance of new TCT. Despite the transfer of the property to Salvador died, followed by Rosalia who died the following month.
Salvador, Rosalia continued to lease and receive rentals from the apartment units. 6. Shortly after, petitioner Zenaida, claiming to be Salvador's heir, demanded
On January 9, 1985, Salvador died, followed by Rosalia who died the following the rent from Antonio Hombrebueno, a tenant of Rosalia.
month. Shortly after, petitioner Zenaida, claiming to be Salvador’s heir, 7. When the latter refused to pay, Zenaida filed and ejectment suit against him
demanded the rent from Antonio Hombrebueno, a tenant of Rosalia. When the with the MTC, which eventually decided in Zenaida's favor.
latter refused to pay, Zenaida filed an ejectment suit against him with the 8. Subsequently, private respondents instituted an action for reconveyance of
Metropolitan Trial Court of Manila, which eventually decided in Zenaida’s favor. property with preliminary injunction against petitioner in the RTC, where
Private respondent instituted an action for reconveyance of property with they alleged that the two deeds of sale executed on January 19, 1959 and
preliminary injunction against petitioner, Zenaida. in the RTC of Manila, where November 20, 1973 were simulated for lack of consideration. They were
they alleged that the two deeds of sale were simulated for lack of consideration. executed to accommodate Salvador in generation funds for his business and
The trial court decided in favor of private respondents in as much as the deeds of providing him with greater business flexibility.
sale were fictitious. Upon appeal, the Court of Appeals affirmed the trial court’s 9. In her Answer, Zenaida denied the material allegations in the complaint as
decision. It held that the subject deeds of sale did not confer upon Salvador the special and affirmative defenses, argued that Salvador was the registered
ownership over the subject property, because even after the sale, the original owner of the property, which could only be subjected to encumbrances or
vendors remained in dominion, control, and possession thereof. Issue is W/N a liens annotated on the title; that the respondents' right to reconveyance was
sale through a public instrument is tantamount to delivery of the thing sold? already barred by prescription and laches; and that the complaint state no
Petitioner in her memorandum invokes Article 1477 of the Civil Code which cause of action.
provides that ownership of the thing sold is transferred to the vendee upon its 10. The trial court decided in favor of private respondents in as much as the deeds
actual or constructive delivery. Article 1498, in turn, provides that when the sale of sale were fictitious, the action to assail the same does not prescribe.
is made through a public instrument, its execution is equivalent to the delivery of 11. Upon appeal, the Court of Appeals affirmed the trial court’s decision. It held
the thing subject of the contract. Petitioner avers that applying said provisions to that the subject deeds of sale did not confer upon Salvador the ownership over
the case, Salvador became the owner of the subject property by virtue of the two the subject property, because even after the sale, the original vendors
deeds of sale executed in his favor. Nowhere in the Civil Code, however, does remained in dominion, control, and possession thereof.
it provide that execution of a deed of sale is a conclusive presumption of
delivery of possession. The Code merely said that the execution shall be ISSUES:
equivalent to delivery. 1. W/N a sale through a public instrument is tantamount to delivery of the thing
sold? – No it is not a conclusive presumption of delivery of possession, thus
DOCTRINE: There is nothing in Article 1498 that provides that execution of a no real transfer of ownership
deed of sale is a conclusive presumption of delivery of possession; presumptive
delivery can be negated by the failure of buyer to take actual possession of the RULING: WHEREFORE, the instant petition is DENIED. The assailed decision
land or the continued enjoyment of possession by the vendor. dated March 10, 1998 of the Court of Appeals, which sustained the judgment of the
Regional Trial Court dated March 17, 1993, in favor of herein private respondents, is
AFFIRMED. Costs against petitioner. 2. It is true that neither tax receipts nor declarations of ownership for taxation
purposes constitute sufficient proof of ownership. They must be supported by
RATIO: other effective proofs.
1. Petitioner in her memorandum invokes Article 1477 of the Civil Code which 3. These requisite proofs are present in this case. As admitted by petitioner,
provides that ownership of the thing sold is transferred to the vendee upon its despite the sale, Jesus and Rosalia continued to possess and administer the
actual or constructive delivery. Article 1498, in turn, provides that when the property and enjoy its fruits by leasing it to third persons.
sale is made through a public instrument, its execution is equivalent to the 4. Both Rosa and Salvador did not exercise any right of ownership over it.
delivery of the thing subject of the contract. Before the second deed of sale to transfer her ½ share over the property was
2. Petitioner avers that applying said provisions to the case, Salvador became executed by Rosa, Salvador still sought she permission of his mother.
the owner of the subject property by virtue of the two deeds of sale executed 5. Further, after Salvador registered the property in his name, he
in his favor. surrendered the title to his mother. These are clear indications that
3. Nowhere in the Civil Code, however, does it provide that execution of a deed ownership still remained with the original owners.
of sale is a conclusive presumption of delivery of possession. The Code 6. In Serrano vs. CA (1985), the Court held that the continued collection of
merely said that the execution shall be equivalent to delivery. The rentals from the tenants by the seller of realty after execution of alleged deed
presumption can be rebutted by clear and convincing evidence. Presumptive of sale is contrary to the notion of ownership.
delivery can be negated by the failure of the vendee to take actual possession 7. Petitioner argues that Salvador, in allowing her mother to use the property
of the land sold. even after the sale, did so out of respect for her and out of generosity, a factual
4. In Danguilan vs. IAC (1988), the SC held that for the execution of a public matter beyond the province of this Court.
instrument to effect tradition, the purchaser must be placed in control of 8. Significantly, in Alcos vs. IAC (1988), the SC noted that the buyer's
the thing sold. When there is no impediment to prevent the thing sold from immediate possession and occupation of the property corroborated the
converting to tenancy of the purchaser by the sole will of the vendor, truthfulness and authenticity of the deed of sale. Conversely, the vendor's
symbolic delivery through the execution of a public instrument is sufficient. continued possession of the property makes dubious the contract of sale
But if, notwithstanding the execution of the instrument, the purchaser between the parties.
cannot have the enjoyment and material tenancy nor make use of it
himself or through another in his name, then delivery has not been
effected.
5. As found by both the trial and appellate courts and amply supported by the
evidence on record, Salvador was never placed in control of the property.
The original sellers retained their control and possession. Therefore,
there was no real transfer of ownership.
6. Moreover, in Norkis Distributors, Inc. vs. CA, the Court held that the critical
factor in the different modes of effecting delivery, which gives legal effect to
the act is the actual intention of the vendor to deliver, and its acceptance by
the vendee. Without that intention, there is no tradition. In the instant case,
although the spouses Jesus and Rosalia executed a deed of sale, they did
not deliver the possession and ownership of the property to Salvador and
Rosa. They agreed to execute a deed of sale merely to accommodate
Salvador to enable him to generate funds for his business venture.

Issue on W/N CA erred in holding Jesus and Rosalia still owned the property
because the spouses continued to pay the realty taxes and possess the property

1. Petitioner, Zenaida, argues that tax declarations are not conclusive evidence
of ownership when not supported by evidence. She avers that Salvador
allowed his mother to possess the property out of respect to her in accordance
with Filipino values.
02 DY, JR. v. CA (Karen) FACTS:
July 8, 1991 | Guiterrez, Jr. J. | Constructive delivery: execution of public instrument 89. The petitioner Perfecto Dy and Wilfredo Dy are brothers. Sometime in 1979,
Wilfredo Dy purchased a truck and a farm tractor through financing extended
PETITIONER: Perfecto Dy Jr. by Libra Finance and Investment Corporation (Libra). Both truck and tractor
RESPONDENTS: Court of Appeals, Gelac Trading Inc., and Antonio V. were mortgaged to Libra as security for the loan.
Gonzales
90. Perfecto wanted to buy the tractor from Wilfredo so he wrote a letter to Libra
SUMMARY: Wilfredo Dy purchased a truck and a farm tractor through Libra requesting that he be allowed to purchase from Wilfredo the said tractor and
which was also mortgaged with the latter, as a security to the loan. Perfecto Dy assume the mortgage debt of the latter. The request was then approved.
expresses his desire to purchased his brother’s tractor in a letter to Libra which
also includes his intention to shoulder its mortgaged. Libra approved the request. 91. Thus, Wilfredo executed a deed of absolute sale in favor of the Perfecto over
At the time that Wilfredo executed a deed of absolute sale in favor of Perfecto, the the tractor in question. At this time, the subject tractor was in the possession
tractor and truck were in the possession of Libra for his failure to pay the of Libra due to Wilfredo’s failure to pay the amortizations.
amortization. When Perfecto finally fulfilled its obligation to pay the tractor, Libra
would only release the same only if he would also pay for the truck. In order to 92. Despite the offer of full payment by Perfecto to Libra for the tractor, the
fulfill Libra’s condition, Perfecto convinced his sister to pay for the remaining immediate release could not be effected because Wilfredo had obtained
truck, to which she released a check of P22,000. Libra, however, insisted that the financing not only for said tractor but also for a truck and Libra insisted on
check must be first cleared before it delivers the truck and tractor. Meanwhile, full payment for both.
another case penned “Gelac Trading Inc vs. Wilfredo Dy” was pending in Cebu
as a case to recover for a sum of money (P12,269.80). By a writ of execution the 93. Perfecto was able to convince his sister, Carol Dy-Seno, to purchase the truck
court in Cebu ordered to seize and levy the tractor which was in the premise of so that full payment could be made for both. A PNB check P22,000 was
Libra, it was sold in a public auction to which it was purchased by Gelac. The issued in favor of Libra, thus settling in full the indebtedness of Wilfredo.
latter then sold the tractor to Antonio Gonzales. RTC rendered in favor of Perfecto. Payment having been effected through an out-of-town check, Libra insisted
CA dismissed the case, alleging that it still belongs to Wilfredo. The issue is WON that it be cleared first before Libra could release the chattels in question.
Wilfredo Dy is still the owner of the tractor when it was seized and levied by the
provincial sheriff. The SC held that per Article 1496 of the Civil Code states that 94. Meanwhile, civil case entitled "Gelac Trading, Inc. v. Wilfredo Dy", a
the ownership of the thing sold is acquired by the vendee from the moment it is collection case to recover P12,269.80 was pending in another court in Cebu.
delivered to him in any of the ways specified in Articles 1497 to 1501 or in any
other manner signifying an agreement that the possession is transferred from the 95. On the strength of an alias writ of execution, the provincial sheriff was able
vendor to the vendee. Articles 1498 and 1499 are applicable in the case at bar. In to seize and levy on the tractor which was in the premises of Libra in Carmen,
the instant case, actual delivery of the subject tractor could not be made. However, Cebu. The tractor was subsequently sold at public auction where Gelac
there was constructive delivery already upon the execution of the public Trading (Gelac) was the lone bidder. Later, Gelac sold the tractor to one of
instrument pursuant to Article 1498 and upon the consent or agreement of the its stockholders, Antonio Gonzales.
parties when the thing sold cannot be immediately transferred to the possession of
the vendee (Article 1499). In this case, the sale of the subject tractor was 96. It was only when the check was cleared that Perfecto learned about Gelac
consummated upon the execution of the public instrument (September 1979). At having already taken custody of the subject tractor. Consequently, Perfecto
this time constructive delivery was already effected. Hence, the subject tractor was filed an action to recover the subject tractor against Gelac with the RTC of
no longer owned by Wilfredo Dy when it was levied upon by the sheriff Cebu City.
(December 1979).
97. The RTC rendered judgment in favor of Perfecto. However, the CA reversed
DOCTRINE: When actual delivery of the subject matter could not be made, the decision and held that tractor in question still belonged to Wilfredo when
constructive delivery is possible upon the execution of the public instrument it was seized and levied by the sheriff by virtue of the alias writ of execution.
pursuant to Article 1498 and upon the consent or agreement of the parties when
the thing sold cannot be immediately transferred to the possession of the vendee 98. Hence, the current petition.
(Article 1499).
ISSUE: Whether or not Wilfredo Dy is still the owner of the tractor when it was seized
and levied by the provincial sheriff. - NO 75. The CA avers that the vendor must first have control and possession of the
thing before he could transfer ownership by constructive delivery. Here, it
RULING: The petition is hereby GRANTED. The decision of the CA is SET ASIDE was Libra which was in possession of the subject tractor due to Wilfredo's
and the decision of the RTC is REINSTATED. failure to pay the amortization as a preliminary step to foreclosure. As
mortgagee, he has the right of foreclosure upon default by the mortgagor in
RATIO: the performance of the conditions mentioned in the contract of mortgage. The
70. The private respondents claim that at the time of the execution of the deed of law implies that the mortgagee is entitled to possess the mortgaged property
sale, no constructive delivery was effected since the consummation of the because possession is necessary in order to enable him to have the property
sale depended upon the clearance and encashment of the check which was sold.
issued in payment of the subject tractor.
76. While it is true that Wilfredo was not in actual possession and control of the
71. However, the SC held that in the case of Servicewide Specialists Inc. v. subject tractor, his right of ownership was not divested from him upon his
Intermediate Appellate Court: “The rule is settled that the chattel mortgagor default. Neither could it be said that Libra was the owner of the subject tractor
continues to be the owner of the property, and therefore, has the power to because the mortgagee can not become the owner of or convert and
alienate the same; however, he is obliged under pain of penal liability, to appropriate to himself the property mortgaged. (Article 2088, Civil Code)
secure the written consent of the mortgagee. The absence of the written Said property continues to belong to the mortgagor. The only remedy given
consent of the mortgagee to the sale of the mortgaged property in favor of a to the mortgagee is to have said property sold at public auction and the
third person, therefore, affects not the validity of the sale but only the penal proceeds of the sale applied to the payment of the obligation secured by the
liability of the mortgagor under the Revised Penal Code and the binding mortgagee. (See Martinez v. PNB, 93 Phil. 765, 767 [1953])
effect of such sale on the mortgagee under the Deed of Chattel Mortgage.”
72. In this case, there is no reason why Wilfredo, as the chattel mortgagor can 77. There is no showing that Libra Finance has already foreclosed the mortgage
not sell the subject tractor. There is no dispute that the consent of Libra was and that it was the new owner of the subject tractor. Undeniably, Libra gave
obtained in the instant case. In a letter, Libra allowed Perfecto to purchase its consent to the sale of the subject tractor to Perfecto. It was aware of the
the tractor and assume the mortgage debt of his brother. The sale between the transfer of rights to Perfecto.
brothers was therefore valid and binding as between them and to the
mortgagee, as well. 78. Where a third person purchases the mortgaged property, he automatically
steps into the shoes of the original mortgagor. His right of ownership shall be
73. Article 1496 of the Civil Code states that the ownership of the thing sold is subject to the mortgage of the thing sold to him. In the case at bar, Perfecto
acquired by the vendee from the moment it is delivered to him in any of the was fully aware of the existing mortgage of the subject tractor to Libra. In
ways specified in Articles 1497 to 1501 or in any other manner signing an fact, when he was obtaining Libra's consent to the sale, he volunteered to
agreement that the possession is transferred from the vendor to the vendee. assume the remaining balance of the mortgage debt of Wilfredo which Libra
Articles 14986 and 14997 are applicable in the case at bar. undeniably agreed to.

74. In the instant case, actual delivery of the subject tractor could not be 79. The payment of the check was actually intended to extinguish the mortgage
made. However, there was constructive delivery already upon the obligation so that the tractor could be released to Perfecto. It was never
execution of the public instrument pursuant to Article 1498 and upon the intended nor could it be considered as payment of the purchase price because
consent or agreement of the parties when the thing sold cannot be the relationship between Libra and Perfecto is not one of sale but still a
immediately transferred to the possession of the vendee. (Art. 1499) mortgage. The clearing or encashment of the check which produced the effect
of payment determined the full payment of the money obligation and the

6 7
Art. 1498. When the sale is made through a public instrument, the Article 1499. The delivery of movable property may likewise be made
execution thereof shall be equivalent to the delivery of the thing which by the mere consent or agreement of the contracting parties, if the thing
is the object of the contract, if from the deed the contrary does not appear sold cannot be transferred to the possession of the vendee at the time of
or cannot clearly be inferred. the sale, or if the latter already had it in his possession for any other
reason.
release of the chattel mortgage. It was not determinative of the consummation
of the sale. The transaction between the brothers is distinct and apart from the
transaction between Libra and Perfecto. The contention, therefore, that the
consummation of the sale depended upon the encashment of the check is
untenable.

80. The sale of the subject tractor was consummated upon the execution of
the public instrument (September 1979). At this time constructive
delivery was already effected. Hence, the subject tractor was no longer
owned by Wilfredo Dy when it was levied upon by the sheriff (December
1979). Well settled is the rule that only properties unquestionably owned by
the judgment debtor and which are not exempt by law from execution should
be levied upon or sought to be levied upon. For the power of the court in the
execution of its judgment extends only over properties belonging to the
judgment debtor. (Consolidated Bank and Trust Corp. v. Court of Appeals,
G.R. No. 78771, January 23, 1991).
03 ADDISON v. FELIX (DANNAH) coconut tree bearing and P5 for each tree not bearing
3 August 1918 | Fisher, J. | Sale Through Public Instrument (Art. 1498) d. Condition: Total price should not exceed P85,000
e. Felix was to deliver to Addison 25% of the value of the products she might
obtain from the four parcels “from the moment she takes possession of them
PLAINTIFF-APPELLANT: A. A. Addison until the Torrens title be issued in her favor”
DEFENDANT-APPELLEES: Marciana Felix and Balbino Tioco f. “Within one year from the date of the title in favor of Felix, she may rescind
the present contract of purchase in which case Felix shall be obliged to
SUMMARY: Addison and Felix, with her husband Tioco’s consent, entered into return to Addison the net value of all the products of the four parcels sold,
an agreement on June 1914 wherein Felix was to purchase from Addison four and Addison be obliged to return to Felix all the sums she may have paid…”
parcels of land. (See installment plan and conditions in the contract in FACT 2). 101. On January 1915, Addison filed in the CFI of Manila a complaint to compel
Felix paid a downpayment of P3,000. Felix to make payment of the first installment (see 2a).
102. Felix, with her husband, answered the complaint and alleged that Addison
On January 1915, Addison filed in the CFI of Manila a complaint to compel Felix had absolutely failed to deliver to Felix the lands which were the subject
to pay the first installment. Felix, on the other hand answered the complaint. She matter of the sale, notwithstanding their demands.
asked to be absolved and that Addison be ordered to return the P3,000 she paid. 103. She asked to be absolved from the complaint, and that Addison be ordered to
The trial court ruled in favor of Felix, holding the contract of sale to be rescinded refund the P3,000 she paid, plus damages.
and ordering the return of the P3,000 with interest (citing a clause from their 104. It is shown that after the execution of the deed of sale, Addison at Felix’
contract, see FACT 2f). From this judgment Addison appealed. request went to Lucena, accompanied by Felix’ representative, for the
purpose of designating and delivering the lands sold.
The SC still ruled in favor of Felix, but for a different reason (since the condition 105. He was able to designate only 2 of the 4 parcels, and more than 2/3 of these
for the rescission of the contract under FACT 2f had not yet been fulfilled). This two was in the possession of one Villafuerte who claimed to be the owner of
is due to the fact that Addison did not deliver the thing sold. The Code imposes the parts he occupied.
upon the vendor the obligation to deliver the thing sold. The thing is considered a. Addison admitted that Felix would have to bring suit to obtain possession
of these lands occupied by Fillafuerte.
to be delivered when it is placed “in the hands and possession of the vendee.”
106. In August 1914, a surveyor named Santamaria went to Lucena at the request
Moreover, the Court cited Art. 1498 (topic of this case in the syllabus), which
of Addison and accompanied by him, in order to survey the land sold to Felix;
states “When the sale is made through a public instrument, the execution thereof
but he surveyed only two parcels which were as mentioned, occupied by
shall be equivalent to the delivery of the thing which is the object of the contract,
Villafuerte.
if from the deed the contrary does not appear or cannot clearly be inferred….”
107. Santamaria did not survey the other parcels, as they were not designated to
(But see doctrine to make sense of the case)
him by Addison.
108. In order to make said survey, it was necessary to obtain from the Land Court
DOCTRINE: It is true that the same article declares that the execution of a public
a writ of injunction against Villafuerte et al.
instruments is equivalent to the delivery of the thing which is the object of the
109. For the purpose of the issuance of said writ, Felix filed in June 1914 an
contract, but, in order that this symbolic delivery may produce the effect of
application with the Land Court for the registration in her name of the four
tradition, it is necessary that the vendor shall have had such control over the
parcels of land in the deed of sale.
thing sold that, at the moment of the sale, its material delivery could have
110. The proceedings were dismissed for failure to present the required plans
been made.
within the period of the time allowed for the purpose.
111. The trial court ruled in favor of Felix, holding the contract of sale to be
FACTS: rescinded and ordering the return of the P3,000 with interest. From this
99. By a public instrument dated June 11, 1914, Addison sold to Felix, with the judgment Addison appealed.
consent of her husband Tioco, four parcels of land described in the
instrument. ISSUE/s:
100. Felix paid, at the time of the execution of the deed P3,000 on account of the 22. WoN Felix can rescind the contract – YES, not because of the stipulation in
purchase price, and bound herself to pay the remainder in installments their contract but because of Article 1498 of the Civil Code
a. First: P2,000 on July 15, 1914
b. Second: P5,000 thirty days after the issuance to her of a certificate of title
RULING: It is therefore held that the contract of purchase and sale entered into by and
under the LRA
c. Further: Within 10 years from the date of issuance of said title, P10 for each between the plaintiff and the defendant on June 11, 1914, is rescinded, and the plaintiff is
ordered to make restitution of the sum of P3,000 received by him on account of the price of the are opposed by the interposition of another will, then fiction yields to reality
sale, together with interest thereon at the legal rate of 6 per annum from the date of the filing of — the delivery has not been effected.
the complaint until payment, with the costs of both instances against the appellant. So ordered. 12. French Civil Code: The word “delivery” expresses a complex idea . . . the
abandonment of the thing by the person who makes the delivery and the
RATIO:
taking control of it by the person to whom the delivery is made.
1. In decreeing the rescission of the contract, the trial judge rested his
conclusion solely on the indisputable fact that up to that time the lands sold 13. The execution of a public instrument is sufficient for the purposes of the
had not been registered under the Torrens system, and on the terms of the abandonment made by the vendor; but it is not always sufficient to permit of
clause in the contract (see 2f please!) the apprehension of the thing by the purchaser.
2. Addison objects, and rightly, that the cross-complaint is not founded on the 14. SC of Spain interpreting the provision: It merely declares that when the sale
conventional rescission relied upon by the court, but on the failure of the land is made through the means of a public instrument, the execution of this latter
sold. is equivalent to the delivery of the thing sold: which does not and cannot
3. He argues that the right to rescind by virtue of the special agreement does not
mean that this fictitious tradition necessarily implies the real tradition of the
accrue until the land is registered.
a. The one year was to be counted “from the date of the certificate of title…” thing sold, for it is incontrovertible that, while its ownership still pertains to
b. The right to elect rescission was subject to a condition: the issuance of the the vendor (and with greater reason if it does not), a third person may be in
title possession of the same thing; wherefore, though, as a general rule, he who
c. The condition has not yet been fulfilled
purchases by means of a public instrument should be deemed . . . to be the
4. However, although not in agreement with the reasoning of the lower court, it
is correct in result. possessor in fact, yet this presumption gives way before proof to the
5. The record shows that Addison did not deliver the thing sold. contrary."
6. With respect to two of the parcels, he was not even able to show them to 15. In the case at hand, it is evident that the mere execution of the instrument was
Felix, and as regards the other two, more than 2/3 of their area was in the not a fulfillment of the vendors' obligation to deliver the thing sold, and that
hostile possession of Villafuerte. from such non-fulfillment arises the purchaser's right to demand, as she has
7. The Code imposes upon the vendor the obligation to deliver the thing sold.
demanded, the rescission of the sale and the return of the price.
The thing is considered to be delivered when it is placed “in the hands and
possession of the vendee.”8 16. If the sale had been made under the express agreement of imposing upon
8. It is true that the same article declares that the execution of a public Felix the obligation to take the necessary steps to obtain the material
instruments is equivalent to the delivery of the thing which is the object of possession of the thing sold, and if it were proven that she knew that the thing
the contract, but, in order that this symbolic delivery may produce the effect was in possession of a third person claiming to have rights therein, such
of tradition, it is necessary that the vendor shall have had such control over agreement would be perfectly valid.
the thing sold that, at the moment of the sale, its material delivery could have 17. But nothing in the instrument indicates that such was the agreement.
been made.9 18. The clause (see 2e please!) obviously shows that it was not foreseen that Felix
9. It is not enough to confer upon the purchaser the ownership and right of might be deprived of her possession during the course of the registration
possession. The thing sold must be placed in his control. proceedings, but that the transaction rested on the assumption that she was to
10. When there is no impediment whatever to prevent the thing sold passing into have, during said period, the material possession and enjoyment of the fourt
parcels of land.
the tenancy of the purchaser by the sole will of the vendor, symbolic delivery
through the execution of a public instrument is sufficient.
11. But if, notwithstanding the execution of the instrument, the purchaser cannot
have the enjoyment and material tenancy of the thing and make use of it
himself or through another in his name, because such tenancy and enjoyment

8
Art. 1497, NCC With regard to movable property, its delivery may also be made by the delivery of the keys of the place or
9
Art. 1498: When the sale is made through a public instrument, the execution thereof shall be equivalent to depository where it is stored or kept
the delivery of the thing which is the object of the contract, if from the deed the contrary does not appear or
cannot clearly be inferred.
04 Danguilan v. IAC (Cyria) 117. Danguilan testified that he was the husband of Isidra Melad, Domingo's
November 28, 1988 | Cruz, J. | Symbolic delivery niece, whom he and his wife Juana Malupang had taken into their home as
their ward as they had no children of their own. He and his wife lived with
PETITIONER: Felix Danguilan the couple in their house on the residential lot and helped Domingo with the
RESPONDENTS: Intermediate Appellate Court, Apolonia Melad cultivation of the farm. Domingo Melad signed in 1941 a private instrument
in which he gave Danguilan the farm and in 1943 another private instrument
SUMMARY: Apolonia filed a complaint against Danguilan for the recovery of a in which he also gave him the residential lot, on the understanding that the
farm lot and residential lot. Apolonia claims that she purchased the property from latter would take care of the grantor and would bury him upon his death.
her father as evidenced by a Deed of Sale. Danguilan claims that the property was 118. Danguilan presented three other witnesses to corroborate his statements and
donated to him and that neither Apolonia nor her mother lived in the land with to prove that he had been living in the land since his marriage to Isidra and
him. SC held that Danguilan owns the land. Apolonia failed to show that she had remained in possession thereof after Domingo Melad's death in 1945.
consummated the contract of sale by actual delivery of the properties to her and Two of said witnesses declared that neither Apolonia nor her mother lived in
her actual possession thereof in concept of purchaser-owner. Symbolic delivery the land with Domingo Melad.
was not effected through the Deed of Sale. 119. CFI: Danguilan was more believable and that the Apolonia’s evidence was
"unpersuasive and unconvincing." Apolonia’s own declaration that she
DOCTRINE: In order that this symbolic delivery may produce the effect of moved out of the property in 1946 and left it in the possession of the
tradition, it is necessary that the vendor shall have had such control over the thing defendant was contradictory to her claim of ownership. She was also
sold that, at the moment of the sale, its material delivery could have been made. inconsistent when she testified first that the defendant was her tenant and later
in rebuttal that he was her administrator. Where there was doubt as to the
ownership of the property, the presumption was in favor of the one actually
FACTS: occupying the same, which in this case was Danguilan.
112. The subject of this dispute is the two lots owned by Domingo Melad which 120. IAC: Exhibits 2-b10 and 3-a,11 by virtue of which Domingo Melad had
is claimed by both Danguilan and Apolonia Melad (Apolonia). conveyed the two parcels of land to Danguilan, were null and void because
113. Apolonia filed a complaint against Danguilan in CFI Cagayan for recovery they were donations of real property and as such should have been effected
of a farm lot and a residential lot which she claimed she had purchased from through a public instrument. It then set aside the appealed decision and
Domingo Melad in 1943 and were now being unlawfully withheld by the declared the respondents the true and lawful owners of the disputed property.
Danguilan.
114. Danguilan averred that he was the owner of the said lots of which he had been
in open, continuous and adverse possession, having acquired them from ISSUE: WoN the symbolic delivery of the property was effected through the Deed of
Domingo Melad in 1941 and 1943. Sale – NO
115. The case was dismissed for failure to prosecute but was refiled in 1967.
116. At the trial, Apolonia presented a deed of sale dated December 4, 1943, RULING: WHEREFORE, the decision of the respondent court is SET ASIDE and
purportedly signed by Domingo Melad and duly notarized, which conveyed that of the trial court REINSTATED, with costs against the private respondent.
the said properties to her for the sum of P80.00. She said the amount was
earned by her mother as a worker at the Tabacalera factory. She claimed to RATIO:
be the illegitimate daughter of Domingo Melad, with whom she and her Donation to Danguilan
mother were living when he died in 1945. She moved out of the farm only 1. Considering the language of the two instruments, that Domingo Melad did
when in 1946 Felix Danguilan approached her and asked permission to intend to donate the properties to the petitioner, as the private respondent
cultivate the land and to stay therein. She had agreed on condition that he contends. We do not think, however, that the donee was moved by pure
would deliver part of the harvest from the farm to her, which he did from that liberality. While truly donations, the conveyances
year to 1958. The deliveries having stopped, she then consulted the municipal were onerous donations as the properties were given to the petitioner in
judge who advised her to file the complaint against Danguilan. Her mother, exchange for his obligation to take care of the donee for the rest of his
her only other witness, corroborated this testimony. life and provide for his burial. Hence, it was not covered by the rule in

10 11
I will deliver this land except to him as he will be the one responsible for me in the event that I will die I have thought of giving him my land because he will be the one to take care of SHELTERING me or
and also for all other things needed and necessary for me, he will be responsible because of this land I am bury me when I die and this is why I have thought of executing this document
giving to him
Article 749 of the Civil Code requiring donations of real properties to be 8. Ownership does not pass by mere stipulation but only by delivery, and the
effected through a public instrument. execution of a public document does not constitute sufficient delivery where
2. Apolonia argues that as there was no equivalence between the value of the the property involved is in the actual and adverse possession of third persons.
lands donated and the services for which they were being exchanged. 9. In our Civil Code it is a fundamental principle in all matters of contracts and
However, no evidence has been adduced to support her contention that the a well- known doctrine of law that "non mudis pactis sed traditione dominia
values exchanged were disproportionate or unequal. rerum transferuntur". In conformity with said doctrine as established in
3. On the other hand, both the trial court and the respondent court have affirmed paragraph 2 of article 609 of said code, that "the ownership and other property
the factual allegation that the petitioner did take care of Domingo Melad and rights are acquired and transmitted by law, by gift, by testate or intestate
later arranged for his burial in accordance with the condition imposed by the succession, and, in consequence of certain contracts, by tradition". And as
donor. It is alleged and not denied that he died when he was almost one the logical application of this disposition article 1095 prescribes the
hundred years old, which would mean that Danguilan farmed the land following: "A creditor has the rights to the fruits of a thing from the time the
practically by himself and so provided for the donee (and his wife) during the obligation to deliver it arises. However, he shall not acquire a real right" (and
latter part of Domingo Melad's life. We may assume that there was a fair the ownership is surely such) "until the property has been delivered to him."
exchange between the donor and the donee that made the transaction an 10. The delivery of a thing constitutes a necessary and indispensable
onerous donation. requisite for the purpose of acquiring the ownership of the same by
virtue of a contract.
Alleged sale to Apolonia 11. As for the argument that symbolic delivery was effected through the deed of
4. The deed of sale was allegedly executed when Apolonia was only three years sale, which was a public instrument, the Court has held: In order that this
old and the consideration was supposedly paid by her mother, Maria Yedan symbolic delivery may produce the effect of tradition, it is necessary that
from her earnings as a wage worker in a factory. This was itself a suspicious the vendor shall have had such control over the thing sold that, at the
circumstance, one may well wonder why the transfer was not made to the moment of the sale, its material delivery could have been made. It is not
mother herself, who was after all the one paying for the lands. The sale was enough to confer upon the purchaser the ownership and the right of
made out in favor of Apolonia Melad although she had been using the possession. The thing sold must be placed in his control.
surname Yedan her mother's surname, before that instrument was signed and 12. There is no dispute that it is Danguilan and not Apolonia who is in actual
in fact even after she got married. possession of the litigated properties. Even if the respective claims of the
5. The averment was also made that the contract was simulated and prepared parties were both to be discarded as being inherently weak, the decision
after Domingo Melad's death in 1945. It was also alleged that even after the should still incline in favor of Danguilan pursuant to the doctrine: If the claim
supposed execution of the said contract, Apolonia considered Domingo of both the plaintiff and the defendant are weak, judgment must be for the
Melad the owner of the properties and that she had never occupied the same. defendant, for the latter being in possession is presumed to be the owner, and
6. Considering these serious challenges, IAC could have devoted a little more cannot be obliged to show or prove a better right.
time to examining Exhibit "E" and the circumstances surrounding its
execution before pronouncing its validity in the manner described above.
While it is true that the due execution of a public instrument is presumed, the
presumption is disputable and will yield to contradictory evidence, which in
this case was not refuted.
7. At any rate, even assuming the validity of the deed of sale, the record
shows that Apolonia did not take possession of the disputed properties
and indeed waited until 1962 to file this action for recovery of the lands
from the petitioner. If she did have possession, she transferred the same to
Danguilan in 1946, by her own sworn admission, and moved out to another
lot belonging to her step-brother. Her claim that Danguilan was her tenant
(later changed to administrator) was disbelieved by the trial court, and
properly so, for its inconsistency. In short, she failed to show that she
consummated the contract of sale by actual delivery of the properties to
her and her actual possession thereof in concept of purchaser-owner.
05 POWER COMMERCIAL & INDUSTRIAL CORPORATION vs. CA ownership and the right of possession unless the thing sold is placed in the
(Gueco) control of the buyer.
June 20, 1997 | Panganiban, J. | Symbolic Delivery
In this case, the lot sold had been placed under the control of Power Commercial;
PETITIONER: Power Commercial and Industrial Corporation thus, the filing of the ejectment suit was subsequently done. It signified that its
RESPONDENTS: Court of Appeals, Spouses Reynaldo and Angelita R. new owner intended to obtain for itself and to terminate said occupants' actual
Quiambao and Philippine National Bank possession thereof. Prior physical delivery or possession is not legally required
and the execution of the deed of sale is deemed equivalent to delivery. This
SUMMARY: Power Commercial, an asbestos manufacturer, needed a bigger deed operates as a formal or symbolic delivery of the property sold and
office space for its products, so it entered into a contract of sale with spouses authorizes the buyer to use the document as proof of ownership. Nothing
Quiambao. The subject matter of the sale was a parcel of land located in Makati. more is required.
In the contract, they agreed that Power Commercial would make a down payment
of P108,000 and the remaining balance to be paid upon the execution of the deed DOCTRINE:
of transfer of the title. Power Commercial also agreed to assume the existing The Civil Code provides that delivery can either be actual or constructive.
mortgage on the land, which the spouses made in favor of PNB. Subsequently, Symbolic delivery is a specie of constructive delivery, and it is effected by the
the spouses again mortgaged the Makati land to PNB because they obtained a transfer of ownership through the execution of a public document. Its efficacy
loan from the said bank. Power Commercial knew about this and even agreed that can, however, be prevented if the vendor does not possess control over the thing
it will pay the loan of the spouses. Pursuant to this agreement, the spouses and sold. The key word is control, not possession, of the land. In order for symbolic
Power Commercial proceeded to execute a Deed of Absolute Sale with delivery to produce the effect of tradition (delivery), it is first necessary that the
Assumption of Mortgage. Afterwards, the general manager of Power Commercial vendor shall have had control over the thing sold. It is not enough to confer upon
submitted to PNB a copy of the deed along with a formal application for the the purchaser the ownership and the right of possession. The thing sold must also
assumption of mortgage. However, this application of mortgage was not accepted be placed in his control.
because Power Commercial failed to submit the necessary papers. As a result, the
loan obtained by the spouses Quiambao became due and demandable. Power
Commercial made partial payments of the loan, but it defaulted in the end. So it FACTS:
filed a civil case against the Spouses for the rescission of their contract plus 1. Power Commercial & Industrial Development Corporation (“Power
damages. Power Commercial alleged that there was a substantial breach in their Commercial”) is an asbestos12 manufacturer. It needed a bigger office space
contract because the spouses failed to deliver actual possession of the property and warehouse for its products, so it entered into a contract of sale with
and also because the spouses failed to eject the lessees that occupied the land. spouses Reynaldo and Angelita Quiambao (“Spouses Quiambao”).
Also, Power Commercial demanded from PNB the return of the payments that it 2. The subject of the sale was a parcel of land located at San Antonio Village of
made to the latter. It contended that since its application for the assumption of Makati. In the contract, they agreed that Power Commercial would pay
mortgage was not accepted, PNB had no right to receive payments from Power P108,000 as down payment and the remaining balance would be paid upon
Commercial. RTC ruled in favor of Power Commercial. It stated that the failure the execution of the deed of transfer of the title. Furthermore, Power
of the spouses to deliver actual possession gave the corporation the right to Commercial assumed the existing mortgage on the land, which was made in
rescind their contract. On appeal, the CA reversed the decision of the RTC. favor of PNB.
ISSUE: WoN the alleged failure of the spouses to deliver actual and physical 3. Subsequently, on June 1979, spouses Quiambao mortgaged (again) the above
possession can be considered a substantial breach of condition—NO. RULING: land to PNB in order to guarantee a loan (P145, 000). Power Commercial was
The Civil Code provides that delivery can either be actual or constructive. aware of this and even agreed to assume the payment of the spouses’ loan.
Symbolic delivery is specie of constructive delivery, and it is effected by the Spouses Quiambao and Power Commercial then executed a Deed of Absolute
transfer of ownership through the execution of a public document. Its efficacy Sale with Assumption of Mortgage. On the same day of the deed’s execution,
can, however, be prevented if the vendor does not possess control over the the general manager of Power Commercial submitted to PNB a copy of the
thing sold. In other words, for symbolic delivery to produce the effect of deed with a formal application for the assumption of mortgage.
tradition (delivery), it is first necessary that the vendor shall have had 4. On February 1980, however, PNB informed Spouses Quiambao that Power
control over the thing sold. It is not enough to confer upon the purchaser the Commercial failed to submit the necessary papers for the approval of their

12
Asbestos refers to a set of six naturally occurring fibrous minerals
application to assume the mortgage; hence, their application with PNB is 5. It is not enough to confer upon the purchaser the ownership and the right
deemed withdrawn and the outstanding balance of the loan is now due. of possession. The thing sold must also be placed in his control. When
5. Power Commercial made partial payments of the loan but they still defaulted there is no impediment whatsoever to prevent the thing sold passing into the
in the end. So it filed a civil case against Spouses Quiambao for rescission tenancy of the purchaser by the sole will of the vendor, symbolic delivery
and damages. through the execution of a public instrument is sufficient. But if,
6. Power Commercial then wrote a letter to PNB demanding the return of the notwithstanding the execution of the instrument, the purchaser cannot have
payments that it made since the assumption of the mortgage was never the enjoyment and material tenancy of the thing and make use of it himself
approved. or through another in his name, because such tenancy and enjoyment are
7. Later on, PNB was impleaded as a party to the case filed by Power opposed by the interposition of another will, then fiction yields to reality —
Commercial because the subject mortgage was foreclosed and it was PNB the delivery has not been effected.
who was the highest bidder at the public auction. 6. In this case, delivery was effected through the execution of the deed. The lot
8. RTC ruled that the Spouses’ failure to deliver actual possession to Power sold had been placed under the control of Power Commercial; thus, the filing
Commercial entitled the latter to rescind the sale. Also, PNB is obliged to of the ejectment suit was subsequently done. It signified that its new owner
return the payments made by Power Commercial since the latter’s assumption intended to obtain for itself and to terminate said occupants' actual possession
of mortgage was never approved. (Side note: bigla nalang talaga nagkaroon thereof. Prior physical delivery or possession is not legally required and
ng ruling regarding delivery.) the execution of the deed of sale is deemed equivalent to delivery. This
9. Spouses Quiambao and PNB appealed. deed operates as a formal or symbolic delivery of the property sold and
10. CA reversed the decision. It ruled that it held that the deed of sale between authorizes the buyer to use the document as proof of ownership. Nothing
the spouses and Power Commercial did not obligate the former to eject the more is required.
lessees from the land in question as a condition of the sale, nor was the
occupation thereof by said lessees a violation of the warranty against eviction.
Hence, there was no substantial breach to justify the rescission of said
contract or the return of the payments made. (Side note: wala din sinabi sa
facts about lessees occupying the land).

ISSUES:
2. WoN the alleged failure of the spouses to deliver actual and physical
possession can be considered a substantial breach of condition—NO.

RULING: WHEREFORE, the petition is hereby DENIED, and the assailed Decision
is AFFIRMED.

RATIO:
1. Power Commercial asserts that there was no delivery because at the execution
of the deed of sale, transfer of possession of said lot was impossible due to
the presence of occupants on the lot sold. The Court does not agree with its
contention.
2. The Civil Code provides that delivery can either be (1) actual (Article 1497)
or (2) constructive (Articles 1498-1501). Symbolic delivery (Article 1498),
as a specie of constructive delivery, effects the transfer of ownership through
the execution of a public document. Its efficacy can, however, be prevented
if the vendor does not possess control over the thing sold.
3. The key word is control, not possession, of the land.
4. In order for symbolic delivery to produce the effect of tradition
(delivery), it is first necessary that the vendor shall have had control over
the thing sold.
06 CHUA v. CA (REINE) 3.) In the morning of July 13, 1989, Chua secured from Philippine Bank of Commerce
9 April 2003| Carpio, J | Steps in Selling Immovables (PBCom) a manager’s check for P480,000. In the afternoon of the same day, Chua and
Petitioner/s:Tomas K. Chua Valdes-Choy met with their respective counsels to execute the necessary documents and
arrange the payments. Valdes-Choy and Chua signed two Deeds of Absolute Sale
(Deeds of Sale). The first Deed of Sale covered the house and lot for the purchase price
Respondent/s:Court of Appeals and Encarnion Valdes-Choy of P8 million. The second Deed of Sale covered the furnishings, fixtures and movable
properties contained in the house for P2.8 million. The computation of capital gains
Summary: Valdes-Choy advertised for sale a house and lot at San Lorenzo Village amounted to P485K.
in which Chua responded and eventually they both agreed on a purchase price of 4.) On July 14, 1989 (the next day) the parties met at the office of Valdes-Choy’s
P10.8 million payable in cash. Chua gave P100,000 as earnest money and P485,000 counsel. Chua handed to Valdes-Choy the PBCom managers check for P485,000.00 so
in order for Valdes-Choy to pay the capital gains tax. However, Chua did not want Valdes-Choy could pay the capital gains tax as she did not have sufficient funds to pay
to pay the balance of the purchase price until Valdes-Choy registered the property the tax. Valdes-Choy issued a receipt showing that Chua had a remaining balance
under the name of Chua but Valdes-Choy stated that it was not part of the agreement of P10,215,000.00 after deducting the advances made by Chua. The receipt reads:
thus the two parties had reached an impasse (Disagreement). The deadline for the “…The total purchase price of the above-mentioned property is TEN MILLION EIGHT
payment of the purchase price had already lapsed (July 15) and there was still no HUNDRED THOUSAND PESOS only, broken down as follows: SELLING
agreement and 2 days after (July 17), Chua filed a complaint for specific PRICE P10,800,000.00 EARNEST MONEY P100,000.00
performance with damages. The Trial Court ruled in favor of Chua but the CA PARTIAL PAYMENT 485,000.00 585,000.00 BALANCE DUE TO ENCARNACION VALDEZ-
reversed the decision. CHOY P10,215,000.00 PLUS P80,000.00 for documentary stamps paid in advance by
seller 80,000.00
The SC affirmed the CA decision and ruled in favor of Valdes-Choy stating that the
contract was not a contract of sale but a contract to sell hence ownership is not 5.) On the same day, Valdes-Choy accompanied by Chua deposited the P485,00 check
passed on to the buyer until payment of the full purchase price. Thus, Chua cannot to her account with Traders Royal Bank (TRB). She then purchased a Traders Royal
withhold payment of the purchase price by stating that the property be first registered Bank managers check for P480,000 payable to the Commission of Internal Revenue for
in his name because it was not part of the agreement. the capital gains tax and both parties returned to the office of Valdes-Choy’s counsel
and gave the TRB check to counsel to pay the capital gains tax. It was also then that
Doctrine: In the absence of a contrary agreement, the submission by an individual Chua showed to Valdes-Choy a PBCom manager’s check for P10, 215,000 representing
seller to the buyer of the following papers would complete a sale of real estate: (1) the balance of the purchase price but Chua did not give this check because the property
owners duplicate copy of the Torrens title; (2) signed deed of absolute sale; (3) tax was still registered under Valdes-Choy. Chua required that the property be registered
declaration; and (3) latest realty tax receipt. The buyer can retain the amount for the first in his name before he would turn over the check to Valdes-Choy which angered
capital gains tax and pay it upon authority of the seller, or the seller can pay the tax, Valdes-Choy who tore up the Deeds of Sale claiming that what Chua required was not
depending on the agreement of the parties. part of the agreement.
Facts: 6.)On July 15 (Deadline for payment of balance of purchase price), Valdes-Choy
1.) Valdes-Choy (Seller) advertised for sale her paraphernal house and lot with 718 suggested to her counsel that Chua should deposit in escrow the P10, 215,000 balance.
square meters located at San Lorenzo Village. Chua (buyer) responded to the Upon such deposit, Valdes-Choy was willing to cause the issuace of new TCT in the
advertisement and after several meetings, Chua and Valdes-Choy agreed on a purchase name of Chua. Valdes-Choy believed this was the only way she could protect herself if
price of P10.8 million payable in cash. Valdes-Choy received from Chua a check for the certificate of title is transferred in the name of Chua before she is fully paid. This
P100,000. was relayed to Chua but there was no response from Chua.
2.) The receipt evidencing the transaction signed by Valdes-Choy as seller and Chua as 7.) On July 17, Chua filed a complaint for specific performance against Valdes-Choy
buyer reads: “RECEIVED from MR. TOMAS K. CHUA PBCom Check No. 206011 in which was dismissed and then he re-filed his complaint for specific performance with
the amount of ONE HUNDRED THOUSAND PESOS ONLY (P100,000.00) as damages. This time, the Trial Court rendered judgment in favor of Chua. The Trial Court
EARNEST MONEY for the sale of the property located at 40 Tampingco cor. Hidalgo, found that the transaction reached an impasse (situation in which no progress is possible
San Lorenzo Village, Makati, Metro Manila (Area : 718 sq. meters).The balance of TEN due to disagreement) wherein Valdes-Choy wanted to be paid first the full consideration
MILLION SEVEN HUNDRED THOUSAND (P10,700,000.00) is payable on or before before a new TCT would be issued in the name of Chua while Chua did not want to pay
15 July 1989. Capital Gains Tax for the account of the seller. Failure to pay balance on unless a new TCT is first issued in his name. The Trial Court held that Chua complied
or before 15 July 1989 forfeits the earnest money. This provided that all papers are in with the terms of the contract to sell such as when he was prepared to pay the
proper order. consideration before the deadline and the purchasing of manager’s checks payable to
Valdes-Choy. On the other hand, Valdes-Choy did not perform her obligation under the relative to the sale which are additional proof that the agreement did not transfer to Chua,
contract to sell to put all the papers in order and the non-payment of the balance by Chua either by constructive or actual delivery the ownership of the Property.
was due to the fault of Valdes-Choy. 4.) It is true that Article 1482 of the Civil Code provides that Whenever earnest money
8.) The CA reversed the ruling of the Trial Court wherein the CA ruled that Chua’s is given in a contract of sale, it shall be considered as part of the price and proof of
stance to pay the consideration only after the Property is registered in his name was not the perfection of the contract. However, this article speaks of earnest money given
the agreement of the parties. Although Chua demonstrated his capacity to pay, this could in a contract of sale. In this case, the earnest money was given in a contract to
not be equated with actual payment which he refused to do. The CA did not consider sell. The Receipt evidencing the contract to sell stipulates that the earnest money is a
the non-payment of the capital gains tax as failure by Valdes-Choy to put the papers in forfeitable deposit, to be forfeited if the sale is not consummated should Chua fail to pay
order since the payment of the capital gains tax has no bearing on the validity of Deeds the balance of the purchase price. Valdes-Choy should have the right to compel Chua to
of Sale (NOTE: Counsel of Valdes-Choy did not pay the capital gains tax so this was pay the balance of the purchase price. Chua, however, has the right to walk away from
one of the defense of Chua of Valdes-Choy not putting the papers in order). the transaction, with no obligation to pay the balance, although he will forfeit the earnest
money. Clearly, there is no contract of sale.
Issue: WoN there is a perfected contract of sale of immovable property or a mere 5.) Since the agreement between Valdes-Choy and Chua is a mere contract to sell, the
contract to sell? Contract to Sell full payment of the purchase price partakes of a suspensive condition. The non-
fulfillment of the condition prevents the obligation to sell from arising and ownership is
WoN Chua can compel Valdes-Choy to cause the issuance of a new TCT in Chua’s retained by the seller without further remedies by the buyer.
name even before payment of the full purchase price? NO Withholding of Payment of Balance of Purchase Price:
6.) Chua insists that he was ready to pay the balance of the purchase price but withheld
Ruling: The Decision of the CA is affirmed in toto. payment because Valdes-Choy did not fulfill her contractual obligation to put all the
Ratio: papers in proper order. Specifically, Chua claims that Valdes-Choy failed to show that
the capital gains tax had been paid after he had advanced the money for its payment. For
1.) Chua has consistently before the Trial Court and Appellate Courts characterized his
the same reason, he contends that Valdes-Choy may not forfeit the earnest money even
agreement with Valdes-Choy as evidenced by Receipt as a contract to sell and this is the
if he did not pay on time. SC Ruling: In a contract to sell, the seller is not obligated
first time he pleads that there is a perfected contract of sale. The SC held that Chua’s
to transfer ownership to the buyer. Neither is the seller obligated to cause the
theory is not well taken since an issue not raised in the lower courts cannot be raised for
issuance of a new certificate of title in the name of the buyer. However, the seller
the first time on appeal as this is offensive to the basic rules of fair play, justice and due
must put all his papers in proper order to the point that he is in a position to
process.
transfer ownership of the real property to the buyer upon the signing of the
2.) The SC held that the agreement between Valdes-Choy and Chua was a contract to contract of sale. In the instant case, Valdes-Choy was in a position to comply with
sell not a contract of sale. The distinction is as follows: In a contract of sale, the title all her obligations as a seller under the contract to sell. First, she already signed
to the property passes to the vendee upon the delivery of the thing sold; in a the Deeds of Sale in the office of her counsel in the presence of the buyer.Second,
contract to sell, ownership is, by agreement, reserved in the vendor and is not to she was prepared to turn-over the owners duplicate of the TCT to the buyer, along
pass to the vendee until full payment of the purchase price.Otherwise stated, in a with the tax declarations and latest realty tax receipt.Clearly, at this point Valdes-
contract of sale, the vendor loses ownership over the property and cannot recover
Choy was ready, able and willing to transfer ownership of the Property to the buyer
it until and unless the contract is resolved or rescinded; whereas, in a contract to as required by the contract to sell. Chua, however, refused to give to Valdes-Choy
sell, title is retained by the vendor until full payment of the price. In the latter
the PBCom managers check for the balance of the purchase price. Chua imposed
contract, payment of the price is a positive suspensive condition, failure of which is
the condition that a new TCT should first be issued in his name, a condition that is
not a breach but an event that prevents the obligation of the vendor to convey title found neither in the law nor in the contract to sell as evidenced by the Receipt.Thus,
from becoming effective. at this point Chua was not ready, able and willing to pay the full purchase price
3.) The Receipt shows the true agreements between the parties was a contract to sell. which is his obligation under the contract to sell.
The Receipt provides that the earnest money will be forfeited in case the buyer fails to 7.) In the absence of a contrary agreement, the submission by an individual seller to
pay the balance of the purchase price on or before July 15 and in such event, Valdes- the buyer of the following papers would complete a sale of real estate: (1) owners
Choy can sell the property to other interested parties. Second, the agreement between
duplicate copy of the Torrens title; (2) signed deed of absolute sale; (3) tax
Chua and Valdes-Choy was embodied in a receipt rather than in a deed of sale,
declaration; and (3) latest realty tax receipt. The buyer can retain the amount for
ownership not having passed between them, the absence of a formal deed of conveyance the capital gains tax and pay it upon authority of the seller, or the seller can pay
is a strong indication that parties did not intend immediate transfer of ownership. Third,
the tax, depending on the agreement of the parties.The buyer has more interest in
Valdes-Choy retained possession of the certificate of title and all other documents
having the capital gains tax paid immediately since this is a pre-requisite to the issuance
of a new Torrens title in his name. Nevertheless, as far as the government is concerned,
the capital gains tax remains a liability of the seller since it is a tax on the sellers gain
from the sale of the real estate. Payment of the capital gains tax, however, is not a
pre-requisite to the transfer of ownership to the buyer. The transfer of ownership
takes effect upon the signing and notarization of the deed of absolute sale.
8.) In this case, Valdes-Choy was ready, able and willing to submit to Chua all the papers
that customarily would complete the sale, and to pay as well the capital gains tax. On
the other hand, Chuas condition that a new TCT be first issued in his name before he
pays the balance of P10,215,000.00, representing 94.58% of the purchase price, is not
customary in a sale of real estate. Such a condition, not specified in the contract to sell
as evidenced by the Receipt, cannot be considered part of the omissions of stipulations
which are ordinarily established by usage or custom. Chua had no reason to fear being
swindled. Valdes-Choy was prepared to turn-over to him the owners duplicate copy of
the TCT, the signed Deeds of Sale, the tax declarations, and the latest realty tax
receipt. There was no hindrance to paying the capital gains tax as Chua himself had
advanced the money to pay the same and Valdes-Choy had procured a managers check
payable to the Bureau of Internal Revenue covering the amount. It was only a matter of
time before the capital gains tax would be paid. Chua acted precipitately in filing the
action for specific performance a mere two days after the deadline of 15 July 1989 when
there was an impasse.
07 Vive Eagle Land v. CA (Steph) 24. Genuino filed a Complaint against VELI for specific performance and
November 26, 2005 | Callejo, Sr., J. | Expenses for registration must be shouldered damages in the RTC. RTC ruled in favor of Genuino and ordered VELI to
by the vendor in the absence of a stipulation to the contrary. pay capital gains tax, transfer title in favor of Genuino, and evict the squatters,
because Genuino is not a party to the initial deed of sale (between Spouses
PETITIONER: Vive Eagle Land Inc. and Virgilio O. Cervantes Flores and TATIC, and between TATIC AND VELI), thus not bound by any
RESPONDENTS: Court of Appeals, Genuino Ice Co., Inc. of that.
SUMMARY: Spouses first sold 2 lots to TATIC, then TATIC sold 2 lots to 25. CA affirmed.
VELI. VELI then sold one of the lots to Genuino. Genuino is ordering VELI to ISSUES: Is VELI obliged to pay for registration and capital gains tax? Yes for
pay the capital gains tax over the lot. (So there are 3 deeds of sale.) However, registration, no for capital gains tax because laws cannot be applied retroactively.
VELI disagrees as in its previous Deeds of Absolute Sale it was agreed that the
Broker Tobias would pay the capital gains tax. The Court ruled that VELI must RULING: IN LIGHT OF ALL THE FOREGOING, the petition is PARTIALLY
shoulder expenses for registration and eviction of squatters because Genuino is GRANTED. The decision of the Court of Appeals in CA-G.R. CV No. 51933 is
not bound by the terms and conditions of the two previous deeds of sale even if hereby AFFIRMED WITH MODIFICATION. That portion of the Decision of the
VELI assigned its rights to Genuino. However, VELI is not liable for capital gains Court of Appeals mandating petitioner Vive Eagle Land, Inc. to pay capital gains tax
tax because the law applicable at the time of the third deed of sale does not require for the November 11, 1988 sale of the property covered by TCT No. 241846 to
them to pay such tax, and the current law cannot be applied retroactively. respondent Genuino Ice Co., Inc. is DELETED. No costs.

DOCTRINE: Under Article 1487, expenses for the registration of the sale should RATIO:
be shouldered by the vendor unless there is a stipulation to the contrary.

FACTS: 7. VELI is Obliged to Cause the Registration of the Deed of Absolute


18. Spouses Flores sold 2 lots to TATIC. Sale in Favor of Genuino, the Issuance of a Torrens Title in the Name
19. TATIC loaned money from Makati Rural Bank to pay for the lots. of Genuino and the Eviction of the Tenants/Occupants from the
20. Spouses Flores entered into a Deed of Absolute Sale in favor of TATIC for Property at the Expense of the VELI.
P 7.9 M. Also, Spouses Flores, Tobias (the broker of TATIC), TATIC, and 8. Remember that there are 3 Deeds of Absolute Sale:
the Bank entered into a Memorandum of Agreement (so 2 contracts) a. Spouses Flores and TATIC (Tobias the Broker of TATIC),
a. Tobias the broker undertook to clear the titles and Spouses Flores wherein Tobias would pay capital gains tax on behalf of
undertook to give necessary documents to broker. TATIC
b. Broker also undertook to pay all taxes, receipts to be presented to
b. TATIC and VELI (TATIC DID NOT OBLIGE TO PAY
the Bank because the Bank will advance money on the loan provided
TATIC executes a promissory note in favor of Bank. The taxes will
CAPITAL GAINS TAX)
be deducted from the purchase price. c. VELI and Genuino
c. Broker and TATIC is in charge of removing squatters from the lots, 9. Indeed, under the third deed of absolute sale, petitioner VELI did not oblige
with help from spouses Flores. itself to spend for the registration of the said deed; to secure a torrens title
d. Estimated expense for all these matters is roughly 790 000, and if it over the property to and under the name of the respondent; or to cause the
exceeds that it will be deducted from whatever the Broker was eviction of the tenants/occupants on the property. Nevertheless, petitioner
supposed to receive (in short kaltas sa kita niya). VELI is liable for registration expenses because, under Article 1487 of
21. While the torrens titles were still with the Bank, TATIC sold the 2 lots to the New Civil Code, the expenses for the registration of the sale should
Vive Eagle Land Inc. (VELI) for P 6.9 M. TATIC warranted in the Deed of be shouldered by the vendor unless there is a stipulation to the contrary.
Sale that it would deliver the torrens titles. 10. By virtue of the deed of assignment yes Genuino got the rights in the first
22. AND THEN, VELI executed a Deed of Absolute Sale and sold one of the deed of sale between Spouses Flores and VELI, HOWEVER, VELI
lots to Genuino Ice Co. for P 4.9 M. VELI assigned its rights in the Deed of CANNOT ENFORCE THE TERMS AND CONDITIONS OF THE FIRST
Absolute Sale between Spouses Flores and TATIC to Genuino, as far as that DEED OF SALE BECAUSE GENUINO IS NOT A PARTY TO THAT.
one lot is concerned. 11. This deed of assignment also did not relieve VELI from its obligation to evict
23. Genuino wrote a letter to VELI demanding that VELI must pay the capital the squatters.
gains tax. VELI rejected these demands. 12. In the third deed of sale, the squatters have not been evicted yet by VELI,
thus it can’t be said that there was delivery already of the property. Moreover,
Genuino had a right to withhold 300 000 of the purchase price if the squatters
haven’t been evicted yet, according to their Deed of Sale.
13. BUT VELI is NOT liable to pay capital gains tax for the sale.
a. During the first and second deeds of sale, the National Internal
Revenue Code (NIRC), as amended by Batas Pambansa Blg. 37, and
Executive Order No. 237 were still in effect. Under these laws,
Spouses Flores were liable as vendors in the first sale, but in the
second sale neither VELI or TATIC had to pay capital gains tax
because they are corporations.
b. During the third deed of sale (which was executed in 1988), VELI
is not liable to pay corporate gains tax. But, NIRC was amended in
1997 and now requires corporations to pay capital gains tax. But,
this cannot be applied retroactively, thus VELI is still not liable.
14. SO, in sum, VELI has to pay registration expenses and evict squatters
and pay for whatever costs they may incur in that, however VELI is not
liable for corporate gains tax.
08 CHUA v. CA (ARIELLE) a contract of sale.
April 9, 2003 | Carpio, J. | Contract of Sale vs. Contract to Sell; Forfeiture of Earnest
Money DOCTRINE: In a contract of sale, the title of the property passes to the vendee
upon the delivery of the thing sold. In a contract to sell, ownership is, by
PETITIONER: Tomas K. Chua agreement, reserved in the vendor and is not to pass to the vendee until full
RESPONDENTS: Court of Appeals and Encarnacion Valdes-Choy payment of the purchase price.

SUMMARY: Valdes-Choy advertised her house and lot, to which Chua


responded to. Valdes-Choy and Chua then agreed on a purchase price of FACTS:
P10,800,000 payable in cash. Chua gave Valdes-Choy a check for P100,000 as 1. Valdes-Choy advertised for sale her paraphernal house and lot with an area
earnest money, as embodied in a receipt. Chua then secured from PBCom a of 718 square meters located at San Lorenzo Village, Makati City. Chua
managers check but the latter immediately gave a stop payment order claiming responded to the advertisement. After several meetings, Chua and Valdes-
that his check for P480,000 was lost. The contracting parties met again with their Choy agreed on a purchase price of P10,800,00.00 payable in cash.
counsels and Chua gave Valdes-Choy a PBCom managers check for P485,000. 2. Valdes-Choy received from Chua a check for P100,000.00. The receipt of the
Valdes-Choy then issued a receipt showing that Chua had a remaining balance of transaction was signed by Valdes-Choy as seller, and Chua as buyer. The
P10,215,000. Chua did not give to Valdes-Choy the managers check because the receipt provided that Valdes-Choy received from Mr. Chua a PBCom Check
TCT was still registered in the name of Valdes-Choy. Chua wanted the property as Earnest Money for the sale of the property in San Lorenzo Village.
to be registered first in his name before he would turn over the check to Valdes- 3. Chua then secured from PBCom a managers check for P480,000.00.
Choy, which the latter claimed was not part of their agreement. Since the deadline Strangely, after securing the managers check, Chua immediately gave
for the payment of the balance was nearing, Valdes-Choy suggested that Chua PBCom a verbal stop payment order claiming that his managers check for
deposit the check in escrow and upon such deposit, Valdes-Choy would cause the P480,000 was lost/misplaced.
issuance of a new TCT in Chua’s name even without receiving the balance. Chua 4. Chua and Valdes-Choy met with their counsels to execute documents and
filed a complaint for specific performance which the trial court dismissed, but arrange the payments. Valdes-Choy as vendor and Chua as vendee signed
upon re-filing, the trial court rendered judgment in favor of Chua. Valdes-Choy two Deeds of Absolute Sale. The first Deed of Sale covered the house and lot
appealed to the CA which reversed the trial court’s decision and declared the for P8M. The second Deed of Sale covered the furniture and movable
P100,000 earnest money as forfeited. Hence this appeal. The issue is WoN there properties contained in the house for P2.8M.
is a perfected contract of sale, and WoN the earnest money can be forfeited. 5. The parties met again at the office of Valdes-Choy’s counsel. Chua handed
to Valdes-Choy the PBCom managers check for P485,000 so Valdes-Choy
The SC held that there is no perfected contract of sale, but only a contract to sell. could pay the capital gains tax as she did not have sufficient funds to pay the
A perusal of the Receipt shows that the true agreement between the parties was a tax.
contract to sell. Ownership over the property was retained by Valdes-Choy and 6. Valdes-Choy then issued a receipt showing that Chua had a remaining
was not to pass to Chua until full payment of the purchase price. The agreement balance of P10,215,000.
between Chua and Valdes-Choy was embodied in a receipt rather than in a deed 7. Valdes-Choy accompanied by Chua deposited the P485,000 managers check
of sale, ownership not having passed between them. The signing of the Deeds of to her account with Traders Royal Bank. She purchased a managers check for
Sale came later when Valdes-Choy was under the impression that Chua was about P480,000 payable to the CIR for the capital gains tax. Chua showed to
to pay the balance. Absence of a formal deed of conveyance is a strong indication Valdes-Choy a PBCom managers check for P10,215,000 representing the
that the parties did not intend immediate transfer of ownership, but only a transfer balance of the purchase price.
after full payment of the purchase price. The SC also held that the earnest money 8. However, Chua did not give this PBCom managers check to Valdes-Choy
can be forfeited in favor of Valdes-Choy because The Receipt provides that the because the TCT was still registered in the name of Valdes-Choy.
earnest money shall be forfeited in case the buyer fails to pay the balance. In such 9. Chua wanted the property to be registered first in his name before he would
event, Valdes-Choy can sell the property to other interested parties. There is in turn over the check to Valdes-Choy. This angered Valdes-Choy who tore up
effect a right reserved in favor of Valdes-Choy not to push through with the sale the Deeds of Sale, claiming that what Chua required was not part of their
upon Chua’s failure to remit the balance before the deadline. It is true that Art. agreement.
1482 of the Civil Code provides that whenever earnest money is given in a 10. Chua then confirmed his stop payment order by submitting to PBCom and
contract of sale, it shall be considered as part of the price and proof of the affidavit of loss. PBCom’s Assistant VP however, testified that the managers
perfection of the contract. However, this article speaks of earnest money given in check was still honored because Chua subsequently verbally advised the bank
that he was lifting the stop payment order due to his special arrangement with rather than a contract to sell. He contends that there was no reservation in the
the bank. contract of sale that Valdes-Choy shall retain title to the property until after
11. The deadline for the payment of the balance of the purchase price was nearing the sale.
so Valdes-Choy suggested to her counsel that to break the impasse, Chua 6. Chua’s theory is not well taken. An issue not raised in the court below cannot
should deposit in escrow the P10,215,000 balance. Upon such deposit, be raised for the first time on appeal because this is offensive to the basic
Valdes-Choy was willing to cause the issuance of a new TCT in Chua’s name rules of fair play.
even without receiving the balance. 7. However, to put doubts to rest, the Court holds that the agreement between
12. Valdes-Choy believed that this was the only way she could protect herself if Chua and Valdes-Choy, as evidenced by the Receipt, is a contract to sell
the certificate of title is transferred in the name of the buyer before she is fully and not a contract of sale.
paid. Valdes-Choy’s counsel promised to relay her suggestion to Chua but 8. In a contract of sale, the title of the property passes to the vendee upon
nothing came out of it. the delivery of the thing sold. In a contract to sell, ownership is, by
13. Chua filed a complaint for specific performance against Valdes-Choy which agreement, reserved in the vendor and is not to pass to the vendee until
the trial court dismissed. Chua re-filed his complaint and the trial court full payment of the purchase price.
rendered judgment in favor of Chua, ordering Valdes-Choy to deliver to the 9. A perusal of the Receipt shows that the true agreement between the
Court the owners duplicate of TCT No. 162955 under her name and the two parties was a contract to sell. Ownership over the property was retained by
deeds of sale, among others. Valdes-Choy and was not to pass to Chua until full payment of the purchase
14. Valdes-Choy appealed to the CA which reversed the decision of the trial price.
court. The CA ruled that Chua’s stance to pay the full consideration only after 10. The agreement between Chua and Valdes-Choy was embodied in a receipt
the property is registered in his name was not the agreement of the parties. rather than in a deed of sale, ownership not having passed between them. The
The CA noted that there is a whale of difference between the phrases all signing of the Deeds of Sale came later when Valdes-Choy was under the
papers are in proper order as written on the Receipt, and transfer of title as impression that Chua was about to pay the balance. Absence of a formal
demanded by Chua. deed of conveyance is a strong indication that the parties did not intend
immediate transfer of ownership, but only a transfer after full payment
ISSUE/s: of the purchase price.
1. WoN there is a perfected contract of sale of immovable property – NO; only 11. Valdes-Choy also retained possession of the certificate of title and all other
a contract to sell documents relative to the sale. When Chua refused to pay Valdes-Choy the
2. WoN the earnest money in the amount of P100,000.00 can be forfeited – YES purchase price, Valdes-Choy also refused to turn over to Chua the documents.
These are additional proof that the agreement did not transfer to Chua, either
RULING: WHEREFORE, the Decision of the Court of Appeals is AFFIRMED in by actual or constructive delivery, ownership of the property.
toto. 12. It is true that Art. 1482 of the Civil Code provides that whenever earnest
money is given in a contract of sale, it shall be considered as part of the
RATIO: price and proof of the perfection of the contract. However, this article
Contract to Sell speaks of earnest money given in a contract of sale.
1. There is no dispute that Valdes-Choy is the absolute owner of the property 13. The Receipt evidencing the contract to sell stipulates that the earnest money
registered under her name, free from all liens and encumbrances. She was is a forfeitable deposit, to be forfeited if the sale is not consummated should
ready, able, and willing to deliver to Chua the owners duplicate copy of the Chua fail to pay the balance of the purchase price.
TCT, the signed Deeds of Sale, tax declarations and realty tax receipt. 14. If there is a contract of sale, Valdes-Choy should have the right to compel
2. There is also no dispute that Valdes-Choy received PBCom check for Chua to pay the balance. However, Chua has the right to walk away from the
P100,000 as earnest money from Chua. transaction with no obligation to pay the balance, although he will forfeit the
3. As embodied in the Receipt, Valdes-Choy and Chua agreed to: earnest money. Clearly, there is no contract of sale.
a. The balance of P10,215,000 is payable on or before July 15 1989; 15. Since the agreement between Valdes-Choy and Chua is a mere contract
b. The capital gains tax is for the account of Valdes-Choy and to sell, the full payment of the purchase price partakes of a suspensive
c. If Chua fails to pay the balance of P10,215,000, Valdes-Choy has the right condition. The non-fulfillment of the condition prevents the obligation to sell
to forfeit the earnest money, provided that all papers are in proper order. from arising and ownership is retained by the seller without further remedies
4. Chua has consistently characterized his agreement with Valdes-Choy as by the buyer.
evidenced by the Receipt, as a contract to sell and not a contract of sale. Earnest Money
5. Chua now pleads for the first time that there is a perfected contract of sale
16. Chua insists that he was ready to pay the balance but withheld payment may become the owner of the real property even if the certificate of title is
because Valdes-Choy did not fulfill her contractual obligation to put all the still registered in the name of the seller.
papers in proper order. Chua then claims that Valdes-Choy cannot forfeit the 28. As between the seller and buyer, ownership is transferred not by the
earnest money even if he did not pay on time. issuance of a new certificate of title but by the execution of the instrument
17. There is no dispute that as long as the papers are in proper order, Valdes- of sale in a public document.
Choy has the right to forfeit the earnest money if Chua fails to pay the 29. Delivery is not only a necessary condition for the enjoyment of the thing, but
balance. is a mode of acquiring dominion and determines the transmission of
18. The Receipt provides that the earnest money shall be forfeited in case ownership, the birth of the real right.
the buyer fails to pay the balance. In such event, Valdes-Choy can sell 30. In a contract of sale of real property, delivery is effected when the instrument
the property to other interested parties. There is in effect a right reserved of sale is executed in a public document. When the deed of absolute sale is
in favor of Valdes-Choy not to push through with the sale upon Chua’s failure signed by the parties and notarized, then delivery of the real property is
to remit the balance before the deadline. deemed made by the seller to the buyer.
19. It is true that Art. 1482 provides that “Whenever earnest money is given in a 31. Similarly, in a contract to sell real property, once the seller is ready, able and
contract of sale, it shall be considered as part of the price and proof of the willing to sign the deed of absolute sale, the seller is in a position to transfer
perfection of the contract.” However, this article speaks of earnest money ownership to the buyer.
given in a contract of sale. In this case, the earnest money was given in a 32. In the instant case, Valdes-Choy was in a position to comply with all her
contract to sell. obligations as a seller under the contract to sell. First, she already signed the
20. The Receipt evidencing the contract to sell stipulates that the earnest money Deeds of Sale in the presence of the buyer.
is a forfeitable deposit, to be forfeited if the sale is not consummated should 33. She also prepared to turn over the owners duplicate of the TCT to Chua, along
Chua fail to pay the balance of the purchase price. with the tax declarations and latest realty tax receipt.
21. The earnest money forms part of the consideration only if the sale is 34. Chua, however, refused to give to Valdes-Choy the PBCom managers check
consummated upon full payment of the purchase price. Clearly, there is for the balance of the purchase price. Chua imposed the condition that a
no contract of sale in this case. The earnest money was given in a contract new TCT should first be issued in his name, a condition that is found
to sell, so Art. 1482 is not applicable. neither in the law nor in the contract to sell.
22. The trial court interpreted the phrase (papers in proper order) to include 35. Thus, at this point, Chua was not ready, able and willing to pay the full
payment of the capital gains tax, with the BIR receipt as proof of payment. purchase price which is his obligation under the contract to sell.
The CA held otherwise, and said that the Revenue Code provides that the 36. The contract to sell stipulated that Chua should pay the balance of the
amount realized from the sale shall be the sum of money received plus the purchase price on or before 15 July 1989. The signed Deeds of Sale also
fair market value of the property received. stipulated that the buyer shall pay the balance of the purchase price upon
23. The CA is correct. In a contract to sell, the obligation of the seller to sell signing of the deeds.
becomes demandable only upon the happening of the suspensive 37. However, on the agreed date, Chua refused to pay the balance of the purchase
condition. In this case, the suspensive condition is the full payment of the price. Chua was therefore in default and has only himself to blame for the
purchase price by Chua. rescission by Valdes-Choy.
24. It is only upon the existence of the contract of sale that the seller becomes 38. Valdes-Choy had all her papers in proper order. In the absence of a contrary
obligated to transfer the ownership of the thing sold to the buyer. agreement, the submission by an individual seller of the owners duplicate
25. Prior to the existence of the contract of sale, the seller is not obligated to copy, signed deed of absolute sale, tax declaration, latest realty tax, would
transfer ownership to the buyer, even if there is a contract to sell between complete a sale of real estate.
them. It is also upon the existence of the contract of sale that the buyer is 39. In this case, Valdes-Choy was ready, able and willing to submit to Chua all
obligated to pay the purchase price to the seller. the papers that customarily would complete the sale, and to pay the capital
26. In a contract of sale, the obligations of the seller are to transfer the gains tax.
ownership of and deliver, as well as warrant the thing which is the object of 40. But Chua’s condition that a new TCT first be issued in his name is not
the sale. customary in a sale of real estate. Such a condition not specified in the
27. The obligation of the seller is to transfer to the buyer the ownership of contract to sell cannot be considered part of the omissions of stipulations
the thing sold. In the sale of real property, the seller is not obligated to which are established by usage or custom.
transfer ownership in the name of the buyer a new certificate of title, but 41. What is increasingly becoming customary is to deposit in escrow the balance
rather to transfer ownership of the real property. This means that the buyer of the purchase price pending the issuance of a new certificate of title. Valdes-
Choy suggested this to Chua but it drew no response.
42. Chua acquired no right to compel Valdes-Choy to transfer ownership to
him because the suspensive condition did not happen.
09 and 11 Behn Meyer v. Yangco (Jolina) Place of Delivery [RELEVANT TOPIC]
Sept. 18, 1918 | Malcolm, J. | Free on Board (FOB); Costs, Insurance, Freight (CIF) 125. The contract provides for “c.i.f. Manila, pagadero against delivery of
documents”.
PETITIONER: Behn, Meyer & Co. (Ltd.)
RESPONDENTS: Teodoro Yangco Time of Delivery
126. The contract provided “Embarque: March 1916”. The merchandise was
SUMMARY: The contract between Behn Meyer and Yangco state that the former shipped from New York on Chinese Prince on April 12, 1916.
ships from New York to the latter in Manila “80 drums of caustic soda of 76%
‘Carabao’ brand” with the goods received from Neuss Hesslein & Co. However, Contract
71 of the shipment of the drums were removed after the ship was detained in 127. The contract between the parties was for 80 drums of caustic soda, 76%
Malaysia by British authorities. Yangco wants to rescind the contract for a ‘Carabao’ brand at the price of $9.75 per 100lbs with cost, insurance and
substantial breach in the failure of delivering the goods while Behn Meyer claims freight included, to be shipped during March 1916, to be delivered at Manila
damages for a supposed fulfillment of the obligation since Yangco already owned and paid for on delivery of the documents.
the goods upon shipment. The Court ruled in favor of Yangco, stating that Manila
being the destination of the delivery, Behn Meyer still had a duty over the goods ISSUE/s:
till it reached its destination as evidenced by the “c.i.f. Manila” found in the 23. WoN Behn Meyer is liable for its duty to deliver the goods as seen in the
contract. contract. –YES, for the goods do not transfer to Yangco till it reaches Manila.

DOCTRINE: CIF found in British contracts stand for costs, insurance, freight, RULING: Trial court judgment is affirmed, ordering the plaintiff take nothing by its
signifying that the price fixed covers not only the cost of goods but also the action without special findings as to costs. (Basically, no damages for Behn Meyer)
expense of freight and insurance to be paid by the seller.
RATIO:
FOB stands for “free on board” from American origin, which means that the seller Subject Matter and Consideration
shall bear all expenses until the goods are delivered where they are to be “FOB”. 81. The soda which Behn Meyer offered was never tendered for it was not the
According as to whether the goods are delivered “FOB” at the point of shipment ‘Carabao’ brand agreed upon and the offer of drums of the other kind of soda
or at the point of destination determines the time when property passes. was not made within the time that a March shipment would normally have
been available as provided by a provision of the contract (seen in fact #6).
Both “CIF” and “FOB” merely make rules of presumption which yield to proof of
contrary intention. Place of Delivery [RELEVANT TOPIC]
82. Determination of place of delivery is always resolved into a question of fact.
If the contract is silent as to the person or mode by which the goods are sent,
FACTS: [case discusses per part of the contract not as a story, so copying that lang] delivery by the vendor to a common carrier in usual and ordinary course of
Subject Matter and Consideration business transfers the property to the vendee.
121. The contract provides for “80 drums Caustic Soda 76% ‘Carabao’ brand al 83. A specification in the contract relative to the payment of freight can indicate
precio de Dollar Gold Nine and 75/100 per 100-lbs” (fact #7 provides a the intention of the parties in regard to the place of delivery. If the buyer pays
clearer explanation of the whole contract). The merchandis was shipped from freight, it is reasonable to suppose he does so because the goods become his
New York on the steamship Chinese Prince. at the point of shipment. If the seller is to pay the freight, the intention is
122. The steamship was detained by the British authorities at Penang, Malaysia, said to be that the seller’s duty is to have the good transported to their
with part of the cargo removed – 71 drums of caustic soda. ultimate destination and that the title to property does not pass until the
123. Yangco refuses to accept delivery of the 9 remaining drums of soda for the goods have reached their destination.
goods were in bad order. He also refused the optional offer of Behn Meyer of 84. CIF found in British contracts stand for costs, insurance, freight, signifying
waiting for the remainder of the shipment until its arrival or accepting a that the price fixed covers not only the cost of goods but also the expense
substitute of the 71 drums of caustic soda of similar grade from Behn Meyer’s of freight and insurance to be paid by the seller.
stock. 85. FOB stands for “free on board” from American origin, which means that the
124. Behn Meyer claims damages computed at 80 drums of caustic soda of seller shall bear all expenses until the goods are delivered where they are
P6,352.89 deducted from the sum of the selling price of P10,063.86. to be “FOB”. According as to whether the goods are delivered “FOB” at the
point of shipment or at the point of destination determines the time when
property passes.

Both “CIF” and “FOB” merely make rules of presumption which yield to
proof of contrary intention.
86. The Court believes that “Manila” in conjunction with “c.i.f” in the
contract must mean that the contract price covering costs, insurance,
and freight signifies that delivery was to be made at Manila. If Behn
Meyer thought that the place of delivery was NY and not Manila, then it
would not have attempted to substitute the goods and just permitted the loss
of the shipment to fall on Yangco.
87. The goods were not shipped nor consigned from NY to Yangco. The bill of
lading was for goods received from Neuss Hesslein & Co. The documents
evidence the shipment sent by Neuss Hesslein to the Bank of the Philippine
Islands with a draft upon Behn Meyer and with instructions to deliver the
same, thus transfering the property to Behn Meyer when and if Behn Meyer
pays the draft.

The place of delivery was Manila and Behn Meyer has not legally
excused default in delivery of the specified merchandise at that place.

Performance
88. Behn Meyer has not proved performance on its part of the condition
precedent in the contract. The warranty of the seller to the buyer has not been
complied with and therefore the buyer may rescind the contract of sale
because of substantial breach.
89. Article 1451 of the CC provides that the vendee can demand the fulfillment
of the contract, and if it be shown to be impossible, is relieved of his
obligation. There being sufficient ground for rescission, Yangco is not liable.
10 GENERAL FOODS v. NACOCO (Megan) port of discharge, this market price to be fixed by the Executive Committee
Nov. 20, 1956 | Reyes, J. | CIF Sales of the National Institute of Oilseeds Products.

PETITIONER: General Foods Corp. xxxx


RESPONDENTS: National Coconut Corporation
PRICE: One hundred and sixty-four dollars ($164) per ton of 2,000
SUMMARY: NACOCO sold to General Foods 1500 tons of copra for $164. The pounds, CIF New York.”
sale was subject to the terms and conditions embodied in its contract (see Facts 4 for
the important parts of the contract). NACOCO shipped 1054.6278 tons of copra. 5. NACOCO shipped 1054.6278 short tons of copra to General Foods on board
The cargo was weighed by Luzon Brokerage and determined the net weight. On the the S. S. “Mindoro”
basis of the net weight, NACOCO prepared the corresponding bills of lading and 6. The cargo was weighed by Luzon Brokerage and determinded the net weight
other documents and withdrew from General Food’s letter of credit a total of $ of the cargo.
136,686.95. Upon arrival in New York, it only weighed 898.792 short tons. General 7. On the basis of the computed net weight, NACOCO prepared and remitted
Foods demanded for refund a refund which NACOCO acknowledged and promised to General Foods the corresponding bills of lading and other documents and
to pay. NACOCO was subsequently abolished and liquidated. General Foods again withdrew from General Food’s letter of credit 95% of the invoice value of
demanded for the refund but they refused payment. The trial court ruled in favor of the shipment or a total of $ 136,686.95. (The price was based on the net
NACOCO however the Supreme Court reversed the ruling and ordered NACOCO to weight)
8. Upon arrival in New York, the net cargo was reweighed and was found to
pay the refund (di ko din alam bakit CFI to SC agad). It found that the express
weigh only 898.792 short tons. (there were tons that was either lost or
agreement of the parties is that the “Net Landed Weights” were to govern. destroyed).
9. General Foods demanded NACOCO for a refund in the amount of $24,
DOCTRINE: C.I.F. found in British contracts stand for costs, insurance, and 154.59.
freight; they signify that the price fixed covers not only the costs of the foods, 10. NACOCO, through its officers-in-charge, acknowledged its liability for the
but the expense of freight and insurance to be paid by the seller. deficiency in the outturn weights through a letter and promised payment as
soon as funds were available.
FACTS: 11. NACOCO was subsequently abolished by EO 3727 and went into liquidation.
1. Appellant General Foods Corp. (General Foods) is a foreign corp. organized 12. General Foods submitted its claim to the Board of Liquidators but they
under the laws of the State of Delaware, USA and licensed to do business in refused to pay.
the Philippines. 13. General Foods therefore files an action in the CFI to recover from NACOCO
2. Appelle National Coconut Corp. (NACOCO) was a corporation create by the amount.
Commonwealth Act No. 518. 14. The CFI ruled in favor of NACOCO and dismissed the complaint.
3. NACOCO sold to General Foods 1500 (later reduced to 1000) tons of copra 15. General Foods appealed to the SC.
at $164 (reduced to 163) per ton of 2000 pounds. 16. General Foods contends that although the sale was quoted a CIF New York
4. The sale was subject to the terms and conditions embodied in its contract. price, the agreement contemplated the payment of the price according to the
(NOTE: take note of the highlighted parts of the contracts as this will be used weight and quality of the cargo upon arrival in New York, therefore the risk
in their arguments) is on the seller.
17. NACOCO on the other hand insists that the contract in question is an ordinary
“CONTRACT NO. RH-3551 CIF agreement wherein delivery to the carrier is delivery to the buyer, and
that the shipment having been delivered to the buyer and the latter having
xxxx paid its price, the sale was consummated.

QUANTITY: Fifteen Hundred (1500) tons of 2,240 pounds each. Seller has ISSUE/s:
the option of delivering 5 per cent more or less of the contracted quantity, 1. WoN the price of the copra should be based on the weight and quality of the
such surplus or deficiency to be settled as follows: On the basis of the cargo upon its arrival in New York – YES
delivered weight up to 3 percent at the contract prince and any excess or
deficiency beyond this 3 percent at market price of the day of arrival at RULING: Wherefore, the judgment appealed from is reversed and the Appellee
National Coconut Corporation is ordered to pay the Appellant General Foods 9. NACOCO contends that as it was only the “balance due to be paid”
Corporation the equivalent in Philippine currency of the amount of $24,154.59, that was to be ascertained and based “upon outturn weights and quality
at port of discharge”, as provided in the contract, there was no more
balance due to be ascertained at the port of discharge because it had
RATIO:
already received full payment of the copra it sent to the General Foods
1. C.I.F. found in British contracts stand for costs, insurance, and
when it withdrew $136,686.95 from the latter’s letter of credit.
freight; they signify that the price fixed covers not only the costs of
10. The argument is untenable. The provision regarding the ascertainment
the foods, but the expense of freight and insurance to be paid by the
of the balance due based upon outturn weight and quality of the
seller. (This is not found in the case but the doctrine in the outline)
shipment at the port of discharge, should not be construed separately
2. under an ordinary C.I.F. agreement, delivery to the buyer is complete
from the stipulation that the “net landed weight” was to control. The
upon delivery of the goods to the carrier and tender of the shipping
manifest intention of the parties was for the total price to be finally
and other documents required by the contract and the insurance policy
ascertained only upon determining the net weight and quality of the
taken in the buyer’s behalf.
goods upon arrival in New York
3. In the transaction now in question, despite the quoted price of CIF
New York, and the right of the seller to withdraw 95 per cent of the
invoice price from the buyer’s letter of credit upon tender of the
shipping and other documents required by the contract, the express
agreement that the “Net Landed Weights” were to govern.
4. The provision that the balance of the price was to be ascertained on
the basis of outturn weights and quality of the cargo at the port of
discharge, indicate an intention that the precise amount to be paid by
the buyer depended upon the ascertainment of the exact net weight of
the cargo at the port of destination.
5. That is furthermore shown by the provision that the seller could
deliver 5 per cent more or less than the contracted quantity, such
surplus or deficiency to be paid “on the basis of the delivered weight”.
6. While the risk of loss was apparently placed on the General Foods
after delivery of the cargo to the carrier, it was nevertheless agreed
that the payment of the price was to be according to the “net landed
weight”
7. Although the evidence shows that the estimated weight of the
shipment when it left Manila was 1,054.6278 tons, the NACOCO had
the burden of proof to show that the shortage in weight upon
arrival in New York was due to risks of the voyage and not the
natural drying up of the copra while in transit, or to reasonable
allowances for errors in the weighing of the gross cargo and the
empty bags in Manila.
8. In the absence of such proof on the part of the shipper, we are
constrained to hold that the net landed weight of the shipment in New
York should control, as stipulated in the agreement, and that therefore,
the NACOCO should be held liable for the amount of $24,154.59
which it had overdrawn from General Food’s letter of credit.
12 PACIFIC VEGETABLE OIL v. ANGEL SINGZON (Armand) 11. Singzon in August 1947, acting through a broker in San Francisco, sold to
1947 |, J. | “c.i.f” Pacific 500 tons of copra for shipment in September and October 1947. The
agreed price to be covered by an irrevocable letter of credit for the contract
PETITIONER: Pacific Vegetable Oil price. Thus, pursuant to this, the Bank of California, on behalf of Pacific,
RESPONDENTS: Angel Singzon opened an irrevocable letter of credit with China Bank in the Philippines.
SUMMARY: Singzon acting through a broker in San Francisco, sold to Pacific 12. Singzon failed to ship the 500 tons of copra, but upon negotiation through the
500 tons of copra. Thus Bank of California, on behalf of Pacific opened an broker, a conditional amicable settlement was arrived at under which Singzon
irrevocable letter of credit with China Bank in the Philippines. Singzon failed to promised to ship on February 1948, the amount of 300 tons of copra with the
ship the 500 tons of copra, but upon negotiation through the broker, a conditional understanding that if he ships the said 300 tons of copra not later than
amicable settlement was arrived at under which Singzon promised to ship on February, the original contract would be considered cancelled. But that
February 1948, the amount of 300 tons of copra with the understanding that if he should he fail to ship said 300 tons, Singzon shall pay Pacific $10,000 as
ships the said 300 tons of copra not later than February, the original contract damages and shall furthermore be obliged to fulfill all his obligations under
would be considered cancelled. But that should he fail to ship said 300 tons, original contract.
Singzon shall pay Pacific $10,000 as damages and shall furthermore be obliged 13. Singzon failed to ship and deliver the 300 tons of copra to Pacific according
to fulfill all his obligations under original contract. Singzon still failed to ship the to their agreement. Thereafter, Pacific demanded from Singzon the payment
300 tons of copra. Pacific then demanded from Singzon the payment of $10,000 of $10,000 but he failed and refused to ship the 500 tons of copra. As a result
to no avail. Pacific was then forced to purchase from another company and thus of the default, Pacific was forced to purchase copra from the world marker
incurred additional expenses. Hence, the present action by Pacific for the and thus incurred additional expenses. Hence, this action is filed by Pacific.
recovery of the sum of P157,760 as damages. Singzon filed a motion to dismiss 14. Singzon, in defense, filed a motion to dismiss on the ground that Pacific
on the ground that Pacific failed to obtain a license to transact business in the Vegetable Oil Corp. (Pacific) failed to obtain license to transact business in
Philippines hence it had no personality to file the action. RTC denied, and so was the Philippines and consequently, it had no personality to file the action. RTC
the MR. CA reversed and dismissed the case holding that Pacific had no denied the motion. It also denied MR.
personality to institute the case even if it afterwards obtained a license on the 15. However, the Court of Appeals reversed and dismissed the case holding that
theory that a belated act of procuring a license would not cure the defect. WoN Pacific had no personality to institute the present case even if it afterwards
appellant transacted business in the Philippines in contemplation of law - NO, it obtained a license to transact business upon the theory that this belated act
was transacted in the US. However a foreign corporation who transacted business did not have the effect of curing the defect.
abroad may still seek redress in Philippine courts. (see ratio #5) WoN Singzon
(vendor) is liable to pay not only the cost of the goods, but also the freight and ISSUES:
insurance expenses (due to the “c.i.f” arrangement) – YES (see doctrine below) 2. WoN appellant transacted business in the Philippines in contemplation of law
- NO, it was transacted in the US. However a foreign corporation who
DOCTRINE: Under an arrangement “c.i.f. U.S. Pacific Coast”, the vendor is to transacted business abroad may still seek redress in Philippine courts. (see
pay not only the cost of the goods, but also the freight and insurance expenses, ratio #5)
and, as it was judicially interpreted, this is taken to indicate that the delivery is to 3. WoN Singzon (vendor) is liable to pay not only the cost of the goods, but also
be made at the port of destination. the freight and insurance expenses (due to the “c.i.f” arrangement) – YES
(see doctrine)
*”c.i.f.” found in British contracts stands for costs, insurance, and freight; they
signify that the price fixed covers not only the costs of the goods, but the expense RULING: Decision of CA is reversed.
of freight and insurance to be paid by the seller. RATIO:
*NOTE: THIS CASE IS UNREPORTED SO I CANT FIND THE CASE 12. It appears from the facts that the copra in question was actually sold by the
ONLINE SO THIS IS JUST A SUMMARY OF THE DIGEST I FOUND defendant to the plaintiff in the US. It also appears that the contract was
COMBINED WITH THE INFO FROM THE SYLLABUS entered into in the US by appellee’s broker and appellant’s representatives.
FACTS: 13. It further appears that the payment of the price was to be made at San
10. This is an action instituted by the plaintiff, a foreign corporation, against the Francisco, California, through a letter of credit to be opened at the Bank of
defendant to recover the sum of P157,760 as damages suffered by plaintiff as California. And with respect to the delivery of copra, it likewise appears that
a consequence of the failure of the defendant to deliver 300 tons of copra the price agreed upon was $142 per 2,000 lbs., c.i.f. Pacific Coast.
which he sold and bound himself to deliver to the plaintiff. 14. This means that the vendor was to pay not only the cost of the goods, but
also the freight and insurance expenses, and, it was judicially
interpreted, this is taken to indicate that the delivery is to be made at the
port of destination.
15. It is therefore clear that the contract covering the copra has not only
entered into in the US but it was agreed to be consummated there. It
follows that Pacific has not transacted business in the Philippines in
contemplation of Sections 68 and 69 of the Corporation Law which require
any foreign corporation to obtain a license before it could transact business,
or before it could have personality to file suit in the Philippines.
16. It appearing that Pacific has not transacted business in the Philippines and as
such it is not required to obtain a license before acquiring personality to bring
court action, it may be stated that the appellant, even if a foreign corporation,
can maintain the present action because, as aptly said by this Court, “it was
never the purpose of the Legislature to exclude a foreign corporation which
happens to obtain an isolated order for business in the Philippines, from
securing redress in the Philippine courts, and thus, in effect, to permit persons
to avoid their contracts made with such foreign corporation.” Wherefore, the
decision appealed from is reversed. Pacific is entitled to prosecute its claim
in the Philippine courts against Singzon.
13 LIETZ v. CA (IYA) the property,
December 19, 2005 | TInga, J. | Topic b. the annulment of the lease agreement between respondents, and
c. the restoration of the amount paid by petitioner in excess of the value
of the property sold to him.
PETITIONER: Rudolf Lietz, Inc.
9. Turatello and Sani filed answers raising the defense of lack of case of action
RESPONDENTS: Court of Appeals, Agapito Buriol, Tiziana Turatello, and
and claimed for damages.
Paola Sani
10. Trial court dismissed Lietz’ complaint and Turatello and Sani’s claim for
damages.
SUMMARY: Buriol sold his parcel of land to Lietz. In the Deed of Absolute
11. The CA affirmed the dismissal of Lietz’ complaint and awarded damages in
Sale, the parcel was described as being 5 hectares. Lietz then later discovered that
favor of Tratello and Sani.
only 3 hectares was delivered to it for Burial only actually owed 4 hectares and 1
12. Hence, the instant petition before the Supreme Court brought by Lietz
hectare was leased to Turatello and Sani.
ISSUE/s:
Lietz then filed a case for the reduction of the purchase price. The RTC and CA
1. Whether or not petitioner Lietz is entitled to the delivery of the entire five
dismissed his petition. The Supreme Court upheld the CA ruling and stated that
hectares or its equivalent
the sale was that of a lump sum, therefore the price in the contract is controlling
and no reduction may be awarded to Lietz.
RULING: WHEREFORE, the instant petition for review on certiorari is GRANTED
in PART. The Court of Appeals Decision in CA-G.R. CV No. 38854 is AFFIRMED
DOCTRINE: According to Article 1542 of the Civil Code, in the sale of real
with the MODIFICATION that the award of moral and exemplary damages is
estate, made for a lump sum and not at the rate of a certain sum for a unit of
DELETED.
measure or number, there shall be no increase or decrease of the price although
there be a greater or lesser area or number than that stated in the contract.
RATIO:
1. Lietz contends that it is entitled to the corresponding reduction of the
FACTS: purchase price because the agreement was for the sale of 5 hectares although
1. Agapito Buriol previously owned a parcel of unregistered land situated at Burial only owned 4 hectares.
Capsalay Island, Port Barton, San Vicente, Palawan. 2. Lietz relies on Article 1539 of the Civil Code which states:
2. On August 15, 1986, respondent Buriol entered into a lease agreement with Art. 1539. The obligation to deliver the thing sold includes that of placing
Flavia Turatello, Tiziana Turatello and Paola Sani, all Italian citizens, in the control of the vendee all that is mentioned in the contract, in
involving one (1) hectare of respondent Buriol’s property. conformity with the following rules:
3. The lease agreement was for a period of 25 years, renewable for another 25
years. The lessees took possession of the land after paying respondent Buriol If the sale of real estate should be made with a statement of its area, at
a down payment of ₱10,000.00. the rate of a certain price for a unit of measure or number, the vendor
4. The lease agreement, however, was reduced into writing only in January shall be obliged to deliver to the vendee, if the latter should demand it,
1987. all that may have been stated in the contract; but, should this be not
5. However, in the year 1986, Buriol sold to Rudolf Lietz, Inc. the same parcel possible, the vendee may choose between a proportional reduction of the
of land for the amount of P30,000 as evidenced by a Deed of Absolute Sale. price and the rescission of the contract, provided that, in the latter case,
6. The Deed mentioned a sale of 5 hectares of land. the lack in the area be not less than one-tenth of that stated.
7. Lietz later discovered that Buriol owned only 4 hectares and that 1 hectare of 3. The Court of Appeals declared as inapplicable the above provision and ruled
it was covered by a lease. Therefore only 3 hectares was only delivered to it. that Lietz is no longer entitled to a reduction of price based on Art 1542 of
8. Lietz then instituted a complaint for Annulment of Lease with Recovery of the Civil Code which states:
Possession with Injunction and Damages against respondents Buriol, et. al. Art. 1542. In the sale of real estate, made for a lump sum and not at the
a. The complaint sought the issuance of a restraining order and a writ rate of a certain sum for a unit of measure or number, there shall be no
of preliminary injunction to prevent Flavia Turatello and increase or decrease of the price, although there be a greater or lesser
respondents Turatello and Sani from introducing improvements on area or number than that stated in the contract.
4. The same rule shall be applied when two or more immovables are sold for a
single price; but if, besides mentioning the boundaries its area or number
should be designated in the contract, the vendor shall be bound to deliver all
that is included within said boundaries.
5. Should he not be able to do so, he shall suffer a reduction in the price, in
proportion to what is lacking in the area or number, unless the contract is
rescinded because the vendee does not accede to the failure to deliver what
has been stipulated.
6. Article 1539 governs a sale of immovable by the unit, that is, at a stated rate
per unit area. In a unit price contract, the statement of area of immovable is
not conclusive and the price may be reduced or increased depending on the
area actually delivered.
7. If the vendor delivers less than the area agreed upon, the vendee may oblige
the vendor to deliver all that may be stated in the contract or demand for the
proportionate reduction of the purchase price if delivery is not possible.
8. In the case where the area of the immovable is stated in the contract based on
an estimate, the actual area delivered may not measure up exactly with the
area stated in the contract.
9. According to Article 1542 of the Civil Code, in the sale of real estate,
made for a lump sum and not at the rate of a certain sum for a unit of
measure or number, there shall be no increase or decrease of the price
although there be a greater or lesser area or number than that stated in
the contract.
10. As correctly noted by the trial court and the Court of Appeals, the sale
between petitioner and respondent Buriol involving the latter’s property
is one made for a lump sum.
11. The Deed of Absolute Sale shows that the parties agreed on the purchase
price on a predetermined area of five hectares within the specified
boundaries and not based on a particular rate per area.
12. In accordance with Article 1542, there shall be no reduction in the
purchase price even if the area delivered to petitioner is less than that
stated in the contract.
13. In the instant case, the area within the boundaries as stated in the contract
shall control over the area agreed upon in the contract.
14 Roble v. Arbasa(Lij) respondents bought from Fidela consisted only of 240 square meters, located
July 31, 2001 | Pardo, J. | More or less at the northern portion of he property. They said that the property was
formerly shoreland but through the efforts of Ireneo Roble (father of Fidela,
PETITIONER: Veronica Roble, Lilibeth R. Portugaliza and Bobby Portugaliza Adelaida, and Gualberto) in 1957, a portion of the sea was reclaimed and
RESPONDENTS: Dominador Arbasa and Adelaida Arbasa filled up.
30. Attached to their answer was the deed of sale describing the property and that
SUMMARY: Arbasa and Adelaida Roble bought from Fidela Roble a parcel of it was 240 square meters more or less.
land. The Deed of sale stated that the total area was 240 square meters. Through 31. There are contravening facts from the start of the case: so I guess the claim
their efforts, they reclaimed a portion of the sea so that the land increased 884 of the respondents was that they were the ones who reclaimed but the other
sqm. Adelaida toterated the stay of her sister Fidela and also their nieces Veronica side found that Ireneo Roble was the one who reclaimed it (father of Fidela,
and Lilibeth. After Fidela’s death, Veronica and lilibeth claimed portion of theland Adleaida and Gualberto (brother)) [note: seems like the sc sided with the
amounting to 664 sqm. There was a dispute on the amount conveyed, whether it respondents were the one who reclaimed; not entirely sure as to that]
was 240 or 884. The sc ruled in the sale of real estate, there shall be no increase 32. So Ireneo reclaimed the land..the 664 square meter was divided into two lots
or decrease of the price although there be a greater or lesser area or number than of equal proportion. Constructed over the 884 square meters were 3 concrete
stated in the contract. This basically means that a difference between actual area houses. One in the portion that was bought and the the other two in the other
and what is stated in the contract is allowed. When the land is sold with the two lots.
description “more or less” or similar description it will pertain only to a reasonable 33. So at the pre trial they agreed to the issue of whether the deed of sale execute
excess or deficiency. The excess of 644 is not reasonable. on Jan 2, 1975 by Fidela Roble in favor of respondents conveyed either the
884 square meters including the house of Fidela or it covered only the 240
DOCTRINE: In a sale of real estate, the description of more or less would allow meters located in te norther portion of the property
a reasonable excess or deficiency in the actual area to the area in the ontract 34. Trial Court: Sale was only for the 240 portion of theland and not thhe entire
884 sq meters. Moreover they added that the house of Fidela was not included
in the sale. They ruled that the deed of sale was the best evidence of the
FACTS: contents of the agreement.
24. Jan. 2, 1976, Sposes Dominador Arbasa and Adelaida Roble (Respondent 35. Respondents appealed to the CA, petitioners appealed insofar as their claims
Spouses) purchased from Fidela Roble an unregistered parcel of land located for damages. CA affirmed the finding of the trial court that the Deed of sale
at Poblacion, Isabel, Leyte. The deed of sale stated that the property had a only conveyed 240 square meters. But they set aside the judgement and
total land area of 240 square meters. Through diligent efforts in reclaiming a declared the respondents the lawful owners of the 884 sqm.
portion of the sea, using stones, sand and gravel, the original size went up to 36. The CA observed from the wording of the deed of sale Fidela sold “whole
884 square meters. parcel or residential land” bounded on the “south by the seashore”. This lent
25. 1976 to present, respondent spouses have been in actual open, peaceful, and credence to the claim of petitioners that they reclaimed the 644 square meters.
continuous possession of the entire parcel of land in the concept of owners. The court said that that if the 644 sqm was already present the deed would be
Included in the sale were the improvements found on the land, consisting phrased differently.
mainly of the house of Fidela. 37. MR was denied. Hence this appeal.
26. Adelaiba tolerated her sister Fidela’s continued stay at the house. Living with
Fidela in the same house were their nieces, Veronica and LIlibeth Roble ISSUE
(dauthers of the deceased brother of Fidela and Adelaida). 1. Whether or not the land conveyed was only
27. Shortly after Fidela died in 1989, petitioner Veronica and Lilibeth Roble
claimed ownership of the house on the southern portion of the land with an RULING: Petition is granted; CA decision is set aside
area of 644 square meters.
28. Respondent spouses tried to have them vacate the house and desist from RATIO:
claiming the parcel of land but this failed. Hence Arbasa filed with the RTC 90. The sale on Jan 2, 1976 was one of cuerpo cierto or a sale for lump sum.
an action for quieting of title with damages. Pursuant to Art. 1542, Civil Code, in the sale of real estate made for a lump
29. Petitioners denied the allegations and said that the the total area which sum and not at the rate of a certain sum for a unit of measure or number, there
shall be no increase or decrease of the price although there be a greter or
lesser are or number than that stated in the contract.
91. This rule admits exception. A vendee of land, when sold in gross or with the
description “more or less” with reference to its area, does not thereby ipso
facto take all risks of quantity in the land. The use of “more or less” or similar
words designating quantity covers only a reasonable excess or deficiency.
92. An area of 644 sq m is not reasonable excess or deficiency to be deemed
included in the subject deed of sale.
93. Moreover, at the time of the sale, the only existing piece was 240 sqm.
Adelaida admitted that the houses of Fidela and Gualberto constructed in
1971 were situated on foreshore lands adjacent to the property. This lends
credence to the claim that only 240 meters were sold.
94. As held by the trial court, when the terms of an agreement ad been reduced
to writing, it is considered as containing all the terms agreed upon. The SC
found no ambiguity in the terms and stipulations of the deed of sale. Contracts
are the laws between the contracting parties. It shall be fulfilled in the literal
sense of their stipulations
95. Sale by its nature is a consensual contract because it is perfected by mere
consent. All the elements of a sale are present: Consent, Subject matter; and
price certain in money or its equivalent.
96. Moreover, parol evidence rule forbids any addition to the terms of a written
instrument by testimony or other evidence. Parties cannot add evidence to
prove practices that would alter the written agreement. This rule is found on
the long experience that written evidence is so much more certain and
accurate than that which rests in fleeting memory only. It would be unsafe to
when parties expressed the terms in writing to admit weaker evidence.
97. Exception to the parol evidence rule: (1) Intrinsic ambiguity, mistake or
imperfection in the written agreement; (2) the failure of written agreement to
express the true intent; (3) validity of the written agreement; (4) the existence
of other terms agreed after the execution of thewritten agreement. None of
them apply to this case.
15 NAAWAN COMMUNITY v. CA (See) proceedings undertaken on the property and the subsequent issuance of a title over
January 13, 2003 | Corona, J. | Double Sales: Primacy of Torrens System the land had under the Torrens system had the legal effect of cleansing title on the
property of all liens and claims not annotated therein.
PETITIONER: Naawan Community Rural Bank Inc.
RESPONDENTS: Court of Appeals and Spouses Alfredo and Annabelle Lumo FACTS:
18. Guillermo Comayas offered to sell to Spouses Alfredo and Annabelle Lumo,
SUMMARY: Spouses Lumo bought property from Comayas. Before buying it, they a 340sqm property with a house located in Pinikitan, Camaman-an, CDO
went to the Office of the Register of Deeds of CDO to check where the property is City.
located and the Bureau of Land to check on the legal status of the vendor’s title. They 19. Wanting to buy the lot, Spouses Lumo went to the Office of the Register of
found out that the property was mortgaged for P8,000 to a certain Mrs. Galupo and Deeds of CDO to check where the property is located and the Bureau of Land
that the owner’s copy of the Certificate of Title to said property was in her to check on the legal status of the vendor’s title.
possession. So Spouses told Comayas to redeem the property and gave P10,000 for 20. They found out that the property was mortgaged for P8,000 to a certain Mrs.
that purpose. The property was released from the mortgage so a deed of sale was Galupo and that the owner’s copy of the Certificate of Title to said property
executed. After obtaining a TCT, the Spouses wanted to have the property tax was in her possession. So Spouses told Comayas to redeem the property and
declared in their name but they found out that it was tax declared in the name of gave P10,000 for that purpose.
Naawan Community Rural bank. Apparently, before Comayas sold the property to 21. On May 30, 1988, a release of the adverse claim of Galupo was annotated on
the Spouses, he obtained a loan from Naawan using the property as security. At the TCT No. T41499 which covered the subject property.
time said contract of mortgage was entered into, the subject property was then an 22. In the meantime, on May 17, 1988, even before the release of Galupo’s
unregistered parcel of residential land, tax declared in the name of a certain Balibay adverse claim, Spouses Lumo and Guillermo Comayas, executed a deed of
while the residential one-storey house was tax declared in the name of Comayas. absolute sale. The subject property was allegedly sold for P125,000 but the
Balibay made an SPA authorizing Comayas to borrow money and use the lot as deed of sale reflected the amount of only P30,000 which was the amount
security but the SPA and the Deed of Real Estate Mortgage was registered not in private respondents were ready to pay at the time of the execution of said
CDO but in Misamis Oriental since at that time, there was only 1 register of deeds deed, the balance payable by installment.
for the entire province. Comayas failed to pay its debt to Naawan so the property was 23. The deed of absolute sale was then registered and a new TCT was issued in
foreclosed with Naawan being the highest bidder. Thereafter, the sheriff’s certificate favor of the Spouses.
of sale was issued and registered under Act 3344 in the Register of Deeds of the 24. After obtaining their TCT, the Spouses requested the issuance of a new tax
Province of Misamis Oriental. The land was then registered under the LRA. A TCT declaration certificate but they found out that the property was declared for
was then issued in the name of Comayas in Register of Deeds in CDO. Comayas and tax purposes in the name of Naawan Community Rural Bank. In the records
Balibay failed to redeem so it was delivered to Naawan and was registered under Act of the City Assessor, the TCT of the Spouses had a note, “This lot is also
3344 and so Naawan got a tax declaration under its name. The issue in this case is declared in the name of Naawan Community under T/D #71210.”
WoN Spouses Lumo should be considered innocent pruchasers for value. – YES. 25. Apparently, before Comayas sold the property to the Spouses, he obtained a
The SC held that the spouses were innocent purchasers for value and that Art 1544 loan from Naawan using the property as security. At the time said contract of
won’t apply in this case since the lands are unregistrered lands. (please see doctrine.) mortgage was entered into, the subject property was then an unregistered
parcel of residential land, tax declared in the name of a certain Sergio A.
DOCTRINE: Invoking the rules on double sales and “priority in time” under Art. Balibay while the residential one-storey house was tax declared in the name
1544 would be misplaced by a first buyer who bought the land not within the Torrens of Comayas.
system but under Act No. 3344, as against the second buyer who bought the same 26. Balibay made an SPA authorizing Comayas to borrow money and use the lot
property when it was already registered under the Torrens system, because: (i) of the as security but the SPA and the Deed of Real Estate Mortgage was registered
“well-known rule in this jurisdiction that persons dealing with registered land have not in CDO but in Misamis Oriental since at that time, there was only 1
the legal right to rely on the fact of the Torrens Certificate of Title and to dispense register of deeds for the entire province.
with the need to inquire further, except when the party concerned has actual 27. Comayas failed to pay its debt to Naawan so the property was foreclosed with
knowledge of facts and circumstances that would impel a reasonably cautious man Naawan being the highest bidder. Thereafter, the sheriff’s certificate of sale
to make such inquiry;” and (ii) the Torrens system rule that formal registration
was issued and registered under Act 3344 in the Register of Deeds of the Article 1473 (now Article 1544) of the Civil Code will apply only if said
Province of Misamis Oriental. execution sale of real estate is registered under Act 496.
28. On April 17, 1984, the subject property was registered in original proceedings 4. Unfortunately, the subject property was still untitled when it was acquired by
under the Land Registration Act. Title was entered in the registration book of Naawan by virtue of a final deed of conveyance. On the other hand, when
the Register of Deeds of Cagayan de Oro City as Original Certificate of Title Spouses Lumo purchased the same property, it was already covered by the
No. 0820, pursuant to Decree No. N189413. Torrens System.
29. On July 23, 1984, Transfer Certificate of Title No. T41499 in the name of 5. Naawan also relies on the case of Bautista vs. Fule where the Court ruled that
Guillermo P. Comayas was entered in the Register of Deeds of Cagayan de the registration of an instrument involving unregistered land in the Registry
Oro City. of Deeds creates constructive notice and binds third person who may
30. Meanwhile, on September 5, 1986, the period for redemption of the subsequently deal with the same property.
foreclosed subject property lapsed and the MTCC Deputy Sheriff of Cagayan 6. However, a close scrutiny of the records reveals that, at the time of the
de Oro City issued and delivered to Naawan the sheriffs deed of final execution and delivery of the sheriff’s deed of final conveyance on
conveyance. This time, the deed was registered under Act 3344 and recorded September 5, 1986, the disputed property was already covered by the
in the registration book of the Register of Deeds of Cagayan de Oro City. Land Registration Act and Original Certificate of Title No. 0820
31. By virtue of said deed, Naawan obtained a tax declaration for the subject pursuant to Decree No. N189413 was likewise already entered in the
house and lot. registration book of the Register of Deeds of Cagayan De Oro City as of
32. Then, Naawan filed a case to eject Comayas from the property. The MCTC April 17, 1984.
decided in favor of Naawan and the RTC affirmed this. The RTC issued a 7. Thus, from April 17, 1984, the subject property was already under the
writ of execution but when the writ was served, the property was no longer operation of the Torrens System. Under the said system, registration is the
occupied by Comayas since it was already the Spouses Lumo that stayed in operative act that given validity to the transfer or creates a lien upon the land.
the property. 8. Moreover, the issuance of a certificate of title had the effect of relieving the
33. Alarmed by the prospect that they might be ejected, the Spouses filed a case land of all claims except those noted thereon. Accordingly, Spouses Lumo,
for quieting of title. The RTC ruled in favor of the Spouses stating that they in dealing with the subject registered land, were not required by law to go
were purchasers in good faith and for value. beyond the register to determine the legal condition of the property. They
34. The CA affirmed the RTC ruling; hence, this petition. were only charged with notice of such burdens on the property as were noted
on the register or the certificate of title. To have required them to do more
ISSUE/s: would have been to defeat the primary object of the Torrens System which is
2. WoN Spouses Lumo should be considered innocent pruchasers for value. – to make the Torrens Title indefeasible and valid against the whole world.
YES 9. The rights created by the PD 1529 of course do not and cannot accrue under
an inscription in bad faith. Mere registration of title in case of double sale is
RULING: Wherefore, petition is hereby denied. So ordered. not enough; good faith must concur with the registration.
10. The “priority in time” principle being invoked by Naawan is misplaced
RATIO: because its registration referred to land not within the Torrens System
1. Both parties cite Article 154413 of the Civil Code which governs the double but under Act 3344. On the other hand, when Spouses Lumo bought the
sale of immovable property. subject property, the same was already registered under the Torrens System.
2. Naawan contends that its earlier registration of the sheriff’s deed of final It is a well-known rule in this jurisdiction that persons dealing with registered
conveyance under Act 3344 should prevail over the later registration of land have the legal right to rely on the face of the Torrens Certificate of Title
Spouses Lumo under Act 496 as amended by PD 1529. and to dispense with the need to inquire further, except when the party
3. This contention has no leg to stand on. It has been held that, where a person concerned has actual knowledge of facts and circumstances that would imple
claims to have superior proprietary rights over another on the ground that he a reasonably cautious make to make such inquiry.
derived his title from a sheriff’s sale registered in the Registry of Property,

13
“x x x. Should it be immovable property, the ownership shall belong to the person acquiring
it who in good faith first recorded it in the Registry of Property.”
11. In this case, the Spouses Lumo exercised the required diligence to be
considered innocent purchaser for value. Before buying the property, they
check the Registry of Deeds and the Bureau of Lands regarding the status of
the vendor. No liens or encumbrances were found to have been annotated on
the certificate of title.
12. Neither were private respondents aware of any adverse claim or lien on the
property other than the adverse claim of a certain Geneva Galupo to whom
Guillermo Comayas had mortgaged the subject property. But this claim of
Galupo was already settled.
16 NAVAL v. CA (PERRAL)
Feb. 22, 2006 | Ynares-Santiago, J. | Double Sales: Immovable When the land is not registered under the Torrens System (as in this case) and
there would be a “double sale”, and the sale was recorded in in the Registry of
PETITIONER: Juanita Naval Property of the Registry of Deeds pursuant to Act no. 3344, such Act is the
RESPONDENTS: Court of Appeals, Juanito Camalla, Jaime Nacion, Conrado applicable law. Act 3344 which provides for the registration of all instrument on
Balila, Ester Moya and Porferia Aguirre. land neither covered by the Spanish Mortgage Law nor the Torrens System. Under
this law, registration by the 1st buyer is constructive notice to the 2nd buyer that
SUMMARY: Idelfonso sold a parcel of land to Gregorio in the year 1969, in which can defeat his right as such buyer in good faith. The issue of good faith and bad
the deed of sale was recorded in the Registry of Property of the Registry of Deeds of faith of buyer is relevant only where the subject of the sale is registered land…
Camarines Sur pursuant to Act 3344 the law governing registration of all instruments
on unregistered lands. Subsequently, Gregorio sold portions of the subject land to FACTS:
different buyers, herein respondents, on different dates. Controversy in this case 1. Dec. 2, 1969: Idelfonso sold a parcel of land located in Sto. Tomas,
arose when an OCT was issued by the Registry of Deeds in favor of Petitioner Naval, Camarines Sur, consisting of 858 sq.m to Gregorio B. Galarosa.
great granddaughter of Idelfonso, on April 1, 1975 (prior to the several sales made 2. The sale was recoded in the Registry of Property of the Registry of Deeds
by Gregorio). She claims that the subject property was sold to her by Idelfonso back of Camarines Sur on Dec. 3 of the same year pursuant to Act no. 3344, the
in the year 1972. With this, Petitioner Naval filed an action against respondent for law governing registration of all instruments on unregistered lands.
the recovery of the subject land contending that she has a superior right over the 3. Subsequently, Gregorio sold portions of the land to different buyers on
propery as eveidence and strengthened by the OCT (under the torrens title) issued in different dates:
her name. However, the case was dimissed due to the failure to prosecute the action a. Balilla- Nov. 1976
for unreasonable length of time. 20 years after, petitioner re-filed the complaint for b. Nacion- Jan. 1977
recovery of possession and damages before the MCTC against respondents. MCTC c. Sps. Moya- July 1977
ruled in favor of petitioner ordering respondents to vacate the premise and dec. that d. Camalla- Sept. 1987
petitioner Naval is the legal owner. Respondents appealed to the RTC which affirmed 4. Controversy arose when Juana Idelfonso, great granddaughter of Idelfonso,
the MCTC’s ruling. Undaunted, respondents brought the matter to the CA, which was issued in her favor, on April 1, 1975 (prior to the subsequent sales
ruled that the registration of the deed of sale of the land pursuant to Act 3344 serves made by the 1st buyer, Gregorio), by the Register of Deeds of Camarines
as a constructive notice to the subsequent buyers, thus reversed the ruling of the RTC. Sur an OCT covering 733 sq. m of the said land. She claimed that she
Hence, petitioner elevated the matter to the SC. SC agreed with CA’s decision that bought the subject property from Idelfonso in 1972.
the respondents have superior right over the subject property, however, did not apply 5. Nov. 1977: petitioner filed complaint for recovery of possession against the
art. 1544 of CC in setting forth this resolution. Instead, it applied Act 3344 which is respondents herein. However, the case was dismissed w/o prejudice,for
the law applicable when the land is not registered under the Torrens System. In failure to prosecute the action for unreasonable length of time.
applying Act 3344, SC held that the registration/recording of the subject land when 6. 20 years after, petitioner Naval re-filed the complaint for recovery of
it was sold to Gregorio, in pursuant to the said act, is considered a constructive notice possession with damages before the MCTC against the respondents (some
to herein petitioner that can defeat her right as such buyer in good faith. Also, are predecessor-in- interest already), MCTC ruled in favor of petitioner
Idelfonso was no longer the owner of the Land when he sold it to Naval since prior Naval declaring her to be the legal owner and ordering respondents to
to this sale, Gregorio was already the owner of the said land. Thus, there was no vacate.
transfer of ownership between Idelfonso and Naval. Lastly, SC held that, Certificate 7. Respondents appealed to the RTC which aAFFIRMED the assailed decision
of title registered under the Torrens System is not an absolute shield or it cannot be of MCTC. Aggrieved, the respondents elevated the case to the CA, latter
used to protect usurper from the true owner. This case falls under the Exception on finding that the prior registration of the deeds of sale between Idelfonso and
the “incontrovertible nature” of Torrens Title. Groegorio with the Register of Deeds as constructive notice to subsequent
buyers, the appellate court REVERED the decision of RTC. Petitioner was
DOCTRINE: The rule in Article 1544 of the Civil Code is inapplicable when the not happy about the said decision, hence, this petition.
subject property was unregistered at the time of the first sale. The registration ISSUE/s:
contemplated in the said Article refer to registration under the Torrens system.
3. W/N the petitioner Naval has a superior right (or title) over the land as 6. Bayoca v Nogales: registration by the first buyer under Act No. 3344 can
against the respondents herein as evidenced and strengthened by the have the effect of constructive notice to the second buyer that can defeat his
OCT under the Torrents title issued in the name of the former– NO. right as such buyer. On account of the undisputed fact of registration
Since Act 3344 is applicable and not Article 1544 of Civil Code. (see under Act No. 3344 by [the first buyers], necessarily, there is absent good
Ratio) faith in the registration of the sale by the [second buyers] for which they
had been issued certificates of title in their names. It follows that their title
RULING: The Petition is DENIED. The SC AFFIRMED the decision rendered by to the land cannot be upheld.
Court of Appeals. 7. Even if petitioner argues that she purchased and registered the subject land
in good faith and without knowledge of any adverse claim thereto,
RATIO: respondents still have superior right over the disputed property. We held
1. It not disputed that the subject land belonged to IDelfonso and that it was not in Rayos v. Reyes: The issue of good faith or bad faith of the buyer is
registered under the Torrens System when it was sold to Gregorio in 1969 relevant only where the subject of the sale is registered land and the
and to petitioner Naval in 1972. Further the deed of sale bet. Idelfonso and purchaser is buying the same from the registered owner whose title to
Gregorio was registered with the Register of Deeds pursuant to Act no. the land is clean…in such case the purchaser who relies on the clean
3344 (as shown by an Inscription at the back). title of the registered owner is protected if he is a purchaser in good
2. In holding that respondent have a better right to possess the subject land in faith for value. Since the properties in question are unregistered lands,
view ot the bona fide registration of the sale with the Register of Deeds by petitioners as subsequent buyers thereof did so at their peril. Their claim of
Idelfonso and Greogorio, CA applied Article 1544 of the Civil Code which having bought the land in good faith, i.e., without notice that some other
the SC, while agreeing with CA that respondents have superior right, person has a right to or interest in the property, would not protect them if it
nevertheless held that Aricle 154414 of CC is not applicable in this case. turns out, as it actually did in this case, that their seller did not own the
3. Article 1544 of CC is not applicable to the case at bar since the subject land property at the time of the sale.
was unregistered at the time of the first sale. The registration contemplated 8. Idelfonso was no longer the owner of the subject property when he sold
under this provision has been held to refere to registration under the Torrens it to Juana Naval.
System, which considers the act of registration as the operative act that binds a. It is an established principle that no one can give what on does not
land. Carumba case: held that art 1544 of CC has not application to land have. Once can sell only what one owns or is authorized to sell,
not registered under Torrens System. and the buyer can acquire no more than what the seller can transfer
4. The law applicable therefore is Act 3344 which provides for the registration legally.
of all instrument on land neither covered by the Spanish Mortgage Law nor b. In the case at bar, since Ildefonso no longer owned the subject land
the Torrens System. Under this law, registration by the 1st buyer is at the time of the sale to the petitioner, he had nothing to sell and
constructive notice to the 2nd buyer that can defeat his right as such the latter did not acquire any right to it.
buyer in good faith. 9. Even if Article 1544 would be applied, the respondents and their pred-in-
5. Bautista v Faule: held that the registration of an instrument involving interest registered first the source of their ownership and possession (the
unregistered land in the Registry of Deeds creates constructive notice and 1969 deed of sale, and possessed the subject land at the earliest time).
binds third person who may subsequently deal with the same property.

14
Article 1544. If the same things should have been sold to different Should there be no inscription, the ownership shall pertain to
vendees, the ownership shall be transferred to the person who may have the person who in good faith was first in possession; and in the absence
first taken possession thereof in good faith, if it should be movable thereof, to the person who presents the oldest title, provided there is
property. good faith.
Should it be immovable property, the ownership shall belong to
the person acquiring it who in good faith recorded it in the Registry of
Property.
Applying the doctrine of "priority in time, priority in rights", respondents o CA correctly held that an action for reconveyance does not
are still entitled to the ownership and possession of the subject land. prescribe when the plaintiff is in possession of the land to be
10. Certificate of title registered under the Torrens System is not an reconveyed.
absolute shield or it cannot be used to protect usurper from the true o Vda. De Cabrera case: held that, “An action for reconveyance of a
owner. (Exception on the incontrovertible nature of Torrens Title) parcel of land based on implied or constructive trust prescribes in
a. True, a certificate of title, once registered, should not thereafter be ten years, the point of reference being the date of registration of the
impugned, altered, changed, modified, enlarged or diminished deed or the date of the issuance of the certificate of title over the
except in a direct proceeding permitted by law. property, but this rule applies only when the plaintiff or the person
b. Moreover, Section 32 of Presidential Decree No. 1529 provides enforcing the trust is not in possession of the property, since if a
that "upon the expiration of said period of one year, the decree of person claiming to be the owner thereof is in actual possession of
registration and the certificate of title shall become the property, as the defendants are in the instant case, the right to
incontrovertible." seek reconveyance, which in effect seeks to quiet title to the
c. However, it does not deprive an aggrieved party of a remedy in property, does not prescribe. The reason for this is that one who is
law. What cannot be collaterally attacked is the certificate of in actual possession of a piece of land claiming to be the owner
title and not the title or ownership which is represented by such thereof may wait until his possession is disturbed or his title is
certificate. Ownership is different from a certificate of title. attacked before taking steps to vindicate his right, the reason for
d. The fact that petitioner was able to secure a title in her name did the rule being, that his undisturbed possession gives him a
not operate to vest ownership upon her of the subject land. continuing right to seek the aid of a court of equity to ascertain and
Registration of a piece of land under the Torrens System does not determine the nature of the adverse claim of a third party and its
create or vest title, because it is not a mode of acquiring effect on his own title, which right can be claimed only by one who
ownership. is in possession.”
e. A certificate of title is merely an evidence of ownership or title
over the particular property described therein.37 It cannot be used
to protect a usurper from the true owner; nor can it be used as
a shield for the commission of fraud; neither does it permit one
to enrich himself at the expense of others.
f. CA correctly held tha, notwithstanding the indefeasibility of the
Torrens title, the registered owner may still be compelled to
reconvey the registered property to its true owners. The rationale
for the rule is that reconveyance does not set aside or re-subject to
review the findings of fact of the Bureau of Lands. In an action for
reconveyance, the decree of registration is respected as
incontrovertible. What is sought instead is the transfer of the
property or its title which has been wrongfully or erroneously
registered in another person’s name, to its rightful or legal owner,
or to the one with a better right.
11. Paramount reason for this exception is based on the theory that registration
proceedings could not be used as a shield for fraud. Moreover, to hold
otherwise would be to put premium on land-grabbing and transgressing the
broader principle in human relations that no person shall unjustly enrich
himself at the expense of another.

- Issue on prescription (just in case atty. Will ask)


17 GOPIAO v. METROPOLITAN BANK & TRUST (PAT) 10. The RTC of Pampanga issued a writ of possession in favor of Metrobank
July 28, 2014 | Peralta, J. | Double Sale when it purchased the said properties at a public auction and subsequently
registered it.
Petitioner: Juanito Gopiao 11. A Notice to Vacate was served on Green Asia Construction and Development
Respondent: Metropolitan Bank & Trust Co.
Corp. represented by the Spouses Renato and Delia Legaspi (Legaspis).
SUMMARY: Case stemmed when Metrobank filed for a Writ of Possession of 3 properties 12. Juanito Gopiao (Gopiao) learned of this notice, and so he filed the following:
of the Legaspis. This is because the Legaspis loaned from Metrobank but they most likely a. Affidavit of Third Party Claim
failed to pay, so the bank foreclosed, a public auction was held, and Metrobank was the b. Very Urgent Motion for Intervention
highest bidder. RTC issued a Writ of Possession in favor of Metrobank and a Notice to c. Recall and/or Stop the Enforcement/Implementation of Writ of
Vacate was served on the Legaspis. Gopiao found out about this and filed the following: (a) Possession.
Affidavit of Third Party Claim, (b) Very Urgent Motion for Intervention, and (c) Recall 13. In these actions, Gopiao claims that he is in actual occupation of the
and/or Stop the Enforcement/Implementation of Writ of Possession. Gopiao claims that he properties and he claimed ownership by virtue of a Deed of Sale.
is in actual possession of the properties and that he owns them based on the fact that a Deed
14. RTC denied his claims, so he filed for a Motion for Reconsideration but it
of Sale was executed between him and Legaspis. RTC denied his claim and MR. On appeal
to the CA, they also denied his claim and MR. Hence, this petition. The issues in this case
was likewise denied.
are whether or not CA erred in ruling that the RTC committed no grave abuse of discretion 15. He elevated his claim to the CA via petition for certiorari but the CA
when it denied Gopiao’s intervention and whether or not CA erred in ruling that there exists dismissed the case; he also filed for a MR but it was also denied.
a double sale in this case and that good faith is material. With regard to first issue, the SC
held that CA did not err. This is because although the issuance of a writ of possession is a ISSUE/S:
ministerial function, thus can’t be enjoined or restrained, the rule admits of an exception: 1. WON CA erred in ruling that the RTC committed no grave abuse of
when a third party in possession of the property claims a right adverse to the debtor- discretion in denying Gopiao’s intervention even if the RTC violated existing
mortgagor. But in Gopiao’s case, he wasn’t able to prove that a sale actually occurred jurisprudence. – NO.
because the titles did not have any annotations evidencing that a sale actually happened. As 2. WON CA erred in ruling that there exists a double sale in this case and that
to the second issue, the SC held that jurisprudence provides that the rule on double sale
would apply in a case wherein 1 of the 2 sales was conducted in a public auction. In addition,
good faith is material. – NO.
the CA’s discussion re double sale and good faith were on the assumption that the Legaspis
sold to Metrobank and Gopiao. But it has already been established that the sale to Gopiao HELD: WHEREFORE, premises considered, the instant petition is DENIED. The
is doubtful. So even assuming arguendo that Gopiao is able to establish his possession, he Orders of the Regional Trial Court and the Decision and Resolution of the Court of
still has to overcome the rule on double sale where good faith of Metrobank is material. Appeals are hereby AFFIRMED.

DOCTRINE: Art. 1544: “If the same thing should have been sold to different vendees, the RATIO:
ownership shall be transferred to the person who may have first taken possession thereof in On whether CA erred in ruling that RTC did not commit GAOD.
good faith, if it should be movable property. Should it be immovable property, ownership
• Gopiao: RTC gravely abused its discretion by ordering the writ of possession
shall belong to the person acquiring it who in good faith recorded it in the Registry of
Property. Should there be no inscription, the ownership shall pertain to the person because since he is a third party in possession of the properties and he is
who in good faith was first in possession; and, in the absence thereof, to the person claiming a right adverse to that of the Legaspis, the writ should not be
who presents the oldest title, provided there is good faith.” Even assuming arguendo implemented against him. Invoked the ff jurisprudene:
that Gopiao is able to establish his possession, he still has to overcome the rule on double o Heirs of Nicolas v. Metrobank: heirs should not be deprived of their
sale where good faith of Metrobank is material. legitime by the enforcement of the writ of possession.
o Dayot v. Chemical Company: RTC was without authority to grant the ex
parte writ of possession in favor of Dayot since Shell has been
FACTS: indisputably in possession of the lots.
9. The case stemmed from LRC Case No. 666 which was a Petition for the o PNB v. CA: Ex parte writ could only be rightfully recognized against the
Issuance of Writ of Possession of 3 real properties, filed by Metropolitan judgment debtors but not against private respondents who assert a right
Bank (Metrobank). adverse to the judgment debtors.
• SC: Writ of possession – executed to enforce judgment to recover possession
of land. Issuance of such is a ministerial function, thus it can’t be enjoined or
restrained, but this rule admits of exceptions: when a third party in possession
of the property claims a right adverse to that of the debtor-mortgagor.
o Cases above are not identical with the facts of this case. Main difference:
certainty of possession. In all those cases, the possession of the adverse
third party is undisputed.
o But in this case, Gopiao’s possession of the property is questionable
since the Deed of Absolute Sale relied upon by him is neither complete
nor in due form. There were essential details that were missing: tax
account numbers, names of the witnesses, and it wasn’t notarized.
o He did not present other evidence to substantiate his claim.
o The claim that there was a Deed of Sale can’t be inferred from the titles
because it was not annotated in the titles.
o Metrobank found the titles and latest tax declarations to be registered still
under the Legaspis and they also found that there were no occupants in
the property.

On whether the CA erred in ruling that there exists a double sale in this case and
thus good faith is material.
• Gopiao: rule on double sales under Art. 1544 of the CC is inapplicable
since there is no double sale to speak of; 1st transaction: sale, 2nd:
mortgage.
• SC: Several jurisprudence would show that the rule on double sale
applies to cases where 1 of the 2 sales was conducted in a public auction.
o Expresscredit v. Sps.Velasco: Property was sold first to spouses by
DOAS and then to Expresscredit in a foreclosure sale. SC ruled in
favor of spouses because Expresscredit was in bad faith since they
were aware of the earlier sale to spouses.
o CA applied good faith of Metrobank in this case because prior to the
approval of the loan application of the Legapsis, they did the
following: a) Checked the records in the Register of Deeds to see if
the titles were clean and b) Inspected premises and found no
occupants. Hence, Metrobank can’t be said to be in bad faith.
o CA’s discussion on double sale and good faith was based on
assumption, for the sake of argument, that Legaspis actually sold to
both Gopiao and Metrobank, but there is doubt whether Legaspis
actually sold to Gopiao.
o What can be derived from CA’s decision: even if Gopiao is able to
establish his possession, he would still have to overcome the rule on
double sale where good faith of Metrobank is material.
18 CARILLO v. CA (MICA) 3. The owner of the land is Priscilla’s son (Aristotle)
Sept. 26, 2006 | Quisumbing, J. | Double sale 4. April 26, 1988 – Gonzales paid P10,000 to Priscilla as downpayment on the
P400,000 purchase price of the lot with improvements since Priscilla had a
PETITIONER: Hon. Dominador F. Carillo, Presiding Judge, RTC XI-19 Digos, special power of attorney from her son
Davao del Sur; Bonifactio J. Guyot, Clerk of Court and Provincial Sheriff of 5. Agreed that the balanced would be paid within 3 months after the execution
Davao del Sur; Alfredo C. Senoy, Deputy Prov. Sheriff assigned to RTC XI-19 of the deed of sale (The deed of sale was not executed even after the period
Digos, Davao del Sur; and Maria Gonzales and repeated demands)
RESPONDENTS: Hon. Court of Appeals, Maria Paz Dabon and Rosalina Dabon 6. Gonzales filed an action for specific performance against the spouses
7. The Spouses failed to answer so they were declared in default and Gonzales
SUMMARY: Gonzales bought a lot with improvements from Priscilla Manio. presented evidence ex parte
Priscilla Manio’s son, Aristotle, is the owner of the property but through a special 8. Trial court rendered judgment in favor of Gonzales.
power of attorney, Priscilla became an agent. Gonzales paid P10,000 as a. Part of judgment: to execute the final deed of sale and transfer of the
downpayment but Priscilla did not execute the deed of sale. Gonzales then, filed property mentioned in par. 4 above to plaintiff, or should the
an action for specific performance with damages and attorney’s fees. TC ruled in defendant refuse to execute the deed of sale, the Clerk of Court be
favor of Gonzales and eventually, declared void the owner’s duplicate of the TCT directed to execute the same upon plaintiff’s depositing of the sum
and ordered that a new TCT be issued in Gonzales’ name. Dabons filed a petition of P390,000 with the Clerk of Court as complete and valid payment
to annul judgment and orders alleging that they were fraudulently deprived of their thereof to defendant Priscilla Manio
right to due process and that there was a lack of jurisdiction over them, who are b. She needs to give 390,000 cause she only paid 10,000 so far.
the real parties in interest. CA cancelled the TCT under the name of Gonzales. The 9. Gonzales deposited the amount with the Clerk of Court and filed a motion for
issue in this case is whether or not the doctrine of double sale holds relevance in execution. However, she withdrew the motion because the trial court’s
this case. NO. No double sale in this case. (check doctrine) SC did not discuss it decision was not properly served on the defendants
because it is outside the scope of the petition for review. The appellate court only 10. After numerous delays, the sheriff personally served a copy of the decision
allowed the reception of extraneous evidence to determine extrinsic fraud. To to Priscilla at 12 midnight
determine which sale was valid, review of evidence is necessary. “This we cannot 11. The Trial court’s decision became final and executory but the writ of
do in this petition.” An action for annulment of judgment is independent of the execution was not served because the sheriff said that the defendants cannot
case where the judgment sought to be annulled is rendered and is not an appeal of be located. The Sheriff also informed the Trial court that the money judgment
the judgment therein. could be satisfied by the petitioner’s cash deposit should the TC grant the
motion to release the cash deposit filed by Gonzales
DOCTRINE: (According to Art. 1544): Where it is immovable property that is 12. Gonzales filed a motion asking the Clerk of Court to be the one to execute a
the subject of double sale, ownership shall be transferred: (1) To the person deed of conveyance. She also filed a motion to withdraw the cash deposit to
acquiring it who in good faith first recorded it in the Registry of Property; (2) In offset the award of damages – granted both but modified to P207,800
default thereof, to the person who in good faith was first in possession; and (3) In 13. Oct. 29, 1990 – Gonzales filed a petition for the nullification of the Owner’s
default thereof, to the person who presents the oldest title, provided there is good Duplicate Certificate and asked that a new certificate be issued in her name
faith. The requirement of the law is two-faold: acquisition in good faith and (granted by the TC)
registration in good faith 14. Dec. 14, 1990 – Maria Paz Dabon and Rosalina Dabon, claiming to have
bought the aforementioned lot from Aristotle Manio, filed before the CA a
petition for annulment of judgment and orders of the RTC in Civil Case No.
FACTS: *it was not stated directly but I think lahat ng before pumasok yung
2647, Maria Gonzales v. Priscilla Manio and Jose Manio
Dabons, yun part ng Civil Case No. 2647
15. The allegations of the Dabons:
1. April 2, 1990 – Maria Gonzales filed a complaint against the spouses Priscilla
a. Judgment of the trial court was void
and Jose Manio with the RTC of Digos, Davao del Sur, Branch19
b. Because of lack of jurisdiction over their persons, as the real parties
2. Gonzales sought the execution of the deed of sale in her favor for the property
in interest
she bought from Priscilla Manio. (Also asked for damages and attorney’s
c. And they were fraudulently deprived of their right to due process
fees)
16. Also prayed for temporary restraining order and for preliminary prohibitory
injunction against Gonzales an agent acting in his own name and for the benefit of an undisclosed principla without
17. Gonzales filed before the TC a motion for the issuance of a writ of possession joining the principal, except when the contract involves things belonging to the
18. The Dabons filed an opposition: principal
a. The writ of possession cannot be enforced because the defendants 5. Real party in interest: party who would be benefited or injured by the judgment or is
the party entitled to the avails of the suit.
named in the writ, the Manios, were no longer in possession of the 6. An attorney-in-fact is not a real party in interest and that there is no law permitting an
property action to be brought by and against an attorney-in-fact
b. They had bought the lot with the improvements therein and had 7. Here, Gonzales filed an action to compel Priscilla to execute a deed of sale, involving
taken possession, although they had not yet registered their real property which, however, does not belong to Priscilla but to Aristotle Manio
ownership with the Register of Deeds 8. Priscilla does not have interest and can have no interest whatever in any judgment
c. The Court did not acquire jurisdiction over them as the real parties rendered
in interest 9. She did not act on her own name nor for the benefit of an undisclosed principal
19. CA: issued a resolution restraining the trial court from implementing its 10. Also, a person need not be a party to the judgment sought to be annulled. What is
essential is that he can prove his allegation that the judgment was obtained by fraud
decision and its subsequent orders until further notice from the CA
or collusion and he would be adversely affected thereby (only needs preponderance
20. CA: of evidence)
a. Contract of sale between Gonzales and Priscilla was unenforceable 11. So, even if the respondents were not parties to the case, they can annul judgment if
because the sale was evidenced by a handwritten note which was there’s a finding that there was extrinsic fraud and this would adversely affect the
vague as to the amount and which was not notarized respondents’ ownership
b. The TC did not acquire jurisdiction over the indispensable parties 12. Sec. 2 of Rule 47 of the Rules on Civil Procedure: 2 grounds for annulment of
c. Proceedings were attended with fraud judgment:
d. Cancelled the TCT under Gonzales’ name a. Extrinsic fraud: when a party has been prevented by fraud or deception from
presenting his case. This prevented a party from having his day in court
(intrinsic fraud: acts of a party at a trail which prevented a fair and just
ISSUE/s:
determination of the case, and which could have been litigated and
1. Whether or not the doctrine of double sale holds relevance in this case - NO determined at the trial or adjudication of the case)
i. CA found that indices of fraud attended the case: deliberately
RULING: The petition is denied for lack of merit. The decision of the CA is affirmed. excluding the Dabons despite knowledge that the Dabons had
alleged that they had bought the land from Aristotle; Sheriff’s
RATIO: return was served on a Saturday, at midnight; TC ordered the
1. Gonzales’ contentions: deposit of full amount then ordered its withdrawal; no notice
a. Respondents do not have standing before the CA to file a petition for given to the person named in the cert of title
annulment of the judgment in Civil Case No. 2647 because respondents ii. Failure to comply with the notification requirement in the petition
were not parties therein for the cancellation of title amounts to extrinsic fraud
b. Respondents have no right that could be adversely affected by the judgment b. Lack of jurisdiction: Under Rule 47: the judgment is void over the persons
because they are not the owners of the property of the real parties in interest (Aristotle and the Dabons)
c. CA should have applied the doctrine of double sale (YUP THIS IS THE
IMPORTANT DOCTRINE IN THIS CASE) to settle the issue of ITO NA YUNG TOPIC FOR SALE FINALLY:
ownership and declare her the true owner of the property 13. Petitioner insists that the contract of sale between her and Priscilla was valid
d. Did not implead Aristotle as defendant in Civil case no. 2647 since a
and enforceable because under the provision on double sale, she onwed the
decision against Priscilla, Aristotle’s attorney-in-fact, would bind Aristotle
also
land because she bought the lot on April 26, 1988, while the same was
2. Maria Paz and Rosalina Dabon: they are parties in interest as buyers, owners and allegedly sold to the Dabons on Oct. 19, 1989
possessors of the contested land and that they had been fraudulently deprived of their 14. Doctrine on double sale holds no relevance in this case
day in court during the proceedings in the trial court 15. Civil Code:
a. No other remedy a. Art. 1544. If the same thing should have been sold to different
3. Gonzales should be reminded of Section 3 of Rule 3 of the Rules on Civil Procedure vendees, the ownership shall be transferred to the person who may
which states that an action should be brought against the real party in interest have first possession thereof in good faith, if it should be movable
4. Also, in case the action is brought against the agent, the action must be brought against party.
Should it be immovable property, the ownership shall belong the
person acquiring it who in good faith recorded it in the Registry of
Property
Should there be no inscription, the ownership shall pertain to the
person who in good faith was first in possession; and in the absence
thereof; to the person who presents the oldest title, provided there is
good faith
16. Where it is immovable property that is the subject of double sale, ownership
shall be transferred:
a. To the person acquiring it who in good faith first recorded it in the
Registry of Property
b. In default thereof, to the person who in good faith was first in
possession; and
c. In default thereof, to the person who presents the oldest title,
provided there is good faith.
17. The requirement of the law is two-faold: acquisition in good faith and
registration in good faith
18. It was emphasized that the action for annulment of judgment under Rule 47
of the Rules of Court does not involve the merits of the final order of the trial
court.
19. The issue of whether this is a case of double sale is outside the scope of the
present petition for review.
20. The appellate court only allowed the reception of extraneous evidence to
determine extrinsic fraud. To determine which sale was valid, review of
evidence is necessary. “This we cannot do in this petition.”
21. An action for annulment of judgment is independent of the case where the
judgment sought to be annulled is rendered and is not an appeal of the
judgment therein.
22. The extraneous evidence presented to the appellate court cannot be used to
supplant the evidence in the records of the specific performance case because
the extraneous evidence was not part of the records on the merits of the case.
23. Again, the extraneous evidence was only allowed merely to prove the
allegations of extrinsic fraud.
24. Annulment of judgment is a recourse equitable in character and allowed only
in exceptional cases as where there is no available or other adequate remedy.
25. This case falls under said exception.
26. In this case, where it was found that the trial court did not have jurisdiction
over the real parties in interest, and that notices were deliberately not given,
amount to extrinsic fraud. The Court of Appeals did not err in granting the
annulment of the judgment in Civil Case No. 2647 and the orders subsequent
thereto, for lack of jurisdiction and extrinsic fraud.
19 Carbonell v. CA (Cristelle)
January 26, 1976| Makasiar, J. | art. 1544 & primus tempore, potior jure Additonal Doctrine: The principle of primus tempore, potior jure (first in time,
stronger in right) gains greater significance in case of a double sale of immovable
PETITIONER: ROSARIO CARBONELL property. When the thing sold twice is an immovable, the one who acquires it and
RESPONDENTS: HONORABLE COURT OF APPEALS, JOSE PONCIO, EMMA first records in the Registry of Property, both made in good faith, shall be deemed the
INFANTE and RAMON INFANTE owner. Verily, the act of registration must be coupled with good faith – that is, the
SUMMARY: Poncio is the owner of the parcel of land subject of this dispute. Such land registrant must have no knowledge of the defect or lack of title of his vendor or must
is also the subject of a mortgage in favor of the Republic Savings Bank. Since he is not have been aware of facts which would have put him upon such inquiry and
unable to continue with the payment of mortgage, he offered to sell it his neighbor, investigation as might be necessary to acquaint him with the defects in the title of his
Carbonell, who was willing to buy it at P9.50 per sqm. They executed a memorandum vendor.
on their agreement. When Carbonell was preparing to execute a formal Deed of Sale
with Poncio, the latter refused to do so because he actually sold the lot to his FACTS:
other neighbors, the Sps. Infante who paid a higher price. Upon learning 1. Jose Poncio, a native of the Batanes Islands, was the owner of the parcel of
this, Carbonell wanted to meet with the Infantes but they did not entertain her request. land herein involve with improvements situated at 179 V. Agan St., San Juan,
Carbonell then registered the land under her name on Feb. 8, 1955. Once the formal Rizal, having an area of some one hundred ninety-five (195) square meters,
Deed of Sale with the Infantes was finalized, they also registerd the land under their more or less, covered by TCT No. 5040 and subject to mortgage in favor of
name on Feb. 12, 1955 with an annotation of the adverse claim of Carbonell. Carbonell the Republic Savings Bank for the sum of P1,500.00. Petitioner Rosario
is now seeking to have the sale between Poncio and Infantes nullified and that she be Carbonell, a cousin and adjacent neighbor of respondent Poncio, and also
declared the rightful owner. The contention of the Infantes, on the other hand, is that the from the Batanes Islands, lived in the adjoining lot at 177 V. Agan Street.
sale with Carbonell is unenforceable 2. This land is subject to mortgage in favor of the Republic Savings Bank
as the same was not written in a public document. for the sum of P1,500. Carbonell is the cousin and adjacent neighbor of
Issue: Whether or not Carbonell had the better right to the lot? SC ruled in favor of Poncio.
Carbonell. In order to claim the benefit of Art. 1544, the buyer of realty must register 3. Carbonell offered to buy the disputed land from Poncio. Since Poncio
the property in good faith. It is a pre-condition to a superior title. In this case, Infante was unable to keep up with the installments due on the mortgage, he
was not in good faith, thus the prior sale to Carbonell must prevail. Infante registered offered to sell the lot to Carbonell, to which she accepted and proposed
her claim 4 days after the adverse claim was registered, she had notice that Carbonell the price of P9.50 per sqm.
paid off the mortgage debt as the mortgage passbook was already in his possession. She 4. Poncio accepted the proposed price on the condition that from the purchase
likewise ignored Carbonell and refused to talk to here. These are badges of bad faith that price would come the money to be paid to the bank.
taint her registration. 5. In the presence of a witness, both parties made and executed a memorandum
It is shown that the Sps. Infante were purchasers in bad faith. They refused to in the Batanes dialect, which was later translated into English, containing
meet with Carbonell. Carbonel was in possession of the mortgage passbook and that the their contract of sale.
Sps. Infante never bothered to inquire why it was with her at the time they purchased the 6. They stipulated that Poncio can still live on the lot sold for 1 year during
land from Poncio. Carbonell’s registration of the land was also earlier than that of the which he will not pay any rent to Carbonell. They went to the Republic
spouses. The memorandum executed between Poncio and Carbonell is sufficient to Saving Bank to seek the consent of the bank president and afterwards,
prove that there was a valid sale between them as the elements of sale, such as Carbonell assumed the obligation of paying the mortgage.
consideration was present. There was also partial performance done by Carbonell so as 7. Carbonell then asked for the preparation of a formal deed of sale with Poncio.
to remove their sale from the restrictions of the Statute of Fraud. 8. Both petitioners Rosario Carbonell and respondent Emma Infante
offered to buy the said lot from Poncio.
DOCTRINE: Ownership of an immovable property which is the subject of a double 9. However, Poncio informed Carbonell that he had actually given the lot
sale shall be transferred: (1) to the person acquiring it who in good faith first recorded it to the Emma Infante and that he could not withdraw the sale to the
in the Registry of Property; (2) in default thereof, to the person who in good faith was Infantes even if he will go to jail.
first in possession; and (3) in default thereof, to the person who presents the oldest titled, 10. In their agreement, Infante bought the lot for P2,357. 52 higher price than
provided there is good faith. The requirement of the law then is two-fold: acquisition in what Carbonell paid and assumed the responsibility of paying the existing
good faith and registration in good faith. Good faith must concur with registration. mortgage debt.
11. Carbonell then sought after the Infante but the latter refused to meet. 2. It is essential that the buyer of realty must act in good faith in registering his
During this time, Carbonell, with the assistance of counsel, initiated the deed of sale to merit the protection of the second paragraph of said Article
registration of the land. The land was registered under Carbonell’s name 1544.
on Feb. 8, 1955. 3. Unlike the first and third paragraphs of said Article 1544, which accord
12. Once the Deed of Sale with Infante was finalized, she registered the lot under preference to the one who first takes possession in good faith of personal or
her name on Feb. 12, 1955. real property, the second paragraph directs that ownership of immovable
13. The Transfer Certificate of Title that was issued to her was annotated with
property should be recognized in favor of one "who in good faith first
the adverse claim of Carbonell. Infante took immeditate possession of the lot
involved and spent an amount of P11,929 worth of improvements. recorded" his right. Under the first and third paragraph, good faith must
14. Carbonell filed a complaint against the Infantes and Poncio praying that: characterize the act of anterior registration.
a. she be declared the lawful owner of the questioned parcel of land 4. Should there be no inscription, the ownership shall pertain to the person who
b. subsequent sale between the Infantes and Poncio declared null and in good faith was first in the possession; and, in the absence thereof, to the
void person who presents the oldest title, provided there is good faith.
c. Poncio be ordered to execute the corresponding deed of conveyance 5. If there is no inscription, what is decisive is prior possession in good faith.
of said land If there is inscription, as in the case at bar, prior registration in good
d. Damages and attorney’s fees faith is a pre-condition to superior title.
15. The respondents filed a motion to dismiss on the grounds that the sale 6. When Carbonell bought the lot from Poncio on January 27, 1955, she was the
between Poncio and Carbonell was unenforceable under the Statue of Frauds only buyer thereof and the title of Poncio was still in his name solely
for not being evidenced by a written document. encumbered by bank mortgage duly annotated thereon. Carbonell was not
16. RTC: initially ruled in favor of Carbonell and declared that the sale between aware and she could not have been aware of any sale of Infante as there was
Poncio and Infante was null and void. But a re-hearing of the trial occurred no such sale to Infante then.
wherein the Infantes presented new evidence regarding the improvements 7. Carbonell's prior purchase of the land was made in good faith. Her good
they made on the land. Thus, the decision was reversed in favor of the faith subsisted and continued to exist when she recorded her adverse
Infantes. claim four (4) days prior to the registration of Infantes's deed of sale.
17. CA: reversed the last decision declaring Carbonell to have a superior right to Carbonell's good faith did not cease after Poncio told her on January 31,
the land and condemning the Infantes to reconvey to the petitioner the 1955 of his second sale of the same lot to Infante.
property and with reimbursement. 8. When she tried to approach Infante to settle the dispute, the latter ignored her
ISSUE: W/N Carbonell had a superior right to the lot? YES with an aristocratic disdain unworth of the good breeding of a good Christian
RULING: WHEREFORE, THE DECISION OF THE SPECIAL DIVISION OF FIVE OF THE COURT and good neighbor. It was then only proper for Carbonell to protect her right
OF APPEALS OF OCTOBER 30, 1968 IS HEREBY REVERSED; PETITIONER ROSARIO and register the land.
CARBONELL IS HEREBY DECLARED TO HAVE THE SUPERIOR RIGHT TO THE LAND IN 9. Indication of bad faith on the part of Infante - Infante refused to see
QUESTION AND IS HEREBY DIRECTED TO REIMBURSE TO PRIVATE RESPONDENTS
INFANTES THE SUM OF ONE THOUSAND FIVE HUNDRED PESOS (P1,500.00) Carbonell, Carbonell was already in possession of the mortgage passbook
and Poncio’s copy of the mortgage contract when Poncio sold the lot to
RATIO: Carbonell, who after paying the arrears of Poncio assumed the balance of his
1. Art. 1544 of the Civil Code states: If the same thing should have been sold to mortgage, Poncio was no longer in possession of his mortgage passbook
different vendees, the ownership shall be transferred to the person who may should have compelled Infante to inquire from Poncio why he was no longer
have first taken possession thereof in good faith, if it should be movable in possession of the mortgage passbook, Carbonell registered on February 8,
property. Should it be immovable property, the ownership shall belong to 1955 her adverse claim, which was accordingly annotated on Poncio's title, 4
the person acquiring it who in good faith first recorded it in the Registry days before Infante registered on February 12, 1955 her deed of sale executed
of Property. Should there be no inscription, the ownership shall pertain to on February 2, 1955. Here she was again on notice of the prior sale to
the person who in good faith was first in the possession; and, in the absence Carbonell. Such registration of adverse claim is valid and effective and
thereof, to the person who presents the oldest title, provided there is good Infante offered to buy at a higher price but Poncio refused. It is therefore
faith. logical to presume that Infante was told by Poncio and consequently knew of
the offer of Carbonell.
10. It can be deduced from the foregoing that Poncio was induced by the Infantes 1. Teehankee Concurring - who between the two buyers in good faith should
to sell the land to them because they offered a higher price. Also, these parties prevail is provided in the second paragraph of Article 1544 which ordains
are neighbors that’s why there is no reason why Infante refuse to meet with that "the ownership of the immovable property shall belong to the person
Carbonell since they just live near each other. They were clearly in bad faith. acquiring it who in good faith first recorded it in the Registry of Property."
11. The existence of the prior sale to Carbonell was also duly established. Since Carbonell (the first buyer) did not have a formal registrable deed
a. The memorandum between Carbonell and Poncio shows that the of sale, she did the next best thing to protect her legal rights and
sale of property in favor of Carbonell is already an accomplished registered on February 8, 1955 with the Rizal Register of Deeds her
act. adverse claim as first buyer entitled to the property. The second buyer
b. There was actualy performance on the part of Carbonell when she Infante registered the deed of sale in her favor with the Rizal Register of
paid P247.76 to Poncio (as shown in his bank deposit book) which Deeds only on February 12, 1955 (notwithstanding its having been executed
would effectively remove it from the ambit of the Statute of Frauds. ten days earlier on February 2, 1955), and therefore the transfer certificate
c. The memorandum also shows a sufficient description of the lot sold of title issued in her favor carried the duly annotated adverse claim of
to Carbonell. Carbonell as the first buyer.
12. There is also adequate consideration or price for the sale in favor of Both these registrations were in good faith and hence, as provided by the cited code
Carbonell. It is to be remembered that Poncio sold the lot because he could article, the first buyer Carbonell as also the first registrant is legally entitled to the
not pay the remainder of his mortgage. To forestall the foreclosure, he sold property.
the lot to Carbonell to which she agreed to assume the obligation of the The governing principle here is prius tempore, portior jure (first in time, stronger
mortgage on top of her payment of P9.50 per sqm to Poncio. in right). Knowledge gained by the first buyer of the second sale cannot defeat
13. While petitioner Carbonell has the superior title to the lot, she must however the first buyer's rights except only as provided by the Civil Code and that is where
refund to respondents Infantes the amount of P1,500.00, which the Infantes the second buyer first registers in good faith the second sale ahead of the first.
paid to the Republic Savings Bank to redeem the mortgage. Such knowledge of the first buyer does not bar her from availing of her rights
14. It appearing that the Infantes are possessors in bad faith, their rights to the under the law, among them, to register first her purchase as against the second
improvements they introduced to the disputed lot are governed by Articles buyer. But in other so knowledge gained by the second buyer of the first sale
546 and 547 of the New Civil Code. defeats his rights even if he is first to register the second sale, since such
a. Their expenses consisting of P1,500.00 for draining the property, knowledge taints his prior registration with bad faith.
filling it with 500 cubic meters of garden soil, building a wall around This is the price exacted by Article 1544 of the Civil Code for the second buyer being
it and installing a gate and P11,929.00 for erecting a bungalow able to displace the first buyer: that before the second buyer can obtain priority over
thereon, are useful expenditures, for they add to the value of the the first, he must show that he acted in good faith throughout the time of acquisition
property until the title is transferred to him by registration or failing registration, by delivery of
15. Reimbursement for improvement: Since the Infantes paid a reasonable possession. The second buyer must show continuing good faith and innocence or lack
amount for the improvement, Carbonell should reimburse them. But, if the of knowledge of the first sale until his contract ripens into full ownership through prior
lawful possessor can retain the improvements introduced by the possessor in registration as provided by law.
bad faith for pure luxury or mere pleasure only by paying the value thereof at 2. Munoz Palma Dissenting -Rosario Carbonell cannot be held to have a title
the time he enters into possession (Article 549), as a matter of equity, the superior to that of Emma Infante for even if We were to concede that the
Infantes, although possessors in bad faith, should be allowed to remove the notation of her adverse claim on February 8, 1955, was in the nature of
aforesaid improvements, unless petitioner Carbonell chooses to pay for their registration of title as required in Art. 1544 the same was not
value at the time the Infantes introduced said useful improvements in 1955 accomplished in good faith. At the time Carbonell caused the
and 1959. annotation of her adverse claim she was, therefore, cognizant of facts
a. The Infantes cannot claim reimbursement for the current value of
which impaired her title to the property in question, and taking
the said useful improvements; because they have been enjoying such
advantage of the situation that the second purchaser hadnot as yet
improvements for about two decades without paying any rent on the
land and during which period herein petitioner Carbonell was registered her deed of sale, she went ahead of the second buyer and
deprived of its possession annotated what was only in the nature of an adverse claim inasmuch as she
Opinion: had no registrable document of sale at the time. That annotation of
Carbonell's adverse claim did not produce any legal effects as to place her
in a preferential situation to that of Infante, the second purchaser, for the
simple reason that a registration made in bad faith is equivalent to no
registration at all. It is a settled rule that the inscription in the registry,
to be effective, must be made in good faith.
20 CHENG v GENATO (CLARK) respondents first in time; it was also registered long before petitioners intrusion
December 29, 1998 | Martinez, J. | Double Sale Rule as a second buyer.

PETITIONER: Ricardo Cheng DOCTRINE: The governing principle of Article 1544 of the New Civil Code15
RESPONDENTS: Ramon B. Genato and Ernesto R. Da Jose & Socorro Da Jose should apply in this situation. Jurisprudence teaches us that the governing
principle is PRIMUS TEMPORE, PORTIOR JURE (first in time, stronger in
SUMMARY: Ramon Genato is owner of 2 parcels of land located in Paradise right). This principle only applies when the special rules provided in the aforecited
Farms, Bulacan. Genato entered an agreement w/ spouses Da Jose at P80/sqm. article of Civil Code do not apply or fit the specific circumstances mandated under
On Oct 4 1989, Da Jose spouses, not having finished verifying the titles, asked said law or by jurisprudence interpreting the article.
for and was granted by Genato an extension of another 30 days to verify titles.
Pending effectivity and without notice to Da Jose spouses, Genato executed an FACTS:
Affidavit to Annul the Contract to Sell on Oct 13, 1989. On Oct. 24, 1989, 1. Ramon B. Genato is the owner of two parcels of land located at Paradise
Ricardo Cheng went to Genato’s residence and expressed interest to buy. Genato Farms, San Jose Del Monte, Bulacan with an aggregate area of 35,821 square
showed Cheng copies of the TCT and annotations at the back of contract to sell. meters. On September. 6, 1989, Genato entered into an agreement with
Genato also showed Affidavit to Annul the Contract to Sell w/c has not been spouses Ernesto and Socorro Da Jose over the two parcels of land. The
annotated at the back of titles. Cheng then issued a handwritten receipt, which agreement culminated in the execution of a contract to sell for which the
caused Genato the registration of the Affidavit to Annul the Contract to Sell in purchase price was P80.00 per square meter. The contract was in a public
Registry of Deeds. In the said Registry of Deeds, Genato met the Da Jose instrument and was duly annotated at the back of the two certificates of title
spouses by coincidence. After being reminded that Genato had given the Da Jose on the same day. Clauses 1 and 3 provide:
spouses an additional 30-day period to finish their verification of his titles, that a. Purchase price: P80/sqm; partial down payment of P50,000 paid by
the period was still in effect, and that they were willing and able to pay the vendee to vendor at the time of execution of this Contract to Sell
balance of the agreed down payment, Genato decided to continue the Contract he b. Vendee, 30 days after execution, and only after having satisfactorily
had with them. Genato sent a letter to Cheng enclosing a BPI Cashier’s Check verified and confirmed the truth and authenticity of documents,
for P50,000 and expressed regret for his inability to consummate his transaction Vendee shall pay Vendor P950,000 representing full payment of the
with him. Cheng instituted a complaint for specific performance to compel agreed Down Payment, after which possession of the property shall
Genato to execute a deed of sale to him of the subject properties plus damages be given to the Vendee
and prayer for preliminary attachment. Cheng averred that the P50,000 check he 2. On October 4, 1989, Da Jose spouses, not having finished verifying the titles
gave was a partial payment to the total agreed purchase price of the subject mentioned in clause 3 as aforequoted, asked for and was granted by
properties and considered as an earnest money for which Genato acceded. Thus, respondent Genato an extension of another 30 days or until November 5,
their contract was already perfected. The RTC ruled in favor of Cheng. The CA 1989. However, according to Genato, the extension was granted on condition
reversed the decision. The issue is W/N was correct in ruling that there was no that a new set of documents is made 7 days from October 4, 1989. This was
valid and effective rescission of resolution of the Da Jose spouses Contract to denied by the Da Jose spouses.
Sell. SC held that the CA was correct. No default can be ascribed to the Da Jose 3. Pending the effectivity of the aforesaid extension period, and without due
spouses since the 30-day extension period has not yet expired. For with or notice to the Da Jose spouses, Genato executed an Affidavit to Annul the
without the aforesaid affidavit their non-payment to complete the full Contract to Sell, on October 13, 1989. Moreover, no annotation of the said
downpayment of the purchase price avoids their contract to sell, it being affidavit at the back of his titles was made right away. The affidavit
subjected to a suspensive condition. not only was the contract between herein contained:

15
Article 1544. If the same thing should have been sold to different vendees, the ownership shall be Should there be no inscription, the ownership shall pertain to the person who in good faith was first in the
transferred to the person who may have first taken possession thereof in good faith, if it should be possession; and, in the absence thereof, to the person who presents the oldest title, provided there is good
movable property. faith. (1473)
Should it be immovable property, the ownership shall belong to the person acquiring it who in good faith
first recorded it in the Registry of Property.
a. That it was agreed between the parties that the agreed downpayment executed an affidavit of adverse claim and had it annotated on the subject
of P950,000 shall be paid 30 days after the execution of the TCTs.
Contract, that is on or before October 6, 1989 11. Consistent with the decision of Genato and the Da Jose spouses to continue
b. The supposed vendees failed to pay the said full downpayment even with their Contract to Sell of September 6, 1989, the Da Jose spouses paid
up to this writing, a breach of contract Genato the complete down payment of P950,000 and delivered to him three
c. That this affidavit is being executed to Annul the aforesaid Contract postdated checks (all dated May 6, 1990, the stipulated due date) in the total
to Sell for the vendee having committed a breach of contract for not amount of P1,865,680 to cover full payment of the balance of the agreed
having complied with the obligation as provided in the Contract to purchase price. However, due to the filing of the pendency of this case, the
Sell three postdated checks have not been encashed.
4. On October 24, 1989, Ricardo Cheng went to Genato’s residence and 12. Cheng instituted a complaint for specific performance to compel Genato to
expressed interest in buying the properties. Genato showed Cheng copies of execute a deed of sale to him of the subject properties plus damages and
his TCT and annotations at the back of his contract to sell with the Da Jose prayer for preliminary attachment. Cheng averred that the P50,000 check he
spouses. Genato also showed the Affidavit to Annul the Contract to Sell gave was a partial payment to the total agreed purchase price of the subject
which has not been annotated at the back of the titles. properties and considered as an earnest money for which Genato
5. Despite these, Cheng went ahead and issued a handwritten receipt for acceded. Thus, their contract was already perfected.
P50,000. 13. Genato alleged that the agreement was only a simple receipt of an option-bid
6. On October 25, 1989, Genato deposited Cheng’s check. Cheng also called up deposit, and never stated that it was a partial payment, nor is it an earnest
Genato reminding him to register the affidavit to annul contract to sell. money and that it was subject to the condition that the prior contract with the
7. on October 26, 1989, acting on Cheng’s request, Genato caused the Da Jose spouses be first cancelled. Also, Da Jose spouses asserted that they
registration of the Affidavit to Annul the Contract to Sell in the Registry of have a superior right to the property as first buyers. They alleged that the
Deeds, Meycauayan, Bulacan unilateral cancellation of the Contract to Sell was without effect and
8. While the Da Jose spouses were at the Office of the Registry of Deeds of void. They also cited Cheng’s bad faith as a buyer being duly informed by
Meycauayan, Bulacan on October 27, 1989, they met Genato by Genato of the existing annotated Contract to Sell on the titles.
coincidence. It was only then that the Da Jose spouses discovered about the 14. The lower Court ruled in favor of Cheng:
affidavit to annul their contract. The latter were shocked at the disclosure and a. Ruled that the receipt issued by Genato to Cheng meant a sale and
protested against the rescission of their contract. After being reminded that not just a priority or an option to buy.
Genato had given the Da Jose spouses an additional 30-day period to finish b. The transaction could not have been subject to condition or
their verification of his titles, that the period was still in effect, and that they reservation like the priority in favor of the Da Jose spouses as first
were willing and able to pay the balance of the agreed down payment, later buyer because if it were otherwise, the receipt would have provided
on in the day, Genato decided to continue the Contract he had with them. The such condition or reservation especially because Genato made the
agreement to continue with their contract was formalized in a conformed receipt in his own hand
letter dated October 27, 1989. c. There was valid recision of Contract to Sell by virtue of Affidavit to
9. Genato advised Cheng of his decision to continue his contract with the Da Annul the Contract to Sell. Time is of the essence in the execution
Jose spouses and the return of Chengs P50,000 check. On October 30, 1989, of the agreement between Genato and Cheng, under this
Cheng’s lawyer sent a letter to Genato demanding compliance with their circumstance demand, extrajudicial or judicial, is not necessary. It
agreement to sell the property to him stating that the contract to sell between falls under the exception to the rule provided in Article 1169 of the
him and Genato was already perfected and threatening legal action. Civil Code. The right of Genato to unilaterally rescind the contract
10. On November 2, 1989, Genato sent a letter to Cheng enclosing a BPI is said to be under Article 1191 of the Civil Code.
Cashier’s Check for P50,000 and expressed regret for his inability to 15. The Court a Quo reversed the decision:
consummate his transaction with him. After having received the letter of a. The prior contract to sell in favor of the Da Jose spouses was not
Genato on November 4, 1989, Cheng, however, returned the said check to validly rescinded, and that the subsequent contract to sell between
the former via RCPI telegram dated November 6, 1989, reiterating that our Genato and Cheng, embodied in the handwritten receipt, was
contract to sell your property had already been perfected. Cheng also without force and effect due to the failure to rescind the prior
contract;
b. That Cheng should pay damages to the respondents herein being 3. Even if the Da Jose spouses defaulted as claimed by Genato, in their Contract
found to be in bad faith to Sell, the execution by Genato of the affidavit to annul the contract is not
even called for. For with or without the aforesaid affidavit their non-payment
ISSUE: to complete the full downpayment of the purchase price avoids their contract
1. WoN CA is correct in ruling that there was no valid and effective rescission to sell, it being subjected to a suspensive condition.
of resolution of the Da Jose spouses Contract to Sell – YES 4. When a contract is subject to a suspensive condition, its birth or
2. Is the contract one of a Contract to Sell or one of a Conditional Contract of effectivity can take place only if and when the event which constitutes
Sale – Contract to Sell the condition happens or is fulfilled. If the suspensive condition does not
take place, the parties would stand as if the conditional obligation had
RULING: WHEREFORE, premises considered, the instant petition for review is never existed.
DENIED and the assailed decision is hereby AFFIRMED EN TOTO. SO ORDERED. 5. Nevertheless, Genato is not relieved from the giving of a notice, verbal or
written, to the Da Jose spouses for decision to rescind their contract.
RATIO: 6. Jurisprudence provides that even though we upheld the validity of a
1. In a Contract to Sell, the payment of the purchase price is a positive stipulation in a contract to sell authorizing automatic rescission for a violation
suspensive condition, the failure of which is not a breach, casual or of its terms and conditions, at least a written notice must be sent to the
serious, but a situation that prevents the obligation of the vendor to defaulter informing him of the same. The act of a party in treating a contract
convey title from acquiring an obligatory force. It is one where the as cancelled should be made known to the other. For such act is always
happening of the event gives rise to an obligation. Thus, for its non- provisional. It is always subject to scrutiny and review by the courts in case
fulfillment there will be no contract to speak of, the obligor having failed the alleged defaulter brings the matter to the proper courts.
to perform the suspensive condition which enforces a juridical relation. a. The party who deems the contract violated may consider it resolved
a. In fact with this circumstance, there can be no rescission of an or rescinded, and act accordingly, without previous court action, but
obligation that is still non-existent, the suspensive condition not it proceeds at its own risk. For it is only the final judgment of the
having occurred as yet. Emphasis should be made that the breach corresponding court that will conclusively and finally settle whether
contemplated in Article 1191 of the New Civil Code is the obligor’s the action taken was or was not correct in law. But the law definitely
failure to comply with an obligation already extant, not a failure of does not require that the contracting party who believes itself injured
a condition to render binding that obligation. must first file suit and wait for a judgment before taking extajudicial
2. No default can be ascribed to the Da Jose spouses since the 30-day extension steps to protect its interest. (UP v de Los Angeles)
period has not yet expired. The Da Jose spouses contention that no further 7. In the case at hand, Cheng’s contention that the Contract to Sell between
condition was agreed when they were granted the 30-days extension period Genato and the Da Jose spouses was rescinded or resolved due to Genato’s
from October 7, 1989 in connection with clause 3 of their contract to sell unilateral rescission finds no support in this case.
dated September 6, 1989 should be upheld for the following reason: 8. Even if we are to assume that the receipt, is to be treated as a conditional
a. If this were not true, Genato could not have been persuaded to contract of sale, it did not acquire any obligatory force since it was
continue his contract with them and later on agree to accept the full subject to suspensive condition that the earlier contract to sell between
settlement of the purchase price knowing fully well that he himself Genato and the Da Jose spouses should first be cancelled or rescinded; a
imposed such sine qua non condition in order for the extension to condition never met, as Genato, to his credit, upon realizing his error,
be valid; redeemed himself by respecting and maintaining his earlier contract
b. Genato could have immediately annotated his affidavit to annul the with the Da Jose spouses. In fact a careful reading of the receipt, alone
contract to sell on his title when it was executed on October 13, 1989 would not even show that a conditional contract of sale has been entered
and not only on October 26, 1989 after Cheng reminded him of the by Genato and Cheng. When the requisites of a valid contract of sale are
annotation; lacking in said receipt, therefore the sale is neither valid nor enforceable.
c. Genato could have sent at least a notice of such fact, there being no 9. Petitioner invokes Coronel v CA to support his theory that the transaction was
stipulation authorizing him for automatic rescission, so as to finally a conditional contract of sale. But the facts in Coronel is not on all fours with
clear the encumbrance of his titles and make it available to other those in the case at bar.
would be buyers
a. In Coronel, the petitioners therein clearly intended to transfer title b. In this light, we see no reason why we should not give priority in
to the buyer which petitioner themselves admitted in their right the annotation made by the Da Jose spouses with respect to
pleading. The agreement of the parties therein was definitively their Contract to Sell dated September 6, 1989.
outline in the Receipt of Down Payment. In the instant case, even by 13. Good faith must concur with registration for such prior right to be
a careful perusal of the receipt alone such, kind of circumstances enforceable. In the instant case, the annotation made by the Da Jose spouses
cannot be ascertained without resorting to the exceptions of the Rule on the titles of Genato of their Contract to Sell more than satisfies this
on Parol Evidence. requirement. Whereas in the case of Genatos agreement with Cheng such is
10. The governing principle of Article 1544 of the New Civil Code16 should unavailing. For even before the receipt was issued to Cheng information of
apply in this situation. Jurisprudence teaches us that the governing such pre-existing agreement has been brought to his knowledge which did
principle is PRIMUS TEMPORE, PORTIOR JURE (first in time, not deter him from pursuing his agreement with Genato.
stronger in right). 14. Cheng himself admitted that it was he who sought Genato in order to inquire
a. For not only was the contract between herein respondents first about the property and offered to buy the same. Since Cheng was fully aware,
in time; it was also registered long before petitioners intrusion or could have been if he had chosen to inquire, of the rights of the Da Jose
as a second buyer. spouses under the Contract to Sell duly annotated on the transfer certificates
b. This principle only applies when the special rules provided in of titles of Genato, he is indeed in bad faith in entering into such agreement.
the aforecited article of Civil Code do not apply or fit the specific 15. Good faith, or lack of it, is in its last analysis a question of intention; but in
circumstances mandated under said law or by jurisprudence ascertaining the intention by which one is actuated on a given occasion, we
interpreting the article. are controlled by the evidence as to the conduct and outward acts by which
11. In the case at bar, the knowledge gained by the Da Jose spouses, as first the inward motive may, with safety, be determined. So it is that the honesty
buyers, of the new agreement between Cheng and Genato will not defeat their of intention, the honest lawful intent, which constitutes good faith implies a
rights as first buyers except where Cheng, as second buyer, registers or freedom from knowledge and circumstances which ought to put a person on
annotates his transaction or agreement on the title of the subject properties in inquiry, and so it is that proof of such knowledge overcomes the presumption
good faith ahead of the Da Jose spouses. Although the Da Jose spouses, as of good faith in which the courts always indulge in the absence of the proof
first buyers, knew of the second transaction it will not bar them from availing to the contrary. (Leung Yee v F.L. Machinery Strong Co
of their rights granted by law, among them, to register first their agreement
as against the second buyer. In contrast, knowledge gained by Cheng of the
first transaction between the Da Jose spouses and Genato defeats his rights
even if he is first to register the second transaction, since such knowledge
taints his prior registration with bad faith.
12. Registration, as defined by Soler and Castillo, means any entry made in the
books of the registry, including both registration in its ordinary and strict
sense and cancellation, annotation, and even marginal notes. In its strict
acceptation, it is the entry made in the registry which records solemnly and
permanently the right of ownership and other real rights.
a. When a Deed of Sale is inscribed in the registry of property on the
original document itself, what was done with respect to said entries
or annotations and marginal notes amounted to a registration of the
sale.

16
Article 1544. If the same thing should have been sold to different vendees, the ownership shall be Should there be no inscription, the ownership shall pertain to the person who in good faith was first in the
transferred to the person who may have first taken possession thereof in good faith, if it should be possession; and, in the absence thereof, to the person who presents the oldest title, provided there is good
movable property. faith. (1473)
Should it be immovable property, the ownership shall belong to the person acquiring it who in good faith
first recorded it in the Registry of Property.
21 SAN LORENZO DEV. CORP. v. CA (Ella) FACTS:
January 21, 2005 | Tinga J. | Contract to Sell 8. respondents Miguel Lu and Pacita Zavalla, (hereinafter, the Spouses Lu)
owned two (2) parcels of land situated in Sta. Rosa, Laguna.
PETITIONER: San Lorenzo Development Corp. 9. On 20 August 1986, the Spouses Lu purportedly sold the two parcels of land
RESPONDENTS: Court of Appeals, Pablo Babasanta, Spouses Miguel & Pacita to respondent Pablo Babasanta, (hereinafter, Babasanta) for the price of
Lu P15.00 per square meter. Babasanta made a downpayment of P50,000.00 as
evidenced by a memorandum receipt issued by Pacita Lu of the same date.
Several other payments totaling P200,000.00 were made by Babasanta.
SUMMARY: Spouses Lu owned 2 parcels of land. They purportedly sold these
lands to Babasanta. Babasanta paid downpayment of P50,000, and several other 10. Sometime in May 1989, Babasanta wrote a letter to Pacita Lu to demand the
payments totaled P200,000. In 1989, Babasant a wrote a letter to Spouses Lu execution of a final deed of sale in his favor so that he could effect full
demanding that a deed of sale be executed in his favor so that he could effect full payment of the purchase price. In the same letter, Babasanta notified the
payment of the price. He also notified the spouses that he received information spouses about having received information that the spouses sold the same
that the latter have sold the land to another w/o his knowledge and consent. property to another without his knowledge and consent. He demanded
Babasanas is demanding that the 2nd sale be cancelled. In their reply, spouses Lu that the second sale be cancelled and that a final deed of sale be issued in his
said that they have indeed agreed to sell the land to Babasanas. But spouses Lu favor.
reminded him that when his request for the reduction of the purchase has been 11. In response, Pacita Lu wrote a letter to Babasanta wherein she acknowledged
refused by the spouses, Babasanas backed out. That their relationship had having agreed to sell the property to him at P15.00 per square meter. She,
originally been creditor (Babasanas) & debtor (Lu), when his request was refused, however, reminded Babasanta that when the balance of the purchase price
the Babasanas rescinded the contract to sell and that the original loan transaction became due, he requested for a reduction of the price and when she refused,
be carried out. Babasanas filed an action against the spouses for specific Babasanta backed out of the sale. Pacita added that she returned P50,000.00
performance. Meanwhile, petitioner San Lorenzo Dev’t. Corp (SLDC), filed a to Babasanta through Eugenio Oya.
motion for intervention. It alleged that the subject property had been sold to it
through a deed of sale with absolute mortgage. It allege that it was a buyer in good 12. So respondent Babasanta, as plaintiff, filed before the Regional Trial Court
faith. Issue is WON SLDC has a better right over the property than Babasanas. SC (RTC), Branch 31, of San Pedro, Laguna, a Complaint for Specific
held that SLDC has a better right. First, contract between Babasanas and Spouses Performance and Damages 1 against his co-respondents herein, the Spouses
Lu was actually a perfected contract to sell not a contract of sale. Spouses Lu never Lu.
intended to transfer ownership to Babasana except upon full payment of the price. 13. The Spouses Lu alleged that Pacita Lu obtained loans from Babasanta and
Babasanta himself recognized that ownership of the property would not be when the total advances of Pacita reached P50,000.00, the latter and
transferred to him until such time as he shall have effected full payment of the Babasanta, without the knowledge and consent of Miguel Lu, had verbally
price. Thus, the receipt issued by the spouses recognizing Babasanas’ partial agreed to transform the transaction into a contract to sell the two parcels of
payments should legally be considered as a perfected contract to sell. The land to Babasanta with the fifty thousand pesos P50,000.00 to be considered
perfected contract to sell imposed upon Babasanta the obligation to pay the as the downpayment for the property and the balance to be paid on or before
balance of the purchase price. But he never did make the proper tender of payment 31 December 1987.
& consignation required by law. Thus, the obligation on the part of the sellers to 14. Respondents Lu added that as of November 1987, total payments made by
convey title never acquired obligatory force. Babasanta amounted to only P200,000.00 and the latter allegedly failed to
pay the balance of P260,000.00 despite demands.
15. Babasanta had purportedly asked Pacita for a reduction of the price from
DOCTRINE: The main distinctions that set apart the Contract to Sell (as opposed
P15.00 to P12.00 per square meter and when the Spouses Lu refused to grant
to a contract of sale) are 1) By agreement the ownership is reserved in the vendor
Babasanta's request, the latter rescinded the contract to sell and declared that
and is not to pass until the full payment of the price. 2) Title is retained by the
the original loan transaction just be carried out in that the spouses would be
vendor until the full payment of the price, such payment being a positive
indebted to him in the amount of P200,000.00
suspensive condition and failure of which is not a breach but an event that prevents
16. On 19 January 1990, herein petitioner San Lorenzo Development
the obligation of the vendor to convey title from becoming effective.
Corporation (SLDC) filed a Motion for Intervention before the trial court.
SLDC alleged that it had legal interest in the subject matter under litigation
because on 3 May 1989, the two parcels of land involved had been sold to it ownership until and unless the contract full payment of the price, such payment
in a Deed of Absolute Sale with Mortgage. It alleged that it was a buyer in is resolved or rescinded being a positive suspensive condition
good faith and for value and therefore it had a better right over the property and failure of which is not a breach but
in litigation. an event that prevents the obligation of
the vendor to convey title from
ISSUE/s: becoming effective.
38. WON SLDC has a better right than Babasanta over the subject parcel of lands
in view of the successive transactions executed by Spouses Lu. = SLDC has
better right.
14. The perfected contract to sell imposed upon Babasanta the obligation to pay
RULING: WHEREFORE, the instant petition is hereby GRANTED. The decision of the balance of the purchase price.
the Court of Appeals appealed from is REVERSED and SET ASIDE and the decision 15. There being an obligation to pay the price, Babasanta should have made the
of the Regional Trial Court, Branch 31, of San Pedro, Laguna is REINSTATED. No proper tender of payment and consignation of the price in court as required
costs. by law. Mere sending of a letter by the vendee expressing the intention to pay
without the accompanying payment is not considered a valid tender of
payment.
RATIO:
16. Babasanta did not even attempted to make the proper consignation of the
amounts due, thus, the obligation on the part of the sellers to convey title
10. The agreement between Babasanta and the Spouses Lu is not a contract of never acquired obligatory force.
sale, but a contract to sell.
11. The receipt signed by Pacita Lu merely states that she accepted the sum of 17. Furthermore there is no delivery (actual or constructive) of the subject matter
fifty thousand pesos (P50,000.00) from Babasanta as partial payment. While of the sale in favor of Babasanta which is essential to transfer of ownersip of
there is no stipulation that the seller reserves the ownership of the property property. Thus, respondent Babasanta did not acquire ownership by the mere
until full payment of the price which is a distinguishing feature of a contract execution of the receipt by Pacita Lu acknowledging receipt of partial
to sell, the subsequent acts of the parties shows that the Spouses Lu never payment for the property. Reasons: 1) the agreement between Babasanta and
intended to transfer ownership to Babasanta except upon full payment of the the Spouses Lu, though valid, was not embodied in a public instrument.
purchase price. Hence, no constructive delivery of the lands could have been effected. 2)
Babasanta had not taken possession of the property at any time after the
12. Babasanta's letter stated therein that despite his repeated requests for the perfection of the sale in his favor or exercised acts of dominion over it despite
execution of the final deed of sale in his favor so that he could effect full his assertions that he was the rightful owner of the lands.
payment of the price. In effect, Babasanta himself recognized that ownership 18. Judicial relationship between parties in case of double sale, principle
of the property would not be transferred to him until such time as he shall of primus tempore, potior jure (first in time, stronger in right) gains
have effected full payment of the price. greater significance in case of double sale of immovable property.
13. Moreover, had the sellers intended to transfer title, they could have easily 19. When the thing sold twice is an immovable, the one who acquires it and first
executed the document of sale in its required form simultaneously with their records it in the Registry of Property, both made in good faith, shall be
acceptance of the partial payment, but they did not. Doubtlessly, the receipt deemed the owner.
signed by Pacita Lu should legally be considered as a perfected contract to 20. SLDC registered the sale with the Registry of Deeds after it had acquired
sell. knowledge of Babasanta's claim. SLDC qualifies as a buyer in good faith
since there is no evidence extant in the records that it had knowledge of the
CONTRACT OF SALE CONTRACT TO SELL prior transaction in favor of Babasanta. At the time of the sale of the property
Title passes to the vendee upon the By agreement the ownership is reserved to SLDC, the vendors were still the registered owners of the property and
delivery of the thing sold in the vendor and is not to pass until the were in fact in possession of the lands. (Notice of lis pendens is irrelevant to
full payment of the price the good faith & bad faith character of the purchaser).
The vendor has lost and cannot recover Title is retained by the vendor until the
22 Adalin v. CA (EMAR) there be no inscription, the ownership shall pertain to the person who in good faith
October 10, 1997| HERMOSISIMA, JR., J.| Conditional Sales/Contracts to Sell and was first in the possession; and, in the absence thereof, to the person who presents
Adverse Claims the oldest title, provided there is good faith.
PETITIONER: CARLOS CALINGASAN, DEMETRIO ADAYA and MAGNO
ADALIN (lessees) LORETO ADALIN (Adalin) FACTS:
RESPONDENTS: THE HON. COURT OF APPEALS, FAUSTINO L. YU, 1. Kado siblings (vendors), were the owners of a parcel of land with a 5-door,
ANTONIO T. LIM, (Original Buyers) ELENA K. PALANCA, JOSE PALANCA, building constructed thereon, fronting the Imperial Hotel
EDUARDA K. VARGAS, JOSE VARGAS, MERCEDES K. CABALLERO, 2. Lessees Calingasan (1 door), Adaya (1) Magno (2) and Adalin (1) paid
EBERHARDO CABALLERO, ISABEL K. VILLAMOR, FEDERICO P1.5k/door/month.
VILLAMOR, JOSE KADO, URSULA KADO, MARIA K. CALONZO, BAYANI 3. Vendors commissioned Bautista to look for and negotiate with prospective
L. CALONZO, TEOFILA KADO, NESTOR KADO and LILIA KADO (Kado buyers for the sale of their property for the price of P3m.
siblings/vendors) 4. Bautista offered the property for sale, to the owners of the Imperial Hotel, Yu
SUMMARY: Vendors commissioned Bautista to sell their 5-door property in front and Lim (original buyers) and each agreed to buy 2 doors and Adalin will
of Imperial Hotel. Bautista offered to sell to Yu and Lim (owners of Imperial Hotel, buy 1 door, all for the price of P2.6m.
original buyers) who agreed to buy together with Adalin. The vendors, original 5. The original buyers inquired from vendor Elena if the lessees were interested
buyers and Adalin had a conference wherein the vendor said to the original buyers to buy the property and she said no.
that the other tenants were not interested to buy the property. They met again to 6. Sept 2 1987: Unknown to the original buyers, Elena sent letters to each of the
finalize their sale, this time, lessee Magno was present and he said that they weren’t lessees informing them that someone was interested to buy the property and
interested to buy the property because they can’t afford it but he said that each lessee requested them to vacate the property within 30d unless all of them could
was entitled to a disturbance fee of P50k. They executed a “Deed of Conditional buy the property at the same price.
Sale” where vendors, original buyers and Adalin agreed that the buyers will pay 7. Sept 2 1987 Conference: original buyers, Adalin and vendors, Atty. Calonzo,
P300k as downpayment and as soon as the former secures the eviction of the tenants, the husband of vendor Maria, Atty. Soyao, counsel of the original buyers and
they will be paid the balance of P2.3m. Vendor Elena sent letters to each of the lessee Magno, in behalf of the lessees, were present.
lessees informing them that someone was interested to buy the property and 8. When asked by the original buyers if the lessees were interested to buy the
requested them to vacate the property within 30d unless all of them could buy the property, Magno replied that they’re not, because they can’t afford its
property at the same price. The vendors filed a letter complaint for unlawful detainer price but he claimed that the lessees were entitled to P50k each as disturbance
because the lessees refused to vacate. The lessees sent letters informing the vendors money, in consideration for their vacating the property, to be borne by the
that they wanted to purchase the doors that they were leasing for the purchase price vendors.
of P600k/door. Vendor Elena accepted the offer and returned the downpayment of 9. Original buyers, Adalin and the vendors agreed that each of the original
the original buyers but they refused to accept. The issue is whether the transaction buyers will buy 2 doors while the 5th door leased by Adalin will be purchased
entered into by the original buyers with the vendors is an absolute sale or conditional by him, all for the purchased price of P2.6m and that the original buyers and
sale as denominated in the document signed by said parties. The lower court ruled Adalin will pay P300k as downpayment for the property, the balance to be
that it was a conditional sale wherein the condition wasn’t fulfilled but the CA payable upon the eviction of the lessees from the property and the execution
reversed the ruling and decided that it was a perfect deed of absolute sale. The SC of a Deed of Absolute Sale.
agreed with the CA using Article 1544 of the Civil Code which states that “if the 10. Atty. Calonzo assured the original buyers that he could secure the eviction of
same thing should have been sold to different lessees, the ownership shall pertain to the lessees from the property within a month because they were close.
the person who in good faith was first in the possession; and, in the absence thereof, 11. Original buyers and Adalin and the vendors, met in the office of the Appellant
to the person who presents the oldest title, provided there is good faith.” The Lessees Yu at the Imperial Hotel and executed the Deed of Conditional Sale (DCS)
are estopped from claiming that the property had not been delivered to the original prepared by Atty. Soyao.
buyers. They cannot use their gross bad faith as a shield to frustrate the enforcement 12. The original buyers and Adalin each contributed P100k and gave the total
of the Deed of Conditional Sale. They were in bad faith for buying the property amount of P300k to Elena as the downpayment for the property for which the
although knowing that there was a previous sale of the same property. vendors signed an Acknowledgment Receipt.
DOCTRINE: If the same thing should have been sold to different vendees, should
13. Original buyers deferred registration of the deed until after the eviction 26. Atty. Calonzo stated that there was no need to show the deed of sale. No
of the lessees and the payment of the balance of the purchase price of the settlement was forged and the Barangay Captain issued the Certification
property to the vendors as agreed upon under the DCS. to File Action.
14. The vendors wrote a letter complaint against the lessees with the Barangay RTC
Captain for unlawful detainer. It’s a conditional sale wherein the condition wasn’t fulfilled.
15. Lessees Magno, Demetrio and Calingasan wrote a letter to the vendors, 27. The original buyers filed their complaint for Specific Performance against
informing their intent to purchase the doors they were leasing for the the vendors and Adalin in the court.
purchase price of P600k/door. 28. The original buyers caused the annotation of a Notice of Lis Pendens at the
16. Inspite of the prior sale of the property to the original buyers and Adalin, the dorsal portion of TCT under the names of the lessees.
vendors decided to back out from said sale to the original buyers and to 29. The lessees filed a Motion for Intervention, adding a copy of the Deed of Sale
sell the property to the lessees and to return the P200k downpayments of of Registered land signed by the vendors.
the original buyers with interest, procuring 2 checks, both worth 30. Lessees filed the Deed of Sale of Registered Land with the Register of Deeds
P101,416.66 payable to original buyers Yu and Lim and transmitted the same on the basis of which TCT No. 24791 over the property was issued under
to the Original buyers with a covering letter. The original buyers were their names. On the same day, the lessees filed in the Court a Motion To
shocked and refused to receive the said letter and checks and insisted, Admit Complaint-In-Intervention. Attached to the Complaint-In-
instead, that the vendors comply with the DCS. Intervention was the 'Deed of Sale of Registered land signed by the lessees.
17. Original buyers wrote a letter to the vendors, and copies were sent to lessees, 31. Original buyers were shocked to learn that the vendors had signed the said
inquiring if the appropriate action has been undertaken towards the eviction deed. As a counter-move, the Appellants filed a motion for leave to amend
of the lessees. Complaint and filed their Amended Complaint adding the lessees as
18. Vendors ignored the letter and instead, signed a Deed of Sale of Registered defendants.
Land under which they sold the said property to the lessees, including Adalin 32. RTC dismissed the Petition of the vendors for consignation.
for the price of only P1m (much lower than the price of the Appellant under 33. Adalin died and was substituted by his heirs.
the DCS). 34. After trial, the Court rendered judgment in favor of the lessees.
19. It appears that the deed was notarized by Atty. Calonzo, however, it does not 35. In the opinion of the court a quo, the lessees became the owners of the parcel
bear any number in the notarial register of the lawyer. of land in question with the 5-door, 1 storey commercial building standing
20. Vendors signed another Deed of Sale of Registered Land under which they thereon, when they purchased the same following the offer and the 30-
sold to the lessees and Adalin the property for the considerably increased day option extended to them by Elena in her letter to them.
price of P3m. The deed was notarized by Atty. Calonzo. However, both 36. The trial court disregarded the fact that the vendor siblings had already
deeds were not filed with the Register of Deeds. finished transacting with original buyers and had in fact entered into a
21. The vendors, signed a third Deed of Sale of Registered land under which they conditional sale with them respecting the same property.
purportedly sold to the lessees and Adalin, the aforesaid property for the 37. The trial court brushed aside this fact as it reasoned that:
much reduced price of only P860k. However, the deed was not immediately 1. In conditional deed of sale, ownership is only transferred after
filed with the Register of Deed. 1. purchase price is fully paid or the fulfillment of the
22. The vendors filed before the RTC a petition against the original buyers for condition and
the consignation of their downpayment of P200k 2. execution of a definite or absolute deed of sale
23. Original buyers filed a complaint with the Barangay captain for Breach of 2. It states in the DCS that the balance of the price of P2.3m shall be
Contract against the vendors. paid only after all the defendants-lessees shall have vacated and
24. The Barangay Captain issued summons to the vendors for them to appear for surrendered the premises to the vendors but the tenants did not
a conference. Invitations were also sent to the lessees. During the conference leave the premises. In fact they opted to buy the property.
the original buyers agreed to buy only 1 door each so that lessee Magno could Moreover, at that time, the property was legally leased to the
purchase only 1 of the 2 doors he was occupying. However, Magno refused, lessees.
claiming that he was already the owner of the 2 doors. 3. The condition set forth in the said DCS between the original buyers
25. When original buyer Lim asked Magno to show the Deed of Sale for the two and vendors was not fulfilled and therefore, there was no transfer
doors, Magno walked out. of ownership of the property from the vendors to the buyers.
4. The letters of Elena to the lessees gave them the 41. A contract of sale sale may be absolute or conditional. (Art. 1458 par.2, CC)
option/preferential right to purchase the property. A contract may be conditional when the ownership of the thing sold is
5. When the vendors accepted lessees’ option to buy, the vendors retained until the fulfillment of a positive suspensive condition, generally
returned the initial payment of P200k to the original buyers but they the payment of the purchase price, the breach of which condition will
refused to accept it. The refusal however did not diminish the prevent the onset of the obligation to deliver title.
effect of the acceptance of the option to buy, which in fact led to 42. A sale of immovables is absolute where the contract does not contain any
the execution of the said Deed of Sale of Registered Land and the provision that title to the property sold is reversed to the Vendors or that
subsequent issuance of the TCT in the names of the lesees. the Vendor is entitled to unilaterally rescind the same.
6. The vendors acted in bad faith when, while during the effectivity 43. The Court believes that the deed is conditional, its efficacy is dependent
of the period of the option to buy they gave to the lessees, they upon a suspensive condition--that of the payment by the original buyers
executed a DCS in favor of the original buyers. This was only 6 of the purchase price of the property, after the lessees shall have vacated
days from date of the option. the same and the Deed of Absolute Sale shall have been executed in favor
7. The conditional sale of the subject property to original buyers and of the original buyers; and, since the condition was not fulfilled, the sale
the sale of the same property to petitioners, did not involve a double never became effective.
sale to warrant the application of Art. 154417 of the Civil Code 44. A reading of the deed will readily show absence of any stipulation that the
1. The plaintiffs assert that this case is one of double sale (1st title to the property was reserved to the Vendors until the balance of the
sale to Lim and Yu; 2nd sale to the lessees) and should be purchase price was paid nor giving them the right to unilaterally rescind
governed by Art. 1544 of the CC. the contract if the original buyers failed to pay the said amount upon the
2. The execution of the DCS did not transfer ownership to eviction of the lessees.
the original buyers, hence, there can be no double sale. 45. Even if there was no delivery of the property to the original buyers, the
3. Mendoza vs. Kalaw: Article 1544 does not apply to vendors may have delayed their delivery of physical possession of the
situations where one sale was subject to a condition property to the original buyers only after the lessees shall have vacated the
which was not complied with. This is because a property, however, the right of retention of the vendors of title to or
conditional sale, before the performance of the ownership over the property cannot thereby be inferred.
condition, can hardly be said to be a sale of property, 46. Non-payment of the balance & the consequent eviction of the lessees were
specially where the condition has not been performed not conditions which suspended the efficacy of the Deed of Conditional
or complied with. Sale. Rather, if due to the fault of the original buyers, merely accorded the
CA vendors the option to rescind the already existing and effective sale.
It’s a perfect deed of absolute sale. 47. Because the original buyers and vendors entered into an absolute sale, the
38. The original buyers appealed to the CA which ruled in their favor and original buyers had every right to demand that the vendors performed their
declared that the DCS executed by the vendors in favor of the original buyers prestation under the deed (eviction of the lessees) so that the Appellants may
was an absolute deed of sale & not a conditional sale. then pay the balance of the purchase price of the property
39. It looked into the context of the contract in its entirety and not merely 48. The contention of the lower court that “the sale had not been consummated
specific words or phrases therein, standing alone, as well as the and title to and ownership over the property had not been transferred to the
contemporaneous and subsequent acts of the parties. It bears stressing that original buyers because there had been neither constructive nor actual
the title of the contract is not conclusive of its nature. delivery of the property” cannot be true because evidence in the record
40. A contract may be denominated a DCS/Agreement to Sell but in reality a shows that the original buyers and vendors met in the house of Appellee Elena
deed of absolute sale or a contract of sale. shows that lessee Magno did not object to the sale of the property to the

17
Art. 1544. If the same thing should have been sold to different absence thereof, to the person who presents the oldest title, provided
vendees, should there be no inscription, the ownership shall pertain to there is good faith.
the person who in good faith was first in the possession; and, in the
original buyers but merely insisted that each of the lessee be given P50k as the property to the Appellants.
disturbance fee by the vendors to which the latter acquiesced 2. It would be the peak of inequity to allow the vendors to invoke the occupation by
49. Moreover, when the vendors wrote letters to the lessees giving them the the lessees, as of the property, as a justification to ignore their obligation to have them
option to match the price offered by the Appellants, they didn’t respond. It evicted and for them to give P50k as disturbance fee for each of the lessees and a
was only after the execution of the Deed of Conditional Sale, that the lessees justification for the latter to hold on to the possession of the property.
finally decided to purchase the property. 3. Non-consummation of the DCS because of non-delivery to of the property does not
50. Lessees are estopped from claiming that the property had not been delivered mean that the Deed of Sale of Registered Land executed by the lessees and vendors
to the original buyers. They cannot use their gross bad faith as a shield to should be given preference. The lower court erred in not applying the second and third
frustrate the enforcement of the Deed of Conditional Sale. paragraphs of Article 154418.
51. The vendors cannot invoke the refusal of the lessees to vacate the property 4. Evidence shows that the lessees were able to register the Deed of Sale of Registered
and their decision to themselves purchase the property as a valid justification Land and procure TCT under their names, and that they were, as of said date, in
to renege on and turn their backs against their obligation to deliver/cause the physical possession of the property, however, the evidence in the record shows that
eviction. the lessees were in gross evident bad faith because at the time they executed their
52. CA declared the sale to the original buyers valid and the sale to the lessees sale (December 1987), they were aware that the vendors and the original buyers
void and ordered: had executed their DCS (September 1987)
1. Original buyers to pay the balance of the purchase price (P1.88m) 5. The SC reiterated the reasons of the CA.
2. Lessees
1. vacate within 30d from the finality of this Decision
2. pay P1.5k/month per door to original buyers, from
November, 1987, up to the time the property is vacated
3.
3. Vendors
1. execute a Deed of Absolute Sale in favor of the original
buyers
2. to refund to the lessees the amount of P840k which they
paid for the property without interest bec. bad faith
4. pay to each of the original buyers the amount of P120k for damages
and P20k for attys fee

ISSUE: WoN the transaction entered into by private respondents Yu and Lim with
private respondents Kado siblings, is one of an absolute sale and not merely a
conditional sale as denominated in the document signed by said parties –DEED OF
ABSOLUTE SALE.

RULING: WHEREFORE, the instant petition is HEREBY DISMISSED.

RATIO:
1. Vendors cannot invoke the refusal of the lessees to vacate the property and the latters
decision to themselves purchase the property as a valid justification to not do their
obligation to cause the eviction of the lessees from and deliver physical possession of

18
If the same thing should have been sold to different vendees, should good faith was first in the possession; and, in the absence thereof, to the
there be no inscription, the ownership shall pertain to the person who in person who presents the oldest title, provided there is good faith.
23 CHENG v. GENATO (Therese)
December 29, 1998| Martinez J. | Contract to Sell
FACTS:
PETITIONER: Ricardo Cheng 1. Respondent Ramon B. Genato(Genato) is the owner of two parcels of land
RESPONDENTS: Ramon Genato and Ernesto R. Da Jose and Socorro B. Da Jose located at Paradise Farms, San Jose Del Monte, Bulacan with an aggregate
area of 35,821 square meters, more or less.
SUMMARY: Genato was the owner of two parcels of land and he entered into a 2. Respondent Genato entered into an agreement with respondent-spouses
contract to sell with spouses Da Jose. In their agreement, the vendee is given 30 days Ernesto R. Da Jose and Socorro B. Da Jose (Da Jose spouses) over the above-
after the execution of the said contract to verify the authenticity of documents and mentioned two parcels of land.
that no restrictions, limitations, and developments imposed on and/or affecting the a. The agreement culminated in the execution of a contract to sell for
subject property and after which they should pay the price. Pending the period given which the purchase price was P80.00 per square meter.
to the spouses, Genato entered into an agreement with Cheng concerning the same b. The contract was in a public instrument and was duly annotated at
land. Cheng knew about the prior agreement but nonetheless pursued the transaction the back of the two certificates of title on the same day.
holding on to Genato’s claim that the contract to sell with the spouses will be c. Clauses 1 and 3 thereof provide:
annulled. Then the spouses knew about this then Genato decided to pursue the i. That the purchase price shall be (P80.00) per square
contract to sell with the spouses, to which Cheng of course, objected to. Court said meter, of which the amount of (P50,000.00) shall be paid
that In a Contract to Sell, the payment of the purchase price is a positive suspensive by the VENDEE to the VENDOR as partial down
condition, the failure of which is not a breach, casual or serious, but a situation that payment at the time of execution of this Contract to Sell.
prevents the obligation of the vendor to convey title from acquiring an obligatory ii. That the VENDEE, thirty (30) DAYS after the execution
force. In this case, the spouses had not yet complied with their obligation to pay and of this contract, and only after having satisfactorily
the period given to them for which to pay has not yet expired. Furthermore, genato verified and confirmed the truth and authenticity of
did not give the spouses notice that their contract had been rescinded. Thus, there documents, and that no restrictions, limitations, and
was no valid cancellation of the contract to sell. As to the nature of the agreement developments imposed on and/or affecting the property
between genato and Cheng, even if it was a conditional contract of sale, it will still subject of this contract shall be detrimental to his
have no obligatory force as the contract with the spouses was still subsisting. Court interest, the VENDEE shall pay to the VENDOR,
also applied Article 1544, wherein it states that if the same thing should have been (P950,000.00) representing the full payment of the
sold to different vendees, Should it be immovable property, the ownership shall agreed Down Payment, after which complete possession
belong to the person acquiring it who in good faith first recorded it in the of the property shall be given to the VENDEE to enable
Registry of Property. Should there be no inscription, the ownership shall him to prepare the premises and any development
pertain to the person who in good faith was first in possession; and in the therein.
absence thereof, to the person who presents the oldest title, provided there is 3. Da Jose spouses, not having finished verifying the titles mentioned asked for
good faith. In this case, Cheng can never be a good faith buyer as he had knowledge and was granted by respondent Genato an extension of another 30 days.
of the transaction between Genato and the Spouses Da Jose. a. However, according to Genato, the extension was granted on
condition that a new set of documents is made seven (7) days from
DOCTRINE: In rescinding a contract, there must be a written notice given to the October 4, 1989.
parties and in case that if the same thing should have been sold to different vendees, b. This was denied by the Da Jose spouses.
Should it be immovable property, the ownership shall belong to the person acquiring 4. Pending the effectivity of the aforesaid extension period, and without due
it who in good faith first recorded it in the Registry of Property. Should there be no notice to the Da Jose spouses, Genato executed an Affidavit to Annul the
inscription, the ownership shall pertain to the person who in good faith was first in Contract to Sell, on October 13, 1989.
possession; and in the absence thereof, to the person who presents the oldest title, a. Moreover, no annotation of the said affidavit at the back of his titles
provided there is good faith. was made right away.
b. The affidavit contained, inter alia, the following paragraphs;
i. That it was agreed between the parties that the agreed a. Chengs lawyer sent a letter to Genato demanding compliance with
downpayment of P950,000.00 shall be paid thirty (30) their agreement to sell the property to him stating that the contract
days after the execution of the Contract, that is on or to sell between him and Genato was already perfected and
before October 6, 1989; threatening legal action.
ii. The supposed VENDEES failed to pay the said full b. Genato returned the P50,000.00 and expressed regret for his
downpayment even up to this writing, a breach of inability to consummate his transaction with him.
contract. c. Cheng, however, returned the said check to the former via RCPI
iii. That this affidavit is being executed to Annul the telegram reiterating that their contract to sell your property had
aforesaid Contract to Sell for the vendee having already been perfected.
committed a breach of contract for not having complied 9. Cheng executed an affidavit of adverse claim and had it annotated on the
with the obligation as provided in the Contract to Sell subject TCTs.
5. Petitioner Ricardo Cheng (Cheng) went to Genatos residence and expressed a. On the same day, consistent with the decision of Genato and the Da
interest in buying the subject properties. Jose spouses to continue with their Contract to Sell, the Da Jose
a. On that occasion, Genato showed to Ricardo Cheng copies of his spouses paid Genato the complete down payment of P950,000.00
transfer certificates of title and the annotations at the back thereof of and delivered to him three (3) postdated in the total amount
his contract to sell with the Da Jose spouses. of P1,865,680.00 to cover full payment of the balance of the agreed
b. Genato also showed him the aforementioned Affidavit to Annul the purchase price.
Contract to Sell which has not been annotated at the back of the b. However, due to the filing of the pendency of this case, the three (3)
titles. postdated checks have not been encashed.
c. Despite these, Cheng went ahead and issued a check for P50,000.00 10. Cheng instituted a complaint for specific performance to compel Genato to
upon the assurance by Genato that the previous contract with the Da execute a deed of sale to him of the subject properties plus damages and
Jose spouses will be annulled for which Genato issued a handwritten prayer for preliminary attachment.
receipt a. In his complaint, Cheng averred that the P50,000.00 check he gave
d. Genato deposited Chengs check. was a partial payment to the total agreed purchase price of the
6. On the same day, Cheng called up Genato reminding him to register the subject properties and considered as an earnest money for which
affidavit to annul the contract to sell. Genato acceded.
a. Acting on Chengs request, Genato caused the registration of the b. Thus, their contract was already perfected.
Affidavit to Annul the Contract to Sell in the Registry of Deeds, 11. Genato alleged that the agreement was only a simple receipt of an option-bid
Meycauayan, Bulacan deposit, and never stated that it was a partial payment, nor is it an earnest
7. While the Da Jose spouses were at the Office of the Registry of Deeds of money and that it was subject to the condition that the prior contract with the
Meycauayan, Bulacan they met Genato by coincidence. Da Jose spouses be first cancelled.
a. It was only then that the Da Jose spouses discovered about the 12. The Da Jose spouses asserted that they have a superior right to the property
affidavit to annul their contract. as first buyers.
b. The latter were shocked at the disclosure and protested against the a. They alleged that the unilateral cancellation of the Contract to Sell
rescission of their contract. was without effect and void.
c. After being reminded that he (Genato) had given them (Da Jose b. They also cited Chengs bad faith as a buyer being duly informed by
spouses) an additional 30-day period to finish their verification of Genato of the existing annotated Contract to Sell on the titles.
his titles, that the period was still in effect, and that they were willing 13. Trial court: contract between Genato and Da Jose spouses rescinded in
and able to pay the balance of the agreed down payment, later on in accordance with Article 1191.
the day, Genato decided to continue the Contract he had with them. 14. CA: reversed the judgment and ruled that the prior contract to sell in favor of
8. Thereafter, Ramon Genato advised Ricardo Cheng of his decision to continue the Da Jose spouses was not validly rescinded, that the subsequent contract
his contract with the Da Jose spouses and the return of Chengs P50,000.00 to sell between Genato and Cheng, embodied in the handwritten receipt, was
check. without force and effect due to the failure to rescind the prior contract; and
that Cheng should pay damages to the respondents herein being found to be was executed on October 13, 1989 and not only on
in bad faith. October 26, 1989 after Cheng reminded him of the
15. Hence this petition annotation;
iii. thirdly, Genato could have sent at least a notice of such
ISSUES: Whether or not the agreement between Genato and the Da Jose was fact, there being no stipulation authorizing him for
validly rescinded so as to entitle Cheng to the sale of the subject properties automatic rescission, so as to finally clear the
RULING: NO. encumbrance of his titles and make it available to other
would be buyers. It likewise settles the holding of the
WHEREFORE, premises considered, the instant petition for review is DENIED trial court that Genato needed money urgently.
and the assailed decision is hereby AFFIRMED EN TOTO. 19. Even assuming in gratia argumenti that the Da Jose spouses defaulted, as
claimed by Genato, in their Contract to Sell, the execution by Genato of the
RATIO:
affidavit to annul the contract is not even called for.
16. The petition must be denied for failure to show that the Court of Appeals a. For with or without the aforesaid affidavit their non-payment to
committed a reversible error which would warrant a contrary ruling. complete the full downpayment of the purchase price ipso
17. In a Contract to Sell, the payment of the purchase price is a positive facto avoids their contract to sell, it being subjected to a suspensive
suspensive condition, the failure of which is not a breach, casual or serious, condition.
but a situation that prevents the obligation of the vendor to convey title from b. When a contract is subject to a suspensive condition, its birth or
acquiring an obligatory force. effectivity can take place only if and when the event which
a. It is one where the happening of the event gives rise to an constitutes the condition happens or is fulfilled.
obligation. c. If the suspensive condition does not take place, the parties would
b. Thus, for its non-fulfillment there will be no contract to speak of, stand as if the conditional obligation had never existed.
the obligor having failed to perform the suspensive condition which 20. Nevertheless, this being so Genato is not relieved from the giving of a notice,
enforces a juridical relation. verbal or written, to the Da Jose spouses for decision to rescind their contract.
c. In fact with this circumstance, there can be no rescission of an a. In many cases, even though we upheld the validity of a
obligation that is still non-existent, the suspensive condition not stipulation in a contract to sell authorizing automatic rescission
having occurred as yet for a violation of its terms and conditions, at least a written
d. Emphasis should be made that the breach contemplated in Article notice must be sent to the defaulter informing him of the same.
1191 of the New Civil Code is the obligors failure to comply with b. The act of a party in treating a contract as cancelled should be made
an obligation already extant, not a failure of a condition to render known to the other.
binding that obligation. c. For such act is always provisional. It is always subject to scrutiny
18. Obviously, the foregoing jurisprudence cannot be made to apply to the and review by the courts in case the alleged defaulter brings the
situation in the instant case because no default can be ascribed to the Da Jose matter to the proper courts.
spouses since the 30-day extension period has not yet expired. d. University of the Philippines vs. De Los Angeles:
a. The Da Jose spouses contention that no further condition was agreed i. the party who deems the contract violated may consider
when they were granted the 30-days extension period from October it resolved or rescinded, and act accordingly, without
7, 1989 in connection with clause 3 of their contract to sell dated previous court action, but it proceeds at its own risk.
September 6, 1989 should be upheld for the following reason: ii. For it is only the final judgment of the corresponding
i. firstly, If this were not true, Genato could not have been court that will conclusively and finally settle whether the
persuaded to continue his contract with them and later on action taken was or was not correct in law.
agree to accept the full settlement of the purchase price iii. But the law definitely does not require that the
knowing fully well that he himself imposed such sine contracting party who believes itself injured must first
qua non condition in order for the extension to be valid; file suit and wait for a judgment before taking
ii. secondly, Genato could have immediately annotated his extajudicial steps to protect its interest.
affidavit to annul the contract to sell on his title when it
iv. Otherwise, the party injured by the others breach will a. We do not agree. Apparently,
have to passively sit and watch its damages accumulate b. the factual milieu in Coronel is not on all fours with those in the
during the pendency of the suit until the final judgment case at bar.
of rescission is rendered when the law itself requires that c. In Coronel, this Court found that the petitioners therein clearly
he should exercise due diligence to minimize its own intended to transfer title to the buyer which petitioner themselves
damages (Civil Code, Article 2203). admitted in their pleading. The agreement of the parties therein was
e. This rule validates, both in equity and justice, contracts such as the definitively outline in the Receipt of Down Payment both as to
one at bat, in order to avoid and prevent the defaulting party from property, the purchase price, the delivery of the seller of the property
assuming the offer as still in effect due to the obligees tolerance for and the manner of the transfer of title subject to the specific
such non-fulfillment. condition that upon the transfer in their names of the subject
i. Resultantly, litigations of this sort shall be prevented and property the Coronels will execute the deed of absolute sale.
the relations among would-be parties may be preserved. d. Whereas, in the instant case, even by a careful perusal of the receipt,
21. Thus, Ricardo Chengs contention that the Contract to Sell between Genato Exh. D, alone such kind of circumstances cannot be ascertained
and the Da Jose spouses was rescinded or resolved due to Genatos unilateral without however resorting to the exceptions of the Rule on Parol
rescission finds no support in this case. Evidence.
26. Article 1544. If the same thing should have been sold to different vendees,
the ownership shall be transferred to the person who may have first taken
Issue on the nature of the agreement between Cheng and Genato: possession thereof in good faith, if it should be movable property.
22. The records of this case are replete with admissions that Cheng believed it to a. Should it be immovable property, the ownership shall belong to
be one of a Contract to Sell and not one of Conditional Contract of Sale which the person acquiring it who in good faith first recorded it in the
he, in a transparent turn-around, now pleads in this Petition. Registry of Property.
a. This ambivalent stance of Cheng is even noted by the appellate b. Should there be no inscription, the ownership shall pertain to
court, thus: the person who in good faith was first in possession; and in the
b. plaintiff-appellee was inconsistent in characterizing the contract he absence thereof, to the person who presents the oldest title,
allegedly entered into. In his complaint, Cheng alleged that provided there is good faith
the P50,000.00 down payment was earnest money. 27. This provision connotes that the following circumstances must concur:
c. And next, his testimony was offered to prove that the transaction i. (a) The two (or more) sales transactions in the issue
between him and Genato on October 24, 1989 was actually a must pertain to exactly the same subject matter,
perfected contract to sell. and must be valid sales transactions.
23. This patent twist only operates against Chengs posture which is indicative of ii. (b) The two (or more) buyers at odds over the rightful
the weakness of his claim. ownership of the subject matter must each represent
24. But even if we are to assume that the receipt, Exh. D, is to be treated as a conflicting interests; and
conditional contract of sale, it did not acquire any obligatory force since it iii. (c) The two (or more) buyers at odds over the rightful
was subject to suspensive condition that the earlier contract to sell between ownership of the subject matter must each have bought
Genato and the Da Jose spouses should first be cancelled or rescinded a from the very same seller.
condition never met, as Genato, to his credit, upon realizing his error, b. These situations obviously are lacking in a contract to sell for
redeemed himself by respecting and maintaining his earlier contract with the neither a transfer of ownership nor a sales transaction has been
Da Jose spouses. consummated. The contract to be binding upon the obligee or the
a. Reading of the receipt alone would not even show that a conditional vendor depends upon the fulfillment or non-fulfillment of an event.
contract of sale has been entered by Genato and Cheng. 28. Court is of the view that the governing principle of Article 1544, Civil
b. When the requisites of a valid contract of sale are lacking in said Code, should apply in this situation.
receipt, therefore the sale is neither valid or enforceable. a. Jurisprudence teaches us that the governing principle is PRIMUS
25. To support his now new theory that the transaction was a conditional contract TEMPORE, PORTIOR JURE (first in time, stronger in right).
of sale, petitioner invokes the case of Coronel vs. Court of Appeals
b. For not only was the contract between herein respondents first in c. Whereas in the case of Genatos agreement with Cheng such is
time; it was also registered long before petitioners intrusion as a unavailing. For even before the receipt, Exh. D, was issued to Cheng
second buyer. information of such pre-existing agreement has been brought to his
c. This principle only applies when the special rules provided in the knowledge which did not deter him from pursuing his agreement
aforcited article of Civil Code do not apply or fit the specific with Genato.
circumstances mandated under said law or by jurisprudence 33. Such bad faith, coupled with his wrongful interference with the contractual
interpreting the article. relations between Genato and the Da Jose spouses, which culminated in his
29. The rule exacted by Article 1544 of the Civil Code for the second buyer to filing of the present suit and thereby creating what the counsel for the
be able to displace the first buyer are: respondents describes as a prolonged and economically unhealthy
a. (1) that the second buyer must show that he acted in good faith (i.e. gridlock on both the land itself and the respondents rights provides ample
in ignorance of the first sale and of the first buyers rights) from the basis for the damages awarded.
time of acquisition until title is transferred to him by registration or 34. Based on these overwhelming evidence of bad faith on the part of herein
failing registration, by delivery of possession; petitioner Ricardo Cheng, we find that the award of damages made by the
b. (2) the second buyer must show continuing good faith and appellate court is in order.
innocence or lack of knowledge of the first sale until his contract
ripens into full ownership through prior registration as provided by
law.
30. Thus, in the case at bar, the knowledge gained by the Da Jose spouses, as
first buyers, of the new agreement between Cheng and Genato will not
defeat their rights as first buyers except where Cheng, as second buyer,
registers or annotates his transaction or agreement on the title of the subject
properties in good faith ahead of the Da Jose spouses.
a. Moreover, although the Da Jose spouses, as first buyers, knew of
the second transaction it will not bar them from availing of their
rights granted by law, among them, to register first their agreement
as against the second buyer.
b. In contrast, knowledge gained by Cheng of the first transaction
between the Da Jose spouses and Genato defeats his rights even
if he is first to register the second transaction, since such
knowledge taints his prior registration with bad faith.
31. We have ruled before that when a Deed of Sale is inscribed in the registry of
property on the original document itself, what was done with respect to said
entries or annotations and marginal notes amounted to a registration of the
sale.
a. In this light, we see no reason why we should not give priority in
right the annotation made by the Da Jose spouses with respect to
their Contract to Sell dated September 6, 1989.
32. Moreover, registration alone in such cases without good faith is not
sufficient.
a. Good faith must concur with registration for such prior right to be
enforceable.
b. In the instant case, the annotation made by the Da Jose spouses on
the titles of Genato of their Contract to Sell more than satisfies this
requirement.
24 BADILLA v. BRAGAT (SEE) 36. Before the issuance of the OCT, Spouses Pastrano, on November 18, 1968,
April 22, 2015 | Peralta, J. | Multiple Sales of the same property sold the lot to Eustaquio Ledesma (Ledesma), as evidenced by a Deed of
Definite Sale of Unregistered Coconut and Residential Land.
PETITIONER: Spouses Magdalino and Cleofe Badilla 37. Petitioners, Spouses Magdalino and Cleofe Badilla (Spouses Badilla) claim
RESPONDENTS: Fe Bragat that in 1970, Ledesma sold to them, on installment basis, 200sqm (Lot 19986-
B) of the lot. The sale was not in writing but possession of the 200sqm was
SUMMARY: Spouses Pastrano owned a 1015sqm parcel of land in Tablon, CDO as transferred to them .
evidenced by and OCT. Before the said OCT was issued, they sold the land to 38. On April 18, 1978, Spouses Florito and Fe Bragat (Spouses Bragat) bought
Ledesma in 1968. Spouses Badilla claim that in 1970, Ledesma sold to them 200sqm 991sqm from Ledesma via Deed of Absolute Sale of Residential Lot. 2 tax
of the land via installment and they have been in possession and occupation of the declarations were allegedly issued as a result of the sale: (1) Lot 19986-A
land since then. In 1978, Spouses Bragat bought 991sqm from Ledesma. In 1984, with an area of 642sqm; and (2) Lot 19986-B with an area of 349sqm.
Spouses Pastrano then again sold to Fe Bragat (Bragat), the whole lot (1015sqm). On 39. On May 5, 1984, Spouses Pastrano executed another Deed of Absolute Sale
the same day, Azur Pastrano, filed and affidavit for the loss of owner’s duplicate of registered land with Fe Bragat (Bragat) for the whole land (1015sqm). On
copy of OCT but it was Bragat who petitioned the court and a new OCT was issued. the same day, Azur filed an Affidavit of Loss reporting the loss if the owner’s
In 1987, Spouses Pastrano then again sold the same 1015sqm lot to Bragat so the duplicate copy of the OCT.
OCT was cancelled and a TCT was issued in Bragat’s name. Bragat then told Spouses 40. It was Bragat, however, who petitioned the court for the issuance of a new
Badilla to vacate but the latter refused. RTC ruled in favor of Bragat which the CA owner’s duplicate copy of the OCT so the RTC issued a new one.
affirmed with modifications (only 991sqm was owned by Bragat and 24 sqm was 41. On October 2, 1987, Spouses Pastrano executed another Deed of Sale of
owned by Spouses Badilla). So Spouses Badilla raised the issue to the SC. The issue registered land in favor of Bragat, with the same land as the subject, the whole
in this case is WoN Bragat, who registered the property first, is the proper owner of property with an area of 1015sqm. So the OCT was cancelled and a TCT was
the whole property. The SC ruled that Bragat only owns 863sqm while Spouses issued in the name of Bragat.
Badilla own 152sqm. In such a situation of multiple sales, Article 1544 of the Civil 42. On March 1991, Bragat, through counsel, demanded that the Spouses Badilla
Code relates that ownership shall belong to the person acquiring the property who, vacate the land. Spouses Badilla, also through counsel, refused and raised the
in good faith, first recorded such acquisition. Presently, however, it cannot be said earlier sale of the land to them by Ledesma who bought the land from Spouses
that Bragat’s recording of her 1987 purchase was in good faith because that sale was Pastrano.
simulated and Bragat was aware of other persons who have an interest on the 43. Bragat filed her Complaint for Recovery of Posession and Damages against
property. That the 1987 sale is void is further revealed by evidence to show that one the spouses Magdalino and Cleofe Badilla on June 5, 1992, alleging therein
of its signatories, Profitiza Pastrana was already dead when it was executed. Bragat that she is the absolute owner of Lot No. 19986, covered by TCT. She claimed
herself also admitted that she knew of the Spouses Badillas’ occupation prior to her to have purchased the property, first, from Ledesma, but later, when she
purchase. In that case, the same Article 1544 of the Civil Code provides that when found out that Ledesma was “unauthorized” to sell, she again allegedly made
neither buyer registered, in good faith, the sale of the properties with the register of another purchase of the same property from Azur Pastrano, on May 5, 1984.
deeds, the one who took prior possession of the properties shall be the lawful owner This led to the cancellation of Pastrano’s OCT and the issuance of Bragat’s
thereof. Such prior possessors, at least with respect to the 152sqm. portion, are TCT. Thus, she prays for the Spouses Badilla to be ordered to vacate the
indisputably the Spouses Badilla. around 149sqm portion that they occupy in the property.
44. On June 11, 1992, the Spouses Badilla filed their own Complaint for Quieting
DOCTRINE: Article 1544 of the Civil Code relates that ownership shall belong to of Title, Declaration of Nullity of TCT and Damages against Bragat, claiming
the person acquiring the property who, in good faith, first recorded such acquisition. that the Spouses Badilla are the lawful owners and possessors of Lot No.
19986-B (a portion of Lot No. 19986), having acquired it in 1970 from
Ledesma. The latter, on his part, allegedly bought the bigger Lot No. 19986
FACTS: from Pastrano earlier on November 18, 1968. The Spouses Badilla alleged
35. Azur Pastrano and his wife, Profitiza Ebaning (Spouses Pastrano) were the that they took possession of and built a house on the property upon their
original owners of Lot 19986 which is 1,015sqm in area, located in Tablon, purchase thereof from Ledesma and has since remained in possession.
Cagayan de Oro City. The OCT was in the name of Azur Pastrano. However, they claimed that Pastrano was subsequently able to obtain a free
patent and a title, OCT, over Lot No. 19986. According to the Badillas,
Pastrano made a sale to Bragat on October 2, 1987, but such sale is not valid 4. WoN Bragat, who registered the property first, is the proper owner of the
since Pastrano was no longer the owner of the property on that date. whole property.-NO
Consequently, the Spouses Badilla prayed that TCT issued to Bragat pursuant
to that sale be declared null and void. RULING: WHEREFORE, premises considered, the petition is GRANTED. The
45. The RTC ruled in favor of Bragat noting the following: assailed Decision dated October 9, 2008 and Resolution dated February 12, 2009 of
1. The sketch map shows the 152sqm portion occupied by the Spouses the Court of Appeals in C.A.G. R. CV No. 70423-MIN are hereby REVERSED and
Badilla is within the titled property of Bragat. SET ASIDE. Transfer Certificate of Title No. T47759 is DECLARED VOID, and, in
2. It also found Bragat’s title as valid for what it saw as the result of a its place, two (2) new transfer certificates of titles are ORDERED ISSUED, namely:
purchase in good faith and for value. (1) in the name of the Spouses Magdalino and Cleofe Badilla, covering the 152 sq. m.
46. The trial court observed a lack of evidence of the Spouses Badilla. that they are occupying, and (2) in the name of Fe Bragat, covering the remaining 863
1. The latter allegedly presented handwritten and typewritten receipts sq. m. of the property, of which measurements are to be based on Exhibits “N” and
which were purportedly signed by Ledesma, dated March 5, 1989, Exhibit “N2.” SO ORDERED.
March 1, 1991 and March 23, 1991 acknowledging Ledesma’s
receipt of certain amounts, but the court claimed that it found no RATIO:
evidence of (Ledesma’s) absolute ownership on these dates. 11. Spouses Pastrano had previously sold the land on November 18, 1968 to
2. The court noted that Ledesma had sold previously to the Spouses Ledesma. So as early as 1968, Spouses Pastrano no longer had ownership of
Bragat via a Deed of Absolute Sale of Residential Land dated April the property. Then, as Ledesma sold, in 1970, a portion of the property to
18, 1978. Hence, in the trial court’s view, on March 5, 1989, March Spouses Badilla, who took possession of it despite payment being in
1, 1991 and March 23, 1991, Ledesma no longer owned the land and installments. That the parties intended for ownership to be transferred may
transferred nothing to the Badillas. be inferred from their lack of any agreement stipulating that ownership of the
47. The CA affirmed the RTC decision but modified it. The CA declared that property is reserved by the seller and shall not pass to the buyer until the latter
only 991sqm was transferred to Bragat in 1978 so the remaining 24sqm was has fully paid the purchase price.33 The fact is, Ledesma even delivered to
validly transferred to Badillas in 1991. It also removed the award of damages. the Badillas the owner’s duplicate copy of OCT No. P2035. The Civil Code
48. Hence this petition by the Spouses Badilla. states that ownership of the thing sold is transferred to the vendee upon the
49. Contention of Spouses Badilla: actual or constructive delivery of the same.
1. ownership of the 200sqm. portion was transferred to them when they 12. Therefore, since Spouses Badilla already owned and occupied the 152sqm
purchased the same and possession was delivered to them by portion, the TCT issued to Bragat covering the said portion was wrongfully
Ledesma in 1970. issued.
2. that when OCT No. P2035 was actually issued in 1980, it was first 13. Also, the TCT issued to Bragat was on the strength of the Deed of Sale dated
delivered by Pastrano to Ledesma and the latter delivered the same October 2, 1987 but this Deed of Sale is void for being simulated since both
to them (the Badillas). Thus, Bragat allegedly falsely claimed the Azur Pastrano and Bragat knew that at the time of its execution of the
“loss” of the title when she petitioned the court for a new duplicate Pastrano’s lack of ownership over Lot No. 19986, the property being sold. At
original, because such title was not lost but had been with the that time, it was not Pastrano but Ledesma who was absolute owner of the
Badillas all along. property by virtue of the latter’s earlier purchase of Lot No. 19986 from the
3. Another fraud that Bragat allegedly committed was the Deed of Sale Spouses Pastrano on November 18, 1968, via a Deed of Definite Sale of
dated October 2, 1987, in which Profitiza Pastrano signed (in marital Unregistered Coconut and Residential Land. Bragat herself knew this, as she
consent) although she had been dead since March 30, 1985 and her husband themselves first bought the property from Ledesma through
50. Bragat claims: a Deed of Absolute Sale of Residential Land dated April 18, 1978.
1. that the sale of October 2, 1987 was only a re-execution of the sale 14. Pastrano could no longer sell any part of the property to Bragaton such later
of May 5, 1984, in order to avoid tax surcharges. dates since he had already sold the same as early as November 18, 1968 to
2. Badilla’s documentary evidence were all executed only after she had Ledesma. Wellsettled is the rule that no one can give what one does not have
the property in her name. — nemo dat quod non habet — and, accordingly, one can sell only what one
owns or is authorized to sell, and the buyer acquires no better title than the
ISSUE/s: seller.44 Thus, the sales made on the dates May 5, 1984 and October 2, 1987
are void for being simulated and for lack of a subject matter. On these sales,
Bragat cannot claim good faith as she herself knew of Pastrano’s lack of
ownership.
15. Bragat’s property which she bought from Ledesma in 1978 does not include
the 152sqm portion already owned by Spouses Badilla. So Bragat is only
entitled to a new TCT based on the Deed of Sale dated April 18, 1978
excluding the 152sqm portion of the Spouses Badilla.
16. 2 new TCTs should be issued. (1) in the name of Spouses Badilla for the
152sqm portion; and (2) in the name of Bragat for the 863sqm portion.
17. In such a situation of multiple sales, Article 1544 of the Civil Code relates
that ownership shall belong to the person acquiring the property who, in good
faith, first recorded such acquisition. Presently, however, it cannot be said
that Bragat’s recording of her 1987 purchase was in good faith because
that sale was simulated and Bragat was aware of other persons who have
an interest on the property. That the 1987 sale is void is further revealed
by evidence to show that one of its signatories, Profitiza Pastrana was
already dead when it was executed. Bragat herself also admitted that she
knew of the Spouses Badillas’ occupation prior to her purchase.
18. In that case, the same Article 1544 of the Civil Code provides that when
neither buyer registered, in good faith, the sale of the properties with the
register of deeds, the one who took prior possession of the properties
shall be the lawful owner thereof. Such prior possessors, at least with
respect to the 152sqm. portion, are indisputably the Spouses Badilla.
25 CRB v. DELA CRUZ (Siapno) must have been made by a party who has an existing right in the thing and the
January 17, 2005| Tinga, J. | 1544 applies when party has right & power to dispose power to dispose of it. It cannot be invoked where the two different contracts of
sale are made by two different persons, one of them not being the owner of the
PETITIONERS: Consolidated Rural Bank (Cagayan Valley), Inc. property sold. And even if the sale was made by the same person, if the second
RESPONDENTS: The Honorable Court of Appeals and Heirs of Teodoro Dela sale was made when such person was no longer the owner of the property,
Cruz because it had been acquired by the first purchaser in full dominion, the second
purchaser cannot acquire any right.
SUMMARY: Madrid brothers subdivided a lot and Rizal sold his share, Lot No.
7036-A-7, to Gamiao (southern portion) and Dayag (northern portion). They FACTS:
failed to register it with the Office of the Register of Deeds in 1957, they only 1. Rizal, Anselmo, Gregorio, Filomeno and Domingo Madrid (Madrid brothers),
declared the property for taxation purposes in their names. In 1964, Gamiao and were the registered owners of Lot 7036-A situated in San Mateo, Isabela. Lot
Dayag sold the land to Dela Cruz and Hernandez. In 1976, the Madrid brothers, at was subdivided into several lots.
that time they were no longer the owners, conveyed all their rights and interests 2. 1957: Rizal sold part of his share identified as Lot No. 7036-A-7, to Aleja
over Lot No. 7036-A-7 to Marquez. Marquez subdivided the lots and some lots Gamiao and Felisa Dayag by virtue of a Deed of Sale, the rest of the Madrid
were mortgaged to CRB, and RBC. The heirs of Dela Cruz filed a case for brothers did not object in their Joint Affidavit.
reconveyance and damages, while the successor-in-interest of Hernandez filed 3. The deed of sale was not registered with the Office of the Register of Deeds of
with leave of court a Complaint in Intervention wherein she claimed the northern Isabela. However, Gamiao and Dayag declared the property for taxation
portion of the lot. Marquez alleged that apart from being the first registrant, he purposes in their names
was a buyer in good faith and for value. He also argued that the sale executed by 4. 1964: Gamiao and Dayag sold the southern half of the lot to Teodoro dela Cruz,
Madrid to Gamiao and Dayag was not binding upon him, it being unregistered. and the northern half, to Restituto Hernandez. They both took possession of and
Banks said they are mortgagees in good faith and that they had the right to rely on cultivated the portions of the property respectively sold to them.
the titles of Marquez which were free from any lien or encumbrance. Lower courts 5. Restituto Hernandez eventually donated the northern half to his daughter,
resolved the present controversy by applying the rule on double sale provided in Evangeline Hernandez-del Rosario. The children of Teodoro dela Cruz
Article 1544 of the Civil Code. The RTC made CRB and the other defendants win, continued possession of the southern half after their father's death
while the CA decided the case in favor of the Heirs. Issue is WON Marquez is 6. 1976: In a Deed of Sale, the Madrid brothers conveyed all their rights and
the rightful owner? SC ruled that both courts erred in using Art 1544. The interests over Lot No. 7036-A-7 to Pacifico Marquez.
provision is not applicable in the present case. It contemplates a case of double or 7. Subsequently, Marquez subdivided Lot No. 7036-A-7 into 8 lots:
multiple sales by a single vendor and covers a situation where a single vendor sold a. Mortagaged 4 lots to the Consolidated Rural Bank, Inc. of Cagayan Valley
one and the same immovable property to two or more buyers. It is necessary that (CRB) to secure a loan of P100,000.00.
the conveyance must have been made by a party who has an existing right in the b. Marquez mortgaged another lot to the Rural Bank of Cauayan (RBC) to
thing and the power to dispose of it. It cannot be invoked where the two different secure a loan of P10,000.00.
contracts of sale are made by two different persons, one of them not being the c. Marquez sold another lot to Romeo Calixto but reacquired the same so walang effect. (just in
owner of the property sold. In the case at bar, the subject property was not case sir asks)
transferred to several purchasers by a single vendor. In the first deed of sale, the 8. Marquez defaulted so CRB caused the foreclosure of the mortgages in its favor
vendors were Gamiao and Dayag whose right to the subject property originated and the lots were sold to it as the highest bidder.
from their acquisition thereof from Madrid. On the other hand, the vendors in the 9. The Heirs of dela Cruz and successors-in-interest of Hernandez claimed all of
other or later deed were the Madrid brothers but at that time they were no longer this to be null and void.
the owners since they had long before disposed of the property in favor of Gamiao 10. Heirs of Teodor Dela Cruz filed a case for reconveyance and damages of the
and Dayag. Second issue is WON the Banks are mortgagees in good faith? SC southern portion of Lot No. 7036-A against Marquez, Calixto, RBC and CRB.
held that CRB is a mortgagee in bad faith because banks are expected to exercise 11. Evangeline del Rosario, the successor-in-interest of Restituto Hernandez, filed
more care and prudence, their business being impressed with public interest, with leave of court a Complaint in Intervention wherein she claimed the
northern portion of Lot No. 7036-A-7.
DOCTRINE: For Article 1544 to apply, it is necessary that the conveyance 12. Marquez alleged that apart from being the first registrant, he was a buyer in
good faith and for value. He also argued that the sale executed by Rizal Madrid
to Gamiao and Dayag was not binding upon him, it being unregistered. acquiring it who in good faith first recorded it in the Registry of Property.
13. CRB & RBC insisted that they were mortgagees in good faith and that they had
the right to rely on the titles of Marquez which were free from any lien or Should there be no inscription, the ownership shall pertain to the person who in
encumbrance. good faith was first in possession; and, in the absence thereof, to the person
14. RTC declared Marquez to be the lawful owner of Lots 7036-A-7 and the who presents the oldest title, provided there is good faith.”
mortgages to CRB and RBC as valid saying that Marquez was a buyer in good 21. The provision is not applicable in the present case. It contemplates a case of
faith because the conveyances were not registered and relied on Article 1544 double or multiple sales by a single vendor. Art 1544 covers a situation where a
(provided in the Ratio) of the Civil Code. single vendor sold one and the same immovable property to two or more buyers.
15. CA reversed RTC’s decision saying that Marquez failed to prove his good faith 22. It is necessary that the conveyance must have been made by a party who has an
and declared null and void the sale between Marquez and Madrid brothers and existing right in the thing and the power to dispose of it. It cannot be invoked
also the mortages. CA ordered Marquez to reconvey lot to the heirs of Teodoro where the two different contracts of sale are made by two different persons, one
dela Cruz and Evangeline Hernandez-del Rosario. of them not being the owner of the property sold.
16. Marquez admitted having knowledge that the subject property was "being 23. And even if the sale was made by the same person, if the second sale was made
taken" by the Heirs at the time of the sale. The Heirs were also in possession of when such person was no longer the owner of the property, because it had been
the land at the time. CA held that circumstances along with the subject acquired by the first purchaser in full dominion, the second purchaser cannot
property's attractive location — it was situated along the National Highway and acquire any right.
was across a gasoline station — should have put Marquez on inquiry as to its 24. In the case at bar, the subject property was not transferred to several purchasers
status. Instead, Marquez closed his eyes to these matters and failed to exercise by a single vendor. In the first deed of sale, the vendors were Gamiao and
the ordinary care expected of a buyer of real estate. Dayag whose right to the subject property originated from their acquisition
17. As to the mortgagees RBC and CRB, CA found that they were mortgagees in thereof from Rizal. On the other hand, the vendors in the other or later deed
bad faith because they merely relied on the certificates of title of the mortgaged were the Madrid brothers but at that time they were no longer the owners
properties. They did not ascertain the status and condition thereof according to since they had long before disposed of the property in favor of Gamiao and
standard banking practice. Dayag. Art 1544 of the Civil Code presupposes the right of the vendor to
18. Hence, the instant CRB petition. However, both Marquez and RBC elected not dispose of the thing sold.
to challenge the Decision of the appellate court. When not all the requisites are present to warrant the application of Art 1544
25. The principle of prior tempore, potior jure or simply "he who is first in time is
ISSUE: WoN Marquez is the rightful owner? NO, Gamiao & Dayag’s heirs preferred in right," should apply. The only one who can invoke this is the first
WoN RBC and CRB are mortgagees in bad faith? YES vendee. Undisputedly, he is a purchaser in good faith because at the time he
bought the real property, there was still no sale to a second vendee.
HELD: WHEREFORE, the Petition is DENIED. The dispositive portion of the 26. In the instant case, the sale to the Heirs by Gamiao and Dayag, who first bought
Court of Appeals' Decision, as modified by its Resolution dated 5 January 1998, is it from Rizal Madrid, was anterior to the sale by the Madrid brothers to
AFFIRMED. Costs against petitioner. Marquez. The Heirs also had possessed the subject property first in time. Thus,
SO ORDERED. applying the principle, the Heirs, without a scintilla of doubt, have a superior
right to the subject property.
RATIO: 27. Moreover, it is an established principle that no one can give what one does not
IMPT: Art 1544 only applies when vendor has the right to dispose the thing sold have — nemo dat quod non habet. Accordingly, one can sell only what one
19. Lower courts, although used Art 1544, arrived at different conclusions. The RTC owns or is authorized to sell, and the buyer can acquire no more than what the
made CRB and the other defendants win, while the CA decided the case in favor seller can transfer legally.
of the Heirs. 28. In this case, since the Madrid brothers were no longer the owners of the subject
20. ART. 1544 provides: “If the same thing should have been sold to different property at the time of the sale to Marquez, the latter did not acquire any right
vendees, the ownership shall be transferred to the person who may have first to it.
taken possession thereof in good faith, if it should be movable property. Assuming arguendo that Art 1544 applies
29. The claim of Marquez still cannot prevail over the right of the Heirs since
Should it be immovable property, the ownership shall belong to the person according to the evidence he was not a purchaser and registrant in good faith.
30. Following Article 1544, in the double sale of an immovable, the rules of standard procedure in its operations, we agree with the Court of Appeals that
preference are: (a) the first registrant in good faith; (b) should there be no entry, CRB is a mortgagee in bad faith.
the first in possession in good faith; and (c) in the absence thereof, the buyer
who presents the oldest title in good faith.
31. Prior registration of the subject property does not by itself confer ownership or
a better right over the property. Article 1544 requires that before the second
buyer can obtain priority over the first, he must show that he acted in good faith
from the time of acquisition until the title is transferred to him by registration or
failing registration, by delivery of possession.
32. The actions of Marquez have not satisfied the requirement of good faith from the
time of the purchase of the subject property to the time of registration. Found by
the CA, Marquez knew at the time of the sale that the subject property was being
claimed or "taken" by the Heirs. This was a detail which could indicate a defect
in the vendor's title which he failed to inquire into. Marquez also admitted that
he did not take possession of the property and at the time he testified he did not
even know who was in possession.
33. The rule of caveat emptor requires the purchaser to be aware of the supposed title
of the vendor and one who buys without checking the vendor's title takes all the
risks and losses consequent to such failure. Instead, Marquez willfully closed his
eyes to the possibility of the existence of these flaws. For failure to exercise the
measure of precaution which may be required of a prudent man in a like situation,
he cannot be called a purchaser in good faith.
34. The requirement of good faith in the possession of the property finds no
application in cases where there is no second sale. In the case at bar, Teodoro
dela Cruz took possession of 18 years before the Heirs had knowledge of the
registration of said sale. Thus, there was no need for the appellate court to
consider the issue of good faith or bad faith with regard to Teodoro dela Cruz's
possession of the subject property.
35. Also, the regularity of the sale to Gamiao and Dayag was never contested by
Marquez. RTC upheld the validity of this sale, holding that the Madrid brothers
are bound by the sale by virtue of their Joint Affidavit.
36. Moreover, as this Court declared in the case of Heirs of Simplicio Santiago v.
Heirs of Mariano E. Santiago, tax declarations "are good indicia of possession
in the concept of an owner, for no one in his right mind would be paying taxes
for a property that is not in his actual or constructive possession."
CRB and RBS are mortgagees in bad faith
37. This rule equally applies to mortgagees of real property. It is a well-settled rule
that a purchaser or mortgagee cannot close his eyes to facts which should put a
reasonable man upon his guard, and then claim that he acted in good faith under
the belief that there was no defect in the title of the vendor or mortgagor.
38. Banks, their business being impressed with public interest, are expected to
exercise more care and prudence than private individuals in their dealings, even
those involving registered lands. Hence, for merely relying on the certificates of
title and for its failure to ascertain the status of the mortgaged properties as is the
26 Carbonell v CA (Mel) which directs that ownership of immovable property should be recognized in
January 26, 1976| Makasiar, J.| First in time, stronger in right favor of one "who in good faith recorded" his right.

PETITIONER: Rosario Carbonell


RESPONDENTS: Honorable Court of Appeals, Jose Poncio, Emma Infante and FACTS:
Ramon Infante 1. Prior to January 27, 1955, respondent Jose Poncio, a native of the Batanes
Islands, was the owner of the parcel of land herein involve with
SUMMARY: Poncio, a Batanes native, owned a parcel of land, which he offered improvements situated at 179 V. Agan St., San Juan, Rizal, having an area of
to sell to Carbonell and Infante. The land was mortgaged to Republic Bank. some 195 square meters covered by TCT No. 5040 and subject to mortgage
Poncio, unable to keep up with the installments due on the mortgage, approached in favor of the Republic Savings Bank for the sum of P1,500.00.
Carbonell one day and offered to sell to the said lot, excluding the house wherein 2. Petitioner Rosario Carbonell, a cousin and adjacent neighbor of respondent
respondent lived. Poncio assumed to pay the mortgage in favor of the bank. Poncio, and also from the Batanes Islands, lived in the adjoining lot at 177 V.
Poncio and Carbonell executed an instrument where Carbonell allowed Poncio to Agan Street.
remain in the premises in spite of the sale for a period of 1 year. Later on, when 3. Both petitioners Rosario Carbonell and respondent Emma Infante offered to
the Formal Deed of Sale was to be executed, Poncio told Carbonell that he could buy the said lot from Poncio.
no longer proceed with the sale as he had already sold the same to Infante for a 4. Poncio, unable to keep up with the installments due on the mortgage,
better price. Carbonell immediately sought to register adverse claim; 4 days later, approached Carbonell one day and offered to sell to the said lot, excluding
Infante registered the sale with the adverse claim annotated thereto. Infante the house wherein he lived.
thereafter introduced significant improvements on the property. The trial court 5. Carbonell accepted the offer and proposed the price of P9.50 per square
declared the claim of Infante to be superior to that of the first buyer Carbonell. meter. Poncio accepted the price proposed by Carbonell, on the condition that
The Court of Appeals (Fifth Division) reversed the decision of the trial court, from the purchase price would come the money to be paid to the bank.
declaring Carbonell to have a superior right to the land in question, and 6. Carbonell and Poncio then went to the Republic Savings Bank and secured
condemning Infante to reconvey to the former, after reimbursement of expenses, the consent of the President thereof for her to pay the arrears on the mortgage
the land in question and all its improvements. On motion for reconsideration, a and to continue the payment of the installments as they fall due. The amount
special division of five of the said appeals court ruled in favor of the respondents. in arrears reached a total sum of P247.26. But because respondent Poncio had
Carbonell moved to reconsider the Resolution of the Special Division of Five, previously told her that the money, needed was only P200.00, only the latter
which motion was denied by Minute Resolution. The issue in this case is who has amount was brought by Carbonell constraining Poncio to withdraw the sum
a better title, Carbonell or Infante? The Supreme Court held that Carbonell has a of P47.00 from his bank deposit with Republic Savings Bank. But the next
better title to the land. In order to claim the benefit of Art. 1544, the buyer of day, Carbonell refunded to Poncio the sum of P47.00.
realty must register the property in good faith. It is a pre-condition to a superior 7. On January 27, 1955, Carbonell and Poncio, in the presence of a witness,
title. In this case, Infante was not in good faith, thus the prior sale to Carbonell made and executed a document in the Batanes dialect, which, translated into
must prevail. There was a showing of bad faith by Infante in the following English, reads:
circumstances: Infante registered her claim 4 days after the adverse claim was CONTRACT FOR ONE HALF LOT WHICH I BOUGHT FROM JOSE PONCIO Beginning
registered, she had notice that Carbonell paid off the mortgage debt as the today January 27, 1955, Jose Poncio can start living on the lot sold by him to me, Rosario
Carbonell, until after one year during which time he will not pay anything. Then if after said one
mortgage passbook was already in his possession and she ignored Carbonell and year, he could not find a place where to move his house, he could still continue occupying the
refused to talk to her. site but he should pay a rent.
8. Carbonell asked Atty. Salvador Reyes to prepare the formal deed of sale,
DOCTRINE: (from syllabus, wasn’t expressly mentioned in the majority which she brought to Poncio together with the amount of some P400.00, the
opinion but was mentioned in Teehankee’s concurring opinion) MAIN RULE balance she still had to pay in addition to her assuming the mortgaged
UNDER ART. 1544: “Primus Tempore, Portior Jure”. – FIRST IN TIME, obligation to Republic Savings Bank.
STRONGER IN RIGHT 9. Upon arriving at Poncio's house, however, Poncio told Carbonell that he
could not proceed any more with the sale, because he had already given the
The buyer of realty must act in good faith in registering his deed of sale to merit lot to Emma Infante; and that he could not withdraw from his deal with
the protection of the second paragraph of Article 1544 of the New Civil Code Infante, even if he were to go to jail. Carbonell then sought to contact Infante
but the latter refused to see her. written in the Batanes dialect aforementioned, Infantes objected to the
10. On February 5, 1955, Carbonell saw Infante erecting a wall around the lot presentation by petitioner of parole evidence to prove the alleged sale
with a gate. between her and respondent Poncio. The trial court sustained the objection
11. Carbonell then consulted Atty. Jose Garcia, who advised her to present an and dismissed the complaint.
adverse claim over the land in question with the Office of the Register of 21. Carbonell appealed to the Supreme Court which ruled that the Statute of
Deeds of Rizal. Atty. Garcia actually sent a letter of inquiry to the Register Frauds, being applicable only to executory contracts, does not apply to the
of Deeds and demand letters to Poncio and Infante. alleged sale between Carbonell and Poncio, which Carbonell claimed to have
12. In his answer, Poncio admitted "that on January 30, 1955, Mrs. Infante been partially performed, so that Carbonell is entitled to establish by parole
improved her offer and he agreed to sell the land and its improvements to her evidence "the truth of this allegation, as well as the contract itself." The order
for P3,535.00" appealed from was thus reversed, and the case remanded to the court a quo
13. In a private memorandum agreement, Poncio indeed bound himself to sell to for further proceedings.
his corespondent Emma Infante, the property for the sum of P2,357.52, with 22. After trial in the court a quo; a decision was, rendered declaring the second
respondent Emma Infante still assuming the existing mortgage debt in favor sale by respondent Jose Poncio to his co-respondents Ramon Infante and
of Republic Savings Bank in the amount of P1,177.48. Infante lives just Emma Infante of the land in question null and void and ordering respondent
behind the houses of Poncio and Rosario Carbonell. Poncio to execute the proper deed of conveyance of said land in favor of
14. On February 2, 1955, respondent Jose Poncio executed the formal deed of Carbonell.
sale in favor of respondent Mrs. Infante in the total sum of P3,554.00 and on 23. Respondent Infantes filed a motion for re-trial to adduce evidence for the
the same date, the latter paid Republic Savings Bank the mortgage proper implementation of the court's decision in case it would be affirmed on
indebtedness of P1,500.00. The mortgage on the lot was eventually appeal which motion was opposed by Carbonell for being premature.
discharged. 24. Before their motion for re-trial could be resolved, respondent Infantes, filed
15. Informed that the sale in favor of respondent Emma Infante had not yet been another motion for new trial, claiming that the decision of the trial court is
registered, Atty. Garcia prepared an adverse claim for petitioner, who signed contrary to the evidence and the law which motion was also opposed by
and swore to and registered the same on February 8, 1955. Carbonell.
16. The deed of sale in favor of respondent Mrs. Infante was registered only on 25. The trial court granted a new trial at which re-hearing only the respondents
February 12, 1955. As a consequence, thereof, a Transfer Certificate of Title introduced additional evidence consisting principally of the cost of
was issued to her but with the annotation of the adverse claim of Carbonell. improvements they introduced on the land in question.
17. Emma Infante took immediate possession of the lot involved, covered the 26. After the re-hearing, the trial court ruled that the claim of the respondents was
same with 500 cubic meters of garden soil and built therein a wall and gate, superior to the claim of Carbonell. Carbonell appealed.
spending the sum of P1,500.00. She further contracted the services of an 27. The Court of Appeals rendered judgment reversing the decision of the trial
architect to build a house; but the construction only started in 1959 — years court, declaring Carbonell to have a superior right to the land in question, and
after the litigation actually began and during its pendency. condemning the defendant Infantes to reconvey to Carbonell after her
18. Carbonell filed a second amended complaint against private respondents, reimbursement to them of the sum of P3,000.00 plus legal interest, the land
praying that she be declared the lawful owner of the questioned parcel of in question and all its improvements
land; that the subsequent sale to respondents Infanet be declared null and 28. Respondents sought reconsideration of said decision and acting on the motion
void, and that Poncio be ordered to execute the corresponding deed of for reconsideration, the Appellate Court, three Justices (Villamor, Esguerra
conveyance of said land in her favor and for damages and attorney's fees. and Nolasco) of Special Division of Five, ruled in favor of the respondents.
19. Respondents first moved to dismiss the complaint on the ground that 29. Carbonell moved to reconsider the Resolution of the Special Division of Five,
Carbonell's claim is unenforceable under the Statute of Frauds, the alleged which motion was denied by Minute Resolution (but with Justices Rodriguez
sale in her favor not being evidenced by a written document and when said and Gatmaitan voting for reconsideration)
motion was denied without prejudice to passing on the question raised therein
when the case would be tried on the merits, respondents filed separate ISSUE:
answers, reiterating the grounds of their motion to dismiss. Who has a better title, Carbonell or Infante? – Carbonell.
20. During the trial, when Carbonell started presenting evidence of the sale of the
land in question to her by Poncio, part of which evidence was the agreement RULING:
WHEREFORE, THE DECISION OF THE SPECIAL DIVISION OF FIVE OF THE COURT OF APPEALS OF Carbonell's good faith did not cease after Poncio told her on January 31, 1955
OCTOBER 30, 1968 IS HEREBY REVERSED; PETITIONER ROSARIO CARBONELL IS HEREBY
DECLARED TO HAVE THE SUPERIOR RIGHT TO THE LAND IN QUESTION AND IS HEREBY of his second sale of the same lot to Infante. Because of that information,
DIRECTED TO REIMBURSE TO PRIVATE RESPONDENTS INFANTES THE SUM OF ONE THOUSAND Carbonell wanted an audience with Infante, which desire underscores
FIVE HUNDRED PESOS (P1,500.00) WITHIN THREE (3) MONTHS FROM THE FINALITY OF THIS Carbonell's good faith.
DECISION; AND THE REGISTER OF DEEDS OF RIZAL IS HEREBY DIRECTED TO CANCEL TRANSFER
CERTIFICATE OF TITLE NO. 37842 ISSUED IN FAVOR OF PRIVATE RESPONDENTS INFANTES 8. With an aristocratic disdain unworthy of the good breeding of a good
COVERING THE DISPUTED LOT, WHICH CANCELLED TRANSFER CERTIFICATE OF TITLE NO. 5040 Christian and good neighbor, Infante snubbed Carbonell like a leper and
IN THE NAME OF JOSE PONCIO, AND TO ISSUE A NEW TRANSFER CERTIFICATE OF TITLE IN refused to see her. So Carbonell did the next best thing to protect her right —
FAVOR OF PETITIONER ROSARIO CARBONELL UPON PRESENTATION OF PROOF OF PAYMENT BY
HER TO THE INFANTES OF THE AFORESAID AMOUNT OF ONE THOUSAND FIVE HUNDRED PESOS she registered her adversed claim on February 8, 1955. Under the
(P1,500.00). PRIVATE RESPONDENTS INFANTES MAY REMOVE THEIR AFOREMENTIONED USEFUL circumstances, this recording of her adverse claim should be deemed to
IMPROVEMENTS FROM THE LOT WITHIN THREE (3) MONTHS FROM THE FINALITY OF THIS have been done in good faith and should emphasize Infante's bad faith
DECISION, UNLESS THE PETITIONER ROSARIO CARBONELL ELECTS TO ACQUIRE THE SAME AND
PAYS THE INFANTES THE AMOUNT OF THIRTEEN THOUSAND FOUR HUNDRED TWENTY-NINE when she registered her deed of sale 4 days later on February 12, 1955.
PESOS (P13,429.00) WITHIN THREE (3) MONTHS FROM THE FINALITY OF THIS DECISION. SHOULD 9. Bad faith arising from previous knowledge by Infante of the prior sale
PETITIONER CARBONELL FAIL TO PAY THE SAID AMOUNT WITHIN THE AFORESTATED PERIOD to Carbonell is shown by the following facts:
OF THREE (3) MONTHS FROM THE FINALITY OF THIS DECISION, THE PERIOD OF THREE (3)
MONTHS WITHIN WHICH THE RESPONDENTS INFANTES MAY REMOVE THEIR AFOREMENTIONED
USEFUL IMPROVEMENTS SHALL COMMENCE FROM THE EXPIRATION OF THE THREE (3) MONTHS (1) Mrs. Infante refused to see Carbonell, who wanted to see Infante after she was
GIVEN PETITIONER CARBONELL TO PAY FOR THE SAID USEFUL IMPROVEMENTS. informed by Poncio that he sold the lot to Infante but several days before Infante
registered her deed of sale. This indicates that Infante knew of the prior sale of the lot
RATIO: by Poncio to Carbonell. Ordinarily, one will not refuse to see a neighbor. Infante lives
1. Article 1544, New Civil Code, which is decisive of this case, recites:
If the same thing should have been sold to different vendees, the ownership shall be transferred to the
just behind the house of Carbonell. Her refusal to talk to Carbonell could only mean
person who may have first taken possession thereof in good faith, if it should movable property. that she did not want to listen to Carbonell's story that she (Carbonell) had previously
Should it be immovable property, the ownership shall belong to the person acquiring it who in good faith bought the lot from Poncio.
first recorded it in the Registry of Property.
Should there be no inscription, the ownership shall pertain to the person who in good faith was first in the
possession; and, in the absence thereof, to the person who presents the oldest title, provided there is good (2) Carbonell was already in possession of the mortgage passbook and Poncio's
faith copy of the mortgage contract, when Poncio sold the lot Carbonell who, after paying
2. It is essential that the buyer of realty must act in good faith in registering the arrearages of Poncio, assumed the balance of his mortgaged indebtedness to the
his deed of sale to merit the protection of the 2nd paragraph of said Art bank, which in the normal course of business must have necessarily informed
1544. Infante about the said assumption by Carbonell of the mortgage indebtedness of
3. Unlike the first and third paragraphs of said Article 1544, which accord Poncio. Before or upon paying in full the mortgage indebtedness of Poncio to the
preference to the one who first takes possession in good faith of personal or Bank.
real property, the second paragraph directs that ownership of immovable
property should be recognized in favor of one "who in good faith first (3) The fact that Poncio was no longer in possession of his mortgage passbook and
recorded" his right. Under the first and third paragraph, good faith must that the said mortgage passbook was already in possession of Carbonell, should have
characterize the act of anterior registration. compelled Infante to inquire from Poncio why he was no longer in possession of the
4. If there is no inscription, what is decisive is prior possession in good faith. If mortgage passbook and from Carbonell why she was in possession of the same. The
there is inscription, as in the case at bar, prior registration in good faith only plausible and logical reason why Infante did not bother anymore to make
is a pre-condition to superior title. such injury was because in the ordinary course of business the bank must have
5. When Carbonell bought the lot from Poncio on January 27, 1955, she told her that Poncio already sold the lot to Carbonell who thereby assumed the
was the only buyer thereof and the title of Poncio was still in his name mortgage indebtedness of Poncio and to whom Poncio delivered his mortgage
solely encumbered by bank mortgage duly annotated thereon. passbook.
6. Carbonell was not aware — and she could not have been aware — of any sale
of Infante as there was no such sale to Infante then. Hence, Carbonell's prior (4) Carbonell registered her adverse claim, which was accordingly annotated on
purchase of the land was made in good faith. Poncio's title, 4 days before Infante registered her deed of sale. Here she was again
7. Her good faith subsisted and continued to exist when she recorded her on notice of the prior sale to Carbonell. Such registration of adverse claim is valid and
adverse claim 4 days prior to the registration of Infantes's deed of sale. effective.
(5) In his answer Poncio alleged that both Mrs. Infante and Mrs. Carbonell offered to 16. It appearing that the Infantes are possessors in bad faith, their rights to the
buy the lot at P15.00 per square meter, which offers he rejected as he believed that his improvements they introduced op the disputed lot are governed by Articles
lot is worth at least P20.00 per square meter. It is therefore logical to presume that 546 and 547 of the New Civil Code.
Infante was told by Poncio and consequently knew of the offer of Carbonell which 17. As a matter of equity, the Infantes, although possessors in bad faith, should
fact likewise should have put her on her guard and should have compelled her to be allowed to remove the aforesaid improvements, unless Carbonell chooses
inquire from Poncio whether or not he had already sold the property to to pay for their value at the time the Infantes introduced said useful
Carbonell. improvements in 1955 and 1959. The Infantes cannot claim reimbursement
for the current value of the said useful improvements; because they have been
10. The Existence of Prior Sale to Carbonell Duly Established. The Supreme enjoying such improvements for about two decades without paying any rent
Court pointed out to the rulings of the lower court which showed that there on the land and during which period Carbonell was deprived of its possession
was a previous sale by Poncio to Carbonell and the document they executed and use.
allowed Poncio to continue stayin in the premises for one year. The document
having been written originally in a dialect well understood by Poncio, he TEEHANKEE, J., concurring:
signed with full knowledge and consciousness of the terms and consequences - The answer to the question of who between the two buyers in good faith
thereof. should prevail is provided in the second paragraph of Article 1544 of the Civil
11. There was adequate consideration or price for the sale in favor of Code which ordains that "the ownership of the immovable property shall
Carbonell. It should be emphasized that the mortgage on the lot was about belong to the person acquiring it who in good faith first recorded it in the
to be foreclosed by the bank for failure on the part of Poncio to pay the Registry of Property."
amortizations thereon. To forestall the foreclosure and at the same time to - In the case at bar, the seller executed on January 27, 1955 the private
realize some money from his mortgaged lot, Poncio agreed to sell the same memorandum of sale of the property in favor of the first buyer Carbonell,
to Carbonell at P9.50 per square meter, on condition that Carbonell [1] should However, six days later on February 2, 1955, the seller sold the property for
pay (a) the amount of P400.00 to Poncio and 9b) the arrears in the amount of a second time for an improved price, this time executing a formal registrable
P247.26 to the bank; and [2] should assume his mortgage indebtedness. deed of sale in favor of the second buyer Infante.
12. The sale did not include Poncio's house on the lot. And Poncio was given the - The facts of record amply show that Carbonell had a written memorandum
right to continue staying on the land without paying any rental for one year, of sale, which was partially executed with the advance payment made by her
after which he should pay rent if he could not still find a place to transfer his for the Poncio’s mortgage account with the bank, and which was perfected
house. All these terms are part of the consideration of the sale to and binding in law by their accord on the subject matter and price.
Carbonell. It is evident therefore that there was ample consideration, and not - The governing principle here is prius tempore, portior jure (first in time,
merely the sum of P200.00, for the sale of Poncio to Carbonell of the lot in stronger in right). Knowledge gained by the first buyer of the second sale
question. cannot defeat the first buyer's rights except only as provided by the Civil
13. But Poncio, induced by the higher price offered to him by Infante, Code and that is where the second buyer first registers in good faith the
reneged on his commitment to Carbonell and told Carbonell, who second sale ahead of the first. Such knowledge of the first buyer does not bar
confronted him about it, that he would not withdraw from his deal with her from availing of her rights under the law, among them, to register first
Infante even if he is sent to jail. her purchase as against the second buyer. But in other so knowledge gained
14. Inevitably evident therefore from the foregoing discussion, is the bad faith by the second buyer of the first sale defeats his rights even if he is first to
of Emma Infante from the time she enticed Poncio to dishonor his register the second sale, since such knowledge taints his prior registration
contract with Carbonell, and instead to sell the lot to her (Infante) by with bad faith.
offering Poncio a much higher price than the price for which he sold the
same to Carbonell. Being guilty of bad faith, both in taking physical MUÑOZ PALMA, J., dissenting:

possession of the lot and in recording their deed of sale, the Infantes cannot - We are here confronted with a double sale made by Poncio, the solution to
recover the value of the improvements they introduced in the lot. which is found in Art. 1544 of the Civil Code, more particularly the second
15. While Carbonell has the superior title to the lot, she must however refund to paragraph thereof which provides that should the thing sold be immovable
respondents Infantes the amount of P1,500.00, which the Infantes paid to the property, the ownership shall belong to the person acquiring it who in good
Republic Savings Bank to redeem the mortgage. with first recorded it in the Registry of property.
- The two purchasers, namely, petitioner Rosario Carbonell and case was decided by the CA, considering that Rosario Carbonell is being
respondent Emma Infante, are both purchasers in good faith. That given the option either to order the removal of the house or to acquire it at
Rosario Carbonell is a buyer in good faith cannot be disputed P13,429.00.
- That Emma Infante is likewise a buyer in good faith is supported by the
express finding of the the lower courts: also must it be remembered that
Emma L. Infante, when she bought the property on 2 February, 1955, neither
had she before then been, preliminary informed of the first sate to Rosario
- Departing from a well-entrenched rule set that factual findings of the trial
court and of the Court of Appeals are generally binding and conclusive, and
that on appeal by certiorari, questions of fact are not to be determined nor
reviewed by Us the Majority Opinion of my colleagues however undertakes
a fact-finding process of its own, and concludes that Emma Infante was a
buyer in bad faith
- The next question to be resolved is who of the two first registered her
purchase or title in good faith. In applying Art. 1544, it is not enough that
the buyer bought the property in good faith, but that the registration of her
title must also be accomplished in good faith. This requirement of good faith
is not only applicable to the second or subsequent purchaser but to the first
as well. Carbonell cannot be held to have a title superior to that of Emma
Infante for even if We were to concede that the notation of her adverse claim
on February 8, 1955, was in the nature of registration of title as required in
Art. 1544, the same was not accomplished in good faith.
- At the time Carbonell caused the annotation of her adverse claim she was
cognizant of facts which impaired her title to the property in question, and
taking advantage of the situation that the second purchaser had not as yet
registered her deed of sale, she went ahead of the second buyer and annotated
what was only in the nature of an adverse claim inasmuch as she had no
registrable document of sale at the time. That annotation of Carbonell's
adverse claim did not produce any legal effects as to place her in a preferential
situation to that of Infante, the second purchaser, for the simple reason that a
registration made in bad faith is equivalent to no registration at all. It is a
settled rule that the inscription in the registry, to be effective, must be made
in good faith.
- The act of the registration of Infante's deed of sale was but a formality in the
sense that it simply formalized what had already been accomplished earlier,
that is, the registration of Infantes purchase as against Carbonell. We have
long accepted the rule that knowledge is equivalent to registration. Infante
also took immediate physical possession of the property by erecting a fence
with a gate around the lot on February 5, at least tree days prior to Carbonell’s
registration.
- On top of all these, equity is on the side of Emma Infante. Under the Majority
Opinion, Emma Infante stands to lose the lot she bought in good faith which
was fully paid for plus the building she erected thereon for which she spent
little less than P14,000.00, or equivalent to about P40,000.00 at the time the
27 CRB v. CA (PAUL) name.
Jan. 5, 2005 | Tinga, J. | Application of Art. 1544 (Double Sale) b. Gamiao and Dayag then sold the southern half to Teodoro Dela
Cruz and the northern half to Restituto Hernandez.
c. Both of them immediately took possession of the land and cultivated
PETITIONER: Consolidated Rural Bank (Cagayan Valley) Inc. it.
RESPONDENTS: The Honorable Court of Appeals, and Heirs of Teodoro Dela d. Restituto Hernandez eventually donated his parcel to his daughter
Cruz Evangeline Del Rosario; and the children of Teodoro dela Cruz
continued the possession of his land after his death.
SUMMARY: Here, Madrid brothers sold the same parcel of land two times. First, 4. Second Sale:
Rizal Madrid, with the concurrence of his other brothers, sold it to Gamiao and a. June 1976: Madrid brothers sold the same lot - 7036-A-7 to Pacifico
Dayag, who subsequently sold it to Teodoro Dela Cruz and Restituto Hernandez. 18 Marquez. This sale was subsequently registered in the Registry of
years after, the brothers sold the same land to Marquez, who later subdivided it, and Deeds in Isabela.
mortgages some of the sub-parcels to Consolidated Rural Bank. When petitioner was b. Marquez then subdivided the lot in to 8 parts. Marquez then
trying to foreclose, Heirs of Teodoro Dela Cruz sought to have the sale to Marquez, mortgaged 4 lots to Consolidated Rural bank (Cagayan Valley)
and the mortgages declared void. RTC: Ruled in favor of Heirs; and the CA ruled in to obtain a loan of Php 100,000; and mortgaged 1 lot to Rural Bank
favor of Marquez. Both courts used Art. 1544. of Cauayan to obtain a loan of Php 10,000. These mortgages were
registered in the Register of Deeds as well.
The SC held that Art. 1544 is not applicable here because the article contemplates c. Marquez defaulted in his payments to both banks sought to foreclose
the sale to 2 or more people by the same vendor. Moreover, that person should have the mortgages.
the right to convey such land. Clearly here, because of the first sale Gamiao and d. Marquez then sold 1 more parcel to Romeo Calixto.
Dayag, the Madrid brothers did not anymore have the right to sell it to Marquez. 5. Because of the foreclosure, Heirs of Dela Cruz wanted to declared the sale to
Moreover, Marquez, and mortgagee banks were not held to be in good faith because Marquez, and the subsequent mortgages null and void.
they ignored the fact that Heirs were in actual possession when the sale and 6. Marquez said that he was a buyer in good faith and for value. Furthermore,
mortgages were executed. he said that the sale to Gamiao and Dayag should not bind him because it was
unregistered.
DOCTRINE: Art. 1544 contemplates a case of double or multiple sales by a single 7. CRB and RBC also argued that they were mortgagees in good faith and they
vendor. Therefore, the conveyance must have been made by a party who has an only relied on Marquez’s titles which were free from any lien or
existing right in the thing and the power to dispose of it. It cannot be invoked encumbrance.
where the different contracts of sale are made by two different persons, one of them 8. RTC: Applied Art. 1544 of the Civil Code which said that if there are two
not being the owner of the property. sales over the same property (immovable), the ownership shall belong to the
person acquiring it in good faith and first records it in the Registry of
Property. They declared Marquez to be the rightful owners. The RTC said
FACTS: that he was a buyer in good faith and there was nothing to show that he had
1. Rizal, Anselmo, Gregorio, Filomeno, and Domingo Madrid (Madrid any knowledge that heirs were claiming ownership.
brothers) were the owners of a parcel of land with Lot. No. 7036-A in Isabela, 9. Heirs then appealed to the CA. CA reversed the RTC. It held that Marquez
covered by a TCT. was not a buyer in good faith because he apparently admitted that he had the
2. This was subdivided into several lots, and one of the resulting lots was Lot. knowledge that the property was “being taken” at the time of sale. Moreover,
No. 7036-A-7. the heirs were actually in possession of the land. Because of these facts, in
3. First Sale: addition to the fact that the property was in an attractive location - National
a. August 1957: Rizal Madrid sold Lot No. 7036-A-7 to Aleja Gamiao Highway + across a gas station - should have put Marquez in inquiry.
and Felisa Dayag evidenced by a deed of sale. The co-owners - Furthermore, mortgagees RBC an CRB relied on Marquez’s certificates of
other brothers - did not object as evidenced by a joint affidavit. This titles but they did not ascertain the status and condition of the property which
deed was not registered in the Registry of Deeds, but both Gamiao is in contravention to the standard banking practice. Thus, they declared the
and Dayag declared the property for taxation purposes under their Heirs to be the rightful owners.
10. CRB argues now that there was no finding that Heirs acted in good faith, nor Even if Art. 1473 (this was the exact replica of Art. 1544 in the Old
did Gamiao and Dayag ever took possession. Furthermore, they argued that Civil Code) does not expressly provide this, this is one of the
Marquez’s claim should prevail because he was the registered owner. circumstances which constitutes an assumption upon which the
article is based.
ISSUE/s: b. In this case, Millante sold the property to Carpio, and subsequently
1. W/N the Heirs are the rightful owners of the land? YES, but not because of sold to Exevea on the same date. The CA and the SC then did not
Art. 1544 which governs double sales. apply Art. 1473 (same as Art. 1544), because Millante did not have
the right anymore over the land.
RULING: WHEREFORE, the Petition is DENIED. The dispositive portion of the 7. In situations where Art. 1544 is not applicable, then the principle of prior
Court of AppealsÊ Decision, as modified by its Resolution dated 5 January 1998, is tempore, potion jure will apply - he who is first in time is preferred in right.
AFFIRMED. Costs against petitioner. a. The only essential requisite of this rule is priority in time - the one
who can invoke this is the first vendee.
RATIO: 8. Clearly, in this case, Gamiao and Dayag who first bought it from Rizal
1. Both RTC and CA tried to use Art. 154419 to justify their decisions. Madrid, are the one who can invoke this. Not only did they buy the property,
2. This is NOT APPLICABLE to this case. Art. 1544 contemplates a case of but they also took possession of it. There is no doubt that they have the
double or multiple sales by a single vendor. Therefore, the conveyance must superior right.
have been made by a party who has an existing right in the thing and the 9. Moreover, as mentioned, one can only sell what one owns or is authorized to
power to dispose of it. sell; and that the buyer can acquire no more than what the seller can legally
3. It cannot be invoked where the different contracts of sale are made by two transfer.
different persons, one of them not being the owner of the property. 10. At the time of the sale to Marquez, the Madrid brothers were no longer the
4. Moreover, even if the second sale was made by the same person when he/she owners of the property, so Marquez did not acquire any right over it.
was no longer the owner, Art. 1544 still cannot be applied because it had 11. Assuming that Art. 1544 is applicable, the Heirs would still have the superior
already been acquired by the first purchaser in full dominion, and therefore, right because Marquez is not a purchaser/registrant in good faith.
the second purchaser cannot acquire any right. 12. In Art. 1544, the rules of preference in a double sale are as follows:
5. Clearly, Art. 1544 is not applicable here. a. The first registrant in good faith;
a. In the first deed of sale (the one talking about the sale to Teodoro b. If there is no entry, the first in possession in good faith; and
Dela Cruz and Restituto Hernandez), the vendors were Gamiao and c. In the absence thereof, the buyer who presents the oldest title in good
Dayag who got their right from Rizal Madrid, in concurrence with faith.
his other brothers in 1957. Gamiao and Dayag also declared the 13. Prior registration does not itself confer ownership or a better right. Art. 1544
property under their names for tax declaration purposes. requires that the second buyer should show that he was in good faith
b. On the other hand, the vendors were the Madrid brothers but they throughout - from the time of acquisition until the title is transferred by
were no longer the owners since they had already disposed it to registration or delivery of possession.
Gamiao and Dayag. 14. As the CA stated, Marquez already knew that the property was being claimed
6. SC cited the case of Carpio v. Exevea: at the time of the sale to him. He also admitted that he did not take possession,
a. The conveyor must have the right and the will to convey the thing. nor did he knows who was in possession then.
Intention to transfer is not sufficient because it only constitutes the 15. The purchaser of real property which is in actual possession of another,
will. If he did not have the right, he could not juridically perform the should make an inquiry with regard to the rights of the actual possessors.
act, because he could not transfer a right which he did not possess. 16. The principle of caveat emptor is applicable here - requires the purchaser to

19
ART. 1544. If the same thing should have been sold to different vendees, the ownership there be no inscription, the ownership shall pertain to the person who in good faith was first in
shall be transferred to the person who may have first taken possession thereof in good faith, if possession; and, in the absence thereof, to the person who presents the oldest title, provided
it should be movable property. Should it be immovable property, the ownership shall belong to there is good faith.
the person acquiring it who in good faith first recorded it in the Registry of Property. Should
be aware of the supposed title of the vender and he who buys without
checking it takes all the risks and losses consequent to such failure.
17. Marquez did not even fight for the possession, and he just willfully closed his
eyes to the possibility of these flaws being true.
18. This rule equally applies to the mortgagees of the property. A mortgagee
cannot just exercise mere refusal to believe that defects exist - this will not
make him a mortgagee in good faith.
19. Banks are impressed with public interest and they are expected to exercise
extraordinary diligence. Petitioner was negligent here because it merely
relied on Marquez’s certificates and it did not ascertain the property’s status
which is in contravention of standard banking procedures.
20. Marquez cannot also argue that just because he had a Torrens Title, means
that he automatically has the superior right. He had actual knowledge that
someone else was possessing the land in the concept of an owner. The
Torrens system cannot be used to shield fraud nor protect an usurper from the
true owner.
21. Here, Teodoro Dela Cruz was the one in good faith - he bought the property
18 years before Marquez did, and he has been taking possession ever since.
22. Moreover, the tax declarations by Gamiao and Dayag were good indications
that possession was in the concept of an owner.
28 SLDC v. CA (Eliel) FACTS:
January 21, 2005 | Tinga, J. | Requisites of Double Sale: Two VALID sales 1. Sps. Miguel and Pacita Lu (Sps. Lu) owned 2 parcels of land in Sta. Rosa
Laguna. Both measuring 15,808 sqms or 3.6 ha. The Sps. Lu purportedly sold
PETITIONER: San Lorenzo Development Corporation the 2 parcels of land to Babasanta, for the price of P15 per sqm.
RESPONDENTS: CA, Pablo Babasanta, Sps. Miguel Lu and Pacita Lu 2. Babasanta made a downpayment of P50,000 as evidenced by a memorandum
receipt issued by Pacita Lu, and several other payments totaling P200,000.
SUMMARY: Sps. Lu purportedly sold 2 parcels of land with the area of 3.6 3. Babasanta wrote a letter to Pacita Lu to demand the execution of a final deed
ha in Laguna to Babasanta at P15 per sqm. Babasanta made a downpayment of sale in his favor so that he could effect full payment of the purchase price.
of P50,000 and several other payments totaling P200,000. Subsequently, In the same letter, Babasanta notified the Sps. Lu about having received
Babasanta demanded that Sps. Lu executed the final deed of sale, and information that the Sps. Lu sold the same property to another without his
notified Sps. Lu about the information he received that the land was being knowledge and consent. He demanded that the second sale be cancelled and
sold to other persons. Sps. Lu replied that she acknowledged the receipt but the that a final deed of sale be issued in his favor.
sale was rescinded when Sps. Lu refused to accept the offer of Babasanta to 4. Pacita Lu wrote a letter to Babasanta wherein she acknowledged having
reduce the price. Babasanta then filed a complaint for specific performance agreed to sell the property to him at P15 per sqm. She, however, reminded
and damages, but Sps. Lu replied that it was only a contract to sell. Babasanta Babasanta that when the balance of the purchase price became due, he
then amended the complaint and prayed for a preliminary injunction to sell the requested for a reduction of the prce and when she refused, Babasanta backed
parcels of land, with which SLDC intervened. SLDC claimed that the parcels out of the sale. Pacita added that she returned the sum of P50,000 to
of land was sold to them evidence by a Absolute Deed of Sale with Mortgage. Babasanta through Oya.
The RTC ruled in their favor but the CA set aside the judgment and ruled in favor 5. Babasanta then filed for a Complaint for Specific Performance and Damages.
of Babasanta because SLDC was a purchaser in bad faith. Hence this petition. The Sps. Lu answered that Pacita Lu obtained loans from Babasanta and
The issue before the court is who between Babasanta and SLDC has a better right when the total advances of Pacita reached P50,000, the latter and Babasanta,
over the parcels of land. without the knowledge and consent of Miguel Lu, had verbally agreed to
transform the transaction into a contract to sell the 2 parcels of land to
The SC ruled in favor of SLDC. First the agreement between Sps. Lu and babasanta with the P50,000 to be considered as the downpayment for the
Babasanta was only a contract to sell, and no transfer of ownership was given property and the balance to be paid on or before 31 December 1987.
to Babasanta. It was conditioned upon the payment of the full purchase price. A 6. Babasanta later filed an Amended Complaint praying for the issuance of a
contract to sell only confers upon the vendee the title but it is not a mode of writ of preliminary injunction to restrain the transfer or conveyance by the
acquiring ownership. The mode of acquiring ownership is through delivery, of Sps. Lu of the subject property to other persons.
which SLDC evidently took constructive possession of the parcels of land 7. SLDC filed a Motion for Intervention and alleged that it had legal interest
evidenced by the Absolute Deed of Sale with Mortgage. And that according to because the 2 parcels of land involved, had been sold to it ina Deed of
Art. 1544 of the Civil Code, if the immovable property is sold to different Absolute Sale with Mortgage. It alleged that it was a buyer in good faith and
vendors, the preference of ownership will be given to the person who for value and therefore it had a better right over the property in litigation.
acquired the property in good faith and first recorded it in the Registry of 8. SLDC averred that the Sps. Lu executed in its favor an Option to Buy the ots
Deeds. In this case, SLDC acquired the property in good faith and indeed subject of the complaint. That it paid an option money in the amount of
recorded the sale of the property as soon as the Deed was executed in their favor. P316,160 out of the total consideration for the purchase of the 2 lots of
Moreover, the SC ruled that the issue on Double sale is moot and academic P1,264,640. After the Sps. Lu received a total amount of P632,320 they
since Art. 1544 does not apply to a contract to sell and a contract of sale. executed a Deed of Absolute Sale with Mortgage in its favor.
9. SLDC added that the certificates of title over the property were delivered to
DOCTRINE: In Dichoso v. Roxas, we had the occasion to rule that Article 1544 it by the spouses clean and free from any adverse claims and/or notice of lis
does not apply to a case where there was a sale to one party of the land itself while pendens.
the other contract was a mere promise to sell the land or at most an actual 10. The RTC ruled in favor of SLDC applying Art 1544 of the Civil Code since
assignment of the right to repurchase the same land. Accordingly, there was no SLDC is the buyer who first acquired possession of the property. The CA
double sale of the same land in that case. however set aside the decision of the RTC and ruled that Babasanta is the
owner because the Deed of Sale with Mortgage of SLDC is null and void
because it is a buyer in bad faith. Hence this petition. Babasanta should have made the proper tender of payment and consignation
of the price in court as required by law.
ISSUES: 7. Glaringly absent from the records is any indication that Babasanta even
1. Who between SLDC and Babasanta has a better right over the 2 parcels of attempted to make the proper consignation of the amounts due, thus, the
land – SLDC obligation on the part of the sellers to convey title never acquired obligatory
force.
RULING: WHEREFORE, the instant petition is hereby GRANTED. The decision 8. On the assumption that the transaction between the parties is a contract
of the Court of Appeals appealed from is REVERSED and SET ASIDE and the of sale and not a contract to sell, Babasanta’s claim of ownership should
decision of the Regional Trial Court, Branch 31, of San Pedro, Laguna is nevertheless fail.
REINSTATED. No costs. 9. Sale, being a consensual contract, is perfected by mere consent and from the
moment, the parties may reciprocally demand performance. The essential
RATIO: elements of a contract of sale, to wit: (1) consent or meeting of the minds,
CONTRACT OF SALE v. CONTRACT TO SELL that is, to transfer ownership in exchange for the price; (2) object certain
1. Contracts, in general, are perfected by mere consent, which is manifested by which is the subject matter of the contract; (3) cause of the obligation which
the meeting of the offer and the acceptance upon the thing which are to is established.
constitute the contract. The offer must be certain and the acceptance absolute 10. Contracts only constitute titles or rights to the transfer or acquisition of
Moreover, contracts shall be obligatory in whatever form they may have been ownership, while delivery or tradition is the mode of accomplishing the same.
entered into, provided all the essential requisites for their validity are present. Therefore, sale by itself does not transfer or affect ownership; the most
2. The receipt signed by Pacita Lu merely states that she accepted the sum of that sale does is to create the obligation to transfer ownership. It is
P50,000 from Babasanta as partial payment of 3.6 hectares of farm lot tradition or delivery, as a consequence of sale, that actually transfers
situated in Laguna. While there is no stipulation that the seller reserves ownership.
the ownership of the property until full payment of the price which is a 11. Following the above disquisition, Babasanta did not acquire ownership by
distinguishing feature of a contract to sell, the subsequent acts of the the mere execution of the receipt by Pacita Lu acknowledging receipt of
parties convince us that the Sps. Lu never intended to transfer ownership partial payment of the property. For one, the agreement between Babasanta
to Babasanta except upon full payment of the purchase price. and the Sps Lu, though valid, was not embodied in a public instrument.
3. In effect, Babasanta himself recognized that ownership of the property Hence, no constructive delivery of the lands could have been effected.
would not be transferred to him until such time as he shall have effected 12. For another, Babasanta had not taken possession of the property at any time
full payment of the price. Moreover, had the sellers intended to transfer title, after the perfection of the sale in his favor or exercised acts of dominion over
they could have easily executed the document of sale in its required form it despite his assertions that he was the rightful owner of the lands.
simultaneously with their acceptance of the partial payment, but they did not. 13. Simply stated, there was no delivery to Babasanta, whether actual or
Doubtlessly, the receipt signed by Pacita Lu should legally be considerd as a constructive, which is essential to transfer ownership of the property.
perfected contract to sell. Thus, even on the assumption that the perfected contract between the
4. The distinction between a contract to sell and a contract of sale is quite parties was a sale, ownership could not have passed to Babasanta in the
germane. In a contract of sale, title passes to the vendee upon the delivery of absence of delivery, since in a contract of sale ownership is transferred
the thing sold; whereas in a contract to sell, by agreement the ownership is to the vendee only upon the delivery of the thing sold.
reserved in the vendor and is not to pass until the full payment of the price.
5. In a contract of sale, the vendor lost and cannot recover ownership until and DOUBLE SALE
unless the contract is resolved or rescinded; whereas in a contract to sell, 14. However, it must be stressed that the juridical relationship between the parties
title is retained by the vendor until the full payment of the price, such in a double sale is primarily governed by Article 1544 which lays down the
payment being a positive suspensive condition and failure of which is not rules of preference between the two purchasers of the same property.
a breach but an event that prevents the obligation of the vendor to 15. Art. 1544. If the same thing should have been sold to different vendees,
convey title from becoming effective. the ownership shall be transferred to the person who may have first
6. The perfected contract to sell imposed upon Babasanta the obligation to pay taken possession thereof in good faith, if it should be movable property.
the balance of the purchase price. There being an obligation to pay the price, 16. Should it be immovable property, the ownership shall belong to the
person acquiring it who in good faith first recorded it in the Registry of faith in contrast to Babasanta, who neither registered nor possessed the
Property. property at any time, SLDC’s right is definitely superior to that of
17. Should there be no inscription, the ownership shall pertain to the person Babasanta’s.
who in good faith was first in the possession; and, in the absence thereof, 26. At any rate, the above discussion on the rules of double sale would be purely
to the person who presents the oldest title, provided there is good faith. academic for as earlier stated in this decision, the contract between Babasanta
18. The principle of primus tempore, potior jure (first in time, stronger in and the Sps. Lu is not a contract of sale but merely a contract to sell.
right) gains greater significance in case of double sale of immovable 27. In Dichoso v. Roxas, we had the occasion to rule that Article 1544 does
property. When the thing sold twice is an immovable, the one who not apply to a case where there was a sale to one party of the land itself
acquires it and first records it in the Registry of Property, both made in while the other contract was a mere promise to sell the land or at most
good faith, shall be deemed the owner. an actual assignment of the right to repurchase the same land.
19. Verily, the act of registration must be coupled with good faith – that is, the Accordingly, there was no double sale of the same land in that case.
registrant must have no knowledge of the defect or lack of title of his
vendor must not have been aware of facts which should have put him
upon such inquiry and investigation as might be necessary to acquaint
him with the defects in the title of his vendor.
20. Admittedly, SLDC registered the sale with the Registry of Deeds after it had
acquired knowledge of Babasanta’s claim. Babasanta, however, strongly
argues that the registration of the sale by SLDC was not sufficient to confer
upon the latter any title to the property since the registration was attended by
bad faith. Specifically, he points out that at the time SLDC registered the sale,
there was already a notice of lis pendens on the file with the Register of
Deeds.
21. It must be stressed that the Sps. Lu executed the Option to Buy in favor of
SLDC upon receiving P316,160 as option money from SLDC. After SLDC
had paid more than one half of the agreed purchase price of P1,264,640, the
Sps. Lu subsequently executed a Deed of Absolute Sale in favor of SLDC.
At the time both deeds were executed, SLDC had no knowledge of the prior
transaction of the Sps. Lu with Babasanta.
22. Simply, from the titme of execution of the first deed up to the moment of
transfer and delivery of possession of the lands to SLDC, it had acted in
good faith and the subsequent annotation of lis pendens has no effect at
all on the consummated sale between SLDC and the Sps. Lu.
23. Assuming ex gratia argumenti that SLDC’s registration of the sale had been
tainted by the prior notice of lis pendens and assuming further for the same
nonce that this is a case of double sale, still Babasanta’s claim could not
prevail over that of SLDC’s.
24. Be it noted that delivery of the property to SLDC was immediately effected
after the execution of the deed in its favor, at which time SLDC had no
knowledge at all of the prior transaction by the Sps. Lu in favor of Babasanta.
25. The law speaks not only of one criterion. The first criterion is priority of
entry in the registry of property there being no priority of such entry,
the second is priority of possession; and, in the absence of two priorities,
the third priority is of the date of title, with good faith as the common
critical element. Since SLDC acquired possession of the property in good
29 ROMAN CATHOLIC CHURCH v PANTE (CHIQUI) the lot as a passageway and that since the spouses Rubi have long been
April 11, 2012 | Brion, J. | First to Possess in Good Faith as Second Priority occupying the 215-square meter lot which included the 32 square meter lot,
then the spouses were the rightful buyers.
PETITIONER: The Roman Catholic Church, represented by Archbishop Caceres 132. RTC: in favor of the Church
RESPONDENTS: Regino Pante a. Church’s consent to the sale was secured through misrepresentation.
RTC annulled the contract between the Church and Pante.
SUMMARY: The Church sold a lot to Pante believing that the latter was an actual b. Also, since the full payment was only made in Sept. 23, 1995, the
occupant. After which, the Church sold to the spouses Rubi the adjacent lot and three-year delay in payment is fatal to Pante’s claim. (Pante
included the lot of Pante. Pante filed an action to annul the subsequent sale. The consigned the balance with the RTC because the Church does not
Church argued that its consent was vitiated when Pante misrepresented that he was want to accept.) RTC upheld the sale between the spouses and the
an actual occupant. W/N Pante is the rightful owner of the subject lot? YES. There Church.
was no misrepresentation. The size of the lot can only be a passageway since it 133. CA: in favor of Pante
was small; it cannot be a place of residence. Thus, it cannot be “actually a. CA characterized the contract as a contract of sale since the Church
occupied”. The Church can also verify it if it did an ocular inspection and this was did not make a reservation of ownership until full payment of the
supported by a sketch plan. Moreover, since both Pante and the spouses Rubi did price is made. As a matter of fact, it even gave the right to repurchase
not register their sale, the lot should be given to the first in possession of the to Pante in case the latter fails to pay the installment. It ruled that
property in good faith. In this case, it was Pante because he was already using it that right is unnecessary of ownership was not transferred to the
as a passageway until he was prevented from doing so. Also, he already placed buyer.
electric connections and water pipes on the lot and this was known to the spouses b. Also, CA applied Art. 1544 (rule on double sales) and stated that
Rubi. since neither of the two sales were registered, the it should be given
to Pante who first possessed the lot by using it as a passageway.
DOCTRINE: After the sale of realty by means of a public instrument, the vendor, 134. Hence, the Church appealed
who resells it to another, does not transmit anything to the second vendee, and if
the latter, by virtue of this second sale takes material possession of the thing, he ISSUE/s:
does it as mere detainer, and it would be unjust to protect this detention against 39. W/N Pante was the rightful owner - YES
the rights of the thing lawfully acquired by the first vendee. (Doctrine in syllabus)
RULING: WHEREFORE, we DENY the petition for review on certiorari, and
AFFIRM the decision of the Court of Appeals dated May 18, 2006, and its resolution
FACTS: dated August 11, 2006, issued in CA-G.R.-CV No. 65069. Costs against the Roman
128. The Church, owned a 32-square meter lot that measure 2x16 meters located Catholic Church.
in Brgy. Dinaga, Canaman, Camarines Sur. On Sept 25, 1992, the Church,
believing that Regino Pante was an actual occupant of the lot, sold it to the RATIO:
latter. The purchase price was at 11, 200 pesos with the intial downpayment 98. No misrepresentation existed vitiating the seller’s consent and invalidating
of 1,120, and the remaining balance payable in 3 years. the contracts.
129. On June 28, 1994, the Church sold the a 215-square meter lot to Nestor and
a. Not every mistake renders a contract voidable. Mistake as to the
Fidela Rubi which included Pante’s lot. Spouses Rubi erected a concrete
identity or qualification of one of the parties will vitiate consent only
fence over Pante’s lot which blocked the latter’s acces from their family home when such identity or qualifications have been the principal cause
to the municipal road. of the contract.
130. Since no settlement could be reached, Pante filed in the RTC an action to b. Contrary to what the Church said, the actual occupancy or residency
annul the sale between the Church and the spouses Rubi regarding the lot of abuyer over the land does not appear to be a necessary
previously sold to him. qualification that the Church requires before it could sell its land.
131. Meanwhile the Church filed a counterclaim, seeking to annul their contract Had this been indeed its policy, then neither Pante nor the spouses
with Pante. The Church stated that its consent was obtained by fraud when Rubi would qualify as buyers of the 32-square meter lot, as none of
Pante misrepresented his actual occupancy when in reality he was only using them actually occupied or resided on the lot. SC notes in this regard
that the lot was only a 2x16-meter strip of rural land used as a i. Actual delivery of a thing sold occurs when it is placed
passageway from Pante's house to the municipal road. under the control and possession of the vendee.
c. SC sides with Pante's argument that, given the size of the lot, it could 1. Pante claimed that he had been using the lot as a
serve no other purpose than as a mere passageway; it is unthinkable passageway, with the Church's permission, since
to consider that a 2x16-meter strip of land could be mistaken as 1963. After purchasing the lot in 1992, he
anyone's residence. In fact, the spouses Rubi were in possession of continued using it as a passageway until he was
the adjacent lot, but they never asserted possession over the 2x16- prevented by the spouses Rubi's concrete fence
meter lot when the 1994 sale was made in their favor. over the lot in 1994. Pante's use of the lot as a
d. It is also unlikely that Pante could successfully misrepresent himself passageway after the 1992 sale in his favor was a
as the actual occupant of the lot; this was a fact that the Church clear assertion of his right of ownership that
(which has a parish chapel in the same barangay where the lot was preceded the spouses Rubi's claim of ownership.
located) could easily verify had it conducted an ocular inspection of Pante also stated that he had placed electric
its own property. The surrounding circumstances actually indicate connections and water pipes on the lot, even
that the Church was aware that Pante was using the lot merely as a before he purchased it in 1992, and the existence
passageway this was supported by the sketch plan, attached to the of these connections and pipes was known to the
contract executed by the Church and Pante, which clearly labeled spouses Rubi. Thus, any assertion of possession
the 2x16-meter lot as a "RIGHT OF WAY"; below these words was over the lot by the spouses Rubi (e.g., the
written the name of "Mr. Regino Pante." construction of a concrete fence) would be
e. There could not have been a deliberate, willful, or fraudulent act considered as made in bad faith because works
committed by Pante that misled the Church into giving its consent had already existed on the lot indicating
to the sale of the subject lot in his favor. The Church was by no possession by another.
means led to believe or do so by Pante's act; there had been no ii. Article 1498. When the sale is made through a public
vitiation of the Church's consent to the sale of the lot to Pante. instrument, the execution thereof shall be equivalent to the
f. From the time the sale to Pante was made and up until it sold the delivery of the thing which is the object of the contract, if
subject property to the spouses Rubi, the Church made no move to from the deed the contrary does not appear or cannot
reject the contract with Pante; it did not even return the down clearly be inferred.
payment he paid. The Church's bad faith in selling the lot to Rubi 1. Under this provision, the sale in favor of Pante
without annulling its contract with Pante negates its claim for would have to be upheld since the contract
damages. executed between the Church and Pante was duly
g. In the absence of any vitiation of consent, the contract between the notarized, converting the deed into a public
Church and Pante stands valid and existing. Any delay by Pante in instrument.
paying the full price could not nullify the contract, since (as d. Under either mode of delivery, Pante acquired prior possession of
correctly observed by the CA) it was a contract of sale. the lot.
99. The rule on double sales
a. Art. 1544 ... Should there be no inscription, the ownership shall
pertain to the person who in good faith was first in the
possession ; and, in the absence thereof, to the person who presents
the oldest title, provided there is good faith. (rule on double sales)
b. As neither Pante nor the spouses Rubi registered the sale in their
favor, the question now is who, between the two, was first in
possession of the property in good faith.
c. Jurisprudence interpreted possession in Article 1544 of the Civil
Code to mean both actual physical delivery and constructive
delivery.
30 Dagupan Trading Company v. Rustico Macam (Valle) the right of redemption. Manila Trading then sold all his rights and title in
May 31, 1965| Dizon,J| Conflicting Sales the property to Dagupan Trading Company.
PETITIONER: Dagupan Trading Company 18. Rustico in his complaint alleged that Sammy maron’s share in the property,
RESPONDENT: Rustico Macam as well as Sammy’s co-heirs, had been acquired by purchase by Rustico
since June 19 and September 21, 1955 before the issuance of the original
SUMMARY: certificate of title. And that at the time that the levy was made on Sammy
What is disputed here is one-eight of a parcel of land that Sammy Maron and his co-heirs Maron’s share, Sammy had no longer any right or interest in the property.
inherited. This land was first unregistered. Pending the approval of the unregistered land, the Rustico also claims that since the sales made in his favor, he had enjoyed
land was sold to Rustico. Rustico then took possession of the land and introduced improvements uninterrupted possession of the property and introduced improvements.
on it. A month later, the Original Certificate of Title was issued in favor of the Marons.
Unfortunately, in another civil case, a decision was rendered against Sammy Maron and in favor ISSUE: Who has better right as between Dagupan and Rustico? - Rustico
of Manila Trading where the property in dispute was levied on and sold at public auction to
Manila Trading. Eventually, Manila Trading sold all his rights and title in the property to
Dagupan. Rustico is now claiming that he is the true owner of the property in question. The RULING: WHEREFORE the decision appealed from is affirmed with costs.
issue here is who has better right over the property. The SC said that Rustico has better right to
the property because at the time that the property was levied on in the separate civil case, the RATIO:
property was no longer Maron’s. It was already in the possession of Rustico who introduced 1. If the property was unregistered land, Rustico would undoubtedly have better right
improvements on the property. To arrive at this decision, the court used Section 35, Rule 39 of in view of the fact that his claim is based on a prior sale coupled with public, exclusive
the ROC. The court said that when first sale is over unregistered land and the second sale is and continuous possession.
when it is registered, the rules on double sale do not apply. 2. on the other hand, were the land involved in the conflict is a registered land,
Dagupan will have a better right because in case of conveyance of registered real
DOCTRINE:
estate, the registration of the deed of sale is the operative act that gives validity to the
When first sale is over unregistered land and the second sale is when it is registered, the rules
on double sale do not apply.
transfer. The deeds of sale executed in Rustico’s favor by the Marons not having been
registered while the levy and the final deed of sale in favor of Dagupan were registered.
FACTS:
3. The present case, however, doesn’t fall within either situation.
12. Dagupan Trading Company prays to be declared owner of a one-eighth
4. In this case, the sale in favor of Rustico was executed before the land subject matter
portion of a piece of land, that a partition of the whole property be made,
was registered, while the conflicting sale in favor of appellant was executed after the
and that Rustico be ordered to pay an amount of Php 500 a year as damages
same property had been registered. The Courts cannot therefore, decide the case in
from 1958 until the said portion be delivered, plus attorney’s fees.
light of whatever adjudicated cases there are covering the two situations.
13. In 1955, Sammy Maron and his seven siblings were pro-indiviso owners of
5. The Court determines the issue using the provisions of the last paragraph of Section
a parcel of unregistered land located in barrio Parayao, Municipality of
35, Rule 39 of the Rules of Court, to the effect that upon the execution and delivery of
Binmaley, Pangasinan.
the final certificate of sale in favor of the purchaser of land sold in an execution sale,
14. While their application for registration of land was pending, they executed,
such purchaser shall be substituted to and acquire all the right, title, interest, and claim
two deeds of sale conveying the property to Rustico. Rustico thereafter took
of the judgment debtor to the property as of the time of the levy.
possession of the land and proceeded to introduce substantial
6. Now the court asks what is the interest and claim of Sammy Maron on the one-eight
improvements.
portion of the property inherited by and his co-heirs at the time of the levy? He had
15. A month later, the original certificate of title covering the land was issued in
none. Because prior to the levy, his interest had already been conveyed to Rustico fully
favor of the Maron family, free of all liens and encumbrances.
and irretrievably. Consequently, subsequent levy made on the prpoperty for the
16. On August 4, 1956, by virtue of a final judgement rendered in Civil Case
purpose of satisfying judgment rendered against Sammy in favor of Manila Trading
No. 42215 against Sammy Maron and in favor of Manila Trading and
was void and of no effect.
Supply Company. Levy was made on whatever interest he had in the
7. Upon the execution of the deed of sale in favor of Rustico, Rustico already took
property. Said interest on the property was sold at public auction to Manila
possession of the land as owner and introduced considerable improvements. To
Trading.
deprive him now of the same by sheer force of technicality would be against both
17. The notice of levy, certificate of sale and the sheriff’s certificate of final
justice and equity.
sale were registered in favor of Manila Trading because nobody exercised
31 CARUMBA vs CA (Nikki V) 6. CA: reversed the decision of the CFI. There having been a double sale of the land,
1970 February 18 | J. Reyes J.B.L. | G.R. No. L-27587 | Balbuena’s title was superior to that of Carumba’s under Art. 1544 of the Civil
When subject of sale is unregistered land Code since the execution sale had been properly registered in good faith and the
PETITIONER: Amado Carumba sale to Carumba was not recorded.
RESPONDENTS: The Court of Appeals, Santiago Balbuena, and Angeles Boaquiña
as Deputy Provincial Sheriff ISSUE: Whether or not the CA erred in ruling that Balbuena has superior title to the
SUMMARY: In 1955, Sps. Canuto sold a parcel of land to Sps. Carumba as evidenced subject land – YES. CFI was correct in holding Carumba as the owner.
by a Deed of Sale of Unregistered Land. Said deed was not registered in the RD of
CamSur. In 1957, Balbuena filed a complaint for a sum of money against Sps. Canuto. RULING: WHEREFORE, the decision of the Court of Appeals is reversed and that
Judgment was rendered in favor of Balbuena. By virtue of said judgment, sheriff of the Court of First Instance affirmed. Costs against Santiago Balbuena.
issued a Definite Deed of Sale of the property now in question. This instrument was
registered in the RD in 1958. The question is: Who has superior title to the land – Sps. RATIO:
Carumba or Balbuena? SC declared Carumba as the owner of the land. When the levy 1. While under Art. 1544 registration in good faith prevails over possession in the
was made by the sheriff20, Sps. Canuto no longer had dominical interest nor any real event of a double sale by the vendor of the same piece of land to different vendees,
right over the land that could pass to Balbuena. said article does not apply in this case even if Balbuena, the later vendee, was
DOCTRINE: Article 1544 is inapplicable to unregistered land because “the purchaser ignorant of the prior sale made by his judgment debtor in favor of Carumba.
of unregistered land at a sheriff’s execution sale only steps into the shoes of the 2. The reason is that the purchaser of unregistered land at a sheriff’s execution sale
judgment debtor, and merely acquires the latter’s interest in the property sold as of the (Balbuena) only steps into the shoes of the judgment debtor (Canuto), and merely
time the property was levied upon,” as expressly provided for in then Sec. 35, Rule 39 acquires the latter’s interest in the property sold as of the time the property was
of the Revised Rules of Court on execution of sale [now Sec. 33, Rule 39, 1997 Rules levied upon. This is specifically provided by Section 3521 of Rule 39 of the Revised
of Civil Procedure]. Rules of Court.
FACTS: 3. While the time of the levy does not clearly appear, it could not have been made
1. 1955: Sps. Canuto, by virtue of a “Deed of Sale of Unregistered Land with prior to 1957, when the decision against Sps. Canuto was rendered.
Covenants of Warranty” sold a parcel of land (partly residential, partly coconut 4. The Deed of Sale of Unregistered Land with Covenants of Warranty had been
land) to Sps. Carumba. The referred deed of sale was never registered in the Office executed two years before, in 1955, and while only embodied in a private
of the Register of Deeds and the Notary was not then an authorized notary public. document, the same, coupled with the fact that the buyer (Sps. Carumba) had taken
*Carumba (buyer) is the brother-in law of Canuto (seller)* possession of the unregistered land sold, sufficed to vest ownership on the said
2. 1957: A complaint for a sum of money was filed by a certain Balbuena against Sps. buyer. When the levy was made by the Sheriff, therefore, the judgment debtor (Sps.
Canuto. In the said case, judgment was rendered in favor of Balbuena and against Canuto) no longer had dominical interest nor any real right over the land that could
the Sps. Canuto. pass to the purchaser at the execution sale. Hence, the Balbuena must yield the land
3. 1968: The sheriff issued a “Definite Deed of Sale” of the property now in question to Carumba.
in favor of Balbuena, which instrument of sale was registered before the Office of 5. Said rule is different in case of lands covered by Torrens titles, where the prior sale
the Register of Deeds in 1958. Said property was declared for taxation purposes in is neither recorded nor known to the execution purchaser prior to the levy; but the
the name of Balbuena in 1958. land here in question is admittedly not registered under Act No. 496.
4. The issue now arises as to who has superior title to the said land: Sps. Carumba or
Balbuena?
5. CFI: ruled in favor of Sps. Carumba and declared him to be the owner of the land
under a consummated sale. After execution of the document, Carumba had taken
possession of the land planting bananas, coffee, and other vegetables thereon.
Moreover, the CFI also held void the execution levy made by the sheriff and
nullified the sale in favor of Balbuena.

20 21
Time of levy: not stated in the case. SC only said – malamang mga 1957 onwards because it was only in 1957 when Balbuena filed a Upon the execution and delivery of said (final) deed the purchaser, redemptioner, or his assignee shall be substituted to and acquire all the
complaint. During that time, Canuto could not have transferred any real right to Balbuena because he does not own the said land anymore. 1955 right, title, interest, and claim of the judgment debtor to the property as of the time of the levy, except as against the judgment debtor in
- Canuto already sold the land to Carumba. possession, in which case the substitution shall be effective as of the time of the deed.
32 ABRIGO v. DE VERA (JP) Rosita shall remain valid and Gloria should vacate the property
June 21, 2004 | Panganiban, J. | When Subject of Sale is Unregistered Land without the need of demand.
138. Gloria failed to buy back the house and lot, so Rosenda & Rosita declared the
PETITIONER: Spouses Noel and Julie Abrigo lot in their name.
RESPONDENTS: Romana de Vera 139. Unknown to Rosenda & Rosita, Gloria obtained a free patent over the parcel
of land. Such patent was however cancelled by TCT No. 212598.
SUMMARY: Gloria sold a house and lot to Rosenda & Rosita. Rosenda & Rosita 140. Rosenda and Rosita sold the house & lot to herein petitioner Spouses Noel
then sold this land to Spouses Abrigo. Gloria however, likewise sold the same and Abrigo (Spouses Abrigo).
property to De Vera. De Vera registered the property and a TCT was issued in her 141. Gloria on the other hand, likewise sold the same house and lot to Respondent
name. Sps. Abrigo not knowing that the property as under the Torrens Title, they Romana de Vera (de Vera).
only registered the land under Act 3344. He relied on the TCT presented by Gloria 142. Personal Note: Memorize the parties from this point. Here an illustration:
that she was the owner of the property. Sps. Abrigo then filed a case for annulment a. Gloria -> (sold to) Rosenda and Rosita
of documents and preliminary injunction against Gloria and De Vera. RTC and b. Rosenda and Rosita -> Sps. Abrigo
CA ruled that De Vera was a purchaser in good faith and was the rightful owner c. Gloria -> de Vera
of the land. d. All involve the same property
143. De Vera registered the sale and TCT No. 22515 was issued in her name.
The SC affirmed. The Torrens Title of De Vera should prevail over registration 144. De Vera filed an action for Forcible Entry and Damages against Sps. Abrigo
under Act 3344 by Sps. Abrigo because the latter is not sufficient registration before the MTC.
required by the law. De Vera is also a buyer in good faith because she had no 145. The parties submitted a Motion for Dismissal in view of their agreement in
knowledge of the controversy of the property. Her reliance on the TCT of De Vera the instant case that neither of them can physically take possession of the
was already sufficient as a TCT proves an indefeasible title. property in question until the instant case is terminated. Hence the ejectment
case was dismissed.
DOCTRINE: The law provides that a double sale of immovables transfers 146. Sps. Abrigo filed the instant case for annulment of documents, injunction,
ownership to (1) the first registrant in good faith; (2) then, the first possessor in preliminary injunction, restraining order and damages against De Vera and
good faith; and (3) finally, the buyer who in good faith presents the oldest title. Gloria.
This principle is in full accord with Section 51 of PD 1529 14 which provides 147. RTC ruled that De Vera was the rightful owner and with better right to
that no deed, mortgage, lease or other voluntary instrument — except a will — possess the property, being an innocent purchaser. Gloria was ordered to pay
purporting to convey or affect registered land shall take effect as a conveyance de Vera and Sps. Abrigo for damages.
or bind the land until its registration. Thus, if the sale is not registered, it is 148. CA ruled:
binding only between the seller and the buyer but it does not affect innocent a. On its first decision: a void title could not give rise to a valid one
third persons. and hence ruled that since Gloria had already transferred ownership
to Rosenda and Rosita, the subsequent sale to De Vera was deemed
Constructive notice to the second buyer through registration under Act 3344 does void.
not apply if the property is registered under the Torrens system. b. On reconsideration: CA amended its decision and now found De
Vera to be a purchaser in good faith and for value.
149. Sps. Abrigo contend that Gloria could not have transferred the property to
FACTS: Respondent De Vera because it no longer belonged to her. They further claim
135. Gloria Villafania (Gloria) sold a house and lot located in Pangasinan to that the sale could not be validated, since respondent De Vera was not a
Rosenda Tigno-Salazar and Rosita Cave-Go (Rosenda & Rosita). purchaser in good faith and for value.
136. The sale became a subject of a suit for annulment of documents.
137. The RTC approved a Compromise Agreement submitted by the parties. ISSUE/s:
a. Gloria was given one year from the date of the Compromise 40. WoN the deed of sale executed by Gloria in favor of De Vera is valid – YES
Agreement to buy back the property. 41. WoN De Vera is a purchaser for value in good faith – YES
b. Failure to do so with mean the previous sale in favor of Rosenda & 42. WoN De Vera has a better title over Sps. Abrigo - YES
3344. De Vera relies on the following insight of Justice Edgardo L. Paras:
RULING: Judgment in question is affirmed. a. “. . . If the land is registered under the Land Registration Act (and
has therefore a Torrens Title), and it is sold but the subsequent sale
RATIO: is registered not under the Land Registration Act but under Act
100. The present case involves what in legal contemplation was a double sale. 3344, as amended, such sale is not considered REGISTERED…”
On May 27, 1993, Gloria Villafania first sold the disputed property to 110. The court agrees with respondent De Vera.
Rosenda Tigno-Salazar and Rosita Cave-Go, from whom petitioners, in turn, 111. It is undisputed that Gloria had been issued a free patent registered as
derived their right. Subsequently, on October 23, 1997, a second sale was Original Certificate of Title (OCT) No. P-30522. The OCT was later
executed by Villafania with Respondent Romana de Vera. cancelled by Transfer Certificate of Title (TCT) No. 212598, also in
101. Article 1544 of the Civil Code states the law on double sale thus: Gloria’s name. As a consequence of the sale, TCT No. 212598 was
a. If the same thing should have been sold to different vendees, the subsequently cancelled and TCT No. 22515 thereafter issued to respondent
ownership shall be transferred to the person who may have first De Vera.
taken possession thereof in good faith, if it should be movable 112. Soriano v. Heirs of Magali: registration must be done in the proper
property. registry in order to bind the land. Since the property in dispute in the
b. Should it be immovable property, the ownership shall belong to the present case was already registered under the Torrens system,
person acquiring it who in good faith first recorded it in the petitioners’ registration of the sale under Act 3344 was not effective for
Registry of Property. purposes of Article 1544 of the Civil Code.
c. Should there be no inscription, the ownership shall pertain to the 113. Inasmuch as the registration of the sale to Respondent De Vera under
person who in good faith was first in the possession; and, in the the Torrens system was done in good faith, this sale must be upheld over
absence thereof, to the person who presents the oldest title, provided the sale registered under Act 3344 to Petitioner-Spouses Abrigo.
there is good faith. 114. Radiowealth Finance Co. v. Palileo explained the difference in the rules of
102. Otherwise stated, the law provides that a double sale of immovables registration under Act 3344 and those under the Torrens system in this wise:
transfers ownership to (1) the first registrant in good faith; (2) then, the a. Under Act No. 3344, registration of instruments affecting
first possessor in good faith; and (3) finally, the buyer who in good faith unregistered lands is ‘without prejudice to a third party with a
presents the oldest title. better right.’
103. There is no ambiguity in the application of this law with respect to b. The aforequoted phrase has been held by this Court to mean that the
lands registered under the Torrens system. mere registration of a sale in one’s favor does not give him any right
104. This principle is in full accord with Section 51 of PD 1529 14 which over the land if the vendor was not anymore the owner of the land
provides that no deed, mortgage, lease or other voluntary instrument having previously sold the same to somebody else even if the earlier
— except a will — purporting to convey or affect registered land shall sale was unrecorded.
take effect as a conveyance or bind the land until its registration. c. The purchaser of unregistered land at a sheriff’s execution sale only
105. Thus, if the sale is not registered, it is binding only between the seller steps into the shoes of the judgment debtor, and merely acquires the
and the buyer but it does not affect innocent third persons. latter’s interest in the property sold as of the time the property was
106. The above provision does not apply if the property is not registered levied upon.
under the Torrens system. 115. Sps. Abrigo cannot validly argue that they were fraudulently misled
107. In the instant case, both Sps. Abrigo and respondent De Vera registered into believing that the property was unregistered. A Torrens title, once
the sale of the property. Since neither Sps. Abrigo nor their predecessors registered, serves as a notice to the whole world. All persons must take
(Tigno-Salazar and Cave-Go) knew that the property was covered by the notice, and no one can plead ignorance of the registration.
Torrens system, they registered their respective sales under Act 3344. 116. Under Uraca v. Court of Appeals:
108. For her part, respondent De Vera registered the transaction under the Torrens a. The prior registration of the disputed property by the second buyer
system because, during the sale, Gloria had presented the transfer certificate does not by itself confer ownership or a better right over the
of title (TCT) covering the property. property. Article 1544 requires that such registration must be
109. Respondent De Vera contends that her registration under the Torrens coupled with good faith.
system should prevail over that of petitioners who recorded theirs under Act b. Jurisprudence teaches us that ‘(t)he governing principle is primus
tempore, potior jure (first in time, stronger in right). a. On lands covered by the Torrens System, the purchaser
c. Knowledge gained by the first buyer of the second sale cannot acquires such rights and interest as they appear in the
defeat the first buyer’s rights except where the second buyer certificate of title, unaffected by any prior lien or encumbrance
registers in good faith the second sale ahead of the first, as not noted therein. The purchaser is not required to explore
provided by the Civil Code. farther than what the Torrens title, upon its face, indicates.
d. Before the second buyer can obtain priority over the first, he The only exception is where the purchaser has actual
must show that he acted in good faith throughout (i.e. in knowledge of a flaw or defect in the title of the seller or of such
ignorance of the first sale and of the first buyer’s rights) — liens or encumbrances which, as to him, is equivalent to
from the time of acquisition until the title is transferred to him registration.
by registration, or failing registration, by delivery of 124. The CA examined the facts to determine whether respondent was innocent
possession.’ purchaser for value.
117. Equally important, under Section 44 of PD 1529, every registered 125. Gloria Villafania, [Respondent] De Vera’s vendor, appears to be the
owner receiving a certificate of title pursuant to a decree of registered owner. The subject land was, and still is, registered in the
registration, and every subsequent purchaser of registered land taking name of Gloria Villafania. There is nothing in her certificate of title and
such certificate for value and in good faith shall hold the same free in the circumstances of the transaction or sale which warrant
from all encumbrances, except those noted and enumerated in the [Respondent] De Vera in supposing that she needed to look beyond the
certificate. title. She had no notice of the earlier sale of the land to Sps. Abrigo.
118. Thus, a person dealing with registered land is not required to go behind the 126. She ascertained and verified that her vendor was the sole owner and in
registry to determine the condition of the property, since such condition is possession of the subject property by examining her vendor’s title in
noted on the face of the register or certificate of title. the Registry of Deeds and actually going to the premises.
119. Following this principle, this Court has consistently held as regards 127. There is no evidence in the record showing that when she bought the land
registered land that a purchaser in good faith acquires a good title as on October 23, 1997, she knew or had the slightest notice that the same was
against all the transferees thereof whose rights are not recorded in the under litigation. She was not even a party to the said case.
Registry of Deeds at the time of the sale. 128. In sum, she testified clearly and positively, without any contrary
120. Citing Santiago v. Court of Appeals, petitioners Sps. Abrigo contend that evidence presented by Sps. Abrigo, that she did not know anything
their prior registration under Act 3344 is constructive notice to respondent about the earlier sale until after she had bought the same, and only
De Vera and negates her good faith at the time she registered the sale. then when she brought an ejectment case with MTC.
a. Santiago v. CA: On account of the undisputed fact of registration 129. To the [Respondent] De Vera, the only legal truth upon which she had
under Act No. 3344 by [the first buyers], necessarily, there is to rely was that the land is registered in the name of Gloria Villafania,
absent good faith in the registration of the sale by the [second her vendor, and that her title under the law, is absolute and
buyers] for which they had been issued certificates of title in their indefeasible.
names. 130. Spouses Abrigo base their position only on the general averment that
121. SC held that Santiago and Bayoca (the cases where Sps. Abrigo relied) are respondent should have been more vigilant prior to consummating the sale.
not in point. In Santiago, the first buyers registered the sale under the They argue that had she inspected the property, she would have found
Torrens system, as can be inferred from the issuance of the TCT in their petitioners to be in possession.
names. There was no registration under Act 3344. In Bayoca, when the first 131. This argument is contradicted, however, by the spouses’ own admission that
buyer registered the sale under Act 3344, the property was still unregistered the parents and the sister of Villafania were still the actual occupants in
land. Such registration was therefore considered effectual. when Respondent De Vera purchased the property. The family members
122. Constructive notice to the second buyer through registration under Act may reasonably be assumed to be Villafania’s agents.
3344 does not apply if the property is registered under the Torrens
system, as in this case.
123. The SC quotes below the additional commentary of Justice Vitug, which
was omitted in Santiago. This omission was evidently the reason why
petitioner misunderstood the context of the citation therein:
33 ACABAL v. ACABAL (HENRY) kay sir)
March 31, 2005 | Carpio-Morales, J. | Subject of sale: Unregistered land 1. Villaner’s parents, Alejandro Acabal and Felicidad Balasbas, owned a parcel
of land situated in Negros Oriental (18.15 hectares). By a Deed of Absolute
PETITIONER: Leonardo Acabal and Ramon Nicolas Sale (DAS) on July 6, 1971, his parents transferred for Php 2,000.00
RESPONDENT: Villaner Acabal, Eduardo Acabal, Grace Acabal, et al. ownership of the said land to him, who was then married to Justiniana Lipajan
2. Sometime after the transfer, Villaner became a widower.
SUMMARY: This case is basically about the dispute as to Villaner’s conveyance 3. April 19, 1990, Villaner executed a deed conveying the same property in
of the property to petitioner Leonardo, who sold it to Nicolas. Villaner claims that favor of Leonardo Acabal.
it was only a lease contract, while the adverse party claims it’s in fact a sale. 4. Villaner later claims that while deed executed on April 1990 now appears to
be a DAS, witnessed by court clerk Carmelo Cadalin and wife Lacorte, what
The case showed the roots of the ownership of the land. First it was owned by he signed was actually a document captioned “Lease Contract” wherein he
Villaner’s parents, who transferred it to him, when he was married to Justiniana. leased for 3 years the property to Leonardo at Php 1,000/hectare and which
He became a widower, and sometime around April 19, 1990, he conveyed the was witnessed by 2 women employees of one Judge Villegas (in other words,
land to Leonardo (here’s where the problem all began). With claims that the yung document daw nung April 19, 1990, lease contract daw yun, not deed
conveyance was only a lease, Villaner came to RTC and filed a case, to which it of absolute sale, accdg to Villaner)
ruled against his favor. Upon his appeal to the CA, it was reversed. Hence, an 5. Villaner filed a complaint before the RTC against Leonardo and Ramon
appeal by petitioners in the SC. Nicolas, to whom Leonardo in turn conveyed the property, for annulment of
deeds of sale
SC after explaining lots of stuff regarding the issue as to whether it was a sale or 6. At the witness stand, Villaner basically claims that the agreement he signed
not finally ruled that it was indeed a sale. Hence, Nicolas bought the land from was that of a contract of lease and not a sale. When asked why he did not read
Leonardo in good faith with assumption that it was indeed Leonardo who owned the document, he said he read the title that said that the document was a Lease
the land. BUT Villaner was persistent, and argued that, such “sale” was contrary Contract.
to law and shall be void, as it was against the Comprehensive Agrarian Reform 7. On the other side of the boat, Leonardo asserts that such document was a
Law. For this issue, SC held that there was no violation, as the land in question DAS, which was for a consideration of Php 10,000.00 which he already paid
was not within the ambit of the statute. Even assuming that there was a violation, and that, he is the absolute owner thereof, and transferred it to Ramon Nicolas
both are in pari delicto, and cannot seek recourse in the courts. on May 19, 1990.
8. Carmelo Cadalin who admittedly prepared the DAS corroborated with
Ultimately, with the land being unregistered, it puts the buying of the land in good Leonardo’s claims.
faith as an irrelevant factor. Such issue is only relevant when the subject of the 9. Later, the complaint was amended to implead Villaner’s 8 children as party
sale is registered. One who purchases an unregisterd land does so at his peril. plaintiffs, they being heirs of his diseased wife.
Nicolas’ claim of having bought the land in good faith is thus irrelevant. 10. RTC ruled in favor of Leonardo and Nicolas, and dismissed the complaint.
(so RTC said, sale daw yun)
With this, SC held that such was in fact a sale, which essentially puts the sale to 11. Villaner et al brought the case on appeal to the CA, which reversed the RTC
Nicolas valid as well, but only for the 5/9 part. ruling, holding that the DAS was fictitious. (CA said naman, na it was a
lease, not a sale)
DOCTRINE: Under Act 3344, registration of instruments affecting unregistered 12. Hence, this petition.
lands is “without prejudice to a third party with a better right,” which means that
mere registration does not give buyer any right over the land if seller was not ISSUES:
anymore owner thereof, having previously sold it to somebody else even if the 1. WoN Nicolas’ claim of having bought the land in good faith is relevant – NO.
earlier sale was unrecorded. The rules on double sale have no application to land panalo parin kahit hindi applicable yung good faith claims
not registered under the Torrens System.
RULING: SC granted the petition. Sale in favor of Leonardo Acabal and subsequent
FACTS: (bear with me, sabog facts ng case, and minimal discussion of the sale in favor of Ramon Nicolas is valid, but only insofar as the 5/9 of the subject
doctrine on unregistered property. I expounded some stuff para madami masabi property is concerned.
conjugal, hence Art 160 of the CC will apply.
RATIO: 11. The property being conjugal, upon death of his spouse, such partnership was
1. On the burden of proof: It is a basic rule in evidence that the burden of proof terminated. With its dissolution, Villaner’s interest in the partnership became
lies on party who makes allegations. If he claims a right granted by law, he actual and vested with respect to an undivided one-half portion. The other
must prove it by competent evidence, relying on the strength of his own half is vested upon her death to her heirs, including Villaner. Hence, an
evidence and not upon weakness of his opponent existence of a regime of co-ownership
2. Allegations of defect in or lack of valid consent to a contract by reason of 12. While Villaner owns 5/9 of the property, he could not claim title to any
fraud or undue influence are NEVER presumed, it must be established by definite portion until its actual partition by agreement or judicial degree. Prior
clear and convincing evidence to partition, all that he has is an ideal abstract quota or proportionate share.
3. Whether the document was a DAS or that of for lease: In the present case, it He may sell his undivided share though, with reference to laws on Property
was incumbent on Villaner to prove that he was deceived into executing the 13. However in this case, Villaner sold the entire property without obtaining
DAS. Except for his bare allegation that the transaction was that of a lease, consent of co-owners (his heirs). Following the well-established principle
he failed to adduce evidence in support thereof. On his claim that two women that the binding vorce of a contract must be recognized as far as it is legally
employees signed as witnesses but that the signatures were not of those two, possible to do so, the disposition affects only Villaner’s share, and the
it must be discredited. transferee only gets what corresponds to his grantor’s share in the partition
4. On the purchase price: Villaner zeroes in the price of Php 10,000.00, which of property owned in common
to him was unusually low if the transaction were one of sale. He even 14. The Court cited Cruz v. Leis, where it held that as a rule, a co-owner could
presented Tax Declarations covering the property for years (1971, 1974, only dispose of her share in the property owned in common.
1977, 1980, 1983, 1985, 1985) to substantiate his claim. However, he failed 15. But, such doctrine is not applicable in the case at bar for the simple reason
to present evidence on the fair market value of the property as of April 19, that the property in dispute is unregistered.
1990. Without this proof, there can be no substance as to claims of 16. Issue of good faith or bad faith of a buyer is relevant only where the subject
inadequacy of price of the sale is a registered land but not where the property is an unregistered
5. Victor Ragay, a staff appointed by RTC to inspect the property, even reported land.
that one Anatolio Cabusog purchased the 6-hectare property adjoining that of 17. One who purchases an unregistered land does so at his peril. Accordingly,
the subject property for only Php 1,600.00 or Php 2.66/hectare. Given that, Nicolas’ claim of having bought the land in good faith is irrelevant.
the 18-hectare subject property been sold at about that same time, would have
fetched the amount of Php 4,800.00, which leads the SC to say that the 10k
purchase price of the property is more than in fact reasonable
6. Even assuming that the price of 10k was below fair market value, mere
inadequacy of price will not rule out the transaction as one of sale.
7. SC likewise held that as far as RA 6657 (Comprehensive Agrarian Reform
Law), subject property is not suitable for agricultural purposes (baka lang
itanong ni sir)
8. On the sale of Leonardo to Nicolas: There was also an argument as to the sale
of Leonardo to Nicolas being void for being violative of the retention limits
provided for by RA 6657 (Comprehensive Agrarian Reform Law), the SC
held that there was no such violation.
9. Even assuming that the sale from Villaner to Leonardo was contrary to law,
he has no remedy as he and Leonardo were in pari delicto, hence, not entitled
to affirmative relief.
10. On relevant topic: The subject property was acquired on July 6, 1971 during
Villaner’s marriage with Justiniana. It cannot be seriously contended that
simply because the tax declarations covering the property was solely in the
name of Villaner, that it is his personal and exclusive property. It is, by nature,
01 MINDANAO v. YAP (Salve) D. Yap.
February 26, 1965 | Makalintal, J. | Co-owners 14. Included in the sale were certain buildings situated on said lands as well as
PETITIONER: Mindanao Academy Inc., Mauricio Bas, Erlinda Diaz, Rosenda laboratory equipment, books, furniture and fixtures used by two schools
De Niqui, Sotero Dionisio Jr., Antolin Diaz established in the respective properties, the Mindanao Academy in Oroquieta
RESPONDENTS: Ildefonso Yap and the Misamis Academy in Ozamis City.
15. The price was P100,700.00, to be paid according to the terms and conditions
SUMMARY: Rosena de Nuqui and her son Sotero Dionisio Jr. sold parcels of specified in the contract.
land and buildings and other improvements on the said land to Ildefonso Yap. 16. Adelaida Dionisio-Nuesa (a daughter of Rosenda) is also named therein as
However, there were other co-owners of the land not named in the deed of sale. co-vendor, but actually did not take part.
Ildefonso Yap obtained possession of the properties. Thus, the said co-owners 17. Rosenda and her two children are referred to in the deed as the owners pro-
brought actions for annulment of sale and rescission. Yap contends that the lower indiviso of the properties sold. The truth, however, was that there were other
court erred "in declaring that the mutual agreement dated May 10, 1954 ... is co-owners of the lands, namely, Erlinda D. Diaz, Ester Aida D. Bas,
entirely void and legally non-existent in that the vendors therein ceded to Rosalinda D. Belleza, and Luz Minda D. Dajao, children also of Rosenda.
defendant-appellant not only their interests, rights, shares and participation in the 18. The school building, equipment, books, furniture and fixtures were owned by
property sold but also those that belonged to persons who were not parties the Mindanao Academy, Inc.
thereto." He also contends that Erlinda Diaz is in no position to ask for annulment 19. The buyer, Ildefonso D. Yap, obtained possession of the properties by virtue
because of her bad faith. WoN the contract of sale is void for failing to state all of the sale, took over the operation of the two schools and even changed their
the owners of the property? YES. WoN bad faith (if ever she was in bad faith) on names to Harvardian Colleges.
the part of Erlinda will militate her action for nullity? NO. The contract is void 20. Two actions were commenced in the Court of First Instance of Misamis
because the contract was not made by all the owners of the property. Bad faith on Occidental.
the part of Erlinda would not militate against the nullity of the sale, considering 21. The first was for annulment of the sale and recovery of rents and damages
that it included not only the lands owned in common by Rosenda Nuqui and her (Civil Case No. 1774, filed May 3, 1955) with the Mindanao Academy, Inc.,
six children but also the buildings and school facilities owned by the Mindanao the five children of Rosenda Nuqui who did not take part in the deed of sale,
Academy, Inc., an entity which had nothing to do with the transaction and which and several other persons who were stockholders of the said corporation, as
could be represented solely by its Board of Trustees. plaintiffs, and the parties who signed the deed of sale as defendants.
22. The second action was for rescission (Civil Case No. 1907, filed July 17,
DOCTRINE: 1956) with Rosenda Nuqui, Sotero Dionisio, Jr. and Erlinda D. Diaz (and the
The contract is void because latter's husband Antolin Diaz) as plaintiffs, and Ildefonso D. Yap as lone
defendant. The other four children of Rosenda did not join, having previously
(a) the contract purported to sell properties of which the sellers were ceded and quitclaimed their shares in the litigated properties in favor of their
not the only owners, since of the four parcels of land mentioned in sister Erlinda D. Diaz.
the deed their shares consisted only of 7/12, (6/12 for Rosenda 23. Ildefonso D. Yap appealed from the judgment of the CFI.
Nuqui and 1/12 for Sotero, Jr.), while in the buildings, laboratory 24. He contends that the lower court erred "in declaring that the mutual
equipment, books, furniture and fixtures they had no participation agreement dated May 10, 1954 ... is entirely void and legally non-existent in
at all, the owner being the Mindanao Academy, Inc.; that the vendors therein ceded to defendant-appellant not only their interests,
(b) the prestation involved in the sale was indivisible, and therefore rights, shares and participation in the property sold but also those that
incapable of partial annulment, inasmuch as the buyer Yap, by his belonged to persons who were not parties thereto."
own admission, would not have entered into the transaction except 25. Yap contends that Rosenda Nuqui who did not take part in the sale, besides
to acquire all of the properties purchased by him. Erlinda Dionisio Diaz, quitclaimed in favor of the latter their interests in the
properties; and that the trial court held that Erlinda as well as her husband
acted in bad faith, because "having reasonable notice of defendants' having
FACTS: unlawfully taken possession of the property, they failed to make reasonable
13. Rosenda A. de Nuqui (widow of deceased Sotero Dionisio) and her son demands for (him) to vacate the premises to respect their rights thereto." It is
Sotero Dionisio, Jr. sold three parcels of residential land in Oroquieta, argued that being herself guilty of bad faith, Erlinda D. Diaz, as owner of
Misamis Occidental, and another parcel in Ozamis City in favor of Ildefonso 5/12 undivided interest in the properties (including the 4/12 ceded to her by
her four sisters), is in no position to ask for annulment of the sale. the said rents in proportion to her own interests in the lands and the interest
in the four co-owners which she had acquired. Rosenda Nuqui and her son
ISSUES: Sotero, it is true, acted in bad faith when they sold the properties as theirs
alone, but so did the defendant Yap when he purchased them with knowledge
2. WoN the contract of sale is void for failing to state all the owners of the of the fact that there were other co-owners.
property? YES 18. Although the bad faith of one party neutralizes that of the other and hence as
3. WoN bad faith (if ever she was in bad faith) on the part of Erlinda will militate between themselves their rights would be as if both of them had acted in good
her action for nullity? NO faith at the time of the transaction, this legal fiction of Yap's good faith ceased
when the complaint against him was filed, and consequently the court's
RULING: WHEREFORE, the judgment appealed from is modified by eliminating declaration of liability for the rents thereafter is correct and proper. A
therefrom the award of attorney's fees of P1,000.00 in favor of Erlinda D. Diaz and possessor in good faith is entitled to the fruits only so long as his possession
her husband, plaintiffs in Civil Case No. 1907, and the award of nominal and is not legally interrupted, and such interruption takes place upon service of
exemplary damages in Civil Case No. 1774; and making the award of attorney's fees judicial summons.
in the sum of P2,000.00 payable to counsel for the account of the Mindanao Academy,
Inc. instead of the plaintiff stockholders. In all other respects the judgment appealed
from is affirmed. No pronouncement as to costs.

RATIO:
13. The contract is void because
(c) the contract purported to sell properties of which the sellers were not the
only owners, since of the four parcels of land mentioned in the deed their
shares consisted only of 7/12, (6/12 for Rosenda Nuqui and 1/12 for
Sotero, Jr.), while in the buildings, laboratory equipment, books,
furniture and fixtures they had no participation at all, the owner being
the Mindanao Academy, Inc.;
(d) the prestation involved in the sale was indivisible, and therefore
incapable of partial annulment, inasmuch as the buyer Yap, by his own
admission, would not have entered into the transaction except to acquire
all of the properties purchased by him.
14. Quitclaim of Erlinda, in the form of an extrajudicial partition, was made on
May 6, 1956, after the action for annulment was filed, wherein the plaintiffs
were not only Erlinda but also the other co-owners who took no part in the
sale and to whom there has been no imputation of bad faith.
15. The trial court's finding of bad faith is an erroneous conclusion induced by a
manifest oversight of an undisputed fact, namely, that on July 10, 1954, just
a month after the deed of sale in question, Erlinda D. Diaz did file an action
against Ildefonso D. Yap and Rosenda Nuqui, among others, asserting her
rights as co-owner of the properties.
16. Finally, bad faith on the part of Erlinda would not militate against the nullity
of the sale, considering that it included not only the lands owned in common
by Rosenda Nuqui and her six children but also the buildings and school
facilities owned by the Mindanao Academy, Inc., an entity which had nothing
to do with the transaction and which could be represented solely by its Board
of Trustees.
17. There can be no doubt that Erlinda D. Diaz is entitled to recover a share of
02 BUCTON v. GABAR (Lorena) 51. This action for specific performance filed by Spouses Bucton prays, inter-
January 31, 1974 | Antonio, J. | Estoppel on the True Owner (Art. 1434) alia, that Spouses Gabar be ordered to execute in favor of Spouses Bucton a
PETITIONER: Felix Bucton and Nicanora Gabar Bucton deed of sale of the western half of a parcel of land in Misamis Oriental
RESPONDENTS: Zosimo Gabar, Josefina Llamaso Gabar and the Honorable Court 52. Nicanora Gabar Bucton (Nicanora), wife of Felix Bucton, (Felix) is the sister
of Appeals of Zosimo Gabar (Zosimo), husband of Josefina Llamoso Gabar (Josefina).
53. In 1946 Josefina bought a parcel of land having an area of 728 sq. m. from
SUMMARY: This action for specific performance filed by Spouses Bucton prays the spouses Villarin on installment basis (P500 down, the balance payable in
that Spouses Gabar be ordered to execute in favor of Spouses Bucton a deed of sale installments)
of the western half of a parcel of land having an area of 728 sq.m. in Misamis 54. Josefina entered into a verbal agreement with her sister-in-law, Nicanora, that
Oriental. Spouses Bucton contends that sometime in 1946 Josefina bought the land the latter would pay one-half of the price (P3,000) and would then own one-
from the spouses Villarin on installment basis, to wit, P500 down, the balance half of the land. Thus, Nicanora gave Josefina the initial amount of P1,000,
payable in installments. Josefina entered into a verbal agreement with her sister-in- for which the latter signed a receipt and subsequently gave Josefina P400 to
law, Nicanora, that the latter would pay one-half of the price (P3,000) and would which she later signed another receipt marked.
then own one-half of the land. Subsequently, the spouses Villarin executed the deed 55. Spouses Bucton gave Spouses Gabar P1,000 in concept of loan, for which
of sale of the land in favor Josefina, to whom was issued a TCT. Spouses Bucton Zosimo signed a receipt marked
then sought to obtain a separate title for their portion of the land in question. Spouses 56. Meanwhile, after Josefina had received in January, 1946 the initial amount of
Gabar repeatedly declined to accommodate Spouses Bucton. The issue in this case is P1,000 as above stated, Spouses Bucton took possession of the portion of the
W/N there was a sale between Josefina and Nicanora. The Court ruled in the land indicated to them by Spouses Gabar and built a modest nipa house
affirmative. There is no question that Nicanora paid P1,500.00 to Josefina as therein.
purchase price of one-half of the lot, for the CA found as a fact "that Spouses Bucton 57. About two years later, Spouses Bucton built behind the nipa house another
really paid for a portion of the lot in question pursuant to their agreement with the house for rent. And, subsequently, they demolished the nipa house and in its
Spouses Gabar that they would own one-half (1/2) of the land." That sale, although place constructed a house of strong materials, with three apartments in the
not consigned in a public instrument or formal writing, is nevertheless valid and lower portion for rental purposes. Spouses Bucton occupied the upper portion
binding between the parties, for the time-honored rule is that even a verbal contract of this house as their residence, until July, 1969 when they moved to another
of sale or real estate produces legal effects between the parties. Although at the time house, converting and leasing the upper portion as a dormitory.
Nicanora paid P1,000.00 as part payment of the purchase price Souses Gabar 58. Spouses Bucton then sought to obtain a separate title for their portion of the
were not yet the owners of the lot, they became such when a deed of sale was land in question. Spouses Gabar repeatedly declined to accommodate them.
executed in their favor by the Villarin spouses. In the premises, Article 1434 of Their excuse: the entire land was still mortgaged with the Philippine National
the Civil Code is applicable. Thus, the payment by Nicanora of P1,000.00, her Bank as guarantee for Spouses Gabar's loan of P3,500
second payment of P400.00, and the compensation, up to the amount of P100.00 (out 59. Spouses Bucton continued enjoying their portion of the land, planting fruit
of the P1,000.00-loan obtained by Spouses Gabar from Spouses Bucton), resulted in trees and receiving the rentals of their buildings. In 1953, with the consent of
the full payment of the purchase price and the consequential acquisition by Spouses Spouses Gabar (who were living on their portion), Spouses Bucton had the
Bucton of ownership over one-half of the lot. Spouses Bucton therefore became entire land surveyed and subdivided preparatory to obtaining their separate
owners of the one-half portion of the lot in question by virtue of a sale which, though title to their portion.
not evidenced by a formal deed, was nevertheless proved by both documentary and 60. After the survey and the planting of the concrete monuments Spouses Gabar
parole evidence. erected a fence from point 2 to point 4 of the plan, which is the dividing line
between the portion of the Spouses Gabar
DOCTRINE: Article 1434 of the Civil Code, which provides that "when a person 61. In the meantime, Spouses Bucton continued to insist on obtaining their
who is not the owner of a thing sells or alienates and delivers it, and later the seller separate title. Spouses Gabar remained unmoved, giving the same excuse.
or grantor acquires title thereto, such title passes by operation of law to the buyer or 62. Frustrated,Spouses Bucton were compelled to employ Atty. Bonifacio
grantee” Regalado to intercede; counsel tried but failed. Spouses Bucton persevered,
FACTS: this time employing Atty. Aquilino Pimentel, Jr. to persuade Spouses Gabar
to comply with their obligation; this, too, failed. Hence, this case, which has
cost plaintiffs P1,500 in attorney's fees.
63. Spouses Gabar's evidence — based only on the testimony of Josefina — 3. Thus, the payment by Nicanora of P1,000.00, her second payment
denies agreement to sell to the spouses Bucton one-half of the land in of P400.00, and the compensation, up to the amount of P100.00 (out
litigation. She declared that the amounts she had received from Nicanora — of the P1,000.00-loan obtained by Spouses Gabar from Spouses
first, P1,000, then P400 — were loans, not payment of one-half of the price Bucton), resulted in the full payment of the purchase price and the
of the land (which was P3,000). consequential acquisition by Spouses Bucton of ownership over
64. The trial court, rendered judgment in favor of the Spouses Bucton one-half of the lot.
65. The Court of Appeals reversed the judgment of the trial court and ordered the 4. Spouses Bucton therefore became owners of the one-half portion of
complaint dismissed the lot in question by virtue of a sale which, though not evidenced
66. Spouses Bucton’s' appeal is predicated on the proposition that owners of the by a formal deed, was nevertheless proved by both documentary and
property by purchase from Spouses Gabar, and being in actual, continuous parole evidence.
and physical possession thereof since the date of its purchase, their action to 20. The action of Nicanora has not yet prescribed
compel the vendors to execute a formal deed of conveyance so that the fact 1. The real and ultimate basis of petitioners' action is their ownership
of their ownership may be inscribed in the corresponding certificate of title, of one-half of the lot coupled with their possession thereof, which
had not yet prescribed when they filed the present action. entitles them to a conveyance of the property.
2. No enforcement of the contract is needed, since the delivery of
ISSUE/s: possession of the land sold had consummated the sale and
5. WoN the action of Nicanora has prescribed – No transferred title to the purchaser, and that, actually, the action for
6. WoN there was a sale between Josefina and Nicanora - Yes conveyance is one to quiet title, i.e., to remove the cloud upon the
appellee's ownership by the refusal of the appellants to recognize the
RULING: WHEREFORE, the decision and resolution of respondent Court of Appeals sale made by their predecessors. It is an established rule of American
appealed from are hereby reversed, and the judgment of the Court of First Instance of jurisprudence (made applicable in this jurisdiction by Art. 480 of the
Misamis Oriental, Branch IV, in its Civil Case No. 3004, is revived. Costs against New Civil Code) that actions to quiet title to property in the
private respondents. possession of the plaintiff are imprescriptible.

RATIO:
19. There was a sale between Josefina and Nicanora
1. There is no question that Nicanora paid P1,500.00 to Josefina as
purchase price of one-half of the lot, for the CA found as a fact "that
Spouses Bucton really paid for a portion of the lot in question
pursuant to their agreement with the Spouses Gabar that they would
own one-half (1/2) of the land." That sale, although not consigned
in a public instrument or formal writing, is nevertheless valid and
binding between the parties, for the time-honored rule is that even a
verbal contract of sale or real estate produces legal effects between
the parties.
2. Although at the time Nicanora paid P1,000.00 as part payment
of the purchase price, Spouses Gabar were not yet the owners of
the lot, they became such owners when a deed of sale was
executed in their favor by the Villarin spouses.
1. Article 1434 of the Civil Code, which provides that
"[w]hen a person who is not the owner of a thing sells
or alienates and delivers it, and later the seller or
grantor acquires title thereto, such title passes by
operation of law to the buyer or grantee," is applicable.
03 City of Manila v. Bugsuk (Celaje) 12. In 1951 and during the 1st, 2nd and 3rd quarters of 1952, the Bugsuk Lumber
Company made sales of lumber to several firms including Pio Barreto &
G.R. No. L-8255 | July 11, 1957 | Felix, J. | Dealer v. Producer Sons, Inc., Gotamco & Sons, Co., Basilan Lumber Co., etc.
PETITIONER: CITY OF MANILA 13. On October 10, 1952, the Office of the Treasurer of the City of Manila sent
RESPONDENTS: BUGSUK LUMBER CO. a demand to the Company for the payment of the amount of P544.50 for
license fees corresponding to the years 1951 and 1952, and P40.00 for the
SUMMARY: The City of Manila sued Bugsuk Lumber Company, alleging that necessary mayor's permit, on the ground that said business firm was found
Bugsuk Lumber Company sold at wholesale to different lumber dealers in Manila to be engaged in the sales of timber products without first securing the
in 1951 and 1952 for which it should have paid a quarterly license tax of P40.000 required licenses and permits pursuant to numerous City Ordinances.
or a total of P280.00 and the mayor's permit of P20.00. Bugsuk claims that it not
liable to pay wholesale dealer taxes because it is not a dealer as contemplated by 14. The Company must have refused or failed to pay said imposts because on
the law, but a producer. Lower courts ruled in favor of City of Manila. Supreme June 11, 1953, the City Fiscal of Manila filed a complaint against the Bugsuk
Court ruled in favor of Bugsuk. SC held that appellant certainly is not a “dealer” Lumber Co., Inc., with the Municipal Court of Manila and prayed that
because what appellant sold was the produce of its concession in Palawan. judgment be rendered ordering the defendant Company to pay the City of
Further, as to Manila’s contention that Bugsuk is a dealer because it “sold” lumber Manila the amount of P584.50 representing license fees and mayor's
at its office in Manila, SC held that the placing of an order for goods and the permit fees, with legal interests thereon.
making of payment thereto at a principal office does not transform said office into 15. Defendant Bugsuk Lumber Co., Inc., filed an answer on October 12, 1953,
a store, for it is a necessary element that there must also be goods or wares stored contesting plaintiff's allegation that it sold lumber at wholesale transactions
therein or on display, and provided also that the firm or person maintaining that because what it actually sold were unprocessed logs; neither did it sell at retail
office is actually engaged in the business of buying and selling. Indeed, the because the timbers were delivered directly from the vessel to the lumber
principal office of Bugsuk alluded to as a store only serves to facilitate the dealers, and set up the affirmative defenses that the Bugsuk Lumber
transactions relative to the sale of its produce, but does not act as a dealer or Company was essentially a producer, having no lumber yard of any kind
intermediary between its field office and its customers. in Manila or elsewhere, nor kept a store where lumber or logs could be sold,
DOCTRINE: A dealer, in the legal meaning of the word, is not one who buys to and that its products (logs) were sold directly from the lumber concession to
keep or makes to sell, but one who buys to sell again; the middleman between the the dealers in Manila; that as such producer, it had paid the taxes required by
producer and the consumer. For an office to be “treated” as a store, it is necessary law; and prayed for the dismissal of the complaint.
that there must also be goods or wares stored therein or on display, and provided 16. Municipal Court ruled in favor of City of Manila and ordered Bugsuk Lumber
also that the firm or person maintaining that office is actually engaged in the to pay City of Manila. On appeal, CIF affirmed. The CIF held that that the
business of buying and selling. Company sold logs to various firms in wholesale and retail transactions and
FACTS: although defendant had no store or lumber yard in the City, this fact alone
cannot destroy the findings of the inspector of the City Treasurer's Office that
11. In Bugsuk Lumber Company, Inc., a domestic corporation with field office it sold logs to different buyers in Manila; that the imposition of the taxes in
at Balabak, Palawan, and principal office at 703 San Fernando, Binondo, question did not constitute double taxation and, consequently, ordered the
Manila, was organized to: defendant Company to pay the sum of P584.50 plus legal interests and costs.
a. (a) (To buy and sell lumber and to engage in general, in any kind of Hence this appeal.
business concerning lumber); (b) (To apply from the Government or ISSUES:
to acquire in any manner permitted by law, lumber concessions if
the business would so require); 3. W/N appellant, maintaining a principal office in Manila, receiving orders for
its products and accepting in said office payments thereto, can be considered
b. (c) (To saw lumber and to buy logs, in case the business of the a dealer in this City and is, therefore, subject to the payment of the license
corporation would so demand; and (d) (To make all kinds of tax and permit fees in question. No.
business that may be directly or indirectly in line with the purposes
for which this corporation has been created). RULING: Wherefore, the decision appealed from is hereby reversed and appellant
declared exempt from the liabilities sought to be charged against it under the instant appeal.
provisions of the aforementioned ordinances, without pronouncement as to costs. It is
29. Appellee, however, in asserting that appellant Company is a dealer relied on
so ordered.
the case of Atlantic Refining Co. vs. Van Valkenburg, 265 Pa. 456; 109 A.
208, wherein it was held that the term dealer includes "one who carries on the
business of selling goods, wares and merchandise manufactured by him at
RATIO:
a store or warehouse apart from his own manufactory", and it was the
24. Appellant does not dispute the power of the Municipal Board of the City of contention of the City Fiscal that the office at 703 San Fernando, Binondo,
Manila to enact Ordinance No. 3000 requiring wholesale and retail dealers to Manila, where appellant received orders and receipted payment for such
secure and pay the mayor's permit annually, neither does it contest the orders is actually a store.
validity of Ordinance No. 3364 which contains the following provision:
30. Appellant admittedly maintained said principal office but averred that it was
a. Group 2. Retail dealers in new (not yet used) merchandise, which used merely to facilitate the payment of the tax obligations of said Company,
dealers are not yet subject to the payment of any municipal tax, such to receive orders of its timber produce and accept payments therefor, and not
as; (1) Retail dealers in General Merchandise and (2) retail dealers for any purpose connected with the business of buying and selling. Did the
exclusively engaged in the sale of electrical supplies; rice; cooking fact that appellant received orders of its goods and accepted payments thereto
utensils and construction materials… in said office make such office a store?
25. nor of Ordinance No. 3420 which provides: 31. Lexicographers defined a store as:
a. SEC. 1. Municipal Tax on wholesalers in General Merchandise. — a. Any place where goods are kept for sale, whether by wholesale or
There shall be paid by every person, firm or corporation engaging retail; a shop (Webster's New International Dictionary).
in business as wholesale dealer in general merchandise, a municipal
b. Any place where goods are deposited and sold by one engaged in
tax based on wholesales….
buying and selling them (Black's Law Dictionary, 4th ed., p. 1589).
26. A dealer has been defined as:
32. It could be seen that the placing of an order for goods and the making of
a. A dealer, in the common acceptation and, therefore, in the legal payment thereto at a principal office does not transform said office into
meaning of the word, is not one who buys to keep or makes to sell, a store, for it is a necessary element that there must also be goods or
but one who buys to sell again; the middleman between the producer wares stored therein or on display, and provided also that the firm or
and the consumer of the commodity. person maintaining that office is actually engaged in the business of
buying and selling. These elements are wanting in the case at bar for it needs
b. It has been said that a dealer stands immediately between the
no further clarification that the principal office alluded to as a store only
producer and the consumer, and depends for his profit, not upon the
serves to facilitate the transactions relative to the sale of its produce, but does
labor he bestows on his commodities, but upon the skill and
not act as a dealer or intermediary between its field office and its customers.
foresight with which he watches the markets.
33. We may further add that this matter was already passed upon by this Court
27. In the light of the above definitions, appellant certainly does not fall
when, through Mr. Justice Alejo Labrador, it held that:
within the common and ordinary acceptation of the word "dealer" for
there is no controversy as to the fact that what appellant sold was the a. It may be admitted that the manufacturer becomes a dealer if he
produce of its concession in Palawan. Even conceding, therefore, that the carries on the business of selling goods or the products
lumber which appellant disposed of comes within the connotation of manufactured by him at a store or warehouse apart from his own
'construction materials' (Group 2, Ordinance No. 3364) and of the term shop or manufactory. But plaintiff-appellee did not carry on the
"general merchandise" (used in Ordinances Nos. 3364 and 3420)…. business of selling sugar at stores or at its warehouses. It entered into
the contracts of sale at its central office in Manila and made
28. We see no reason why a producer or manufacturer selling its own deliveries of the sugar sold from its warehouses. It does not appear
produce or manufactured goods would be considered a dealer just to
that the plaintiff keeps stores at its warehouses and engages in
make it liable for the corresponding dealer's tax, as is the case in the
selling sugar in said stores. Neither does it appear that any one who
desires to purchase sugar from it may go to the warehouses and there
purchase sugar. All that it does was to sell the sugar it manufactured;
it does not open stores for the sale of such sugar. Plaintiff-appellee
did not, therefore, engage in the business of selling sugar. (Central
Azucarera de Don Pedro vs. City of Manila et al).
04 TAGATAC vs JIMENEZ (Abs) registered the car in his name. On August 18, 1952, Feist sold the car to Felix
February 22, 1957 | Ocampo, J. | Sale by Seller having voidable title Sanchez. The latter then registered the car in his name.
13. Sanchez then offered to sell the car to Liberato C. Jimenez. After some
PETITIONER: Trinidad C. Tagatac investigation and finding out that everything is in order, Jimenez bought the
RESPONDENT: Liberato C. Jimenez car for 10,000php.
14. On August 21, 1952, Jimenez brought the car to California Car Exchange to
SUMMARY: Tagatac sold his car to a certain Feist who paid him via check. be displayed for sale. This is where Tagatac discovered her car. She
However, when he was going to encash it, the bank refused and informed him demanded from the manager of said exchange for the delivery of the car to
that Feist has no account in said bank. Meanwhile, Feist notarized the deed of sale her but the latter refused.
between him and Tagatac, and registered the car in his name. He then sold it to 15. On October 20, 1952, Trinidad C. Tagatac filed a suit for the recovery of the
Sanchez who, in turn, sold it to Jimenez. Jimenez then displayed it in California possession of the car. After trial, the lower court decided against Tagatac and
Car Exchange for sale. Tagatac saw the car and demanded that the car’s confirmed the ownership and possession of Jimenez. Tagatac appealed.
possession be returned to him. The lower court decided against him and
confirmed Jimenez’ ownership of the car. The issue is whether or not Tagatac can
still recover his car. The Supreme Court held that Jimenez has a better title ISSUES: WoN Jimenez is a purchaser in good faith and thus entitled to the possession
because Tagatac was not unlawfully deprived of his car in the legal sense of the car in question? –YES.
provided in Article 559 of NCC. He voluntarily departed with his car, thus
perfecting a contract of sale. The fact that Feist paid him with a bouncing check RULING: Wherefore, the Petition is GRANTED.
went into the performance of the sale, not its perfection. Therefore, the sale being
legally perfected albeit voidable, Jimenez is considered an innocent purchaser. RATIO:

DOCTRINE: Whenever there is an underlying sale which grants to the culprit- 1. Tagatac claims that Jimenez was a purchaser in bad faith since the estafa was
buyer a voidable title, even when this is accompanied by the criminal act of estafa committed by Feist who then transferred the car to Jimenez basing such
or swindling, Article 1506 would grant to the buyer in good faith a better title as presumption from Rule 123 of the Rules of Court. Said provision states that
against the original owner even though the latter may be classified to have been there is a disputable presumption that a person found in possession of a thing
“unlawfully deprived” of the subject matter under Article 559. taken in the doing of a recent wrongful act is the taker and the doer of the
whole act. This presumption usually arises in cases of stolen goods.
FACTS: 2. We cannot make the same presumption in this case because (1) the car was
8. On October 1951, Trinidad C. Tagatac bought a car for $4,500 from not stolen from Tagatac and (2) Jimenez came into possession of the car two
Danielson and Kavarno Motors of Sta. Barbara, California, USA. On May months after Feist swindled Tagatac so even assuming that it was stolen,
27, 1952 she brought the car to the Philippines. Jimenez could not have been presumed to have been the wrongdoer for the
9. Sometime in June 1952, Tagatac went to a friend’s house and met one Warner wrongful act that was done two months before the car came into his
L. Feist aka Warner L. Levy who was posing as a wealthy man. While talking, possession.
Feist was able to make Tagatac believe that he was “rich” and offered to buy 3. Tagatac also claimed that when the car was transferred to Ruben Masalonga
her car for 15,000php. Tagatac agreed to sell the car to Feist. of the California Car Exchange and re-transferred to him, Jimenez was
10. On June 18, 1952, the deed of sale was made. Tagatac signed said private already informed of the criminal case filed against Feist, making him a buyer
document. Feist paid the price to Tagatac by means of a postdated check in bad faith. However, it is a fact beyond dispute that when Jimenez first
which he delivered on the same day. Tagatac then delivered the car to Feist. acquired the car, he had no knowledge of any flaw in the title of the person
11. The next day, Tagatac tried to cash the check with PNB, however, the bank from whom he acquired it. He merely transferred the car to Masalonga
refused the check and informed her that Feist has no account and no funds to facilitate the sale of the car.
with PNB. Tagatac then notified all enforcement agencies of the estafa 4. Tagatac also alleges that the lower court blatantly disregard that Feist was
committed by Feist. The authorities failed to apprehend him and the car was criminally convicted of Estafa. He also avers that it is part of Feist’s civil
already gone. liability to get the subject matter from a third person. This is based from
12. Meanwhile, Feist managed to have the private deed of sale notarized and Article 559 of the New Civil Code which states that although a third person
who acquired a property in good faith, the one who has lost it or has been
unlawfully deprived thereof, may nevertheless recover it from the possession
of the same.
5. However, Tagatac was not unlawfully deprived of his property
considering he voluntarily departed with it. The giving of Feist of a
bouncing check is illegal but that does not render fraudulent or illegal the sale
which bears all the earmarks of legality.
6. As regards the contract of sale between Tagatac and Feist, due to the fraud
and deceit practiced by Feist, it is considered voidable. And because it was
not annulled, it is deemed ratified and cleansed of all its defects.
7. When Feist sold the car to Sanchez, the former’s title had been avoided and
therefore conferred a good title on the latter. There being no proof that
Sanchez acted in bad faith, it is safe to assume that he acted in good faith, for
value and without notice of any defect in Sanchez’ title, then he acquired a
good title to the car. Good title means an indefeasible title to the car even
as against the original owner of the car.
8. Even under the rules of equity, Jimenez has a better right. Despite both the
parties being innocent, it was the gullibility of Tagatac that made
possible the swindle committed by Feist. Thus, it is Tagatac that should
bear the consequences of the swindle.
05 EDCA PUBLISHING v. SANTOS (Ram) prepared and delivered the same together with an invoice.
April 26, 1990 | Cruz, J. | Article 559 ; Good faith is equivalent to a title 2. In turn Cruz issued a personal check covering the purchase price of
P8,995.65. Cruz then sold 120 of the books to Leonor Santos (Santos) who,
PETITIONER: EDCA Publishing & Distributing Corporation after verifying the seller's ownership from the invoice he showed her,
RESPONDENTS: The Spouses Leonor and Gerardo Santos, doing business paid him P1,700.00.
under the name and style of “Santos Bookstore” and the CA 3. EDCA then became suspicious when Cruz placed a second order, since his
first check was not yet cleared. EDCA made an inquiry with the De la Salle
SUMMARY: An impostor identifying himself as a professor, Cruz ordered College where Cruz had claimed to be a dean. EDCA was informed that there
books from EDCA for which he issued a check. EDCA delivered the books to was no such person in its employ.
Cruz with a sales invoice that the books had been paid for on delivery – vesting 4. Moreover, it was found out that Cruz had no more account or deposit with
ownership to Cruz. Cruz then sold some of the books to Santos. The check issued the Philippine Amanah Bank, against which he had drawn the payment check.
by Cruz bounced and it was found that he was an impostor. EDCA is now 5. With the aid of policemen Cruz was trapped and arrested. His real name is
claiming recovery of the books from Santos. EDCA argues that Cruz never Tomas de la Peña. It was found out that 120 of the books he had ordered from
acquired title to the books because the failure of the consideration nullified the EDCA were sold to Spouses Santos.
sale. Therefore, the Spouses Santos could not establish ownership over the books. 6. EDCA and the police then went to Santos’ store and seized the subject books
MTC, RTC and CA all sided with Santos. The issue in this case is WON EDCA and threatened Santos for buying stolen property. They seized the books
has been unlawfully deprived of the books because the check issued by Cruz was without a warrant.
dishonored. The SC held that there was already a perfected contract of sale. 7. The Spouses Santos sued for recovery of the books after demand for their
Moreover, interpreting Article 559 provides that "the possession of movable return was rejected by EDCA.
property acquired in good faith is equivalent to a title," thus dispensing with 8. It was only after a writ of preliminary attachment was issued that EDCA
further proof. finally surrendered the books to Spouses Santos.
9. The MTC ruled in favor of Spouses Santos, which was sustained by the RTC.
DOCTRINE: Note that the doctrine in the sales outline refers to the Tagatac and The CA affirmed the same. Hence, this petition.
EDCA. Please see additional discussion under C.Tiu’s reviewer. 10. EDCA argues that
Non-payment only creates a right to demand payment or to rescind the contract, a. The impostor “Cruz” acquired no title to the books that he could not
or to criminal prosecution in the case of bouncing checks. But absent the have validly transferred the books to the Spouses.
stipulation above noted, delivery of the thing sold will effectively transfer b. As the payment check bounced for lack of funds, there was a failure
ownership to the buyer who can in turn transfer it to another. of consideration that nullified the contract of sale between it and
Cruz.
Article 559 provides that "the possession of movable property acquired in good c. Spouses Santos could not establish ownership because they could
faith is equivalent to a title," thus dispensing with further proof. not even produce a receipt to prove that they bought the books.

CLV’s Comments on the cases: The implication of the Tagatac and EDCA ISSUES:
Publishing rulings is that Article 1506 represents an operative act which would 3. WON EDCA has been unlawfully deprived of the books because the check
constitute a further exception to the provisions of Article 559, which means that issued by Cruz was dishonored. - NO.
if the owner has been unlawfully deprived by means of deceit pertaining to the
non-payment of the purchase price, but the one who takes the movable is able to RULING: Wherefore, the challenged decision is AFFIRMED and the petition is
sell and deliver the movable to another person who takes it in good faith and for DENIED, with costs against the petitioner.
value before the owner is able to rescind the earlier sale, the buyer obtains good
title and the original owner has no cause of action to recover. RATIO:
Perfection of a Contract of Sale
FACTS: 1. The contract of sale is consensual and is perfected once agreement is reached
1. A person identifying himself as Professor Jose Cruz (Cruz) placed an order between the parties on the subject matter and the consideration.
by telephone with EDCA for 406 books, payable on delivery. EDCA 2. It is clear that ownership in the thing sold shall not pass to the buyer until full
payment of the purchase price only if there is a stipulation to that effect. thereof, may recover it from the person in possession of the same.
Otherwise, the rule is that such ownership shall pass from the vendor to the
vendee upon the actual or constructive delivery of the thing sold even if the If the possessor of a movable lost or of which the owner has been unlawfully deprived
purchase price has not yet been paid. has acquired it in good faith at a public sale, the owner cannot obtain its return without
3. Non-payment only creates a right to demand payment or to rescind the reimbursing the price paid therefor.
contract, or to criminal prosecution in the case of bouncing checks. But absent
the stipulation above noted, delivery of the thing sold will effectively transfer Article 1506. Where the seller of goods has a voidable title thereto, but his title has
ownership to the buyer who can in turn transfer it to another. not been avoided at the time of the sale, the buyer acquires a good title to the goods,
4. Actual delivery of the books having been made, Cruz acquired ownership provided he buys them in good faith, for value, and without notice of the seller's defect
over the books, which he could then validly transfer to the private of title. (Note: This was not mentioned in the EDCA case)
respondents.
5. The fact that he had not yet paid for them to EDCA was a matter between DISCUSSION IN CLARENCE TIU REVIEWER:
him and EDCA and did not impair the title acquired by the Spouses Santos CLV’s Comments on the cases: The implication of the Tagatac and EDCA Publishing
to the books. rulings is that Article 1506 represents an operative act which would constitute a further
exception to the provisions of Article 559, which means that if the owner has been
Article 559 - Good faith is equivalent to a title unlawfully deprived by means of deceit pertaining to the non-payment of the purchase
1. Article 559 provides that "the possession of movable property acquired in price, but the one who takes the movable is able to sell and deliver the movable to
good faith is equivalent to a title," thus dispensing with further proof. another person who takes it in good faith and for value before the owner is able to
2. Leonor Santos took care to ascertain first that the books belonged to Cruz rescind the earlier sale, the buyer obtains good title and the original owner has no
before she agreed to purchase them. She did not have to go beyond that cause of action to recover
invoice to satisfy herself that the books being offered for sale by Cruz
belonged to him; yet she did. The EDCA invoice Cruz showed her assured
her that the books had been paid for on delivery.
3. To Santos, Cruz must have been only one of the many sellers she was dealing
with. Cruz was selling the books for a discount because he was in financial
need. It is not bad faith to buy the books at a discount and resell them for a
profit.
4. EDCA was the one who was less cautious in dealing with the impostor as it
readily delivered the books although it had never transacted with him before.
It did not wait to clear the check and it indicated in the sales invoice that the
books had been paid for on delivery – vesting ownership to Cruz.
5. Although the title of Cruz was presumed under Article 559 by his mere
possession of the books, these being movable property, Leonor Santos
nevertheless demanded more proof before deciding to buy them.
6. It would certainly be unfair now to make the Spouses Santos bear the
prejudice sustained by EDCA as a result of its own negligence. We cannot
see the justice in transfer ring EDCA’s loss to the Santoses who had acted in
good faith, and with proper care, when they bought the books from Cruz.
7. EDCA should go after “Cruz” and not the Spouses Santos.

RELEVANT PROVISION:
ART. 559. The possession of movable property acquired in good faith is equivalent to
a title. Nevertheless, one who has lost any movable or has been unlawfully deprived
06 AZNAR V YAPDIANGCO (Nicolin) a. In the afternoon of May 28, 1959, a certain L. De Dios, claiming to
March 31, 1965 | Regala, J | Sale by seller having voidable title (Article 1506, as an be a nephew of Vicente Marella, went to Santos’ residence to
exception to Art 559) answer the ad.
b. However, Teodoro Santos was out and so his son, Irineo Santos,
PLAINTIFF-APPELLANT: Jose B. Aznar received and talked with De Dios. Teodoro instructed his son to
DEFENDANT-APPELLEE: Rafael Yapdiangco see Vicente the following day at 1642 Crisostomo Street,
INTERVENOR-APPELLEE: Teodoro Santos Sampaloc, Manila. And so, during the meeting, Vicente agreed to
buy the car for Php 14,700 on the understanding that the price
SUMMARY: Teodoro Santos advertised the sale of his FORD FAIRLANE 500 in would be paid only after the car had been registered in his
a newspaper. On L. De Dios went to the house of Teodoro and talked to his son name.
Ireneo Santos and said that his uncle Vicente Marella is interested in buying the said c. Teodor, Irineo and DeDios executed the sale for the car at the
car. The next day, Ireneo went to the house of Marella and they agreed to the price office of Atty. Jose Padolina. The parties proceeded to the Motor
of P14,700 on the understanding that it will be paid after the car has been registered Vehicle’s Office in QC to register the car in the name of Vicente.
in the latter’s name. A deed of sale was executed and the registration was changed to (Purchase price still not paid at this stage)
the name of Marella. Ireneo went to Marella to get the payment and deliver the car d. Teodoro gave the registration papers and a copy of the deed of sale
who informed him that he is P2,000 short of the money and that they need to go to to Irineo and instructed him not to give them to Vicente until he
his sister to get it. Ireneo, together with De Dios and an unidentified man went to a has given the full payment for the car.
house. Once inside, De Dios asked Ireneo to wait in the sale. After waiting in vain, e. Then, only Irineo and De Dios proceeded to 1642 Crisostomo
he went down and discovered that the car was gone. Marella was able to sell the car where Irineo demanded the payment from Vicente. The cash he
to plaintiff-appellant Jose Aznar and while attending to registration, the car was had on hand was short of Php2,000.
seized by Phil. Constabulary due to the report of the incident. Aznar filed a complaint f. They went to Vicente’s sister’s house for him to borrow the money
for replevin against Captail Rafael Yapdiangco, the head of the PC which seized the but before they went to the house, Vicente requested for the
car. Lower court ruled awarding the car to Teodoro as Teodoro was unlawfully registration papers and the deed of sale from Irineo under the
deprived of his personal property by Vicente (from whom Aznar traced his right). So pretext that he was going to show the papers to his lawyers. (Irineo
Aznar appealed said decision. The issue before the court is who has a better right to gave the documents, sigh)
the possession of the disputed automobile. The SC held that Teodoro has better right g. De Dios went with Irineo inside the house and made Irineo wait in
to the vehicle. Marella did not have any title to the property under litigation because the sala. After a considerable length of time waiting, Irineo
the same was never delivered to him. He may have the contract but he never acquired discovered that neither the car nor their unidentified companion
valid title. Although the keys to the car may have been given to the unidentified were there. There was also no such person as the sister. The 1642
companion, it may be done only because that companion took them to the place Crisostomo house was likewise closed and Vicente was gone.
where the sister of Marella was supposed to live. The car was evidently stolen and h. Teodoro reported to police the said incident.
that the buyer did not acquire any valid title thereto. 2. May 9, 1959, Vicente sold the car to Jose B. Aznar for Php 15,000. SC
found that Aznar is a buyer in good faith, for a valuable consideration and
DOCTRINE: When owner did not voluntarily deliver possession of the car, and in without notice of the defect of vendor’s title.
effect it was stolen from him, then one who buys the car even in good faith from the a. While Aznar was registering the car in his name, agents of the
thief will lose the car to the owner who is deemed to have been unlawfully Philippine Constabulary (PC) seized and confiscated the car in
deprived.Ownership is not transferred by contact merely but by tradition or delivery. consequence of the report to them by Teodoro.
Contracts only constitute titles or rights to the transfer or acquisition of ownership 3. Action: Aznar filed a complaint for replevin against Captail Rafael
while delivery or tradition is the mode of accomplishing the same. Yapdiangco, the head of the PC which seized the car. He prayed for its
deliver of the car.
a. Teodoro moved and was allowed to intervene by lower court.
FACTS: b. Lower court ruled awarding the car to Teodoro as Teodoro was
1. May 1959, Teodoro Santos advertised in two metropolitan papers the sale unlawfully deprived of his personal property by Vicente (from
of his FORD FAIRLANE 500. whom Aznar traced his right)
c.
Further, Azanr is entitled to recovery (as GF buyer) pursuant to 3.
Under Article 712 of CC, “"ownership and other real rights over property are
Article 559 of NCC22 acquired and transmitted by law, by donation, by testate and intestate
4. From this decision, Aznar appeals succession, and in consequence of certain contracts, by tradition.”
ISSUE/s: a. SC interpretation: ownership is not transferred by contact merely but
1. Between Teodoro Santos and the plaintiffappellant, Jose B. Aznar, who has by tradition or delivery. Contracts only constitute titles or rights to
a better right to the possession of the disputed automobile? Teodoro Santos the transfer or acquisition of ownership while delivery or tradition
is the mode of accomplishing the same.
RULING: UPON ALL THE FOREGOING, the instant appeal is hereby dismissed b. Article 712 contemplates that the act be coupled with intent of
and the decision of the lower court affirmed in full. Costs against the appellant. delivering the thing.
c. No evidence Irineo voluntarily delivered the key to the car and so
did not vest the title to the vehicle to De Dios (as agent of Vicente).
RATIO: 4. In the case on hand, the car in question was never delivered to the vendee
ARTICLE 559 APPLIES TO THE CASE AT BAR (Vicente) by the vendor (Teodoro) as to complete or consummate the
1. SC found the contention of Aznar unmeritorious. transfer of ownership by virtue of the contract. It should be recalled that
a. Aznar contends that the applicable provision of CC is Article 150623 while there was indeed a contract of sale between Vicente Marella and
not Article 559 Teodoro Santos, Vicente took possession of the subject matter thereof by
b. Under the 1506, it is essential that the seller should have a voidable stealing the same while it was in the custody of the latter's son.
title at least. It is very clearly inapplicable where the seller had no MISPLACED CONFIDENCE NOT APPLICABLE
title at all. 1. Aznar contends that the right of owner to recover personal property acquired
c. Vicente did not have any title to the property under litigation in good faith by another is based on his being dispossessed without his
because the same was never delivered to him. He sought ownership consent.
or acquisition of it by virtue of the contract, Vicente could have 2. SC held that the common law principle that where one of two innocent
acquired ownership or title only by delivery or tradition of the persons must suffer by a fraud perpetrated by another, the law imposes the
car to him. loss upon the party who, by his misplaced confidence has enabled the fraud,
2. SC held that the lower court was correct in applying Article 559 of the Civil cannot be applied in a case covered by the NCC, specifically 559. Between a
Code to the case at bar. common law principle and a statutory provision, the statutory provision must
a. For under it, the rule is to the effect that if the owner has lost the prevail in this jurisdiction.
thing, or if he has been unlawfully deprived of it, he has a right to
recover it, not only from the finder, thief or robber, but also
from the third person who may have acquired it in good faith
from such finder, thief or robber
b. Exceptions to the general rule of irrevindicability:
i. Owner lost the thing
ii. Unlawfully deprived thereof
c. In these cases, possessor cannot retain the thing as against the
owner, who may recover it without paying any indemnity, except
when the possessor acquired it in a public sale.

TRANSFER OF OWNERSHIP NOT CONSUMMATED

22 23
"Art. 559. The possession of movable property acquired in good faith is equivalent to title. Nevertheless, "Art. 1506. Where the seller of goods has a voidable title thereto, but his title has not been voided at the
one who has lost any movable or has been unlawfully deprived thereof, may recover it from the person in time of the sale, the buyer acquires a good title to the goods, provided he buys them in good faith, for value,
possession of the same. "If the possessor of a movable lost or of which the owner has been unlawfully and without notice of the seller's defect of title."
deprived, has acquired it in good faith at a public sale, the owner cannot obtain its return without reimbursing
the price paid therefor.
07 CRUZ v. PAHATI (JOSEF) 3. Bulahan on his part claims the following:
April, 13, 1956 | Bautista Angelo, J. | Sale by seller having voidable title o That he acquired the automobile from Jesusito Belizo (co-
defendant) for value and without having any knowledge of any
PETITIONER: Jose R. Cruz (plaintiff-appellant) defect in the title of the latter (from Belizo à Bulahan à Pahati)
RESPONDENTS: Reynaldo Pahati, Felixberto Bulahan and Jesusito Belizo. o That Cruz had previously acquired title to said automobile by
(defendants-appellees) purchase from Belizo as evidenced by a deed of sale executed to
that effect. (Connect this with ratio #1 kasi totoong previously
SUMMARY: This case involves an action of replevin instituted by plaintiff Cruz for naacquire ni Cruz yung car kay Belizo, but later under ratio #5
the recovery of the possession of his car. Pahati, one of the defendants, testified that nagkaron na ng falsification)
he acquired the car from Bulahan but their transaction was later on cancelled when o That later Cruz delivered the possession of the automobile to
the Manila Police Department impounded the automobile so the property reverted Belizo for resale and to facilitate it he gave the Belizo a letter of
back to Bulahan. Thereafter, Bulahan, another defendant, testified that he acquired authority to secure a new certificate of registration in his name
the property from Belizo, another co-defendant, and that he acquired it in good faith (Cruz’s name) and that by having clothed Belizo with an apparent
without any knowledge of any defect in the title of Belizo. The lower court rendered ownership or authority to sell the automobile, Cruz is now estopped
judgment in favor of Bulahan. Now the issue is who has the better right between to deny such ownership or authority. (Connect this with ratio #3)
Bulahan and Cruz. The SC held that Cruz has the better right. The court held than o Bulahan claims that between two innocent parties, he who gave
when a person has been deprived of his property in consequence of a crime such as occasion, through his conduct, to the falsification committed by
estafa in this case, the latter has the right to recover the property even when the Belizo, should be the one to suffer the loss and this one is Cruz.
property is in the possession of an honest purchaser. The only exception to this is 4. Lower court rendered judgment declaring defendant Bulahan entitled to the
that when the third party has acquired the property in good faith at a public sale, automobile in question and consequently ordered the plaintiff to return it to
which is not present in this case because Bulahan acquired the car through a private said defendant and, upon his failure to do so, to pay him the sum of P4,900,
transaction with Belizo. with legal interest from the date of the decision.
5. Hence, this appeal.
DOCTRINE: Whoever may have been deprived of his property in consequence of
a crime is entitled to the recovery thereof, even if such property is in the possession
of a third party who acquired it by legal means. The only exception to this is when ISSUE/s:
the third party has acquired the property in good faith at a public sale. 7. WoN Cruz has better right with regard to the property against Bulahan - Yes,
Cruz has better right because he was deprived of his property in consequence
of a crime which is estafa

RULING: Wherefore, the decision appealed from is reversed. The Court declares
FACTS: plaintiff (Cruz) to be entitled to recover the car in question, and orders defendant
1. This is an action of replevin instituted by Jose Cruz (plaintiff) in the Court Jesusito Belizo to pay him the sum of P5,000 as moral damages, plus P2,000 as
of First Instance of Manila to recover the possession of an automobile and attorney's fees. The Court absolves defendants Bulahan and Pahati from the complaint
certain amount as damages and attorney's fees resulting from his illegal as regards the claim for damages, reserving to Bulahan whatever action he may deem
deprivation thereof. proper to take against Jesusito Belizo. No costs.
2. Pahati (defendant) on his part claims the following:
o Admitted having bought the automobile from Bulahan (co-
defendant), for the sum of P4,900 which he paid in check. RATIO:
o When the Manila Police Department impounded the automobile, he SC’s perusal of RTC’s decision
cancelled the sale and stopped the payment of the check and as a
result he returned the automobile to Bulahan who in turn 1. The lower court found that the automobile in question was originally owned
surrendered the check for cancellation (basically I think this was a by the Northern Motors, Inc. which later sold it to Chinaman Lu Dag. This
rescission so there was mutual restitution). Chinaman sold it afterwards to Jesusito Belizo and the latter in turn sold it to
Cruz. Belizo was then a dealer in second hand cars. One year thereafter, been unlawfully deprived, has acquired it in good faith at a public
Belizo offered Cruz to sell the latter’s automobile for him, Belizo claiming sale, the owner cannot obtain its return without reimbursing the
to have a buyer for it. (Northern Motors, Inc. à sold to Chinaman Lu Dag price paid therefor."
à sold to Belizo à sold to Cruz à after 1 year, delivered the car to
Belizo for resale) 8. It appears that "one who has lost any movable or has been unlawfully
2. Cruz agreed. At that time, Cruz’s certificate of registration was missing deprived thereof, may recover it from the person in possession of the
and, upon the suggestion of Belizo, Cruz wrote a letter addressed to the same" and the only defense the person in possession may have is if
Motor Section of the Bureau of Public Works for the issuance of a new he "has acquired it in good faith at a public sale" in which case
registration certificate alleging as reason the loss of the one previously "the owner cannot obtain its return without reimbursing the price paid
issued to him and stating that he was intending to sell his car. therefor." (However, in this case, Bulahan did not acquire the car at a
3. This letter was delivered to Belizo on March 3, 1952. He also turned over public sale but rather by purchase from Belizo which is a private
to Belizo the automobile on the latter's pretext that he was going to show transaction)
it to a prospective buyer. 9. And supplementing this provision, Article 1505 of the same Code
4. On March 7, 1952, the letter was falsified and converted into an provides that "where goods are sold by a person who is not the owner
authorized deed of sale in favor of Belizo by erasing a portion thereof and thereof, and who does not sell them under authority or with the
adding in its place the words "sold the above car to Mr. Jesusito Belizo of 25 consent of the owner, the buyer acquires no better title to the goods
Valencia, San Francisco del Monte, for Five Thousand Pesos (P5,000)." than the seller had, unless the owner of the goods is by his conduct
5. Armed with this deed of sale, Belizo succeeded in obtaining a certificate of precluded from denying the seller's authority to sell."
registration in his name on the same date, March 7, 1952, and also on the 10. Applying the above legal provisions to the facts of this case, one is
same date, Belizo sold the car to Felixberto Bulahan who in turn sold it to inevitably led to the conclusion that plaintiff Cruz has a better
Reynaldo Pahati, a second hand car dealer. These facts show that the letter right to the car in question than defendant Bulahan for it cannot
was falsified by Belizo to enable him to sell the car to Bulahan for a be disputed that plaintiff had been illegally deprived thereof
valuable consideration. because of the ingenious scheme utilized by Belizo to enable him
to dispose of it as if he were the owner thereof.
With the foregoing, the SC determined that this is a case which involves a confict 11. Cruz therefore can still recover the possession of the car even if
of rights of two persons who claim to be the owners of the same property, plaintiff defendant Bulahan had acted in good faith in purchasing it from
Cruz and defendant Bulahan. Who has, therefore, a better right of the two over Belizo.
the car? 12. Nor can it be pretended that the conduct of Cruz in giving Belizo
a letter to secure the issuance of a new certificate of registration
6. Both were found by the lower court to be innocent and to have acted in constitutes a sufficient defense that would preclude recovery
good faith (NOTE based on syllabus: Although the SC did not expressly because of the undisputed fact that letter was falsified and this fact
declare, nonetheless, in relation to ratio #12, the lack of exercise of reasonable can be clearly seen by a cursory examination of the document. If
diligence on the part of Bulahan also barred him from claiming good faith). Bulahan had been more diligent he could have seen that the
They were found to be the victims of Belizo who falsified the letter given him pertinent portion of the letter had been erased which would have
by Cruz to enable him to sell the car of Bulahan for profit. placed him on guard to make an inquiry as regards the authority
7. The law applicable to the case is Article 559 of the new Civil Code which of Belizo to sell the car. This he failed to do.
provides: 13. The right of the plaintiff to the car in question can also be justified
under the doctrine laid down in U. S. vs. Sotelo, 28 Phil., 147. This is
"ART. 559. The possession of movable property acquired in a case of estafa wherein one Sotelo misappropriated a ring belonging
good faith is equivalent to a title. Nevertheless, one who has lost to Alejandra Dormir.
any movable or has been unlawfully deprived thereof, may 14. In the course of the decision, the Court said that "Whoever may have
recover it from the person in possession of the same. been deprived of his property in consequence of a crime (estafa in
this case) is entitled to the recovery thereof, even if such property
"If the possessor of a movable lost or of which the owner has is in the possession of a third party who acquired it by legal means
other than those expressly stated in Article 464 of the Civil Code
(now art 559)."
15. The Court further said: It is a fundamental principle of our law of
personal property that no man can be divested of it without his
own consent; consequently, even an honest purchaser, under a
defective title, cannot resist the claim of the true owner. The
maxim that 'No man can transfer to another a better title than he has
himself "obtain in the civil as well as in the common law."

Common Law vs. Civil Law (not VERY important)

16. Counsel for appellee places much reliance on the common law
principle that "Where one of two innocent parties must suffer by a
fraud perpetrated by another, the law imposes the loss upon the party
who, by his misplaced confidence, has enabled the fraud to be
committed", and contends that, as between plaintiff and Bulahan, the
former should hear the loss because of the confidence he reposed in
Belizo which enabled the latter to commit the falsification.
17. But this principle cannot be applied to this case, which is covered
by an express provision of our new Civil Code (Art. 559 in this
case). Between a common law principle and a statutory provision, the
latter must undoubtedly prevail in this jurisdiction. Moreover, we
entertain serious doubt if, under the circumstances obtaining, Bulahan
may be considered more innocent than the plaintiff in dealing with the
car in question. We prefer not to elaborate on this matter it being
unnecessary considering the conclusion we have reached.
08 DIZON V. SUNTAY (MAYUMI) 19. It was pledged for P2,600 to Dominador who owns and operates a pawnshop.
September 29, 1972 | Fernando, J. | Sale by Seller Having Voidable Title This was a violation of the terms of the agency.
20. Lourdes insistently demanded from Clarita the return of her ring. Thereafter,
PETITIONER: Dominador Dizon, doing business under the firm name Clarita delivered to the former the pawnshop ticket.
“Pawnshop of Dominador Dizon” 21. When Lourdes found out that Clarita pledged her ring, she took steps to file
RESPONDENTS: Lourdes Suntay a case of estafa against Clarita. Subsequently, Lourdes wrote, through her
lawyer, a letter, asking for the delivery of her ring which was pledged in
SUMMARY: Lourdes entered into a transaction where she delivered her ring to Dominador’s pawnshop.
Clarita who would sell it for commission. Clarita, without the knowledge of 22. Lourdes tried to recover possession thereof from Dominador who refused.
Lourdes, pledged the ring to Dominador who owns a pawnshop. When Lourdes Thus, Lourdes filed an action for its recovery.
was asking for the ring back, Clarita could not comply, but eventually gave the 23. The trial court and CA ruled in her favor based on Article 559 of the Civil
pawnshop ticket. When Lourdes found out that her ring was pledged, she filed a Code. (The lower court issued a writ of replevin)
case of Estafa against Clarita, and wrote Dominador asking that the ring pledged 24. Dominador then elevated this case to the SC.
be delivered back to her. Dominador refused, and so Lourdes filed a writ of 25. The SC notes how ordinarily they would not give due course to such a petition
replevin. The lower courts ruled in favor of Lourdes, thus this petition. The SC for review. However, the vigorous plea grounded on estoppel by
affirmed the lower courts’ decision stating that under Art. 559, one who has lost Dominador’s counsel, Atty. Velarde, persuaded the court to act otherwise.
any movable or has been unlawfully deprived thereof may recover it from the
person in possession of the same. The only exception the law allows is when there ISSUES:
is acquisition in good faith of the possessor at a public sale, in which case the 3. WoN Lourdes has the right to get possession of the ring –YES
owner cannot obtain its return without reimbursing the price. Thus, in this case
Lourdes clearly has the right to have her ring returned to her possession. RULING: WHEREFORE, the decision of the Court of Appeals of May 19, 1969 is
affirmed,
DOCTRINE: Owner of diamond ring may recover it from pawnshop where
owner’s agent had pledged it without authority to do so; Art. 559 applies and the RATIO:
defense that the pawnshop acquired possession without notice of any defect of the 1. The controlling provision is Art. 559 of the CC. The possession of movable
pledgor-agent is unavailing. property acquired in good faith is equivalent to a title. Nevertheless, one who
has lost any movable or has been unlawfully deprived thereof may
recover it from the person in possession of the same. If the possessor of a
Caveat: This is really what the case said. The doctrine placed was taken from the
movable lost of which the owner has been unlawfully deprived, has acquired
syllabus as well. It really didn’t involve any sale transaction.
it in good faith at a public sale, the owner cannot obtain its return without
reimbursing the price paid therefor. xxx (De Gracia v. CA)
FACTS:
2. The right of the owner cannot be defeated even by proof that there was good
16. Clarita Sison and Lourdes Suntay are close friends. In fact, in 1961 Clarita
faith in the acquisition by the possessor. (Cruz v. Pahati)
sold jewelry for Suntay.
3. Suffice it to say in this regard that the right of the owner to recover personal
17. On June 13, 1962, Lourdes and Clarita entered into a transaction wherein
property acquired in good faith by another, is based on his being
Lourdes’s diamond ring was turned over to Clarita for sale on commission
dispossessed without his consent. The common law principle that where
along with other pieces of jewelry (AKA Clarita is the agent of Lourdes). The
one of two innocent persons must suffer by a fraud perpetrated by another,
diamond ring was valued at P5,500. Upon receiving the ring, Clarita executed
the law imposes the loss upon the party who, by his misplaced confidence,
and delivered to Lourdes a receipt.
has enabled the fraud to be committed, cannot be applied in a case which is
18. After the lapse of a considerable time, Lourdes demanded that Clarita return
covered by an express provision of the new Civil Code, specifically Article
her ring, but the latter could not comply because three days after the ring was
559. (Aznar v. Yapdiangco)
received by Clarita, the ring was pledged by Melia (the niece of the husband
4. It must have been a recognition of the compulsion exerted by the above
of Clarita Sison).
authoritative precedents that must have caused petitioner to invoke the
principle of estoppel. There is clearly a misapprehension.
5. For estoppel to exist though, it is indispensable that there be a declaration, act unlawfully deprived thereof, may recover it from the person in possession of the same.
or omission by the party who is sought to be bound. It is equally a requisite If the possessor of a movable lost or of which the owner has been unlawfully deprived,
that he, who would claim the benefits of such a principle, must have altered has acquired it in good faith at a public sale, the owner cannot obtain its return without
his position, having been so intentionally and deliberately led to comport reimbursing the price paid therefor."
himself thus, by what was declared or what was done or failed to be done. A
court is to see to it then that there is no turning back on one's word or a
repudiation of one's act.
6. a person claimed to be estopped must have knowledge of the fact that his
voluntary acts would deprive him of some rights because said voluntary acts
are inconsistent with said rights
7. A party should not be permitted "to go against his own acts to the prejudice
of [another]” (Rodriguez v. Martinez)
8. How then can petitioner in all seriousness assert that his appeal finds support
in the doctrine of estoppel? Neither the promptings of equity nor the mandates
of moral right and natural justice come to his rescue.
9. He is engaged in a business where presumably ordinary prudence would
manifest itself to ascertain whether or not an individual who is offering a
jewelry by way of a pledge is entitled to do so. If no such care be taken,
perhaps because of the difficulty of resisting opportunity for profit, he should
be the last to complain if thereafter the right of the true owner of such jewelry
should be recognized.
10. Moreover, while the activity he is engaged in is no doubt legal, it is not to be
lost sight of that it thrives on taking advantage of the necessities precisely of
that element of our population whose lives are blighted Moreover, while the
activity he is engaged in is no doubt legal, it is not to be lost sight of that it
thrives on taking advantage of the necessities precisely of that element of our
population whose lives are blighted

Teehankee: Concurring
1. Confirms the long settled and uniform jurisprudence based on Art. 559 of the
CC that the owner "who has lost any movable or has been unlawfully
deprived thereof may recover it from the person in possession of the same,"
the only exception expressly provided in the codal article being that "if the
possessor of a movable lost of which the owner has been unlawfully
deprived, has acquired it in good faith at a public sale, the owner cannot
obtain its return without reimbursing the price paid therefor"1
2. Senator Tolentino's submittal in his commentaries on the Civil Code "that the
better view is to consider 'unlawfully deprived' as limited to unlawful
taking, such as theft or robbery, and should not include disposition through
abuse of confidence.

NOTES:
Article 559 of the NCC: "The possession of movable property acquired in good faith
is equivalent to a title. Nevertheless, one who has lost any movable or has been
01 ROMAN v. GRIMALT (Regine)
April 11, 1906 | Torres, J. | Effect of Loss or Deterioration of Thing Sold Before FACTS:
Perfection 1. Counsel for Pedro Roman filed a complaint in the CFI against Andres
PETITIONER: Pedro Roman Grimalt, praying that judgment be entered in his favor and against the
RESPONDENTS: Andres Grimalt defendant (1) for the purchase price of the schooner Santa Marina, to wit,
1,500 pesos or its equivalent in Philippine currency, payable by installments
SUMMARY: Roman was the alleged owner of a schooner Santa Marina, a sailing in the manner stipulated; (2) for legal interest on the installments due on the
vessel. He entered into negotiations with Grimalt, who was interested in purchasing dates set forth in the complaint; (3) for costs of proceedings; and (4) for such
it. They agreed upon the sale of the schooner for the sum of P1,500, payable in three other and further remedy as might be considered just and equitable.
installments, provided the title papers to the vessel were in proper form. This 2. The court made an order sustaining the demurer filed by Grimalt to the
condition was agreed upon because the title to the schooner was not in the name of complaint and allowing Roman ten days within which to amend his
Roman but one Paulina Giron. Roman promised to perfect his title over the schooner. complaint. To this order the Roman duly excepted.
Before Roman could fulfill his promise and make subsequent delivery, the schooner 3. Counsel for Roman amended his complaint and alleged:
sunk in the bay during a severe storm. Roman filed a complaint against Grimalt to a. That between the June 13 and June 23, 1904 both parties, through
demand the purchase price. In his answer, Grimalt contends that he agreed to one Fernando Agustin Pastor, verbally agreed upon the sale of the
purchase the vessel only when the papers were found to be satisfactory. The schooner said schooner;
was not in the name of Roman. Grimalt insisted that Roman perfect his title – but he b. That the Grimalt in a letter dated June 23 had agreed to purchase the
did not do so. He should not be compelled to pay the price since there was no said schooner and offered to pay it in three installment of 500 pesos
perfected contract of sale. Thus, the risk of loss should be borne by Roman. CFI ruled each, on July 15, September 15, and November 15, adding in his
in favor of Grimalt. Issue: WoN Grimalt is liable for the loss. SC said no. A sale shall letter that if the Roman accepted the plan of payment suggested by
be considered perfected and binding as between vendor and vendee when they have him the sale would become effective on the following day;
agreed as to the thing which is the object of the contract and as to the price, even c. That Roman on June 24 notified Grimalt through Agustin Pastor that
though neither has been actually delivered. (Art. 1450 of the Civil Code.) Ownership he accepted the plan of payment suggested by him and that from that
is not considered transmitted until the property is actually delivered and the purchaser date the vessel was at his disposal, and offered to deliver the same
has taken possession of the value and paid the price agreed upon, in which case the at once to Grimalt if he so desired;
sale is considered perfected. Roman, the owner, and Grimalt, the purchaser, had been d. That the contract having been closed and the vessel being ready for
negotiating for the purchase of the schooner Santa Marina. They agreed upon the sale delivery to the purchaser, it sunk about 3 o'clock p. m., June 25, in
of the vessel for the sum of 1,500 pesos, payable in three installments, provided the the harbor of Manila and is a total loss, as a result of a severe storm;
title papers to the vessel were in proper form. The sale of the schooner was not e. And that on the June 30, demand was made upon the Grimalt for the
perfected and the purchaser did not consent to the execution of the deed of transfer payment of the purchase price of the vessel in the manner stipulated
for the reason that the title of the vessel was in the name of one Paulina Giron and and defendant failed to pay.
not in the name of Roman, the alleged owner. Roman promised, however, to perfect 4. Grimalt in his answer asked that the complaint be dismissed. He alleged that:
his title to the vessel, but he failed to do so. The papers presented by him did not a. That both parties met in a public establishment of this city and the
show that he was the owner of the vessel. If no contract of sale was actually executed Roman personally proposed to the Grimalt the sale of the said
by the parties, the loss of the vessel must be borne by its owner and not by a party vessel, Roman stating that the vessel belonged to him and that it was
who only intended to purchase it and who was unable to do so on account of failure then in a sea worthy condition;
on the part of the owner to show proper title to the vessel and thus enable them to b. That Grimalt accepted the offer of sale on condition that the title
draw up the contract of sale papers were found to be satisfactory, also that the vessel was in a
seaworthy condition; that both parties then called on Calixto Reyes,
a notary public, who, after examining the documents, informed them
DOCTRINE: If no contract of sale was actually executed by the parties, the loss of that they were insufficient to show the ownership of the vessel and
the vessel must be borne by its owner and not by a party who only intended to to transfer title thereto;
purchase it c. That Roman then promised to perfect his title and on June 23
``called on Grimalt to close the sale, and Grimalt believing that
Roman had perfected his title, wrote to him on June 23 and set the 7. The vessel was sunk in the bay on the afternoon of June 25, 1904, during a
following day for the execution of the contract, but, upon being severe storm and before the owner had complied with the condition exacted
informed that Roman had done nothing to perfect his title, he by the proposed purchaser, to wit, the production of the proper papers
insisted that he would buy the vessel only when the title papers were showing that the plaintiff was in fact the owner of the vessel in question.
perfected and the vessel duly inspected 8. Grimalt was under no obligation to pay the price of the vessel, the purchase
5. Grimalt also denied the other allegations of the complaint inconsistent with of which had not been concluded. The conversations had between the parties
his own allegations. and the letter written by defendant to plaintiff did not establish a contract
6. CFI ruled in favor of Grimalt and dismissed the complaint of Roman. sufficient in itself to create reciprocal rights between the parties.
7. Hence, this petition 9. It follows, therefore, that article 1452 of the Civil Code relative to the injury
or benefit of the thing sold after a contract has been perfected and articles
1096 and 1182 of the same code relative to the obligation to deliver a
ISSUE/s: WoN Grimalt is liable for the loss. NO. specified thing and the extinction of such obligation when the thing is either
lost or destroyed, are not applicable to the case at bar.
RULING: The judgment of the court below is affirmed and the complaint is dismissed 10. The first paragraph of article 1460 of the Civil Code and section 335 of
with costs against the Roman. the Code of Civil Procedure are not applicable. These provisions
contemplate the existence of a perfected contract which can not, however, be
RATIO: enforced on account of the entire loss of the thing or made the basis of an
action in court through failure to conform to the requisites provided by law.
1. A sale shall be considered perfected and binding as between vendor and
vendee when they have agreed as to the thing which is the object of the
NOTE:
contract and as to the price, even though neither has been actually delivered.
(Art. 1450 of the Civil Code.)
2. Ownership is not considered transmitted until the property is actually 1. This case was decided under the Spanish Civil Code (April 11, 1906). It
delivered and the purchaser has taken possession of the value and paid the exemplifies the Civil Law concept of risk of loss (Roman law is embodied in
price agreed upon, in which case the sale is considered perfected. the Spanish Civil Code). If the would-be subject matter is lost before
3. When the sale is made by means of a public instrument the execution thereof perfection, the owner bears the loss.
shall be equivalent to the delivery of the thing which is the object of the
contract. (Art. 1462 of the Civil Code.)
4. Pedro Roman, the owner, and Andres Grimalt, the purchaser, had been for
several days negotiating for the purchase of the schooner Santa Marina —
from June 13 to June 23, 1904 They agreed upon the sale of the vessel for the
sum of 1,500 pesos, payable in three installments, provided the title papers to
the vessel were in proper form. It is so stated in the letter written by the
purchaser to the owner on June 23, 1904.
5. The sale of the schooner was not perfected and the purchaser did not
consent to the execution of the deed of transfer for the reason that the title
of the vessel was in the name of one Paulina Giron and not in the name of
Pedro Roman, the alleged owner. Roman promised, however, to perfect his
title to the vessel, but he failed to do so. The papers presented by him did not
show that he was the owner of the vessel.
6. If no contract of sale was actually executed by the parties the loss of the
vessel must be borne by its owner and not by a party who only intended
to purchase it and who was unable to do so on account of failure on the part
of the owner to show proper title to the vessel and thus enable them to draw
up the contract of sale.
02 LAWYERS COOP VS TABORA(Mark) 6. Tabora failed to pay the monthly installments, the LCPC demanded payment
April 30, 1965 |Bautista Angleo J. | loss after delivery of the installments due. Tabora did not pay.
7. LCPC filed an action before the Court of First Instance of Manila for the
PETITIONER: LAWYERS COOPERATIVE PUBLISHING COMPANY recovery of the balance of the obligation. LCPC also prayed that Tabora be
RESPONDENTS: PERFECTO A. TABORA ordered to pay 25% of the amount due as liquidated damages, and the cost of
action.
SUMMARY: Tabora bought books from LCPC and it was delivered to him, and 8. Tabora, in his answer, pleaded force majeure as a defense. He alleged that
payment was in installments. A fire burned the books. Tabora did not pay the the books bought from the LCPC were burned during the fire and since the
installments and LCPC filed a case against him. Tabora used force majure as a loss was due to force majeure he cannot be held responsible for the loss. He
defense that he is exempt from liability. In the contract it indicated that ownership prayed that the complaint be dismissed and that he be awarded moral
of the books remain with seller until full payment and loss after delivery would be damages in the amount of P15,000.00.
borne by the buyer. The issue is WoN Tabora is liable to pay for the books, the SC 9. The CFI ruled in favor of LCPC. It ordered the Tabora to pay the sum of
held that he is liable and that even though generally loss of the thing is borne by the P1,382.40, with legal interest thereon from the filing of the complaint, plus a
owner, the contract between the parties are binding and Art 1504 applies. Also force sum equivalent to 25% of the total amount due as liquidated damages, and
majure applies only to determinate things, and since the books were not the cost of action.
determinate, he is not exempt to pay. 10. Tabora took the case to the CA, but the same is now before us by virtue of a
certification issued by that Court that the case involves only questions of law.
DOCTRINE: Article 1504 of our Civil Code provides: 11. Tabora bought from Lawyers Coop one set of American Jurisprudence,
Where delivery of the goods has been made to the buyer or to a bailee for the buyer, including one set of general index, payable on installment plan. It was
in pursuance of the contract and the ownership in the goods has been retained by provided in the contract that "title to and ownership of the books shall remain
the seller merely to secure performance by the buyer of his obligations under the with the seller until the purchase price shall have been fully paid. Loss or
contract, the goods are at the buyer's risk from the time of such delivery. damage to the books after delivery to the buyer shall be borne by the buyer."
The total price of the books, including the cost of freight, amounts to
P1,682.40. Tabora only made a down payment of P300.00 thereby leaving a
FACTS: balance of P1,382.40.
1. On May 3, 1955, Perfecto A. Tabora bought from the Lawyers Cooperative 12. Tabora now contends that since it was agreed that the title to and the
Publishing Company (LCPC) one complete set of American Jurisprudence ownership of the books shall remain with the seller until the purchase price
consisting of 48 volumes with 1954 pocket parts, plus one set of American shall have been fully paid, and the books were burned or destroyed
Jurisprudence, General Index, consisting of 4 volumes, for a total price of immediately after the transaction, LCPC should be the one to bear the loss
P1,675.50 which, in addition to the cost of freight of P6.90, makes a total of for, as a result, the loss is always borne by the owner.
P1,682.40. 13. Even assuming that the ownership of the books were transferred to Tabora
2. Tabora made a partial payment of P300.00, leaving a balance of P1,382.40. after the perfection of the contract he should not answer for the loss since the
The books were duly delivered and receipted for by Tabora in his law office same occurred through force majeure. There is no evidence that Tabora has
in Naga City contributed in any way to the occurrence of the conflagration.
3. On the same day a big fire broke out which destroyed and burned all the
buildings standing on one whole block including at the law office and library ISSUE: WoN Tabora is liable to pay for the books. -YES
of Tabora.
4. The books bought from the LCPC as above stated, together with Tabora's RULING: WHEREFORE, the decision appealed from is modified by eliminating that
important documents and papers, were burned during the conflagration. portion which refers to liquidated damages. No costs.
5. This unfortunate event was immediately reported by Tabora to the LCPC in
a letter he sent on May 20, 1955. On May 23, the LCPC replied and as a token RATIO:
of goodwill it sent to Tabora free of charge volumes 75, 76, 77 and 78 of the 1. While as a rule the loss of the object of the contract of sale is borne by the
Philippine Reports. owner or in case of force majeure the one under obligation to deliver the
object is exempt from liability, it cannot be applied here because the law on
the contract entered into.
2. The contract entered into between the parties LCPC agreed that the
ownership of the books shall remain with it until the purchase price shall have
been fully paid, but such stipulation cannot make the it liable in case of loss
not only because such was agreed merely to secure the performance by the
buyer of his obligation but in the very contract it was expressly agreed that
the "loss or damage to the books after delivery to the buyer shall be borne by
the buyer."
3. Any such stipulation is sanctioned by Article 1504 of our Civil Code, which
in part provides:
(1) Where delivery of the goods has been made to the buyer or to a
bailee for the buyer, in pursuance of the contract and the ownership
in the goods has been retained by the seller merely to secure
performance by the buyer of his obligations under the contract, the
goods are at the buyer's risk from the time of such delivery.
4. Tabora’s defense of force majure would not apply in this case. The rule only
applies when the obligation consists in the delivery of a determinate thing
and there is no stipulation holding him liable even in case of fortuitous event.
Here these qualifications are not present. The obligation does not refer to a
determinate thing, but is pecuniary in nature, and the obligor bound himself
to assume the loss after the delivery of the goods to him. In other words,
Tabora agreed to assume any risk concerning the goods from the time of their
delivery, which is an exception to the rule provided for in Article 1262 of our
Civil Code.
5. Tabora likewise contends that the CFI erred in sentencing him to pay
attorney's fees. This is merely the result of a misapprehension for what the
court a quo ordered Tabora to pay is not 25% of the amount due as attorney's
fees, but as liquidated damages, which is in line with an express stipulation
of the contract.
6. We believe, however, that the Tabora should not be made to pay any damages
because his denial to pay the balance of the account is not due to bad faith.
01 Delta Motor Sales Corp v. Niu Kim Duan (Izzy) 4. After paying almost P6,966, NKD failed to pay at least two monthly
September 2, 1992 | Nocon, J. | Unpaid seller’s remedies not cumulative installments. The remaining unpaid obligation of NKD amounted to P12,920.
5. The statements of accounts were sent to defendants NKD, and the Delta's
collectors personally went to the former to effect collections but they failed
PLAINTIFF-APPELLEE: Delta Motor Sales Corporation
to do so.
DEFENDANTS-APPELLANTS: Niu Kim Duan and Chan Fue Eng
6. Delta tried to recover extrajudicially, but it failed to do so.
7. Delta prayed for the issuance of a wire of replevin, which the Court granted
SUMMARY: Niu Kim Duan purchased from Delta Motors 3 air conditioning
in its Order dated February 28, 1977.
units. Niu paid the downpayment, the balance payable in 24 installments. Title to
8. In view of the failure of the defendants to pay their obligations, the amount
the property remained with Delta until the payment of the full purchase price.
of P6,966.00 which had been paid by way of installments were treated as
Under the agreement, failure to pay 2 monthly installments makes the obligation
rentals for the units in question for two (2) years pursuant to the provisions
entirely due and demandable. The units were delivered, Niu failed to pay. Thus,
of paragraph 5 of the Deed of Conditional Sale.
Delta filed a complaint for Replevin and applied the installments paid by Niu as
9. The trial court ruled in favour of Delta.
rentals; Niu contends that the contractual stipulations are unconscionable. Issue
10. NKD assails the Deed of Conditional Sale under which they purchased the
is WoN the remedy Delta availed of was unconscionable. The Court said NO. A
three (3) Daikin air-conditioners from plaintiff- appellee as being contrary to
stipulation in the contract treating installments as rentals in case of failure to pay
law, morals, good custom, public order or public policy. In particular, they
is VALID—so long as they are not unconscionable. The provision in this case is
point to the contract’s paragraphs 5 and 7 as iniquitous and unconscionable.
reasonable.
11. Defendants-appellants claim that for the use of the plaintiff-appellee’s three
air-conditioners, from July 5, 1975 to April 11, 1977, or for a period of about
DOCTRINE: An unpaid seller has 3 alternative (not cumulative) remedies: (1)
22 months, they, in effect, paid rentals in the amount of P6,429.92, or roughly
exact fulfillment of the obligation, (2) cancel the sale for default in 2 installments,
one-third (1/3) of the entire price of said air- conditioners which was
and (3) foreclose the chattel mortgage (if any) but the seller cannot anymore claim
P19,350.00.
the unpaid balance of the price. Delta chose the 2nd remedy. Having done so, it is
now barred from claiming the balance of the purchase price. ISSUE/S:
1. WoN the remedy that Delta availed of is unconscionable – NO

FACTS: RULING: WHEREFORE, the judgment of the trial court in Civil Case No. 25578 is
1. On July 5, 1975, Niu Kim Duan (NKD), and Chan Fue Eng, the defendants, hereby SET ASIDE and the complaint filed by plaintiff- appellee Delta Motor Sales
purchased from the plaintiff three (3) units of ‘DAIKIN’ air-conditioner all Corporation is hereby DISMISSED. No costs.
valued at P19,350.00 as evidenced by the Deed of Conditional Sale.
Aforesaid deed of sale had the following terms and conditions: RATIO:
a. NKD would pay a downpayment of 774 and the balance of 18k+ 1. A stipulation in a contract that the installments paid shall not be
shall be paid in 24 monthly installments; returned to the vendee is valid insofar as the same may not be
b. Title to properties would remain with Delta until the price is fully unconscionable under the circumstances is sanctioned by Article 1486 of
paid; the New Civil Code.
c. If any 2 installments are not paid by NKD on their due dates, the the 2. The monthly installment payable by defendants-appellants was P774.00. The
whole of the principal sum remaining unpaid would become due; P5,655.92 installment payments correspond only to seven (7) monthly
d. In case of a suit, the defendants shall pay an amount equivalent to installments.
25% of the remaining unpaid obligation as damages, penalty and 3. Since they admit having used the air- conditioners for twenty-two (22)
attorney’s fees; months, this means that they did not pay fifteen (15) monthly installments on
2. To secure the payment of the balance (P18,576), NKD jointly and severally the said air-conditioners and were thus using the same FREE for said
executed in favor of Delta a promissory note. period—to the prejudice of plaintiff-appellee. Under the circumstances, the
3. The airconditioners were delivered and received as shown by a delivery treatment of the installment payments as rentals cannot be said to be
receipt. unconscionable.
4. The vendor in a sale of personal property payable in installments may attorney’s fees;
exercise one of three remedies, namely, (1) exact the fulfillment of the
obligation, should the vendee fail to pay; (2) cancel the sale upon the vendee’s
failure to pay two or more installments; (3) foreclose the chattel mortgage, if
one has been constituted on the property sold, upon the vendee’s failure to
pay two or more installments.
5. The third option or remedy, however, is subject to the limitation that the
vendor cannot recover any unpaid balance of the price and any agreement to
the contrary is void (Art. 1484).
6. The three (3) remedies are alternative and NOT cumulative. If the creditor
chooses one remedy, he cannot avail himself of the other two.
7. It is not disputed that the plaintiff-appellee, Delta, had taken possession
of the three air-conditioners, through a writ of replevin when
defendants- appellants, NKD, refused to extra-judicially surrender the
same.
8. This was done pursuant to paragraphs 5 and 7 of its Deed of Conditional Sale
when defendants-appellants failed to pay at least two (2) monthly
installments, so much so that as of January 6, 1977, the total amount they
owed plaintiff-appellee, inclusive of interest, was P12,920.08.
9. The case Delta filed was to seek a judicial declaration that it had validly
rescinded the Deed of Conditional Sale. Clearly, Delta chose the second
remedy of Article 1484 in seeking enforcement of its contract with
defendants-appellants. This is shown from the fact that its Exhibit “F” which
showed the computation of the outstanding account of defendants-appellants
as of October 3, 1977 took into account “the value of the units repossessed.”
10. Having done so, it is barred from exacting payment from defendants-
appellants of the balance of the price of the three air-conditioning units
which it had already repossessed. It cannot have its cake and eat it too.

Just in case sir asks: Paragraphs 5 & 7 of their contract

Paragraph 5: Should BUYER fail to pay any of the monthly installments when due, or
otherwise fail to comply with any of the terms and conditions herein stipulated, this
contract shall automatically become null and void; and all sums so paid by BUYER
by reason thereof shall be considered as rental and the SELLER shall then and there
be free to take possession thereof without

Paragraph 7: Should SELLER rescind this contract for any of the reasons stipulated in
the preceding paragraph, the BUYER, by these presents obligates himself to
peacefully deliver the PROPERTY to the SELLER in case of rescission, and should a
suit be brought in court by the SELLER to seek judicial declaration of rescission and
take possession of the PROPERTY, the BUYER hereby obligates himself to pay all
the expenses to be incurred by reason of such suit and in addition to pay the sum
equivalent to 25% of the remaining unpaid obligation as damages, penalty and
02 TAJANLANGIT V. SOUTHERN MOTORS, INC. (CASTRO) 3. The note stipulated that if default be made in the payment of interest or of
28 May 1957| Bengzon, J. | Remedies of Vendor on Installment any installment, then the total principal sum still unpaid with interest shall at
once become demandable
PETITIONER: Amador Tajanlangit, Et. al 4. The spouses Tajanlangit failed to meet any installment. They were sued for
RESPONDENTS: Southern Motors, Inc. et. al the amount of the promissory note
5. The spouses defaulted, and the court after listening to the Southern Motors’
SUMMARY: April 1953, Amador Tajanlangit and his wife Angeles, residents of evidence entered judgment in the amount stipulated in the promissory note
Iloilo bought from Southern MOTORS inc. two tractors and a thresher. In payment together with interest at 12% per annum of the total amount due as attorney’s
for the same, they executed the promissory note whereby they undertook to satisfy fees and costs of collection
the total purchase price of P24,755.75 in several installments with interest 6. Carrying out the order of execution, the sheriff levied on the same
payable. The note stipulated that if default be made in the payment of interest or machineries and farm implements which had been bought by the spouses and
of any installment, then the total principal sum still unpaid with interest shall at later sold them at public auction to the highest bidder – which turned out to
once become demandable. The spouses Tajanlangit failed to meet any installment. be Southern Motors itself for the total sum of P10,000
They were sued for the amount of the promissory note. Still, the spouses defaulted 7. As its judgment called for much more, the Southern Motors subsequently
and the court entered a judgment ordering the spouses to pay the amount stipulated asked and obtained, an alias writ of execution and pursuant thereto, the
in the PN together with 12% interest. The sheriff levied on the same machineries provincial sheriff levied attachment on the Tajanlangits’ rights and interests
and implements bought by the spouses and it was sold at public acution. The in certain real properties – with a view to another sale on execution
highest bidder was Southern Motors. However, the auction sale was only able to 8. To prevent such sale, the Tajanlangits instituted this action in the Iloilo court
raise 10,000php so a judgment was called for much more, Southern Motors of first instance for the purpose among others, of annulling the alias writ of
subsequently asked and obtained writ of execution for other real properties of execution and all proceedings subsequent thereto. Their two main theories:
Tajanlangits. To prevent the sale the spouses filed before CFI of Iloilo an (1) They had returned the machineries and farm implements to the Southern
annulment of the writ of execution. Southern motors denied alleged settlement and Motors Inc., the latter accepted them, and had thereby settled their accounts;
understanding. CFI ruled against the spouses for the reason that CFI Branch IV for that reason, said spouses did not contest the action in Civil Case, (2) as
cannot invalidate the writ of execution ordered by CFI Branch I. The matter was the Southern Motors Inc. had repossessed the machines purchased on
brought to the CA but for expediency it was brought to the SC. installment (and mortgaged) the buyers were thereby relieved from further
responsibility, in view of the Recto Law, now article 1484 of the New Civil
The SC ruled that indeed Southern Motors as a vendor on installment can still Code.
claim beyond from the proceeds of the auction sale. Furthermore, the spouses 9. The company denied the alleged “settlement and understanding” during the
Tajanlangit was sued on the basis of the PN and other properties not the ones pendency of the civil case. It also denied having repossessed the machineries,
mortgaged hence, Southern motors cannot be deprived of its right to collect from the truth being that they were attached by the sheriff and then deposited by
the other properties. the latter in its shop for safekeeping before the the sale at public auction
10. The CFI mentioned that it cannot grant the action sought by Tajanlangit for
DOCTRINE: Southern motors had a right to select among the three remedies the CFI Branch IV of Iloilo cannot invalidate the judgment or orders of
provided in Article 1484 in choosing to sue on the note, it was not thereby limited another CFI which is CFI Branch I who ordered the sale at public auction
to the proceeds of the sale, on execution, of the mortgaged good 11. The matter was brought to the CA, but the latter forwarded the expediente,
being of the opinion that th appeal involved questions of jurisdiction and/or
law
FACTS: 12. Hence this petition
1. In April 1953, Amador Tajanlangit and his wife Angeles, residents of Iloilo
bought from Southern MOTORS inc. of Iloilo two tractors and a thresher. ISSUE/s:
2. In payment for the same, they executed the promissory note whereby they
• WoN Southern Motors as a vendor on installment is limited to the proceeds
undertook to satisfy the total purchase price of P24,755.75 in several
of the sale? - No
installments with interest payable on stated dated (May 18, 1953 to December
10, 1955)
RULING: The decision dismissing the complaint, is affirmed, with costs against
appellants. So ordered. to foreclose the mortgage on the truck. . . .

As the plaintiff has chosen to exact the fulfillment of the defendant's


RATIO: obligation, the former may enforce execution of the judgement rendered in
its favor on the personal and real properties of the latter not exempt from
1. The Tajanlangits would invoke the last paragraph of Art. 148424, but there
execution sufficient to satisfy the judgment. That part of the judgement
has been no foreclosure of the chattel mortgage nor a foreclosure sale
depriving the plaintiff of its right to enforce judgment against the
(NOTE: Southern motors sued Tajanlangit on the basis of the notes,
properties of the defendant except the mortgaged truck and discharging
furthermore what was sold in the auction are the threshers bought not the
the writ of attachment on his other properties is erroneous
properties mortgaged) Therefore the prohibition against further collection
does not apply

2. At any rate it is the actual sale of the mortgaged chattel in accordance with
Act no 1508 that would bar the creditor (who chooses to foreclose) from
recovering any unpaid balance

3. It is true that there was a chattel mortgage on the goods sold, but the
Southern motors elected to sue on the note exclusively i.e to exact
fulfillment of the obligation to pay.

4. Southern motors had a right to select among the three remedies provided
in Article 1484 in choosing to sue on the note, it was not thereby limited
to the proceeds of the sale, on execution, of the mortgaged good.

5. In Southern Motors Inc. vs. Magbanua, (100 Phil., 155) a similar situation
arose in connection with the purchase on installment of a Chevrolet truck by
Magbanua. Upon the latter's default, suit on the note was filed, and the truck
levied on together with other properties of the debtor. Contending that the
seller was limited to the truck, the debtor obtained a discharge of the other
properties. This court said:

By praying that the defendant be ordered to pay the sum of P4,690 together
with the stipulated interest at 12% per annum from 17 March 1954 until fully
paid, plus 10 per cent of the total amount due as attorney's fees and cost of
collection, the plaintiff acted to exact the fulfillment of the obligation and not

24
ART. 1484. In a contract of sale of personal property the price of which is payable in installments, the (2) Cancel the sale, should the vendee's failure to pay cover two or more installments;
vendor may exercise of the following remedies: (3) Foreclose the chattel mortgage on the thing sold, if one has been constituted, should the vendee's failure
to pay cover two or more installments. In this case, he shall have no further action against the purchaser to
(1) Exact fulfillment of the obligation, should the vendee fail to pay; recover any unpaid balance of the price. Any agreement to the contrary shall be void
03 NONATO v. IAC (KC) was repossessed by IFC.
November 22, 1985 | Escolin, J. | Recission 152. IFC subsequently demanded from Spouses Nonato to pay the balance of the
price of the car saying that the value of the car was not sufficient to cover the
PETITIONER: SPOUSES RESTITUTO NONATO and ESTER NONATO balance of the purchase price. IFC then filed a case before the CFI of Negros
RESPONDENTS: THE HONORABLE INTERMEDIATE APPELLATE Occidental.
COURT and INVESTOR'S FINANCE CORPORATION 153. Spouses Nonato in their defense alleges that when the company repossessed
the vehicle, it had effectively cancelled the sale of the vehicle and therefore
SUMMARY: Spouses Nonato bought a vehicle from People’s Car, Inc on is barred from exacting recover of the unpaid balance.
installment basis. They executed a promissory note and chattel mortage in favor 154. The trial court rendered a decision in favor of IFC. The IAC likewise affirmed
of People’s Car, Inc. to ensure payment of the price. Subsequently, Peopler’s Car, this decision. Hence, this appeal.
Inc. assigned its rights and interests over the P/N and mortgage to IFC. The
Spouses weren’t able to pay the installments despite demands so IFC repossessed ISSUE/s: Whether a vendor who had cancelled the sale of a motor vehicle for failure
the car. IFC later on instituted a case against the Spouses to demand payment of of the buyer to pay two or more of the stipulated installments, may also demand
the balance of the purchase price because the value of the car was not sufficient to payment of the balance of the purchase price. - NO
cover the entire amount. Both the RTC and IAC ruled in favor of IFC. The
Supreme Court on the other hand reversed the decision said that the remedies in RULING: WHEREFORE, the judgment of the appellate court in CAG.R. No. 69276-
Art. 1484 are alternative and NOT cumulative; such that the exercise of one R is hereby set aside and the complaint filed by respondent Investors Finance
excludes the exercise of the others. In the case at bar, when IFC repossessed the Corporation against petitioner in Civil Case No. 13852 should be, as it is hereby,
car without returning the vehicle upon appraisal of the current amount, the SC held dismissed. No costs.
that this was tantamount to a cancellation of the sale. This was also evidence of
SO ORDERED.
the receipt issued by IFC to the Spouses wherein it said that the Spouses may
redeem the said vehicle within 15 days. Furthermore, the witness for IFC also
RATIO:
confirmed that the car wouldn’t be returned if the Spouses failed to pay the
1. The applicable law in the case at bar is Article 1484 as it involves a sale of
balance.
personal property on installment:
“In a contract of sale of personal property the price of which is payable in
DOCTRINE: Jurisprudence provides that Art. 1484 means: Should the vendee or
purchase of a personal property default in the payment of two or more of the installments, the vendor may exercise any of the following remedies:
agreed installments, the vendor or seller has the option to avail of any of these
three remedies: (1) Exact fulfillment of the obligation, should the vendee fail to pay;
a. Exact fulfillment by the purchaser of the obligation
b. Cancel the sale (2) Cancel the sale, should the vendee's failure to pay cover two or more
c. Foreclose the mortgage on the purchased personal property installments;
These remedies have been recognized as alternative, NOT cumulative. Thus, the
exercise of one would bar the exercise of the others. (3) Foreclose the chattel mortgage on the thing sold, if one has been
constituted, should the vendee's failure to pay cover two or more installments.
FACTS: In this case, he shall have no further action against the purchaser to recover
150. Spouses Restituto Nonato and Ester Nonato purchased one unit of any unpaid balance of the price. Any agreement to the contrary shall be void.”
Volkswagen Sakbayan from People’s Car, Inc. on installment basis. To 132. Jurisprudence provides that Art. 1484 means: Should the vendee or purchase
secure the payment thereof, they executed a promissory note and a chattel of a personal property default in the payment of two or more of the agreed
mortgage in favor of said company. installments, the vendor or seller has the option to avail of any of these three
151. People’s Car, Inc. assigned its rights and interests over the note and the remedies
mortgage in favor of Investor’s Finance Corporation (IFC). For failure of the a. Exact fulfillment by the purchaser of the obligation
Spouses Nonato to pay two or more instalmments, despite demands, the car b. Cancel the sale
c. Foreclose the mortgage on the purchased personal property
These remedies have been recognized as alternative, NOT cumulative. Thus,
the exercise of one would bar the exercise of the others.
133. It is not disputed that IFC had taken possession of the car. Spouses Nonato
contend that IFC exercised its option to cancel the contract of sale, IFC on
the other hand denies such and alleges that the repossession was only for the
purpose of appraising its value and for storage and safekeeping pending full
payment of the purchasing price.
134. However, the receipt issued by IFC to Spouses Nonato proves otherwise
because it said that the vehicle could be redeemed within 15 days. This could
only mean that should the Spouses Nonato fail to redeem, IFC would retain
permanent possession of the vehicle, as it did in fact. Also, this had been
confirmed by Mr. Ernesto Carmona (IFC’s witness) who testified:
“ATTY. PAMPLONA:

So that Mr. Witness, it is clear now that, per your receipt and your answer,
the company will not return the unit without paying a sum of money, more
particularly the balance of the account?

WITNESS: Yes, sir.”


135. The contention of IFC that the repossession was for mere purpose of
appraising the current value is also untenable for even after it notified
Spouses Nonato the value of the car, it did not make any attempts to return
the vehicle.
136. Since the acts performed by IFC are wholly consistent with the conclusion
that it had opted to cancel the contract of sale, it is thus bared from exacting
payment from Spouses Nonato of the balance of the price.
04 Zayas, Jr. v. Luneta Motor Company (Paolo) Luneta Motor Company filed a civil case for the recovery of the balance of
October 23, 1982 | Justice Gutierrez Jr. | Remedy of Foreclosure P1,551.74 plus interest,
16. Zayas countered that he had no more outstanding liabilities because
PETITIONER: Eutropio Zayas Jr. according to Art. 148425 of the New Civil Code, his obligation found in the
RESPONDENTS: Luneta Motor Company and Hon. Juan Reyes, CFI of Manila promissory note was extinguished by the sale at public auction of the motor
vehicle.
SUMMARY: Zayas bought a Ford Thames from Escano Enterprises, the dealer of 17. Luneta Motor Company denied the applicability of Art.1484 of the Civil
Luneta Motor Company. Zayas paid the downpayment and was supposed to pay the Code ... for the simple reason that the contract involved between the parties
balance in 26 installments. Zayas mortgaged the same car in favor of LMC. Zayas is not one for a sale on installment.
eventually stopped paying the installments and LMC foreclosed the chattel mortgage 18. It maintains that the contract between the company and Zayas was only an
and bought the car in a public auction. LMC wanted to collect the rest of what was ordinary loan removed from the coverage of Art. 1484. (So Luneta Motor
due from Zayas. The SC ruled that Art. 1484 was clear and supported by Company insists that Zayas bought the car from Escano Enterprises and that
jurisprudence, when a vendee used the remedy of foreclosure over the property LMC only financed Zayas through a loan)
mortgaged, he is barred from collecting the deficiency from the foreclosure sale. 19. The City Court ruled in favor of Zayas, but the CFI remanded the case, so
Zayas appealed to the SC because of this order to remand the case.
DOCTRINE: Remedy of Foreclosure – When the seller assigns his credit to
another person, assignee is likewise bound by the same law. ISSUES:
12. Whether or not a deficiency amount after the motor vehicle, subject of
FACTS: the chattel mortgage, has been sold at public auction could still be
11. Eutropio Zayas (Zayas) purchased a motor vehicle, which was a Ford Thames recovered – No, the auction sale extinguishes the obligation.
Freighter with a PUJ Body, on installment basis, from Mr. Roque Escaño of
the Escaño Enterprises in Cagayan de Oro City, dealer of respondent Luneta RULING: WHEREFORE, the instant petition is hereby granted. The orders
Motor Company (LMC). remanding the case are annulled and the appeal of LMC is dismissed.
12. The terms and conditions stated that the total selling price was P8,926.82,
with a payment on delivery (downpayment) of P1,006.82. The rest of the RATIO:
selling price amounting to P7,920.00 would be payable in 24 months at 12%
interest per annum. • The Escaño Enterprises of Cagayan de Oro City was an agent of Luneta
13. The motor vehicle was delivered to Zayas and so he paid the initial paymentt Motor Company. The proof of this relationship is the certification from the
of P1,006.82, executed a Promissory Note for the P7,920.00, which was the cashier of Escano Enterprises on the monthly installments paid by Zayas. In
balance of the purchase price, which stated the amounts and dates of payment the certification, the promissory note in favor of Luneta Motor Company was
for twenty-six (26) installments for the balance of the price. He also executed specifically mentioned.
a chattel mortgage on the motor vehicle in favor of the Luneta Motor • Escano Enterprises, a dealer of Luneta Motor Company, was merely a
Company. collecting-agent as far as the purchase of the subject motor vehicle was
14. Zayas was only able pay the amount of P3,148.00 when he stopped paying concerned. The principal and agent relationship is clear.
the monthly installments. This prompted the Luneta Motor Company to • But even assuming that the "distinct and independent entity" theory of LMC
extra-judicially foreclose the mortgage and the company was the highest is valid, the nature of the transaction as a sale of personal property on
bidder in the public auction for P5,000. installment basis remains. (Escano assigned its “vendor rights” to Luneta
15. Since the foreclosure could not cover the total amount of the promissory note, Motor Company, so still a sale on installment)

• 25Art. 1484. In a contract of sale of personal property the price of which is payable in (3) Foreclose the chattel mortgage on the thing sold, if one has been constituted, should the
installments, the vendor may exercise any of the following remedies: vendee's failure to pay cover two or more installments. In this case, he shall have no further
(1) Exact fulfillment of the obligation, should the vendee fail to pay; action against the purchaser to recover any unpaid balance of the price. Any agreement to
the contrary shall be void.
(2) Cancel the sale, should the vendee's failure to pay cover two or more installments;
• The established rule is to the effect that the foreclosure and actual sale of a
mortgaged chattel bars further recovery by the vendor of any balance on the
purchaser's outstanding obligation not satisfied by the sale. And the reason
for this doctrine was aptly stated in the case of Bachrach Motor Co. vs.
Millan:
◦ This prevents mortgagees from seizing the mortgaged property, buying
it at foreclosure sale for a low price and then bringing suit against the
mortgagor for a deficiency judgment, avoiding a situation where the
mortgager would lose the property because of foreclosure and still owing
the amounts not satisfied by the auction sale.
05 RIDAD v. FILIPINAS INVESTMENT (Elach) Barring effect would cover a third-party mortgage, when it was the chattel
January 27, 1983 | De Castro, J. | Foreclosure; chattel mortgage mortgage that was first foreclosed

PLAINTIFFS-APPELLESS: Luis Ridad and Lourdes Ridad


FACTS:
DEFENDANTS-APPELLANTS: Filipinas Investment and Finance
13. Ridad purchased from the Supreme Sales Development Corporation two (2)
Corporation, Jose D. Sebastian and Jose San Agustin, in his capacity as Sheriff
brand new Ford Consul Sedans complete with accessories, for P26,887
payable in 24 monthly installments.
SUMMARY: The spouses Ridad purchased from the Supreme Sales
14. To secure payment thereof, plaintiffs executed on the same date a promissory
Development Corporation 2 brand new Ford Consul Sedans complete with note covering the purchase price and a deed of chattel mortgage not only on
accessories. To secure payment thereof, plaintiffs executed on the same date a the two vehicles purchased but also on another car (Chevrolet) and
promissory note covering the purchase price and a deed of chattel mortgage not Ridad’s franchise or certificate of public convenience granted by the
only on the two vehicles purchased but also on another car (Chevrolet) and their defunct Public Service Commission for the operation of a taxi fleet.
franchise or certificate of public convenience granted by the defunct Public 15. Then, with the conformity of the plaintiffs, the vendor (Supreme Sales)
Service Commission for the operation of a taxi fleet with Filipinas Investment. assigned its rights, title and interest to the above-mentioned promissory note
and chattel mortgage to defendant Filipinas Investment and Finance
Due to the failure of the plaintiffs to pay their monthly installments as per
Corporation.
promissory note, Filipinas Investment foreclosed on the chattel mortgage on the 16. Due to the failure of Ridad to pay their monthly installments as per
Ford Consul Sedans. The foreclosure sale had a deficiency. Consequently, the promissory note, Filipinas Investment foreclosed the chattel mortgage extra-
corporation foreclosed the mortgage constituted on the Chevrolet and their judicially, and at the public auction sale of the two Ford Consul cars, of which
franchise or certificate of public convenience. Issue in this case is Whether the plaintiffs were not notified, in which the Filipinas Investment was the
foreclosing the second mortgage to recover the deficiency on the first mortgage is highest bidder and purchaser.
17. Another auction sale was held, involving the remaining properties subject
valid. According to the SC, No. Article 1484 of the Civil Code is applicable. Under
of the deed of chattel mortgage since Ridad’s obligation was not fully
this article, the vendor of personal property the purchase price of which is payable satisfied by the sale of the aforesaid vehicles, and at the public auction sale,
in installments, has the right, should the vendee default inthe payment of two or the franchise of plaintiffs to operate five units of taxicab service was sold
more of the agreed installments, to exact fulfillment by the purchaser of the for P8,000 to the highest bidder, herein defendant corporation, which
obligation, or to cancel the sale, or to foreclose the mortgage on the purchased subsequently sold and conveyed the same to herein defendant Jose D.
personalproperty, if one was constituted. The vendor can only choose one option. Sebastian, who then filed with the Public Service Commission an application
If the vendor avails himself of the right to foreclose the mortgage, the law prohibits for approval of said sale in his favor.
18. Thus, Ridad filed an action for annulment of contract before the CFI against
him from further bringing an action against the vendee for the purpose of
Filipinas Investment and Finance Corporation, Jose D. Sebastian and Sheriff
recovering whatever balance of the debt secured is not satisfied by the foreclosure Jose San Agustin, as party-defendants.
sale. Purpose of the law is to prevent mortgagees from seizing the mortgaged 19. By agreement of the parties, the case was submitted for decision in the lower
property, buying it at foreclosure sale for a low price and the bringing suit against court on the basis of the documentary evidence adduced by the parties during
the mortgagor for a deficiency judgment. Without the law, the mortgagor-buyer the pre-trial conference. Thereafter, the lower court rendered judgment
would find himself without the property and still owing practically the full amount declaring to be null and void in so far as the taxicab franchise and the
of his original debt. used Chevrolet car of plaintiffs are concerned, and the sale at public
auction conducted by the City Sheriff of Manila concerning said taxicab
DOCTRINE: If the vendor avails himself of the right to foreclose his mortgage, franchise.
the law prohibits him from further bringing an action against the vendee for the 20. Defendants went to the CA which certified the said appeal to the SC
purpose of recovering whatever balance of the debt secured not satisfied by the
ISSUE/s
foreclosure sale; and
W/N the chattel mortgage in so far as the franchise and the subsequent sale of the
Chevrolet are valid? – NO Hermanos, Inc. vs. Pacific Commercial Co., the facts of which are similar to
those in the case at bar.
RULING: WHEREFORE, the petition is DISMISSED for lack of merit with costs a. There, we have the same situation wherein the vendees offered as
against petitioner. security for the payment of the purchase price not only the motor
vehicles which were bought on installment, but also a residential lot
RATIO: and a house of strong materials.
10. Art. 1484. In a contract of sale of personal property the price of which is b. This Court sustained the pronouncement made by the lower court on
payable in installments, the vendor may exercise of the following remedies: the nullity of the mortgage in so far as it included the house and lot
a. Exact fulfillment of the obligation, should the vendee fail to pay; of the vendees, holding that under the law, should the vendor choose
b. Cancel the sale, should the vendee's failure to pay cover two or more to foreclose the mortgage, he has to content himself with the
installments; proceeds of the sale at the public auction of the chattels which were
c. Foreclose the chattel mortgage on the thing sold, if one has been sold on installment and mortgaged to him and having chosen the
constituted, should the vendee's failure to pay cover two or more remedy of foreclosure, he cannot nor should he be allowed to insist
installments. In this case, he shall have no further action against the on the sale of the house and lot of the vendees, for to do so would
purchaser to recover any unpaid balance of the price. Any agreement be equivalent to obtaining a writ of execution against them
to the contrary shall be void. concerning other properties which are separate and distinct from
11. Under the above-quoted article of the Civil Code, the vendor of personal those which were sold on installment.
property the purchase price of which is payable in installments, has the right, c. This would indeed be contrary to public policy and the very spirit
should the vendee default in the payment of two or more of the agreed and purpose of the law, limiting the vendor's right to foreclose the
installments, to exact fulfillment by the purchaser of the obligation, or to chattel mortgage only on the thing sold.
cancel the sale, or to foreclose the mortgage on the purchased personal
property, if one was constituted.
12. Whichever right the vendor elects, he cannot avail of the other, these
remedies being alternative, not cumulative.
13. Furthermore, if the vendor avails himself of the right to foreclose his
mortgage, the law prohibits him from further bringing an action against
the vendee for the purpose of recovering whatever balance of the debt
secured not satisfied by the foreclosure sale.
14. The precise purpose of the law is to prevent mortgagees from seizing the
mortgaged property, buying it at foreclosure sale for a low price and then
bringing suit against the mortgagor for a deficiency judgment, otherwise, the
mortgagor-buyer would find himself without the property and still owing
practically the full amount of his original indebtedness.
15. In the instant case, Filipinas Investment elected to foreclose its mortgage
upon default by the plaintiffs in the payment of the agreed installments.
16. Having chosen to foreclose the chattel mortgage, and bought the purchased
vehicles at the public auction as the highest bidder, it submitted itself to the
consequences of the law as specifically mentioned, by which it is deemed to
have renounced any and all rights which it might otherwise have under the
promissory note and the chattel mortgage as well as the payment of the unpaid
balance.
17. The lower court rightly declared the nullity of the chattel mortgage in
question in so far as the taxicab franchise and the used Chevrolet car of
plaintiffs are concerned, under the authority of the ruling in the case of Levy
06 EQUITABLE SAVINGS BANK v. PALCES (Karen) chattel mortgage contract - and not a contract of sale of personal property in
March 9, 2016 | Perlas-Bernabe, J. | Remedy of Foreclosure under Art. 1484 installments - was entered into by the parties with Palces standing as the debtor-
mortgagor and BDO as the creditor-mortgagee. Therefore, the conclusion of the
PETITIONER: Equitable Savings Bank (now known as BDO Unibank Inc.) CA that Article 1484 finds application in this case is misplaced, and thus, must be
RESPONDENTS: Rosalinda C. Palces set aside. Further, there is nothing in the Promissory Note with Chattel Mortgage
that bars BDO from receiving any late partial payments from Palces. If at all,
SUMMARY: Palces purchased a vehicle through a loan granted by BDO of BDO's acceptance of Palces’ late partial payments in the aggregate amount of
P1,196,100. With this, Palces executed a Promissory Note with Chattel P103,000 will only operate to reduce her outstanding obligation to BDO from
Mortgage in favor of BDO wherein it was stated that (a) she will pay monthly P664,500 to P561,500. Such a reduction in Palces’ outstanding obligation should
installments; (b) if she defaulted in paying the installments, the remaining balance be accounted for when BDO conducts the impending foreclosure sale of the
become due and payable; and (c) BDO has the right to declare the entire obligation subject vehicle. Once such foreclosure sale has been made, the proceeds thereof
due and payable and has an option to foreclose the mortgage or file an ordinary should be applied to the reduced amount of Palces' outstanding obligation, and the
civil action allowed under the law. For CY 2005 to 2006, Palces paid on time the excess of said proceeds, if any, should be returned to her.
monthly installments. However, in the first 2 months of CY 2007, she failed to
pay thereby triggering the acceleration clause contained in the Promissory Note DOCTRINE: Art. 148426 of the Civil Code governs the sale of personal
with Chattel Mortgage and prompting BDO to send a demand letter to pay the properties in installments and it would not apply if there is no vendor-vendee
remaining balance. As the demand was remained unheeded, BDO filed a relationship. In this case, a judicious perusal of the records would reveal that
complaint against Palces and a writ of replevin was issued and served. Palces mortgagor-buyer never bought the subject vehicle from financing company but
admitted that she defaulted but she had asked permission for such delay and from a third party, and merely sought financing from mortgagee for its full
thereafter, she gave P103,000. The RTC ruled in BDO's favor which was affirmed purchase price.
by the CA with modification wherein it ordered BDO to return the P103,000. The
CA opined that by choosing to recover the subject vehicle via a writ of replevin, FACTS:
BDO already waived its right to recover any unpaid installments, pursuant to 43. On August 15, 2005, Rosalinda C. Palces (Palces) purchased a Hyundai
Article 1484 of the Civil Code. As such, the CA concluded that Palces is entitled Starex GRX Jumbo (subject vehicle) through a loan granted by BDO
to the recovery of the aforesaid amount. Hence, the current petition. Unibank Inc. (BDO) of P1,196,100. With this, Palces executed a Promissory
Note with Chattel Mortgage in favor of BDO, stating, that:
The issue in this case is whether or not the CA correctly ordered BDO to return to a. Palces shall pay BDO the amount in 36-monthly installments of P33,225
Palces P103,000 representing the latter’s late installment payments. per month, beginning September 18, 2005 and every 18th of the month
thereafter until full payment of the loan;
The SC held that the CA is not correct. It held that Art. 1484 of the Civil Code b. Palces' default in paying any installment renders the remaining balance
governs the sale of personal properties in installments. In this case, there was no due and payable; and
vendor-vendee relationship between Palces and BDO. A judicious perusal of the c. Palces' failure to pay any installments shall give BDO the right to declare
records would reveal that Palces never bought the subject vehicle from BDO but the entire obligation due and payable and may likewise, at its option, x
from a third party, and merely sought financing from BDO for its full purchase x x foreclose this mortgage; or file an ordinary civil action for collection
price. In order to document the loan transaction between BDO and Palces, a and/or such other action or proceedings as may be allowed under the law
Promissory Note with Chattel Mortgage was executed wherein, inter alia, Palces
acknowledged her indebtedness to BDO of P1,196,100 and placed the subject 44. From September 18, 2005 to December 21, 2006, Palces paid the monthly
vehicle as a security for the loan. Indubitably, a loan contract with the accessory installment. However, she failed to pay the monthly installments in January

26
Article 1484. In a contract of sale of personal property the price of which is payable in 3. Foreclose the chattel mortgage on the thing sold, if one has been constituted, should
installments, the vendor may exercise any of the following remedies: the vendee's failure to pay cover two or more installments. In this case, he shall have no
1. Exact fulfillment of the obligation, should the vendee fail to pay; further action against the purchaser to recover any unpaid balance of the price. Any
2. Cancel the sale, should the vendee's failure to pay cover two or more installments; agreement to the contrary shall be void. (Emphases and underscoring supplied)
and February 2007, thereby triggering the acceleration clause contained in
the Promissory Note with Chattel Mortgage and prompting BDO to send a 50. Hence, the current petition.
demand letter dated February 22, 2007 to compel her to pay the remaining
balance of the loan of P664,500. ISSUE: Whether or not the CA correctly ordered BDO to return to Palces P103,000
representing the latter’s late installment payments. -NO
45. As the demand went unheeded, BDO filed the instant Complaint for
Recovery of Possession with Replevin with Alternative Prayer for Sum of RULING: The petition is PARTLY GRANTED. The Decision and the Resolution of
Money and Damages against Palces before the RTC, praying that the court a the Court of Appeals are hereby SET ASIDE. In case foreclosure proceedings on the
quo: (a) issue a writ of replevin ordering the seizure of the subject vehicle subject chattel mortgage has not yet been conducted/concluded, BDO
and its delivery to BDO; or (b) in the alternative as when the recovery of the is ORDERED to commence foreclosure proceedings on the subject vehicle in
subject vehicle cannot be effected, to render judgment ordering Palces to pay accordance with the Chattel Mortgage Law, i.e., within thirty (30) days from the
the remaining balance of the loan, including penalties, charges, and other finality of this Decision. The proceeds therefrom should be applied to the reduced
costs appurtenant thereto. outstanding balance of Palces of P561,500, and the excess, if any, should be returned
to her.
46. A writ of replevin was issued and served to Palces personally. In her
defense, while admitting that she indeed defaulted on her installments for RATIO:
January and February 2007, she insisted that she called BDO regarding such 137. The SC held that the CA was wrong in citing Art. 1484 of of the Civil Code,
delay in payment and spoke to a bank officer, Rodrigo Dumagpi, who gave specifically par. 3 thereof. The CA ruled that BDO had already waived its
his consent thereto. right to recover any unpaid installments when it sought - and was granted - a
writ of replevin in order to regain possession of the subject vehicle. As such,
47. Palces then maintained that in order to update her installment payments, she BDO is no longer entitled to receive Palces' late partial payments in the
paid BDO a total of P103,000 on March 8 and 20, 2007. Despite the aforesaid aggregate amount of P103,000.
payments, Palces was surprised when BDO filed the instant complaint,
resulting in the sheriff taking possession of the subject vehicle. 138. Art. 148427 of the Civil Code governs the sale of personal properties in
installments.
48. The RTC ruled in BDO's favor and, accordingly, confirmed BDO's right and
possession over the subject vehicle and ordered Palces to pay the former 139. In this case, there was no vendor-vendee relationship between Palces and
P15,000 as attorney's fees as well as the costs of suit. BDO. A judicious perusal of the records would reveal that Palces never
bought the subject vehicle from BDO but from a third party, and merely
49. The CA affirmed the RTC ruling with modification: (a) ordering BDO to sought financing from BDO for its full purchase price. In order to
return the amount of P103,000.00 to Palces; and (b) deleting the award of document the loan transaction between BDO and Palces, a Promissory Note
attorney's fees in favor of BDO for lack of sufficient basis. It held that while with Chattel Mortgage was executed wherein, inter alia, Palces
Palces was indeed liable to BDO under the Promissory Note with Chattel acknowledged her indebtedness to BDO of P1,196,100 and placed the subject
Mortgage, BDO should not have accepted Palces' late partial payments of vehicle as a security for the loan. Indubitably, a loan contract with the
P103,000. In this regard, the CA opined that by choosing to recover the accessory chattel mortgage contract - and not a contract of sale of
subject vehicle via a writ of replevin, BDO already waived its right to recover personal property in installments - was entered into by the parties with
any unpaid installments, pursuant to Article 1484 of the Civil Code. As such, Palces standing as the debtor-mortgagor and BDO as the creditor-
the CA concluded that Palces is entitled to the recovery of the aforesaid mortgagee. Therefore, the conclusion of the CA that Article 1484 finds
amount. application in this case is misplaced, and thus, must be set aside.

27
Article 1484. In a contract of sale of personal property the price of which is payable in 3. Foreclose the chattel mortgage on the thing sold, if one has been constituted, should
installments, the vendor may exercise any of the following remedies: the vendee's failure to pay cover two or more installments. In this case, he shall have no
1. Exact fulfillment of the obligation, should the vendee fail to pay; further action against the purchaser to recover any unpaid balance of the price. Any
2. Cancel the sale, should the vendee's failure to pay cover two or more installments; agreement to the contrary shall be void. (Emphases and underscoring supplied)
140. In light of the stipulations in the Promissory Note with Chattel Mortgage (see
Fact No. 1), BDO is justified in filing his Complaint before the RTC seeking
for either the recovery of possession of the subject vehicle so that it can
exercise its rights as a mortgagee, i.e., to conduct foreclosure proceedings
over said vehicle; or in the event that the subject vehicle cannot be recovered,
to compel Palces to pay the outstanding balance of her loan. Since it is
undisputed that BDO had regained possession of the subject vehicle, it is only
appropriate that foreclosure proceedings, if none yet has been
conducted/concluded, be commenced in accordance with the provisions of
Act No. 1508 (The Chattel Mortgage Law), as intended. Otherwise, Palces
will be placed in an unjust position where she is deprived of possession of the
subject vehicle while her outstanding debt remains unpaid, either in full or in
part, all to the undue advantage of BDO - a situation which law and equity
will never permit.

141. Further, there is nothing in the Promissory Note with Chattel Mortgage that
bars BDO from receiving any late partial payments from Palces. If at all,
BDO's acceptance of Palces’ late partial payments in the aggregate amount
of P103,000 will only operate to reduce her outstanding obligation to BDO
from P664,500 to P561,500. Such a reduction in Palces’ outstanding
obligation should be accounted for when BDO conducts the impending
foreclosure sale of the subject vehicle. Once such foreclosure sale has been
made, the proceeds thereof should be applied to the reduced amount of Palces'
outstanding obligation, and the excess of said proceeds, if any, should be
returned to her.

142. In sum, the CA erred in ordering BDO to return the P103,000 to Palces. In
view of BDO's prayer for and subsequent possession of the subject vehicle in
preparation for its foreclosure, it is only proper that BDO be ordered to
commence foreclosure proceedings, if none yet has been
conducted/concluded, over the vehicle in accordance with the provisions of
the Chattel Mortgage Law, i.e., within thirty (30) days from the finality of
this Decision.
07 MACONDRAY & CO. v. EUSTAQUIO (DANNAH) (From the Syllabus) “Barring” Effects of Foreclosure: All amounts due from the
16 July 1937 | Imperial, J. | “Barring” Effects of Foreclosure sale, including damages and attorneys fees, are barred from recovery.

PLAINTIFF-APPELLANT: Macondray & Co., Inc., FACTS:


DEFENDANT-APPELLEE: Urbando Eustaquio 155. This is an appeal from the judgment of the CFI of Manila dismissing the
complaint of plaintiff Macondray & Co. (Macondray).
SUMMARY: Macondray sold to Eustaquio a De Soto Car, Sedan for P595. This 156. Macondray sold to Eustaquio a De Soto car, Sedan, for P595.
was to be paid in twelve monthly installments, with an interest rate of 12% p.a. A 157. For this, Eustaquio executed a note dated May 22, 1934.
stipulation in the contract states that upon failure to pay an installment, the whole a. He undertook to pay the car in twelve monthly installments
amount becomes due and demandable. Moreover, Eustaquio mortgaged the same b. Interest of 12% p.a.
car to Macondray to guarantee his obligation. c. Should he fail to pay any installment with interest, the remaining installment
would be due and demandable, and he must pay 20% as fees and etc.
Eustaquio was able to pay but one installment, hence Macondray called upon the 158. To guarantee performance of his obligation, Eustaquio on the same date
sheriff to take possession of the car. Eustaquio refused to give up possession of mortgaged the purchased car in favor of Macondray, and bound himself under
said car, which prompted Macondray to file a replevin suit. After taking the same conditions stipulated in the note (5a-c)
possession of the car, it was sold at a public option and Macondray was the highest 159. The mortgage deed was registered on June 11, 1934. On the 22nd of the same
bidder at P250. According to the liquidation, Eustaquio was still indebted in the month, Eustaquio paid P43.75 as the first installment, and failed to pay the
amount of P342.20 with interest and fees. remaining ones.
160. In accordance with the terms of the mortgage, Macondray called upon the
Macondray wanted to enforce payment for the remaining balance, and the lower sheriff to take possession of the car, but Eustaquio refused to yield
court denied his complaint, hence he appealed said judgment to the SC. He possession.
assigned three errors, which the SC ruled as untenable. It ruled as follows: 161. Macondray filed a suit for replevin and thereby succeeded in taking
possession of the car. The car was sold at a public auction to Macondray for
First, Eustaquio not appearing at the trial did not amount to a waiver of his rights P250.
under Act 4122. 162. According to the liquidation, Eustaquio was still indebted in the amount of
P342.20 with interest and fees.
Second, Act 4122 is valid. In the Philippines, the Chattel Mortgage Law did not 163. Macondray brought an action against defendant Urbando Eustaquio
expressly provide for a deficiency judgment upon the foreclosure of a mortgage. (Eustaquio) to obtain the possession of an automobile Eustaquio mortgaged,
The controlling purpose of Act No. 4122 is revealed to be to close the door to and to recover the balance, interest, attorney’s fees, expenses and etc.
abuses committed in connection with the foreclosure of chattel mortgages when 164. Eustaquio was summoned but he failed to appear and file an answer, hence
sales were payable in installments. The public policy, obvious from the statute, he was declared in default and the appealed judgment was rendered
was defined and established by legislative authority. It is for the courts to accordingly.
perpetuate it.
ISSUE/s:
Third, if the vendor has chosen to foreclose the mortgage he shall have no further 51. WoN there was a misapplication of Act No. 4122 – NO
action against the purchaser for the recovery of any unpaid balance owing by the 52. WoN Act No. 4122 is invalid – NO
same, and any agreement to the contrary shall be null and void 53. WoN granting said act is valid, the court should have ordered the defendant
to at least pay the stipulated interest, attorney’s fees and costs – NO
DOCTRINE: The proviso to the right to foreclose is, that if the vendor has chosen
such remedy, he shall have no further action against the purchaser for the recovery RULING: In view of the foregoing, the appealed judgment is affirmed, with the costs
of any unpaid balance owing by the same. In other words, as we see it, the Act of this instance to the plaintiff and appellant. So ordered.
does no more than qualify the remedy.
RATIO:
On the first issue a sufficient remedy remaining.
143. Judgment by default against a defendant who has neither appeared nor filed 10. Three remedies are available to the vendor who has sold personal property
his answer does not imply a waiver of right except that of being heard and of on the installment plan:
presenting evidence in his favor. a. He may elect to exact the fulfillment of the obligation
a. Macondray was contending that since Eustaquio did not appear in b. If the vendee shall have failed to pay two or more installments, the vendor
trial, he already waived his rights under Act 4122 and was already may cancel the sale
declared in default c. If the vendee shall have failed to pay two or more installments, the vendor
may foreclose the mortgage, if one has been given on the property
11. The basis of the first option is the Civil Code. The basis of the last two option
On the second issue
is Act No. 4122, amendatory of the Civil Code.
1. The amendment of the Act prevents mortgagees from seizing the mortgaged
12. The proviso to the right to foreclose is, that if the vendor has chosen such
property, buying it at foreclosure sale for a low price and then bringing suit
remedy, he shall have no further action against the purchaser for the recovery
against the mortgagor for a deficiency judgment. The almost invariable result
of any unpaid balance owing by the same. In other words, as we see it, the
of this procedure was that the mortgagor found himself minus the property
Act does no more than qualify the remedy.
and still owing practically the full amount of his original indebtedness.
13. The proper approach in cases of this character should be to resolve all
2. Under this amdendment, the vendor of personal property, the purchase price
presumptions in favor of the validity of an act in the absence of a clear conflict
of which is payable in installments, has the right to cancel the sale or foreclose
between it and the constitution. All doubts should be resolved in its favor.
the mortgage if one has been given on the property.
14. The controlling purpose of Act No. 4122 is revealed to be to close the door
3. Whichever right the vendor elects he need not return to the purchaser the
to abuses committed in connection with the foreclosure of chattel mortgages
amount of the full installment already paid, “if there be an agreement to that
when sales were payable in installments. The public policy, obvious from the
effect.” Furthermore, if the vendor avails himself of the right from foreclose
statute, was defined and established by legislative authority. It is for the
the mortgage this amendment prohibits him from bringing an action against
courts to perpetuate it.
the purchaser for the unpaid balance.”
15. The law seeks to remedy an evil which the Legislature wished to suppress;
4. In other words, under this amendment, in all proceedings for the foreclosure
this legislative body has power to promulgate the law; the law does not
of chattel mortgages, executed on chattels which have been sold on the
completely deprive vendors on the installment basis of a remedy, but requires
installment plan, the mortgagee is limited to the property included in the
them to elect among three alternative remedies; the law, on the other hand,
mortgage.
does not completely exonerate the purchasers, but only limits their liabilities
5. An Act was passed in Washington which provided “that in all proceedings
and, finally, there is no vested right when a procedural law is involved,
for the foreclosure of mortgages hereafter executed or on judgments rendered
wherefore the Legislature could enact Act No. 4122 without violating the
upon the debt thereby secured the mortgagee or assignee shall be limited to
aforesaid organic law.
the property included in the mortgage.”
6. In Oregon, an Act abolishing deficiency judgment upon the foreclosure of
On the third issue
mortgages to secure the unpaid balance of the purchase price of real property
1. This question involves the interpretation of the pertinent portion of the law,
was unanimously sustained by the Supreme Court of that State.
reading: "However, if the vendor has chosen to foreclose the mortgage he
7. Bronzon v. Kinzie
a. Had under consideration a law passed in Illinois, which provide that the
shall have no further action against the purchaser for the recovery of any
equitable estate of the mortgagor should not be extinguished for twelve unpaid balance owing by the same, and any agreement to the contrary shall
months after sale on decree, and which prevented any sale of the mortgaged be null and void."
property unless two-thirds of the amount at which the property had been 2. This paragraph refers to the mortgage contract executed by the parties,
valued by appraisers should be bid therefor. whereby the purchaser mortgages the chattel sold to him on the installment
8. In the Philippines, the Chattel Mortgage Law did not expressly provide for a basis in order to guarantee the payment of its price, and the words "any unpaid
deficiency judgment upon the foreclosure of a mortgage. Indeed, it required balance" should be interpreted as having reference to the deficiency judgment
decisions of this court to authorize such a procedure. to which the mortgagee may be entitled where, after the mortgaged chattel is
9. But the practice became universal enactment regarding procedure. To a sold at public auction, the proceeds obtained therefrom are insufficient to
certain extent the Legislature has now disauthorized this practice, but has left cover the full amount of the secured obligations which, in the case at bar as
shown by the note and by the mortgage deed, include interest on the principal,
attorney's fees, expenses of collection, and the costs.
3. The fundamental rule which should govern the interpretation of laws is to
ascertain the intention and meaning of the Legislature and to give effect
thereto.
08 Northern Motors v. Sapinoso (Cyria) (e) ordinary civil action to exact fulfillment of the mortgage contract. It was
May 29, 1970 | Villamor, J. | Foreclosure further stipulated that "[w]hichever remedy is elected by the mortgagee, the
mortgagor expressly waives his right to reimbursement by the mortgagee of
PETITIONER: Northern Motors, Inc. any and all amounts on the principal and interest already paid by him."
RESPONDENTS: Casiano Sapinoso and John Doe 3. Sapinoso failed to pay the first installment of P361.00 due on July 5, 1965,
and the second, third, fourth and fifth installments of P351.00 each due on
SUMMARY: Sapinoso purchased from Northern Motors an Opel Kadett car, the 5th day of August, September, October and November, 1965,
payable on installments. Sapinoso executed a promissory note and chattel respectively.
mortgage on the car, providing therein that upon default, Nothern Motors may 4. Several payments were, however, made by Sapinoso, to wit: P530.52 on
elect any of the following remedies: (1) sale of the car, (2) cancellation of the November 21, 1965, P480.00 on December 21, 1965, and P400.00 on April
contract of sale, (3) extrajudicial foreclosure, (4) judicial foreclosure, or (5) 30, 1966. The first and third payments aforesaid were applied to accrued
ordinary civil action to exact fulfillment of the mortgage contract. It was further interest up to April 17, 1966, while the second payment was applied partly
provided that whichever remedy is elected, Northern Motors expressly waives his (P158.10) to interest, and partly (P321.90) to the principal, thereby reducing
right to reimbursement. Sapinoso defaulted so Northern Motors filed the the balance unpaid to P10,218.10.
complaint. Before filing his answer, Sapinoso made two payments on the 5. Sapinoso having failed to make further payments, Northern Motors, Inc. filed
promissory note. The trial court ruled that Northern Motors had acquired the right the present complaint against Sapinoso and a certain "John Doe." Northern
to foreclose the chattel mortgage but in doing so, Northern Motors had waived Motors, Inc. stated that it was availing itself of the option given it under the
any claim it may have on the promissory note. Northern Motors was then ordered mortgage contract of extrajudicially foreclosing the mortgage, and prayed
to return the amount it received after filing the case. Northern Motors appealed to that a writ of replevin be issued upon its filing of a bond for the seizure of the
the SC, arguing that the trial court erred in ordering it to reimburse the amount car and for its delivery to it.
because it is the exercise, no the mere election, of the remedy of foreclosure that 6. Subsequent to the commencement of the action, but before the filing of his
bars the creditor from recovering the unpaid balance of debt. answer, defendant Sapinoso made two payments on the promissory note, the
first on August 22, 1966, for P500.00, and the second on September 27, 1966,
The SC ruled that the trial court erred in ordering Northern Motors to reimburse for P750.00.
the amount paid by Sapinoso. *see doctrine* 7. Sapinoso alleged, however, that he had paid the total sum of P4,230.52,
leaving a balance of only P5,987.58; that upon demand he immediately
DOCTRINE: Action of replevin in order to foreclose on the chattel mortgage surrendered the possession of the car to the plaintiff's representative; and that
does not produce the barring effect under the Recto Law; for it is the fact of the value of the car was only about P5,000.00, and not P10,000.00 as alleged
foreclosure and actual sale of the mortgaged chattel that bar further recovery by in the complaint. He failed to pay the installments due because the car was
the seller of any balance on the buyer’s outstanding obligation not satisfied by defective, and Northern Motors failed to have it fixed although he had
the sale. The voluntary payment of the installment by the buyer-mortgagor is repeatedly called the Northern Motors’ attention thereto, hence, Sapinoso had
valid and not recoverable in spite the restrictive provisions of Art. 1484(3). to procrastinate in his payments in order to move Northern Motors to repair
the car; and that although the car could not be used, he paid P700.00 to
FACTS: Northern Motors upon the latter's assurance that the car would be fixed.
1. Casiano Sapinoso purchased from Northern Motors, Inc. an Opel Kadett car 8. The court a quo held that Sapinoso having failed to pay more than two (2)
for the price of P12,171.00, making a down payment and executing a installments, Northern Motors acquired the right to foreclose the chattel
promissory note for the balance of P10,540.00 payable in installments with mortgage, which it could avail of by filing an action of replevin to secure
interest at 12% per annum. possession of the mortgaged car as a preliminary step to the foreclosure sale
2. To secure the payment of the promissory note, Sapinoso executed in favor of contemplated in the Chattel Mortgage Law; and that the foreclosure of the
Northern Motors, Inc. a chattel mortgage on the car. The mortgage contract chattel mortgage and the recovery of the unpaid balance of the price are
provided, among others, that upon default by the mortgagor in the payment alternative remedies which may not be pursued conjunctively, so that in
of any part of the principal or interest due, the mortgagee may elect any of availing itself of its right to foreclose the chattel mortgage, Northern Motors
the following remedies: (a) sale of the car by the mortgagee; (b) cancellation thereby renounced whatever claim it may have had on the promissory note,
of the contract of sale; (c) extrajudicial foreclosure; (d) judicial foreclosure; and, therefore, Northern Motors has no more right to the collection of the
attorney's fees stipulated in the promissory note, and should return to 6. If the mortgage creditor, before the actual foreclosure sale, is not precluded
Sapinoso the sum of P1,250.00 which Northern Motors had received from from recovering the unpaid balance of the price although he has filed an
Sapinoso after having filed the present case, and elected to foreclose the action of replevin for the purpose of extrajudicial foreclosure, or if a mortgage
chattel mortgage. creditor who has elected to foreclose but who subsequently desists from
9. In this appeal, Northern Motors claims that the court a quo erred in ordering proceeding with the auction sale, without gaining any advantage or benefit,
it to reimburse to Sapinoso the sum of P1,250.00 which the latter had paid. It and without causing any disadvantage or harm to the vendee-mortgagor, is
contends that under Article 1484 of the Civil Code it is the exercise, not the not barred from suing on the unpaid account, there is no reason why a
mere election, of the remedy of foreclosure that bars the creditor from mortgage creditor should be barred from accepting, before a foreclosure sale,
recovering the unpaid balance of the debt; that what the said Article 1484 payments voluntarily tendered by the debtor-mortgagor who admits a
prohibits is "further action" to collect payment of the deficiency after the subsisting indebtedness.
creditor has foreclosed the mortgage; and that in paying Northern Motors the
sum of P1,250.00 Sapinoso filed his answer, and in not filing a counterclaim
for the recovery thereof, the Sapinoso in effect renounced whatever right he
might have had to recover the said amount.

ISSUE: WoN Northern Motors should pay Sapinoso P1,250 - NO

RULING: PREMISES CONSIDERED, the judgment appealed from is modified by


setting aside the portion thereof which orders Northern Motors to pay Sapinoso the
sum of P1,250.00, with costs in this instance against the Sapinoso.

RATIO:
1. In issuing a writ of replevin, and, after trial, in upholding Northern Motors’
right to the possession of the car, and ratifying and confirming its delivery to
Northern Motors, the court below correctly considered the action as one of
replevin to secure possession of the mortgaged vehicle as a preliminary step
to this foreclosure sale contemplated in Section 14 of Act No. 1508
2. The said court however erred in concluding that the legal effect of the filing
of the action was to bar plaintiff-appellant from accepting further payments
on the promissory note.
3. That the ultimate object of the action is the foreclosure of the chattel
mortgage, is of no moment, for it is the fact of foreclosure and actual sale of
the mortgaged chattel that bar further recovery by the vendor of any balance
on the purchaser's outstanding obligation not satisfied by the sale.
4. In any event, what Article 1484(3) prohibits is "further action against the
purchaser to recover any unpaid balance of the price;" and although this Court
has construed the word "action" in said Article 1484 to mean "any judicial or
extrajudicial proceeding by virtue of which the vendor may lawfully be
enabled to exact recovery of the supposed unsatisfied balance of the purchase
price from the purchaser or his privy", there is no occasion at this stage to
apply the restrictive provision of the said article, because there has not yet
been a foreclosure sale resulting in a deficiency.
5. The payment of the sum of P1,250.00 by Sapinoso was a voluntary act on his
part and did not result from a "further action" instituted by Northern Motors.
09 CRUZ vs. FILIPINAS INVESTMENT & FINANCE CORPORATION interest per annum, until fully paid.
(Gueco) 9. As evidence of the above indebtedness, Cruz executed a chattel mortgage
May 27, 1968 | Reyes, J.B.L. J. | Alternative character of Art. 1484 over the subject vehicle in favor of Far East Motor.
10. Since Cruz did not give a down payment, he was required by Far East Motor
PETITIONER: Ruperto G. Cruz, et al., plaintiff-appellees to give an additional security. For this, Felicidad Vda. de Reyes (“Mrs.
RESPONDENTS: Filipinas Investment & Finance Corporation, defendant- Reyes”) executed a second mortgage in favor of Far East Motor over her land
appellant. plus the buildings and improvements thereon. The said land, at that time, was
mortgaged to Development Bank of the Philippines (“DBP”) because Mrs.
SUMMARY: Reyes obtained a loan from DBP.
Cruz purchased an Isuzu Diesel Bus on installment from Far East Motor. To 11. Far East Motor then indorsed the promissory note and assigned all its rights
secure payment of the vehicle, Cruz executed two mortgages: (1) a chattel and interest in chattel mortgage to Filipinas Investment.
mortgage on the subject vehicle, and (2) a real estate mortgage executed by Mrs. 12. Cruz defaulted in the payment of the promissory note (he only paid 500 pesos
Reyes over her land and the improvements therein. The second mortgage was which amount was applied to the payment of interest) and despite repeated
required of Cruz because he did not make any down payment. demands, he made no payment on any of the installments.
13. As such, Filipinas Investment foreclosed the chattel mortgage. The proceeds
Subsequently, Far East Motor assigned his right to collect from Cruz to Filipinas of the sale of the bus, however, was not sufficient to cover the expenses of
Investment. When Cruz defaulted in the payment of his installments, Filipinas the principal obligation, interest, and attorney’s fees. As such, Filipinas
Investment foreclosed the chattel mortgage and sold it at public auction where it Investment moved to close the real estate mortgage on Mrs. Reyes’ land.
won as the highest bidder. The proceeds from the sale of the chattel mortgage 14. In preparation of the foreclosure of the real estate mortgage, Filipinas
were, however, insufficient to cover the debt of Cruz. Hence, Filipinas Investment Investment paid DBP the unpaid balance of Mrs. Reyes’ loan.
moved for the foreclosure of the real estate mortgage. Cruz opposed this and filed 15. Thereafter, Filipinas Investment proceeded with the foreclosure by
an action in Court. ISSUE: WoN Filipinas Investment may also extrajudicially requesting the provincial sheriff to sell the land at public auction.
foreclose the real estate mortgage despite having already foreclosed the chattel 16. Cruz requested Filipinas Investment to cancel the sale but the latter refused
mortgage—NO, FILIPINAS INVESTMENT IS BARRED FROM stating that the former’s request had no legal basis.
FORECLOSING THE REAL ESTATE MORTGAGE. RULING: The remedies 17. Cruz thus filed an action in court and the provincial sheriff held in abeyance
under Art. 1484 are alternative and not cumulative. Hence, the election of one the sale of the mortgaged land pending the result of the case.
bars the election of another. In this case, when Filipinas Investment sought to 18.
foreclose the chattel mortgage on the subject vehicle, it thus waived its right to
elect the other remedies. The purpose behind the alternative character of Art. 1484 ISSUES:
is to prevent mortgagees from seizing the mortgaged property, buying it at 3. WoN Filipinas Investment may also extrajudicially foreclose the real estate
foreclosure sale for a low price, and then bringing suit against the mortgagor for mortgage despite having already foreclosed the chattel mortgage—NO,
the deficiency. FILIPINAS INVESTMENT IS BARRED FROM FORECLOSING THE
REAL ESTATE MORTGAGE.
DOCTRINE:
The remedies under Art. 1484 are alternative and not cumulative, such that the RULING: WHEREFORE, the decision appealed from is modified, by ordering
exercise of one would bar the exercise of the others. Otherwise stated, the plaintiff-appellee Felicidad Vda. de Reyes to reimburse to defendant- appellant
foreclosure and actual sale of a chattel mortgage bars further recovery by the Filipinas Investment & Finance Corporation the sum of P2,148.07, with legal interest
seller of any balance on the purchaser's outstanding obligation not so satisfied by thereon from the finality of this decision until it is fully paid. In all other respects, the
the sale. judgment of the court below is affirmed, with costs against the defendant- appellant.

FACTS: RATIO:
8. Ruperto G. Cruz (“Cruz”) purchased an Isuzu Diesel Bus on installment from 1. ART. 1484. In a contract of sale of personal property the price of which is
Far East Motor Corporation (“Far East Motor”). The vehicle is valued at P44, payable in installments, the vendor may exercise any of the following
616.24 and payable at P1,487.20 per month for thirty months, with 12% remedies:
1) Exact fulfillment of the obligation, should the vendee fail to pay; such proceeding.
2) Cancel the sale, should the vendee's failure to pay cover two or more
installments;
3) Foreclose the chattel mortgage on the thing sold, if one has been
constituted, should the vendee's failure to pay cover two or more
installments. In this case, he shall have no further action against the
purchaser to recover any unpaid balance of the price. Any agreement
to the contrary shall be void.
2. The said provision is clear and simple: should the vendee or purchaser of a
personal property default in the payment of two or more of the agreed
installments, the vendor or seller has the option to avail of any one of these
three remedies — either to exact fulfillment by the purchaser of the
obligation, or to cancel the sale, or to foreclose the mortgage on the purchased
personal property, if one was constituted. These remedies have been
recognized as alternative, not cumulative such that the exercise of one
would bar the exercise of the others. Otherwise stated, the foreclosure
and actual sale of a chattel mortgage bars further recovery by the seller
of any balance on the purchaser's outstanding obligation not so satisfied
by the sale.
3. The objective of Art. 1484 is to prevent the abuses committed in connection
with the foreclosure of chattel mortgages. It prevents mortgagees from
seizing the mortgaged property, buying it at foreclosure sale for a low price
and then bringing suit against the mortgagor for the deficiency.
4. In this case, Filipinas Investment sought to collect the supposed deficiency
of Cruz’s obligation by going against the second real estate mortgage. He
argued that Art. 1484 only prohibits the seller from recovering "against the
purchaser" and not a recourse to the additional security put up, not by the
purchaser himself, but by a third person.
5. The Court did not sustain its (Filipinas Investment) contention, for doing so
would overlook the fact that if the guarantor should be compelled to pay the
balance of the purchase price, the guarantor will in turn be entitled to recover
what she has paid from the debtor vendee. Ultimately, it will be the vendee
who will be made to bear the payment of the balance of the price, despite the
earlier foreclosure of the chattel mortgage given by him. Thus, the protection
given by Article 1484 would be indirectly subverted, and public policy
overturned. Basically, the Court is saying that there is still an indirect
subversion of Art. 1484 because in the end, it’s still the buyer, Cruz, who will
pay for the purchase price. This is because the guarantor, which in this case
is Mrs. Reyes, will still be entitled to reimbursement from Cruz.
6. Also, the word "action" in Art. 1484(3) may be construed as referring to any
judicial or extrajudicial proceeding by virtue of which the vendor may
lawfully be enabled to exact recovery of the supposed unsatisfied balance of
the purchase price from the purchaser or his privy.
7. In this case, the extrajudicial foreclosure of a real estate mortgage is one of
10 Borbon II v. Servicewide Specialists (REINE) of notice or demand, in installments of the amounts following and at the dates
11 July 1996| Vitug, J | Barring Effects of Foreclosure hereinafter set forth, to wit: P10,238.00 monthly for 12 months due and payable on
Petitioner/s: Daniel Borbon II & Francisco Borbon the 7 day of each month starting January, 1985, provided that a late payment charge
Respondent/s: ServiceWide Specialists, Inc & CA of 3% per month shall be added on each unpaid installment from due date thereof
until fully paid. It is further agreed that if upon such default, attorney's services are
availed of, an additional sum equal to twenty 25% of the total sum due thereon,
Summary: Daniel & Francisco Borbon (D & F Borbon) signed a which shall not be less than five hundred pesos, shall be paid to the holder hereof for
promissory note in favor of Pangasinan Auto Mart. To secure the attorney's fees plus an additional sum equivalent to25% of the total sum due which
promissory note, they executed a Chattel Mortgage on a 1 Brand New likewise shall not be less than five hundred pesos for liquidated damages, aside from
1984 Isuzu Crew Cab. The rights of the Auto Mart (Assignor) were expenses of collection and the legal costs provided for in the Rules of Court. It is
assigned to Filinvest then Filinvest assigned all its rights, interest & title expressly agreed that all legal actions arising out of this note or in connection with
over the PN & chattel mortgage to Servicewide (Assignee). Servicewide the chattel(s) subject hereof shall only be brought in or submitted to the jurisdiction
attempted to collect by sending a demand letter to D & F Borbon. of the proper court.'Acceptance by the holder hereof of payment of any installment
However, D & F Borbon contend that Auto Mart was first guilty of not or any part thereof after due dated shall not be considered as extending the time for
fulfilling its obligation when it did not deliver the car that D & F had the payment or any of the installments aforesaid or as a modification of any of the
conditions hereof. Nor shall the failure of the holder hereof to exercise any of its right
ordered. Thus, since the assignee steps into the shoes of its assignor,
under this note constitute or be deemed as a waiver of such rights.”
neither party would not incur in delay because the other did not comply 2.) To secure the Promissory Note, D. Borbon & F. Borbon executed a Chattel
with its obligation. The RTC ruled that ruled that D & F Borbon cannot Mortgage (1 Brand New 1984 Isuzu KCD 20 Crew Cab). The rights of Pangasinan
avoid liability under the promissory note and the chattel mortgage that Auto Mart, Inc (Auto Mart/Assignor) was assigned to Filinvest Credit Corporation
secured it since Servicewide took the note for value & in good faith. The (Filinvest) with notice to D. & F. Borbon. Filinvest then assigned all its rights, interest
CA affirmed such decision and it also upheld the award of liquidated & title over the Promissory Note & chattel mortgage to Servicewide Specialists
damages & attorney’s fees in favor of Servicewide. (Servicewide/Assignee)
The SC held that there should be no award of liquidated damages but 3.) After the accounts were assigned to Servicewide, they attempted to collect by
only attorney’s fees since this was reasonable. The Court further held sending a demand letter to D. & F. Borbon for them to pay their entire obligation which
that in this case, the parties concede that the action for replevin has been as March 1985 totaled P185257.80.
4.) For the defense of D. Borbon & F. Borbon, they claim that what they intended to
instituted for the foreclosure of the vehicle in question (now in the
buy from Auto Mart was a jeepney Isuzu K.C. Cab. The vehicle that they bought was
possession of Servicewide). Thus, D & F Borbon hold that under Article not delivered. Instead, through misrepresentation & machination, Auto Mart delivered
1484, the vendor-mortgagee or its assignees loses any right "to recover an Isuzu crew cab, as this is the unit available at their warehouse. Later the
any unpaid balance of the price" and any "agreement to the contrary representative of the Auto Mart, Inc. told D & F Borbon that their available stock is
would be void. an Isuzu Cab but minus the rear body, which D & F Borbon agreed to deliver with the
understanding that the Auto Mart, Inc. will refund the D & F Borbon the amount of
Doctrine: Foreclosure on chattel mortgage prevents further action on the P10,000.00 to have the rear body completed.
supporting real estate mortgage whether the chattel mortgage is first 5.)Despite Communications with the Auto Mart, the latter was not able to replace the
foreclosed and vice versa when the real estate mortgage is first vehicle until the vehicle delivered was seized by order of this court. D & F Borbon
foreclosed. argue that an assignee stands in the place of an assignor which, to the mind of the
court, is correct. The assignee exercise all the rights of the assignor. D & F
Borbon further claim that they are not in default of their obligation because the Auto
Facts: Mart was first guilty of not fulfilling its obligation in the contract. D & F Borbon claim
1.) Daniel Borbon (D. Borbon) & Francisco Borbon (F. Borbon) signed a promissory that neither party incurs delay if the other does not comply with his obligation.
note which states the following: “For value received (installment price of the chattel/s 6.) The RTC ruled that D & F Borbon cannot avoid liability under the promissory note
purchased), I/We jointly and severally promised to pay Pangasinan Auto Mart, Inc. and the chattel mortgage that secured it since Servicewide took the note for value &
or order,..the sum of P122,856.00, Philippine Currency, to be payable without need in good faith. The CA affirmed such decision and it also upheld the award of liquidated
damages & attorney’s fees in favor of Servicewide. there has been a foreclosure of the chattel mortgage that the vendee-mortgagor would
7.) Hence, D & F Borbon are seeking a modification of the decision of the appellate be permitted to escape from a deficiency liability. Thus, if the case is one for specific
court in the award of liquidated damages & attorney’s fees. performance, even when this action is selected after the vendee has refused to
surrender the mortgaged property to permit an extrajudicial foreclosure, that property
may still be levied on execution and an alias writ may be issued if the proceeds thereof
Issue: WoN Servicewide is entitled to liquidated damages and attorney’s fees?-
are insufficient to satisfy the judgment credit.
Attorney’s Fees only
5.) In this case, the parties concede that the action for replevin has been instituted
Ruling: WHEREFORE, the appealed decision is modified by deleting therefrom the
for the foreclosure of the vehicle in question (now in the possession of
award for liquidated damages, in all other respects such as the grant of attorney’s fees,
Servicewide). Thus, D & F Borbon hold that under Article 1484, the vendor-
the judgment of the appellate court is AFFIRMED.
mortgagee or its assignees loses any right "to recover any unpaid balance of the
price" and any "agreement to the contrary would be void.
Ratio:
6.) In the case of Macondray & Co v Eustaquio, the Court held that any unpaid balance
1.) D & F Borbon invoke Art. 1484: “ In a contract of sale of personal property the can only mean the deficiency judgment to which the mortgagee may be entitled to
price of which is payable in installments, the vendor may exercise any of the when the proceeds from the auction sale are insufficient to cover the "all amount of
following remedies:(a) Exact fulfillment of the obligation, should the vendee fail the secured obligation which x x x include interest on the principal, attorney's fees,
to pay; (b) Cancel the sale, should the vendee's failure to pay cover two or more expenses of collection, and costs. The legislative intent is not only to limit the
installments; (c) Foreclose the chattel mortgage or the thing sold, if one has been proscription of any further action to the "unpaid balance of the principal" to all other
constituted, should the vendee's failure to pay cover two or more installments. In claims that may likewise be called for in the accompanying promissory note against
this case, he shall have no further action against the purchaser to recover any unpaid the buyer-mortgagor or his guarantor, including costs and attorney's fees.
balance of the price. Any agreement to the contrary shall be void. The remedies under
Article 1484 of the Civil Code are not cumulative but alternative and exclusive.
7.)The Court struck down the award for liquidated damages but upheld the grant of
attorney’s fees which the Court found reasonable. The promissory note may appear to
2.) Thus, when the seller assigns his credit to another person, the latter is likewise have been a negotiable instrument, however D & F Borbon, clearly cannot claim
bound by the same law. Accordingly, when the assignee forecloses on the mortgage, unawareness of its accompanying documents so as to thereby gain a right greater than
there can be no further recovery of the deficiency, and the seller-mortgagee is that of the assignor.
deemed to have renounced any right thereto. A contrario, in the event the seller-
mortgagee first seeks, instead, the enforcement of the additional mortgages,
guarantees or other security arrangements, he must then be held to have lost by
waiver or non-choice his lien on the chattel mortgage of the personal property
sold by any mortgaged back to him, although, similar to an action for specific
performance, he may still levy on it.

3.) In ordinary alternative obligations, a mere choice categorically and unequivocally


made and then communicated by the person entitled to exercise the option concludes
the parties. The creditor may not thereafter exercise any other option, unless the
chosen alternative proves to be ineffectual or unavailing due to no fault on his part.

4.) Hence, there is a difference between alternative obligations on one hand &
alternative remedies in the other. Alternative remedies is one wherein the choice
generally becomes conclusive only upon the exercise of the remedy.For instance,
in one of the remedies expressed in Article 1484 of the Civil Code, it is only when
11 Filipinas Investment v. Ridad (Steph) instituted extrajudicial foreclosure proceedings. The car was sold at a public
November 28, 1969 | Castro, J. | Exclusivity of remedies auction and Filipinas Investment was he highest bidder.
30. Ridad Spouses allegedly did not receive any summons and did not appear at
PETITIONER: Filipinas Investment and Finance Corporation the hearing, so they were declared in default.
RESPONDENTS: Lourdes Ridad and Luis Ridad 31. Ridad Spouses filed motion to set aside order of default, which the court
SUMMARY: Ridad Spouses bought a Ford Consul sedan from Supreme Sales denied. So they appealed to the CFI.
& Development Coroporation (succeeded by Filipinas Investment, the party to 32. CFI ruled that Filipinas Investment was entitled to recover the amount of
this case). Ridad Spouses defaulted in paying their monthly installments, so P163.65 which represents the expenses incurred by the plaintiff in the seizure
Filipinas Investment filed for replevin (for return of car or payment of unpaid of the car involved in this case, plus attorney’s fees of P 300 (original amount
balance), and Ridad Spouses refused to comply wth their obligation. Thus, was 500, but CFI reduced it).
pending this case Filipinas Investment foreclosed the mortgage and sold the car 33. Ridad Spouses appeal to the Supreme Court.
at public auction. The Ridad Spouses were declared in default since they did not ISSUES: WON CFI was correct in condemning Ridad Spouses to pay attorney’s fees
appear, and were ordered to pay attorney’s fees and expenses related to seizure and expenses incurred in seizure of the car? YES.
of the car, and costs of auction. Ridad Spouses bring this matter to court, saying
that they should not be compelled to pay because Art. 1484 (3) says foreclosure RULING: ACCORDINGLY, the judgment a quo is affirmed. No costs.
and payment of unpaid balance are mutually exclusive and you can only choose
one remedy as seller. Filipinas Investment on the other hand contends that what RATIO:
is prohibited there is not applicable here, since what is contemplated in the present
case is payment of attorney’s fees and costs, not unpaid balance. The Supreme
Court ruled in favor of Filipinas Investment, and that mortgagees should be 15. Contention of Ridad Spouses: the action against them is for payment
protected against perverse mortgagors who plainly refuse to deliver the thing of unpaid balance of purchase price with a prayer for replevin. So
upon demand. The matter reached the court only because of the refusal of the when Filipinas Investment seized the car and extrajudicially
mortgagor to comply with his obligation to return the thing mortgaged on default foreclosed the mortgage, it renounced any and all rights which it might
and foreclosure of mortgage, thus the mortgagor should pay for the expenses have had under the promissory note and the unpaid balance, and
incurred by the mortgagee. attorney’s fees.
16. Contention of Filipinas Investment: they are entitled to attorney’s fees and
DOCTRINE: What is prohibited under 1484 (3) is choosing both to foreclose the actual expenses and cost of suit because of unjustifiable failure and refusal of
mortgage AND recover unpaid balance of price, because the seller can only Ridad Spouses to comply with their obligations (one of which is to surrender
choose one remedy. This is to avoid unjust enrichment. However, seller may the car upon Filipinas Investment’s demand).
recover expenses, costs, and attorney’s fees incurred in enforcing his rights in the a. They claim that what is prohibited under Art. 1484 (3) of the Civil
manner delineated by law (in short bringing the matter before the court), since Code is recovery of unpaid balance of the purchase price by
this is different from the unpaid balance. means of an action other than a suit for replevin.
b. They also claim that Luneta Motor Co. v. Salvador is inapplicable
FACTS: to the present case because the remedy sought in that case was
26. Ridad Spouses bought a Ford Consul sedan for P 13 371.40 from Supreme conjunctive and not alternative.
Sales & Development Corporation (Filipinas Investment’s assignor-in- c. On the contrary in the present case, the court awarded
interest). attorney’s fees, cost of suit and expenses incurred in relation to
27. P 1160 was paid on delivery and the balance of P 12 211.50 was to be paid the seizure of the motor vehicle by virtue of the writ of replevin
in 24 equal monthly installments, with interest of 12% per annum, secured in the same action, because Filipinas Investment was compelled
by a promisory note and a chattel mortgage on the car. to do that on account of Ridad Spouses’ unjustifiable failure and
28. Spouses failed to pay 5 consecutive installments on the remaining balance of refusal to fulfill their obligation to return the car.
P 5 274.53. d. IN ANY CASE, award of attorney’s fees and costs of suit with
29. So, Filipinas Investment filed a replevin suit for the seizure of the car or expenses incurred were all stipulated in both the promissory note
recovery of unpaid balance. While the case was pending, Filipinas Investment and chattel mortgage contract. Even in the absence of stipulation
regarding attorney’s fees, awarding of this is discretionary on the
part of the court anyway.
17. Art. 148428 governs this case at hand.
a. Ultimate and salutary purpose is to prevent the vendor from
circumventing the Recto Law. Congress sought to protect the buyers
on installment who more often than not have been victimized by
sellers who, before the enactment of this law, succeeded in unjustly
enriching themselves at the expense of the buyers. Before Recto
Law and New Civil Code, aside from recovering the goods sold,
upon default of the buyer in the payment of two installments sellers
still retained for themselves all amounts already paid, and in
addition, were adjudged entitled to damages, such as attorney's fees,
expenses of litigation and costs.
18. The Court agrees with this legislative intent, however does not agree that the
mortgagee is not entitled to protection against perverse mortgagors who
plainly refuse to deliver the thing mortgaged upon foreclosure.
19. Since the mortgagee would enforce his rights through the means and within
the limits delineated by law (so in short he’d file a case in court), then it
logically follows that the necessary expenses incurred in the prosecution by
the mortgagee of the action for replevin so that he can regain possession of
the chattel, should be borne by the mortgagor.
20. The matter reached the court only because of the refusal of the
mortgagor to comply with his obligation to return the thing mortgaged
on default and foreclosure of mortgage, thus the mortgagor should pay
for the expenses incurred by the mortgagee.
21. Recoverable expenses would, in our view, include expenses properly
incurred in effecting seizure of the chattel and reasonable attorney's fees in
prosecuting the action for replevin.

28 (3) Foreclose the chattel mortgage on the thing sold, if one has been constituted, should the
In a contract of sale of personal property the price of which is payable in installments, the
vendor may exercise any of the following remedies: vendee's failure to pay cover two or more installments. In this case, he shall have no further
(1) Exact fulfillment of the obligation, should the vendee fail to pay; action against the purchaser to recover any unpaid balance of the price. Any agreement to the
(2) Cancel the sale, should the vendee's failure to pay cover two or more installments; contrary shall be void.
12 PCI Leasing v. Giraffe-X Creative (ARIELLE) pay the balance of the unpaid rentals and would be able to keep the equipment,
July 12, 2007 | Garcia, J. | Purported lease with option to buy then the real contract between the parties was a sale of movable on installment
disguised as a lease agreement.
PETITIONER: PCI Leasing and Finance, Inc.
RESPONDENTS: Giraffe-X Creative Imaging, Inc.
FACTS:
SUMMARY: PCI Leasing and Giraffe entered into a Lease Agreement where 1. Petitioner PCI Leasing and respondent Giraffe entered into a Lease
PCI leased to Giraffe a set of office equipment, payable in installments for 36 Agreement, whereby PCI leased out to Giraffe one set of Silicon High Impact
months. A year into the agreement, Giraffe defaulted in its monthly rental. Graphics and accessories worth P3.9 million and one ujit of Oxberry
Because of this, PCI a pay-or-surrender demand letter to Giraffe. The demand Cinescan worth P6.5 million.
went unheeded. PCI then instituted a case against Giraffe and prayed for the 2. In connection with the agreement, the parties signed two separate documents,
issuance of a writ of replevin for the recovery of the leased equipment. The trial denominated as Lease Schedule. Parts of the lease agreement were two
court then issued the writ and allowed PCI to secure the seizure and delivery of separate documents denominated Disclosure Statements of Loan/Credit
the equipment. Giraffe then filed a Motion to Dismiss, claiming that the seizure Transaction (Single Payment or Installment Plan) that Giraffe executed for
of the two leased equipment stripped PCI of a cause of action because according each of the leased equipment.
to Art. 1484, PCI is barred from further pursuing any claim arising from the lease 3. The disclosure statements described Giraffe, vis-à-vis the two
agreement. Giraffe also claimed that the agreement between them is in reality a aforementioned equipment, as the borrower who acknowledged the net
lease of movables with option to buy, under Arts. 1484 and 1485. PCI on the other proceeds of the loan, the net amount to be financed, the financial charges,
hand maintains that the contract is a straight lease without an option to buy, the total installment payments that it must pay monthly for 36 months,
claiming that the relationship between the parties is one between an ordinary exclusive of 36% per annum late payment charges.
lessor and lessee. Trial court granted Giraffe’s motion to dismiss (see Fact #14 4. Thus, for the Silicon High Impact Graphics, Giraffe agreed to pay P116,878
for grounds). Hence, this petition. The issue in this case is WoN the lease monthly, and for Oxberry Cinescan, P181,362 monthly.
agreement is a straight lease or a lease with option to buy. 5. According to the Lease Agreement, Giraffe undertook to remit the
P3,120,000 by way of guaranty deposit, a sort of performance and
The SC held that the contract is a lease with option to buy. The demand letter sent compliance bond for the two equipment.
to Giraffe stated PCI’s claim for the full payment of the P8.2 unpaid balance OR 6. The same agreement also embodied a standard acceleration clause, operative
the surrender of the financed asset under pain of legal action. Evidently, the letter in the event Giraffe fails to pay any rental or other accounts due.
did not make a demand for the payment AND return of the equipment; only either 7. A year into the agreement, Giraffe defaulted in its monthly rental. Following
of the two was required. The demand could only be that the respondent need not a three-month default, PCI addressed a formal pay-or-surrender-equipment
return the equipment if it paid P8.2 outstanding balance, suggesting that Giraffe type of demand letter to Giraffe. The demand went unheeded.
can keep possession of the equipment if it exercises its option to acquire the same 8. PCI then instituted the instant case against Giraffe and prayed for the issuance
by paying the unpaid balance. the transaction in this case is a lease in name only. of a writ of replevin for the recovery of the leased property.
The so-called monthly rentals are in truth monthly amortizations of the price of 9. Upon PCI Leasing’s posting of a replevin bond, the trial court issued a writ
the leased office equipment. On the whole, the Court rules that the PCI LEASING of replevin, allowing PCI to secure the seizure and delivery of the equipment
– GIRAFFE lease agreement is in reality a lease with an option to purchase the covered by the basic lease agreement.
equipment. This has been made manifest by the actions of PCI itself, foremost of 10. Instead of an answer, Giraffe filed a Motion to Dismiss, arguing that the
which is the declarations made in its demand letter to Giraffe. There could be no seizure of the two leased equipment stripped PCI of a cause of action. Giraffe
other explanation than that if Giraffe paid the balance, then it could keep the argues that according to Art. 1484 of the Civil Code on installment sales of
equipment; if not, then it should return them. This is clearly an option to purchase personal properties, PCI is barred from further pursuing any claim arising
given to Giraffe. Being so, Art. 1485 should apply. from the lease agreement and the companion contract documents, adding that
the agreement between the parties is in reality a lease of movables with option
DOCTRINE: Where a lease agreement over equipment is without an express to buy.
option to purchase, but nevertheless when a final demand is given prior to suit, 11. According to Giraffe, the given situation squarely brings into applicable play
the demand letter indicates clearly it was within the option of the lessee to fully
Arts. 14841 and 14852 of the Civil Code, commonly referred to as the Recto made to appear as a financial leasing3, a term defined in Sec. 3(d) of RA
Law. 8556.
12. It is Giraffe’s position that Art. 1484 applies to its contractual relation with 45. The Court takes into account the following mix:
PCI because the lease agreement in question is really a lease with option to a. The imperatives of equity
buy under Art. 1485. Consequently, so Giraffe argues, upon the seizure of the b. Contractual stipulations in question
leased equipment pursuant to the writ of replevin, which is equivalent to c. Actuations of the parties vis-à-vis their contract
foreclosure, PCI has no further recourse against it. 46. In BA Finance Corporation vs. CA which involved a motor vehicle, the Court
13. In its Opposition to the motion to dismiss, PCI maintains that its contract with has treated the purported financial lease as actually a sale of movable property
PCI is a straight lease without an option to buy. PCI then rejects the on installments and prevented recovery beyond the buyers’ arrearages.
applicability of Art. 1484 in relation to Art. 1485, claiming that the 47. In BA Finance, the Court said that the transaction is one of a “financial lease”
relationship between the parties is one between an ordinary lessor and an or “financial leasing,” where a financing company would, in effect, initially
ordinary lessee. purchase a mobile equipment and turn around to lease it to a client who gets,
14. The trial court granted Giraffe’s motion to dismiss, on the ground that: in addition, an option to purchase the property at the expiry of the lease
a. The lease agreement package, as memorialized in the contract documents, is akin period.
to the contract contemplated in Art. 1485 48. In the instant case, PCI acquired the office equipment in question for
b. Giraffe’s loss of possession of the leased equipment consequent to the their subsequent lease to Giraffe, with the latter undertaking to pay a
enforcement of the writ of replevin is akin to foreclosure, the condition precedent monthly fixed rental for 36 months.
for application of Arts. 1484 and 1485. 49. As a measure of good faith, Giraffe made an up-front guarantee deposit for
15. PCI contends that the financial leasing arrangement it concluded with Giraffe P3,120,000 or a total of P10,531,116 for the whole 36 months.
represents a straight lease covered by RA 5980 or the Financing Company 50. The basic agreement provides that in the event Giraffe fails to pay, PCI
Act, and is outside the application and coverage of the Recto Law. To PCI, shall have cumulative remedies, but not limited to the following:
Ra 5980 defines and authorizes its existence and business. a. Obtain possession of the equipment
b. Retain all amounts paid to it
ISSUE/s: c. Recover all accrued and unpaid rentals
3. WoN the lease agreement between PCI and Giraffe is a straight lease or a d. Recover all rentals for the remaining term of the lease had it not been
lease with option to buy – It is a lease with option to buy. cancelled, as additional penalty
e. Recovery of any and all amounts advanced by PCI for Giraffe’s account
f. Recover all expenses incurred in repossessing, removing, repairing, and
RULING: The instant petition is DENIED and the trial court’s decision is storing the property
AFFIRMED. g. Recover all damages suffered by PCI by reason of the default.
51. In addition, Sec. 61 of the Lease Agreement states that the guaranty deposit
RATIO: shall be forfeited in the event that Giraffe returns the equipment before the
43. PCI’s recourse is without merit. To PCI, the absence of an option-to-buy expiration of the lease.
stipulation in a financial leasing agreement prevents the application of 52. The demand letter sent to Giraffe stated PCI’s claim for the full payment
Arts. 1484 and 1485. This is not correct. of the P8.2 unpaid balance OR the surrender of the financed asset under
44. The Court can allow that the underlying lease agreement has the earmarks or
3
1 A mode of extending credit through a non-cancelable lease contract under which the lessor purchases or
In a contract of sale of personal property the price of which is payable in installments, the vendor may
acquires, at the instance of the lessee, machinery, equipment, office machines, and other movable or
exercise any of the following remedies:
immovable property in consideration of the periodic payment by the lessee of a fixed amount of money
1) Exact fulfillment of the obligation, should the vendee fail to pay;
sufficient to amortize at least seventy (70%) of the purchase price or acquisition cost, including any
2) Cancel the sale, should the vendee’s failure to pay cover two or more installments;
incidental expenses and a margin of profit over an obligatory period of not less than two (2) years during
3) Foreclose the chattel mortgage on the thing sold, if one has been constituted, should the
which the lessee has the right to hold and use the leased property but with no obligation or option on his
vendee’s failure to pay cover two or more installments. In this case, he shall have no further
part to purchase the leased property from the owner-lessor at the end of the lease contract.
action against the purchaser to recover any unpaid balance of the price. Any agreement to the
contrary shall be void.
2
The preceding article shall be applied to contracts purporting to be leases of personal property with option
to buy, when the lessor has deprived the lessee of the possession or enjoyment of the thing.
pain of legal action. Evidently, the letter did not make a demand for the turn, precludes PCI from maintaining an action for recovery of accrued
payment AND return of the equipment; only either of the two was rentals or the recovery of the balance of the purchase price plus interest.
required.
53. The demand could only be that the respondent need not return the equipment Additional Notes:
if it paid P8.2 outstanding balance, suggesting that Giraffe can keep
possession of the equipment if it exercises its option to acquire the same 1. Giraffe had paid the equivalent of about a year’s lease rentals, or a total of
by paying the unpaid balance. P3.5 million, more or less. Throw in the guaranty deposit and Giraffe had
54. If Giraffe was not minded to exercise its option of acquiring the equipment made a total cash outlay of P6.6 million in favor of PCI.
by returning them, then it need not pay the outstanding balance. This is the 2. The replevin-seized leased equipment had an estimated residual value of P6.9
logical import of the letter: that the transaction in this case is a lease in million at the time the civil case was instituted. Adding all cash advances, the
name only. The so-called monthly rentals are in truth monthly total value which PCI had obtained by virtue of its lease agreement with
amortizations of the price of the leased office equipment. Giraffe amounts to P13.5 million.
55. On the whole, the Court rules that the PCI LEASING – GIRAFFE lease 3. The acquisition cost for both the Silicon High Impact Graphics and the
agreement is in reality a lease with an option to purchase the equipment. Oxberry Cinescan was P8,1 million. Subtracting the acquisition cost of P8.1
This has been made manifest by the actions of PCI itself, foremost of million from the total amount P13.5 million creditable to Giraffe, it would
which is the declarations made in its demand letter to Giraffe. appear that PCI realized a gross income of P5.4 million from its lease
56. There could be no other explanation than that if Giraffe paid the balance, then transaction with Giraffe. The P5.4 million is not yet a final figure as it does
it could keep the equipment; if not, then it should return them. This is clearly not include rentals in arrears, penalties, and interest earned by the guaranty
an option to purchase given to Giraffe. Being so, Art. 1485 should apply. deposit.
57. The present case reflects a situation where the financing company can 4. A financing arrangement has a purpose which is at once practical and
withhold and conceal its intention to sell the property subject of the finance salutary. RA 8556 was precisely enacted to regulate financing companies
lease, in order that the provisions of the Recto Law may be circumvented. operations with the end in view of strengthening their critical role in
58. It may be, as PCI pointed out, that the basic lease agreement does not providing credit and services to small and medium enterprises and to curtail
contain a purchase option contract. acts and practices prejudicial to the public interest.
59. The absence, however, does not necessarily argue against the idea that 5. As a regulated activity, financing arrangements are not meant to quench only
what the parties are into is not a straight lease, but a lease with option to the thirst for profit.
purchase. 6. RA 5980 partakes of a supervisory or regulatory legislation, merely providing
60. The Court has, to be sure, been long aware of the practice of vendors of a regulatory framework for the organization, registration, and regulation of
personal property of denominating a contract of sale on installment as one of the operations of financing companies.
lease to prevent the ownership of the object of the sale from passing to the 7. It does not specifically define the rights and obligations of parties to a
vendee until and unless the price is fully paid. financial leasing arrangement.
61. In choosing, through replevin, to deprive Giraffe of possession of the 8. PCI foists the argument that the Recto Law (the Civil Code provisions on
equipment, PCI waived its right to bring an action to recover unpaid installment sales of movable property) does not apply to a financial leasing
rentals on the leased items. Par 3 of Art. 1484 in relation to Art. 1485 agreement because such agreement, by definition, does not confer on the
cannot be any clearer. lessee the option to buy the property subject of the financial lease.
62. The remedies provided for in Art. 1484 are alternative, not cumulative.
The exercise of one bars the exercise of the others. This limitation applies to
contracts purporting to be leases of personal property with option to buy by
virtue of the same Art. 1485.
63. The condition that the lessor has deprived the lessee of possession or
enjoyment of the thing for the purpose of applying Art. 1485 was fulfilled in
this case by the filing by PCI of the complaint for a sum of money with prayer
for replevin to recover possession of the equipment.
64. By virtue of the writ of seizure, PCI has deprived Giraffe of their use, and in
13 Hermanosa and Legarda v. Saldaña (Leonardo) lots.
Jan. 28, 1974 | J. Teehankee | Remedy for anticipatory breach for immovables 168. Hermanos and Legarda replied that since he failed to complete the total 120
installments by May 1958 as stipulated in the contract to sell, “pursuant to
PETITIONER: Legarda Hermanos, Jose Legarda the provisions, all the amounts paid in accordance to the agreement together
RESPONDENTS: Felipe Saldaña, CA 5th Division with the improvements have been considered rents paid and payment for
damages for such failure to pay for the lots. Cancellation is in order.”
SUMMARY: Saldaña contracted with Hermanos and later on with Legarda for 169. Trial court sustained the cancellation of the contracts, but the CA reversed it
the purchase of 2 lots in Hermanos’ subdivision. Saldaña paid for 95 (out of the and ordered Hermanos and Legarda to deliver to Saldaña the possession of 1
120) installments in the span of 8 (out of the 10) years. However afterwards, he of the 2 lots at his choice and to execute the corresponding deed of
failed to pay the rest of the balance till the due date. Saldaña subsequently informs conveyance. Hermanos and legarda now bring this petition for review before
the Hermanos and Legarda that he has plans to build a house and contrinue to pray the SC.
for the remaining balance. The two contend that his failure caused for the contract
to be cancelled according the stipulation of the contract. The trial court sided with ISSUE/s:
Hermanos and Legarda but the CA reversed this to say that Saldaña should be 54. WoN Hermanos and Legarda may cancel the sale due to Saldaña’s failure to
given 1 of the 2 lots at his choosing since the amount he has paid for is already pay the whole amount in the stipulated period even though he has paid for
more than the amount of one lot. The SC upheld the CA’s decision and argued in more than 1 lot. – NO, this is contrary to the broad principles of equity and
the basis of justice and equity for such decision. justice.

DOCTRINE: Under Art. 1234 of the Civil Code, “If the obligation has been RULING: SC affirms the CA decision in that 1 out of the 2 lots should be given to
substantially performed in good faith, the obligor may recover as though there had Saldaña (land of his choice).
been a strict and complete fulfillment, less damages suffered by the oblige.”
RATIO:
144. The money paid by Saldaña was more than the value of 1 lot of P1,500 and
FACTS: hence 1 of the 2 as chosen by Saldaña should be considered as fully paid. It
165. Saldaña entered into 2 written contracts with Legarda Hermanos, a is only fair and just to do in accordance with law an equity. And as stated, the
subdivision owner. Hermanos agreed to sell Lots Nos. 7 and 8 of Block 5N monthly payments for 8 years made were applied to Saldaña’s account
of the subdivision with an area of 150 sqm. each. The sum of P1,500,000 per without specifying between the 2 lots which this account applies to.
lot was to be paid over the span of 10 years divided into 120 equal monthly 145. Hermanos and Legarda have not been denied substantial justice for under Art.
installments of P19.83 with 10% interest per annum to commence on May 1234 of the Civil Code, “If the obligation has been substantially performed
26, 1948 (date of execution of contracts). Subsequently, Hermanos in good faith, the obligor may recover as though there had been a strict and
partitioned the subdivision among the brothers and sisters, and 2 lots were complete fulfillment, less damages suffered by the oblige” and “ that in the
among the allotted to Jose Legarda. interest of justice and equity, the decision appealed from may be upheld upon
166. Saldaña faithfully paid for 8 continuous years about 95 (out of the 120) the authority of Art. 1234.”
monthly installments totaling P3,582.06 up to the month of Feb. 1956. This 146. [Not expressly stated in the case but just trying to connect it to the syllabus
was evidenced by his own statement of account, without distinguishing under Art. 1591, read with caution LOL] Art. 1591 provides:
which of the 2 lots the payment was for, but saying that he had paid more “Should the vendor have reasonable grounds to fear the
than the stipulated purchase price of P1,500 for 1 lot.. He failed to make loss of immovable property sold and its price, he may
any further payments since then till the filing of the suit in the CFI of Manila. immediately sue for the rescission of the sale.
167. Saldaña wrote a letter to Hermanos and Legarda, stating his intent to build a Should such ground not exist, the provisions of Article
house on the lots but was prevented from doing so since there was no road to 119129 shall be observed.”
these lots yet and that he was going to continue to pay his due balance on the In this case, there was no demand from Hermanos and Legarda for the

29
Art. 1191: The power to rescind obligations is implied in reciprocal ones, in case one of the obligors
should not comply with what is incumbent upon him.
remaining balance of the amount due and no prior cancellation of the contract
before Saldaña’s letter was given to them regarding his intentions. Seems
more of the exception for the rule under the provision than an interpretation
of Art. 1591 in its applicability, and using the broad principles of justice and
equity.
14 JESTRA DEV. AND MANAGEMENT CORP. V. PACIFICO
(Megan) (guys I suggest you read the long version ng digest cause ang hirap nung case may
June 30, 2007 | Carpio-Morales, J. | How to determine years of installment compute compute pa huhu)

PETITIONER: Jestra Development and Management Corporation DOCTRINE: R.A. No. 6552 was enacted to protect buyers of real estate on
RESPONDENT: Daniel Ponce Pacifico, represented by his attorney-in-fact Jordan installment against onerous and oppressive conditions. While the seller has under the
M. Pizarras Act the option to cancel the contract due to non-payment of installments, he must
afford the buyer a grace period to pay them and, if at least two years installments
SUMMARY: Pacifico signed a Reservation Application for the purchase of a house have already been paid, to refund the cash surrender value of the payments.
and lot in Parañaque. He paid 20,000 for the reservation fee. The total purchase price
is 2.5M. The payment scheme provides that 30% would be paid for the down FACTS:
payment and such would be paid in 6 monthly installments interest free. Upon full 67. Daniel Ponce Pacifico signed a Reservation Application with Fil-Estate
payment of the down payment, a Contract to Sell would be signed between Pacifico Marketing Association for the purchase of a house and lot in Brgy. La Huerta,
and Jestra, the owner and developer. The 70% balance would be payable in 10 years Municipality of Parañaque.
but it did not state when the payment of the balance would commence. Pacifico was 68. He paid the reservation fee of Php 20,000.
unable to comply with the schedule so he requested Jestra that he pay the down 69. Total purchase price would be Php 2.5M and the downpayment is 30% or
payments in an amount that he can afford. Jestra acceded provided that penalties and Php 750K interest free to be paid in 6 monthly installments every 5th of the
surcharges would be paid. Despite not having paid the full payment of the down month starting July 1996 until Dec 1996. (note that yung downpayment lang
payment, a Contract to Sell was executed. In such contract it provided that Pacifico yung 6 installments)
should have had completely paid the down payment and that the 70% balance 70. The 20,000 reservation fee formed part of the down payment therefore the
payment should have commenced. A few months after, Pacifico requested for a payment was set to be Php 121,666.66 every month for the down payment.
restructuring of the agreement. Jestra then through counsel, sent Pacifico a final 71. Under the Reservation Application, upon the full payment of the down
demand for the payment of the 11 installments due on the 70% balance. Further, payment, a Contract to Sell was to be signed with the owner and developer,
Jestra demanded payment of the as penalty for the belated settlement of the down Joprest Development and Management Corp. (now Jestra instead of Joprest)
payment. Jestra reminded Pacifico that he had the by virtue of Sec. 5 of the contract 72. The 70% balance was to be payable in 10 years with an interest of 21% per
it has the right to automatically cancel or rescind the same if the fails to comply with annum. However, the Reservation Application was silent on when the
the terms. Another restructuring was made as per request of Pacifico. However, payment of the balance would commence.
Pacifico informed Jestra that due to sudden financial difficulties, he was suspending 73. Since Pacifico was unable to comply with the scheduled payments, Pacifico
the payment of his obligation and that he requested that the postdated checks he requested Jestra to allow him to pay the down payments in an amount that he
issued be returned to him. Jestra denied Pacifico’s request to suspend payment and could afford.
for the return of the postdated checks but allowed him to sell the property but only 74. Jestra acceded provided that penalties and surcharges would be paid.
until April 15, 1998. If he fails to sell the property within the given date, Jestra would 75. Despite not having paid the full payment of the down payment, (may
re-open it for sale. However, Pacifico filed a complaint before the HLURB claiming balance ba ng Php 260K) Pacifico and Jestra executed a Contract to Sell on
that despite his full payment of the down payment, Jestra failed to deliver to him the March 6, 1997.
property and Jestra instead sold the property to another. The Housing and Land Use 76. The contract was silent on the unsettled balance of the down payment.
Arbiter ruled in favor of Pacifico since Jestra failed to comply with Sec. 3 of RA 77. However under the Contract to Sell, it provides that Pacifico should have has
6552 requiring that the seller should give the buyer what he paid for. The Arbiter fully paid the down payment on November 5, 1996, and that the installments
ordered Jestra to pay and reimburse to the complainant the total payments.The Board for the 70% balance should have commenced on Dec. 7, 1996.
of Commissioners affirmed the decision, as well as the Office of the President and 78. A few months after, Pacifico requested Jestra that the balance be restructured
the CA. However, the SC reversed the decision that since Pacifico was not able to in ligh of the “present business condition” (case did not state what this is
pay at least 2 years worth of installments, he is not protected under such law. As about or what happened)
respondent failed to pay at least two years of installments, he is not entitled to a 79. Pacifico had fully paid the 30% down payment on Nov. 27, 1997.
refund of the cash surrender value of his payments. 80. By Dec. 4, 1997, he paid a total of Php 846,600.
81. Jestra then through counsel, sent Pacifico a final demand for the payment of 96. The appellate court concluded that Pacifico had already paid at least two
the 11 installments due on the 70% balance of the purchase price, or a total years of installments, wherein it adopted the formula used by the HLURB
of Php 444,738.88 inclusive of 21% interest per annum. by dividing the amount of P846,600 by the monthly amortization of P34,983
82. Further, Jestra demanded payment of the Php 73,750 as penalty for the to thus result to a quotient of 24.2 months.
belated settlement of the down payment. 97. However, Jestra contests the computation and claims that the amount of
83. Jestra reminded Pacifico that he had the by virtue of Sec. 5 of the contract it 76,600 represents as penalty and is separate from the 30% down payment.
has the right to automatically cancel or rescind the same if the fails to comply Therefore, it should be excluded in determining the total number of
with the terms. installments. It claims that the proper divisor is not P34,983 but P39,468
84. Pacifico later requested Jetra to restructure his unsettled obligation. His since the parties agreed to restructure the amortizations owing to respondent's
request was granted on condition that the interest due on Dec. 1996 to Nov. inability to comply with the schedule of payments
1997, would be added to the 70% balance on the purchase price. 98. The two year-installment threshold is important because the law provides that
85. Pacifico issued 12 postdated Security bank checks to cover his monthly if at least two years installments have already been paid, the seller has
amortizations. the obligation to refund the cash surrender value of the payments.
86. However, Pacifico informed Jestra that due to sudden financial difficulties,
he was suspending the payment of his obligation and that he requested that ISSUE/s:
the postdated checks he issued be returned to him. 8. WoN Pacifico was able to pay at least two years of installments – NO.
87. To help him with his financial condition, he wanted to dispose of the property
(etong property na di niya mabayaran) to recover his investment. RULING: WHEREFORE, the petition is GRANTED. The assailed Decision and
88. Jestra denied Pacifico’s request to suspend payment and for the return of the Resolution dated January 31, 2005 and March 16, 2005 of the Court of Appeals are
postdated checks but allowed him to sell the property but only until April 15, hereby REVERSED and SET ASIDE. The complaint of respondent, Daniel Ponce
1998. If he fails to sell the property within the given date, Jestra would re- Pacifico, is DISMISSED.
open it for sale.
89. Jestra then subsequently sent Pacifico a notarial Notice of Cancellation RATIO:
notifying him of exercising its right to cancel the Contract to Sell. 21. This Court finds that neither of the parties' computations is in order.
90. In a separate move, Jestra sent a letter demanding payment of the monthly 22. While, under Section 3 of R.A. No. 6552, the down payment is included in
amortizations inclusive of the penalties. It gave him until June 1, 1998 and computing the total number of installment payments made, the proper divisor
that if he fails, the Contract to Sell would be automatically cancelled and re- is neither P34,983 nor P39,468, but P121,666.66, the monthly installment
opened for sale. on the down payment.
91. However, Pacifico filed a complaint before the HLURB claiming that despite 23. The P750,000 down payment was to be paid in six monthly installments. If
his full payment of the down payment, Jestra failed to deliver to him the the down payment of P750,000 is to be deducted from the total payment of
property and Jestra instead sold the property to another. He prayed that the P846,600, the remainder is only P96,600. Since Pacifico was able to pay the
2nd sale be declared a nullity and that Jestra be ordered to deliver the property down payment in full eleven (11) months after the last monthly installment
to him. was due, and the sum of P76,600 representing penalty for delay of payment
92. The Housing and Land Use Arbiter held Jestra liable for failure to comply is deducted from the remaining P96,600, only a balance of P20,000 remains.
with Sec. 3 of RA 6552 requiring payment by the seller of the cash surrender TOTAL AMOUNT PAID: Php 846,600.
value of the buyer’s payments (basically the seller should pay/give the buyer 1. DOWN PAYMENT: 750,000
what the buyer paid for) and ordered it to pay and reimburse to the 96,000
complainant the total payments. 2. PENALTY: 76,000
93. The Board of Commissioners affirmed the decision modifying only by = 20,000
deleting the award for damages. Bale Pacifico only paid 20K for the 70% balance purchase price.
94. The Office of the President also affirms the decision and by petition for 24. R.A. No. 6552 was enacted to protect buyers of real estate on installment
review, the CA affirmed the orders of the OP. against onerous and oppressive conditions. While the seller has under the Act
95. Hence, this Petition. the option to cancel the contract due to non-payment of installments, he must
afford the buyer a grace period to pay them and, if at least two years
installments have already been paid, to refund the cash surrender value
of the payments.
25. As respondent failed to pay at least two years of installments, he is not
entitled to a refund of the cash surrender value of his payments. What
applies to the case instead is Section 4 of the same law:
SECTION 4. In case where less than two years of installments were paid,
the seller shall give the buyer a grace period of not less than sixty days
from the date the installment became due.

If the buyer fails to pay the installments due at the expiration of the grace
period, the seller may cancel the contract after thirty days from receipt
by the buyer of the notice of cancellation or the demand for rescission
of the contract by a notarial act.

26. While Pacifico issued checks to cover the same, the first two were dishonored
due to insufficiency of funds.
27. When he was notified of the dishonor of the checks, he took no action
thereon, hence, the 60 days grace period lapsed. Pacifico made no further
payments thereafter. Instead, he requested for suspension of payment and for
time to dispose of the property to recover his investment.
28. Pacifico admits that Jestra was justified in canceling the contract to sell via
the notarial Notice of Cancellation which he received on May 13, 1998. The
contract was deemed cancelled 22 30 days from May 13, 1998 or on June 12,
1998.
15 MCLAUGHLIN V. CA (MAYUMI) receipt thereof. A mere notice or letter, would not suffice. (Syllabus doctrine, but
October 10, 1986 | Feria, Actg. C.J. | How Cancellation of Contract Can Be Effected not found verbatim in the case)

PETITIONER: Luisa McLaughlin In case where less than two years of installments were paid, the seller shall give
RESPONDENTS: The Court of Appeals and Ramon Flores the buyer a grace period of not less than sixty days from the date the installment
became due. If the buyer fails to pay the installments due at the expiration of the
SUMMARY: grace period, the seller may cancel the contract after thirty days from receipt
Luisa (seller) and Ramon (buyer) entered into a contract of conditional sale for a by the buyer of the notice of the cancellation or the demand for rescission of
property at the price of) P140,000. In their deed of conditional sale, Ramon was the contract by a notarial act. (from the case)
to pay P26,550 upon the execution of the deed, and to pay the balance not later
than May 31, 1977. Luisa filed a complaint to rescind the contract citing that FACTS:
Ramon failed to pay the balance due. However, the parties entered into a [This case involves a lot of dates. Please check the end of the digest to see the
compromise agreement wherein Ramon would settle his obligation. He would do timeline of dates for clarification]
this by paying P50,000 upon signing the agreement, and pay the balance in two 26. On Feb 28, 1977, Luisa McLaughlin (Luisa) and Ramon Flores (Ramon)
equal installments. Further, Ramon was to pay P1,000 per month as rentals for the entered into a contract of conditional sale of real property for P140,000.
use of the property from December 1979 until full payment. Furthermore, the Luisa as the seller and Ramon as the buyer. According to the deed of
compromise agreement stated that if Ramon fails to pay, Luisa will be entitled to conditional sale, the payment shall be made:
a writ of execution rescinding the contract of conditional sale, and all money paid a. P26,550 upon the execution of deed, and
by Ramon will be considered liquidated damages. One day, Luisa demanded b. P113,450 (balance) not later than may 31, 1977
Ramon to pay the balance of both installments. 15 days later, Ramon sent a letter 27. The parties also agreed that the balance shall bear interest at 1% per month
back signifying his willingness and intention to pay. However, Luisa filed a starting from December 1, 1976 until full price is paid.
motion for writ of execution stating that Ramon still did not pay. The trial court 28. On June 19, 1979, Luisa filed a complaint with the CFI of Rizal for the
granted the writ. Ramon filed for a motion for reconsideration and also tendered a rescission of the deed of conditional sale because of Ramon’s failure to pay
Pacific Banking Corporation certified manager’s check payable to the order of the balance.
Luisa which would cover the entire obligation. The trial court denied the 29. On December 27, 1979, the parties submitted a compromise agreement which
reconsideration. Ramon elevated this to the CA who ruled in his favor. Thus, Luisa the court used as basis to render their decision in January 22, 1980. In the
elevated this case to the SC. compromise agreement:
a. Ramon acknowledged his indebtedness to Luisa under the deed of
The SC ruled that according to the Maceda Law, Luisa could cancel the contract conditional sale in the amount of P119,050.71
after thirty days from receipt by Ramon of the notice of cancellation. Considering b. The said debt shall be payable as follows
Luisa’s motion for execution as a notice of cancellation, she could cancel the i. P50,000 upon signing the agreement, and
contract of conditional sale after thirty days from receipt by Ramon of said motion. ii. P69,059.71 (balance) in two equal installments on June 30,
However, Ramon’s tender of payment together with his motion for 1980 and December 31, 1980
reconsideration was well within the 30-day period granted by law. Moreover, that c. Ramon will pay P1,000 monthly rental beginning December 5, 1979
a certified bank manager’s check payable was a valid tender of payment. However, until the debt is paid so that he may use the property (subject matter
because Ramon did not tender the payment when Luisa refused to accept such, he of sale). [Paragraph 3]
should have consigned it to the court. Because he did not, consign it, the payment d. That in the event Ramon fails to comply with his obligations herein
for rentals still continued. Thus, upon full payment of the amount his debt, and the provided, the Luisa will be entitled to the issuance of a writ of
rentals in arrears, Ramon shall be entitled to a deed of absolute sale in his favor of execution rescinding the Deed of Conditional Sale of Real Property.
the real property in question. In such eventuality, Ramon hereby waives his right to appeal to
(from) the Order of Rescission and the Writ of Execution which the
DOCTRINE: At end of grace period, seller shall furnish buyer with a notarial Court shall render in accordance with the stipulations herein
notice of cancellation or demand for rescission, effective 30 days from buyer’s provided for. [Paragraph 6]
e. That in the event of execution, all payments made by Ramon will be the height of inequity to have this amount forfeited in favor McLaughlin.
forfeited in favor of Luisa as liquidated damages. [Paragraph 7] Under the questioned orders, McLaughlin would get back the property
30. Ramon paid Luisa P50,000 upon signing the agreement. He also paid an and still keep P101,550.00.”
“escalation cost” of P25,000.
31. On October 15, 1980, Luisa wrote to Ramon demanding him to pay the 38. Hence, Luisa filed elevated this case to the SC contending that the CA erred
balance on or before October 31, 1980. This demand included the amount of in not observing the provisions of Article 1306 of the Civil Code, and in
both installments (fact #4(b)i) abusing its judicial discretion by disregarding the penal clause stipulated in
32. 15 days later, Ramon sent a letter to Luisa signifying his willingness and the compromise agreement which was the basis of the decision of the lower
intention to the pay the full balance price, and at the same time demanded to court.
see the certificate of title of the property and the tax receipts.
33. Luisa filed a Motion for Writ of Execution on November 10, 1980 alleging ISSUES:
that Ramon failed to pay the installments due as well as the monthly rentals. 4. WoN the deed of conditional sale was cancelled –NO
Thus, Luisa prayed that:
a. The deed of conditional sale to be rescinded with forfeiture of all RULING: WHEREFORE, the decision of the Court of Appeals is AFFIRMED with
payments as liquidated damages, the following modifications:
b. The court order Ramon to pay the P1,000 back rentals since June
1980, and (a)Petitioner is ordered to accept from private respondent the Metrobank
c. Eviction of Ramon Cashier’s Check No. CC 004233 in her favor in the amount of P76,059.71
34. The trial court granted the motion for writ of execution on November 14, or another certified check of a reputable bank drawn in her favor in the same
1980 amount;
35. Ramon filed a motion for reconsideration on November 17, 1980 tendering
at the same time a Pacific Banking Corporation certified manager’s check in (b)Private respondent is ordered to pay petitioner, within sixty (60) days
the amount of P76,059.71, payable to the order of Luisa and covering the from the finality of this decision, the rentals in arrears of P1,000.00 a month
entire obligation including the installment due on December 31, 1980 from January 1, 1981 until full payment thereof; and
36. The trial court denied the motion for reconsideration, and issued the writ of
execution on November 25, 1980.
37. Ramon filed a petition for certiorari and prohibition with the CA who set (c)Petitioner is ordered to execute a deed of absolute sale in favor of private
aside the orders of the lower court. According to the CA respondent over the real property in question upon full payment of the
amounts as provided in paragraphs (a) and (b) above. No costs.
“It is significant to note that on November 17, 1980, or just seventeen
(17) days after October 31, 1980, the deadline set by McLaughlin, Flores RATIO:
tendered the certified manager’s check. We hold that the Song Fo ruling 1. The SC agrees with the CA that it would be inequitable to cancel the contract
is applicable herein considering that in the latter case, there was a 20-day of conditional sale and to have the amount of P101,550.00 already paid by
delay in the payment of the obligation as compared to a 17-day delay in him under said contract, excluding the monthly rentals paid, forfeited in favor
the instant case.” of petitioner, particularly after private respondent had tendered the amount of
xxx P76,059.71 in full payment of his obligation.
“as held in the recent case of New Pacific Timber & Supply Co., Inc. vs. a. NOTE: according to Ramon the amount of the manager’s check was
Hon. Alberto Seneris, L-41764, December 19, 1980, it is the accepted actually P148,126.97
practice in business to consider a cashier’s or manager’s check as cash 2. In the analogous case of De Guzman vs. Court of Appeals, this Court
and that upon certification of a check, it is equivalent to its acceptance sustained the order of the respondent judge denying the petitioners’ motion
and the funds are thereby transferred to the credit of the creditor” for execution on the ground that the private respondent had substantially
xxx complied with the terms and conditions of the compromise agreement, and
“Considering that Flores had already paid P101,550.00 under the directing the petitioners to immediately execute the necessary documents
contract to sell, excluding the monthly rentals paid, certainly it would be transferring to the private respondent the title to the properties.
3. In the case at bar, there was also substantial compliance with the compromise property, he did not follow it with a consignation or deposit of the sum due
agreement. with the court.
4. Luisa is invoking the SC’s ruling in Luzon Brokerage Co v. Maritame 12. Pursuant to a resolution on September 1, 1986, both parties submitted their
Building Co wherein the court talked about the Maceda law and how it respective manifestations which confirm that the Manager’s Check in
“recognizes and reaffirms the vendor’s right to cancel the contract to sell question was subsequently withdrawn and replaced by cash, but the cash was
upon breach and nonpayment of the stipulated installments but requires a not deposited with the court.
grace period after at least two years of regular installment payments x x x.” 13. According to Article 1256 of the Civil Code of the Philippines, if the creditor
5. On the other hand, Ramon also invokes the Maceda law (Ra. 6552) to whom tender of payment has been made refuses without just cause to
specifically Section 4 which states: accept it, the debtor shall be released from responsibility by the consignation
of the thing or sum due, and that consignation alone shall produce the same
In case where less than two years of installments were paid, the seller shall effect in the five cases enumerated therein
give the buyer a grace period of not less than sixty days from the date the 14. Article 1257 provides that in order that the consignation of the thing (or sum)
installment became due. If the buyer fails to pay the installments due at the due may release the obligor, it must first be announced to the persons
expiration of the grace period, the seller may cancel the contract after thirty interested in the fulfillment of the obligation; and Article 1258 provides that
days from receipt by the buyer of the notice of the cancellation or the demand consignation shall be made by depositing the thing (or sum) due at the
for rescission of the contract by a notarial act. disposal of the judicial authority and that the interested parties shall also be
notified thereof.
And section 7, which states “Any stipulation in any contract hereafter entered 15. Tender of payment must be distinguished from consignation. Tender is the
into contrary to the provisions of Sections 3, 4, 5 and 6, shall be null and antecedent of consignation, that is, an act preparatory to the consignation,
void” which is the principal, and from which are derived the immediate
consequences which the debtor desires or seeks to obtain. Tender of payment
6. The record does not contain the complete text of the compromise agreement, may be extrajudicial, while consignation is necessarily judicial, and the
and the decision approving it. However, assuming that under the terms of said priority of the first is the attempt to make a private settlement before
agreement the 2nd installment was due and payable when Luisa demanded proceeding to the solemnities of consignation.
payment of the balance on or before October 31, 1980, Luisa could cancel 16. In the case at bar, although Ramon had preserved his rights as a vendee in the
the contract after thirty days from receipt by Ramon of the notice of contract of conditional sale of real property by a timely valid tender of
cancellation. payment of the balance of his obligation which was not accepted by Luisa,
7. Considering Luisa’s motion for execution as a notice of cancellation, she he remains liable for the payment of his obligation because of his failure to
could cancel the contract of conditional sale after thirty days from deposit the amount due with the court.
receipt by Ramon of said motion. 17. Inasmuch as Luisa did not accept the payment, it was incumbent on Ramon
8. Ramon’s tender of payment together with his motion for reconsideration was to deposit the same with the court in order to be released from responsibility.
well within the 30-day period granted by law. (The motion for execution was Since he did not deposit said amount with the court, his obligation was not
filed November 7, and the motion for reconsideration was in November 17). paid and he is liable in addition for the payment of the monthly rental of
9. The tender made by Ramon via a certified bank manager’s check payable was P1,000.00 from January 1, 1981 until said obligation is duly paid, in
a valid tender of payment. The certified check covered not only the balance accordance with paragraph 3 of the Compromise Agreement.
of the purchase price, but also the arrears in the rental payments from June to 18. Upon full payment of the amount of P76,059.71 and the rentals in arrears,
December, 1980. Ramon shall be entitled to a deed of absolute sale in his favor of the real
10. Section 49, Rule 130 of the Revised Rules of court, “an offer in writing to property in question.
pay a particular sum of money or to deliver a written instrument or specific
property is, if rejected, equivalent to the actual production and tender of the TIMELINE:
money, instrument, or property” Feb 28, 1977: the parties entered into a contract of conditional sale
11. However, although Ramon had made a valid tender of payment which June 19, 1979: Luisa filed a complaint to rescind the contract for non-payment
preserved his rights as a vendee in the contract of conditional sale of real December 27, 1979: Compromise agreement
October 15, 1980: Luisa demanded payment of the two installments on or before
October 30, 1981
October 30, 1981: Ramon sent a letter to Luisa signifying his willingness and intention
to the pay the full balance price
November 7, 1980: Luisa filed a writ of execution to rescind the conditional contract
of sale
November 14, 1980: Trial court granted the writ of execution
November 17, 1980: Ramos filed a MR + tendered a certified manager’s check
November 25, 1980: Lower court issued the writ of execution (MR was dnied)
16 GARCIA V CA (Armand) installment a year and a half after the stipulated date, that was beyond the sixty-
July 24, 2012 | Perlas-Bernabe, J. | Cancellation of Contract; Maceda law day grace period under Section 4 of the Maceda Law. The buyers cannot use the
second sentence of Section 4 of the Maceda Law against the sellers’ alleged
PETITIONER: SPOUSES FAUSTINO AND JOSEFINA GARCIA, SPOUSES failure to give an effective notice of cancellation or demand for rescission because
MELITON GALVEZ AND HELEN GALVEZ, and CONSTANCIA ARCAIRA the sellers merely sent the notice to the address supplied by the buyers in the
represented by their Attorney-in-Fact JULIANA O. MOTAS Contract to Sell.
RESPONDENTS: COURT OF APPEALS, EMERLITA DE LA CRUZ, and
DIOGENES G. BARTOLOME FACTS:
SUMMARY: Spouses Garcia and Spouses Dela Cruz entered into a contract to 16. Spouses Garcia and Spouses Galvez and Emerlita dela Cruz entered into a
sell wherein the latter agreed to sell to the former, for P3,170,220, 5 parcels of contract to sell wherein the latter agreed to sell to the former, for P3,170,220,
land in Cavite. At the time of the execution of the said contract, 3 of the lots were 5 parcels of land in Cavite.
registered in the name of Angel Abedila. Spouses Garcia agreed upon terms of 17. At the time of the execution of the said contract, 3 of the lots were registered
payment namely a downpayment of P500k, then subsequently another P500k, and in the name of Angel Abelida from whom defendant acquires said properties
finally a final installment of P1.6M on Dec 31, 1993. Spouses Garcia failed to by virtue of a Deed of Absolute Sale dated March 31, 1989.
make good on the last payment but offered to pay a year and a half later, but de 18. As agreed upon, Spouses Garcia shall make a down payment of P500,000
la Cruz refused to accept because she had sold the same parcels to Bartolome for upon signing of the contract. The balance of P2,670,220 shall be paid in
P7M. Spouses Garcia filed a complaint for specific performance in the RTC installments.
alleging that they delayed payment due to alleged infirm and spurious contract a. P500,000 on Aug 30 1993
between the former owner and dela Cruz and that the signature of the former b. P1,670,220 on Dec 31 1993
owner was falsified. Dela Cruz denied such allegation in her reply and argued 19. On its due date on Dec 31, Spouses Garcia failed to pay the last installment.
that the provisio in the contract allowing for automatic rescission of the contract Sometime in July 1995, Spouses offered to pay the unpaid amount which
upon failure to pay the purchase price in full plus forfeiture of half the amount of had already been delayed by one and a half years which defendant refused to
amount paid was valid. RTC ruled in favor of Spouses Garcia. RTC also applied accept.
the Maceda law and stated that dela Cruz was not allowed to unilaterally cancel 20. On Sept 23, 1995, defendant sold the same parcels of land to intervenor
the contract to sell and found that the Spouses Garcia were justified in Diogenes G. Bartolome for P7,793,000.
withholding payment due to the alleged spurious sale. CA reversed RTC decision 21. In order to compel the defendant to accept payment in full satisfaction of the
and held that the obligation under the contract to sell did not arise due to the purchase price, and thereafter execute the necessary document of transfer,
failure of Spouses Garcia to pay in full the agreed purchase price on the stipulated Spouses Garcia filed a complaint for specific performance in the RTC.
date. Moreover, judicial action for the rescission of a contract is not necessary 22. They alleged in their complaint that they discovered infirmity in the Deed of
where the contract provides that it may be revoked and cancelled for Absolute Sale covering 3 lots, between their former owner Angel Abedila
violation of any of its terms and conditions. Issue is WoN Maceda Law is and the defendant, the same being spurious because the signature of Angel
applicable- NO, and WoN CA erred in failing to consider that dela Cruz acted in Abedila and his wife were falsified; and that at the time of the execution of
bad faith in selling such parcels of land and rescinding their contract thereby said deed, the spouses were in the U.S.; that due to their apprehension
justifying Spouses Garcia’s suspension of payment – NO. See doctrine for why regarding the authenticity of the document, they withheld payment of the last
Maceda Law is inapplicable. For second issue, the court held that contracts are installment which was due on Dec 31, 1993; that they tendered payment of
law between the parties, and they are bound by its stipulations. It is clear from the the unpaid balance sometime in July 1995 after Angel Abedila ratified the
above-quoted provisions that the parties intended their agreement to be a Contract sale made in favor of the defendant but the defendant refused to accept
to Sell: Dela Cruz retains ownership of the subject lands and does not have the payment for no justifiable reason.
obligation to execute a Deed of Absolute Sale until petitioners' payment of the 23. In her answer, defendant denied the allegation the the deed was spurious and
full purchase price. The validity of the stipulation in the contract providing for argued that the spouses Garcia failed to pay in full on its due date despite
automatic rescission upon non-payment cannot be doubted. It is in the repeated demands; that the contract to sell contains a proviso that failure
nature of an agreement granting a party the right to rescind a contract unilaterally plaintiffs to pay the purchase price in full shall cause the rescission of the
in case of breach without need of going to court. contract and forfeiture of ½ of the total amount paid to defendant; that a
DOCTRINE . Where buyers under a contract to sell offers to pay the last notarized letter stating the intended rescission of the contract to sell and
forfeiture was sent to the spouses at their last known address but it was stipulated date; and (3) Dela Cruz did not want to accept petitioners'
returned with a notation “insufficient address”. offer of payment and did not want to execute a document of transfer in
24. Intervenor Diogenes Bartolome filed a complaint in intervention alleging that petitioners' favor
the Contract to Sell between the Spouses and the defendants was rescinded 18. Contracts are law between the parties, and they are bound by its stipulations.
and became ineffective due to unwarranted failure of the Spouses to pay the It is clear from the above-quoted provisions that the parties intended their
unpaid balance on the stipulated date; that he became interested in the subject agreement to be a Contract to Sell: Dela Cruz retains ownership of the subject
parcels due to their clean titles and that he purchased the same from defendant lands and does not have the obligation to execute a Deed of Absolute Sale
by virtue of an Absolute deed of sale executed on Sept 23, 1995 for until petitioners' payment of the full purchase price.
P7,793,000. 19. Payment of the price is a positive suspensive condition, failure of which is
25. RTC ruled that Dela Cruz’s rescission was not valid. Trial court applied RA not a breach but an event that prevents the obligation of the vendor to convey
6552 (Maceda Law) and stated that dela Cruz was not allowed to unilaterally title from becoming effective. Strictly speaking, there can be no rescission or
cancel the contract to sell. It found that the spouses Garcia are justified in resolution of an obligation that is still non-existent due to the non-
withholding the payment of the balance due to the alleged spurious sale happening of the suspensive condition.
between Angel Abedila and defendant Emerlita dela Cruz. Moreover, 20. Dela Cruz is thus not obliged to execute a Deed of Absolute Sale in
intervenor Bartolome is not a purchaser in good faith because he was aware petitioners' favor because of petitioners' failure to make full payment on the
of the spouses Garcia’s interest in the subject parcels of land. RTC ordered stipulated date.|||
Emerlita dela Cruz to accept the balance of the purchase price within 10 days 21. In Pangilinan v CA, the Court ruled that: Article 1592 of the New Civil Code,
after judgement, plus P400,000 in moral damages and P100,000 as exemplary requiring demand by suit or by notarial act in case the vendor of realty wants
damages, and P100,000 as atty’s fees. to rescind does not apply to a contract to sell but only to contract of sale. In
26. CA reversed trial court’s decision. Dela Cruz’s obligation under the contract contracts to sell, where ownership is retained by the seller and is not to pass
to sell did not arise due to Spouses Garcia’s undue failure to pay in full the until the full payment, such payment, as we said, is a positive suspensive
agreed purchase price on the stipulated date. Moreover, judicial action for the condition, the failure of which is not a breach, casual or serious, but simply
rescission of a contract is not necessary where the contract provides that it an event that prevented the obligation of the vendor to convey title from
may be revoked and cancelled for violation of any of its terms and acquiring binding force. To argue that there was only a casual breach is to
conditions. Hence, this petition. proceed from the assumption that the contract is one of absolute sale, where
non-payment is a resolutory condition, which is not the case. The applicable
ISSUES: provision of law in instant case is Article 1191 of the New Civil Code which
4. WoN CA failed to consider the RA 6552 (Maceda Law) - NO makes it available to the injured party alternative remedies such as rescission
5. WoN CA failed to consider that Dela Cruz could not pass title over the 3 or fulfillment of the contract with damages in either case if the obligor does
parcels at the time she entered into a contract to sell as her purported not comply with what is incumbent upon him.
ownership was tainted with fraud, thereby justifying Spouses Garcia’s 22. The rationale for the foregoing is that in contracts providing for automatic
suspension of payment - NO revocation, judicial intervention is necessary not for purposes of obtaining a
6. WoN CA erred when it failed consider Dela Cruz’s rescission was done in judicial declaration rescinding a contract already deemed rescinded by
bad faith and malice on account of a second sale with Bartolome for a much virtue of an agreement providing for rescission even without judicial
bigger amount- NO intervention, but in order to determine whether or not the rescission was
7. WoN the CA erred when it failed to declare Bartolome as NOT an innocent proper.
purchaser for value despite the presence of evidence of bad faith- NO 23. The validity of the stipulation in the contract providing for automatic
rescission upon non-payment cannot be doubted. It is in the nature of an
RULING: Petition DENIED. CA Decision affirmed in toto. agreement granting a party the right to rescind a contract unilaterally in
case of breach without need of going to court.
RATIO: 24. Petitioners justify the delay in payment by stating that they had notice that
17. Petition has no merit. Both parties admit the following: (1) the contract Dela Cruz is not the owner of the subject land, and that they took pains to
between petitioners and Dela Cruz was a contract to sell; (2) petitioners failed rectify the alleged defect in Dela Cruz's title. Be that as it may, Angel
to pay in full the agreed purchase price of the subject property on the Abelida's (Abelida) affidavit confirming the sale to Dela Cruz only serves to
strengthen Dela Cruz's claim that she is the absolute owner of the subject
lands at the time the Contract to Sell between herself and petitioners was
executed. Dela Cruz did not conceal from petitioners that the titles to said
parcels still remained under Abelida's name, and the Contract to Sell
even provided that spouses Garcia should shoulder the attendant
expenses for the transfer of ownership from Abelida to Dela Cruz.
25. RTC erred in applying the Maceda Law as it applies to contracts of sale of
real estate on installment payments, including condomium apartments but
excluding industrial lots, commercial buildings, and sales to tenants. The
subject lands do not comprise of residential real estate within the
contemplation of the Maceda Law. Also, where buyers under a contract to
sell offers to pay the last installment a year and a half after the stipulated date,
that was beyond the sixty-day grace period under Section 4 of the Maceda
Law. The buyers cannot use the second sentence of Section 4 of the Maceda
Law against the sellers’ alleged failure to give an effective notice of
cancellation or demand for rescission
26. It is undeniable that petitioners failed to pay the balance of the purchase price
on the stipulated date of the Contract to Sell. Thus, Dela Cruz is within her
rights to sell the subject lands to Bartolome. Neither Dela Cruz nor Bartolome
can be said to be in bad faith.|||

PERTINENT PORTIONS OF THE CONTRACT (for reference):

“It is hereby agreed and covenanted that possession shall be retained by the VENDOR
until a Deed of Absolute Sale shall be executed by her in favor of the Vendees.
Violation of this provision shall authorize/empower the VENDOR [to] demolish any
construction/improvement without need of judicial action or court order.
That upon and after the full payment of the balance, a Deed of Absolute Sale shall be
executed by the Vendor in favor of the Vendees.
That the duplicate original of the owner's copy of the Transfer
Certificate of Title of the above subject parcels of land shall remain in the
possession of the Vendor until the execution of the Deed of Absolute Sale”
01 ADELFA v. CA (IYA) Dominador, while the western portion was allocated to Rosario and Salud.
January 25, 1995 | Regalado, J. | Contract of Sale v Contract to Sell 4. Adelfa expressed interest in buying the western portion of the property from
Rosario and Salud. Accordingly an “Exclusive Option to Purchase” was
executed. The following are the terms:
PETITIONER: Adelfa Properties, Inc.
a. Purchase price would be 2,856,150.00
RESPONDENTS: Court of Appeals, Rosario Jimenez-Castañeda and Salud
b. Sum of 50,000 received is option money and shall be credited as
Jimenez
partial payment of the purchase price.
c. Default on the part of Adelfa will result to the cancellation of the
SUMMARY: Adelfa was to purchase a portion of a parcel of land from Rosario
option and 50% of the option money will be forfeited. The
and Salud Jimenez. They executed an Exclusive Option to Purchase. Because of
remaining 50% will be returned.
a civil case, seeking the annulment of the sale of property instituted by nephews
d. All expenses will be born by the vendors.
and heirs of the Jimenezes, Adelfa suspended the payment of the purchase price.
5. Considering that the owner’s copy of the certificate title issued to Salud had
Upon dismissal of the civil case, Adelfa did not may any tender of payment or
been lost, a petition for the re-issuance of a new owner’s copy was filed. Atty.
consignation, it merely sent notice to Rosario and Salud that it wishes to exercise
Bernardo acted as counsel of Salud who received and kept in possession the
their option to purchase.
new certificate issued which it later gave to Adelfa.
6. Before payment was made, Adelfa received a copy of a complaint filed by
Rosario and Salud already sold the parcel to a third party and wish to recover
the nephews and nieces of Rosario and Salud against Jose and Dominador
from Adelfa the certificate title it previously gave them. The SC in deciding the
and Adelfa for the annulment of the deed of sale of the eastern portion.
case had to identify the nature of the contract between the parties. It ruled that
7. As a consequence, Adelfa informed Salud and Rosario that it would hold
Adelfa actually made an option to buy in favor of Rosario and Salud. Upon
payment of the full purchase price and suggested that the case first be settled.
acceptance of Rosario and Salud of such option, what was perfected was a
8. Salud refused to heed the suggestion of Adelfa and attributed the suspension
contract to sell and not a contract of sale. Because it is a contract to sell and no
of payment of the purchase price to “lack of word of honor”
full payment was made by Adelfa, ownership of the property was still with
9. Adelfa caused to be annotated on the title of the lot its option contract with
Rosario and Salud. They could therefore validly sell the property to a third party.
Rosario and Salud and its contract of sale with Jose and Dominador.
10. Rosario and Salud asked Atty. Bernardo to inform Adelfa that they were
cancelling the transaction.
DOCTRINE: In a contract of sale, title to the property passes to buyer upon the
11. RTC of Makati dismissed the complaint of the nephews on February 23,
delivery of the thing sold; in a contract to sell, ownership is, by agreement,
1990.
reserved in the seller and is not to pass to buyer until full payment of purchase
12. Rosario and Salud executed a Deed of Conditional Sale in favor of a certain
price. Otherwise stated, in a contract of sale, seller loses ownership over the
Emylene Chua for the western potion for P3M+, P1.5M was paid to them on
property and cannot recover it until and unless the contract is resolved or
the execution with the balance to be pain on the transfer of title.
rescinded, whereas in a contract to sell, title is retained by the seller until full
13. Atty. Bernardo informed Rosario and Salud that in view of the dismissal of
payment of the price. In the latter contract, payment of the price is a positive
the case, Adelfa was willing to pay the purchase price and he requested that
suspensive condition, failure of which is not a breach but an event that prevents
the corresponding deed of absolute sale be executed. This was ignored.
the obligation of the vendor to convey title from becoming effective.
14. Rosaro and Salud wrote a letter to Adelfa with the check of P25,000
representing the 50% refund. They also requested the return of the duplicate
FACTS: copy of the certificate of title.
1. Jose and Dominador Jimenez were the registered co-owners of a parcel of 15. Adelfa faield to surrender the certificate which led to the filing in the RTC
land consisting of 17,710 square meters. for the annulment of contract with damages, praying that het exclusive option
2. On July 28, 1988, Jose and Dominador sold their share consisting of half of to purchase be declared null and void.
the said parcel (eastern poriton) to Adelfa Properties pursuant to a “Kasulatan
sa Bilihan ng Lupa. ISSUE/s:
3. A Confirmatory Extrajudicial Partition Agreement was executed by the 1. WON the contract is rescinded – YES
Jimenezes where the eastern portion of the lot was adjudicated to Jose and 2. WON the “Exclusive Option to Purchase” executed by Adelfa and Rosario
and Salud is an option contract – NO. The Court’s treated it as a contract to iv. It should be noted that under the law and jurisprudence, a
sell. contract which contains this kind of stipulation is
considered a contract to sell.
RULING: WHEREFORE, on the foregoing modificatory premises, and considering b. It has NOT been shown that there was delivery of the property,
that the same result has been reached by respondent Court of Appeals with respect to actual or constructive, made to Adelfa. Neither did Adelfa take
the relief awarded to private respondents by the court a quo which we nd to be correct, actual, physical possession of the property at any given time.
its assailed judgment in CA-G.R. CV No. 34767 is hereby AFFIRMED. 3. The controverted document should legally be considered as a perfected
contact to sell.
RATIO: 4. The important task in contract interpretation is always the ascertainment of
1. The Supreme Court treated the alleged option contract as a contract to sell. the intention of the contracting parties and that task is discharged by looking
CONTRACT OF SALE CONTRACT TO SELL to the words they used to project that intention I their contract.
Title passes to the vendee upon the By agreement, the ownership is 5. Moreover, judging from the subsequent acts of the parties, it is undeniable
delivery of the thing sold reserved in the vendor and is not that the intention of the parties was to enter into a contract to sell.
to pass until the full payment of 6. The title of the contract does not necessarily determine its true nature.
the price Additional discussion by the SC:
Vendor has lost and cannot recover Title is retained by the vendor 7. An option, as used in the law on sales, is a continuing offer or contract by
ownership until and unless the until the full payment of the which the owner stipulates with another that the latter shall have the right to
contract is resolved or rescinded price, such payment being a buy the property at a fixed price within a certain time or under certain terms
positive suspensive condition and and conditions.
failure of which is not a breach a. An option is not of itself a purchase, but merely secures the privilege
bun an event that prevents the to buy.
obligation of the vendor to convey b. It is not a sale of property but a sale of the right to purchase.
title from being effective OPTION CONTRACT CONTRACT OF SALE
Unaccepted offer Fixes definitely the relative rights and
2. There are two features which convince the Court that the parties never obligations of both parties at the time
intended to transfer ownership to Adelfa except upon full payment of the of its execution
purchase price: States the terms and conditions on Offer and the acceptance are
a. Exclusive option to purchase, although it provided for automatic which the owner is willing to sell his concurrent since the minds of the
rescission of the contract and partial forfeiture of the amount already land, if the holder elects to accept parties meet in the terms of the
paid in case of default, DOES NOT mention that Adelfa is obliged them within the time limited agreement
to return possession or ownership of the property as a consequence If he does so elect, he must give notice
of the property as a consequence of non-payment. to the other party and the accepted
i. With the absence of such stipulation, although there is a offer becomes a valid and biding
provision on the remedies available to the parties in case of contract
breach, it may be legally be inferred that the parties never If an acceptance is not made within
intended to transfer ownership to the petitioner prior the time fixed, the owner is no longer
to completion of payment of the purchase price. bound by his offer
ii. In effect, there was an implied agreement that ownership
shall not pass to the purchaser until he had fully paid the 8. The test in determining whether a contract is a "contract of sale or
price. purchase" or a mere "option" is whether or not the agreement could be
iii. Article 1478 of the Civil Code does not require that such a specifically enforced.
stipulation be expressly made. Consequently, an implied 9. There is no doubt that the obligation of petitioner to pay the purchase price
stipulation to that effect is considered valid and, therefore, is specific, definite and certain, and consequently binding and enforceable.
binding and enforceable between the parties. 10. In interpreting the contract of this case, it can be shown that there is indeed
a concurrence of Adelfa’s offer to buy the property and the acceptance of
such by Rosario and Salud.
11. It must be stressed that there already existed a perfected contract between
the parties at the time the alleged counter-offer was made.
12. Thus, any new offer by a party becomes binding only when it is accepted by
the other.
13. In the case of Rosario and Salud, they actually refused to concur in said
offer of Adelfa, by reason of which the original terms of the contract
continued to be enforceable.
a. At any rate, the same cannot be considered a counter-offer for the
simple reason that petitioner's sole purpose was to settle the civil
case in order that it could already comply with its obligation.
14. Rosario and Salud may no longer be compelled to sell and deliver the subject
property for two reasons
a. Petitioners failure to duly effect the consignation of the purchase
price after the disturbance had ceased
b. fact that the contract to sell had been validly rescinded.
15. The alleged option money of P50,000.00 was actually earnest money which
was intended to form part of the purchase price. The amount of P50,000.00
was not distinct from the cause or consideration for the sale of the property,
but was itself a part thereof.
a. It is a statutory rule that whenever earnest money is given in a
contract of sale, it shall be considered as part of the price and as
proof of the perfection of the contract.
b. It constitutes an advance payment and must, therefore, be deducted
from the total price. Also, earnest money is given by the buyer to
the seller to bind the bargain.
16. Issue not related to topic: Whether there is a valid suspension of payment?
17. Adelfa already knew of the dismissal of the civil case by the nephews in
February. However, it was only in April did they write to Rosario and Salud
expressing their willingness to pay the balance of the purchase price.
18. It was merely a notice to pay. There was no proper tender of payment nor
consignation in this case as required by law.
19. The intention to pay without accompanying payment is not considered a valid
tender of payment.
20. A mere tender of payment is not sufficient to compel Rosario and Salud to
deliver the property and execute the deed of absolute sale.
21. It is consignation which is essential in order to extinguish petitioner’s
obligation to pay the balance of the purchase price.
22. Obligation to pay the balance arose and resumed after Adelfa received notice
of the dismissal of the case. It however failed to seasonably make the payment
and failed to do so up to present time, or even to deposit the money with the
trial court.
02 Coronel v CA (Lij) the said property exclusively to the prospective buyer upon fulfillment of the
October 7, 1996 | Melo, J. | Contract to sell condition agreed upon

PETITIONER: Romulo Coronel, etal FACTS:


RESPONDENTS: Court of Appeals, Concepcion Alcaraz, and Ramona Alcaraz 170. The Coronels executed a document entitled “Recipt of Down Payment” in
SUMMARY: favor of Ramona. There was a down payment of 50,000 and a remaining
The coronels executed a document entitled “Recipt of Down Payment” of 50,000 balance of 1,190,000 pesos for a house and lot inherited from their father.
in favor of Ramona, binding themselves to transfer the ownership of the land in 171. The conditions: payment of 50,000 as downpayment; the Coronels will
their name from their deceased father, after which Alcaraz would pay the balance transfer in their name of the title of the property registered in their dead
of 1,190,000. Feb 6, property was transferred to the Coronels name. Feb 18, fathers name upon receipt of the 50k; Upon transfer in their names, Coronels
Coronels sold to a third party and hence Ramona and her mother filed an action would execute a deed of absolute sale in favor of Ramona and the latter will
for specific performance. pay the remainder of the balance.
The heart of the controversy is the determination of what kind of contract was 172. On the same day, Jan 15, 1985, the mom of Ramona (concepcion) paid the
constituted between the parties. The issue was whether it was a contract TO sell downpayment.
or a contract of sale. Ramona and her mom argue that there was a perfected 173. On Feb 18, 1985, the coronels would sell the same property to Catalina for
contract of sale while the Coronels argue that it was simply a contract to sell. The 1,580,000. Because of this Coronels canceled and rescinded the contract with
court explained the differences between a contract to sale and that of a contract Ramona by depositing the downpayment paid in the bank in trust for
TO sell. A contract TO sell is a bilateral contract whereby the prospective seller, Ramona.
while expressly reserving the ownership of the subject property despite delivery 174. Concepcion et al filed a complaint for specific performance against the
therof to the prospective buyer, binds himself to sell the said property exclusively Coronels and cause the annotation of a notice of lis pendens at the back of
to the prospective buyer upon fulfillment of the condition agreed upon. It is the property title. Catalina also caused an annotation of a notice of adverse
different from a sale whereby the parties have already consented to the transfer of claim. Later on the coronels would execute a Deed of absolute sale in favor
the property in exchange of a price. In a contract TO sell, they only agree that of Catalina and would transfer the property in her name.
upon fulfillment of the condition that the seller will sell. A contract TO sell is also 175. RTC would render a decision in favor of Ramona and her mom (plaintiffs).
not the same as a conditional contract of sale. The fulfillment of the condition in MR of said decision was denied. CA fully agreed with the trial court.
a contract TO sell will not perfect the sale. If the property was previously 176. The heart of the controversy is the precise determination of the legal
delivered, the ownership thereof will not automatically transfer. While the significance of the document entitled “Recipet of Down Payment”
fulfillment of the condition of a contract of sale will complete the sale and if the 177. The respondents (Ramona) argue that the document embodies a perfected
property was previously delivered, the ownership would automatically transfer. contract of sale which they seek to enforce by means of an action for specific
The importance in distinguishing is especially significant in cases involving a sale performance. The opposing party argue that the document only signified a
to a third party. The fulfillment of the condition in a contract TO sell will not make mere executory contract to sell, subject to certain suspensive conditions
a buyer one in bad faith since there was still no sale. In a contract of sale, the
fulfillment would mean that there was completed salehence theparty who actually ISSUE:
or constructively knows such and still buys the party is a buyer in bad faith. In this 55. Was the “receipt of downpayment” a contract to sell or a contract of sale?
case, the court took the plain language of the receipt of down payment into Contract of sale
consideration and concluded that the nature of the contract was a contract of sale.
It was clear that the sellers made no reservation as to the ownership of the property. RULING: The petition is denied, judgement appealed from is affirmed.(in favor of
They intended to transfer it but they couldn’t do so since the property was still in Ramona and Mom)
their dead father’s name. They already agreed to sell and undertook to transfer the
title to their names and then immediately execute a deed of absolute sale RATIO:
DOCTRINE: A contract TO sell can be defined as a bilateral contract whereby 147. A contract of Sale is perfect by mere consent. The essential elements are
the prospective seller, while expressly reserving the ownership of the subject Consent as to transfer ownership in exchange for the price; determinate
property despite delivery thereof to the propspective buyer, binds himself to sell subject matter and a price certain in money or its equivalent
148. A contract TO sell may not be considered as a Contract of sale because the deed of absolute sale.
first element is lacking. The prospective seller does not yet agree or consent 157. It was clear that they didn’t just promise to sell under certain conditions. They
to transfer ownership of the property until the happening of an event. It is were completely willing to transfer full ownership of the subject house and
subject to a suspensive condition, the non-fulfillment of which prevents the lot to the buyerif the documents were then in order. It just so happened that
obligation to sell from arising and thus, ownership still remains with eh the title was still in the name of their father.
prospective seller. 158. The plain language of the Receipt of Down payment shows that the parties
149. A contract TO sell can be defined as a bilateral contract whereby the agreed to a conditional contract of sale, consummation of which is subject
prospective seller, while expressly reserving the ownership of the subject only to the successful transfer of the certificate of title from the father to the
property despite delivery thereof to the prospective buyer, binds himself to heirs.
sell the said property exclusively to the prospective buyer upon fulfillment of 159. The condition was fulfilled on Feb 6, 1985, hence the contract of sale hence
the condition agreed upon. became obligatory. Additionally court said that it is significant to note that in
150. A contract TO sell is not a conditional contract of sale. In a conditional the petitioners petition they admitted that it was a sale subject to a suspensive
contract of sale, the fulfillment of the suspensive condition will prefect the condition
sale. If the property was previously delivered, ownership automatically
transfers. In a contract TO sell, the fulfillment of the suspensive condition
will not automatically transfer the ownership.
151. It is important to distinguish between a contract to sell and a conditional
contract of sale especially in cases where the subject is sold to a third party
such as in this case. In a contract TO sell, there is no previous sale so that a
person buying such property despite the fulfillment of the suspensive
condition cannot be deemed a buyer in bad faith and the prospective buyer
cannot seek the relief of reconveyance of the property.
152. In a conditional contract of sale, upon fulfillment of the suspensive condition,
the sale becomes absolute and this will affect the owner’s title. If there was
previous delivery, the ownership automatically transfers to the buyer. A
second buyer who had actual or constructive knowledge of such defect, or
was charged with the obligation to discover such cannot be registrant in good
faith.
153. What is the real nature of the contract? It is canon that words in a contract are
given their natural and ordinary meaning unless a technical meaning was
intended.
154. Taking the Receipt of Down payment, in its entirety, it becomes apparent that
the petitioners intended to transfer title to the buyer, but it could not be made
because it was still in the name of the father. They could not effect the transfer
although the buyer was willing and able to pay the price.
155. The agreement could not have been a contract to sell because the sellers
herein made no express reservation of ownership or title to the property.
Furthermore, the circumstances which prevented them from entering into an
absolute contract of sale pertained to the sellers themselves and not the full
payment of the purchase price.
156. The sellers in this case also did not merely promise to sell the property to
private respondent upon the fulfillment of a suspensive condition. On the
contrary, they already agreed to sell the subject property, they undertook to
have the title changed to their names and to immediately execute a written
03 PNB v. CA (Perral) contract to sell, by agreement, ownership is reserved in the vendor and is not to pass
Sept, 26, 1996 | Hermosisima Jr, J. | Contract to sell v cont of sale until the full payment of the price. In a contract of sale, the vendor has lost and cannot
recover ownership until and unless the contract is resolved or rescinded, whereas in
PETITIONER: Philippine National Bank a contract to sell, title is retained by the vendor until the full payment of the price,
RESPONDENTS: Court of Appeals, Lapaz Kaw Ngo such payment being a positive suspensive condition, failure of which is not a breach
but an event that prevented the obligation of the vendor to convey title from
SUMMARY: Lapaz made a formal offer to purchase a parcel of land owned by PNB. becoming effective.
Due to this formal offer, PNB and Lapaz executed a letter of agreement (1st) in which Note: guys sorry mahaba talaga yung case and ma detail.
PNB gave several conditions for the completion of their agreement. Unfortunately, FACTS:
the sale between PNB and Lapaz under the 1st letter of agreement was never 1. July, 14 1983: Private respondent Lapaz made a formal offer to purchase the
materialized because of Lapaz’ failure to pay the amount stipulated in their parcel of land consisting of 1,250 sqm. Located at the corner of Carlos
agreement. However, due to Lapaz’ request to revive their agreement, PNB and Palanca and Helios St Manila, owned and registered in the name of PNB
Lapaz executed a second letter of agreement, again, with terms and conditions, 2. PNB advised Lapaz of its approval of the latter’s offer to purchase the subject
wherein Lapaz was not agreeable to term no. 6. This prompted PNB to canel their property subject to the terms and conditions stated in its official
agreement and forfeit Lapaz’ deposit money. Lapaz filed an action before the court communication with Lapaz (see last page for the conditions)
for specific performance. RTC and CA ruled in favor of Lapaz, ;PNB filed a petition 3. Lapaz signified her comformity to the above letter-agreement by affixing her
before the SC and latter reversed the lower courts’ ruling, holding that, there was signature thereon.
never a contract of sale between PNB and Lapaz but rather there was only a contract 4. On of the conditions in the agreement was to clear the subject property of its
to sell, thus there was no transfer of title yet especially that the conditions set forth then occupants; thus. Lapaz undertook the ejectment of the squatters at her
were not complied with. Also, SC held that the 1st and 2nd letter of agreements are own expense. Subsequently, Lapaz, citing the then prevailing credit squeeze,
different and the effects of which should not be tacked with each other. Lastly, SC requested for adjustment of payment proposals.
said that the deposit money was not an earnest money that can be construed as a proof 5. Feb 28, 1984: PNB wrote Lapaz reminding her of her failure to remit the
of a perfection of a sale. amount of P978,860 as embodied in its letter dated Dec 6, 1983..and of her
refusal to send her letter of conformity to the letter agreement.Lapaz was
DOCTRINE: A distinction must be made between a contract of sale in which title liewise advised to remit her cash payment of the full price amounting to
passes to the buyer upon delivery of the thing sold and a contract to sell,where by P5,378,902;otherwise, the subject property shall be sold to other interested
agreement the ownership is reserved in the seller and is not to pass until the full party and her deposit forfeited.Lapaz request for adjustment of payments was
payment of the purchase price is made. In the first case, non-payment of the price is also denied.
a negative resolutory condition; in the second case, full payment is a positive 6. March 1, 1984: In a letter, Lapaz, due to a significant reduction in the land
suspensive condition. Being contraries, their effect in law cannot be identical. In the area being purchased, requested for the reduction of the selling price from
first case, the vendor has lost and cannot recover the ownership of the land sold until P5,394,300 to P5,135,599 on cash basis or a total of P6,066,706 on
and unless the contract of sale is itself resolved and set aside. In the second case, installment.
however, the title remains in the vendor if the vendee does not comply with the 7. PNB favorably acted on Lapaz’s request. However, when PNB received no
condition precedent of making payment at the time specified in the contract further payment, PNB notified the Lapaz by telegram that it was giving her a
last chance to pay the balance or the required doenpayment of P563,341and
In contract to sell ownership is retained by the seller and is not to pass to the buyer failure to do so will result to the cancellation of the sale and forfeiture of her
until full payment of the price or the fulfillment of some other conditions either of 100K deposit.
which is a future and uncertain event the non-happening of which is not a breach, 8. The sale between Lapaz and PNB never materialized because of Lapaz’
casual or serious, but simply an event that prevents the obligation of the vendor to failure to pay the agreed amount; sale was cancelled and deposit money
convey title from acquiring binding force. forfeited and later on PNB leased the property to a certain Morse Rivera.
9. Lapaz requested for a refund of her deposit in the total of P550K with the
Distinction bet contract to sell and contract of sale is important for in a contract of further requested that since the Bank was willing to refund to her her deposit
sale, the title passes to the vendee upon the delivery of the thing sold, whereas in a provided that the P100,000.00 is forfeited in favor of the Bank, the amount
of P100,000.00 be reduced to P30,000.00 because her deposit of P550,000.00 c.Won PNB’s condition no.6 is material and should be agreed upon at
had, after, all, already accumulated to a sizable amount of interest and, inception of contract.
besides there was a delay in the approval of the contract or proposal. d. Won the area of agreement in PNB’s counter offer/acceptance
10. Lapaz further intimated that her request for refund shall be subject to the extends to condtion no.6 together with all other conditions PNB
release of the fund within 1 week from receipt thereof; otherwise, she would specified.
insist on purchasing the property subject to mutually agreed grace period. 10. WoN Lapaz’ refusal to pay P827,199 downpayment is not a valid basis for
11. Subsequently PNB released in favor of Lapaz P550K as refund of deposit PNB’s cancellation of the approved sale.
made on the offer to purchase the subject property.
12. Lapaz then wrote a letter to then Pres. Marcos, requesting for the lifting of RULING: SC GRANTED the petition for REVIEW. Decision of the CA and decision
the directive suspending the sale of the subject property. This letter was of RTC is hereby reversed and set aside. Lapaz’ complaint for specific performance
transmitted to the Pres. Of PNB. PNB then advised Lapaz of the approval of and damages is DISMISSED
her request for revival of the previously approved offer to purchase the
subject property subject to terms and conditions (see last page 2nd terms and RATIO: In general, petition is meritorious.
conditions) 29. THERE ARE 2 SEPARATE TRANSACTIONS IN THE INSTANT CASE,
13. In a letter, Lapaz informed PNB management that the terms and conditions THE FIRST HAVING BEEN UNCONDITIONALLY CANCELLED,
were acceptable to her ecept for condition no. 6. She requested for the EFFECTS THEREOF CANNOT BE DEEMED APPLICABLE TO THE 2ND
deletion of the said condition because she had already defrayed the expenses TRANSACTION:
for the ejectmenet of the previous occupants of the premises in compliance - Even private respondent admits in her pleadings that she failed to remit
with the condition in the original approved offer to purchase and that the the required downpayment under the 1st letter agreement, thus it was just
present occupants are not the squatters but rather the lessees of PNB. proper for PNB to cancel the agreement to protect its interest.
14. This request for modification was not acceptable to PNB, thus, she was given - Appellate court erred in holding that the conformity to the 2nd letter of
up to July 10, 1986 to submit, duly signed, the letter-conforme dated May 14, agreement would be superfluous being merely a revival of the first
1986 and to remit the initial amount of P827,119.83 to comply with the agreement which was approved by the bank and the terms and conditions
approved terms and conditions; otherwise, the approved sale will be thereof accepted by Lapaz. SC said that this holding is in complete
cancelled and her deposit of P200,000.00 forfeited disregard of the status of the first letter agreement as being non- existent
15. Lapaz thru counsel informed PNB that she is willing to pay the amount stated and totally inefficaious as a result of its cancellation.
above as the balance of the 20% downpayment of the purchased price as soon - It was wrong for the appellate court to perceived the 2 letter agreements
as the property was cleared of its present tenants. Unfortunately, the bank did to be a single transaction such that it justified Lapaz’ non-compliance
not grant her any extension to pay the said amount, cancelled on Jan 30,1987 with condition no. 6 in the 2nd letter agreement by invoking her earlier
the approved sale for being stale and unimplemented and forfeited her deposit compliance with the same comdition in the 1st letter agreement.
of 200K. o When the first letter agreement was cancelled by petitioner, and
16. Lapaz tried to ask PNB for reconsideration but to no avail. This cancellation private respondent agreed to that cancellation upon receiving
prompted Lapaz to file an action for Specific Performance.RTC ruled in favor P550K as refund of her aggregate deposit, all the effects of that
of Lapaz and this was affirmed by CA, thus PNB filed a petition for review agreement were terminated. Upon mutual assent to that
before SC. (reasoning of RTC and CA are reflected in the RATIO) cancellation, the agreement so cancelled thereafter no longer
exist.
ISSUE/s: o Thus, the compliance by private respondent with the terms and
9. WoN there was a perfected contract between PNB and Lapaz Despite their conditions of that first agreement serve the purposes of that
disagreement on the term and condition no. 6. agreement and cannot be made to serve the purposes of the 2nd
a. Won PNB’s acceptance of Lapaz’ offer to revive her purchase of letter agreement. Appellate court erred in tacking the 2
subject lot was subject to certain substantive condition agreements and commingled their effects; it incorrectly
b. Won PNB’s acceptance of Lapaz’ was in fact a couner-offer whivh considered petitioner’s successful ejectmenet of the subject
she rejected by her insistence that PNB delete condition no. 6 propert in 1983 occupants under the 1st letter agreement to be
sufficient compliance under the 2nd letter agreement.
- PNB’s acceptance of Lapaz’ offer was a qualified acceptance which is a vendor if the vendee does not comply with the condition
counter offer. Refusing to bind herself to bear the expenses for a 2nd precedent of making payment at the time specified in the
ejectment suit involving the subject property, Lapaz’ in effect rejected contract.”
petitioner’s counter offer of at the least accepted the same subject to the o In contract to sell ownership is retained by the seller and is
deletion of condition no. 6 not to pass to the buyer until full payment of the price or the
17. BOTH LETTER- AGREEMENT ARE IN THE NATURE OF fulfillment of some other conditions either of which is a
CONTRACTS TO SELL; NON COMPLIANCE WOTH THE future and uncertain event the non-happening of which is
SUSPENSIVE CONDITIONS SET FORTH THEREIN PREVENTS THE not a breach, casual or serious, but simply an event that
OBLIGATION OF THE VENDORTO CONVEY TITLE FROM HAVING prevents the obligation of the vendor to convey title from
OBLIGATORY FORCE: acquiring binding force.
- The fundamental flaw in the reasoning of both the trial court and - In the case at bench, Lapaz does not dispute the fact that under identical
appellate court is their admitted premise that both letter agreements are proviso in the 2 letter of agreemens, her obligation was to deposit an
contracts of sale the perfection of which are proven by the earnest moey initial amount (100K 1st agreement; 200k 2nd agreement) and then
tendered to and accepted by PNB in the form of 100K and 200K under subsequently to deposit an additional amount of 20% of the purchase
the first and sec letter agreements, respectively. price (P978,860 1st agreement; P827,119 2nd agreement). Under both
o SC said that the letter agreements shows that they are letter-agreements, the consequences of private respondent's failure to
contracts to sell and not contract of sale. remit the additional deposit, are unequivocal and plainly
§ Contact to sell is akin to a conditional sale where the comprehensible: deposit shall be forfeited and for this purpose, the Bank
efficacy or obligatory force of the vendor’s obligation can sell the property to other interested parties due to your failure to
to trqnsfer title is subordinated to the happening of a consummate the previously-approved sale.
future and uncertain event so that if the suspensive - The right reserved by PNB to cancel the agreement to sell upon
condition does not take place, the parties would stand failure of petitioner to remit the additional deposit and to
as if the conditional obligation had never existed. The consequently open the subject property anew to purchase offers, is
suspensive condition is commonly full payment of the in the nature of a stipulation reserving title in the vendor until
purchase price fullpayment of the purchase price or giving the vendor the right to
§ If it were not full payment of the purchase price upon unilaterally rescind the contract the moment the vendee fails to pay
which depends the passing of title from the vendor to within fixed period.
the vendee, it may be some other condition or - SC found that the intention of the private parties to make the sale
conditions that have been stipulated and must be dependent on petitioner's compliance with certain other conditions, is
fulfilled before the contract is converted from a undeniable and plainly evident in the letter-agreements. Identical
contract to sell or at the most an executory sale into an provisions therein relating to petitioner's waiver of her right to warranty
executed one. against eviction and her accountability for the expenses for the ejectment
- Difference bet. Contract to sell and contract of sale: proceedings, are not so called "standard" provisions that are more of a
o “A distinction must be made between a contract of sale in which rhetorical device than conditions genuinely meant by the parties to be
title passes to the buyer upon delivery of the thing sold and a suspensive conditions in the legal sense.
contract to sell,where by agreement the ownership is reserved - Also, letter agreements of PNB and Lapaz are not deed of sal, thus,
in the seller and is not to pass until the full payment of the no title having been passed from PNB to Lapaz.
purchase price is made. In the first case, non-payment of the o Distinction bet contract to sell and contract of sale is
price is a negative resolutory condition; in the second case, full important for in a contract of sale, the title passes to the
payment is a positive suspensive condition. Being contraries, vendee upon the delivery of the thing sold, whereas in a
their effect in law cannot be identical. In the first case, the contract to sell, by agreement, ownership is reserved in the
vendor has lost and cannot recover the ownership of the land vendor and is not to pass until the full payment of the price.
sold until and unless the contract of sale is itself resolved and In a contract of sale, the vendor has lost and cannot recover
set aside. In the second case, however, the title remains in the ownership until and unless the contract is resolved or
rescinded, whereas in a contract to sell, title is retained by (2nd TERMS AND CONDITIONS)
1. That the selling price shall be P5,135,599.17 (P200,000.00) already deposited . . .
the vendor until the full payment of the price, such payment
2. A.That upon your failure to pay the additional deposit of P827,119.83 upon receipt of advice of
being a positive suspensive condition, failure of which is not approval, your P200,000.00 deposit shall be forfeited and for this purpose, the Bank can sell the
a breach but an event that prevented the obligation of the property to other interested parties;
vendor to convey title from becoming effective. xxx xxx xxx
3. That your previous deposit of P100,000.00 which was forfeited by the Bank due to your failure
o In this case it is clear that there was no transfer of title. not even to consummate the previously- approved sale, shall not be considered as part of the purchase
mention of such a transfer in the future, considering that all the price;
parties were aware of the occupancy of the subject property by 4. That the bank sells only whatever rights interests and participation it may have in the property
third persons. This circumstance all the more reinforces our and you are charged with full knowledge of the nature and extent of said rights, interest and
participation and waives your right to warranty against eviction;
finding that the transaction contemplated under the letter-
Xxx xxx xxx
agreements was a contract to sell or a conditional sale which 6. That the property shall be cleared of its present tenants but all expenses to be incurred in
absolutely depends, for its efficacy, upon the happening of the connection woth the ejectment proceedings shall be for your account;
conditions specified in the said letter-agreements. 7. That the sale shall be subject to all terms and conditions covering sale of similar acquired real
estate properties;
- Deposit given was not an earnest money; not a proof of the 8. That the sale shall also be subject to all terms and conditions that the legal dept may impose to
perfection of the sale. And even if it is, it is just a disputable protect the interest of the Bank.
presumption that can be rebutted, which in this case, was rebutted
by sufficient evidence.
o The letter of agreementsthemselves are the evidence of an
intention on the part of herein private parties to enter into
negotiations leading to a contract of sale that is mutually
acceptable as to absolutely bind them to the performance of
their obligations thereunder.
o Letter of agreements are replete with substantial condition
precedents, acceptance of which on the part of private
respondent must first be made in order for petitioners to proceed
to the next step in the negotiations. The initial deposits under
the two letter-agreements, therefore, should rather be construed,
not strictly as earnest money, but as part of the consideration
for petitioner's promise to reserve the subject property for
private respondent.

(1st TERMS AND CONDITIONS) . . . your offer to purchase the Bank-acquired property . . . was approved
by the Bank, subject to the following terms and conditions:
1. That the selling price shall be P5,394,300.00 (P100,000.00 already deposited) . . .
2. (a) That upon your failure to pay the additional deposit of P978,860.00 upon receipt of advice
accepting your offer, your P100,000.00 initial deposit shall be forfeited and for this purpose the
Bank shall be authorized to sell the property to other interested parties.
xxx xxx xxx
3. That the Bank sells only whatever rights, interests and participation it may have in the property
and you are charged with full knowledge of the nature and extent of said rights, interests and
participation and waives [sic] your right to warranty against eviction.
xxx xxx xxx
4. That the property shall be cleared of its present tenants/occupants but all expenses to be incurred
in connection with the ejectment proceedings shall be for your account.
5. That the sale shall be subject to such other terms and conditions that the Legal Department may
impose to protect the interest of the Bank.
04 BABASA v. CA (PAT) Tabangao agreed on a total purchase price of Php 2,121,920 and stipulated
May 21, 1998 | Bellosillo, J. | Effect of condition imposed on the performance on the following:
a. Php 300k upon signing of the contract.
Petitioner: Spouses Vivencio Babasa and Elena Babasa b. Php 1,821,920 upon presentation of the Babasas of TCTs in their
Respondent: Court of Appeals, Tabangao Realty, Inc., and Shell Gas Philippines, Inc.
name, free from all liens and encumbrances, and delivery of
registrable documents of sale in favor of Tabangao within 20 months
SUMMARY: Babasas and Tabangao entered into a Conditional Sale of Registered Lands.
from the signing of the contract.
They were already able to agree with the price and the subject matter but there was a
condition imposed that Tabangao would give the rest of the payment when the Babasas
c. Retained balance of the purchase price would earn interest at 17%
are able to deliver the clean TCTs within 20 months from signing and in addition, Tabangao per annum or Php 20,648.43/mo payable to the Babasas until
was also granted the absolute and unconditional right to take immediate possession of December 1982.
the lots and make improvements. They were able to substantially comply with their parts d. Tabangao would have the absolute and unconditional right to take
but 2 days before the 20-month period, the Babasas asked Tabangao if they could have an immediate possession of the lots as well as introduce any
indefinite extension of the period since the case to clean the titles was still pending, but improvements.
Tabangao refused. So the Babasas executed a notarized unilateral rescission and also 18. Tabangao leased the lots to Shell and Shell constructed a Liquefied Petroleum
demanded that Shell vacate the lots (Tabangao leased the property to Shell). Tabangao Gas Terminal Project; Tabangao was the real estate arm of Shell.
filed for specific performance and damages in the RTC and Shell intervened. The Babasas 19. Babasas and Tabangao substantially complied with the terms of the contract:
moved for a dismissal saying that since the 20-month period has already expired, the a. Tabangao paid Php 300k first installment to Babasas.
contract is null and void. While the case was pending, the Babasas put several structures b. Babasas delivered actual possession of the lots to them.
so that Shell and Tabangao could not move certain things on the property. The RTC ruled c. Tabangao also paid Php 379,625 to the tenants of the lots as disturbance
in favor of Tabangao and Shell saying that when the 20-month period expired, it did not compensation and also Php 334,700 to the owners of the house in the
mean that the obligations would be extinguished. They also said that the rescission was area.
void. On appeal to the CA, the CA affirmed the decision of the RTC. They also said that the d. Tabangao also paid the monthly interest for the 20-month period.
contention of the Babasas that the contract is a lease and not a sale does not hold water. e. Babasas filed a civil case to transfer the TCTs of the lots in their name.
Hence, this petition. The issues in this case are: (1) whether or not the contract is one of 20. But 2 days before the 20-month period expires, Babasas asked Tabangao
sale or lease and whether or not the non-happening of the condition extinguished the for an indefinite extension so that they could deliver clean title and also
efficacy of the sale. As to the first issue, the SC said that basing from the contract itself,
asked Tabangao to continue paying the monthly interest since the civil case
the words used and basing from the intent, the contract is indeed one of sale and not of a
that the Babasas filed to get the titles back have not been resolved with
lease. As to the second issue, the SC said that the contract is an absolute sale even if it was
finality, but Tabangao refused.
denominated as conditional. A deed of sale is absolute although denominated as
conditional if no stipulations on rescission and no proviso regarding reserving title are
21. In retailiation, Babasas executed a notarized unilateral rescission, and
present. And the expiration of the 20-month period was a condition imposed not on the Tabangao replied that the Babasas were the ones who did not comply with
perfection but upon the performance of the obligation. And a condition imposed on the their contractual obligation to deliver the clean titles within the 20-month
performance merely grants to the buyer the right to rescind or waive the condition and period, hence they have no right to rescind the contract.
enforce performance on the seller. 22. Babasas insisted on the rescission and demanded that Shell vacate the lots.
23. Tabango filed an action for specific performance with damages in the RTC
DOCTRINE: When buyer’s obligation to pay the purchase price was made subject to the to compel the Babasas to fulfill their obligation to deliver clean titles over the
condition that seller first delivers clean TCTs over the parcel bought, such condition is properties. Tabangao alleged that Babasas were in the position to give them
imposed merely on the performance of the obligation, as distinguished from a condition the clean titles considering that the civil case has ended and the court ruled in
imposed on the perfection. The non-happening of the condition merely granted the buyer their favor.
FACTS:
the right to rescind or waive the condition and enforce performance on the seller. 24. Babasas moved for a dismissal, saying that the contract with Tabangao
16. A Conditional Sale of Registered Lands was executed between Vivencio became null and void with the expiration of the 20-month period; Shell
and Elena Babasa (Babasas) as vendors and Tabangao Realty, Inc. intervened.
(Tabangao) as vendee over 3 parcels of land in Batangas City. 25. While the case was pending, the Babasas put several structures over the lots
17. The certificates of title over these lots were in the names of 3rd persons who in the case, they also claimed 12 heads of cattle which belonged to Shell, and
had executed deeds of reconveyance in favor of the Babasas, so Babasas and threatened to collect levy from all buyers.
26. RTC ruled in favor of Tabangao and Shell saying that the 20-month period a. A threat of eminent domain proceedings cannot be considered as the kind
was never meant to be its term in that when it expires, the obligations would of injury that could vitiate consent.
be extinguished. What the expiration of the period means is that it merely
gave rise to the right of Tabangao to rescind or demand that Babasas comply On whether the non-happening of the condition extinguished the efficacy of the sale
with the obligation; the rescission by Babasas was void; in addition, Babasas 1. Babasas: Ownership over the 3 lots was never transferred to Tabangao since
were also ordered to deliver the clean TCTs. the contract was rendered lifeless when the 20-month period expired without
27. Babasas appealed to the CA but the CA affirmed the decision of the RTC. them being able to deliver clean TCTs.
They also rejected the claim that the contract was of lease and not sale. CA 2. SC: Although the contract says Conditional Sale of Registered Lands, the
said that it is an absolute sale though denominated conditional. contract is an absolute sale.
28. Hence, this petition. a. Aside from the terms, there is no proviso reserving title in the Babasas
until full payment of the purchase.
ISSUE/S: b. There was also no stipulation giving them the right to unilaterally rescind
3. WON the contract was a contract of lease and not sale. – NO. It is a SALE. the contract in case of non-payment.
4. On whether the non-happening of the condition extinguished the efficacy of c. A deed of sale is absolute although denominated as conditional
the sale. – NO. absent such stipulations.
d. Ownership of the thing sold passes to the vendee upon constructive or
HELD: WHEREFORE, the petition is DENIED. The appealed decision of the CA actual delivery and in this case, the ownership has been passed to
affirming that of the RTC is AFFIRMED. Tabangao both constructive and actual delivery.
i. Constructive since the contract was without any reservation of title.
RATIO: ii. Actual since Tabangao took unconditional possession and leased it
On whether the contract is a lease or a sale to Shell.
1. Babasas: The contract was a lease. 3. Babasas: The contract lost efficacy when the 20-month period expired, so
2. SC: CA is correct in saying that the contract was a sale and not a lease. Tabangao cannot seek specific performance.
a. The contract in itself shows that it is actuall a sale. 4. SC: In Romero v. CA and Lim v. CA they distinguished between a condition
i. The whereas clause says that the parties: desire and mutually agreed imposed on the perfection of the contract and a condition imposed
on the sale and purchase of the 3 parcels of land. merely on the performance.
ii. They agreed that Babasas were vendors and Tabangao was the a. Condition imposed on the perfection – Failure to comply with the
vendee. condition results in the failure of the contract.
iii. There was a purchase price for the lots stated. b. Condition imposed merely on performance – Failure to comply
iv. Tabangao was granted absolute and unconditional right to take merely gives the other party to option to either refuse the sale or to
immediate possession of the premises while Babasas warranted that waive the condition.
possession forever. c. In this case, there is already a perfected contract of absolute sale since
v. Tabangao would shoulder the capital gains tax. they agreed on the determinate subject matter (3 lots), the price certain
vi. Babasas would executed a Final Deed of Absolute Sale in favor of and without any condition or reservation of title.
Tabangao for the issuance of TCTs. d. The obligation of Tabangao to pay the full amount of the purchase price
b. Just because the word ownership was not mentioned doesn’t mean that was made subject to the condition that Babasas would deliver the clean
it is already a lease. TCTs. If Babasas had done so, they would get the balance.
c. In addition, Babasas did not object to the terms in the contract, e. Since the condition was imposed merely on the performance, the failure
considering that they were assisted by Atty. E.M. Carreon. of the Babasas to deliver the clean titles merely gives Tabangao the
3. Babasas: We never intended to sell our ancestral lots; we were forced because option to refuse to proceed with the sale or to waive the condition
Tabangao gave a threat of expropriation. imposed. Hence, the act of Babasas in unilaterally rescinding is
4. SC: Contracts are valid even though one of the parties entered into it against unwarranted.
his own wish and desire or even against his better judgment. f. When buyer’s obligation to pay the purchase price was made subject to
the condition that seller first delivers clean TCTs over the parcel bought,
such condition is imposed merely on the performance of the obligation,
as distinguished from a condition imposed on the perfection. The non-
happening of the condition merely granted the buyer the right to rescind
or waive the condition and enforce performance on the seller.
05 VALDEZ v. CA (Mica) FACTS: (Bold yung lots kasi nakakalito)
Sept. 24, 2004 | Callejo, Sr., J. | Essential Stipulations to constitute a contract to sell 1. Carlos Valdez, Sr. (Carlos Sr.) and Josefina de Leon Valdez were the owners
of a parcel of land with an area of 24,725sqm located in the commercial
PETITIONER: Josefina L. Valdez and Carlos L. Valdez, Jr. district of Isulan, Sultan Kudarat
RESPONDENTS: Court of Appeals and Jose Lagon 2. When Carlos Sr. died intestate on Mar. 26, 1966, he was survived by Josefina
and their children, including Carlos Jr. (a practicing lawyer)
SUMMARY: Carlos Sr. and Josefina de leon valdez owned a parcel of land in 3. Dec. 28, 1978- Josefina caused the subdivision survey of the property into 8
Sultan Kudarat. When Carlos Sr. died, he was survived by Josefina and their lots, all fronting the national road.
children, including Carlos Jr., who is a practicing lawyer. Josefina divided the lot 4. To enhance the value of the property, she decided to sell a portion to Jose
into 8. Through an SPA authorizing Carlos Jr., she sold a portion (4,094sqm) to Lagon, a successful businessman in Sultan Kudarat who owned a
Lagon. Part of the consideration of the transaction was that Lagon would cause construction firm as well as real estate and business enterprises (the Lagon
the transfer of the Rural Bank there and would build a commercial building. These Enterprises and the Rural Bank of Isulan). He’s also of the clients of her son
were not part of the deed of absolute sale that was executed. In the deed it was 5. May 1, 1979- Josefina executed a SPA authorizing Carlos Jr. to sell a protion
said that the land was to be sold for P80,000 cash and it was paid already, but in of Lot No. 3-C and Lot No. 3-D to Lagon (the subject lots: 4,094 sqm, with
reality, it was purchased for P163,760 including the account of Carlos Jr to Lagon. a frontage of 64.3sqm)
Lagon has not remitted the whole 163k. Lagon issued checks. One check worth 6. Part of the consideration of the transaction was the condition that Lagon cause
8k was encashed but the other check worth 81k was returned cause Lagon’s wife the transfer of the Rural Bank of Isulan to the subject property and construct
paid 20k so there’s still a balance. Lagon acknowledged the earlier consideration a commercial building beside the bank
in an affidavit. If he fails to comply, the sale would be declared null and void. 7. May 9 – they executed a Deed of Absolute Sale. However, the condition
Lagon failed to comply and to pay the balance so the petitioners refused to give imposed by Josefina was not incorporated in the deed. What was appended
the title. First demand letter said if they still won’t comply within 10 days, then it was the SPA. It was indicated in the deed that the property was to be sold for
would be rescinded. But then Carlos wrote again proposing for the reduction of P80,000 cash and the Lagon had already paid to Carlos Jr
the property instead. Lagon did not reply. In the meantime, a geodetic engr. 8. In reality, however, Lagon purchased the 4,094 sqm property at P40.00/sqm
subdivided Lot 3-C into 3 (lot 3-C-1, lot 3-C-2, lot 3-C-3). Lagon paid for his or for the amount of P163,760, inclusive of Carlos Jr.’s personal account to
services. Lot 3-C-2 was sold to PCIB and lot 3-D was sold to Engr. Delfin. Carlos Lagon in the amount of P73,760. Lagon has not remitted the whole amount
Jr. reminded Lagon about the considerations. He requested for a conference 9. April 21, 1981 – Lagon gave to Carlos Jr. PCIB Check in the amount of
through his counsel but it Carlos Jr was not available on the said date. Josefina P8,1996 dated April 21, 1981, and PCIB Check postdated June 15, 1981 in
mortgaged Lot 3-C-3 to DBIP as security for her P150,000 loan. Lot 3-C-1 was the amount of P81,880 (total: P90,076 in full payment of the property, after
sold to Carlos Jr. Lagon filed a complaint for specific performance and damages. deducting the accoung of Carlos Jr)
Josefina and Carlos said that Lagon does not have a cause of action because he 10. Josefina acknowledged the checks and signed through Carlos Jr.
failed to comply with the conditions of the sale and his undertaking. TC and CA 11. Carlos Jr. was able to encash the P8,196 but returned the other check to
ruled in favor of Lagon saying that it was a deed of absolute sale and not a contract Lagon’s wife, Nenita, after the latter paid him P20,000 thereby leaving a
to sell. WoN it was a contract to sell. No. Court held that it was a contract of sale balance of P61,880
because Josefina did not reserve the title over the property until full payment and 12. Carlos, Jr. prepared an Affidavit dated April 27, 1981 signed by Lagon, where
she did not have the right to unilaterally rescind the contract in case of non- the latter undertook to transfer the Rural Bank of Isulan to the property and
payment (doctrine in this case). The contract was however, unenforceable but construct a commercial building thereon, to be in full operation within a
Josefina ratified it by receiving the partial payments. period of five (5) years from the date of the deed of absolute sale, or until
May 9, 1984
DOCTRINE: A contract is one of sale, absent any stipulation therein (a) reserving 13. If Lagon failed to do so, the deed of absolute sale shall be declared null and
title over the property to the vendee until full payment of the purchase price, and void without need of demand therefor.
(b) giving the vendor the right to unilaterally rescind the contract in case of non- 14. Lagon also made it clear in the said affidavit that the consideration of the said
payment Deed of Absolute Sale was not only the P80,000.00 purchase price, but also
that the subject property be commercialized.
15. However, Lagon failed to start the construction of a commercial building and
to transfer the rural bank thereon; he, likewise, failed to pay the balance of SPA, executed by Josefina in favor of Carlos, Jr., the deed of absolute sale
the purchase price amounting to P61,880.00. over Lot No. 3 in favor of Lagon and the deed of absolute sale executed by
16. Consequently, Josefina and Carlos, Jr. refused to deliver to Lagon a torrens Josefina in favor of PCIB, among others. Lagon, through his counsel, Atty.
title over the purchased property. Rex Rico, reiterated his request for a conference on May 23, 1988. However,
17. On September 4, 1981, Carlos, Jr. demanded the payment of P61,800.00 Carlos, Jr. was not available on the said date. cSEaTH
within ten days from notice thereof, otherwise, the sale would be considered 30. Josefina executed a real estate mortgage over Lot No. 3-C-3 covered by TCT
rescinded. Still, Lagon failed to pay or even respond to the letter. No. 18818 in favor of the Development Bank of the Philippines (DBP) as
18. Carlos, Jr. again wrote Lagon on September 25, 1981, and this time proposed security for a loan of P150,000.00.
the reduction of the area of the property subject of the sale to correspond to 31. Josefina executed a deed of absolute sale over Lot No. 3-C-1 in favor of her
the payment so far made by Lagon in the total amount of P90,676.00. There son, Carlos, Jr.. In the meantime, in 1984, Carlos, Jr. had an edifice
was no response from Lagon. constructed on the property where he put up his law office, a nipa hut behind
19. In the meantime, the TCT was cancelled on Oct. 9, 1981 by 8 titles under the PCIB branch, the Ivy Pharmacy, the "K House" and the headquarters of
Josefina Valdez (cause pinadivide nga ni Josefina diba so naging: 3-A to 3- the Nationalista Party
H na talaga) 32. Sept. 24, 1990 – Lagon filed a complaint against Josefina and Carlos Jr., in
20. Alhambra, a geodetic engr., conducted a subdivision survey of lot no. 3-C, his capacity as attorney-in-fact, for specific performance and damages with a
covered by TCT no. 16438 into 3 subdivision lots with the ff. areas: lot no. prayer for a temporary restraining order and writ of preliminary injunction
3-C-1 (4499sqm); lot no. 3-C-2 (350sqm); and lot no. 3-C-3 (17,355sqm). a. Wants the injunction to be permanent
He also submitted this to the Bureau of Lands on Dec. 12, 1983. Lagon paid b. Deliver the TCT over the remaining area of that parcel of land sold
for the professional services to him
21. Porfirio L. Cubar, the Bank Manager of the Philippine Commercial Industrial c. Pay opportunity loss, attorney’s fees, litigation expenses and
Bank (PCIB) in Isulan talked offered to buy, in behalf of the PCIB, Lot No. damages
3-C-2 for P100.00 per square meter. Carlos, Jr. agreed. Josefina executed a 33. Lagon contends that Josefina failed to deliver the title to the property
deed of absolute sale on May 8, 1984, over Lot No. 3-C-2 for P35,000.00 in purchased from her, as well as the possession thereof. Hence, he was not
favor of PCIB. certain of the boundaries of the property and could not secure a building
22. Carlos, Jr. later learned that Lagon had been saying that he was responsible permit for the construction of the Rural Bank and the commercial building
for the sale of Lot No. 3-C-2 to the PCIB, but PCIB informed Carlos, Jr. in a 34. Besides, Carlos, Jr. secured his permission for the construction of the PCIB
Letter dated that Lagon had nothing to do with the sale. commercial building on Lot No. 3-C-2 which was sold to him by Josefina,
23. the deed of extrajudicial settlement earlier executed by the heirs of Carlos and even agreed to the deduction of the purchase price thereof; hence, the
Valdez, Sr. was filed and registered in the Office of the Register of Deeds. balance was only P26,880.
(transferring their rights to Josefina) 35. Lagon demanded that the title to the property be turned over to him and the
24. Josefina sold Lot 3-D in favor of Engr. Rolendo Delfin occupants thereof be evicted therefrom so that he could comply with the
25. August 1987- a question ensued in connection with Lagon’s failure to pay the conditions of the sale for the construction of the commercial building and the
balance of the purchase price of the property, to cause the construction of a transfer of the Isulan Rural Bank.
commercial building and the transfer of the Rural Bank of Isulan to Lot No. 36. However, Carlos, Jr. dilly-dallied, saying that the heirs of Carlos, Sr. needed
3, as undertaken by him in his Affidavit time to execute the extrajudicial settlement of his estate, and thus failed to
26. As a reminder, Carlos Jr. gave Lagon with a machine copy of the affidavit deliver said title to him. Lagon averred that his consent to the construction by
27. Lagon’s counsel, Atty. Ernesto Catedral, wrote Carlos, Jr., pointing out that the PCIB of its branch on a portion of the property he had purchased from
he had earlier sought Lagon’s consent for the construction of the PCIB Josefina constituted substantial compliance of his undertaking under the deed
Branch in Lot No. 3. Catedral posited that by consenting to the sale of the of absolute sale and the affidavit he executed in favor of Josefina. He also
property to PCIB and the construction of its branch office, Lagon thereby alleged that he signed the affidavit prepared by Carlos, Jr. without reading
substantially complied with his undertaking under the deed of absolute sale. and understanding the same. He pointed out that although Lot No. 3 had
28. The lawyer asked Carlos, Jr. to set a conference to thresh out possibilities of already been sold to him by Josefina, she still sold Lot No. 3-C-3 to her son,
an amicable settlement of the matter. Carlos, Jr.; Lot No. 3-D to Engr. Rolendo Delfin; and mortgaged Lot No. 3-
29. Carlos, Jr. furnished Atty. Catedral with copies of documents, including a D to DBP which acquired title over the property.
37. Josefina and Carlos Jr. said that Lagon has no cause of action cause he failed of the conjugal nature of the property, since there is no showing that it was
to comply with the terms of the deed of absolute sale, his undertaking under acquired during the marriage of the Spouses Carlos Valdez, Sr. and Josefina
the affidavit, and the purchase price of the property in full. Carlos also denied L. Valdez.
securing Lagon’s consent for the PCIB Branch and agreeing to deduct P35k 4. The presumption under Article 160 of the New Civil Code, that property
from Lagon’s balance acquired during marriage is conjugal, does not apply where there is no
38. Lagon withdrew his petition for a writ of preliminary injunction, which was showing as to when the property alleged to be conjugal was acquired.
granted by the trial court 5. The presumption cannot prevail when the title is in the name of only one
39. TC: in favor of Lagon so Josefina and Carlos Jr. appealed. CA reversed the spouse and the rights of innocent third parties are involved.
decision based on the deed of absolute sale, the SPA, the affidavit. CA ruled 6. Moreover, when the property is registered in the name of only one spouse
that the parties had executed a contract to sell. CA reversed again (affiriming and there is no showing as to when the property was acquired by same spouse,
RTC) in the MR. It declared that it was a deed of absolute sale, that SPA was this is an indication that the property belongs exclusively to the said spouse.
not part of that deed, and that the affidavit was not part of the deed too 7. In this case, there is no evidence to indicate when the property was acquired
because it was a mere afterthought. Also, CA agreed with the inability of by petitioner Josefina. Thus, we agree with petitioner Josefina’s declaration
Lagon to know the boundaries because Josefina refused to deliver the title to in the deed of absolute sale she executed in favor of the respondent that she
the property. CA said that based on Art. 1186 of the NCC, the conditions of was the absolute and sole owner of the property.
the sale are deemed fulfilled. And the procedure for rescission was not 8. We are convinced that the declaration in the deed of extrajudicial settlement
followed of the estate of the late Carlos Valdez, Sr., that the property formed part of
40. Petitioners assert that it was a contract to sell and that the three documents his estate and that his children waived their rights and claims over the
formed integral parts containing the terms and conditions of one and the same property in favor of their mother, was done merely to facilitate the issuance
transaction. They emphasized theat Lagon knew this because he did not of a torrens title over the property in petitioner Josefina’s name with her
acquire a torrens title over the property nor took possession thereof after the marital status as widow.
deed and Lagon even failed to register the deed. Art. 1592 only applies to
contract of sale Josefina Valdez and Lagon entered into a contract of sale over the subject property
1. It is not disputed by the parties that Josefina executed a SPA in favor of
ISSUE/s: Carlos, Jr., as her attorney-in-fact, authorizing him to sell the subject
1. WoN the subject property is the exclusive property of Josefina de leon Valdez property, and that executed a deed of absolute sale over it
– Yes 2. She also acknowledged receipt of partial payments of the purchase price, the
2. WoN it was a contract to sell – no (it was a contract of sale) balance of the purchase price thus stood at P61,880.00 There is, likewise, no
dispute that Lagon signed the affidavit on April 27, 1981.
RULING: The Amended Decision of the CA is reversed and set aside. Lagon’s 3. The parties, however, differ on the real nature of their transaction and on
complaint is dismissed. Petitioners are directed to refund the amount of P101,880 whether the said affidavit formed an integral part of the deed of absolute sale
executed by petitioner Josefina in favor of the respondent.
RATIO: 4. The real nature of a contract may be determined from the express terms
The Subject Property is the Exclusive Property of Josefina of the written agreement and from the contemporaneous and subsequent
1. In the deed of absolute sale, Josefina declared that she was the absolute owner acts of the parties thereto.
of the property. However, in the deed of extrajudicial settlement of the estate 5. In the construction or interpretation of an instrument, the intention of the
of Carlos Sr. the property was declared as part of the estate of the deceased parties is primordial and is to be pursued. If the terms of a contract are clear
2. RTC and CA held that it was only after the execution of the deed of and leave no doubt upon the intention of the contracting parties, the literal
extrajudicial settlement that Josefina became the absolute owner of the meaning of its stipulations shall control. If the contract appears to be contrary
property. SC said both erred. to the evident intentions of the parties, the latter shall prevail over the former.
3. SC note that TCT No. T-19529 (T-1902) covering the property was issued on The denomination given by the parties in their contract is not conclusive of
August 18, 1967, during the marriage of the Spouses Carlos Valdez, Sr. and the nature of the contents.
Josefina, under the name "Josefina L. Valdez married to Carlos Valdez, Sr." 6. The agreement of the parties may be embodied in only one contract or in two
The issuance of the title in the name solely of one spouse is not determinative or more separate writings. In such event, the writings of the parties should be
read and interpreted together in such a way as to render their intention the property. This retroacted to the date executed the deed of sale
effective. 18. Court also said that the affidavit was not a mere afterthought. Lagon admitted
7. A sale is at once perfected when the seller obligates himself, for a price in his complaint that he undertook to construct the sid building and transfer
certain, to deliver and to transfer ownership of a specified thing or right to the Rural Bank. In his testimony, he also said that he was supposed to comply
the buyer. From the time the contract is perfected, the parties are bound from May 9, 1979 to 1984 (May 9 yung date ng absolute sale). Counsel of
not only to the fulfillment of what has been expressly stipulated but also Lagon also admitted the binding effect of the affidavit (it is an unequivocal
to all the consequences which, according to their nature, may be in undertaking)
keeping with good faith, usage and law. 19. The refusal of Lagon to pay the balance has no basis. As such, Josefina's
8. In a contract of sale, the title to the property passes to the vendee upon the refusal to deliver the torrens title over the subject property under the
constructive or actual delivery thereof, as provided for in Article 1477 of the respondent’s name was justified, precisely because of his refusal to comply
New Civil Code. with his obligation to pay the balance of the purchase price.
9. The vendor loses ownership over the property and cannot recover it until and 20. Had he paid the purchase price of the property, such failure on the part of
unless the contract is resolved or rescinded by a notarial deed or by judicial Josefina to deliver the torrens title to and under the name of the respondent
action as provided for in Article 1540 of the New Civil Code. would have warranted the suspension of the five-year period agreed upon for
10. A contract is one of sale, absent any stipulation therein reserving title over the construction of a fully operational commercial building, as well as the
the property to the vendee until full payment of the purchase price nor giving transfer of the aforesaid bank to the property. This is so because absent such
the vendor the right to unilaterally rescind the contract in case of non- torrens title under the name of the respondent, no building permit for the
payment. construction of the buildings could be secured.
11. In a contract of sale, the non-payment of the price is a resolutory 21. Thus, failure to comply with his undertaking is through his own fault
condition which extinguishes the transaction that, for a time, existed and 22. There was no need for petitioner Josefina to make a notarized demand to the
discharges the obligations created thereunder. respondent or file an action to rescind the deed of absolute sale to enable her
12. In a contract to sell, ownership is, by agreement, reserved in the vendor to recover the ownership of the property.
and is not to pass to the vendee until full payment of the purchase price. 23. This is so because they had agreed that upon Lagon’s failure to construct a
Such payment is a positive suspensive condition, failure of which is not a new and fully operational commercial building and to cause the transfer of
breach but an event that prevents the obligation of the vendor to convey the Rural Bank of Isulan to the property on or before May 9, 1984, the deed
title from becoming effective. of absolute sale would be deemed null and void without need of any demand
13. The deed specifically states that the property is sold and delivered to the from the petitioners. Such agreement is evidenced by the affidavit executed
respondent as vendee. Josefina did not reserve the ownership over the by the respondent himself on April 27, 1981.
property, as well as any right to unilaterally rescind the contract. 24. Court does not agree with the contention that Carlos, Jr., had agreed to the
14. There has been, by the execution of the deed, a constructive delivery of the sale of a portion of the property, the construction of the PCIB branch office
property to Lagon; hence, the latter acquired ownership over the same. thereon, and the crediting of the amount paid by the PCIB to the respondent’s
15. Upon payment of the purchase price, petitioner Josefina was obliged to account, and deducted from the balance of the purchase price. There’s no
deliver the torrens title over the property to and under the name of the evidence that Josefina knows of this and that she authorized Carlos Jr. to enter
respondent as the new owner and place him, as vendee, in actual possession this agreement
thereof; otherwise, the failure or inability to do so constitutes a breach of the 25. In sum, then, the respondent had no cause for specific performance against
contract sufficient to justify its rescission. the petitioners. However, the petitioners are obliged to refund to the
16. However, the deed is unenforceable as of the date of its execution because respondent the latter’s partial payments for the subject property.
under the SPA, Carlos Jr. was authorized to sell on cash basis only and 26. The petitioners failed to adduce sufficient evidence to prove their
Josefina required some conditions to be incorporated on the deed. So, Carlos counterclaims, and, as such, the counterclaims must forthwith be dismissed.
Jr. acted beyond the scope of his authority. He also falsely declared the
purchase price was P80k and that he already received the amount, when in
fact, it was sold for P163,760 and that Lagon has not yet paid
17. Thus, the effectivity would depend upon the ratification by Josefina. Court
said that she did when she received partial payments of the pruhase price of
06 De Leon v. Ong (Cristelle) FACTS:
February 2, 2010 | Corona, J. | Chapter 11 REMEDY OF RESCISSION IN SALES 1. Petitioner Raymundo S. de Leon sold three parcels of land with
OF IMMOVABLES: CONTRACT OF SALE VERSUS CONTRACT TO SELL improvements situated in Antipolo, Rizal to respondent Benita T. Ong. As
PETITIONER: RAYMUNDO S. DE LEON these properties were mortgaged to Real Savings and Loan Association,
RESPONDENTS: VELASCO NACHURA & PERALTA, JJ. BENITA T. ONG. Incorporated (RSLAI), petitioner and respondent executed a notarized deed
of absolute sale with assumption of mortgage. (March 10, 1993)
SUMMARY: On March 10, 1993, Raymundo S. De Leon (petitioner) sold 3 parcels 2. That for and in consideration of the sum of ONE MILLION ONE
of land to Benita T. Ong(respondent). The said properties were mortgaged to a HUNDRED THOUSAND PESOS (P1.1 million), Philippine currency, the
financial institution; Real Savings & Loan Association Inc. (RSLAI). The parties receipt whereof is hereby acknowledged from [RESPONDENT] to the entire
then executed a notarized deed of absolute sale with assumption of mortgage. As satisfaction of [PETITIONER], said [PETITIONER] does hereby sell,
indicated in the deed of mortgage, the parties stipulated that the petitioner (de Leon) transfer and convey in a manner absolute and irrevocable, unto said
shall execute a deed of assumption of mortgage in favor of Ong (respondent)after [RESPONDENT], his heirs and assigns that certain real estate together with
full payment of the P415,000. They also agreed that the respondent (Ong) shall the buildings and other improvements existing thereon, situated in [Barrio]
assume the mortgage. The respondent then subsequently gave petitioner P415,000 Mayamot, Antipolo, Rizal under the following terms and conditions:
as partial payment. On the other hand, de Leon handed the keys to Ong and de Leon a. That upon full payment of [respondent] of the amount of FOUR
wrote a letter to inform RSLAI that the mortgage will be assumed by Ong. HUNDRED FIFTEEN THOUSAND FIVE HUNDRED
Thereafter, the respondent took repairs and made improvements in the properties. (P415,000), [petitioner] shall execute and sign a deed of assumption
Subsequently, respondent learned that the same properties were sold to a certain of mortgage in favor of [respondent] without any further cost
Viloria after March 10, 1993 and changed the locks, rendering the keys given to her whatsoever;
useless. Respondent proceeded to RSLAI but she was informed that the mortgage b. That [respondent] shall assume payment of the outstanding loan of
has been fully paid and that the titles have been given to the said person. Respondent SIX HUNDRED EIGHTY-FOUR THOUSAND FIVE HUNDRED
then filed a complaint for specific performance and declaration of nullity of the PESOS (P684,500) with REAL SAVINGS AND LOAN,4 Cainta,
second sale and damages. The petitioner contended that respondent does not have a Rizal… (emphasis supplied)”
cause of action against him because the sale was subject to a condition which 3. Pursuant to this deed, respondent gave petitioner P415,500 as partial
requires the approval of RSLAI of the mortgage. Petitioner reiterated that they only payment. Petitioner, on the other hand, handed the keys to the properties and
entered into a contract to sell. The RTC dismissed the case. On appeal, the CA wrote a letter informing RSLAI of the sale and authorizing it to accept
upheld the sale to respondent and nullified the sale to Viloria. Petitioner moved for payment from respondent and release the certificates of title. Thereafter,
reconsideration to the SC. Issue: Whether the parties entered into a contract of respondent undertook repairs and made improvements on the properties.
sale or a contract to sell? In a contract of sale, the seller conveys ownership of the 4. Subsequently, respondent learned that petitioner again sold the same
property to the buyer upon the perfection of the contract. The non-payment of the properties to one Leona Viloria and changed the locks, rendering the keys he
price is a negative resolutory condition. Contract to sell is subject to a positive gave her useless. Respondent thus proceeded to RSLAI to inquire about the
suspensive condition. The buyer does not acquire ownership of the property until credit investigation. However, she was informed that petitioner had already
he fully pays the purchase price. In the present case, the deed executed by the parties paid the amount due and had taken back the certificates of title.
did not show that the owner intends to reserve ownership of the properties. The 5. Petitioner: since the transaction was subject to a condition (i.e., that RSLAI
terms and conditions affected only the manner of payment and not the immediate approve the assumption of mortgage), they only entered into a contract to sell.
transfer of ownership. It was clear that the owner intended a sale because he Inasmuch as respondent did apply for a loan from RSLAI, the condition did
unqualifiedly delivered and transferred ownership of the properties to the not arise.
respondent 6. Consequently, the sale was not perfected and he could freely dispose of the
properties.
DOCTRINE: Essential Stipulations to Constitute a Contract to Sell – A contract is 7. RTC: Because respondent was a licensed real estate broker she knew that the
one of sale, absent any stipulation therein (a) reserving title over the property to the validity of the sale was subject to a condition. The perfection of a contract of
vendee until full payment of the purchase price, and (b) giving the vendor the right sale depended on RSLAI’S approval of the assumption of mortgage. Since
to unilaterally rescind the contract in case of non-payment RSLAI did not allow respondent to assume petitioner’s obligation, the RTC
held that the sale was never perfected.
8. CA upheld the sale to respondent and nullified the sale to Viloria. c. The totality of petitioner’s acts clearly indicates that he had
unqualifiedly delivered and transferred ownership of the properties
ISSUES: to respondent. Clearly, it was a contract of sale the parties entered
1. Is the 1st sale a Contract of Absolute Sale or Contract to Sell? CONTRACT into.
OF SALE 5. Furthermore, even assuming arguendo that the agreement of the parties was
2. Is the 2nd sale: Void Sale Or Double Sale? DOUBLE SALE subject to the condition that RSLAI had to approve the assumption of
mortgage, the said condition was considered fulfilled as petitioner prevented
RULING: WHEREFORE, the July 22, 2005 decision and November 11, 2005 its fulfillment by paying his outstanding obligation and taking back the
resolution of the Court of Appeals in CA-G.R. CV No. 59748 are hereby AFFIRMED certificates of title without even notifying respondent. In this connection,
with MODIFICATION insofar as respondent Benita T. Ong is ordered to pay Article 1186 of the Civil Code provides: Article 1186. The condition shall be
petitioner Raymundo de Leon P684,500 representing the balance of the purchase price deemed fulfilled when the obligor voluntarily prevents its fulfillment.
as provided in their March 10, 1993 agreement. 6. Void Sale Or Double Sale? This case involves a double sale as the disputed
properties were sold validly on two separate occasions by the same seller to
RATIO: the two different buyers in good faith. Respondent is a purchaser in good
1. Contract of sale: seller conveys ownership of the property to the buyer upon faith.
the perfection of the contract. Should the buyer default in the payment of the 7. Respondent purchased the properties, knowing they were encumbered only
purchase price, the seller may either sue for the collection thereof or have the by the mortgage to RSLAI. According to her agreement with petitioner,
contract judicially resolved and set aside. The non-payment of the price is respondent had the obligation to assume the balance of petitioner’s
therefore a negative resolutory condition. outstanding obligation to RSLAI. Consequently, respondent informed
2. Contract to sell: subject to a positive suspensive condition. The buyer does RSLAI of the sale and of her assumption of petitioner’s obligation. However,
not acquire ownership of the property until he fully pays the purchase price. because petitioner surreptitiously paid his outstanding obligation and took
For this reason, if the buyer defaults in the payment thereof, the seller can back her certificates of title, petitioner himself rendered respondent’s
only sue for damages. obligation to assume petitioner’s indebtedness to RSLAI impossible to
3. There was absolute and irrevocable transfer: The deed executed by the perform.
parties stated that petitioner sold the properties to respondent "in a manner 8. Since respondent’s obligation to assume petitioner’s outstanding balance
absolute and irrevocable " for a sum of P1.1 million. Nothing in said with RSLAI became impossible without her fault, she was released from the
instrument implied that petitioner reserved ownership of the properties until said obligation. Moreover, because petitioner himself willfully prevented the
the full payment of the purchase price. On the contrary, the terms and condition vis-à-vis the payment of the remainder of the purchase price, the
conditions of the deed only affected the manner of payment, not the said condition is considered fulfilled pursuant to Article 1186 of the Civil
immediate transfer of ownership (upon the execution of the notarized Code. For purposes, therefore, of determining whether respondent was a
contract) from petitioner as seller to respondent as buyer. Otherwise stated, purchaser in good faith, she is deemed to have fully complied with the
the said terms and conditions pertained to the performance of the contract, condition of the payment of the remainder of the purchase price.
not the perfection thereof nor the transfer of ownership. 9. Respondent was not aware of any interest in or a claim on the properties other
4. There was delivery: than the mortgage to RSLAI which she undertook to assume. Moreover,
a. Settled is the rule that the seller is obliged to transfer title over the Viloria bought the properties from petitioner after the latter sold them to
properties and deliver the same to the buyer. In this regard, Article respondent. Respondent was therefore a purchaser in good faith. Hence, the
1498 of the Civil Code provides that, as a rule, the execution of a rules on double sale are applicable.
notarized deed of sale is equivalent to the delivery of a thing sold. 10. When neither buyer registered the sale of the properties with the registrar of
b. In this instance, petitioner executed a notarized deed of absolute sale deeds, the one who took prior possession of the properties shall be the lawful
in favor of respondent. Moreover, not only did petitioner turn over owner thereof. In this instance, petitioner delivered the properties to
the keys to the properties to respondent, he also authorized RSLAI respondent when he executed the notarized deed and handed over to
to receive payment from respondent and release his certificates of respondent the keys to the properties. For this reason, respondent took actual
title to her. possession and exercised control thereof by making repairs and
improvements thereon. Clearly, the sale was perfected and consummated on
March 10, 1993. Thus, respondent became the lawful owner of the properties.
Nonetheless, while the condition as to the payment of the balance of the
purchase price was deemed fulfilled, respondent’s obligation to pay it
subsisted. Otherwise, she would be unjustly enriched at the expense of
petitioner.
11. Therefore, respondent must pay petitioner P684,500, the amount stated in the
deed.
07 DIGNOS v CA (CLARK) 2. On November 25, 1965, the Dignos spouses sold the same land in favor of
February 29, 1988 | Bidin, J. | Deed of Absolute Sale Luciano Cabigas and Jovita L. De Cabigas (Cabigas spouses) who were then
US citizens for P35,000. A deed of absolute sale was executed by the Dignos
PETITIONER: Silvestre Dignos and Isabel Lumungsod spouses in favour of the Cabigas spouses and was registered in the Office of
RESPONDENTS: Hon. Court of Appeals and Atilano G. Jabil the Register of Deeds pursuant to provisions of Act No. 3344.
3. As the Dignos spouses refused to accept from Jabil the balance of the
SUMMARY: On June 7, 1965, Spouses Dignos sold their parcel of land in Opon, purchase price of the land, and as Jabil discovered the second sale made by
Lapu–Lapu to Antonio Jabil for the sum of P28,000 payable for two installments, Dignos spouses to Cabigas spouses, Jabil brought the present suit.
with an assumption of indebtedness with the First Insular Bank of Cebu in the sum 4. CFI of Cebu declared:
of P12,000 and the next installment of P4,000 to be paid in September 1965. On a. That the deed of sale executed by Isablea L. de Dignos (wife) in
November 1965, the spouses Dignos sold the same parcel of land for P35,000 to favor of Luciano Cabigas, a US citizen, is null and void ab initio
spouses Cabigas who were then US citizens, and executed in their favor an b. The deed of sale executed by Dignos spouses (in favor of Jabil) not
Absolute Deed of Sale duly registered in the Office of the Register of Deeds. Upon rescinded. Thus, Atilano G. Jabil is ordered to pay P16,000 to
discovery of the 2nd sale of the subject land, Jabil filed the case at bar in the CFI Dignos spouses upon the execution of Deed of Absolute Sale of Lot
of Cebu declared the 2nd sale to the spouses Cabigas null and void ab initio and No. 3453, Opon Cadastre
the 1st sale to Jabil not rescinded. The CFI of Cebu also ordered Jabil to pay the c. Jabil to reimbeurse Cabigas spouses a reasonable amount
remaining P16,000 to the spouses Dignos and to reimburse the spouses Cabigas a corresponding to expenses or costs of the hollow block fence
reasonable amount corresponding the expenses in the construction of hollow block constructed
fences in the said parcel of land. The spouses Dignos were also ordered to return d. Dignos spouses to return to Cabigas spouses the sum of P35,000
the P35,000 to the spouses Cabigas. CA affirmed. The spouses Dignos contested 5. CA affirmed the decision. MR was also filed but was denied for lack of merit.
that the contract between them and Jabil was merely a contract to sell and not a
deed of sale. The SC held that the contract between the parties is a contract of sale. ISSUES:
All the elements of a valid contract of sale are present in the document and that 1. WoN the subject contract is a deed of absolute sale or a contract to sell –
Spouses Dignos never notified Jabil by notarial act that they were rescinding the Deed of Absolute Sale
contract, and neither did they file a suit in court to rescind the sale. There is no 2. WoN there was a valid rescission – NO
showing that Jabil properly authorized a certain Cipriano Amistad to tell
petitioners that he was already waiving his rights to the land in question. RULING: WHEREFORE, the petition filed is hereby Dismissed for lack of merit and
the assailed decision of the Court of Appeals is Affirmed in toto.
DOCTRINE: A deed of sale is absolute in nature although dominated as a “Deed
of Conditional Sale” where nowhere in the contract in question is a proviso or RATIO:
stipulation to the effect that title to the property sold is reserved in the vendor until 1. The contract in question (Exhibit C) is a Deed of Sale, with the following
full payment of the purchase price, nor is there a stipulation giving the vendor the conditions:
right to unilaterally rescind the contract the moment the vendee fails to pay within a. That Atilano G..Jabilis to pay the amount of Twelve Thousand Pesos
a fixed period. P12,000.00) Phil. Philippine Currency as advance payment;
b. That Atilano G. Jabil is to assume the balance of Twelve Thousand
FACTS: Pesos (P12,000.00) Loan from the First Insular Bank of Cebu;
1. Dignos spouses were owners of a parcel of land (Lot No. 3453) of the c. That Atilano G. Jabil is to pay the said spouses the balance of Four.
cadastral survey of Opon, Lapu-Lapu City. On June 7, 1965, Dignos spouses Thousand Pesos (P4,000.00) on or before September 15,1965;
sold that land to Atilano Jabil for the sum of P28,000 payable in two d. That the said spouses agrees to defend the said Atilano G. Jabil from
instalments with an assumption of indebtedness with the First Insular Bank other claims on the said property;
of Cebu for P12,000 which was paid and acknowledged by the vendors in the e. That the spouses agrees to sign a final deed of absolute sale in favor
deed of sale executed in favor of Jabil. The next instalment in the sum of of Atilano G. Jabil over the above-mentioned property upon the
P4,000 will be paid on or before September 15, 196. payment of the balance of Four Thousand Pesos.
2. Petitioner’s Contentions: Dignos spouses reiterated their contention that the
Deed of Sale is a mere contract to sell and not an absolute sale; known as Jabil's Beach Resort in March, 1965; Mactan White Beach Resort
a. That the same is subject to two positive suspensive conditions, on January 15,1966 and Bevirlyn's Beach Resort on September 1, 1965.
namely (1) the payment of the balance of P4,000.00 on or before 7. SECOND ISSUE: Dignos spouses also claim that when they sold the land
September 15,1965 and (2) the immediate assumption of the to the Cabigas spouses, the contract of sale was already rescinded.
mortgage of P12,000.00 with the First Insular Bank of Cebu. 8. The contract of sale being absolute in nature is governed by Article 1592 of
b. That in said contract, title or ownership over the property was the Civil Code. It is undisputed that Dignos spouses never notified Jabil by
expressly reserved in the vendor, the Dignos spouses until the notarial act that they were rescinding the contract, and neither did they file a
suspensive condition of full and punctual payment of the balance of suit in court to rescind the sale. The most that they were able to show is a
the purchase price shall have been met. So that there is no actual sale letter of Cipriano Amistad who, claiming to be an emissary of Jabil, informed
until full payment is made. the Dignos spouses not to go to the house of Jabil because the latter had no
c. That there is absolutely nothing that indicates that the vendors money and further advised Dignos spouses to sell the land in litigation to
thereby sell, convey or transfer their ownership to the alleged another party.
vendee 9. There is no showing that Amistad was properly authorized by Jabil to make
d. Dignos spouses insist that it is a private instrument and the absence such extra-judicial rescission for the latter who, on the contrary, vigorously
of a formal deed of conveyance is a very strong indication that the denied having sent Amistad to tell Dignos spouses that he was already
parties did not intend “transfer of ownership and title but only a waiving his rights to the land in question. Under Article 1358 of the Civil
transfer after full payment.” Code, it is required that acts and contracts which have for their object the
e. Dignos spouses anchored their contention on the very terms and extinguishment of real rights over immovable property must appear in a
conditions of the contract, more particularly paragraph four and public document.
condition number five 10. Dignos spouses laid considerable emphasis on the fact that p Jabil had no
3. FIRST ISSUE: It has been held that a deed of sale is absolute in nature money on the stipulated date of payment on September 15,1965 and was able
although denominated as a “Deed of Conditional Sale” where nowhere to raise the necessary amount only by mid-October 1965. It has been ruled,
in the contract in question is a proviso or stipulation to the effect that however, that “where time is not of the essence of the agreement, a slight
title to the property sold is reserved in the vendor until full payment of delay on the part of one party in the performance of his obligation is not a
the purchase price, nor is there a stipulation giving the vendor the right sufficient ground for the rescission of the agreement.”
to unilaterally rescind the contract the moment the vendee fails to pay 11. Considering that Jabil has only a balance of P4,000.00 and was delayed in
within a fixed period. payment only for one month, equity and justice mandate as in the aforecited
4. In this case, there is no such stipulation in the contract reserving the title case that Jabil be given an additional period within which to complete
of the property on the vendors nor does it give them the right to payment of the purchase price.
unilaterally rescind the contract upon non-payment of the balance
thereof within a fixed period.
5. On the contrary, all the elements of a valid contract of sale under Article 1458
of the Civil Code, are present, such as: (1) consent or meeting of the minds;
(2) determinate subject matter; and (3) price certain in money or its
equivalent. In addition, Article 1477 of the same Code provides that “The
ownership of the thing sold shall be transferred to the vendee upon actual or
constructive delivery thereof.” In the absence of stipulation to the contrary,
the ownership of the thing sold passes to the vendee upon actual or
constructive delivery thereof.
6. While it may be conceded that there was no constructive delivery of the land
sold in the case at bar, as subject Deed of Sale is a private instrument, it is
beyond question that there was actual delivery thereof. The Dignos spouses
delivered the possession of the land in question to Jabil as early as March
27,1965 so that the latter constructed thereon Sally's Beach Resort also
08 UNIVERSITY OF THE PHILIPPINES vs. DE LOS ANGELES (Elach) law.
September 9, 1970 | Reyes JBL, J. | Remedies for breach/Rescission

PETITIONERS: University of the Philippines


RESPONDENTS: Walfrido De Los Angeles, In His Capacity as Judge of The
Court Of First Instance In Quezon City, et al.

SUMMARY: On November 2, 1960, UP and ALUMCO entered into a logging


agreement whereby the latter was granted exclusive authority to cut, collect and
remove timber from the Land Grant for a period starting from the date of
agreement to December 31, 1965, extendible for a period of 5 years by mutual
agreement. On December 8, 1964, ALUMCO incurred an unpaid account of
P219,362.94. Despite repeated demands, ALUMCO still failed to pay, so UP sent
a notice to rescind the logging agreement. On the other hand, ALUMCO executed
an instrument entitled “Acknowledgment of Debt and Proposed Manner of
Payments. It was approved by the president of UP, which stipulated the following:
3. In the event that the payments called for are not sufficient to liquidate the
foregoing indebtedness, the balance outstanding after the said payments have
been applied shall be paid by the debtor in full no later than June 30, 1965. 5. In
the event that the debtor fails to comply with any of its promises, the Debtor agrees
without reservation that Creditor shall have the right to consider the Logging
Agreement rescinded, without the necessity of any judicial suit… ALUMCO
continued its logging operations, but again incurred an unpaid account. On July
19,1965, UP informed ALUMCO that it had, as of that date, considered rescinded
and of no further legal effect the logging agreement, and that UP had already taken
steps to have another concessionaire take over the logging operation. ALUMCO
filed a petition to enjoin UP from conducting the bidding. The lower court ruled
in favor of ALUMCO, hence, this appeal. Issue in this case is W/N UP can treat
its contract with ALUMCO rescinded, and may disregard the same before any
judicial pronouncement to that effect? According to the SC, Yes. In the first place,
UP and ALUMCO had expressly stipulated that upon default by the debtor, UP
has the right and the power to consider the Logging Agreement of December 2, FACTS:
1960 as rescinded without the necessity of any judicial suit. As to such special 21. 3 orders of the CFI, are sought to be annulled in this petition for certiorari and
stipulation and in connection with Article 1191 of the Civil Code, the Supreme prohibition, filed by herein petitioner University of the Philippines against
Court, stated in Froilan vs. Pan Oriental Shipping Co: “There is nothing in the law the above-named respondent judge and the Associated Lumber
that prohibits the parties from entering into agreement that violation of the terms Manufacturing Company, Inc. (or ALUMCO).
of the contract would cause cancellation thereof, even without court intervention. 22. The first order, enjoined UP from awarding logging rights over its timber
In other words, it is not always necessary for the injured party to resort to court concession (or Land Grant), situated at the Lubayat areas in the provinces of
for rescission of the contract.” Laguna and Quezon; the second order adjudged UP in contempt of court, and
directed Sta. Clara Lumber Company, Inc. to refrain from exercising logging
DOCTRINE: Party who deems the contract violated may consider it resolved or rights or conducting logging operations on the concession; and the third order
rescinded, and act accordingly, without previous court action, but it proceeds at denied reconsideration of the order of contempt.
its own risk. For it is only the final judgment of the corresponding court that will 23. On November 2, 1960, UP and ALUMCO entered into a logging agreement
conclusively and finally settle whether the action taken was or was not correct in
whereby the latter was granted exclusive authority to cut, collect and remove second petition for preliminary injunction; and, on 25 February 1966,
timber from the Land Grant for a period starting from the date of agreement respondent judge issued the first of the questioned orders, enjoining UP from
to December 31, 1965, extendible for a period of 5 years by mutual awarding logging rights over the concession to any other party.
agreement. 31. That UP received the order of 25 February 1966 after it had concluded its
24. On December 8, 1964, ALUMCO incurred an unpaid account of contract with Sta. Clara Lumber Company, Inc., and said company had
P219,362.94. Despite repeated demands, ALUMCO still failed to pay, so UP started logging operations.
sent a notice to rescind the logging agreement. On the other hand, ALUMCO 32. That, on motion dated 12 April 1966 by ALUMCO and one Jose Rico, the
executed an instrument entitled “Acknowledgment of Debt and Proposed court, in an order dated 14 January 1967, declared petitioner UP in contempt
Manner of Payments. of court and, in the same order, directed Sta. Clara Lumber Company, Inc., to
25. It was approved by the president of UP, which stipulated the following: refrain from exercising logging rights or conducting logging operations in the
a. In the event that the payments called for in Nos. 1 and 2 of this concession.
paragraph are not sufficient to liquidate the foregoing indebtedness 33. The UP moved for reconsideration of the aforesaid order, but the motion was
of the DEBTOR in favor of the CREDITOR, the balance denied on 12 December 1967.
outstanding after the said payments have been applied shall be paid 34. Except that it denied knowledge of the purpose of the Land Grant, which
by the DEBTOR in full no later than June 30, 1965; purpose, anyway, is embodied in Act 3608 and, therefore, conclusively
b. In the event that the DEBTOR fails to comply with any of its known, respondent ALUMCO did not deny the foregoing allegations in the
promises or undertakings in this document, the DEBTOR agrees petition. In its answer, respondent corrected itself by stating that the period
without reservation that the CREDITOR shall have the right and the of the logging agreement is five (5) years - not seven (7) years, as it had
power to consider the Logging Agreement dated December 2, 1960 alleged in its second amended answer to the complaint in Civil Case No.
as rescinded without the necessity of any judicial suit, and the 9435.
CREDITOR shall be entitled as a matter of right to Fifty Thousand 35. It reiterated, however, its defenses in the court below, which maybe boiled
Pesos (P50,000.00) by way of and for liquidated damages; down to: blaming its former general manager, Cesar Guy, in not turning over
26. ALUMCO continued its logging operations, but again incurred an unpaid management of ALUMCO, thereby rendering it unable to pay the sum of
account, for the period from 9 December 1964 to 15 July 1965, in the amount P219,382.94; that it failed to pursue the manner of payments, as stipulated in
of P61,133.74, in addition to the indebtedness that it had previously the "Acknowledgment of Debt and Proposed Manner of Payments" because
acknowledged. the logs that it had cut turned out to be rotten and could not be sold to Sta.
27. That on 19 July 1965, petitioner UP informed respondent ALUMCO that it Clara Lumber Company, Inc., under its contract "to buy and sell" with said
had, as of that date, considered as rescinded and of no further legal effect the firm, and which contract was referred and annexed to the "Acknowledgment
logging agreement that they had entered in 1960; and on 7 September 1965, of Debt and Proposed Manner of Payments"; that UP's unilateral rescission
UP filed a complaint against ALUMCO for the collection or payment of the of the logging contract, without a court order, was invalid; that petitioner's
herein before stated sums of money and alleging the facts hereinbefore supervisor refused to allow respondent to cut new logs unless the logs
specified, together with other allegations; it prayed for and obtained an order, previously cut during the management of Cesar Guy be first sold; that
dated 30 September 1965, for preliminary attachment and preliminary respondent was permitted to cut logs in the middle of June 1965 but
injunction restraining ALUMCO from continuing its logging operations in petitioner's supervisor stopped all logging operations on 15 July 1965; that it
the Land Grant. had made several offers to petitioner for respondent to resume logging
28. That before the issuance of the aforesaid preliminary injunction UP had taken operations but respondent received no reply.
steps to have another concessionaire take over the logging operation, by 36. The basic issue in this case is whether petitioner U.P. can treat its contract
advertising an invitation to bid; that bidding was conducted, and the with ALUMCO rescinded, and may disregard the same before any judicial
concession was awarded to Sta. Clara Lumber Company, Inc.; the logging pronouncement to that effect.
contract was signed on 16 February 1966. 37. Respondent ALUMCO contended, and the lower court, in issuing the
29. That, meantime, ALUMCO had filed several motions to discharge the writs injunction order of 25 February 1966, apparently sustained it (although the
of attachment and preliminary injunction but were denied by the court; order expresses no specific findings in this regard), that it is only after a final
30. That on 12 November 1965, ALUMCO filed a petition to enjoin petitioner court decree declaring the contract rescinded for violation of its terms that
University from conducting the bidding; on 27 November 1965, it filed a U.P. could disregard ALUMCO's rights under the contract and treat the
agreement as breached and of no force or effect. compensation in damages, it becomes plain that the acts of the court a quo in
enjoining petitioner's measures to protect its interest without first receiving
ISSUE/s evidence on the issues tendered by the parties, and in subsequently refusing
W/N UP can treat its contract with ALUMCO rescinded, and may disregard the same to dissolve the injunction, were in grave abuse of discretion, correctible by
before any judicial pronouncement to that effect? – YES certiorari, since appeal was not available or adequate. Such injunction,
therefore, must be set aside.
RULING: WHEREFORE, the writ of certiorari applied for is granted, and the order
of the respondent court of 25 February 1966, granting the Associated Lumber
Company's petition for injunction, is hereby set aside. Let the records be remanded for
further proceedings conformably to this opinion.

RATIO:
18. In the first place, UP and ALUMCO had expressly stipulated that upon
default by the debtor, UP has the right and the power to consider the Logging
Agreement of December 2, 1960 as rescinded without the necessity of any
judicial suit.
19. As to such special stipulation and in connection with Article 1191 of the Civil
Code, the Supreme Court, stated in Froilan vs. Pan Oriental Shipping Co:
“There is nothing in the law that prohibits the parties from entering into
agreement that violation of the terms of the contract would cause cancellation
thereof, even without court intervention.
20. In other words, it is not always necessary for the injured party to resort to
court for rescission of the contract.”
21. The law definitely does not require that the contracting party who believes
itself injured must first file suit and wait for a judgment before taking
extrajudicial steps to protect its interest. Otherwise, the party injured by the
other's breach will have to passively sit and watch its damages accumulate
during the pendency of the suit until the final judgment of rescission is
rendered when the law itself requires that he should exercise due diligence to
minimize its own damages.
22. In the light of the foregoing principles, and considering that the complaint of
petitioner University made out a prima facie case of breach of contract and
defaults in payment by respondent ALUMCO, to the extent that the court
below issued a writ of preliminary injunction stopping ALUMCO's logging
operations, and repeatedly denied its motions to lift the injunction
23. That it is not denied that the respondent company had profited from its
operations previous to the agreement of 5 December 1964
("Acknowledgment of Debt and Proposed Manner of Payment"); that the
excuses offered in the second amended answer, such as the misconduct of its
former manager Cesar Guy, and the rotten condition of the logs in private
respondent's pond, which said respondent was in a better position to know
when it executed the acknowledgment of indebtedness, do not constitute on
their face sufficient excuse for non-payment; and considering that whatever
prejudice may be suffered by respondent ALUMCO is susceptibility of
09 PALAY INC. v. CLAVE (Ella) by TCT No. 90454, and owned by said corporation. The sale price was
Sept. 21, 1983 | Melencio-Herrera J. | Extrajudical Rescission P23,300.00 with 9% interest per annum, payable with a downpayment of
P4,660.00 and monthly installments of P246.42 until fully paid.
PETITIONER: Palay Inc. and Albert Onstott 2. Paragraph 6 of the contract provided for automatic extrajudicial rescission
RESPONDENTS: Jacobo C. Clave, Presidential Executive Assistant, National upon default in payment of any monthly installment after the lapse of 90 days
Housing Authority and Nazario Dumpit, from the expiration of the grace period of one month, without need of notice
and with forfeiture of all installments paid.
3. Respondent Dumpit paid the downpayment and several installments
SUMMARY: Petitioner Palay Inc. and private respondent Nazario Dumpit
amounting to P13,722.50. The last payment was made on December 5, 1967
entered into a contract to sell a parcel of land in favor of the latter. Par. 6 of the
for installments up to September 1967.
contract provided for automatic extrajudicial rescission upon default in payment
4. On May 10, 1973, or almost six (6) years later, Dumpit wrote Palay Inc.
of any monthly installment after the lapse of 90 days from the expiration of the
offering to update all his overdue accounts with interest, and seeking its
grace period of 1 month, without need of notice & with forfeiture of all
written consent to the assignment of his rights to a certain Lourdes Dizon. He
installments paid. Dumpit was able to pay some of the installments but it was later
followed this up with another letter dated June 20, 1973 reiterating the same
in default. 6 years from his last payment, Dumpit wrote Palay Inc. offering to
request.
update all his overdue accounts but the latter informed him that the contract to sell
5. Replying, Palay Inc. informed Dumpit that his Contract to Sell had long been
had long been rescinded pursuant to said Par. 6. Dumpit then questioned the
rescinded pursuant to paragraph 6 of the contract, and that the lot had already
validity of the rescission. Palay Inc. maintains that the rescission without prior
been resold.
notice was justified considering that it was expressly provided for in their contract,
and the condition of default occurred. SC said that while extrajudicial rescission 6. Questioning the validity of the rescission of the contract, Dumpit filed a letter
is valid, Palay Inc. should have given prior notice to Dumpit informing the latter complaint with the National Housing Authority (NHA) for reconveyance
of the rescission. Notice is indispensable in extrajudicial rescission. However, with an alternative prayer for refund.
when the party in default questions the validity of the extrajuducial rescission, 7. In a Resolution, dated July 10, 1979, the NHA, finding the rescission void in
judicial determination of the rescission is also necessary. Also in a contract of the absence of either judicial or notarial demand, ordered Palay, Inc. and
adhesion where only one party makes the stipulations and the other only adheres Alberto Onstott, in his capacity as President of the corporation, jointly and
to it such that there is no freedom to stipulate, a waiver of notice must be certain, severally, to refund immediately to Nazario Dumpit the amount of
unequivocal, and intelligently made. Such waiver only follows where freedom of P13,722.50 with 12% interest from the filing of the complaint.
choice has been fully accorded. Finally, as a consequence of rescission, subject lot 8. Palay Inc.'s Motion for Reconsideration of said Resolution was denied by the
must be restored in favor of Dumpit, or an available lot. In case there are no NHA in its Order dated October 23, 1979.
available lots and the subject lot has been transferred to a 3rd person, then 9. On appeal to the Office of the President, upon the allegation that the NHA
installments paid by Dumpit must be refunded plus legal interest. Resolution was contrary to law, Presidential Executive Assistant, on May 2,
1980, affirmed the Resolution of the NHA. Reconsideration sought by Palay
DOCTRINE: Judicial action for the rescission of a contract is not necessary Inc. was denied for lack of merit. Thus, the present petition was filed.
where the contract provides that it may be revoked and cancelled for violation of
any of its terms and conditions. However, notice informing the party in default of ISSUE/s:
the rescission is necessary. Also, extrajudicial rescission has legal effect where the 1. (Main Issue) WON notice or demand is indispensible under the
other party does not oppose it. Where it is objected to, a judicial determination of circumstances in a stipulation in a contract to sell = YES, NOTICE IS
the issue is still necessary. INDISPENSIBLE
2. WON the installments paid by private respondent Dumpit must be refunded
= YES
FACTS:
1. On March 28, 1965, petitioner Palay, Inc., through its President, Albert RULING: WHEREFORE, the questioned Resolution of respondent public official,
Onstott, executed in favor of private respondent, Nazario Dumpit, a Contract dated May 2, 1980, is hereby modified. Petitioner Palay, Inc. is directed to refund to
to Sell a parcel of Land (Lot No. 8, Block IV) of the Crestview Heights respondent Nazario M. Dumpit the amount of P13,722.50, with interest at twelve
Subdivision in Antipolo, Rizal, with an area of 1,165 square meters, covered (12%) percent per annum from November 8, 1974, the date of the filing of the
Complaint. The temporary Restraining Order heretofore issued is hereby lifted. No 7. The contention that Dumpit had waived his right to be notified under
costs. SO ORDERED. paragraph 6 of the contract is neither meritorious because it was a contract
of adhesion, a standard form of petitioner Palay Inc., and private respondent
RATIO: Dumpit had no freedom to stipulate. A waiver must be certain and
Notice is indispensable. unequivocal, and intelligently made; such waiver follows only where
liberty of choice has been fully accorded. Moreover, it is a matter of
1. Palay Inc. maintains that it was justified in cancelling the contract to sell public policy to protect buyers of real estate on installment payments
without prior notice or demand upon Dumpit in view of paragraph 6 of their against onerous and oppressive conditions. Waiver of notice is one such
contract with Dumpit. SC said Palay Inc. is wrong. onerous and oppressive condition to buyers of real estate on installment
payments.
2. Well settled is the rule, as held in previous jurisprudence, that judicial action
for the rescission of a contract is not necessary where the contract provides Installments paid by Dumpit must be refunded
that it may be revoked and cancelled for violation of any of its terms and 1. Art. 138530 of the Civil Code gives petitioner Palay Inc. the obligation to
conditions. However, even in previous jurisprudence, there was at least a refund the installments paid by private respondent Dumpit as a consequence
written notice sent to the defaulter informing him of the rescission. The act of the rescission.
of a party in treating a contract as cancelled should be made known to
the other. 2. Rights to the lot should be restored to Dumpit or the same should be replaced
by another acceptable lot. However, considering that the property had
3. Extrajudicial rescission has legal effect where the other party does not already been sold to a third person (this fact was not mentioned in the facts,
oppose it. Where it is objected to, a judicial determination of the issue is only in the ratio) and there is no evidence on record that other lots are still
still necessary. In other words, resolution of reciprocal contracts may be available, Dumpit is entitled to the refund of installments paid plus interest
made extrajudicially unless successfully impugned in Court. If the debtor at the legal rate of 12% computed from the date of the institution of the
impugns the declaration, it shall be subject to judicial determination. action.
4. In this case, private respondent Nazario Dumpit has denied that rescission is 3. It would be most inequitable if Palay Inc. was to be allowed to retain
justified and has resorted to judicial action (since he questioned the validity Dumpit’s payments and at the same time appropriate the proceeds of the
of the rescission). It is now for the Court to determine whether resolution of second sale to another.
the contract by Palay Inc. was warranted.

5. SC held that resolution by Palay Inc. of the contract was ineffective and
inoperative against Dumpit for lack of notice of resolution, as held in the
U.P. vs. Angeles case.

6. The indispensability of notice of cancellation to the buyer was to be later


underscored in Republic Act No. 6551 entitled "An Act to Provide
Protection to Buyers of Real Estate on Installment Payments." which
took effect on September 14, 1972.

30
ART. 1385. Rescission creates the obligation to return the things which Neither shall rescission take place when the things which are the object of the
were the object of the contract, together with their fruits, and the price with contract are legally in the possession of third persons who did not act in bad
its interest; consequently, it can be carried out only when he who demands faith. In this case, indemnity for damages may be demanded from the person
rescission can return whatever he may be obliged to restore. causing the loss
10 AFP Mutual Benefit Assn. v. CA (EMAR) FACTS:
September 10, 2001| Pardo, J. | Cancellation of Contracts to Sell 1. September 1976: Investco & Solid Homes entered into an agreement to
G.R. No. 104769 buy and to sell. Under the contract to sell, the vendors bound themselves to
PETITIONER: AFP MUTUAL BENEFIT ASSOCIATION, INC. cause the titles to the land to be transferred in the name of Investco after
RESPONDENT: COURT OF APPEALS, SOLID HOMES, INC., INVESTCO, which, should Solid Homes complete the installment payments, Investco
INC., and REGISTER OF DEEDS OF MARIKINA would execute a Deed of Absolute Sale in favor of Solid Homes and Solid
G.R. No. 135016 Homes would execute a first preferred mortgage in favor of Investco. The
PETITIONER: AFP MUTUAL BENEFIT ASSOCIATION, INC. deed of absolute sale would replace the contract to sell. Only then would
RESPONDENT: INVESTCO, INC., substituted by ARMED FORCES OF THE Solid Homes be entitled to take possession of the QC and Marikina parcels
PHILIPPINES MUTUAL BENEFIT ASSOCIATION, INC. of land and introduce improvements thereon.
SUMMARY: Investco and Solid Homes entered into a contract to sell for parcels of 2. March 21, 1979: titles to the Marikina property were issued in the name of
land in QC and Marikina but Solid Homes wasn’t able to pay its obligation in full, Investco. However, it did not execute a deed of absolute sale in favor of Solid
so Investco extra-judicially rescinded the contract between them and later on entered Homes because Solid Homes never paid in full its stipulated obligation
into a contract of absolute sale with AFP, involving the same property. Solid Homes payable in installments. In fact, Solid Homes did not even bother to register
was claiming that AFP is not a purchaser in good faith and for value because the its contract to sell with the Register of Deeds pursuant to the Property
subject properties it has bought has a notice of lis pendence31 on its title. The court Registration Decree (PD1529).
ruled that AFP is a purchaser in good faith and for value through stating that the 3. With the same property as the subject matter, Investco & AFP entered into
Rules of Court allows the annotation of a notice of lis pendens in actions affecting a contract of absolute sale. Investco sold the property which AFP paid for in
the title or right of possession of real property, or an interest in such real property full, causing the transfer of titles in the name of AFP.
and that the basis of the annotation pending with the trial court was an action for 4. This case is an MR by Solid Homes of the SC decision ordering the
collection of a sum of money and did not involve the titles to, possession or Register of Deeds to cancel the notice of lis pendens on the titles issued to
ownership of the subject property or an interest therein. Secondly, the court stated AFP, declaring AFP as a buyer in good faith and for value32.
that upon Solid Homes’ failure to comply with its obligation to pay the installments 5. Solid Homes’ position is based on the preposition that a notice of lis
for the purchase of the properties on the dates specified in the contract to sell, there pendens was duly annotated on the vendor’s title that must be deemed carried
was no need to judicially rescind the contract to sell. Failure by one of the parties to over to the titles issued to AFP, subjecting it to final result of the litigation as
abide by the conditions in a contract to sell resulted in the rescission of the contract. transferee pendente lite.
It would be unfair if Solid Homes which has failed to pay anything since 1981 and
defaulted since 1982, would now get the property by performance of the very ISSUES:
contract which it violated. With the passage of time, more than 14y, and appreciation 1. WON AFP was a purchaser in good faith and for value - YES
in the value of real estate which is now worth billions of Pesos, thus enriching Solid 2. WON the Court erred in failing to appreciate Solid Homes’cause of action - NO
Homes for its violation of the contract and default on its obligation. 3. WON the Court erred in denying Solid Homes’ petition to set aside the trial courts
order denying its motion to execute its decision - NO
DOCTRINE: In a contract to sell, upon failure of buyer to comply with its
obligation, there was no need to judicially rescind the contract to sell. Failure by RULING: IN VIEW WHEREOF, we DENY Solid Homes, Inc.’s motion for
one of the parties to abide by the conditions in a contract to sell resulted in the reconsideration, for lack of merit. The denial is final.
rescission of the contract.
RATIO:
On WON AFP was a purchaser in good faith

31
A notice of lis pendens is an announcement to the whole world that a particular real property is in for the same, at the time of such purchase, or before he has notice of the claim or interest of some other
litigation, serving as a warning that one who acquires interest over said property does so at his own risk, or person in the property.
that he gambles on the result of the litigation over the said property.
32
A purchaser in good faith and for value is defined by the court as one who buys the property of another
without notice that some other person has a right to or interest in such property and pays a full and fair price
1. The Rules of Court allows the annotation of a notice of lis pendens in Contract of Sale vs. Contract to Sell
actions affecting the title or right of possession of real property, or an 8. The contractual relation between Investco and Solid Homes, is based on
interest in such real property. It is declared that the rule of lis pendens an agreement executed as a contract to sell and to buy, in which AFP is not
applied to suits brought to establish an equitable estate, interest, or right a party.
in specific real property or to enforce any lien, charge, or encumbrance 9. The relationship between AFP and Investco arose out of a contract of
against it x x x. absolute sale after Solid Homes defaulted on its contract to sell, and
2. Pencil markings are not an accepted form of annotating a notice of lis Investco rescinded extra-legally such contact to sell with Solid Homes.
pendens. SC cannot accept the argument that such pencil annotation can be AFP did not acquire from Solid Homes its rights or interest over the
considered as a valid annotation of notice of lis pendens, and thus an effective property in question; Investco sold the property itself which AFP paid in
notice to the whole world as to the status of the title to the land. The law full, causing transfer of titles in the name of AFP.
requires proper annotation, not provisional annotation of a notice of lis 10. When the contract was entered into between Solid Homes and Investco
pendens. in September 1976, the titles to the QC and Marikina property had not
3. To allow provisional annotations as a valid form of annotation of notice of been transferred in the name of Investco as assignee of the owners. Hence,
lis pendens will erode the very value of the indefeasibility of the Torrens Investco merely agreed to sell, and Solid Homes to buy the rights and
system. If there were a valid annotation of notice of lis pendens, the same interest of Investco in the property which at the time was still registered in
would have been carried over to the titles issued to AFP, but there was none, the names of Staley and Perez, Investco’s predecessors-in-interest.
so the TCTs of the vendor Investco conveyed to AFP were clean and without 11. Under the contract of sale, the vendors bound themselves to cause the
any encumbrance. titles to the land to be transferred in the name of Investco after which,
4. There could be no valid annotation on the titles issued to AFP because the should Solid Homes complete the installment payments, Investco would
case used as basis of the annotation pending with the trial court was an action execute a Deed of Absolute Sale in favor of Solid Homes & Solid Homes
for collection of a sum of money and did not involve the titles to, would execute a first preferred mortgage in favor of Investco. The deed
possession or ownership of the subject property or an interest therein. of absolute sale would replace the contract to sell. Only then would Solid
The SC, categorized the action initiated by Investco against Solid Homes as: Home be entitled to take possession of the QC and Marikina parcels of
“An action for collection of sums of money, damages and attorneys fees was land and introduce improvements thereon.
filed with the RTC…” 12. Around March 1979, the titles to the property were issued in the name
5. Such action did not directly involve titles to, ownership or possession of Investco but Investco did not execute a deed of absolute sale in favor of
of the subject property, & therefore, was not a proper subject of a notice Solid Homes because Solid Homes never paid in full its stipulated
of lis pendens. obligation payable in installments. Solid Homes did not even bother to
6. The Torrens system was believed to be the most effective measure to register its contract to sell with the Register of Deeds pursuant to the
guarantee the integrity of land titles and to protect their indefeasibility Property Registration Decree (PD 1529).
once the claim of ownership is established and recognized. If a person 13. Solid Homes’ contention that the transaction between AFP, Investco and
purchases a piece of land on the assurance that the sellers title thereto is valid, Solid Homes is in the nature of a double sale is untenable. The transaction
he should not run the risk of being told later that his acquisition was between:
ineffectual after all. This would not only be unfair to him; if this were (a) Investco and Solid Homes: contract to sell and to buy (not
permitted, public confidence in the system would be eroded and land consummated because Solid Homes defaulted on its payments)
transactions would be attended by complicated and not necessarily (b) Investco and AFP: absolute sale (culminated in the registration of the
conclusive investigations and proof of ownership. The further consequence deeds + issuance of certificate of titles in favor of AFP)
would be that land conflicts could be even more numerous and complex than 14. Salazar v. CA: Differentiated the Contract to Sell and Contract of Sale
they are now and possibly also more abrasive, if not even violent. Contract to Sell Contract of Sale
7. All persons dealing with property covered by the Torrens certificate of title Ownership is, by agreement, Title to the property passes to the
are not required to go beyond what appears on the face of the title. The reserved in the vendor and is not vendee upon the delivery of the
buyer is not obligated to look beyond the certificate to investigate the titles
to pass to the vendee until full thing sold
of the seller appearing on the face of the certificate. Hence, AFP is a buyer in
good faith and for value. payment of the purchase price.
Title is retained by the vendor until Vendor loses ownership over the 14y, and appreciation in the value of real estate, the property is now worth
full payment of the price. property and cannot recover it until billions of pesos,thus enriching Solid Homes for its violation of the contract
and unless the contract is resolved or and default on its obligation.
rescinded
Payment of the price is a suspensive
condition, failure of which is not a
breach but an event that prevents
the obligation of the vendor to
convey title from becoming
effective.
15. Upon Solid Homes’ failure to comply with its obligation to pay the
installments for the purchase of the properties on the dates specified in the
contract to sell, there was no need to judicially rescind the contract to sell.
Failure by one of the parties to abide by the conditions in a contract to sell
resulted in the rescission of the contract.

Solid Homes has no Personality to Move for Execution


16. The court found untenable Solid Homes’ last contention that when the
decision in the previous Civil Case became final, there was no one to move
for execution of the decision since Investco had absconded, and had re-
sold the property. Investco was the party which had the right to demand
execution.
17. Once a judgment becomes final and executory, the prevailing party can
have it executed as a matter of right, and the issuance of a writ of
execution becomes a ministerial duty of the court. In fact, the prevailing
party is the one really entitled to file a motion for the issuance of a writ
of execution.
18. In this case, it was Solid Homes that filed a motion for execution of
judgment in the court of origin which court denied the motion. Hence, 2 years
later, Solid Homes filed a petition for certiorari with the SC.
19. Assuming that AFP was bound by the judgment in the previous and be
substituted for Investco, it is clear that Investco prevailed in the case. It was
the winning party which is entitled as a matter of right to a writ of
execution in its favor. It is not an option of the losing party to file a motion
for execution of judgment to compel the winning party to take the
judgment. As the losing party, Solid Homes can not now insist on the
performance of the very contract on which it defaulted for more than
14y. Hence, Solid Homes has no personality to move for execution of the
final judgment in the Civil Case. The trial court correctly denied its motion
for execution.
20. It would be unfair if Solid Homes which has failed to pay anything since
1981 and defaulted since 1982, would now get the property by performance
of the very contract which it violated. With the passage of time, more than
01 ROBERTO Z. LAFORTEZA, GONZALO Z. LAFORTEZA, MICHAEL Z. imposed on the performance of an obligation only gives the other party the option
LAFORTEZA, DENNIS Z. LAFORTEZA, and LEA Z. LAFORTEZA either to refuse to proceed with sale or waive the condition.
vs. ALONZO MACHUCA (sarmiento)
June 16, 2000 | Gonzaga-Reyes, J. | Contract of sale
PETITIONER: Roberto Laforteza et al. FACTS:
RESPONDENTS: Alonzo Machuca
1. The property involved consists of a house and lot located at No. 7757
SUMMARY: The subject property of the case is registered in the name of Francisco Sherwood Street, Marcelo Green Village, Paraaque, Metro Manila. The
Laforteza. Francisco already died thus his heirs were in the process of settling his subject property is registered in the name of the late Francisco Q. Laforteza,
estate, including the subject property. The heirs, Lea Laforteza, Michael Laforteza although it is conjugal in nature
and Dennis Laforteza authorized Roberto and Gonzalo Laforteza to sell the property.
The agents thus entered into a Memorandum of Agreement (Contract to sell) with 2. Defendant Lea Zulueta-Laforteza, Michael Laforteza and Dennis Laforteza
Machuca. In the agreement, it contained a provision stating that upon issuance by the executed a Special Power of Attorney in favor of defendants Roberto Z.
proper Court of the new title, the BUYER-LESSEE shall be notified in writing and Laforteza and Gonzalo Z. Laforteza, Jr., appointing both as her Attorney-in-
said BUYER-LESSEE shall have thirty (30) days to produce the balance fact authorizing them jointly to sell the subject property and sign any
of P600,000.00 which shall be paid to the SELLER-LESSORS upon the execution document for the settlement of the estate of the late Francisco Q. Laforteza
of the Extrajudicial Settlement with sale. Mahuca paid earnest money of P30,000.
Then he asked for an extension within which to pay the balance. It was approved by 3. In the exercise of the above authority, the heirs of the late Francisco Q.
Roberto but not by Gonzalo. Thus when Machuca said that he already had the money Laforteza represented by Roberto Z. Laforteza and Gonzalo Z. Laforteza, Jr.
to pay the full amount, the sellers told him that they were cancelling the sale. entered into a Memorandum of Agreement (Contract to Sell) with the
Machuca thus filed a complaint for specific performance. The court ruled that the plaintiff over the subject property for the sum of (P630,000.00) payable as
sellers should perform their obligation to deliver and transfer ownership over the follows:
property to the buyer, as there was already a perfected contract of sale. The six-month a. P30,000.00 as earnest money, to be forfeited in favor of the
period during which the respondent would be in possession of the property as lessee, defendants if the sale is not effected due to the fault of the plaintiff;
was clearly not a period within which to exercise an option. the six-month period b. P600,000.00 upon issuance of the new certificate of title in the
merely delayed the demandability of the contract of sale and did not determine its name of the late Francisco Q. Laforteza and upon execution of an
perfection for after the expiration of the six-month period, there was an absolute extra-judicial settlement of the decedents estate with sale in favor of
obligation on the part of the petitioners and the respondent to comply with the terms the plaintiff.
of the sale. Furthermore, The issuance of the new certificate of title in the name of
the late Francisco Laforteza and the execution of an extrajudicial settlement of his 4. Significantly, the fourth paragraph of the Memorandum of Agreement
estate was not a condition which determined the perfection of the contract of sale. (Contract to Sell) contained a provision as follows:
The petitioners fail to distinguish between a condition imposed upon the perfection xxx. Upon issuance by the proper Court of the new title, the
of the contract and a condition imposed on the performance of an obligation. Failure BUYER-LESSEE shall be notified in writing and said BUYER-
to comply with the first condition results in the failure of a contract, while the failure LESSEE shall have thirty (30) days to produce the balance
to comply with the second condition only gives the other party the option either to of P600,000.00 which shall be paid to the SELLER-LESSORS upon
refuse to proceed with the sale or to waive the condition. What further militates the execution of the Extrajudicial Settlement with sale.
against petitioners argument that they did not enter into a contract of sale is the fact
that the respondent paid thirty thousand pesos (P30,000.00) as earnest money. 5. Plaintiff paid the earnest money of (P30,000.00), plus rentals for the subject
Whenever earnest money is given in a contract of sale, it is considered as part of the property
purchase price and proof of the perfection of the contract
6. Defendant heirs, through their counsel wrote a letter to the plaintiff furnishing
DOCTRINE: Failure to comply with condition imposed upon perfection of the the latter a copy of the reconstituted title to the subject property, advising him
contract results in failure of a contract, while the failure to comply with a condition that he had thirty (30) days to produce the balance of (P600,000.00) under
the Memorandum of Agreement which plaintiff received on the same date
14. Petitioners contend that the Memorandum of Agreement is merely a lease
7. Plaintiff sent the defendant heirs a letter requesting for an extension of the 30 agreement with "option to purchase". As it was merely an option, it only gave
days deadline up to November 15, 1989 within which to produce the balance the respondent a right to purchase the subject property within a limited period
of (P600,000.00). Defendant Roberto Z. Laforteza, signed his conformity to without imposing upon them any obligation to purchase it. Since the
the plaintiffs letter request. The extension, however, does not appear to have respondents tender of payment was made after the lapse of the option
been approved by Gonzalo Z. Laforteza, the second attorney-in-fact as his agreement, his tender did not give rise to the perfection of a contract of sale.
conformity does not appear to have been secured.
15. It is further maintained by the petitioners that the Court of Appeals erred in
8. Plaintiff informed the defendant heirs, through defendant Roberto Z. ruling that rescission of the contract was already out of the question.
Laforteza, that he already had the balance of (P600,000.00). However, the Rescission implies that a contract of sale was perfected unlike the
defendants, refused to accept the balance. Defendant Roberto Z. Laforteza Memorandum of Agreement in question which as previously stated is
had told him that the subject property was no longer for sale. allegedly only an option contract.

9. Defendants informed the plaintiff that they were canceling the Memorandum 16. Petitioner adds that at most, the Memorandum of Agreement (Contract to
of Agreement (Contract to Sell) in view of the plaintiffs failure to comply Sell) is a mere contract to sell, as indicated in its title. The obligation of the
with his contractual obligations. petitioners to sell the property to the respondent was conditioned upon the
issuance of a new certificate of title and the execution of the extrajudicial
10. Thereafter, plaintiff reiterated his request to tender payment of the balance of partition with sale and payment of the P600,000.00. This is why possession
(P600,000.00). Defendants, however, insisted on the rescission of the of the subject property was not delivered to the respondent as the owner of
Memorandum of Agreement. Thereafter, plaintiff filed the instant action for the property but only as the lessee thereof. And the failure of the respondent
specific performance. to pay the purchase price in full prevented the petitioners obligation to convey
title from acquiring obligatory force.
11. Lower court: judgment is hereby rendered in favor of plaintiff Alonzo
Machuca and against the defendant heirs of the late Francisco Q. Laforteza, 17. Petitioners also allege that assuming for the sake of argument that a contract
ordering the said defendants. Defendants are ordered to accept the balance of of sale was indeed perfected, the Court of Appeals still erred in holding that
P600,000.00 as full payment of the consideration for the purchase of the respondents failure to pay the purchase price of P600,000.00 was only a
house and lot. "slight or casual breach".

12. CA: affirmed the decision of the lower court 18. The petitioners also claim that the Court of Appeals erred in ruling that they
were not ready to comply with their obligation to execute the extrajudicial
13. Hence this petition. settlement. The Power of Attorney to execute a Deed of Sale made by Dennis
Z. Laforteza was sufficient and necessarily included the power to execute an
ISSUE: Whether or not there was a perfected contract of sale-YES extrajudicial settlement. At any rate, the respondent is estopped from
claiming that the petitioners were not ready to comply with their obligation
for he acknowledged the petitioners ability to do so when he requested for an
RULING:
extension of time within which to pay the purchase price. Had he truly
believed that the petitioners were not ready, he would not have needed to ask
ACCORDINGLY, the decision of the Court of Appeals in CA G.R. CV No. for said extension.
47457 is AFFIRMED and the instant petition is hereby DENIED.
19. Finally, the petitioners allege that the respondents uncorroborated testimony
RATIO: that third persons offered a higher price for the property is hearsay and should
not be given any evidentiary weight. Thus, the order of the lower court
Petitioner’s CONTENTIONS: awarding moral damages was without any legal basis.
COURT: a. In the present case, the six-month period merely delayed the
demandability of the contract of sale and did not determine its
20. A perusal of the Memorandum Agreement shows that the transaction between perfection for after the expiration of the six-month period, there was
the petitioners and the respondent was one of sale and lease. The terms of the an absolute obligation on the part of the petitioners and the
agreement read: (See Annex A) respondent to comply with the terms of the sale.
b. The parties made a "reasonable estimate" that the reconstitution of
21. A contract of sale is a consensual contract and is perfected at the moment the lost title of the house and lot would take approximately six
there is a meeting of the minds upon the thing which is the object of the months and thus presumed that after six months, both parties would
contract and upon the price. From that moment the parties may reciprocally be able to comply with what was reciprocally incumbent upon them.
demand performance subject to the provisions of the law governing the form c. The fact that after the expiration of the six-month period, the
of contracts. respondent would retain possession of the house and lot without
need of paying rentals for the use therefor, clearly indicated that the
22. In the case at bench, there was a perfected agreement between the petitioners parties contemplated that ownership over the property would
and the respondent whereby the petitioners obligated themselves to transfer already be transferred by that time.
the ownership of and deliver the house and lot located at 7757 Sherwood St.,
Marcelo Green Village, Paraaque and the respondent to pay the price 24. The issuance of the new certificate of title in the name of the late Francisco
amounting to six hundred thousand pesos (P600,000.00). Laforteza and the execution of an extrajudicial settlement of his estate was
a. All the elements of a contract of sale were thus present. not a condition which determined the perfection of the contract of sale.
b. However, the balance of the purchase price was to be paid only upon a. Petitioners contention that since the condition was not met, they no
the issuance of the new certificate of title in lieu of the one in the longer had an obligation to proceed with the sale of the house and
name of the late Francisco Laforteza and upon the execution of an lot is unconvincing.
extrajudicial settlement of his estate. b. The petitioners fail to distinguish between a condition imposed upon
c. Prior to the issuance of the "reconstituted" title, the respondent was the perfection of the contract and a condition imposed on the
already placed in possession of the house and lot as lessee thereof performance of an obligation.
for six months at a monthly rate of three thousand five hundred c. Failure to comply with the first condition results in the failure of a
pesos (P3,500.00). contract, while the failure to comply with the second condition only
d. It was stipulated that should the issuance of the new title and the gives the other party the option either to refuse to proceed with the
execution of the extrajudicial settlement be completed prior to sale or to waive the condition.
expiration of the six-month period, the respondent would be liable
only for the rentals pertaining to the period commencing from the 25. Thus, Art. 1545 of the Civil Code states:
date of the execution of the agreement up to the execution of the a. "Art. 1545. Where the obligation of either party to a contract of sale
extrajudicial settlement. is subject to any condition which is not performed, such party may
e. It was also expressly stipulated that if after the expiration of the six refuse to proceed with the contract or he may waive performance of
month period, the lost title was not yet replaced and the extrajudicial the condition. If the other party has promised that the condition
partition was not yet executed, the respondent would no longer be should happen or be performed, such first mentioned party may also
required to pay rentals and would continue to occupy and use the treat the nonperformance of the condition as a breach of warranty.
premises until the subject condition was complied with by the Where the ownership in the things has not passed, the buyer may
petitioners. treat the fulfillment by the seller of his obligation to deliver the same
as described and as warranted expressly or by implication in the
23. The six-month period during which the respondent would be in possession of contract of sale as a condition of the obligation of the buyer to
the property as lessee, was clearly not a period within which to exercise an perform his promise to accept and pay for the thing.
option. An option is a contract granting a privilege to buy or sell within an
agreed time and at a determined price. 26. In the case at bar, there was already a perfected contract. The condition was
imposed only on the performance of the obligations contained therein.
Considering however that the title was eventually "reconstituted" and that the ready to comply with what was incumbent upon them, i.e. the
petitioners admit their ability to execute the extrajudicial settlement of their delivery of the reconstituted title of the house and lot.
fathers estate, the respondent had a right to demand fulfillment of the b. It was only on September 18, 1989 or nearly eight months after the
petitioners obligation to deliver and transfer ownership of the house and lot execution of the Memorandum of Agreement when the petitioners
informed the respondent that they already had a copy of the
27. What further militates against petitioners argument that they did not enter reconstituted title and demanded the payment of the balance of the
into a contract of sale is the fact that the respondent paid thirty thousand pesos purchase price.
(P30,000.00) as earnest money. Earnest money is something of value to show c. The respondent could not therefore be considered in delay for in
that the buyer was really in earnest, and given to the seller to bind the bargain. reciprocal obligations, neither party incurs in delay if the other party
Whenever earnest money is given in a contract of sale, it is considered as part does not comply or is not ready to comply in a proper manner with
of the purchase price and proof of the perfection of the contract what was incumbent upon him.

28. We do not subscribe to the petitioners view that the Memorandum Agreement 31. Even assuming for the sake of argument that the petitioners were ready to
was a contract to sell. There is nothing contained in the Memorandum comply with their obligation, we find that rescission of the contract will still
Agreement from which it can reasonably be deduced that the parties intended not prosper. The rescission of a sale of an immovable property is specifically
to enter into a contract to sell. governed by Article 1592 of the New Civil Code, which reads:
a. There is clearly no express reservation of title made by the a. "In the sale of immovable property, even though it may have been
petitioners over the property, or any provision which would impose stipulated that upon failure to pay the price at the time agreed upon
non-payment of the price as a condition for the contracts entering the rescission of the contract shall of right take place, the vendee
into force. may pay, even after the expiration of the period, as long as no
b. Although the memorandum agreement was also denominated as a demand for rescission of the contract has been made upon him either
"Contract to Sell", we hold that the parties contemplated a contract judicially or by a notarial act. After the demand, the court may not
of sale. grant him a new term.
c. In such cases, ownership of the thing sold passes to the vendee upon
actual or constructive delivery thereof. The mere fact that the 32. It is not disputed that the petitioners did not make a judicial or notarial
obligation of the respondent to pay the balance of the purchase price demand for rescission. The November 20, 1989 letter of the petitioners
was made subject to the condition that the petitioners first deliver informing the respondent of the automatic rescission of the agreement did not
the reconstituted title of the house and lot does not make the contract amount to a demand for rescission, as it was not notarized.
a contract to sell for such condition is not inconsistent with a a. It was also made five days after the respondents attempt to make the
contract of sale. payment of the purchase price. This offer to pay prior to the demand
for rescission is sufficient to defeat the petitioners right under article
29. The next issue to be addressed is whether the failure of the respondent to pay 1592 of the Civil Code.
the balance of the purchase price within the period allowed is fatal to his right
to enforce the agreement. We rule in the negative. 33. Besides, the Memorandum Agreement between the parties did not contain a
clause expressly authorizing the automatic cancellation of the contract
30. Admittedly, the failure of the respondent to pay the balance of the purchase without court intervention in the event that the terms thereof were violated.
price was a breach of the contract and was a ground for rescission thereof. a. A seller cannot unilaterally and extrajudicially rescind a contract of
The extension of thirty (30) days allegedly granted to the respondent by sale where there is no express stipulation authorizing him to
Roberto Z. Laforteza (assisted by his counsel Attorney Romeo Gutierrez) was extrajudicially rescind. Neither was there a judicial demand for the
correctly found by the Court of Appeals to be ineffective inasmuch as the rescission thereof.
signature of Gonzalo Z. Laforteza did not appear thereon as required by the b. Thus, when the respondent filed his complaint for specific
Special Powers of Attorney. performance, the agreement was still in force inasmuch as the
a. However, the evidence reveals that after the expiration of the six- contract was not yet rescinded. At any rate, considering that the six-
month period provided for in the contract, the petitioners were not month period was merely an approximation of the time it would take
to reconstitute the lost title and was not a condition imposed on the the execution of an Extrajudicial Settlement of his estate
perfection of the contract and considering further that the delay in with sale in favor of BUYER-LESSEE free from lien or
payment was only thirty days which was caused by the respondents any encumbrances.
justified but mistaken belief that an extension to pay was granted to
him, we agree with the Court of Appeals that the delay of one month 3. Parties reasonably estimate that the issuance of a new title in place
in payment was a mere casual breach that would not entitle the of the lost one, as well as the execution of extrajudicial settlement
respondents to rescind the contract. of estate with sale to herein BUYER-LESSEE will be completed
c. Rescission of a contract will not be permitted for a slight or casual within six (6) months from the execution of this Agreement. It is
breach, but only such substantial and fundamental breach as would therefore agreed that during the six months period, BUYER-
defeat the very object of the parties in making the agreement. LESSEE will be leasing the subject property for six months period
at the monthly rate of PESOS: THREE THOUSAND FIVE
34. Petitioners insistence that the respondent should have consignated the amount HUNDRED (P3,500.00). Provided however, that if the issuance of
is not determinative of whether respondents action for specific performance new title and the execution of Extrajudicial Partition is completed
will lie. Petitioners themselves point out that the effect of consignation is to prior to the expiration of the six months period, BUYER-LESSEE
extinguish the obligation. It releases the debtor from responsibility shall only be liable for rentals for the corresponding period
therefor. The failure of the respondent to consignate the P600,000.00 is not commencing from his occupancy of the premises to the execution
tantamount to a breach of the contract for by the fact of tendering payment, and completion of the Extrajudicial Settlement of the estate,
he was willing and able to comply with his obligation. provided further that if after the expiration of six (6) months, the lost
title is not yet replaced and the extra judicial partition is not
_______________ executed, BUYER-LESSEE shall no longer be required to pay
rentals and shall continue to occupy, and use the premises until
Annex A: subject condition is complied by SELLER-LESSOR;

4. It is hereby agreed that within reasonable time from the execution


"1. For and in consideration of the sum of PESOS: SIX HUNDRED of this Agreement and the payment by BUYER-LESSEE of the
THIRTY THOUSAND (P630,000.00) payable in a manner herein amount of P30,000.00 as herein above provided, SELLER-
below indicated, SELLER-LESSOR hereby agree to sell unto LESSORS shall immediately file the corresponding petition for the
BUYER-LESSEE the property described in the first WHEREAS of issuance of a new title in lieu of the lost one in the proper Courts.
this Agreement within six (6) months from the execution date Upon issuance by the proper Courts of the new title, the BUYER-
hereof, or upon issuance by the Court of a new owners certificate of LESSEE shall have thirty (30) days to produce the balance of
title and the execution of extrajudicial partition with sale of the P600,000.00 which shall be paid to the SELLER-LESSORS upon
estate of Francisco Laforteza, whichever is earlier; the execution of the Extrajudicial Settlement with sale.

2. The above-mentioned sum of PESOS: SIX HUNDRED THIRTY


THOUSAND (P630,000.00) shall be paid in the following manner:

P30,000.00- as earnest money and as consideration for this


Agreement, which amount shall be forfeited in favor of
SELLER-LESSORS if the sale is not effected because of
the fault or option of BUYER-LESSEE;

P600,000.00- upon the issuance of the new certificate of


title in the name of the late Francisco Laforteza and upon
02 Heirs of Escanlar v CA (Siapno) 4. The Cari-ans, executed the Deed of Sale of Rights, Interests and Participation
October 23, 1997 | Romero, J. | Stipulation affecting effectivity, not validity in favor of Escanlar and Hogaldo which had the ff stipulations:
a. P275,000.00 is to be paid by Escanlar and Hogaldo to the Cari-ans, except the
PETITIONER: Heirs of Pedro Escanlar, Francisco Holgado, and Spouses Edwin share of the minor child of Leonardo Cari-an, which should be deposited with
Jayme and Elisa Tan-Jayme the Municipal Treasurer of Himamaylan, Negros Occidental, by the order of
RESPONDENTS: The Honorable Court of Appeals, Generosa Martinez, the CFI of Negros Occidental, Branch VI, Himamaylan
Carmen Cari-An, Rodolfo Cari-An, Nelly Chue Cari-An, for herself and as b. Cari-ans sell, cede, transfer, convey, by way of absolute sale all the rights,
guardian ad litem of her minor son, Leonell Cari-An, Fredisminda Cari-An, the interests, and participation of the Vendors as to the ½ portion pro-indiviso of
Spouses Paquito Chua and Ney Sarrosa-Chua and The Register of Deeds of the 2 lots (Fishpond), of the Kabankalan Cadastre, pertaining to the one-half
Negros Occidental (1/2) portion pro-indiviso of the late Victoriana Cari-an
c. That this Contract of Sale shall become effective only upon the approval
SUMMARY: The Heirs of Cari-an executed a Deed of Sale of Rights, Interests, by the Honorable Court of First Instance of Negros Occidental, Branch
and Participation over a parcel of undivided land in favor of Escanlar and VI-Himamaylan.
Hogaldo. It was stipulated that “the contract shall become effective only upon d. Escanlar and Holgado were concurrently the lessees of the lots. They stipulated
approval of the CFI of Negros Occidental.” The Heirs of Escanlar failed to pay that the balance of the purchase price P225,000.00 shall be paid on or before
the balance of the purchase price, but the Heirs of Cari-an never demanded May 1979 in a Deed of Agreement and that pending the complete payment
payment and continued to accept belated payments. They later on sold their thereof, Vendees shall not assign, sell, lease, nor mortgage the rights, interests
interests over the same land to the Chuas and assailed the validity of the Deed of and participation thereof
Sale they executed with the Heirs of Escanlar. The lower courts annulled the e. Vendees’ failure to pay the balance of said purchase price on May 31, 1979 and
contract for not having the approval of the court as stipulated. Issue is WON the the cancellation of said Contract of Sale is made thereby, P50,000.00 shall be
Deed of Sale to the Heirs of Escanlar is valid. SC said yes. There is a distinction deemed as damages thereof to Vendors.
between the validity and effectivity. Only the effectivity was made subject to the 5. Escanlar and Hogaldo were unable to pay the heirs but said heirs received at
condition. So long as all the requisites (consent, subject matter, and price) are least 12 installments from petitioners after May 1979. Cari-ans were fully
present, as in this case, the contract is already perfected. Nonetheless, the intent compensated for their individual shares, per receipts given in evidence. The
of the parties clearly manifests their intention to give efficacy to the contract. In minor Leonell’s share was deposited with the RTC.
fact, the vendors continued to accept payments. That being the case, the sale in 6. Being former lessees, petitioners continued in possession of the 2
favor of the Heirs of Escanlar must be preferred as it is a valid and subsisting one. lots. Interestingly, they continued to pay rent based on their lease contract.
7. The probate court approved Cari-ans’ sale of shares in eight parcels of land
DOCTRINE: There is a distinction between the validity and effectivity. Only the including the 2 lots to the Spouses Chua for P1,850,000.00.
effectivity was made subject to the condition. 8. Cari-ans opposed the motion for approval of the sale of Escanlar and Holgado
in the probate proceedings of Nombre and Cari-an as the buyers of private
FACTS: respondent Cari-ans share in the 2 lots because of petitioners failure to pay the
1. Spouses Guillermo Nombre and Victoriana Cari-an died. Nombres heirs balance of the purchase price by May 31, 1979 and alleged that they only
include his nephews and grandnephews. Victoriana Cari-an was succeeded by received a total of P132,551.00 in cash and goods.
her late brothers son, Gregorio Cari-an. Gregorio was declared as Victorianas 9. Petitioners replied that the Cari-ans, having been paid, had no right to resell the
heir in the estate proceedings for Nombre and his wife. subject lots; that the Chuas were purchasers in bad faith; and that the court
2. After Gregorio died, his wife, Generosa Martinez, and children, Rodolfo, approval of the sale to the Chuas was subject to their existing claim over said
Carmen, Leonardo and Fredisminda, all surnamed Cari-an, were also adjudged properties.
as heirs by representation to Victorianas estate. Leonardo Cari-an passed away, 10. Escanlar and Holgado also sold their rights and interests in the 2 lots to the
leaving his widow, Nelly Chua vda. de Cari-an and minor son Leonell, as his Jaymes for P735,000.00 and turned over possession of both lots to the
heirs. (heirs of Cari-an) latter. The Jaymes in turn, were included in the civil case as fourth-party
3. Two parcels of land of the Kabankalan Cadastre formed part of the estate of defendants.
Nombre and Cari-an. 11. The probate court approved the sale without prejudice to whatever rights,
claims and interests over any of those properties of the estate which cannot be
properly and legally ventilated and resolved by the court in the same intestate the rates specified above with legal interest from date of demand.
proceedings.
12. The certificates of title over the eight lots sold by the heirs of Nombre and Cari- RATIO:
an were later issued in the name of Spouses Chua. The stipulation only affected the effectivity and not the validity of the sale*
13. Escanlar and Holgado alleged that the Cari-ans conspired with the Chuas when 1. There has arisen here a confusion in the concepts of validity and the efficacy of
they executed the second sale on and that the latter sale is illegal and of no a contract. The essential requisites of a contract (consent, subject matter, price)
effect. Spouses Chua countered that they did not know of the earlier sale of were all present, the result is a valid contract.
one-half portion of the subject lots to Escanlar and Holgado. 2. However, some parties introduce various kinds of restrictions or modalities, the
14. Trial court approved Spouses Chua’s motion to file a fourth-party complaint lack of which will not, however, affect the validity of the contract.
against the spouses Jayme. Spouses Chua alleged that the Jaymes refused to 3. In this case, the Deed of Sale, the essential requisites being complete, is a valid
vacate said lots despite repeated demands; and that by reason of the illegal one. However, it did not bear the stamp of approval of the court. Only the
occupation of 2 Lots by the Jaymes, they suffered from uncollected rentals. effectivity and not the validity of the contract is affected.
15. RTC of Himamaylan held that the properties of the estate of Guillermo Nombre 4. Heirs of Escanlar are correct in saying that the need for approval by the
and Victoriana Cari-an had long been disposed of by the rightful heirs of probate court exists only where specific properties of the estate are sold and
Guillermo Nombre and Victoriana Cari-an. not when only ideal and indivisible shares of an heir are disposed of.
16. Therefore, there is no need to resolve the Motion for Subrogation of Movants 5. When Cari-ans sold their rights, interests and participation in the 2 lots, they
Escanlar and Holgado to be subrogated to the rights of the heirs of Victoriana could legally sell the same without the approval of the probate court.
Cari-an since all the properties of the estate had been transferred and titled to 6. As a general rule, the pertinent contractual stipulation (requiring court approval)
in the name of Spouses Chua. should be considered as the law between the parties. However, the presence of
17. The seminal case at bar was resolved by the trial court in favor of cancellation two factors militate against this conclusion.
of the sale to Escanlar and Holgado. Said transaction was nullified because it a. First, the evident intention of the parties appears to be contrary to the
was not approved by the probate court as required by the contested deed mandatory character of said stipulation. Receipt and acceptance of the
of sale of rights, interests and participation and because the Cari-ans were numerous installments on the balance of the purchase price by the Cari-ans
not fully paid. Sale to Spouses Chua was upheld. and leaving petitioners in possession of the 2 lots reveal their intention to
18. Escanlar and Holgado raised the case to the CA and it affirmed the decision of effect the mutual transmission of rights and obligations. It was only after
the trial court and held that the questioned deed of sale of rights, interests private respondents Cari-an sold their shares in the subject lots again to the
and participation is a contract to sell because it shall become effective only spouses Chua that these same heirs filed the case at bar for the cancellation
upon approval by the probate court and upon full payment of the purchase price. of the conveyance to petitioners. This reveals the original objective of the
parties to give effect to the deed of sale even without court approval.
ISSUE: b. Second, we hold that the requisite approval was virtually rendered impossible
1. WoN the Contract of Sale was invalid because of the lack of approval by the CFI by the Cari-ans because they opposed the motion for approval of the sale filed
of Negros Occidental? NO, it merely affected the effectivity of sale [ISSUE IN by petitioners and sued them for the cancellation of that sale.
SYLLABUS] 7. Having provided the obstacle and the justification for the stipulated
2. WoN the Deed of Sale of Rights, Interests and Participation is a contract to sell? approval not to be granted, private respondents Cari-an should not be allowed to
NO, it was a SALE cancel their first transaction with petitioners because of lack of approval by the
probate court, which lack is of their own making
RULING: WHEREFORE, the petitions are hereby GRANTED. The decision of the
Court of Appeals under review is hereby REVERSED AND SET ASIDE. The case is Failure to pay is not a breach of contract
REMANDED to the Regional Trial Court of Negros Occidental, Branch 61 for 8. SC disagrees with the CA, stating that in contracts to sell, ownership is retained
petitioners and private respondents Cari-an or their successors-in-interest to determine by the seller and is not passed until the full payment of the price. Such payment
exactly which 1/2 portion of Lot Nos. 1616 and 1617 will be owned by each party, at is a positive suspensive condition, the failure of which is not a breach of contract
the option of petitioners. The trial court is DIRECTED to order the issuance of the but simply an event that prevented the obligation of the vendor to convey title
corresponding certificates of title in the name of the respective parties and to resolve from acquiring binding force.
the matter of rental payments of the land not delivered to the Chua spouses subject to
9. Although a deed of conditional sale is denominated as such, absent a proviso that 16. However, despite all her claims, Fredismindas testimony fails to convince this
title to the property sold is reserved in the vendor until full payment of the Court that they were not fully compensated by petitioners. Fredisminda admits
purchase price nor a stipulation giving the vendor the right to unilaterally rescind that her mother and her sister signed their individual receipts of full payment on
the contract the moment the vendee fails to pay within a fixed period, by its their own and not in her presence. The receipts presented in evidence show that
nature, it shall be declared a deed of absolute sale the rest of the heirs were paid by Escanlar. Court thus finds it incredible that a
10. The Deed of sale of rights, interests and participation to Escanlar and Hogaldo as mature woman like Fredisminda Cari-an, would sign a receipt for money she did
to 1/2 portion pro indiviso of the two subject lots is a contract of sale for the not receive. Worth noting too is the absence of supporting testimony from her
following reasons: co-heirs and siblings Carmen Cari-an, Rodolfo Cari-an and Nelly Chua vda.
a. First, private respondents as sellers did not reserve unto themselves the de Cari-an.
ownership of the property until full payment of the unpaid balance
b. Second, there is no stipulation giving the sellers the right to unilaterally rescind Continuing to pay the rent does not mean they have not fully paid Cari-ans
the contract the moment the buyer fails to pay within the fixed period. Upon sale 17. The trial court reasoned out that petitioners, in continuing to pay the rent for the
to them of the rights, interests and participation as to the 1/2 portion pro indiviso, parcels of land they allegedly bought, admit not having fully paid the Cari-ans.
they remained in possession, not in concept of lessees anymore but as owners Petitioners response, that they paid rent until 1986 in compliance with their lease
now through symbolic delivery contract, only proves that they respected this contract and did not take undue
c. Under Article 1477 of the Civil Code, the ownership of the thing sold is acquired advantage of the heirs of Nombre and Cari-an who benefited from the
by the vendee upon actual or constructive delivery thereof. lease. Moreover, it is to be stressed that petitioners purchased the hereditary
11. In a contract of sale, the non-payment of the price is a resolutory condition which shares solely of the Cari-ans and not the entire lot.
extinguishes the transaction that, for a time, existed and discharges the 18. The foregoing discussion ineluctably leads us to conclude that the Cari-ans were
obligations created. The remedy of an unpaid seller in a contract of sale is to seek indeed paid the balance of the purchase price, despite having accepted
either specific performance or rescission. installments therefor belatedly. There is thus no ground to rescind the contract of
sale because of non-payment.
Vendee may pay, even after the expiration of the period, as long as no demand for
rescission of the contract has been made Division of the lots
12. With respect to rescission of a sale of real property, Article 1592 of the Civil 19. It must be emphasized that what was sold was only the Cari-ans hereditary shares
Code governs, “the vendee may pay, even after the expiration of the period, as in 2 lots being held pro indiviso by them and is thus a valid conveyance only of
long as no demand for rescission of the contract has been made upon him either said ideal shares. Specific or designated portions of land were not involved.
judicially or by a notarial act. After the demand, the court may not grant him a 20. Consequently, the subsequent sale of 8 parcels of land, including 2 subject lots,
new term.” to the spouses Chua is valid except to the extent of what was sold to Escanlar. It
13. In the instant case, the sellers gave the buyers until May 1979 to pay the balance must be noted however, that the probate court in Special Proceeding No. 7-7279
of the purchase price. After the latter failed to pay installments due, the former desisted from awarding the individual shares of each heir because all the
made no judicial demand for rescission of the contract nor did they execute any properties belonging to the estate had already been sold. Thus it is not certain
notarial act demanding the same, as required under Article 1592. how much private respondents Cari-an were entitled to with respect to the two
14. Upon the expiration of the period to pay, the sellers made no move to rescind but lots, or if they were even going to be awarded shares in said lots.
continued accepting late payments, an act which cannot but be construed as a 21. The proceedings surrounding the estate of Nombre and Cari-an having attained
waiver of the right to rescind. They were thus estopped. finality for nearly a decade now, the same cannot be re-opened. The protracted
15. The sole witness in the cancellation of sale case was private respondent herein proceedings which have undoubtedly left the property under a cloud and the
Fredisminda Cari-an Bustamante. She claims that Escanlar and Holgado made parties involved in a state of uncertainty compels us to resolve it definitively.
her sign the receipt of the P5,000 late in the afternoon and promised to give the 22. The decision of the probate court declares private respondents Cari-an as the sole
money to her the following morning when the banks opened but never received heirs by representation of Victoriana Cari-an who was indisputably entitled to
them. She also claimed that the total of all the installments of amounts paid with half of the estate. There being no exact apportionment of the shares of each heir
the rest of the Cari-ans is P132,551. She thus claimed that they were defrauded and no competent proof that the heirs received unequal shares in the disposition
because petitioners are wealthy and private respondents are poor. of the estate, it can be assumed that the heirs of Victoriana Cari-an collectively
are entitled to half of each property in the estate. More particularly, private
respondents Cari-an are entitled to half of the 2 subject lots.
23. Consequently, Heirs of Escanlar, as their successors-in-interest, own said half of
the subject lots and ought to deliver the possession of the other half, as well as
pay rents thereon, to the Spouses Chua but only if the petitioners remained in
possession thereof
03 Power Commercial and Industrial Corporation vs CA (Mel) 3. The parties agreed that Power Commercial would pay Spouses Quiambao
June 20, 1997 | Panganiban, J.| Conditions vs Warranties P108,000.00 as down payment, and the balance of P295,000.00 upon the
execution of the deed of transfer of the title over the property. Further, Power
PETITIONER: Power Commercial and Industrial Corporation Commercial assumed, as part of the purchase price, the existing mortgage on
RESPONDENTS-APPELLEE: Court of appeals, Spouses Reynaldo and the land. In full satisfaction thereof, Power Commercial paid P79,145.77 to
Angelita R. Quiambao and Philippine National Bank Respondent PNB.
4. Spouses Quiambao mortgaged again said land to PNB to guarantee a loan of
SUMMARY: Power Commercial, an industrial asbestos manufacturer, needed P145,000.00, P80,000.00 of which was paid to Spouses Quiambao. Power
a bigger office space and warehouse for its products. So it entered into a contract Commercial agreed to assume payment of the loan.
of sale with Spouses Quiambao. Power Commercial agreed to assume the 5. The parties executed a Deed of Absolute Sale With Assumption of Mortgage
mortgages thereon. A Deed of Absolute Sale with Assumption of Mortgage was (see end of digest for the terms and conditions). On the same date, Mrs. C.D.
executed. Power Commercial failed to settle the mortgage debt contracted by the Constantino, then General Manager of Power Commercial, submitted to PNB
spouses, thus it could not undertake the proper action to evict the lessees on the said deed with a formal application for assumption of mortgage.
lot. Power Commercial Corp thereafter sought to rescind the contract of sale 6. PNB informed Spouses Quiambao that, for Power Commercial’s failure to
alleging that it failed to take actual and physical possession of the lot. The trial submit the papers necessary for approval, the application for assumption of
court ruled that the failure of Spouses Quiambao to deliver actual possession to mortgage was considered withdrawn; that the outstanding balance of
Power Commercial entitled the latter to rescind the sale. On appeal, the Court of P145,000.00 was deemed fully due and demandable; and that said loan was
Appeals reversed the trial court. It held that the deed of sale between Spouses to be paid in full within 15 days from notice.
Quiambao and Power Commercial did not obligate the Spouses Quiambao to 7. Power Commercial paid PNB P41,880.45 on June 24, 1980 and P20,283.14
eject the lessees from the land in question as a condition of the sale, nor was the on December 23, 1980, payments which were to be applied to the outstanding
occupation thereof by said lessees a violation of the warranty against eviction. loan.
Hence, there was no substantial breach to justify the rescission of said contract 8. PNB received a letter from Power Commercial which said that they desired
or the return of the payments made. The issues are WON there was an imposed to make use of the lot for their own purpose and requested that the assumption
condition? No. First, such a condition that the Quiambao spouses would evict of mortgage be given favorable consideration, and that the mortgage and title
the lessees therein was not stipulated in the contract. In fact, Power Commercial be transferred to their name
was well aware of the presence of the tenants therein. Also in this case, Power 9. PNB replied that there was lack of payment and in order to place their account
Commercial was given control over the said lot and it endeavored to terminate in current form, Power Commercial had to remit payments to cover interest,
the occupation of the actual tenants. (2) WON there was a breach of warranty charges and at least part of the principal.
against eviction? – NO. The facts of the case do not show the requisites for a 10. Later on, Power Commercial filed a case against Spouses Quiambao for
breach of warranty. (check ruling for other issues) rescission and damages. Power Commercial also demanded from PNB the
return of the payments it made as the assumption of mortgage was never
DOCTRINE: (from book) There can be no action for breach of the said warranty approved. While the case was pending, the mortgage was foreclosed. The
when the buyer was well aware of the presence of the tenants at the time the property was bought by PNB during the public auction. Thus, an amended
buyer entered into the sale transaction, and it even undertook the job of ejecting complaint was filed impleading PNB as party defendant.
the squatters which in fact filed suit to eject the occupants. 11. The trial court ruled that the failure of Spouses Quiambao to deliver actual
possession to Power Commercial entitled the latter to rescind the sale, and in
FACTS: view of such failure and of the denial of the latter's assumption of mortgage,
1. Petitioner Power Commercial & Industrial Development Corporation (Power PNB was obliged to return the payments made by the latter.
Commercial), an industrial asbestos manufacturer, needed a bigger office 12. On appeal, the Court of Appeals reversed the trial court. It held that the deed
space and warehouse for its products. For this purpose, it entered into a of sale between Spouses Quiambao and Power Commercial did not obligate
contract of sale with the spouses Reynaldo and Angelita R. Quiambao, herein the former to eject the lessees from the land in question as a condition of the
private respondents (Spouses Quiambao). sale, nor was the occupation thereof by said lessees a violation of the warranty
2. The contract involved a 612-sq. m. parcel of land located at the corner of against eviction. Hence, there was no substantial breach to justify the
Bagtican and St. Paul Streets, San Antonio Village, Makati City. rescission of said contract or the return of the payments made.
ISSUES: stipulation therefor, we cannot say that the parties intended to make its
1. (RELATED ISSUE) WON there was an imposed condition? – No. nonfulfillment a ground for rescission. If they did intend this, their
2. WON there was delivery? – Yes. contract should have expressly stipulated so.
3. (RELATED ISSUE) WON there was a breach of warranty against eviction?
– No. Effective Symbolic Delivery
4. WON there was a mistake in payment? –No. 1. The Court disagrees with Power Commercial's allegation that the Spouses
Quiambao failed to deliver the lot sold. Power Commercial asserts that the
RULING: WHEREFORE, the petition is hereby DENIED, and the assailed Decision legal fiction of symbolic delivery yielded to the truth that, at the execution of
is AFFIRMED. the deed of sale, transfer of possession of said lot was impossible due to the
presence of occupants on the lot sold. We find this misleading.
RATIO: 2. Although most authorities consider transfer of ownership as the primary
Conspicuous Absence of an Imposed Condition purpose of sale, delivery remains an indispensable requisite as our law does
1. The petition is devoid of merit. It fails to appreciate the difference between a not admit the doctrine of transfer of property by mere consent.
condition and a warranty and the consequences of such distinction. 3. The Civil Code provides that delivery can either be (1) actual (2)
2. The alleged "failure" of Spouses Quiambao to eject the lessees from the constructive. Symbolic delivery as a species of constructive delivery, effects
lot in question and to deliver actual and physical possession thereof the transfer of ownership through the execution of a public document. Its
cannot be considered a substantial breach of a condition for two reasons: efficacy can, however, be prevented if the vendor does not possess control
first, such "failure" was not stipulated as a condition — whether over the thing sold, in which case this legal fiction must yield to reality.
resolutory or suspensive — in the contract; and second, its effects and 4. The key word is control, not possession, of the land as Power Commercial
consequences were not specified either. would like us to believe.
3. The provision adverted to by Power Commercial does not impose a condition 5. Considering that the deed of sale between the parties did not stipulate or infer
or an obligation to eject the lessees from the lot. The deed of sale provides in otherwise, delivery was effected through the execution of said deed.
part: We hereby also warrant that we are the lawful and absolute owners of 6. The lot sold had been placed under the control of Power Commercialr;
the above described property, free from any lien and/or encumbrance, and thus, the filing of the ejectment suit was subsequently done. It signified
we hereby agree and warrant to defend its title and peaceful possession that its new owner intended to obtain for itself and to terminate said
thereof in favor of the said Power Commercial and Industrial Development occupants' actual possession thereof.
Corporation, its successors and assigns, against any claims whatsoever of 7. Prior physical delivery or possession is not legally required and the execution
any and all third persons; subject, however, to the provisions hereunder of the deed of sale is deemed equivalent to delivery. This deed operates as a
provided to wit… formal or symbolic delivery of the property sold and authorizes the buyer to
4. By his own admission, Anthony Powers, General Manager of the Power use the document as proof of ownership. Nothing more is required.
Commercial, did not ask the corporation's lawyers to stipulate in the contract
that Reynaldo Quiambao was guaranteeing the ejectment of the occupants, There was no Breach of Warranty.
because there was already a proviso in said deed of sale that the sellers were 1. Despite its protestation that its acquisition of the lot was to enable it to set up
guaranteeing the peaceful possession by the buyer of the land in question. a warehouse for its asbestos products and that failure to deliver actual
Any obscurity in a contract, if the above-quoted provision can be so possession thereof defeated this purpose, still no breach of warranty
described, must be construed against the party who caused it. Power against eviction can be appreciated because the facts of the case do not
Commercial itself caused the obscurity because it omitted this alleged show that the requisites for such breach have been satisfied.
condition when its lawyer drafted said contract. 2. A breach of this warranty requires the concurrence of the following
5. If the parties intended to impose on Spouses Quiambao the obligation to eject circumstances: (1) purchaser has been deprived of the whole or part of the
the tenants from the lot sold, it should have included in the contract a thing sold;
(2) This eviction is by a final judgment;
(3) The basis thereof is
provision similar to that referred to in Romero vs. Court of Appeals, where by virtue of a right prior to the sale made by the vendor; and
(4) The vendor
the ejectment of the occupants of the lot sold by private respondent was the has been summoned and made co-defendant in the suit for eviction at the
operative act which set into motion the period of petitioner's compliance with instance of the vendee.
his own obligation, i.e., to pay the balance of the purchase price. Absent a 3. Power Commercial argues in its memorandum that it has not yet ejected the
occupants of said lot, and not that it has been evicted therefrom. As correctly with offices located at 252-C Vito Cruz Extension, we hereby by these presents SELL,
pointed out by Respondent Court, the presence of lessees does not constitute TRANSFER and CONVEY by way of absolute sale the above described property with
an encumbrance of the land, nor does it deprive Power Commercial of its all the improvements existing thereon unto the said Power Commercial and Industrial
control thereof. Development Corporation, its successors and assigns, free from all liens and
4. Power Commercial's deprivation of ownership and control finally occurred encumbrances.
when it failed and/or discontinued paying the amortizations on the mortgage,
causing the lot to be foreclosed and sold at public auction. But this We hereby certify that the aforesaid property is not subject to nor covered by the
deprivation is due to it’s own fault, and not to any act attributable to the provisions of the Land Reform Code — the same having no agricultural lessee and/or
vendor-spouses. tenant.
5. Because Power Commercial failed to impugn its integrity, the contract is
presumed, under the law, to be valid and subsisting. We hereby also warrant that we are the lawful and absolute owners of the above
described property, free from any lien and/or encumbrance, and we hereby agree and
Absence of Mistake in Payment warrant to defend its title and peaceful possession thereof in favor of the said Power
1. Solutio indebiti does not apply in this case. Commercial and Industrial Development Corporation, its successors and assigns,
2. This doctrine applies where: (1) a payment is made when there exists no against any claims whatsoever of any and all third persons; subject, however, to the
binding relation between the payor, who has no duty to pay, and the person provisions hereunder provided to wit:
who received the payment, and (2) the payment is made through mistake, and
not through liberality or some other cause. That the above described property is mortgaged to the Philippine National Bank,
3. In this case, Power commercial was under obligation to pay the amortizations Cubao, Branch, Quezon City for the amount of one hundred forty-five thousand pesos,
on the mortgage under the contract of sale and the deed of real estate Philippine, evidenced by document No. 163, found on page No. 34 of Book No. XV,
mortgage. Under the deed of sale, both parties agreed to abide by any and all Series of 1979 of Notary Public Herita
L. Altamirano registered with the Register of
the requirements of PNB in connection with the real estate mortgage. Power Deeds of Pasig (Makati), Rizal . . . ;
Commercial was aware that the deed of mortgage made it solidarily and,
therefore, primarily liable for the mortgage obligation:
4. Therefore, it cannot be said that it did not have a duty to pay to PNB the
amortization on the mortgage.
5. Also, Power Commercial insists that its payment of the amortization was a
mistake because PNB disapproved its assumption of mortgage after it failed
to submit the necessary papers for the approval of such assumption.
6. But even if Power Commercial was a third party in regard to the mortgage of
the land purchased, the payment of the loan by Power Commercialwas a
condition clearly imposed by the contract of sale. This fact alone disproves
Power Commercial's insistence that there was a "mistake" in payment. On the
contrary, such payments were necessary to protect its interest as a "the buyers
and new owners of the lot."

DEED OF ABSOLUTE SALE terms and conditions

That for and in consideration of the sum of Two Hundred Ninety-Five Thousand Pesos
(P295,000.00) Philippine Currency, to us in hand paid in cash, and which we hereby
acknowledge to be payment in full and received to our entire satisfaction, by POWER
COMMERCIAL AND INDUSTRIAL DEVELOPMENT CORPORATION, a 100%
Filipino Corporation, organized and existing under and by virtue of Philippine Laws
04 GUINHAWA v. PEOPLE (Paul) 2. In 1995, Guinhawa purchased a brand new Mitsubishi L-300 Versa Van from
Aug. 25, ’05 | Callejo, Sr., J. | Caveat Emptor does not apply if there is deceit Union Motors Corp (UMC).
3. While the van was being driven from Manila to Naga by Guinhawa’s driver,
Leopoldo Olayan, the latter suffered a heart attack.
PETITIONER: Jaime Guinhawa 4. As a result, the van was damaged, and the left front tire had to be replaced.
RESPONDENTS: People of the Philippines 5. The van was then repaired and offered for sale in Guinhawa’s showroom.
6. A few months after, spouses Ralph and Josephine Silo, wanted to buy a van
SUMMARY: Guinhawa was engaged in the business of selling brand new cars under
the name Guinrox Motor Sales, and Azotea is the General Manager. In 1995, he bought
for their garment business. They went to Guinhawa’s showroom and saw the
a Mitsubishi L-300 from UMC. As his driver was using the van, the driver suffered a same L-300 van.
heart attack, and he met the accident. As a result, the car was damaged, and it had to be 7. The couple inspected the interior and liked it. They no longer inspected the
repaired. After the repair of the van, Guinhawa put it in the showroom for selling. A chases since they presumed that the vehicle was brand new. Unaware that
few months after, spouses Silo bought the van because they needed one for their the van had been damaged, and repaired due to an accident, they made a
garments business. Convinced that the van was brand new, they only inspected mostly downpayment of Php 118,000. The balance was to be paid by United Coconut
the interior. They immediately bought the van, as sold to them by Azotea. The spouses, Planters Bank (UCPB) on installments, with the van as collateral.
accompanied by spouses Pingol went to Manila using the van. When they were going 8. The deed of sale was subsequently furnished and the Spouses were given a
back to Naga, Bayani Pingol heard a squeaking sound coming from below the van. receipt and a service manual which contained the warranty terms and
Upon inspection, he did not see any defect, but because the squeaking sound persisted, conditions.
he brought the car to Shell, thinking that it only needed grease. The mechanic of Shell, 9. Ralph Silo did not conduct a test drive, because again, they assumed that the
upon inspection, said that some of the parts under the car were only welded. Spouses van was new.
Silo wanted the van to be exchanged for 2 Charade-Daihatsu vehicles but Guinhawa 10. After a few days, Josephine Silo, accompanied by spouses Glenda and Bayani
did not agree. Josephine Silo then filed a complaint with the Prosecutor for a violation Pingol, went to Manila using the van. On their way back to Naga, Bayani
of Art. 318 (other deceits). Guinhawa now argues that he should not be convicted heard a squeaking sound which was coming from underneath the van. When
because of the principle of caveat emptor, and therefore, the Silo spouses should bear Bayani examined the van, he didn’t find any defect.
the loss of their negligence. The Supreme Court ruled in favor of Silo spouses and said 11. Believing that the van only needed grease, they brought it to Shell and they
that the principle of caveat emptor does not apply when the seller deceives the buyer. were informed by the mechanic that some parts underneath were just welded.
In a contract of sale, if the vendor knowingly allowed the vendee to be deceived as to
12. Because of this, Silo requested Guinhawa to just change the van with 2
the thing sold in a material manner by failing to disclose an intrinsic circumstance
that is vital to the contract, knowing that the vendee is acting upon the presumption
Charade-Daihatsu vehicles. The spouses also stopped paying the
that NO FACT EXISTS, deceit is accomplished by suppression of the truth. amortization on the loan, pending the replacement of the van.
Clearly, Guinhawa and Azotea had the duty to disclose to the spouses that what they 13. Guinhawa initially agreed to their proposal but he changed his mind and told
were buying a only a repaired car, especially, because such belief that the car was new, them that he wanted to sell the van first.
was the compelling reason why they bought the car in the first place. Moreover, since 14. Josephine Silo then filed for rescission and refund before the DTI. But when
the spouses were only experts in the garments industry, they were not expected to know she confronted Guinhawa, she found out that the latter bought the van from
the details of a car to the extent that Guinhawa argues as regards the repaired parts. UMC, and that it was involved in a car accident. The complaint in DTI was
withdrawn.
DOCTRINE: Jurisprudence has held that the principle of caveat emptor only requires 15. Josephine then filed a criminal complaint for par. 1, Art. 318 of the RPC
that the purchaser exercises such care and attention that is usually exercised by (OTHER DECEITS).
ordinarily prudent men in like business affairs, and only applies to defects which 16. Guinhawa was arguing that he did not use the van, it was brand new, and that
are open and patent to the service of one exercising such care. it was not involved in any accident.
17. He said that Azotea informed him that spouses Silo had complained about
alleged defects but the van already had a kilometer reading of 4,000 km. He
FACTS (This is the same case we read for SPL):
allegedly made no false statement of fraudulent misrepresentation. He also
1. Jaime Guinhawa was engaged in the business of selling brand new motor denied meeting Bayani Pingol.
vehicles under the business name of Guinrox Motor Sales; and Gil Azotea is
18. Azotea also alleged that the buyers inspected the car, and even drove it.
his manager. 19. RTC found Guinhawa guilty. It held that Guinhawa made false pretenses and
misrepresentations that the van was brand new when in fact, it had been which he caused.
damaged in an accident. 9. Jurisprudence has held that the principle of caveat emptor only requires that
20. The CA affirmed the decision and held that spouses had the right to assume the purchaser exercises such care and attention that is usually exercised by ordinarily
that the van was brand new because Guinhawa held himself to be a dealer prudent men in like business affairs, and only applies to defects which are open
of brand new vans. The act of displaying the van in the showroom without and patent to the service of one exercising such care.
notice that it was not brand new was tantamount to deceit. 10. The principle can only be applied where it is shown or conceded that the
parties to the contract stand on equal footing and have equal knowledge or equal means
ISSUES: of knowledge, and there is no relation of trust or confidence between them.
1. W/N the doctrine of caveat emptor applies, and therefore Guinhawa 11. BUT, where one party undertakes to sell to another property situated at a
should be acquitted? - No, because Guinhawa is guilty of deceit distance of which he has or claims to have personal knowledge, and of which buyer
knows nothing except as he is informed by the seller, the buyer may rightfully rely
RULING: IN LIGHT OF ALL THE FOREGOING, the petition is DENIED. The on the truth of the seller’s representations as to its kind, quality, and value made in
assailed Decision and Resolution are AFFIRMED WITH MODIFICATION. the courts of negotiation for the purpose of inducing the purchase. If, in such case, the
Considering the surrounding circumstances of the case, the petitioner is hereby representations prove to be false, neither law nor equity will permit the seller to escape
sentenced to suffer a straight penalty of six (6) months imprisonment. The petitioner liability.
shall suffer subsidiary imprisonment in case of insolvency. 12. Applying these doctrines to the case at bar, Azotea and Guinhawa had every
opportunity to reveal to private complainant that the van was defective. They chose to
RATIO: stay silent, and this caused prejudice to spouses Silo, who were garment merchants
THE ONLY RELEVANT PORTION: CAVEAT EMPTOR DOES NOT APPLY who had no special parts of motor vehicles.
1. Art. 1389 of the NCC provides that failure to disclose facts when there is a 1. Personal reflection: So assuming that the Silo spouses were car collectors
duty to reveal them constitutes fraud. instead, it seems as if SC MIGHT HAVE ruled otherwise.
2. In a contract of sale, a buyer and seller do not deal from equal bargaining 13. Based on these surrounding circumstances, Josephine Silo relied on her belief that
positions WHEN the latter has knowledge of a material fact which, if the van was new.
communicated to the buyer, would render the grounds unacceptable, or, at least,
substantially less desirable. OTHER ISSUES (I doubt sir will ask these but read away if you wanna make sure)
3. In a contract of sale, if the vendor knowingly allowed the vendee to be GUINHAWA, LIABLE FOR ACTS OF AZOTEA
deceived as to the thing sold in a material manner by failing to disclose an intrinsic 1. Guinhawa argues that he was not the one who dealt with the buyers, but Azotea, his
circumstance that is vital to the contract, knowing that the vendee is acting upon the general manager.
presumption that NO FACT EXISTS, deceit is accomplished by suppression of 2. It is well settled that the act or transaction of a given affair, when confided to an
the truth. agent, the authority to so act will, in accordance with a general rule often referred
4. Here, Guinhawa knew that the van had been damaged in an accident, and was to, carry with it, by implication, the authority to do all of the collateral acts which
subsequently repaired. Despite this, the van was placed in the showroom, making it are the natural, ordinary incidents of the main act or business authorized.
appear that it was a brand new unit. Based on Art. 1389, Guinhawa was mandated
to reveal the facts to spouses Silo. THE CRIME IS UNDER ART. 318, NOT ART. 315 PAR. 2(A) OF THE RPC
5. But, even in court, Guinhawa and Azotea declared that the vehicle was brand 1. The elements of other deceits are as follows:
new to spouses Silo. a. False pretense, fraudulent act or pretense other than those in the preceding
6. Guinhawa is not relived of criminal liability even if spouses Silo made a articles;
visual inspection of the van before they agreed to buy it, but they just failed to b. Such false pretense, fraudulent act, or pretense must be made or executed prior
inspect the chasis. to or simultaneously with the commission of the fraud;
7. In Burnett v. Boyer, it has been held that when the vendee made only a partial c. As a result, the offended party suffered damage or prejudice
investigation and relies, in part, upon representation of the vendee, and is deceived 2. Guinhawa argues that he did not commit any false pretense.
by such representation, he may maintain an action for such deceit. 3. First, as mentioned, Azotea in open court, declared that the car was brand new.
8. The seller cannot say that the vendee should not have relied on the fraudulent 4. Assuming that there was no express statement that the car was indeed new, the SC
concealment. Negligence is not a defense to unjustly enrich the vendor from the fraud held that fraud or deceit may be committed by omission.
5. As mentioned as well, fraudulent concealment implies a purpose or design to hide
facts which the other party sought to know. Again, the act of putting the car in the
showroom, with the representation that Guinhawa deals brand new cars in general,
is enough to be punishable under this article.
6. As the Rules of Criminal Procedure says, the information alleges acts or omissions
complained of, and it is not the designation or title given by the prosecutor. Such
allegations are clearly under Art. 318.
7. The jurisdiction is also with MTC, not the RTC as the petitioner argues. The crime
is punishable by arresto mayor - within the exclusive jurisdiction of the MTC
because less than 6 years of imprisonment.
05 ANG v. CA (Eliel) 12. Ang, a buyer and seller of used vehicles, later offered the Mitsubishi GSR for
September 29, 2008 | Carpio-Morales, J. | Warranty against Eviction sale through Far Eastern Motors, a second-hand auto display center. The
vehicle was eventually sold to a certain Paul Bugash for P225,000 by a Deed
PETITIONER: Jaime D. Ang of Absolute Sale.
RESPONDENTS: CA and Bruno Soledad 13. Before the deed could be registered in Bugash’s name, the vehicle was seized
by virtue of a writ of replevin, on account of the alleged failure of Ronaldo
SUMMARY: Ang and Soledad entered into a car swapping agreement. Ang, a Panes, the owner of the vehicle prior to Soledad, to pay the mortgage debt
buyer and seller of used vehicles, offer the Mitsubishi GSR for sale through Far constituted thereon.
Eastern Motors, and was eventually sold to Bugash. Before the car was registered 14. To secure the release of the vehicle, Ang paid BA Finance the amount of
to Bugash, a writ of replevin seized the vehicle on account of the failure to pay of P62,038.47. Soleded refused to reimburse the said amount, despite repeated
Panes, the previous owner prior to Soledad. In order to release the vehicle from demands, drawing Ang to charge him for Estafa with abuse of confidence
the writ, Ang paid BA Finance. Thereafter, Ang demanded recovery from before the Office of the City Prosecutor, Cebu City. But was dismissed for
Soledad in the amount of P62,038.47. But Soledad refused, and so Ang filed a insufficience of evidence.
complaint for damages in the MTCC, which dismissed the complaint of Ang 15. Ang thereafter sought for complaint of damages in the MTCC, which was
because of prescription. The RTC likewise affirmed the MTCC decision, but dismissed on the ground of prescription, alleging that the period for filing
granted reimbursement based on Unjust Enrichment. On appeal to the CA, it should have been within 6 months in cases of warranty.
reversed the RTC’s decision regarding the reimbursement for unjust enrichment. 16. Ang appealed to the RTC, declared that the period has not prescribed, because
Hence this petition. Ang contends that the warranty is expressed and has not yet it was an express warranty (general law on prescription- since written
prescribed. contract 10 years) and not an implied warranty (6 months). However, the
RTC dismissed the complaint for damages because it lacked the requirements
The SC maintains that the warranty has already prescribed. The provision in the to recover under the warranty. Nevertheless, Soleded was required to pay
Deed of Absolute Sale (Ratio #8), stating that the title is free from liens and based on Unjust Enrichment.
encumbrances, and that Soledad will defend such from any claims is an implied 17. Soledad then appealed to the CA, which reversed the decision of the RTC on
warranty, rather a warranty against eviction. And such action prescribed 6 months the ground of prescription, and it was not Soledad who was unjustly enriched.
after the delivery of the vehicle. Clearly, it was more than 6 months when Ang 18. Hence this petition.
filed since the deed was executed in 1992 and the complaint in 1995. Moreover,
given the fact that the action has not yet prescribed, the requirements for recover ISSUES:
under warranty against eviction (Ratio #12) were not met, and thus Ang has no 2. WoN the warranty is express and has not prescribed – NO
action. With regard to the unjust enrichment, it was shown that Soledad was not
actually the one who benefitted from the payment of Ang to BA Finance, instead RULING: WHEREFORE, the petition is, in light of the foregoing disquisition,
it was Ang’s own volition in order to secure the conveyance of the vehicle to DENIED.
Bugash.
RATIO:
DOCTRINE: The seller, in declaring that he owned and had clean title to the 28. A warranty is a statement or representation made by the seller of goods,
vehicle, gave an implied warranty of title, and in pledging that he “will defend contemporaneously and as part of the contract of sale, having reference
the same from all claims or any claim whatsoever [and] will save the vendee to the character, quality or title of the goods, and by which he promises
from any suit by the government of the Republic of the Philippines,” he gave or undertakes to insure that certain facts are or shall be as he then
a warranty against eviction, and the prescriptive period to file a breach represents them.
thereof is six months after the delivery of the vehicle 29. Warranties by the seller may be express or implied.
30. Art. 1546. Any affirmation of fact or any promise by the seller relating
FACTS: to the thing is an express warranty if the natural tendency of such
11. Under a car-swapping scheme, Soledad sold his Mitsubishi GSR sedan 1982 affirmation or promise is to induce the buyer to purchase the same, and
model to Ang by Deed of Absolute Sale. Ang in return, conveyed his if the buyer purchases the thing relying thereon. No affirmation of the
Mitsubishi Lancer model 1988, also by Deed of Absolute Sale. As Ang’s car value of the thing, nor any statement purporting to be a statement of the
was of a later model, Soledad paid him an additional P55,000. seller’s opinion only, shall be construed as a warranty, unless the seller made
such affirmation or statement as an expert and it was relied upon by the buyer. c. The basis thereof is by virtue of a right prior to the sale made by
31. On the other hand, an implied warranty is that which the law derives by the vendor; and
application or inference from the nature of the transaction or the relative d. The vendor has been summoned and made co-defendant in the
situation or circumstances of the parties, irrespective of any intention of suit for eviction at the instance of the vendee
the seller to create it. e. In the absence of these requisites, a breach of the warranty agasint
32. Among the implied warranty provision of the Civil Code are: as to the seller’s eviction under Art. 1547 cannot be declared.
title (Art. 1548), against hidden defects and encumbrances (Art. 1561), as to 40. For one, there is no judgment which deprived Ang of the vehicle. For another,
fitness or merchantability (Art. 1562), and against eviction (Art. 1548). there was no suit for eviction in which Soledad as seller was impleaded as
33. Engineering & Machinery Corp, states that “the prescriptive period for co-defendant at the instance of the vendee.
instituting actions based on a breach of express warranty is that specified in 41. Finally, even under the principle of solution indebiti, which the RTC applied,
the contract, and in the absence of such period, the general rule on rescission Ang cannot recover from Soleded that amount he paid BA Finance. For, as
of contract, which is four years. the appellate court observed, Ang settled the mortgage debt on his own
34. As for actions based on breach of implied warranty, the prescriptive volution under the supposition that he would resell the car. It turned out that
period s, under Art. 1571 (warranty against hidden defects of or he did pay BA Finance in order to avoid returning the payment made by the
encumbrances upon the thing sold) and Art. 1548 (warranty against ultimate Buyer Bugash.
eviction), six months from the ate of delivery of the thing sold. 42. It need not be stressed that Soledad did not benefit from Ang’s paying BA
35. “I hereby covenant my absolute ownership to (sic) the above-described Finance, he not being the one who mortgaged the vehicle, hence, did not
property and the same is free from all liens and encumbrances and I will benefit form the proceeds thereof.
defend the same from all claims or any claim whatsoever; will save the
vendee from any suit by the government of the Republic of the
Philippines.”
36. In declaring that he owned and had clean title to the vehicle at the time
the Deed of Absolute Sale was forged, Soledad gave an implied warranty
of title. In pledging that he “will defend the same from all claims or any
claim whatsoever [and] will save the vendee from any suit by the
government of the Republic of the Philippines,” Soledad gave a warranty
of eviction.
37. Given Ang’s business of buying and selling used vehicles, he could not have
merely relied on Soledad’s affirmation that the car was free from all liens and
encumbrances. He was expected to have thoroughly verified the car’s
registration and related documents.
38. Since what Soledad, as seller, gave was an implied warranty, the
prescriptive period to file a breach thereof is six months after the
delivery of the vehicle, following Art. 1571. But even if the date of filing
of the action is reckoned form the date Ang instituted his first complaint
for damages on November 9, 1993 and not on July 15, 1996 when he filed
the complaint subject of the present petition, the action just the same had
prescribed.
39. On the merits of his complaint for damages, even if Ang invokes breach of
warranty against eviction as inferred form the second part of the earlier-
quoted provision (Ration #8) of the Deed of Absolute Sale,the following
essential requisites for such breach have not been met.
a. The purchaser has been deprived of the whole or part of the thing
sold.
b. This eviction is by a final judgment.
06 NUTRIMIX FEEDS CORPORATION v. CA (CHIQUI and RICA) respondents.
Oct. 25, 2004 | Callejo, Sr. J. | Hidden warranty
In essence, the Court holds that the Evangelistas failed to prove that the Nutrimix is
PETITIONER: Nutrimix Feeds Corporation guilty of breach of warranty due to hidden defects. It is, likewise, rudimentary that
RESPONDENTS: Court of Appeals and Spouses Efren and Maura Evangelista common law places upon the buyer of the product the burden of proving that the
seller of the product breached its warranty.
SUMMARY:
The Spouses Efren buys animal feeds from Nutrimix Feeds Corporation. The In any case, the Evangelistas have already admitted, both in their testimonies and
Spouses Efren were good paying customers. But at some instances, the Spouses pleadings submitted, that they are indeed indebted to Nutrimix for the unpaid animal
failed to issue checks despite deliveries. The Spouses consequently incurred an feeds delivered to them. For this reason alone, they should be held liable for their
aggregate unsettled amount with Nutrimix. When the Spouses issued checks to unsettled obligations to the Nutrimix.
answer to this, the checks were dishonored because Maura had already closed her
account in the bank. There was a trial regarding this but what is vital in the case is DOCTRINE:
that in the trial, the Spouses presented witnesses that theorized that Nutrimix is liable 1. A hidden defect is one which is unknown or could not have been known to
for hidden defects. The Spouses claimed that several deliveries of animal feeds (three the buyer. Under the law, the requisites to recover on account of hidden
deliveres) were made and that on the same day the feeds were delivered, they had defects are as follows: Defect must: (a) be hidden; (b) exist at the time the
fed these to their animals. The night of the delivery, 18, 000 broiler chickens died. sale was made; (c) ordinarily have been excluded from the contract; and (d)
The Spouses tried to point out that the chickens died because of the animal feeds of be important to render the thing unfit or considerably decreases fitness; and
Nutrimix (see facts for more details). Some of those witnesses testified that they 2. The action must be instituted within the statute of limitations.
found traces of rat poison in the feeds. Other witnesses testified that the feeds 2. In order to enforce the implied warranty that the goods are reasonably fit
delivered to them for testing were contained in unmarked sacks (basically, the feeds and suitable to be used for the purpose which both parties contemplated, the
could have already been tampered with). The RTC ruled in favor of Nutrimix but the following must be established: (a) that the buyer sustained injury because
CA reversed. of the product; (b) that the injury occurred because the product was
defective or unreasonably unsafe; and finally (c) the defect existed when the
The SC said that in the sale of animal feeds, there is an implied warranty that it is product left the hands of the petitioner.
reasonably fit and suitable to be used for the purpose which both parties 3. A manufacturer or seller of a product cannot be held liable for any damage
contemplated. To be able to prove liability on the basis of breach of implied warranty, allegedly caused by the product in the absence of any proof that the product
three things must be established by the Evangelistas. The first is that they sustained in question is defective, which was present upon the delivery or
injury because of the product; the second is that the injury occurred because the manufacture of the product; or when the product left the seller’s or
product was defective or unreasonably unsafe; and finally, the defect existed when manufacturer’s control; or when the product was sold to the purchaser; or
the product left the hands of Nutrimix. the product must have reached the user or consumer without substantial
change in the condition it was sold.
A manufacturer or seller of a product cannot be held liable for any damage allegedly 4. The remedy against violation of warranty against hidden defects is either to
caused by the product in the absence of any proof that the product in question was withdraw from the contract (accion redhibitoria) or to demand a
defective. proportionate reduction of the price (accion quanti minoris), with damages
in either case.
A review of the facts of the case would reveal that Nutrimix delivered the animal
feeds, allegedly containing rat poison, on July 26, 1993; but it is astonishing that the FACTS:
Evangelistas had the animal feeds examined only on October 20, 1993, or barely 1. Spouses Efren and Maura Evangelista started to directly procure various
three months after their broilers and hogs had died. kinds of animal feeds from Nutrimix Feeds Corporation. Evangelista gave
the Spouses Efren a credit period of thirty to forty-five days to postdate
The Court finds it difficult to believe that the feeds delivered on July 26 and 27, 1993 checks to be issued in payment for the delivery of the feeds. The
and fed to the broilers and hogs contained poison at the time they reached the accommodation was made because of the company president's close
friendship with Eugenio Evangelista, the brother of respondent Efren 9. On July 26, 1993, three various kinds of animals feeds (130 bags) were
Evangelista. delivered to the residence of the Spouses. The deliveries were then fed to the
2. The various animal feeds were paid and covered by checks with due dates animals on the same day. Around 8:30 om of the same day, Maura received
from July 1993 to September 1993. Initially, the Evangelistas were good a radio message from a worker in her farm that the chickens were dying in
paying customers. In some instances, however, they failed to issue checks rapid intervals. The Spouses went to the farm and witnessed the death of 18,
despite the deliveries of animal feeds which were appropriately covered by 000 broilers (kind of chicken).
sales invoices. Consequently, the Evangelistas incurred an aggregate 10. The Spouses received another delivery of 160 bags from Nutrimix, some
unsettled account with Nutrimix in the amount of P766,151.00. were distributed to contract growers of the Spouses. Maura requested a
3. When the above-mentioned checks were deposited at the Nutrimix’s representative of Nutrimix to notify Mr. Bartolome (the company president)
depository bank, the same were, consequently, dishonored because Maura that their broilers died. She also asked that a technician or a veterinarian be
Evangelista had already closed her account. Nutrimix made several demands sent to oversee the untoward occurrence. But the feeds that were delivered
for the respondents to settle their unpaid obligation, but the latter failed and that day were still fed to the animals.
refused to pay their remaining balance with Nutrimix. 11. Efren suffered from a heart attack and was hospitalized. This, they said, was
4. On December 15, 1993, Nutrimix filed with the Regional Trial Court of a consequence of the massive death of their animals.
Malolos, Bulacan, a complaint, against the Evangelistas for sum of money 12. Another set of animal feeds were delivered. The spouses didn’t return the
and damages with a prayer for issuance of writ of preliminary attachment. feeds still because at this time, the Spouses don’t know that the cause of death
a. In their answer with counterclaim, the Evangelistas admitted their of their animals was the polluted feeds.
unpaid obligation but impugned their liability to Nutrimix. They 13. The Spouses eventually met with Mr. Bartolome. Maura attributed the
asserted that the nine checks issued by Maura Evangelista were improbable incident to the animal feeds and asked Mr. Bartolome for
made to guarantee the payment of the purchases, which was indemnity. Mr. Bartolome disavowed liability and filed a case against the
previously determined to be procured from the expected proceeds in Spouses.
the sale of their broilers and hogs. They contended that inasmuch as 14. After the meeting, Maura requested Dr. Sanchez, a vet, to conduct an
the sudden and massive death of their animals was caused by the inspection in the poultry. The Spouses also took amounts of feeds and gave
contaminated products of Nutrimix, the nonpayment of their them to government agencies for lab examination.
obligation was based on a just and legal ground. 15. The Clinical Laboratory Report revealed that the feeds were negative of
5. On January 19, 1994, the Evangelistas also lodged a complaint for damages salmonella and that the very high aflatoxin level found would not cause
against Nutrimix for the untimely and unforeseen death of their animals instantaneous death if taken orally by birds.
supposedly effected by the adulterated animal feeds the petitioner sold to 16. Dr. Diaz, a vet who accompanied Efren at the Bureau of Animal Industry,
them. testified that sometime in October 1993, Efren sought for his advice regarding
a. Nutrimix filed its Answer with Counterclaim, alleging that the death the death of the chickens. He suggested that the remaining feeds be brought
of the respondents' animals was due to the widespread pestilence in to the lab for examination. Dr. Diaz claimed that the feeds brought to the lab
their farm. It also received information that the respondents were in came from one bag of sealed Nutrimix feeds which was covered with a sack.
an unstable financial condition and even sold their animals to settle 17. Samples of the blood and bone marrow were taken for chromosome analysis,
their obligations from other enraged and insistent creditors. It also which showed pulverized chromosomes both from marrow and blood
theorized that it was the Evangelistas who mixed poison to its feeds chromosomes.
to make it appear that the feeds were contaminated. 18. On cross examination, Dr. Medina (a cytogenetic analysis who conducted the
6. A joint trial thereafter ensued. exam on number 17), admitted that the feeds brought to him were merely
7. During the hearing, Nutrimix presented Rufino Arenas (Nutrimix assistant placed in a small unmarked plastic bag and that he had no way of ascertaining
manager). Arenas testified that he met the Spouses to discuss the settlement whether the feeds were indeed manufactured by Nutrimix.
for their unpaid account. The Spouses pleaded for Nutrimix to continue to 19. A witness for the Sppuses, Magsipoc, a forensic chemist verified that the
supply them because their livestock were supposedly suffering from a sample feeds yielded positive results to Coumatetralyl compound, the active
disease. component of racumin, a brand name for a commercially known rat poison.
8. Maura Evangelista testified that the Nutrimix president gave them a discount According to Magsipoc, the presence of the compound in chicken feeds will
of Php12 per bag and a credit term of 45 to 75 days. be fatal to internal organs.
20. Austria, Chief of the Pesticide Analytical Section of the Bureau of Plants 2. A hidden defect is one which is unknown or could not have been known
Industry conducted a test to determine the presence of pesticide. The tests to the vendee. Under the law, the requisites to recover on account of
disclosed no pesticide residue but discovered that the feeds were positive for hidden defects are as follows:
Warfarin, a rodenticide, which is in the chemical family of Coumarin. a. the defect must be hidden;
21. Trial Court ruled in favor of Nutrimix. It said that Nutrimix cannot be held b. the defect must exist at the time the sale was made;
liable under “hidden defects” of commodities sold. The subject feeds were c. the defect must ordinarily have been excluded from the
contaminated sometime between their storage at the bodega and their contract;
consumption by the animals. The contamination was made by unidentified d. the defect, must be important (renders thing UNFIT or
ill-meaning mischief-makers. considerably decreases FITNESS);
22. The Spouses sought a review for the decision, arguing that the TC erred in e. the action must be instituted within the statute of limitations.
holding that they failed to prove that their broilers and hogs died as a reault 3. In the sale of animal feeds, there is an implied warranty that it is reasonably
of the feeds. fit and suitable to be used for the purpose which both parties contemplated.
23. The CA modified the decision of the TC. It ruled that the Spouses were not To be able to prove liability on the basis of breach of implied warranty,
obligated to pay the outstanding obligation to Nutrimix in view of the breach three things must be established by the Evangelistas. The first is that they
of warrant against hidden defects. It gave credecnce to the testimony of Dr. sustained injury because of the product; the second is that the injury
Diaz. Dr. Diaz’s declarations were not effectively impugned during cross occurred because the product was defective or unreasonably unsafe; and
examination, nor was there any contrary evidence. finally, the defect existed when the product left the hands of Nutrimix.
24. Hence this petition. 4. A manufacturer or seller of a product cannot be held liable for any
damage allegedly caused by the product in the absence of any proof that
ISSUE/s: the product in question was defective.
11. W/N Nutrimix is liable - NO 5. Tracing the defect to Nutrimix requires some evidence that there was no
tampering with, or changing of the animal feeds. The nature of the animal
RULING: WHEREFORE, in light of all the foregoing, the petition is GRANTED. feeds makes it necessarily difficult for the Evangelistas to prove that the
The assailed Decision of the Court of Appeals, dated February 12, 2002, is defect was existing when the product left the premises of the Nutrimix.
REVERSED and SET ASIDE. The Decision of the Regional Trial Court of Malolos, 6. A review of the facts of the case would reveal that Nutrimix delivered the
Bulacan, Branch 9, dated January 12, 1998, is REINSTATED. No costs. animal feeds, allegedly containing rat poison, on July 26, 1993; but it is
astonishing that the Evangelistas had the animal feeds examined only on
RATIO: October 20, 1993, or barely three months after their broilers and hogs had
1. The provisions on warranty against hidden defects are found in Articles 1561 died.
and 1566 of the New Civil Code of the Philippines, which read as follows: a. We find it difficult to believe that the feeds delivered on July 26 and
a. Art. 1561. The vendor shall be responsible for warranty against 27, 1993 and fed to the broilers and hogs contained poison at the
hidden defects which the thing sold may have, should they render it time they reached the respondents.
unfit for the use for which it is intended, or should they diminish its i. In a span of three months, the feeds could have already
fitness for such use to such an extent that, had the vendee been aware been contaminated by outside factors and subjected to
thereof, he would not have acquired it or would have given a lower many conditions unquestionably beyond the control of the
price for it; but said vendor shall not be answerable for patent defects petitioner. In fact, Dr. Garcia, one of the witnesses for the
or those which may be visible, or for those which are not visible if respondents, testified that the animal feeds submitted to her
the vendee is an expert who, by reason of his trade or profession, for laboratory examination contained very high level of
should have known them. aflatoxin, possibly caused by mold (aspergillus flavus).
b. Art. 1566. The vendor is responsible to the vendee for any hidden b. We agree with the contention of Nutrimix that there is no evidence
faults or defects in the thing sold, even though he was not aware on record to prove that the animal feeds taken to the various
thereof. This provision shall not apply if the contrary has been governmental agencies for laboratory examination were the same
stipulated, and the vendor was not aware of the hidden faults or animal feeds given to the respondents' broilers and hogs for their
defects in the thing sold. consumption. Moreover, Dr. Diaz even admitted that the feeds that
were submitted for analysis came from a sealed bag.
c. It bears stressing, too, that the chickens brought to the Philippine
Nuclear Research Institute for laboratory tests were healthy animals,
and were not the ones that were ostensibly poisoned. There was even
no attempt to have the dead fowls examined. Neither was there any
analysis of the stomach of the dead chickens to determine whether
the petitioner's feeds really caused their sudden death. Mere sickness
and death of the chickens is not satisfactory evidence in itself to
establish a prima facie case of breach of warranty.
d. There was evidence tending to show that the Evangelistas combined
different kinds of animal feeds and that the mixture was given to the
animals. Maura Evangelista even testified that it was common
practice among chicken and hog raisers to mix animal feeds.
e. In essence, we hold that the Evangelistas failed to prove that the
Nutrimix is guilty of breach of warranty due to hidden defects. It is,
likewise, rudimentary that common law places upon the buyer of
the product the burden of proving that the seller of the product
breached its warranty.
7. For these reasons, the expert testimonies lack probative weight. The
Evangelista’s case of breach of implied warranty was fundamentally based
upon the circumstantial evidence that the chickens and hogs sickened,
stunted, and died after eating Nutrimix feeds; but this was not enough to raise
a reasonable supposition that the unwholesome feeds were the proximate
cause of the death with that degree of certainty and probability required. The
rule is well-settled that if there be no evidence, or if evidence be so slight
as not reasonably to warrant inference of the fact in issue or furnish
more than materials for a mere conjecture, the court will not hesitate to
strike down the evidence and rule in favor of the other party.
8. It must be stressed, however, that the remedy against violations of warranty
against hidden defects is either to withdraw from the contract (accion
redhibitoria) or to demand a proportionate reduction of the price (accion
quanti minoris), with damages in either case.
9. In any case, the Evangelistas have already admitted, both in their testimonies
and pleadings submitted, that they are indeed indebted to Nutrimix for the
unpaid animal feeds delivered to them. For this reason alone, they should be
held liable for their unsettled obligations to the Nutrimix.
001 ROBERTS v. PAPIO (Nikki V) settled their obligation, the corporation returned the owner’s duplicate TCT
09 February 2007 | J. Callejo, Sr. | G.R. No. 166714 | which was then delivered to Amelia Roberts.
Promise to sell v. Agreement to repurchase 9. METC: Papio’s claim that the transfer of property was one with right of
repurchase lacks merit since the Deed of Sale is termed absolute and
PETITIONER: Amelia S. Roberts unconditional.
RESPONDENT: Martin B. Papio 10. RTC: affirmed MeTC as regards the nature of the contract.
11. CA: METC and RTC erred in ignoring Papio’s defense of equitable mortgage
SUMMARY: The Spouses Papio were the owners of a residential lot located in and in not finding that the transaction convered by the deed of absolute sale
Makati. In order to secure the loan from the Amparo Investments Corp, they executed by and between the parties was one of equitable mortgage under Art. 160233
a real estate mortgage on the property. Upon Papio’s failure to pay the loan, the of the New Civil code. CA ruled that Papio retained the ownership of the
corporation filed a petition for the extrajudicial foreclosure of the mortgage. Since the property and its peaceful possession. By virtue of this CA ruling, Roberts
couple needed money to redeem the property and to prevent the foreclosure of the real brought the case to the SC.
estate mortgage, they executed a Deed of Absolute Sale over the property in favor of 12. ROBERTS: Papio is barred from raising the issue of equitable mortgage
Martin Papio’s cousin, Amelia Roberts. As soon as the spouses had settled their because his defense in the RTC and METC was that he had repurchased the
obligation, the corporation returned the owner’s duplicate TCT which was then property from Roberts. By such representation, he had impliedly admitted the
delivered to Amelia Roberts.Roberts and Papio then executed a 2-year lease subject to existence and validity of the deed of absolute sale whereby ownership of the
renewa. However, Papio failed to pay rentals and Roberts instituted this case. Papio property was transferred to Roberts but reverted to him [Papio] upon the
claims that he repurchased the said land under an equitable mortgage. The issue is exercise of the said right. In fact, Papio even filed a complaint for specific
WON the contract between Roberts and Papio is an equitable mortgage. The SC said performance where Papio claims that his transaction with Roberts was a sale
that it is not; it is only a contract of sale where no right to repurchase was reserved. with pacto de retro. Roberts posits that Art. 1602 applies only when the
Moreover, Papio admitted the existence of sale by insisting he repurchased the land defendant [Papio] specifically alleges this defense. Consequently, CA was
which is incompatible with the defense of equitable mortgage as an equitable mortgage proscribed from finding that Papio and Roberts had entered into an equitable
does not transfer ownership. mortgage under the deed of absolute sale.
13. PAPIO: the defense of equitable mortgage need not be particularly stated to
DOCTRINE: An agreement to repurchase becomes a promise to sell when made after apprise Roberts of the nature and character of the repurchase agreement since
the sale because when the sale is made without such agreement the purchaser acquires he had amply discussed in his pleadings before the trial and appellate courts
the thing sold absolutely; and, if he afterwards grants the vendor the right to all the surrounding circumstances of the case and the negotiations that led to
repurchase, it is a new contract entered into by the purchaser as absolute owner. An the repurchase agreement. He then argues that CA correctly ruled that the
option to buy or a promise to sell is different and distinct from the right of repurchase contract was one of equitable mortgage. He insists that Roberts allowed him
that must be reserved by means of stipulations to that effect in the contract of sale. to redeem and reacquire the property, and accepted his full payment through
FACTS: Roberts’ agent as shown by the receipts.
7. The Spouses Papio were the owners of a 274 sq m residential lot located in
Makati. In order to secure a 59k loan from the Amparo Investments Corp, ISSUE:
they executed a real estate mortgage on the property. Upon Papio’s failure to 1. WON the transaction entered into between the parties under the Deed of Absolute
pay the loan, the corporation filed a petition for the extrajudicial foreclosure Sale and the Contract of Lease is an equitable mortgage over the property – NO.
of the mortgage. The contract is one of absolute sale and not one with right to repurchase.
8. Since the couple needed money to redeem the property and to prevent the
foreclosure of the real estate mortgage, they executed a Deed of Absolute RULING: IN LIGHT OF ALL THE FOREGOING, the petition is GRANTED. The
Sale over the property in favor of Martin Papio’s cousin, Amelia Roberts. Of assailed Decision of the Court of Appeals in CA-G.R. CV No. 69034 is REVERSED
the 85k purchase price, 59k was paid to the Amparo Investments Corp, while and SET ASIDE. The Decision of the Metropolitan Trial Court, affirmed with
the 26k difference was retained by the spouses. As soon as the spouses had modification by the Regional Trial Court, is AFFIRMED.

3333 Art. 1602. The contract shall be presumed to be an equitable mortage, in any of the following cases: (2) when the vendor remains in possession as lessee or otherwise
RATIO: allegation and evidence of palpable mistake on the part of respondent, or a fraud
1. Papio alleged the following: on the part of Roberts.
a. Roberts had granted him the right to redeem or repurchase the property at any Art. 1601. Conventional redemption shall take place when the vendor reserves the
time and for a reasonable amount; right to repurchase the thing sold, with the obligation to comply with the provisions of
b. and that Papio had, in fact, repurchased the property in July 1985 for Php250k Art. 1616 and other stipulations which may have been agreed upon.
which he remitted through Roberts’ agent who signed receipts therefor;
c. Papio acquired ownership and juridical possession of the property after his
repurchase thereof in 1985
d. Consequently, Roberts was obliged to execute a deed of absolute sale over the
property in Papio’s favor.
2. As gleaned from the April 13, 1982 deed, the right of Papio to repurchase the
property is not incorporated therein. The contract is one of absolute sale and not
one with right to repurchase. The law states that if the terms of a contract are clear
and leave no doubt upon the intention of the contracting parties, the literal meaning
of its stipulations shall control.
3. The right of repurchase is not a right granted the vendor by the vendee in a
subsequent instrument, but is a right reserved by the vendor in the same instrument
of sale as one of the stipulations of the contract. Once the instrument of absolute
sale is executed, the vendor can no longer reserve the right to repurchase, and any
right thereafter granted the vendor by the vendee in a separate instrument cannot
be a right of repurchase but some other right.
4. An equitable mortgage is one that, although lacking in some formality, form or
words, or other requisites demanded by a statute, nevertheless reveals the intention
of the parties to charge a real property as security for a debt and contain
nothing impossible or contrary to law.
5. A contract between the parties is an equitable mortgage if the following requisites
are present: a. the parties entered into a contract denominated as a contract of sale
and b. the intention was to secure an existing debt by way of mortgage. The
decisive factor is the intention of the parties. In an equitable mortgage, the
mortgagor retains ownership over the property but subject to foreclosure and
sale at public auction upon failure of the mortgagor to pay his obligation.
6. In contrast, in a pacto de retro sale, ownership of the property sold is
immediately transferred to the vendee a retro subject only to the right of the
vendor a retro to repurchase the property upon compliance with legal
requirements for the repurchase. The failure of the vendor a retro to exercise the
right to repurchase within the agreed time vests upon the vendee a retro, by
operation of law, absolute title over the property. One repurchases only what one
has previously sold. The right to repurchase presupposes a valid contract of sale
between same parties.
7. By insisting that he had repurchased the property, Papio thereby admitted that the
deed of absolute sale executed by him and Roberts was in fact and in law a deed of
absolute sale and not an equitable mortgage; he had acquired ownership over the
property based on said deed. Papio, is thus estopped from asserting that the
contract under the deed of absolute sale is an equitable mortgage unless there is an
002 MISTERIO v. CEBU STATE COLLEGE (JP) 180. Asuncion Sadaya-Misterio (Misterio) executed a Deed of Sale of a parcel
June 23, 2005 | Callejo, J. | Redemption Period of land denominated as Lot No. 1064 of the Banilad Friar Lands Estate, in
favor of the SAHS.
PETITIONER: Luis Misterio, Gabriel Misterio, Francis Misterio, et al. 181. The sale was subjet to the right of the vendor (Misterio) to repurchase the
RESPONDENTS: Cebu State College of Science and Technology (CSCST), property after the high school shall have ceased to exist, or shall have
represented by its President, Dr. Jose Tan transferred its site elsewhere.
182. Consequently, the old TCT was cancelled and a new TCT in the name of
SUMMARY: Misterio executed a Deed of Sale of a parcel of land of the Banilad SAHS was issued by the Registry of Deeds of Cebu. The right of the vendor
Friar Lands Estate in favor of Sudol Agricultural High School (SAHS). The sale to repurchase the property was annotated at the dorsal portion thereof.
was subject to the right of Ministerio as vendor to repurchase the property after 183. On March 18, 1960, the Provincial Board of Cebu, through Resolution
the high school has ceased to exist. In the meantime, the Provincial Board of Cebu No. 491, donated the aforementioned 41 lots to SAHS (from Fact No. 2),
donated 41 parcels of the Banilad Friar Lands Estate to SAHS, the previous parcel subject to two (2) conditions:
of land subject of the sale being part of the estate. a. that if the SAHS ceases to operate, the ownership of the lots would
automatically revert to the province, and
BP 412 then took effect. The law incorporated certain schools in the Cebu b. that the SAHS could not alienate, lease or encumber the properties.
Province into one Cebu State College of Science and Technology, including the 184. Personal note: we have two transactions here, a pacto de retro sale in Fact
SAHS. The Provincial Board of Cebu then tried to recover the property by No. 3 and a donation in Fact No. 6. But as the case progresses, it seems
claiming that the donation they made was void because SAHS did not anymore only the pacto de retro sale is relevant.
exist, because of the consolidation. Upon hearing the recovery of the Cebu 185. On June 10, 1983, Batas Pambansa (B.P.) Blg. 412, entitled "An Act
Province of the parcels of land, the heirs of Misterio exercised their intention to Converting the Cebu School of Arts and Trades in Cebu City into a Chartered
repurchase the lot sold to them. The RTC held that the donation was void. CA College to be Known as the Cebu State College of Science and Technology,
reversed and said that although it is true that SAHS ceased to exist, the action for Expanding its Jurisdiction and Curricular Programs" took effect.
repurchase has prescribed. 186. The law incorporated and consolidated as one school system certain
vocational schools in the province of Cebu, including the SAHS, and which
The SC held that according to Art. 1606 of the NCC, if the contract as in this case, became an extension of the Cebu State College of Science and Technology
did not expressly agree on the period for repurchase, the vendor may extend the (CSCST).
period until 4 years from the happening of the condition in the deed. In this case, 187. In the meantime, the province of Cebu decided to recover the 41 lots it had
the petitioner heirs had 4 years after the abolishment of the SAHS to repurchase earlier donated to SAHS on the ground that the said deed was void.
the property but they failed to do so. (See doctrine for additional ruling). Their 188. The province of Cebu opined that based on the initial report of its provincial
action has prescribed. attorney, the SAHS had no personality to accept the donation.
189. In the meantime, Asuncion died intestate.
DOCTRINE: The four-year period to repurchase is not suspended merely 190. When her heirs learned that the province of Cebu was trying to recover the
because there is a divergence of opinion between the parties as to the precise property it had earlier donated to SAHS, they went to the province of Cebu,
meaning of the phrase providing for the condition upon which the right to informing it of their intention to exercise their right to repurchase the property
repurchase is triggered. The existence of seller a retro’s right to repurchase the as stipulated in the aforecited deed of sale executed by their predecessor-in-
proper is not dependent upon the prior final interpretation by the court of the said interest.
phrase. 191. The province of Cebu (represented by then Governor Osmeña), and the
CSCST (represented by then DECS Secretary Quisumbing), entered into a
Memorandum of Agreement over the 40 parcels of land, allocating 53
FACTS: hectares to the province of Cebu, and 51 hectares for the SAHS.
178. The Sudlon Agricultural High School (SAHS) was established in Cebu 192. In a Letter, the heirs of the late Asuncion Sadaya-Misterio informed CSCST
Province. of the heirs' intention to exercise the option to repurchase Lot No. 1064
179. In 1952, the Provincial Board of Cebu granted the usufruct of 41 parcels of granted to them under the deed of sale, as the SAHS had ceased to exist.
land, covering 104.54 hectares of the Banilad Friar Lands Estate to the SAHS. 193. In response thereto, Jesus T. Bonilla, as Vocational School Superintendent II
of CSCST, wrote the heirs, informing the latter that the SAHS still existed the property, was annotated on the title of the land, the right to exercise the
and "in fact, from a purely secondary school it is now offering collegiate same is imprescriptible.
courses." He explained that "what has been changed is only the name of the 4. They argue that they had been vigilant of their right to repurchase the
school [to CSCST] which does not imply the loss of its existence." property, as far back as 1973. In fact, they made tender of payment in March
194. Luis, Gabriel, Francis, Thelma, all surnamed Misterio, and Estella S. 1990, well within the ten-year prescriptive period.
Misterio-Tagimacruz, the legitimate heirs of the late Asuncion Sadaya- 5. They point out that the CSCST had abandoned its defense of prescription by
Misterio and herein petitioners, filed a Complaint before the RTC "Nullity of contending that the condition for repurchase had not yet become operational.
Sale and/or Redemption." 6. The essence of a pacto de retro sale is that title and ownership of the property
195. RTC ruled in favor of the petitioner heirs and declared the Deed of Sale sold is immediately rested in the vendee a retro, subject to the restrictive
entered into by and between Misterio and SAHS as null and void for the condition of repurchase by the vendor a retro within the period provided in
latter’s lack of juridical personality to acquire real property. Article 1606 of the New Civil Code.
196. The RTC ruled that the donation was void ab initio as the SAHS, in the first 7. Art. 1606:
place, did not have the personality to be a donee of real property. Moreover, i. The right referred to in Article 1601, in the absence of an express
with the enactment of B.P. Blg. 412, the SAHS ceased to exist and to operate agreement, shall last four years from the date of the contract.
as such. ii. Should there be an agreement, the period cannot exceed ten years.
197. The CA reversed. While it affirmed the RTC ruling that the SAHS had ceased iii. However, the vendor may still exercise the right to repurchase
to exist when B.P. Blg. 412 took effect, the appellate court ruled that the within thirty days from the time final judgment was rendered in a
period for the petitioner heirs to repurchase the property expired on June civil action on the basis that the contract was a true sale with right
1987, four years after the enactment of B.P. Blg. 412. to repurchase.
198. Their MR was denied; hence, the appeal. 8. In this case, the vendor a retro and the vendee a retro did not agree on
any period for the exercise of the right to repurchase the property.
ISSUE/s: Hence, the vendor a retro may extend the said right within four years
from the happening of the allocated condition contained in the deed: (a)
160. WoN SAHS still retained its personality as such school or was still in the cessation of the existence of the SAHS, or (b) the transfer of the school
existence - NO to other site.
161. WoN the petitioner heirs had the right to exercise the right to repurchase the 9. As gleaned from the petitioner heirs’ complaint before the trial court,
property – NO they alleged that the SAHS ceased to exist on June 10, 1983, when B.P.
162. WoN the petitioner heirs’ action for repurchase has prescribed - YES Blg. 412 took effect, abolishing therein the SAHS which, in the meantime,
had been converted into the Sudlon Agricultural College.
RULING: Petition is DENIED DUE COURSE. 10. The CA found the position of the petitioners to be correct, and declared
that conformably to the condition in the deed of sale, and under Article
RATIO: 1606 of the New Civil Code, the right of the petitioners as successors-in-
1. Citing Article 1606(3) of the New Civil Code, the petitioner heirs argue that interest of the vendee a retro commenced to run on June 10, 1983.
"prescription should start to run from the time it is legally feasible for the 11. Hence, they had until June 10, 1987 within which to repurchase the
party to redeem the land, which is the time when the action to redeem has property; however, they failed to do so.
accrued." The petitioners argue that this is so since the issue of whether the 12. It is true that respondent CSCST, through counsel, was of the view that
SAHS had ceased to exist had still yet to be resolved. despite the effectivity of B.P. Blg. 412, the structure and facilities of the
2. The petitioner heirs posit that unless and until judgment would be rendered SAHS remained in the property and, as such, it cannot be said that the said
stating that the SAHS has ceased to exist, the period to repurchase the school had ceased to exist.
property would not start to run. It is only from the finality of the said judgment 13. It argued that the phrase "SAHS ceased to exist" in the deed meant that the
that the right to repurchase the property may be exercised; hence, they still structure and facilities of the school would be destroyed or dismantled, and
had thirty (30) days from the date of the promulgation of the CA decision had no relation whatsoever to the abolition of the school and its integration
within which to repurchase the property. into the Cebu State College for Science and Technology.
3. The petitioner heirs further aver that since the lien, their right to repurchase 14. The four-year period for the petitioners to repurchase the property was
not suspended merely and solely because there was a divergence of
opinion between the petitioners, on the one hand, and the respondent, on
the other, as to the precise meaning of the phrase "after the SAHS shall
cease to exist" in the deed of sale.
15. The existence of the petitioners' right to repurchase the property was not
dependent upon the prior final interpretation by the court of the said
phrase.
16. The petitioner heirs, by specifically alleging in the complaint that the
SAHS has ceased to exist, are estopped from changing on appeal their
theory of the case in the trial court and in the CA.
17. The SC agrees with the contention of the OSG that the annotation of the
petitioners' right to repurchase the property at the dorsal side of TCT No.
15959 has no relation whatsoever to the issue as to when such right had
prescribed.
18. The annotation was only for the purpose of notifying third parties of the
petitioners' right to repurchase the property under the terms of the deed of
sale, and the law.
003 SOLID HOMES, INC. v. CA (HENRY) of State Financing Center (SFC) its properties (2 TCTs, both in Pasig) to
July 8, 1997 | Panganiban, J. | Pactum Commissorium secure a loan of Php 10M.
17. A year later, Solid Homes applied for and was granted a loan by SFC
PETITIONER: Solid Homes, Inc. amounting to Php 1,511,270.03, to which Solid Homes executed an Amended
RESPONDENTS: CA, State Financing Center, Inc., and RD for Rizal REM (June 1980) where the credits secured increased to Php 11,511,270.03.
SUMMARY: This case is basically about the contention of whether or not a 18. Solid Homes then acquired additional credits and financing facilities from
subsequent MOA/Dacion en pago entered by both parties in lieu of a prior REM SFC in the sum of Php 1,499,811.97 which led them to execute another
constitutes pactum commisorium, to which the SC ruled in the negative. Amended REM (March 1982) which further increased the total credit to Php
Solid Homes executed a REM in favor of SFC for two of its properties (where 13,011,082.00.
VV Soliven Towers II was erected), to secure a loan of 10M, which eventually 19. The loans became due and unfortunately despite repeated demands, Solid
increased to about 13M in total until it became due. Despite repeated demands by Homes failed to pay, this lead SFC to file for a petition for extrajudicial
SFC, Solid Homes failed to pay, which lead to the filing of a petition for foreclosure of the mortgages with the Provincial Sheriff of Rizal.
extrajudicial foreclosure of SFC, that led to the posting of the sheriff of the notice 20. Sheriff issued then a Notice of Sheriff’s Sale where the mortgaged properties
of the sale of the properties that were mortgaged. of Solid Homes and the improvements (including the VV Soliven Towers II
Prior to the scheduled auction sale, Solid Homes was able to induce SFC to forego Building) were set for public auction to satisfy Sold Home’s indebtedness,
of the mortgage, but that they enter into a MOA/Dacion en Pago, which interest, and fees and expenses.
essentially translated the agreement from that of REM to a sale with the right to 21. Prior to the scheduled public auction sale, Solid Homes made representations
repurchase. Pertinent provisions of the MOA included the obligation of Solid and induced SFC to forego with the foreclosure. This lead to both parties
Homes to pay loan (amounting to 14M, which counted the interests, costs, among executing a MOA, with the following terms, among others:
others), and the fact that if by 180 days there is failure to pay, SFC will be in a. Solid Homes acknowledges his debt (Php 14,225,178.40 + 14%
possession of the property, with 10 months right to repurchase after the said 180 interest/annum + 16% penalty/annum from March 1, 1983 until fully
days. paid) and that he agrees to pay the total amount within 180 days from
Apparently even this deal wasn’t enough and Solid Homes was unable to pay date of signing the instrument
once again, which necessarily meant that its properties are to be transferred under b. Should Solid Homes be able to pay SFC an amount equivalent to
the names of SFC. (small issue arose here when the right to repurchase wasn’t 60% of the principal (this is around Php 8.5M) within the first 180
annotated in the title when it was transferred to SFC, to which the lower court days, SFC shall allow the balance to be restructured at a rate of
already fixed, so nvm) interest amenable to both parties.
Solid Homes is contesting that such MOA shall be struck down as void because c. If Solid Homes fails to comply within 180 days, this MOA shall
it is in violation of the CC for being pactum commissorium, as he claims that his operate to be an instrument of dacion en pago without the need of
mortgaged properties were automatically transferred to SFC as creditor when he executing any document to such an effect and Solid Homes binds
was unable to satisfy his debt. Both RTC and CA ruled in favor of SFC and said itself to transfer, convey, and assign to SFC by way of dacion en
that the MOA is not a pactum commisorium. pago, its heirs, successors, and assigns, and SFC accepts the
This was echoed in the ruling of the SC, wherein they said that such MOA/Dacion conveyance and transfer of the 2 real properties + improvements
en pago was in fact a sale with the right to repurchase. This essentially means that d. State Financing grants Solid Homes the right to repurchase
the possession of the property really goes to SFC, and when there is no right to (properites + improvements) within 10 months counted from and
repurchase exercised by Solid Homes within the provided period, it shall after 180 days from signing of MOA
completely transfer ownership to SFC. 22. Solid Homes failed to pay SFC an amount equivalent to 60% (in
DOCTRINE: The provision in MOA/Dacion en pago with a right to repurchase contravention of the MOA, letter b). Hence, the document “operated to be an
that if borrower fails to comply with the new terms of restructuring the loan, the instrument of dacion en pago without the need of executing any document to
agreement shall automatically operate as a dacion en pago without need of such an effect. Now, both properties are transferred under the name of SFC.
executing any new document does not constitute pactum commissorium. 23. When through a letter SFC demanded that the VV Soliven Towers II
Building be transferred to their possession, Solid Homes asked for 10 days
FACTS: to categorize its position—well after the 10 days, they sent a reply to SFC
16. June 4, 1979, Solid Homes executed a Real Estate Mortgage (REM) in favor and said that the MOA they entered into is null and void because its essence
is that, as mortgagee creditor, SFC would be able to appropriate unto itself mortgaged or under antichresis shall automatically become property of the
the properties mortgaged by Solid Homes, which is in contravention of Art. creditor in the event of nonpayment of the debt within the term fixed (Art.
2088 of the CC. Of course, SFC replied with a letter saying that Art. 2088 2088, CC).
does not apply to the MOA they entered into. 69. Looking closely in the facts of this case, specifically Fact #6, the MOA was
24. Solid Homes haggled with SFC and even sent a letter proposing for entered into in order to forego with the foreclosure of the mortgaged
repayment schemes, to which SFC did not agree with. There were also properties. The MOA is necessarily an entirely new contract—a new
conferences where both were negotiating the arrangement on the outstanding agreement. Moving forward, there was no “mortgage” anymore. Reading
obligation. plainly the MOA itself (refer to the items enumerated under Fact#6 for more
25. Both ends never met and with the right to repurchase nearing (June 27, 1984), details), it can be inferred that now it became a sale with pacto de retro.
Solid Homes requested that it be extended to another 60 days, and that the 70. It became as such, when the “loan” became the payment of SFC for the
obligation to pay the loan (14M) be extended for another year. properties that were previously mortgaged. So basically what we’re looking
26. No response from SFC was received, and so a day before the expiry of the at is a change in the contract: it was no longer a real estate mortgage, it was
right to repurchase, a petition was filed before the RTC, who ruled that the now a sale, by virtue of the MOA.
MOA/Dacion en pago was valid and binding. 71. Now, looking at it as a sale with pacto de retro, it is evident that the properties
27. Both appealed: Solid Homes assails the denial of its claim for damages, which shall go with SFC unless Solid Homes exercises its right of redemption in
it claims to have been resulted from the bad faith and malice of SFC in failing accordance with the period granted to it by the SC.
to annotate their right to repurchase in the titles.
28. State Financing on the other hand questioned the ruling as far as the grant of
the court as to the 30 days more for the right of repurchase, among others.
29. CA ruled in favor of SFC and among others, ordered Solid Homes to deliver
possession of the subject properties to SFC, on the ground that in a sale with
pacto de retro, the vendee shall immediately acquire title over and possession
of the real property sold, subject only to the vendor’s right of redemption.
Hence, this petition.

ISSUE/s:
4. WoN the MOA/Dacion en pago constitutes pactum commissorium – NO

RULING: The SC affirmed the CA ruling, with modification that the redemption price
shall not include the registration and other expenses incurred by SFC in the issuance
of new certificates of title in its name, as this was doen without proper judicial order
required under Art. 1607 of the CC.
RATIO:
65. In a sale with pacto de retro, the vendee has a right to the immediate
possession of the property sold, unless otherwise agreed upon.
66. It is basic that in a pacto de retro sale the title and ownership of the property
sold are immediately vested in the vendee a retro, subject only to the
resolutory condition of repurchase by the vendor a retro within the stipulated
period.
67. (guys, until number 2 lang talaga yung sa ratio related sa relevant issue so
moving forward below, allow me to elaborate how this case is related to the
topic. Feel free to read the originals if you’re not confident with what I’m
about to right)
68. Pactum commisorium is defined as a stipulation whereby a thing pledged or
004 A. FRANCISCO REALTY v. CA (Salve) the mortgaged property in favor of the mortgagee, A. Francisco Realty.
October 30, 1998 | Mendoza, J. | Pactum Commissorium 27. The interest on the said loan was to be paid in four installments.
PETITIONER: A. Francisco Realty and Development Corporation 28. The promissory note expressly provided that upon failure of the
RESPONDENTS: Court of Appeals and Spouses Romulo S.A. Javillonar and MORTGAGOR [Spouses Javillonar] to pay the interest without prior
Erlinda P. Javillonar arrangement with the MORTGAGEE [A. Francisco Realty], full possession
SUMMARY: A. Francisco Realty granted a loan to Spouses Javillonar. The of the property will be transferred and the deed of sale will be registered. For
promissory note stated that upon failure of the Spouses to pay the interest, full this purpose, the owners duplicate of TCT No. 58748 was delivered to A.
possession of the property will be transferred and the deed of sale will be Francisco Realty.
registered to A. Francisco Realty. A. Francisco Realty claims that the spouses 29. A. Francisco Realty claims that Spouses Javillonar failed to pay the interest
failed to pay so the sale of the land was registered in its favor and a title was and, as a consequence, it registered the sale of the land in its favor. As a result,
issued in its name. RTC declared A. Francisco the owner of the property. CA a title was issued in the name of A. Francisco Realty.
reversed RTC’s decision and declared the the sale was void for being a pactum 30. Regional Trial Court rendered a decision declaring as legal and valid, the
commissorium. WoN the deed of sale is void for being in fact a pactum right of ownership of A. Francisco Realty And Development Corporation,
commissorium. YES. This Court categorically ruled that a mortgagees mere act over the property subject of this case situated at No. 56 Dragonfly Street,
of registering the mortgaged property in his own name upon the mortgagors Valle Verde VI, Pasig, Metro Manila.
failure to redeem the property amounted to the exercise of the privilege of a 31. Spouses appealed to the Court of Appeals which reversed the decision of the
mortgagee in a pactum commissorium. The act of applicant in registering the trial court and dismissed the complaint against them. The appellate court
property in his own name upon mortgagors failure to redeem the property would ruled that the Regional Trial Court had no jurisdiction over the case because
amount to a pactum commissorium which is against good morals and public it was actually an action for unlawful detainer which is exclusively
policy. Thus, in the case at bar, the stipulations in the promissory notes providing cognizable by municipal trial courts.
that, upon failure of respondent spouses to pay interest, ownership of the property 32. Furthermore, CA ruled that, even presuming jurisdiction of the trial court, the
would be automatically transferred to A. Francisco Realty and the deed of sale in deed of sale was void for being in fact a pactum commissorium which is
its favor would be registered, are in substance a pactum commissorium. They prohibited by Art. 2088 of the Civil Code.
embody the two elements of pactum commissorium as laid down in Uy Tong v.
Court of Appeals. ISSUES:
1. WoN the deed of sale is void for being in fact a pactum commissorium. YES.
DOCTRINE: The two elements for pactum commissorium to exist: (1) that there
should be a pledge or mortgage wherein a property is pledged or mortgaged by RULING: WHEREFORE, the decision of the Court of Appeals is AFFIRMED,
way of security for the payment of the principal obligation; (2) that there should insofar as it dismissed petitioners complaint against respondent spouses on the ground
be a stipulation for an automatic appropriation by the creditor of the thing pledged that the stipulations in the promissory notes are void for being a pactum
or mortgaged in the event of non-payment of the principal obligation within the commissorium, but REVERSED insofar as it ruled that the trial court had no
stipulated period. jurisdiction over this case. The Register of Deeds of Pasig City is hereby ORDERED
to CANCEL TCT No. PT-85569 issued to petitioner and ISSUE a new one in the name
ART. 2088. The creditor cannot appropriate the things given by way to pledge or of respondent spouses.
mortgage, or dispose of them. Any stipulation to the contrary is null and void.
SO ORDERED.

FACTS: RATIO:
26. A. Francisco Realty and Development Corporation granted a loan of P7.5
Million to the spouses Romulo and Erlinda Javillonar, in consideration of 1. A. Francisco Realty’s action could not succeed because the deed of sale on which
which the latter executed the following documents: (a) a promissory note, it was based was void, being in the nature of a pactum commissorium prohibited
dated November 27, 1991, stating an interest charge of 4% per month for six by Art. 2088 of the Civil Code which provides:
months; (b) a deed of mortgage over realty covered by TCT No. 58748, 2. ART. 2088. The creditor cannot appropriate the things given by way to pledge or
together with the improvements thereon; and (c) an undated deed of sale of mortgage, or dispose of them. Any stipulation to the contrary is null and void.
3. A. Francisco Realty denies, however, that the promissory notes contain a pactum
commissorium. Before Article 2088 can find application herein, the subject deed
of mortgage must be scrutinized to determine if it contains such a provision giving
the creditor the right to appropriate the things given by way of mortgage without
following the procedure prescribed by law for the foreclosure of the mortgage.
4. IN SHORT, THE PROSCRIBED STIPULATION SHOULD BE FOUND IN
THE MORTGAGE DEED ITSELF.
5. The contention is patently without merit. To sustain the theory of petitioner would
be to allow a subversion of the prohibition in Art. 2088.
6. In Reyes v. Sierra this Court categorically ruled that a mortgagees mere act of
registering the mortgaged property in his own name upon the mortgagors failure
to redeem the property amounted to the exercise of the privilege of a mortgagee
in a pactum commissorium.
7. The act of applicant in registering the property in his own name upon mortgagors
failure to redeem the property would amount to a pactum commissorium which is
against good morals and public policy.
8. Thus, in the case at bar, the stipulations in the promissory notes providing that,
upon failure of respondent spouses to pay interest, ownership of the property
would be automatically transferred to A. Francisco Realty and the deed of sale in
its favor would be registered, are in substance a pactum commissorium. They
embody the two elements of pactum commissorium as laid down in Uy Tong v.
Court of Appeals, to wit:
9. The two elements for pactum commissorium to exist: (1) that there should be a
pledge or mortgage wherein a property is pledged or mortgaged by way of security
for the payment of the principal obligation;
(2) that there should be a stipulation for an automatic appropriation by the creditor
of the thing pledged or mortgaged in the event of non-payment of the principal
obligation within the stipulated period.
10. The subject transaction being void, the registration of the deed of sale, by virtue
of which petitioner A. Francisco Realty was able to obtain TCT No. PT-85569
covering the subject lot, must also be declared void, as prayed for by respondents
in their counterclaim.
005 HOME GUARANTY CORPORATION illicit stipulation is ineffectual. It did not vest ownership in Home Guaranty
vs. LA SAVOIE DEVELOPMENTCORPORATION (Lorena) Corporation.
January 28, 2015 | Leonen, J. | Pactum sommissorium
FACTS:
PETITIONER: Home Guaranty Corporation
RESPONDENTS: La Savoie Development Corporation 1. La Savoie Development Corporation (La Savoie) is a domestic corporation
incorporated engaged in the business of "real estate development, subdivision
SUMMARY: La Savoie found itself unable to pay its obligations to its creditors
due to the onset of the Asian financial crisis in 1997. Thus, La Savoie filed before and brokering."
the RTC, Makati City a "petition for the declaration of state of suspension of 2. La Savoie found itself unable to pay its obligations to its creditors due to the
payments with approval of proposed rehabilitation plan" under the Interim Rules onset of the Asian financial crisis in 1997.
of Procedure on Corporate Rehabilitation (Interim Rules). Home Guaranty a. Thus, La Savoie filed before the RTC, Makati City a "petition for
the declaration of state of suspension of payments with approval of
Corporation filed an Opposition even though "it [was] not a creditor of
Petitioner.” It asserted that it had a "material and beneficial interest in the . . . proposed rehabilitation plan" under the Interim Rules of Procedure
Petition, in relation to the interest of Philippine Veterans Bank (PVB), on Corporate Rehabilitation (Interim Rules).
3. The proceedings before the RTC were initially held in abeyance
Planters Development Bank (PDB), and Land Bank of the Philippines (LBP),
which are listed as creditors of La Savoie vis-à-vis certain properties or assets that as La Savoie failed to attach to its Petition some of the requirements under
might have been taken cognizance of, and placed under the custody of the RTC Rule 4, Section 2 of the Interim Rules.
and[/]or the appointed Rehabilitation Receiver." Home Guaranty Corporation a. With La Savoie's compliance and finding its "petition to be
noted that through the "La Savoie Asset Pool Formation and Trust sufficient in form and substance," then RTC Judge Estela Perlas-
Agreement" (Trust Agreement), La Savoie obtained financing for some of its Bernabe issued the Stay Order staying the enforcement of all claims
projects through a securitization process in which Planters Development Bank as against La Savoie.
nominal issuer issued PI50 million in asset participation certificates dubbed as i. La Savoie is prohibited:
the "La Savoie Development Certificates" (LSDC certificates) to be sold to 1. selling, encumbering, transferring, or disposing in
investors. Home Guaranty Corporation argued that they should be excluded from any manner any of its properties except in the
the coverage of La Savoie's Petition for Rehabilitation because it and the investors ordinary course of business
on the LSDC certificates had "preferential rights" over the properties making up 2. making any payment of its liabilities outstanding
the Asset Pool as these "were conveyed as security or collaterals for the as of the date of the filing of the petition
redemption of the [LSDC certificates]." The issue in this case is WoN the ii. La Savoie’s suppliers of goods or services are likewise
properties comprising the Asset Pool should be excluded from the proceedings prohibited from withholding supply of goods and services
on La Savoie Development Corporation's Petition for Rehabilitation. The Court in the ordinary course of business for as long as it makes
ruled in the negative. The transfer of the properties comprising the Asset Pool payments for the services and goods supplied after the
was made in favor of Home Guaranty Corporation is invalid. Thus, the properties issuance of the stay order.|||
would then no longer be under the dominion of La Savoie. They would thus be iii. La Savoie is directed to pay in full all administrative
within the reach of rehabilitation proceedings and susceptible to the rule against expenses incurred after the issuance of the stay order.|||
preference of creditors. 4. Following the issuance of the Stay Order, La Savoie's creditors —
DOCTRINE: In this case, Sections 13.1 and 13.2 of the Contract of Guaranty Planters Development Bank, Philippine Veterans Bank, and Robinsons
call for the "prompt assignment and conveyance to [Home Guaranty Corporation] Savings Bank — filed their Comments and/or Oppositions.
of all the corresponding properties in the Asset Pool" that are held as security in 5. Home Guaranty Corporation filed an Opposition even though "it [was] not a
favor of the guarantor. Moreover, Sections 13.1 and 13.2 dispense with the need creditor of Petitioner.”
of conducting foreclosure proceedings, judicial or otherwise. Albeit requiring the a. It asserted that it had a "material and beneficial interest in the . . .
intervention of the trustee of the Asset Pool, Sections 13.1 and 13.2 spell out what Petition, in relation to the interest of Philippine Veterans Bank
is, for all intents and purposes, the automatic appropriation by the paying (PVB), Planters Development Bank (PDB), and Land Bank of the
guarantor of the properties held as security. This is thus a clear case of pactum Philippines (LBP), which are listed as creditors of La Savoie vis-à-
commissorium. It is null and void. Accordingly, whatever conveyance was made vis certain properties or assets that might have been taken
by Planters Development Bank to Home Guaranty Corporation in view of this cognizance of, and placed under the custody of the RTC and[/]or the
appointed Rehabilitation Receiver." conditions. Specifically, it asserted that for the assignment to take
6. Home Guaranty Corporation noted that through the "La Savoie Asset Pool effect, Home Guaranty Corporation had to first pay the holders of
Formation and Trust Agreement" (Trust Agreement), La Savoie obtained the LSDC certificates. Thus, La Savoie claimed that the properties
financing for some of its projects through a securitization process in which comprising the Asset Pool remained to be its assets
Planters Development Bank as nominal issuer issued PI50 million in asset 12. In the interim, a Verification Report on Accuracy of Petition was filed by the
participation certificates dubbed as the "La Savoie Development Rehabilitation Receiver.
Certificates" (LSDC certificates) to be sold to investors. 13. The RTC denied La Savoie’s Petition. La Savoie, then filed an Appeal before
a. The projects financed by these certificates consisted of the CA.
the development of real properties in General Trias, Cavite; Sto. 14. In the meantime, Home Guaranty Corporation approved and processed the
Tomas, Batangas; Los Baños, Laguna; and Quezon City. call on the guaranty for the redemption of the LSDC certificates.
b. The same properties were conveyed in trust by La Savoie, as trustor, a. Thus, Home Guaranty Corporation, through Planters Development
to Planters Development Bank, as trustee, and constituted into Bank, paid a total of P128.5 million as redemption value to
the La Savoie Asset Pool (Asset Pool). certificate holders. Acting on this, Planters Development Bank
7. The redemption of the LSDC certificates upon maturity and the interest executed a "Deed of Assignment and Conveyance" in favor of Home
payments on them were "backed/collateralized by the assets that were Guaranty Corporation through which, in the words of Home
conveyed by [La Savoie] to the Trust." Moreover, the LSDC certificates were Guaranty Corporation, Planters Development Bank "absolutely
covered by a guaranty extended by Home Guaranty Corporation through a conveyed and assigned to [Home Guaranty Corporation] the
"Contract of Guaranty" entered into by Home Guaranty Corporation with La ownership and possession of the entire assets that formed part of the
Savoie and Planters Development Bank. La Savoie Asset Pool."
8. Section 17 of the Contract of Guaranty designates Home Guaranty b. Home Guaranty Corporation claims, in addition, that, through the
Corporation to "undertake financial controllerships of the Projects." Thus, in same Deed, Planters Development Bank "absolutely conveyed and
its Opposition, Home Guaranty Corporation noted that it was "charged with assigned to [Home Guaranty Corporation] the right to collect from
the duty of ensuring that all funds due to the Asset Pool are collected, and [La Savoie] cash receivables . . . representing the amount collected
that funds are disbursed for the purposes they were intended for." by [La Savoie] from sales in the course of the development of the
9. Home Guaranty Corporation added that in the course of its business, La projects which it failed to remit to the Trust."
Savoie collected P60,569,134.30 from the buyers of some of the properties 15. The CA reversed the RTC’s order and reinstated the Stay Order, gave due
covered by the Asset Pool. course to the Petition for Rehabilitation, and remanded the case to the trial
a. This amount, however, was not remitted by La Savoie to the trust. court for further proceedings.
10. With La Savoie's failure to complete some of its projects and failure to remit 16. Home Guaranty Corporation filed before this court the present Petition for
sales collections: Review on Certiorari under Rule 45 of the 1997 Rules of Civil Procedure.
a. the Asset Pool defaulted in redeeming and paying interest on the
LSDC certificates. Thus, La Savoie's investors placed a call on the ISSUES:
guaranty. 1. WoN the properties comprising the Asset Pool should be excluded from the
b. With La Savoie's failure to remit collections, Home Guaranty proceedings on La Savoie Development Corporation's Petition for
Corporation held in abeyance the settlement of the investors' call. Rehabilitation
This settlement was then overtaken by the filing of La Savoie's a. WoN the conveyance to Home Guaranty Corporation of the
Petition for Rehabilitation. properties comprising the Asset Pool was valid and effectual – NO
11. Home Guaranty Corporation argued that they should be excluded from the i. WoN following the issuance of the RTC’s Order and
coverage of La Savoie's Petition for Rehabilitation because it and the pending La Savoie's Appeal, Home Guaranty Corporation
investors on the LSDC certificates had "preferential rights" over the was barred from making payment on the guaranty call, and
properties making up the Asset Pool as these "were conveyed as security or Planters Development Bank, concomitantly barred from
collaterals for the redemption of the [LSDC certificates]." conveying the properties comprising the Asset Pool to
a. La Savoie argued in a Consolidated Answer that the assignment of Home Guaranty Corporation - NO
assets to the Asset Pool was not absolute and subject to certain ii. WoN the payment by Home Guaranty Corporation and the
conveyance of the properties by Planters Development reference to orders on appeal, affirms that it is equally
Bank made Home Guaranty Corporation a creditor of La applicable to final orders. We entertain no doubt that Rule
Savoie - YES 3, Section 5 of the Interim Rules covered the trial court's
iii. WoN recognizing the validity of the transfer made to Home Order dismissing the Petition for Rehabilitation and lifting
Guaranty Corporation was tantamount to giving it the Stay Order The same Order was thus immediately
inordinate preference as a creditor -NO executory.
2. WoN Atty. Danilo C. Javier was authorized to sign the verification and c. The filing of La Savoie's Appeal did not restrain the
certificate of non-forum shopping of Home Guaranty Corporation's Petition effectivity of the Order.
-YES d. Rehabilitation proceedings are not bound by procedural
3. WoN Home Guaranty Corporation engaged in forum shopping -YES rules spelled out in the Rules of Court but under the Interim
Rules, which (at the time of the pivotal incidents in this case)
governed rehabilitation proceedings.
RULING: WHEREFORE, the Petition is DENIED. The Regional. Trial Court,
e. In Rule 3, Section 5, the Interim Rules explicitly carved an
Branch 142, Makati City is directed to proceed with dispatch in resolving the
exception to the general principle that an appeal stays the
Petition for Rehabilitation filed by respondent La Savoie Development Corporation.
judgment or final order appealed from. It explicitly requires
SO ORDERED. the issuance by the appellate court of an order enjoining or
restraining the order appealed from.
RATIO: f. Thus, the Order, lifting the restrictions on the payment of
1. The properties comprising the Asset Pool should be included in the claims against La Savoie, remained in effect.
proceedings on La Savoie Development Corporation's Petition for i. La Savoie's creditors were then free to enforce their
Rehabilitation
claims.
a. the conveyance to Home Guaranty Corporation of the properties ii. Conversely, La Savoie and "its guarantors and sureties
comprising the Asset Pool was invalid and ineffectual not solidarity liable with it" were no longer restrained
i. Home Guaranty Corporation was not barred from making from effecting payment.
payment on the guaranty call, and Planters Development Bank,
ii. The payment by Home Guaranty Corporation and the
concomitantly was not barred from conveying the properties
conveyance of the properties by Planters Development Bank
comprising the Asset Pool to Home Guaranty Corporation due to
made Home Guaranty Corporation a creditor of La Savoie
the lifting of the RTC’s Order 1. Section 13 of the Contract of Guaranty provides for how
1. Rule 3, Section 534 of the Interim Rules governs the effectivity guaranty claims are to be processed and paid by Home Guaranty
of orders issued in proceedings relating to the rehabilitation of Corporation. Likewise, it echoes Section 3.4 of the Trust
corporations, partnerships, and associations under Presidential Agreement in providing for transfer of the Asset Pool in the
Decree No. 902-A, as amended. event of a call on the guaranty:
a. Nowhere from its text can it be gleaned that it does not cover
a. Per Sections 13.135 and 13.236 of the Contract of Guaranty,
orders such as those issued by the trial court
the consequence of this payment was the execution by
b. If at all, its second sentence, which explicitly makes Planters Development Bank, as trustee of the Asset Pool,

34 a) Deed of Assignment and Conveyance to HIGC of the entire Asset Pool pursuant to the Trust
Sec. 5. Executory Nature of Orders. - Any order issued by the court under these Rules is immediately
Agreement;
executory. A petition for review or an appeal therefrom shall not stay the execution of the order unless
b) All tax declarations, transfer certificates of title, original certificates of title and official receipts
restrained or enjoined by the appellate court. The review of any order or decision of the court or an appeal
of payments of real estate taxes covering properties comprising the Asset Pool; and,
therefrom shall be in accordance with the Rules of Court: Provided, however, that the reliefs ordered by
c) All other documents and papers in the Asset Pool, as defined in the Trust Agreement.
the trial or appellate courts shall take into account the need for resolution of proceedings in a just, 36 Upon receipt of the acknowledgment by HIGC of the guaranty claim, the Trustee shall submit the
equitable, and speedy manner.
35 Upon receipt of the guaranty claim filed by the Trustee, HIGC shall have thirty (30) working days to documents and make a prompt assignment and conveyance to- HIGC of all the corresponding properties
in the Asset Pool pursuant to the Trust Agreement.
evaluate the guaranty claim. Within such period, HIGC shall acknowledge the guaranty claim and require
from the Trustee submission of the required documents, as follows:
of a Deed of Conveyance in favor of Home Guaranty does not apply to corporations who have sought to put
Corporation. themselves under receivership but, for lack of judicial
b. Ostensibly, all formal and substantive requisites for the sanction, have not been put under or are no longer under
execution of this Deed, as per the Trust Agreement and receivership.
the Contract of Guaranty, were fulfilled. b. The trial court's Order denied due course to and dismissed
c. La Savoie failed to intimate that any such condition or La Savoie's Petition for Rehabilitation.
requisite was not satisfied. i. It superseded the trial court's Stay Order appointing
iii. Recognizing the validity of the transfer made to Home Guaranty Rito C. Manzana as rehabilitation receiver and thereby
Corporation was not tantamount to giving it inordinate relieving him of his duties and removing La Savoie
preference as a creditor from receivership.
1. The execution of a Deed of Conveyance, which holds the transfer 2. Atty. Danilo C. Javier was authorized to sign the verification and
made to Home Guaranty Corporation, without resorting to certificate of non-forum shopping of Home Guaranty Corporation's
foreclosure is indicative of pactum commissorium. Petition
a. Sections 13.1 and 13.2 of the Contract of Guaranty a. Home Guaranty Corporation’s board of directors issued Board
i. call for the "prompt assignment and conveyance to Resolution No. 30, Series of 2001, "specifically authorizing its
[Home Guaranty Corporation] of all the corresponding President to designate the officer to institute the appropriate legal
properties in the Asset Pool" that are held as security in actions[.]"
favor of the guarantor. b. It was pursuant to this resolution that Atty. Danilo C. Javier, Home
ii. dispense with the need of conducting foreclosure Guaranty Corporation's then Officer-in-Charge and Vice President
proceedings, judicial or otherwise. for Legal, was made signatory to the present Petition's verification
b. Albeit requiring the intervention of the trustee of the Asset and Certification of non-forum shopping.
Pool, Sections 13.1 and 13.2 spell out what is, for all intents 3. Home Guaranty Corporation engaged in forum shopping
and purposes, the automatic appropriation by the paying a. However, the present Petition or Appeal, being a mere offshoot of
guarantor of the properties held as security. La Savoie's original Petition for Rehabilitation, is not the act
c. This is thus a clear case of pactum commissorium. It is null constitutive of forum shopping
and void. b. Forum shopping was committed through the filing of Civil Case No.
i. Accordingly, whatever conveyance was made by 05314 before the Regional Trial Court.
Planters Development Bank to Home Guaranty i. Another case between Home Guaranty Corporation and La
Corporation in view of this illicit stipulation is Savoie, docketed as Civil Case No. 05314, was pending
ineffectual. It did not vest ownership in Home Guaranty before the Makati City RTC.
Corporation. ii. Civil Case No. 05314 is an action for injunction,
2. La Savoie cites Article 2067 of the Civil Code and argues that mandamus, specific performance, and sum of money with
with Home Guaranty Corporation's payment of the LSDC application for temporary restraining order and/or
certificates' redemption value, Home Guaranty Corporation was preliminary prohibitory and mandatory injunction.
subrogated into the rights of La Savoie's creditors (i.e., the c. In both cases, Home Guaranty Corporation is invoking the same
certificate holders). It asserts that "effectively, HGC is already right and is proceeding from the same cause of action
the creditor of La Savoie" and that as creditor, it cannot be given d. True, there is divergence in the details of the specific reliefs it is
a preference over the assets of La Savoie, something that is seeking, but Home Guaranty Corporation is seeking the
"prohibited, in rehabilitation proceedings." same basic relief, i.e., the recognition of its alleged ownership.
3. It is true, as La Savoie asserts, that the suspension of the e. The exclusion of the properties from corporate rehabilitation
enforcement of claims against corporations under proceedings and the remittance to it of payments are mere incidents
receivership is intended "to prevent a creditor from obtaining of this basic relief.
an advantage or preference over another." f. Accordingly, in simultaneously pursuing the present case and Civil
a. It applies only to corporations under receivership.Plainly, it Case No. 05314, Home Guaranty Corporation engaged in forum
shopping.
006 Abilla v. Gobongseng that the transaction between the parties was not an
equitable mortgage. Citing Villarica v. Court of Appeals,[6]it ratiocinated that
G.R. No. 146651 | January 17, 2002 | Ynares-Santiago, J. neither was the said transaction embodied in the Deed of Sale and Option
PETITIONER: RONALDO P. ABILLA and GERALDA A. DIZON, to Buy a pacto de retro sale, but a sale giving respondents until August 31,
1983 within which to buy back the seventeen lots subject of the
RESPONDENTS: CARLOS ANG GOBONSENG, JR. and THERESITA controversy. The dispositive portion thereof reads:
MIMIE ONG
a. 1. Ordering defendants to pay plaintiffs the sum of
SUMMARY: Petitioners filed a case against respondents in regards to a certain P171,483.40 representing the total expenses incurred by plaintiffs in
transaction. Respondents raised the defense that the transaction covered by the the preparation and registration of the Deed of Sale, amount paid to
Deed of Sale and Option to Buy, which appears to be a Deed of Sale with Right the Bank of Asia and America (IBAA) and capital gains tax with
of Repurchase, was in truth, in fact, in law, and in legal construction, a mortgage. legal rate of interest from the time the same was incurred by
Respondents first argued that the transaction was a mrotgage, however, plaintiffs up to the time payment is made by defendants…
respondents abandoned their theory that the transaction was an equitable
mortgage and adopted the finding of the Court of Appeals that it was in fact 20. On appeal by respondents, the Court of Appeals ruled that the transaction
a pacto de retro sale. Respondents now insist that they are entitled to exercise the between the parties was a pacto de retro sale, and not an equitable mortgage.
right to repurchase pursuant to the third paragraph of Article 1606 of the Civil MR denied. Petitioner for review with SC denied for being filed out of time.
Code. However, the SC ruled that respondents are only entitled to the right MR also denied.
granted in the third paragraph of Art. 1606 and agreed with the the Court of 21. Undaunted, respondents filed a second motion for reconsideration, claiming
Appeals that if respondents really believed that the transaction was indeed an that since the transaction subject of the controversy was declared a pacto de
equitable mortgage, as a sign of good faith, they should have, at the very least, retro sale by the Court of Appeals, they can therefore repurchase the property
consigned with the trial court the amount of P896,000.00, representing their pursuant to the third paragraph of Article 1606 of the Civil Code. The issue
alleged loan, on or before the expiration of the right to repurchase on August 21, of the applicability of Article 1606 of the Civil Code was raised by the
1983. respondents only in their motion for clarification with the Court of Appeals,
DOCTRINE: The application of the third paragraph of Article 1606 is predicated and not before the trial court and on appeal to the Court of Appeals. Thus,
upon the bona fides of the vendor a retro. It must appear that there was a belief respondents second motion for reconsideration was denied.
on his part, founded on facts attendant upon the execution of the sale 22. On February 23, 1999, respondents filed with the trial court in Civil Case No.
with pacto de retro, honestly and sincerely entertained, that the agreement was in 8148 an urgent motion to repurchase the lots in question with tender of
reality a mortgage, one not intended to affect the title to the property ostensibly payment. The motion was, however, denied on November 10, 1999[14] by
sold, but merely to give it as security for a loan or other obligation. Judge Ibarra B. Jaculbe, Jr., who subsequently inhibited himself from the
case.

FACTS: 23. On January 14, 2001, Branch 41 of the RTCof Dumaguete City, to which the
case was reraffled, set aside the November 10, 1999 order and granted
17. Petitioner spouses instituted against respondents an action for specific respondents motion to repurchase.
performance, recovery of sum of money and damages, seeking the
reimbursement of the expenses they incurred in connection with the 24. Hence, the instant recourse.
preparation and registration of two public instruments, namely a Deed of ISSUES:
Sale[3] and an Option to Buy.
4. May the vendors in a sale judicially declared as a pacto de retro exercise the
18. In their answer, respondents raised the defense that the transaction covered right of repurchase under Article 1606, third paragraph, of the Civil Code,
by the Deed of Sale and Option to Buy, which appears to be a Deed of Sale after they have taken the position that the same was an equitable mortgage?
with Right of Repurchase, was in truth, in fact, in law, and in legal No.
construction, a mortgage.
RULING: WHEREFORE, in view of all the foregoing, the instant petition is
19. On October 29, 1990, the trial court ruled in favor of petitioners and declared
GRANTED and the January 14, 2001 Order of to give it as security for a loan or other obligation.
the Regional Trial Court of Dumaguete City, Branch 41, in Civil Case No. 8148,
b. In that event, if the matter of the real nature of the contract is
is REVERSED and SET ASIDE
submitted for judicial resolution, the application of the rule is meet
RATIO: and proper; that the vendor a retro be allowed to repurchase the
property sold within 30 days from rendition of final judgment
34. At the outset, it must be stressed that it has been respondents consistent claim
declaring the contract to be a true sale with right to
that the transaction subject hereof was an equitable mortgage and not
repurchase. Conversely, if it should appear that the parties
a pacto de retro sale or a sale with option to buy. Even after the Court of
agreement was really one of sale transferring ownership to the
Appeals declared the transaction to be a pacto de retro sale, respondents
vendee, but accompanied by a reservation to the vendor of the right
maintained their view that the transaction was an equitable mortgage.
to repurchase the property and there are no circumstances that may
35. Seeing the chance to turn the decision in their favor, however, respondents reasonably be accepted as generating some honest doubt as to the
abandoned their theory that the transaction was an equitable mortgage and parties' intention, the proviso is inapplicable. The reason is quite
adopted the finding of the Court of Appeals that it was in fact a pacto de obvious. If the rule were otherwise, it would be within the power of
retro sale. Respondents now insist that they are entitled to exercise the right every vendor a retro to set at naught a pacto de retro, or resurrect
to repurchase pursuant to the third paragraph of Article 1606 of the Civil an expired right of repurchase, by simply instituting an action to
Code, which reads: reform the contract known to him to be in truth a sale with pacto de
retro into an equitable mortgage.
a. However, the vendor may still exercise the right to repurchase
within thirty days from the time final judgment was rendered in a c. This Court has already had occasion to rule on the proper
civil action on the basis that the contract was a true sale with right interpretation of the provision in question. In Adorable v. Inacala,
to repurchase. where the proofs established that there could be no honest doubt as
to the parties intention, that the transaction was clearly and definitely
36. The question now is, can respondents avail of a sale with pacto de retro, the Court adjudged the vendor a retro not
the aforecited provision? Following the theory of the respondents which was to be entitled to the benefit of the third paragraph of Article 1606.
sustained by the trial court, the scenario would be that although respondents
failed in their effort to prove that the contract was an equitable mortgage, they 38. In the case at bar, both the trial court and the Court of Appeals were of the
could nonetheless still repurchase the property within 30 days from the view that the subject transaction was truly a pacto de retro sale; and that none
finality of the judgment declaring the contract to be truly a pacto de of the circumstances under Article 1602 of the Civil Code exists to warrant a
retro sale. However, under the undisputed facts of the case at bar, this cannot conclusion that the transaction subject of the Deed of Sale and Option to Buy
be allowed. was an equitable mortgage. The Court of Appeals correctly noted that if
respondents really believed that the transaction was indeed an equitable
37. In the parallel case of Vda. de Macoy v. Court of Appeals, the petitioners
mortgage, as a sign of good faith, they should have, at the very least,
therein raised the defense that the contract was not a sale with right to consigned with the trial court the amount of P896,000.00, representing their
repurchase but an equitable mortgage. They further argued as an alternative alleged loan, on or before the expiration of the right to repurchase on August
defense that even assuming the transaction to be a pacto de retro sale, they
21, 1983.
can nevertheless repurchase the property by virtue of Article 1606, third
paragraph of the Civil Code. It was held that the said provision was 39. Clearly, therefore, the declaration of the transaction as a pacto de retro sale
inapplicable, thus: will not, under the circumstances, entitle respondents to the right of
repurchase set forth under the third paragraph of Article 1606 of the Civil
a. The application of the third paragraph of Article 1606 is predicated Code.
upon the bona fides of the vendor a retro. It must appear that there
was a belief on his part, founded on facts attendant upon the
execution of the sale with pacto de retro, honestly and sincerely
entertained, that the agreement was in reality a mortgage, one not
intended to affect the title to the property ostensibly sold, but merely
007 FRANCISCO vs BOISER (Abs) another sister of petitioner.
May 31, 2000 | Mendoza, J. | Summary of case-law on Article 1623 18. On August 5, 1992, petitioner received summons, filed by respondent
PETITIONER: Adalia B. Francisco demanding her share in the rentals being collected by petitioner from the
RESPONDENT: Zenaida F. Boiser tenants of the building. Petitioner then informed respondent that she was
exercising her right of redemption as a co-owner of the subject property. On
SUMMARY: Francisco and three of her sisters were co-owners of four parcels August 12, 1992, she deposited the amount of P10,000.00 as redemption
of land. They sold 1/5 of their undivided shares to their mother, Adela Blas. In price with the Clerk of Court. However, said case was dismissed after
1986, without knowledge of her co-owner daughters, Blas sold her 1/5 share to respondent was declared non-suited with the result that petitioners
Boiser. In 1992, Boiser filed a civil case against Francisco for the collection of counterclaim was likewise dismissed.
her share of the rentals. Francisco informed Boiser that she will exercise her right 19. On September 14, 1995, petitioner instituted a civil case before the RTC in
of redemption as co-owner and deposited 10K as redemption price but said case Caloocan City. She alleged that the 30-day period for redemption under Art.
was dismissed. In 1995, Francisco filed a case wherein she alleged that the 30- 1623 of the Civil Code had not begun to run against her since the vendor,
day period for redemption under Art 1623 of NCC had NOT yet run against her Adela Blas, never informed her and the other owners about the sale to
since vendor Blas never informed her co-owners about the sale to Boiser. She respondent. She learned about the sale only on August 5, 1992, after she
only learned about it in 1992 after being informed of the complaint filed against received the summons of the case filed against her.
her by Boiser. Boiser, on the other hand, contends that Francisco knew of the sale 20. Respondent, on the other hand, contended that petitioner knew about the sale
earlier when she wrote a letter to Francisco about getting her share of the rentals. as early as May 30, 1992, because, on that date, she wrote petitioner a letter
Francisco confirmed that she received said letter. RTC dismissed said case informing the latter about the sale, with a demand that the rentals
because Art 1623 does not prescribe any particular form of notifying co-owners corresponding to her 1/5 share of the subject property be remitted to her. Said
of the sale for them to exercise their right of redemption. Thus, knowledge of the letter was sent with a copy of the Deed of Sale between respondent and Adela
sale via the letter is enough substantial compliance for notification under Art Blas. On the same date, letters were likewise sent by respondent to the tenants
1623. The sole issue is whether or not the letter served as substantial compliance of the building, informing them of the sale and requesting that, thenceforth,
of notification under Art 1623. The Supreme Court held that said letter by buyer they pay 1/5 of the monthly rentals to respondent. That petitioner received
Boiser is not enough because *see doctrine* these letters is proved by the fact that on June 8, 1992, she wrote the buildings
DOCTRINE: tenants advising them to disregard respondents request and continue paying
•For the 30-day redemption period to begin to run, notice must be given by seller full rentals directly to her.
(as in by Blas); notice given by the buyer or even by the Register of Deeds is not 21. RTC dismissed the petitioners complaint for legal redemption. It ruled that
sufficient. Art. 1623 does not prescribe any particular form of notifying co-owners about
•When notice is given by the proper party (seller), no particular form of written a sale of property owned in common to enable them to exercise their right of
notice is prescribed under Art. 1623, so that the furnishing of the copies of the legal redemption. While no written notice was given by the vendor, Adela
deeds of sale to the co-owner would be sufficient. Blas, to petitioner or the other owners, petitioner herself admitted that she
•Filing of suit for ejectment or collection of rentals against a co-owner actually had received respondents letter of May 30, 1992 and was in fact furnished a
dispenses with the written notice, and commences running of period to exercise copy of the deed evidencing such sale.
the right of redemption, since filing of the suit amounted to actual knowledge of 22. The trial court considered the letter sent by respondent to petitioner with a
the sale. copy of the deed of sale as substantial compliance with the required written
FACTS: notice under Art. 1623 of the New Civil Code. Consequently, the 30-day
16. Petitioner Adalia B. Francisco and three of her sisters, Ester, Elizabeth and period of redemption should be counted not from August 5, 1992, when
Adeluisa, were co-owners of four parcels of registered lands on which stands petitioner received summons in the first civil case, but at the latest, from June
the Ten Commandments Building at 689 Rizal Avenue Extension, Caloocan 8, 1992, the date petitioner wrote the tenants of the building advising them to
City. On August 6, 1979, they sold 1/5 of their undivided share in the subject continue paying rentals in full to her. Petitioner failed to redeem the property
parcels of land to their mother, Adela Blas, for P10,000.00, thus making the within that period.
latter a co-owner of said real property to the extent of the share sold. 23. CA affirmed RTC’s decision.
17. On August 8, 1986, without the knowledge of the other co-owners, Adela
Blas sold her 1/5 share for P10,000.00 to respondent Zenaida Boiser who is ISSUES: WoN the letter of May 30, 1992 sent by respondent to petitioner notifying
her of the sale on August 8, 1986 of Adela Blas 1/5 share of the property to respondent, and not by the buyer, are easily divined. The seller of an undivided interest
containing a copy of the deed evidencing such sale, can be considered sufficient as is in the best position to know who are his co-owners that under the law
compliance with the notice requirement of Art. 1623 for the purpose of legal must be notified of the sale. Also, the notice by the seller removes all doubts
redemption? –NO. as to fact of the sale, its perfection, and its validity, the notice being a
RULING: WHEREFORE, in view of the foregoing, the petition is GRANTED and reaffirmation thereof; so that that party notified need not entertain doubt that
the decision of the Court of Appeals is REVERSED and the Regional Trial Court, the seller may still contest the alienation. This assurance would not exist if
Branch 122, Caloocan City is ordered to effect petitioners exercise of her right of legal the notice should be given by the buyer.
redemption in Civil Case No. C-17055. 14. Art. 1623 of the Civil Code is clear in requiring that the written
RATIO: notification should come from the vendor or prospective vendor, not from
9. The trial court and the Court of Appeals relied on the ruling in Distrito v. any other person. There is, therefore, no room for construction.
Court of Appeals that Art. 1623 does not prescribe any particular form of 15. Now, it is clear that by not immediately notifying the co-owner, a vendor can
written notice, nor any distinctive method for notifying the redemptioner. delay or even effectively prevent the meaningful exercise of the right of
They also invoked the rulings in De Conejero v. Court of Appeals and Badillo redemption. In the present case, for instance, the sale took place in 1986, but
v. Ferrer that furnishing the redemptioner with a copy of the deed of sale is it was kept secret until 1992 when vendee (herein respondent) needed to
equivalent to giving him the written notice required by law. notify petitioner about the sale to demand 1/5 rentals from the property sold.
10. On the other hand, petitioner points out that the cited cases are not relevant Compared to serious prejudice to petitioners right of legal redemption, the
because the present case does not concern the particular form in which notice only adverse effect to vendor Adela Blas and respondent-vendee is that the
must be given. Rather, the issue here is whether a notice sent by the vendee sale could not be registered. It is non-binding, only insofar as third persons
may be given in lieu of that required to be given by the vendor or prospective are concerned. It is, therefore, unjust when the subject sale has already
vendor. been established before both lower courts and now, before this Court, to
11. In ruling that the notice given by the vendee was sufficient, the appellate court further delay petitioners exercise of her right of legal redemption by
cited the case of Etcuban v. Court of Appeals. In said case, notice to the co- requiring that notice be given by the vendor before petitioner can
owners of the sale of the share of one of them was given by the vendees exercise her right. For this reason, we rule that the receipt by petitioner of
through their counterclaim in the action for legal redemption. Despite the summons in Civil Case No. 15510 on August 5, 1992 constitutes actual
apparent meaning of Art. 1623, it was held in that case that it was "of no knowledge on the basis of which petitioner may now exercise her right of
moment" that the notice of sale was given not by the vendor but by the redemption within 30 days from finality of this decision.
vendees. "So long as the [co-owner] is informed in writing of the sale and the 16. Our ruling is NOT without precedent. In Alonzo v. Intermediate Appellate
particulars thereof, the 30 days for redemption start running, and the Court, we dispensed with the need for written notification considering that
redemptioner has no cause to complain," so it was held. The contrary doctrine the redemptioners lived on the same lot on which the purchaser lived and
of Butte v. Manuel Uy and Sons, Inc. was thus overruled sub silencio. were thus deemed to have actual knowledge of the sales. We stated that the
12. However, in the later case of Salatandol v. Retes, decided a year after the 30-day period of redemption started, not from the date of the sales in 1963
Etcuban case, the Court expressly affirmed the ruling in Butte that the notice and 1964, but sometime between those years and 1976, when the first
required by Art. 1623 must be given by the vendor. In Salatandol, the notice complaint for redemption was actually filed. For 13 years, however, none of
given to the redemptioner by the Register of Deeds of the province where the the co-heirs moved to redeem the property. We thus ruled that the right of
subject land was situated was held to be insuffucient. Resolving the issue of redemption had already been extinguished because the period for its exercise
whether such notice was equivalent to the notice from the vendor required had already expired.
under Art. 1623, this Court stated: The appeal is impressed with merit. In 17. In the present case, as previously discussed, receipt by petitioner of summons
Butte vs. Manuel Uy and Sons, Inc., the Court ruled that Art. 1623 of the in the first civil case on August 5, 1992 amounted to actual knowledge of the
Civil Code clearly and expressly prescribes that the thirty (30) days for sale from which the 30-day period of redemption commenced to run.
making the pre-emption or redemption are to be counted from notice in Petitioner had until September 4, 1992 within which to exercise her right
writing by the vendor. of legal redemption, but on August 12, 1992 she deposited the P10,000.00
13. The test of Article 1623 clearly and expressly prescribes that the thirty days redemption price. As petitioners exercise of said right was timely, the
for making the redemption are to be counted from notice in writing by the same should be given effect.
vendor. The reasons for requiring that the notice should be given by the seller,
008 SORIANO v. BAUTISTA (Ram) b. Spouses Soriano – Mortgagees
December 29, 1962 | Makalintal, J. | Right to Redeem a Mortgaged Property & 12. In May 30, 1956, Spouses Bautista entered into an agreement entitled
Option to Buy Kasulatan ng Sanglaan (mortgage) in favor of Spouses Soriano for the
PLAINTIFFS-APPELLEES: Ruperto Soriano et al (1st case) ; Basilio Bautista amount of P1,800.
et al (2nd case) a. The mortgage will last for 2 years
DEFENDANTS-APPELLANTS: Basilio Bautista and Sofia De Rosas (1st b. The fruits of the farm will belong to the Spouses Soriano as guaranty
case); Ruperto Soriano (2nd case) for the loan
c. Taxes shall be paid by the Spouses Bautista
SUMMARY: Spouses Bautista are the absolute and registered owners of a parcel d. The said land cannot be further mortgaged without the consent of
of land in Rizal. In May 30, 1956, Spouses Bautista mortgaged the said land in the Spouses Soriano
favor of Spouses Soriano for the amount of P1,800 with a stipulation in paragraph e. The Spouses Soriano may purchase the said land absolutely within
5, “the Spouses Soriano may purchase the said land absolutely within the 2-year the 2-year term of the mortgage for P3,900 if their financial
term of the mortgage for P3,900 if their financial condition permits.” A certain condition permits
Atty. Angel O. Ver informed the spouses Bautista that the spouses Soriano have f. However, failure to pay or failure to comply with the conditions
decided to purchase the subject property pursuant to par. 5 of the document. The within the said period the law on Foreclosure of Mortgages, Judicial
spouses Bautista refused to comply with Soriano’s demand. As such, spouses or Extrajudicial will apply
Soriano filed a case, praying that they be allowed to consign or deposit with the 13. Simultaneously with the signing of the deed, the spouses Bautista transferred
Clerk of Court the sum of P1,650 as the balance of the purchase price of the land the possession of the subject property to spouses Soriano. The spouses
in question. Subsequently spouses Bautista filed another case against Soriano, Soriano have, since that date, been in possession of the property and are still
asking the court to order Soriano to accept the payment of the principal obligation enjoying the produce thereof to the exclusion of all other persons.
and release the mortgage and to make an accounting the harvest for the 2 harvest 14. Sometime after May 1956, the spouses Bautista received from spouses
seasons (1956-1957). CFI ruled in Soriano’s favor and ordered the Spouses Soriano the sum of P450 pursuant to the conditions agreed upon in the
Bautista to execute of the deed of sale upon payment of the Spouses Soriano of document. However, no receipt was issued. The said amount was returned by
P1,650. Spouses Bautista argued that as mortgagors, they cannot be deprived of the spouses Bautista.
the right to redeem the mortgaged property, as such right is inherent in and 15. In May 13, 1958, a certain Atty. Angel O. Ver informed the spouses Bautista
inseparable from a mortgage. WoN Spouses Bautista are entitled to redeem the that the spouses Soriano have decided to purchase the subject property
subject property? – NO. pursuant to par. 5 of the document which states that “…the mortgagees
DOCTRINE: may purchase the said land absolutely within the 2-year term of the mortgage
While the transaction is undoubtedly a mortgage and contains the customary for P3,900.”
stipulation concerning redemption, it carries the added special provision, which 16. Despite the receipt of the letter, the spouses Bautista refused to comply with
renders the mortgagor’s right to redeem defeasible at the election of the Soriano’s demand
mortgagees. There is nothing illegal or immoral in this as this is allowed under 17. As such, spouses Soriano filed a case, praying that they be allowed to consign
Art 1479 NCC. The mortgagor’s promise was in the nature of a continuing or deposit with the Clerk of Court the sum of P1,650 as the balance of the
offer, non-withdrawable during a period of 2 years, which upon acceptance purchase price of the land in question
by the mortgagees gave rise to a perfected contract of sale. Bautista’s offer to 18. Spouses Bautista also filed a complaint, but was dismissed for lack of
redeem could be defeated by Soriano’s preemptive right to purchase within the jurisdiction.
period of 2 years from May 30, 1956. 19. Subsequently spouses Bautista filed another case against Soriano, asking the
court to order Soriano to accept the payment of the principal obligation and
A stipulation to render the right to redeem defeasible by an option to buy on the release the mortgage and to make an accounting the harvest for the 2 harvest
part of the creditor. Note: (from the syllabus) seasons (1956-1957).
20. CFI decision
FACTS: a. Held in Soriano’s favor and ordered the Spouses Bautista to execute
11. Spouses Bautista are the absolute and registered owners of a parcel of land in of the deed of sale upon payment of the Spouses Soriano of P1,650
Rizal. (balance of the price agreed upon plus amount previously received
a. Spouses Bautista – Mortgagors through the loan – from P3,900)
21. Spouses Bautista argued that as mortgagors, they cannot be deprived of the the right within one year after the sale of the real estate, to redeem the property by
right to redeem the mortgaged property, as such right is inherent in and paying the amount due under the mortgage deed, with interest thereon at the rate
inseparable from a mortgage. specified in the mortgage, and all the costs and expenses incurred by the bank or
institution from the sale and custody of said property less the income derived
ISSUES: therefrom. However, the purchaser at the auction sale concerned whether in a judicial
4. WoN Spouses Bautista are entitled to redeem the subject property? – NO. or extrajudicial foreclosure shall have the right to enter upon and take possession of
such property immediately after the date of the confirmation of the auction sale and
RULING: The judgment appealed from is affirmed, with costs. administer the same in accordance with law. Any petition in court to enjoin or restrain
the conduct of foreclosure proceedings instituted pursuant to this provision shall be
RATIO: given due course only upon the filing by the petitioner of a bond in an amount fixed
1. While the transaction is undoubtedly a mortgage and contains the by the court conditioned that he will pay all the damages which the bank may suffer
customary stipulation concerning redemption, it carries the added special by the enjoining or the restraint of the foreclosure proceeding.
provision, which renders the mortgagor’s right to redeem defeasible at the
election of the mortgagees. There is nothing illegal or immoral in this as this
is allowed under Art 1479 NCC which states:
a. “A promise to buy and sell a determinate thing for a price certain is
reciprocally demandable. An accepted unilateral promise to buy or
to sell a determinate thing for a price certain is binding upon the
promisor if the promise supported by a consideration apart from the
price.”
2. In the case at bar, the mortgagor’s promise is supported by the same
consideration as that of the mortgage itself, which is distinct from the
consideration in sale should the option be exercised. The mortgagor’s
promise was in the nature of a continuing offer, non-withdrawable during
a period of 2 years, which upon acceptance by the mortgagees gave rise to
a perfected contract of sale.
3. Tender Ineffective as Preemptive Right to Purchase by Other Party has been
a. The tender of P1,800 to redeem the mortgage by spouses Bautista
was ineffective for the purpose intended.
b. Such tender must have been made after the option to purchase
had been exercised by spouses Soriano.
c. Bautista’s offer to redeem could be defeated by Soriano’s
preemptive right to purchase within the period of 2 years from May
30, 1956.
d. Such right was availed of and spouses Bautista were accordingly
notified by Soriano. Offer and acceptance converged and gave rise
to a perfected and binding contract of purchase and sale.

ADDITIONAL NOTES:
Redemption in Judicial Foreclosure of Mortgage (Sec. 47, R.A. 8791)

SECTION 47. Foreclosure of Real Estate Mortgage. — In the event of foreclosure,


whether judicially or extrajudicially, of any mortgage on real estate which is security
for any loan or other credit accommodation granted, the mortgagor or debtor whose
real property has been sold for the full or partial payment of his obligation shall have
001 NYCO SALES CORP V BA FINANCE (Nicolin)
August 16, 1991 | Paras, J | Warranties of Assignor

PETITIONER: Nyco Sales Corporation


RESPONDENTS: BA Finance Corporation, Judge Rosalio De Leon (RTC), IAC,
First Civil Cases Division

SUMMARY: Nyco Sales Corporation, represented by Ruffino Yao (as president and FACTS:
GM), granted Sanshell, represented by the Fernandez Brothers (Santiago and 5. Nyco Sales Corporation (Nyco) is engaged in the business of selling
Renato), discounting privileges, which Nyco had with BA Finance. The Fernandez construction materials with principal office in Davao City. Ruffino Yao
brothers approached Yao to discount Sanshell’s post-dated BPI check, which Nyco (Yao) is the president and general manager.
(through Yao) endorsed in favor of BA Finance. In turn, BA Finance issued a check 6. Sometime in 1978, the brothers Santiago and Renato Fernandez (Brothers),
payable to Nyco who endorsed it to Sanshell. Subsequently, Sanshell made both acting in behalf of Sanshell Corporation, approached Rufino Yao for
use/negotiated the check. Upon presentment for payment, the BPI check was credit accommodation.
dishonored. When BA Finance reported this to the Fernandez brothers, they issed a a. The brothers requested Nyco (through Yao) to grant Sanshell
SBTC check as substitute, but was also dishonored. Despite repeated demands, Nyco discounting privileges which Nyco had with BA Finance.
and the brothers failed to settle their obligation with BA Finance, so BA Finance b. Yao acquiesced so the brothes went to Yao to discount Sanshell’s
instituted an action in court. Lower court ruled in favor of BA Finance ordering Nyco post-dated check (BPI-Davao Branch Check No. 499648) dated
and the Brothers to pay jointly and severally. Nyco appealed the said decision but February 17, 1979, for Php 60,000. The check was payable to Nyco.
CA upheld the decision in favor of BA Finance. Hence, this petition. The issue is 7. Following the discounting process agreed upon, Nyco, thru Yao, endorsed
whether or not the assignor (Nyco) is liable to its assignee (BA Finance) for its the check in favor of BA Finance. BA Finance issued a check payable to
dishonored checks. SC held yes. According to Article 1628 of the Civil Code, the Nyco which endorsed it in favor of Sanshell. Sanshell then made use of
assignor-vendor warrants both the credit itself (its existence and legality) and the and/or negotiated the check.
person of the debtor (his solvency), if so stipulated, as in the case at bar. Hence, the a. Accompanying the exchange of checks, a Deed of Assignment was
assignor-vendor should be held answerable for the invalidity of whatever he assigned executed by Nyco in favor of BA Finance with the conformity of
to the assignee-vendee i.e. by virtue of the Deed of Assignment executed by Nyco in Sanshell where the subject of the discounting was the BPI check.
favor of BA finance, Nyco shall be held liable if there be any breach of the above i. At the back of every deed of assignment was the
warranties. Further, Nyco’s pretension that it had not been notified of the fact of Continuing Suretyship Agreement whereby the brothers
dishonor of the SBTC check is belied by the formal demand letter issued by BA unconditionally guaranteed to BA Finance the full, faithful
Finance and the frequent contacts between Nyco and Sanshell before, during and and prompt payment and discharge of any and all
after the dishonor. There was also no novation when BA Finance accepted the SBTC indebtedness of Nyco.
check as substitute because there was no express agreement that BA Finance's b. However, the BPI check was dishonored by BPI upon presentment
acceptance of the SBTC check will discharge Nyco from liability. Lastly, Nyco is for payment. BA Finance immediately reported to the brothers who
estopped from denying Rufino Yao's authority as far as the latter's transactions with then issued a substitute check (Security Bank and Trust Company
BA Finance are concerned. Check No. 183157) for the same amount in favor of BA Finance,
which was also dishonored.
DOCTRINE: General rule: Assignor warrants only the existence or legality of the 8. Despite repeated demands, Nyco and the brothers failed to settle their
credit but not the solvency of the debtor. Exception: if so stipulated, as in the case obligation with BA Finance, so BA Finance instituted an action in court.
at bar. a. Both Nyco and the brothers were served summons and copies of the
complaint but failed to file their answer and subsequently declared
in default.
9. Lower court ruled in favor of BA Finance ordering Nyco and the Brothers to
pay jointly and severally the sum of Php 65, 536.67 plus 14% interest per
annum from July 1, 1979 and attorney’s fees in the amount of Php 3,000 and i. Check merely evidenced the credit, which was actually
costs. Nyco moved to set aside the order of default assigned to BA Finance. Designation is immaterial as it
a. Nyco, however, moved to set aside the order of default, to have its could be any other check
answer admitted and to be able to implead Sanshell. Prayer was ii. Nyco cannot be held liable for both BPI and SB checks as
granted and decision was set aside only as to Nyco. Judgment nowhere in the decisions of lower and appellate courts said
rendered in favor of BA Finance and against Nyco. BA Finance can recover from two checks.
10. On appeal, CA also upheld BA Finance but modified the running period to
Feb 19 1979 until paid, not from Feb 1, 1979. MR denied. Hence, this NOTICE OF DISHONOR WAS GIVEN AND LIABILITY ON CREDIT
petition. SUBSISTS
2. Nyco's pretension that it had not been notified of the fact of dishonor (of the
ISSUE/s: SB check) is belied not only by the formal demand letter but also by the
1. W/N the assignor (Nyco) is liable to its assignee (BA Finance) for its findings of the trial court that Rufino Yao of Nyco and the Fernandez
dishonored checks – Yes. Assignor-vendor is held liable for the invalidity of Brothers of Sanshell had frequent contacts before, during and after the
whatever he assigned to assignee-vendee. dishonor
3. Also, as long as the credit remains outstanding, Nyco shall continue to be
RULING: PREMISES CONSIDERED, the decision appealed from is AFFIRMED. liable to BA Finance as its assignor. The cause of action stems from the
SO ORDERED. breach of the warranties embodied in the Deed of Assignment, and not from
the dishonoring of the check alone (See Article 1628 CC)
a. The dishonor of an assigned check simply stresses its liability and
RATIO:
the failure to give a notice of dishonor will not discharge it from
EXECUTION OF DEED OF ASSIGNMENT BY NYCO
such liability.
1. An assignment of credit is the process of transferring the right of the
assignor to the assignee, who would then be allowed to proceed against the
debtor. It may be done either gratuitously or generously, in which case, the NO NOVATION
assignment has an effect similar to that of a sale. 1. SC found the defense of novation took place when BA finance accepted the
SB check in replacement of the dishonored BPI check untenable.
2. Article 1628 of the Civil Code37, the assignor-vendor warrants both the
a. First, novation must be explicitly stated and declared in unequivocal
credit itself (its existence and legality) and the person of the debtor (his
solvency), if so stipulated, as in the case at bar. terms as novation is never presumed (Mondragon v. Intermediate
a. Consequently, if there be any breach of the above warranties, the Appellate Court)
assignor-vendor should be held answerable. b. Secondly, the old and the new obligations must be incompatible on
every point.
b. There is no question then that the assignor-vendor is indeed liable
2. In the instant case, there was no express agreement that BA Finance's
for the invalidity of whatever he assigned to the assignee-vendee
acceptance of the SBTC check will discharge Nyco from liability. Neither is
3. It is undisputed that Nyco executed a deed of assignment in favor of BA
there incompatibility because both checks were given precisely to terminate
Finance with Sanshell Corporation as the debtor-obligor
a. BA Finance is actually enforcing said deed and the check covered a single obligation arising from Nyco's sale of credit to BA Finance. As
thereby is merely an incidental or collateral matter novation speaks of two distinct obligations, such is inapplicable to this case

YAO AUHTORIZED TO APPLY FOR CREDITACCOMMODATION

37
Article 1628. The vendor in good faith shall be responsible for the existence and Even in these cases he shall only be liable for the price received and for the expenses
legality of the credit at the time of the sale, unless it should have been sold as doubtful; specified in No. 1 of Article 1616.
but not for the solvency of the debtor, unless it has been so expressly stipulated or
unless the insolvency was prior to the sale and of common knowledge. The vendor in bad faith shall always be answerable for the payment of all expenses,
and for damages (1529).
1. Nyco disowns its President's acts claiming that it never authorized Yao to
even apply to BA Finance for credit accommodation. It supports its argument
with the fact that it did not issue a Board resolution giving Yao such authority.
However, the very evidence on record readily belies Nyco's contention.
2. Its corporate By-Laws clearly provide for the powers of its President, which
include, inter alia, executing contracts and agreements, borrowing money,
signing, indorsing and delivering checks, all in behalf of the corporation.
3. Also, there was already a previous transaction of discounting of checks
involving the same personalities wherein any enabling resolution from Nyco
was dispensed with and yet BA Finance was able to collect from Nyco and
Sanshell was able to discharge its own undertakings.
a. Thus, Nyco is placed under estoppel in pais which arises when
one, by his own representations or admissions, or by his silence
when he ought to speak out, intentionally or through culpable
negligence, induces another to believe certain facts to exist and such
other rightfully relies and acts on such belief, so that he will be
prejudiced if the former is permitted to deny the existence of such
facts.
4. Nyco remained silent in the course of the transaction and spoke out only later
to escape liability. This cannot be countenanced. Nyco is estopped from
denying Rufino Yao's authority as far as the latter's transactions with BA
Finance are concerned.
002 LICAROS v. GATMAITAN (JOSEF) placements and potentially profitable capital ventures in Hongkong, Europe
August, 9, 2001 | Gonzaga-Reyes, J. | Conventional Subrogation v. Assignment and the United States for the purpose of maximizing the returns on those
investments.
PETITIONER: Abelardo B. Lecaros 2. Abelardo Licaros, a Filipino businessman, decided to make a fund placement
RESPONDENTS: Antonio B. Gatmaitan with said bank.
3. Unfortunately, Licaros, after having invested in Anglo-Asean, encountered
SUMMARY: Abelardo Licaros (Licaros) decided to make a fund placement (invest) tremendous and unexplained difficulties in retrieving, not only the interest or
in the Anglo-Asean Bank and Trust Limited (Anglo-Asean). Unfortunately, when profits, but even the very investments he had put in Anglo-Asean.
Licaros decided to retrieve his investment together with the interest, he encountered 4. Licaros then decided to seek the counsel of Antonio P. Gatmaitan, a reputable
difficulty and subsequently he failed to retrieve such. Licaros then decided to seek banker and investment manager who had been extending managerial,
the counsel of Antonio Gatmaitan (Gatmaitan). To Licaros relief, Gatmaitan financial and investment consultancy services to various firms and
voluntarily offered to assume the payment of the indebtedness of Anglo-Asean to corporations both here and abroad.
Licaros and their agreement was formalized in a Memorandum of Agreement. 5. To Licaros relief, Gatmaitan was only too willing enough to help. Gatmaitan
However, when Gatmaitan made the effort himself of collecting the amount of voluntarily offered to assume the payment of Anglo-Aseans indebtedness
investments from Anglo-Asean by showing the Memorandum of Agreement, the to Licaros subject to certain terms and conditions. In order to effectuate and
former was not able to elicit any formal response from the latter. Because of this, formalize the parties respective commitments, the two executed a notarized
Gatmaitan reneged on his promise to undertake the obligation of Anglo-Asean to MEMORANDUM OF AGREEMENT.
Licaros. Licaros now filed the complaint to collect from Gatmaitan the amount of the 6. Thereafter, Gatmaitan presented to Anglo-Asean the Memorandum of
PN that the latter issued. The issues are: 1.) WoN the Memorandum of Agreement is Agreement earlier executed by him and Licaros for the purpose of collecting
an assignment or conventional subrogation – Conventional Subrogation as this was the latters placement thereat of U.S.$150,000.00. However, no formal
shown by the WHEREAS clause of their Memorandum of Agreement (please see response was ever made by said bank to either Licaros or Gatmaitan
ratio no. 6) wherein the parties intended to secure the express consent of Anglo- (take note of this because fact because as can be gleaned later from the ratio
Asean; 2.) WoN the Memorandum of Agreement was perfected – No, because as part, the consent of Anglo-Asean bank being the original debtor is necessary
earlier said from the first issue, the agreement requires the consent of Anglo-Asean. in this case). To date, Anglo-Asean has not acted on Gatmaitans monetary
However, the latter did not give its express consent, thereby making the claims.
Memorandum of Agreement as ineffective as to any of the parties thereto. 7. Because of his inability to collect from Anglo-Asean, Gatmaitan did not
bother anymore to make good his promise to pay Licaros the amount stated
DOCTRINE: (This is the doctrine in the syllabus and this is basically the same as in his promissory note. Licaros, however, thought differently. He felt that he
the one in the ratio part but just condensed) Subrogation extinguishes the obligation had a right to collect on the basis of the promissory note regardless of the
and gives rise to a new one; assignment refers to the same right, which passes from outcome of Gatmaitan's recovery efforts. Thus, Licaros, thru counsel,
one person to another. Nullity of an old obligation may be cured by subrogation, such addressed successive demand letters to Gatmaitan, demanding payment of
that a new obligation will be perfectly valid; but such nullity is not remedied by the the latters obligations under the promissory note. Gatmaitan, however, did
assignment of the creditor’s right to another. In an assignment of credit, the consent not accede to these demands.
of the debtor is not necessary in order that the assignment may fully produce legal 8. Hence, Licaros filed a complaint in the RTC to collect for the amount.
effects; whereas, conventional subrogation requires an agreement among the three 9. Trial court rendered judgment in favor of Licaros and found Gatmaitan liable
parties concerned – original creditor, debtor, and new creditor. It is a new contractual under the Memorandum of Agreement and Promissory Note.
relation based on the mutual agreement among all the necessary parties. 10. Respondent Gatmaitan appealed to the CA and the appellate court reversed
the decision of the trial court and held that respondent Gatmaitan did not at
any point become obligated to pay to petitioner Licaros the amount stated in
FACTS: the promissory note. The MR was also denied.
1. The Anglo-Asean Bank and Trust Limited (Anglo-Asean), is a foreign 11. Hence this petition for review on certiorari.
private bank. Its business consists primarily in receiving fund placements
by way of deposits from institutions and individual investors from different ISSUE/s: (note: guys basically the issue is WoN Licaros can collect from Gatmaitan
parts of the world and there after investing such deposits in money market but the court finds it necessary – and they did not directly address the issue as WoN
licaros can collect from Gatmaitan – to answer first the questions below because the 2. In an assignment of credit, the consent of the debtor is not necessary in order
issue of collection hinges on the answer of the issues below since related yung answer that the assignment may fully produce legal effects. What the law requires in
sa issue no.2 sa answer sa no.1) an assignment of credit is not the consent of the debtor but merely notice to
12. (RELEVANT IN OUR TOPIC) WoN the Memorandum of Agreement him (debtor) as the assignment takes effect only from the time he has
between Licaros and Gatmaitan is one of assignment of credit or one of knowledge thereof. A creditor may, therefore, validly assign his credit and
conventional subrogation – CONVENTIONAL SUBROGATION as its accessories without the debtors consent.
shown by the WHEREAS clause (fact no. 5) of the Memorandum of
Agreement of the parties wherein Licaros and Gatmaitan stipulated that 3. On the other hand, conventional subrogation requires an agreement among
the express consent of Anglo-Asean as the original debtor must be given the three parties concerned the original creditor (Licaros), the debtor
13. WoN the Memorandum of Agreement was perfected (If perfected, then (Anglo-Asean), and the new creditor (Gatmaitan). It is a new contractual
Gatmaitan still has subsisting commitment to pay Licaros, otherwise the suit relation based on the mutual agreement among all the necessary
for collection must fail) – NO, the absence of consent on the part of one of parties. Thus, Article 1301 of the Civil Code explicitly states that
the parties (Anglo-Asean) prevented the agreement to become effective Conventional subrogation of a third person requires the consent of the
original parties and of the third person.
RULING: WHEREFORE, the instant petition is DENIED and the Decision of the
Court of Appeals dated February 10, 2000 and its Resolution dated April 7, 2000 are 4. The appellate court (correct ruling) held that the agreement was one of
hereby AFFIRMED. conventional subrogation which necessarily requires the agreement of all the
parties concerned. The Court of Appeals thus ruled that the
RATIO: Memorandum of Agreement never came into effect due to the failure of
FIRST ISSUE the parties to get the consent of Anglo-Asean Bank (debtor) (remember
that in Conventional subrogation, the consent of ALL parties are required)
Assignment Subrogation to the agreement and, as such, respondent never became liable for the
The process of transferring the right of The transfer of all the rights of the amount stipulated.
the assignor to the assignee who would creditor to a third person, who
then have the right to proceed against substitutes him in all his rights. 5. Pronouncement of the CA:
the debtor.
Immediately discernible from above is the common feature of contracts
May be done gratuitously or onerously May either be legal (by operation of involving conventional subrogation, namely, the approval of the debtor to the
(same effect with sale) law) or conventional (by agreement of subrogation of a third person in place of the creditor. That Gatmaitan and
the parties) Licaros had intended to treat their agreement as one of conventional
subrogation is plainly borne by a stipulation in their Memorandum of
Consent of debtor is not required In Conventional subrogation, consent of Agreement, to wit:
original debtor is required
Refers to the same right which passes Extinguishes the obligation and creates WHEREAS, the parties herein have come to an agreement on the
from one person to another. a new one nature, form and extent of their mutual prestations which they
now record herein with the express conformity of the third
The nullity of an obligation is not The nullity of an old obligation may be parties concerned, which third party is admittedly Anglo-Asean
remedied by the assignment of the cured by subrogation, such that a new Bank.
creditors right to another. obligation will be perfectly valid.
6. Aside for the whereas clause cited by the appellate court in its
decision, we likewise note that on the signature page, right under the
1. For our purposes, the crucial distinction deals with the necessity of the place reserved for the signatures of petitioner and respondent, there is,
consent of the debtor in the original transaction. typewritten, the words WITH OUR CONFORME. Under this
notation, the words ANGLO-ASEAN BANK AND TRUST were
written by hand. To our mind, this provision which contemplates conformity of third parties, which admittedly was Anglo-Asean Bank, is a
the signed conformity of Anglo-Asean Bank, taken together with mere surplusage which is not necessary to the validity of the agreement.
the aforementioned preambulatory clause leads to the conclusion o SC: As previously discussed, the intention of the parties to treat
that both parties intended that Anglo-Asean Bank should signify the Memorandum of Agreement as embodying a conventional
its agreement and conformity to the contractual arrangement subrogation is shown not only by the whereas clause but also by
between petitioner and respondent. the signature space captioned WITH OUR CONFORME
reserved for the signature of a representative of Anglo-Asean
7. Had the intention been merely to confer on appellant the status of a Bank (Ratio No. 6). These provisions in the aforementioned
mere assignee of appellees credit, there is simply no sense for them to Memorandum of Agreement may not simply be disregarded or
have stipulated in their agreement that the same is conditioned on the dismissed as superfluous. It is a basic rule in the interpretation of
express conformity thereto of Anglo-Asean Bank. contracts that (t)he various stipulations of a contract shall be
interpreted together, attributing to the doubtful ones that sense
SECOND ISSUE which may result from all of them taken jointly.

8. Here, it bears stressing that the subject Memorandum of Agreement 11. 3rd argument: Petitioner next argues that assuming that the conformity of
expressly requires the consent of Anglo-Asean to the Anglo-Asean was necessary to the validity of the Memorandum of
subrogation. Upon whom the task of securing such consent devolves, Agreement, respondent only had himself to blame for the failure to secure
be it on Licaros or Gatmaitan, is of no significance. What counts most such conformity as was, allegedly, incumbent upon him under the
is the hard reality that there has been an abject failure to get Anglo- memorandum.
Aseans nod of approval over Gatmaitans being subrogated in the place o SC: Basically the court said here that either of them (Licaros or
of Licaros. Doubtless, the absence of such conformity on the part Gatmaitan) could secure the consent of Anglo-Asean.
of Anglo-Asean, which is thereby made a party to the same
Memorandum of Agreement, prevented the agreement from 12. Last argument: Petitioner argues that respondent himself allegedly admitted
becoming effective, much less from being a source of any cause of in open court that an assignment of credit was intended.
action for the signatories thereto. o Respondent apparently used the word assignment in his testimony
in the general sense. Respondent is not a lawyer and as such, he is
not so well versed in law that he would be able to distinguish
Other contentions of the petitioner and the answer of SC in case sir asks between the concepts of conventional subrogation and of
assignment of credit.
9. 1st argument: Petitioner argues that the parties to the Memorandum of
Agreement could not have intended the same to be a conventional
subrogation considering that no new obligation was created.
o SC: It is true that conventional subrogation has the effect of
extinguishing the old obligation and giving rise to a new
one. However, the extinguishment of the old obligation is the
effect of the establishment of a contract for conventional
subrogation. It is not a requisite without which a contract for
conventional subrogation may not be created. As such, it is not
determinative of whether or not a contract of conventional
subrogation was constituted.

10. 2nd argument: Petitioner next argues that the consent or conformity of Anglo-
Asean Bank is not necessary to the validity of the Memorandum of
Agreement. He claims that the preambulatory clause requiring the express
002 DBP v. JUDGE OF RTC OF MANILA (Regine) Manufacturing Corporation (Pioneer). Development Bank of the Philippines
February 5, 1990 | Fule, J. | Bulk Sales Law (DBP) and Union Glass and Container Corporation (Union) as defendants.
100. The complaint alleged that during the period from October 21, 1977 to March
PETITIONER: Development Bank of the Philippines and Union Glass & Container 22, 1978, Pioner purchased from Yu equipment parts worth P7,019.00 which
Corporation upon demand Pioneer failed or refused to pay; that without informing Yu,
RESPONDENTS: Judge of the Regional Trial Court of Manila, Branch XXV, et.al; Pioneer transferred all its assets to DBP in a “deed of cession of property in
Antonio Yu payment of obligation or dacion en pago”; that DBP in turn sold these asssets
to Union in an agreement; that the transfer of assets to DBP is fraudulent,
SUMMARY: Pioner purchased from Yu equipment parts worth P7,019.00 which hence, void by reason of which Yu suffered actual and other damages.
upon demand Pioneer failed or refused to pay and without informing Yu, Pioneer 101. Union filed a motion to dismiss on grounds of lack of cause of action and
transferred all its assets to DBP in a “deed of cession of property in payment of lack of jurisdiction over the subject matter of the action
obligation or dacion en pago”. DBP in turn sold these asssets to Union in an 102. Pioneer acknowledge indebtedness to Yu on the purchase of the equipment
agreement; that the transfer of assets to DBP is fraudulent, hence, void by reason of parts however, it interposed the affirmative defenses that by virture of the
which Yu suffered actual and other damages. Union filed a motion to dismiss on the dacion en pago in favor of DBP, its business operations had ceased; that the
grounds of lack of cause of action and lack of jurisdiction over the subject matter of cession of its assets was made known to all its creditor; and that by virture of
the action. Pioneer argued that by virture of the dacion en pago in favor of DBP, its the Board of Governors Res. No. 3509. DBP agreed to assume its obligations
business operations had ceased. DBP denied the material allegations and interposed to creditors including Yu under a payment scheme which is pending
that the bank is not a privy to the sale between Yu and Pioneer and Yu holds no lien implementation.
over Pioneer’s properties and that the validity of the dacion en pago being placed in 103. DBP denied the material allegations of the complaint and interposed as an
issue, the action is one incapable of pecuniary estimation, hence, outside the affirmative defenses lack of cause of action in that the complaint does not
jurisdiction of the MTC. Important Issue: WoN the Bulk Sales Law covers the allege that the good sold to Pioner were among those ceded to DBP; that the
conveyance in question (its violation would make DBP, Union Glass, and Pioneer bank is not a privy to the sale between Yu and Pioneer; that Yu holds no lien
Glass liable to Yu). NO. The technical terms convey the intention that the Bulk Sales over Pioneer’s properties; and that the validity of the dacion en pago being
Law applies to merchants who are in the business of selling goods and wares and placed in issue, the action is one incapable of pecuniary estimation, hence,
similar merchandise, hence the said law was held not to apply to sale of assets by a outside the jurisdiction of the MTC.
manufacturer since the nature of his business does not partake of merchandising. In 104. MTC issued an order denying the two motions to dismiss complaint on the
the case at bar, there is undisputed evidence the Pioneer manufactures glass only on ground that the said motions are prohibited pleadings.
specific orders; that it does not sell directly to consumers but manufactures its 105. Motion for reconsideration was also denied.
products only for particular clients. It cannot be said therefore that Pioneer is a 106. On appeal to the RTC, the decision of the MTC was affirmed.
merchandiser. This is not all. In the dacion en pago between Pioneer and DBP, the 107. Hence, this appeal.
former ceded, transferred and conveyed the bulk of its corporate assets to extinguish
its outstanding obligations to DBP in the total amount of about 63.2 million pesos. ISSUE/s:
The subject matter of the deed of cession were therefore assets, not stock-in-trade, of 14. WoN the cause of action is incapable of pecuniary estimation, hence, the
which it had none. Such conveyance is clearly outside the ambit of the Bulk Sales Metropolitan Trial Court has no jurisdiction. NO
Law. 15. (IMPT) WoN the Bulk Sales Law covers the conveyance in question (its
violation would make DBP, Union Glass, and Pioneer Glass liable to Yu).
DOCTRINE: Bulk Sales Law applies to merchants who are in the business of selling NO.
goods and wares and similar merchandise, and cannot cover the sale of assets by a
manufacturer since the nature of his business does not partake of merchandise. RULING: Wherefore, the decision of the Metropolian Trial Court dated July 12,
1984, as affirmed by the memorandum decision dated June 26, 1985 of respondent
Regional Trial Court is hereby MODIFIED and judgement is hereby rendered
FACTS: ordering only defendant Pioneer Glass Manufacturing Corporation to pay the plaintiff,
99. Antonio Yu, doing business under the name and style “Ancar Equipment Antonio D. Yu, the amount of P7,019.00 plus 12% interest from July 12, 1984 until
Parts” and “Tonicar” as plaintiff, instituted an action against Pioneer Glass the same is fully paid; attorney’s fees in the amount of P1,000.00; and to pay the costs.
1. As to whether the transaction is fraudulent, it bears emphasis that contrary
to the findings of the trial court, there was no concealment of the fact that
RATIO: Yu was an existing small trade creditor of Pioneer. DBP was furnished a list
of all such small creditors and the former, in Resolution No. 3509 of its
FIRST ISSUE: Board of Governors, provided for a liquidation schedule for paying all such
30. As correctly settled by the trial court: As to the jurisdiction of this Court, the other creditors. The concurrence of the foregoing circumstances
declaration of nullity of the transaction of the transfer to the DBP insofar as convinces this Court that there are no badges of fraud in the execution
Yu is concerned is merely incidental to the proceedings. The principal action of the dacion en pago.
is still recovery of the sum of P7,019.00 which is within the jurisdiction of
this court. IMPORTANT PROVISIONS OF THE BULK SALES LAW:
31. A reading of the complaint in the Civil Case confirms that the cause of action
is one of collection of a sum of money wherein judgement is prayed, ordering Sec. 2. Sale and transfer in bulk. — Any sale, transfer, mortgage or assignment of a stock of
the DBP, Union Glass and Pioneer Glass to pay Yu the sum of P7,019.00 goods, wares, merchandise, provisions, or materials otherwise than in the ordinary course of
with interest, consequential, moral and exemplary damages and attorney’s trade and the regular prosecution of the business of the vendor, mortgagor, transferor, or
fees. assignor, or sale, transfer, mortgage or assignment of all, or substantially all, of the business or
trade theretofore conducted by the vendor, mortgagor, transferor, or assignor, or of all, or
SECOND ISSUE: substantially all, of the fixtures and equipment used in and about the business of the vendor,
mortgagor, transferor, or assignor, shall be deemed to be a sale and transfer in bulk, in
1. Under the Bulk Sales Law, the terms "goods" and "merchandise,"
contemplation of this Act: Provided, however, That if such vendor, mortgagor, transferor or
having acquired a fixed meaning, refer to things and articles, which are assignor, produces and delivers a written waiver of the provisions of this Act from his creditors
kept for sale by a merchant. Likewise, the term "fixtures" has been as shown by verified statements, then, and in that case, the provisions of this section shall not
interpreted to mean the chattels, which the merchants usually possess apply.
and annex to the premises occupied by them in order to enable the latter
to store, handle and display their goods and wares. Sec. 3. Statement of creditors. — It shall be the duty of every person who shall sell, mortgage,
2. These technical terms convey the intention that the Bulk Sales Law applies transfer, or assign any stock of goods, wares, merchandise, provisions or materials in bulk, for
to merchants who are in the business of selling goods and wares and similar cash or on credit, before receiving from the vendee, mortgagee, or his, or its agent or
merchandise, hence the said law was held not to apply to sale of assets by representative any part of the purchase price thereof, or any promissory note, memorandum, or
a manufacturer since the nature of his business does not partake of other evidence therefor, to deliver to such vendee, mortgagee, or agent, or if the vendee,
merchandising. mortgagee, or agent be a corporation, then to the president, vice-president, treasurer, secretary
3. In our jurisdiction, this Court has unequivocally ruled in People v. Wong Szu or manager of said corporation, or, if such vendee or mortgagee be a partnership firm, then to a
member thereof, a written statement, sworn to substantially as hereinafter provided, of the
Tung that the sale of a foundry shop- a purely manufacturing concern-
names and addresses of all creditors to whom said vendor or mortgagor may be indebted,
together with its goodwill and credits is not contemplated nor covered by together with the amount of indebtedness due or owing, or to become due or owing by said
the Bulk Sales Laws. vendor or mortgagor to each of said creditors, which statement shall be verified by an oath to
4. In the case at bar, there is undisputed evidence the Pioneer manufactures glass the following effect…
only on specific orders; that it does not sell directly to consumers but
manufactures its products only for particular clients. It cannot be said Sec. 4. Fraudulent and void sale, transfer or mortgage. — Whenever any person shall sell,
therefore that Pioneer is a merchandiser. This is not all. In the dacion en mortgage, transfer, or assign any stock of goods, wares, merchandise, provisions or materials,
pago between Pioneer and DBP, the former ceded, transferred and in bulk, for cash or on credit, and shall receive any part of the purchase price, or any promissory
conveyed the bulk of its corporate assets to extinguish its outstanding note, or other evidence of indebtedness for said purchase price or advance upon mortgage,
obligations to DBP in the total amount of about 63.2 million pesos. without having first delivered to the vendee or mortgagee or to his or its agent or representative,
5. The subject matter of the deed of cession were therefore assets, not stock- the sworn statement provided for in section three hereof, and without applying the purchase or
in-trade, of which it had none. Such conveyance is clearly outside the ambit mortgage money of the said property to the pro rata payment of the bona fide claim or claims
of the creditors of the vendor or mortgagor, as shown upon such sworn statement, he shall be
of the Bulk Sales Law.
deemed to have violated this Act, and any such sale, transfer or mortgage shall be fraudulent
and void.
ISSUE OF FRAUD: HON. CORNELIO BALMACEDA V. UNION CARBIDE
PHILIPPINES (CASTRO) 3. The amendatory provision of the PD added two more paragraphs which was
30 September 1983 | Fernando, CJ. | Retail business the basis for the three previous decisions of the court.
4. The relevant section states that:
PETITIONER: Hon. Cornelio Balmaceda, now Leonides Virata in his capacity a. Retail business- occupation or calling of habitually selling direct to
as Secretary of Commerce and Industry the general public merchandise, commodities or goods for
RESPONDENTS: Union Carbide Philippines and Hon. Alikpala, Presiding consumption.
Judge b. According to the amendment, retail business excludes: a
manufacturer or processor selling to the industrial and commercial
PETITIONER: Hon. Marcelo Balatbat in his capacity as Secretary of Comerce users or consumers who use the products brought by them to render
and Industry service to general public and/or produce or manufacture goods
RESPONDENTS: Union Carbide Philippines, Inc. which are in turn sold to them.
5. Hence, this appeal before the SC was sought to clarify the definition of retail
SUMMARY: The Solicitor general was trying to clarify the definition of retail business
business given the fact that the Retail act was amended and at the same time they
ISSUE/s:
wanted to appeal the decision of the lower court declaring that Union Carbide is 1. WoN Union Carbide Philippines is engaged in the retail business - No
not engaged in the retail business.
RULING: WHEREFORE, the Court affirms the lower court decision holding
The Supreme Court enunciated its decision by saying that it is in line with the that Union Carbide Philippines, Inc. is not engaged in the "retail business" as
previously decided cases, comments by economists, Central Bank Governor, and this term is defined in Section 4 of Republic Act No. 1180 and malting permanent
judicial opinions that the goods contemplated in a retail business refer to the final the restraining order of June 22, 1964 issued in this case. No costs
and end [uses] of a product which directly satisfy human wants and desires and
are needed for home and daily life. Furthermore, SC believes that what removes
Union Carbide from the definition of a retail business was that “The goods handled RATIO:
by the five remaining departments of Industrial Products Division are generally 6. The case at bar is in accord with the ruling of the Supreme Court in the Ichong
raw materials used in the manufacture of other goods, or if not, as one of the case regarding the nature and kind of goods a retailer handles. Under the
component raw materials, or at the least as elements utilized in the process of situation, the Court is persuaded to hold that the goods for consumption
production or manufacturing” mentioned in Republic Act No. 1180 should be construed to refer to the final
and end [uses] of a product which directly satisfy human wants and desires
DOCTRINE: The goods handled by the five remaining departments of Industrial and are needed for home and daily life.
Products Division are generally raw materials used in the manufacture of other
goods, or if not, as one of the component raw materials, or at the least as elements 7. Accordingly, the goods which the Industrial Products Division handle
utilized in the process of production or manufacturing” (commonly referred to as intermediate goods), do not fall and cannot be
classified as consumption goods
FACTS:
1. The question raised in the said petition filed by the Solicior general is to 8. Unionbide has two divisions, the Consumer Products Division and the
review the decision of the late judge Hon Alikpala declaring that Union Industrial Products Division. As to the former, it effected its sales through
Carbide is not engaged in the retail business retail outlets, dealers and distributors. Thus, there was no question as to the
2. The court believes that the said case is in harmony with the previous
character of its business. It was not embraced in the category of retail. As to
jurisprudence laid down (namely Goodrich Philippines v. Reyes Sr.,
Goodyear Tire v. Reyes Sr., and Mobil Oil Philippines v. Reyes Sr.) and the the Industrial Products Division, its Agricultural Chemicals Department sold
doctrines therein announced applying the Presidential decree amending the its products through exclusive distributors. Again, it could be concluded that
retail trade is in point. such Department was not covered by the Act even before its amendment.
9. Again, it could be concluded that such Department was not covered by the
Act even before its amendment. The products handled by the five other
departments of the Industrial Products Division, namely, the Metals and
Carbide; Plastics; Industrial Chemicals; Linde, Haynes Stellite and Carbon
Products and Polyethylene Bags were generally sold to producers,
processors, fabricators and to industries. While these departments had a
limited fixed clientele, still there was no prohibition as to the general public
malting similar purchases from them

10. What removed these departments from the operation of the Retail Trade Act
was pointed out in the appealed decision in these words: "The goods handled
by the five remaining departments of Industrial Products Division are
generally raw materials used in the manufacture of other goods, or if not,
as one of the component raw materials, or at the least as elements utilized
in the process of production or manufacturing."

11. After considering the statutory definition in the Retail Trade Act itself, its
definition by economists, and in judicial opinions, as well as the view of
former Central Bank Governor Cuaderno as to the adverse consequences in
terms of increased cost to consumers, loss of official assistance from
producers, elimination of much needed foreign capital and loss of technical
assistance, the lower court held it was not engaged in the retail business
GOODYEAR TIRE v. REYES (KC) RATIO:
July 2, 1983 | Fernando, C.J. | Meaning of “General Public” 163. This legal question has been put to rest by PD 714, which amended the Retail
Trade Nationalization Law.
PETITIONER: The Goodyear Tire & Rubber Co. of the Philippines, Ltd. 164. As originally worded, the term “retail business” covers any act, occupation
INTERVENOR: Firestone Tire & Rubber Co. of the Philippines or calling of habitually selling direct to the general public merchandise,
RESPONDENTS: The Honorable Teofilo Reyes, Sr., in his capacity as Acting commodities or goods for consumption but shall not include:
Secretary of Commerce and Industry a. A manufacturer, processor, laborer or worker selling to the general
public the products manufactured, processed or produced by him if
SUMMARY: Goodyear Tire and Firestone Tire claim that they are exempt from his capital does not exceed P5,000.
RA 1180 which regulates the retail business arguing that their costumers do not b. Farmer or agriculturist selling the product of his farm.
fall under the category “general public”. The lower court ruled partially in favor c. A manufacturer or processor selling to the industrial and
of both parties. The SC affirmed this saying that in so far as the proprietary commercial users or consumers who use the products bought by
planters, entities engaged in the exploitation of natural resources, and the them to render to the general public and/or to produce or
employees and officers of both parties, their selling of rubber should be considered manufacture goods which are in turn sold to them
as retail. The legal question has long been settled when PD 714 amended Retail d. A hotel-owner or keeper operating a restaurant irrespective of the
Trade Nationalization Law. As in the case of Goodrich, the SC found therein amount of capita, provided that the restaurant is necessarily included
petitioner engaged in retail business as to its employees and officers, proprietory in, or incidental to the hotel business.
planters, and persons engaged in the exploitation of natural resources costumers. 165. As noted by the decision of the lower court Goodyear Tire and Firestone Tire
sold their rubber products to certain types or class of customers:
DOCTRINE: Even when consumer goods is limited only to the officers of the a. The government and all its instrumentalities
company, the same would still constitute retail trade covered by the Law. b. Public utilities
c. Agricultural enterprises, proprietary planters, agricultural
processing plants and agricultural cooperatives
FACTS: d. Logging, mining and other entities engaged in exploitation of
199. Both Goodyear Tire & Rubber Co. of the Philippines and (intervenor) natural resources
Firestone Tire & Rubber Co. of the Philippines appealed from the decision of e. Automotive assembly plants
the lower court holding that they are engaged in retail business as to their f. Industrial and commercial enterprises engaged in the sale of prime
customers – which are proprietory planters, persons engaged in the and essential commodities
exploitation of natural resources, and employees and officers of the g. Employees and officers of the Goodyear Tire and Firestone Tire
petitioner. 166. Goodyear Tire and Firestone year are not exempt from RA 1180 (An Act to
200. Hence, this appeal. Regulate the Retail Business) in so far as the said costumers. As in the case
of Goodrich, a similar conclusion is proper. The SC in that case held that:
ISSUE/s: a. “It is dear from above that proprietary planters and persons engaged
56. WoN Goodyear Tire and Firestone Tire is engaged in retail business – YES in the ploration of natural resources are included x x x. The lower
but only as to letters C, D, and G under Fact #3. court decision x x x is in accordance with law insofar as employees
and officers of petitioner38 are concerned.”
RULING: WHEREFORE, the lower court decision is affirmed declaring that
petitioner and intervenor are not engaged in retail business within the purview of
Section 4 of Republic Act No. 1180 and Presidential Decree No. 714, except as to its
sales to its employees and officers. The injunction issued is likewise made permanent
but subject to the above qualification. No costs.

38
Not petitioner in this case. This was lifted from the Goodrich case.
DBP v. JUDGE OF RTC OF MANILA (Paolo; but I just edited the Manufacturing Corporation (Pioneer). Development Bank of the Philippines
digest of Regine) (DBP) and Union Glass and Container Corporation (Union) as defendants.
February 5, 1990 | Fule, J. | Meaning of General Public 21. The complaint alleged that during the period from October 21, 1977 to March
22, 1978, Pioner purchased from Yu equipment parts worth P7,019.00 which
PETITIONER: Development Bank of the Philippines and Union Glass & Container upon demand Pioneer failed or refused to pay; that without informing Yu,
Corporation Pioneer transferred all its assets to DBP in a “deed of cession of property in
RESPONDENTS: Judge of the Regional Trial Court of Manila, Branch XXV, et.al; payment of obligation or dacion en pago”; that DBP in turn sold these asssets
Antonio Yu to Union in an agreement; that the transfer of assets to DBP is fraudulent,
hence, void by reason of which Yu suffered actual and other damages.
SUMMARY: Pioner purchased from Yu equipment parts worth P7,019.00 which upon 22. Union filed a motion to dismiss on grounds of lack of cause of action and
demand Pioneer failed or refused to pay and without informing Yu, Pioneer transferred lack of jurisdiction over the subject matter of the action
all its assets to DBP in a “deed of cession of property in payment of obligation or dacion 23. Pioneer acknowledge indebtedness to Yu on the purchase of the equipment
en pago”. DBP in turn sold these asssets to Union in an agreement; that the transfer of parts however, it interposed the affirmative defenses that by virture of the
assets to DBP is fraudulent, hence, void by reason of which Yu suffered actual and dacion en pago in favor of DBP, its business operations had ceased; that the
other damages. Union filed a motion to dismiss on the grounds of lack of cause of action cession of its assets was made known to all its creditor; and that by virture of
and lack of jurisdiction over the subject matter of the action. Pioneer argued that by the Board of Governors Res. No. 3509. DBP agreed to assume its obligations
virture of the dacion en pago in favor of DBP, its business operations had ceased. DBP to creditors including Yu under a payment scheme which is pending
denied the material allegations and interposed that the bank is not a privy to the sale implementation.
between Yu and Pioneer and Yu holds no lien over Pioneer’s properties and that the 24. DBP denied the material allegations of the complaint and interposed as an
validity of the dacion en pago being placed in issue, the action is one incapable of affirmative defenses lack of cause of action in that the complaint does not
pecuniary estimation, hence, outside the jurisdiction of the MTC. Bulk Sales Issue: allege that the good sold to Pioneer were among those ceded to DBP; that the
WoN the Bulk Sales Law covers the conveyance in question (its violation would make bank is not a privy to the sale between Yu and Pioneer; that Yu holds no lien
DBP, Union Glass, and Pioneer Glass liable to Yu). NO. The technical terms convey over Pioneer’s properties; and that the validity of the dacion en pago being
the intention that the Bulk Sales Law applies to merchants who are in the business of placed in issue, the action is one incapable of pecuniary estimation, hence,
selling goods and wares and similar merchandise, hence the said law was held not to outside the jurisdiction of the MTC.
apply to sale of assets by a manufacturer since the nature of his business does not 25. MTC issued an order denying the two motions to dismiss complaint on the
partake of merchandising. In the case at bar, there is undisputed evidence the Pioneer ground that the said motions are prohibited pleadings.
manufactures glass only on specific orders; that it does not sell directly to consumers 26. Motion for reconsideration was also denied.
but manufactures its products only for particular clients. It cannot be said therefore that 27. On appeal to the RTC, the decision of the MTC was affirmed.
Pioneer is a merchandiser. This is not all. In the dacion en pago between Pioneer and 28. Hence, this appeal.
DBP, the former ceded, transferred and conveyed the bulk of its corporate assets to
extinguish its outstanding obligations to DBP in the total amount of about 63.2 million ISSUE/s:
pesos. The subject matter of the deed of cession were therefore assets, not stock-in- 13. WoN the cause of action is incapable of pecuniary estimation, hence, the
trade, of which it had none. Such conveyance is clearly outside the ambit of the Bulk Metropolitan Trial Court has no jurisdiction. NO
Sales Law. [For Chapter 16 issue; look at end of ratio]
14. WoN the Bulk Sales Law covers the conveyance in question (its violation
DOCTRINE: Where company manufactures glass products only on specific would make DBP, Union Glass, and Pioneer Glass liable to Yu). NO.
orders, it does not sell directly to consumers but manufacturers its products
only for the particular clients, it cannot be said that it is a merchandiser. 15. WoN Pioneer corp sells goods to the general public. NO. only
manufactures goods for specific clients.
FACTS:
20. Antonio Yu, doing business under the name and style “Ancar Equipment RULING: Wherefore, the decision of the Metropolian Trial Court dated July 12,
1984, as affirmed by the memorandum decision dated June 26, 1985 of respondent
Parts” and “Tonicar” as plaintiff, instituted an action against Pioneer Glass
Regional Trial Court is hereby MODIFIED and judgement is hereby rendered
ordering only defendant Pioneer Glass Manufacturing Corporation to pay the plaintiff, trade, of which it had none. Such conveyance is clearly outside the ambit of
Antonio D. Yu, the amount of P7,019.00 plus 12% interest from July 12, 1984 until the Bulk Sales Law.
the same is fully paid; attorney’s fees in the amount of P1,000.00; and to pay the costs.
ISSUE OF FRAUD:
a.i.1. As to whether the transaction is fraudulent, it bears emphasis that
RATIO: contrary to the findings of the trial court, there was no concealment of the
fact that Yu was an existing small trade creditor of Pioneer. DBP was
FIRST ISSUE: furnished a list of all such small creditors and the former, in Resolution No.
• As correctly settled by the trial court: As to the jurisdiction of this Court, the 3509 of its Board of Governors, provided for a liquidation schedule for
declaration of nullity of the transaction of the transfer to the DBP insofar as paying all such other creditors. The concurrence of the foregoing
Yu is concerned is merely incidental to the proceedings. The principal action circumstances convinces this Court that there are no badges of fraud in the
is still recovery of the sum of P7,019.00 which is within the jurisdiction of execution of the dacion en pago.
this court.
• A reading of the complaint in the Civil Case confirms that the cause of action MEANING OF “GENERAL PUBLIC”: (this is literally the only took worth
is one of collection of a sum of money wherein judgement is prayed, ordering taking from the case for this chapter)
the DBP, Union Glass and Pioneer Glass to pay Yu the sum of P7,019.00
with interest, consequential, moral and exemplary damages and attorney’s • According to CLV: “General Public” must mean that the activities of the
fees. seller must be such that the target clientele or customers must not only
be a particular person or group of persons.
SECOND ISSUE: • This is not determined by the nature of the goods sold on whether they
a.i.1. Under the Bulk Sales Law, the terms "goods" and "merchandise," would be acceptable or usable only by a sector of society.
having acquired a fixed meaning, refer to things and articles, which are kept • So in this case, Pioneer Corp. is not a merchandiser that sells goods to
for sale by a merchant. Likewise, the term "fixtures" has been interpreted to the general public. The business operations of the company show that it
mean the chattels, which the merchants usually possess and annex to the only manufactures glass products for particular clients. It is not a
premises occupied by them in order to enable the latter to store, handle and merchandiser that sells indiscriminately to just any customer, hence it
display their goods and wares. cannot be conducting a business enterprise that serves the interests of
the “General Public”.
2. These technical terms convey the intention that the Bulk Sales Law applies
to merchants who are in the business of selling goods and wares and similar
merchandise, hence the said law was held not to apply to sale of assets by a
manufacturer since the nature of his business does not partake of
merchandising.
3. In our jurisdiction, this Court has unequivocally ruled in People v. Wong Szu
Tung that the sale of a foundry shop- a purely manufacturing concern-
together with its goodwill and credits is not contemplated nor covered by the
Bulk Sales Laws.
4. In the case at bar, there is undisputed evidence the Pioneer manufactures
glass only on specific orders; that it does not sell directly to consumers
but manufactures its products only for particular clients. It cannot be
said therefore that Pioneer is a merchandiser. This is not all. In the dacion
en pago between Pioneer and DBP, the former ceded, transferred and
conveyed the bulk of its corporate assets to extinguish its outstanding
obligations to DBP in the total amount of about 63.2 million pesos.
5. The subject matter of the deed of cession were therefore assets, not stock-in-

You might also like